You are on page 1of 490

Mathematics

GEOMETRY
for JEE (Main & Advanced)
VOL. 4
Mathematics
GEOMETRY
for JEE (Main & Advanced)
VOL. 4

Dr. G.S.N. Murti


Reader and HOD of Maths (Retd.)
Rajah R. S. R. K. R. R. College, Bobbili, Andra Pradesh, India

Wiley India Pvt. Ltd.


Mathematics
GEOMETRY
for JEE (Main & Advanced)

VOL. 4

Copyright © 2013 by Wiley India Pvt. Ltd., 4435-36/7, Ansari Road, Daryaganj, New Delhi-110002.
All rights reserved. No part of this book may be reproduced, stored in a retrieval system, or transmitted in any form or by
any means, electronic, mechanical, photocopying, recording or scanning without the written permission of the
publisher.
Limits of Liability: While the publisher and the author have used their best efforts in preparing this book, Wiley and the
author make no representation or warranties with respect to the accuracy or completeness of the contents of this book,
and specifically disclaim any implied warranties of merchantability or fitness for any particular purpose. There are no
warranties which extend beyond the descriptions contained in this paragraph. No warranty may be created or extended
by sales representatives or written sales materials.
Disclaimer: The contents of this book have been checked for accuracy. Since deviations cannot be precluded entirely,
Wiley or its author cannot guarantee full agreement. As the book is intended for educational purpose, Wiley or its author
shall not be responsible for any errors, omissions or damages arising out of the use of the information contained in the
book. This publication is designed to provide accurate and authoritative information with regard to the subject matter
covered. It is sold on the understanding that the Publisher is not engaged in rendering professional services.
Other Wiley Editorial Offices:
John Wiley & Sons, Inc. 111 River Street, Hoboken, NJ 07030, USA
Wiley-VCH Verlag GmbH, Pappellaee 3, D-69469 Weinheim, Germany
John Wiley & Sons Australia Ltd, 42 McDougall Street, Milton, Queensland 4064, Australia
John Wiley & Sons (Asia) Pte Ltd, 1 Fusionopolis Walk #07-01 Solaris, South Tower Singapore 138628
John Wiley & Sons Canada Ltd, 22 Worcester Road, Etobicoke, Ontario, Canada, M9W 1L1

First Edition: 2013


ISBN: 978-81-265-3583-5
ISBN:978-81-265-8073-6 (ebk)

www.wileyindia.com
Printed at: Beekam Print & Pack (P) Ltd., Delhi
Dedication

Dedicated to
Sri. Poojya Ganapati Sachchidananda Swamyjee,
Dattanagar, Mysore, Karnataka

Dr. G. S. N. Murti
Acknowledgments

1. My special thanks to Mr. Paras Bansal, Publisher and Ms. Meenakshi Sehrawat, Senior Developmental
Editor, of Wiley India for their constant help and support.

2. My sincere and heartful thanks to Wiley India Pvt. Ltd. for giving me the opportunity to write this
textbook.

Dr. G. S. N. Murti
Features and Benefits
at a Glance
Feature Benefit to student
Chapter Opener Peaks the student’s interest with the chapter opening vignette, definitions
of the topic, and contents of the chapter.
Clear, Concise, and Inviting Students are able to Read this book, which reduces math anxiety and
Writing Style, Tone and Layout encourages student success.
Theory and Applications Unlike other books that provide very less or no theory, here theory is
well matched with solved examples.
Theorems Relevant theorems are provided along with proofs to emphasize
conceptual understanding.
Solved Examples Topics are followed by solved examples for students to practice and
understand the concept learned.
Examples Wherever required, examples are provided to aid understanding of
definitions and theorems.
Quick Look Formulae/concepts that do not require extensive thought but can be
looked at the last moment.
Try It Out Practice problems for students in between the chapter.
Subjective Problems Solved subjective problems for the preceding sections.
Summary Key formulae, ideas and theorems are presented in this section in
each chapter.
Worked Out Problems The problems are presented in the form of
Single Correct Choice Type Questions
Multiple Correct Choice Type Questions
Matrix-Match Type Questions
Comprehension-Type Questions
Integer Answer Type Questions
In-depth solutions are provided to all problems for students to
understand the logic behind.
Exercises Offer self-assessment. The questions are divided into subsections which
include large number of Multiple Choice Questions as per requirements
of JEE (Main & Advanced).
Answers Answers are provided for all exercise questions for students to validate
their solution.
Note to the Students
The JEE (Main & Advanced) is one of the hardest exams to crack for
students, for a very simple reason – concepts cannot be learned by rote,
they have to be absorbed, and IIT believes in strong concepts. Each
question in the JEE (Main & Advanced) entrance exam is meant to push
the analytical ability of the student to its limit. That is why the questions
are called brainteasers!
Students find Mathematics the most difficult part of JEE (Main &
Advanced). We understand that it is difficult to get students to love
mathematics, but one can get students to love succeeding at mathematics.
In order to accomplish this goal, the book has been written in clear, concise,
and inviting writing style. It can be used as a self-study text as theory is well
supplemented with examples and solved examples. Wherever required,
figures have been provided for clear understanding.
If you take full advantage of the unique features and elements of this
textbook, we believe that your experience will be fulfilling and enjoyable.
Let’s walk through some of the special book features that will help you in
your efforts to crack JEE (Main & Advanced).
To crack mathematics paper for JEE (Main & Advanced) the five things
to remember are:
1. Understanding the concepts
2. Proper applications of concepts
3. Practice
4. Speed
5. Accuracy

About the Cover Picture


The picture on the cover is Jatiyo Smriti Soudho or National Martyrs'
Memorial – a monument located in Savar, Bangladesh – which is built
to commemorate the valour and the sacrifice of those killed in the
Bangladesh Liberation War of 1971. The monument is composed of seven
isosceles triangular pyramid shaped towers, with the middle one being the
tallest, that is, 150 feet. The arrangement of the seven towers is unique. The
planes are arranged uniquely so that one can see its distinctive patterns
when looking at it from different angles.
A. PEDAGOGY

CHAPTER OPENER
Rectangular
Each chapter starts with an opening vignette,
Coordinates, Basic
Formulae, Locus and
Change of Axes
1 definition of the topic, and contents of the
chapter that give you an overview of the
chapter to help you see the big picture.

y Contents
1.1 Rectangular Coordinates
1.2 Basic Formulae
Basic Formulae, Locus and

1.3 Locus
−x x
Rectangular Coordinates,

1.4 Change of Axes

Worked-Out Problems
−y
Summary
Exercises
y
Answers

−z x

The locus of a point is the


−x
Change of Axes

z path traced out by the point


when it moves according to a
given rule (or rules). In other
−y
words, a locus is the path of
90°
a single moving point that
obeys certain conditions.

180° 0°

270°
Equiangular spiral

CLEAR, CONCISE, AND INVITING WRITING


g p p g y

Special attention has been paid to present an DEFINITION 4.1 Cone Let S be a non-empty set of Fpoints
HHE in the space. Then, S is called a cone if there ex-
ists a point V S such that the line VP is Fcontained in S for all points P in S. This point V is
HHE
engaging, clear, precise narrative in the layout called the vertex of the cone and the line VP where P S is called generator of the cone S.

that is easy to use and designed to reduce Examples

(1) Every line is a cone with every point on the line as (3) Two intersecting planes form a cone with every point
math anxiety students may have. vertex and the line is the only generator. on their line of intersection as vertex.
(2) Every plane is a cone with all of its points as vertices.

DEFINITION 4.2 Degenerate and Non-degenerate Cones The cones described in the examples of Definition 4.1
are called degenerate cones. Generally, cones that are having more than one vertex are called

DEFINITIONS degenerate cones. Cones which do not degenerate are called non-degenerate cones. Using the
three-dimensional analytic geometry (Chapter 6), we can verify that the locus represented by the
equation x 2 y2 z2  0 is a cone with origin as the vertex.

DEFINITION 4.3 Base Curve or Guiding Curve If a plane is not passing through the vertex and intersects all
the generators of a cone, then the intersection of the plane and the cone are called base curve
Every new topic or concept starts with or guiding curve.

defining the concept for students. Related DEFINITION 4.4 Circular Cone and Right Circular Cone If the base curve is a circle, then it is called a circular
cone (see Fig. 4.1). If the base curve is a circle and the line connecting the centre of the base
and the vertex of the cone is perpendicular to the plane of the circle, then the cone is called
examples to aid the understanding follow the right circular cone.

definition.
EXAMPLES
Example 2.13

Write 2x 3y 5  0 in the normal form. What is the dis- To make the RHS positive, we multiply both sides with
tance of the line from origin? (−1). Thus, the normal form is
¥ 2 ´ ¥ 3 ´ 5
Examples pose a specific problem using
Solution: In 2x 3y 5  0, the constant 5 should be ¦§ µ x ¦§ µy
taken to the RHS of the equation, that is
where
13 ¶ 13 ¶ 13
concepts already presented and then work
2x 3y  −5

Dividing both sides with a 2 b2  2 2 32  13 , we


cos A 
2
13
and sin A 
3
13
through the solution. These serve to enhance
get
2 3 5
Now, the distance of the line from the orgin is the students’ understanding of the subject
x y c 5 5
13 13 13 p
a 2 b2

13

13 matter.
Example 2.14

Find the normal form of the line 3x 4y − 10  0 and its where


distance from the origin.
3 4
cos A  and sin A 
Solution: The equation 3x 4y − 10  0 can be 5 5
written as 3x 4y  10. Dividing both sides with
Now the distance of the line from the origin is
a 2 b2  32 4 2  5, we get
10 10
3 4 p  2
x y 2 5 5
5 5

T H E O R E M 2.11 Let ax by c  0 be a straight line. Then

¥ a ´ ¥ b ´ c
1. ¦ µ x ¦ 2 µy is the normal form of the given line if c  0.
§ a 2 b2 ¶ § a b2 ¶ a 2 b2
¥ a ´ ¥ b ´ c
2. ¦ µ x ¦ 2 µy is the normal form of the given line if c  0.
§ a 2 b2 ¶ § a b2 ¶ a 2 b2
PROOF Suppose x cos A y sin A  p is the normal form of ax by c  0. Therefore, by Theorem 2.10,
there exists a real K x 0 such that cos@  Ka, sin@  Kb and −p  Kc. Now,
cos2@ sin2@  1
1
implies K±
a 2 b2
Also −Kc  p  0 (since p is the distance of the line from origin) implies that
« 1
® 2 if c  0
® a b2
L¬
® 1
THEOREMS ®­ a 2 b2

Therefore, if c  0, then the normal form of the line is


if c  0

¥ a ´ ¥ b ´ c
x¦ µ y¦ 2 µ
§ a 2 b2 ¶ § a b2 ¶ a 2 b2
Relevant theorems are provided along with or if c  0, the normal form of the line is

proofs to emphasize conceptual understand- ¥



a ´ ¥
µ y¦ 2
b ´
µ
c
§ a 2 b2 ¶ § a b2 ¶ a 2 b2
ing rather than rote learning.

QUICK LOOK 1

x2 y2 5. b  a.
The properties of the curve  1 are as follows:
QUICK LOOK
a2 b2 6. If P(x, y) is a point on the ellipse

1. The curve is symmetric about both axes. x2 y2



2. For any point (x, y) on the curve, we have a b x b a a2 b2
and b b y b b. then we have
3. The x-axis meets the curve at A(a, 0) and Aa(−a, 0).
The y-axis meets the curve at B(0, b) and Ba(0, −b).
SP  e(PM)  e(NZ)  e(CZ − CN)
¥a ´
Some important formulae and concepts that
4. For each value of x,  e ¦ xµ  a ex

y pb 1
x 2
§e ¶
7. Since the curve is symmetric about both axes, there
do not require exhaustive explanation, but
a2 must be second focus and directrix. Another focus
and for each value of y,
Sa(−ae, 0) and its corresponding directrix is their mention is important, are presented in
a
x  pa 1
y2 x
e this section. These are marked with a magni-
b2
fying glass.
TRY IT OUT Try it out Try Example 5.15 for the hyperbola x 2 y2  a 2 whose asymptotes are x p y  0.

Within each chapter the students would find


problems to reinforce and check their under-
standing. This would help build confidence
as one progresses in the chapter. These are
marked with a pointed finger.

Subjective Problems SUBJECTIVE PROBLEMS


1. If p1 and p2 are the distances between the opposite Also
sides of a parallelogram and @ is one of its angles,
then show that the area of the parallelogram is p1p2 a1a2 b1b2
cos A 
cosec@. (a12 b12 )(a22 b22 ) Since geometry requires a lot of practice, some
Solution: ABCD is a parallelogram (see Fig. 2.18). AM Therefore
 p1, DN  p2 and BAD  @.
(a1a2 b1b2 ) 2 chapters in addition provide numerous solved
sin 2 A  1 cos2 A  1
Area of the parallelogram  (AB) p2 (2.17) (a12 b12 )(a22 b22 )
Now from $AMP, sin@  p1/AB and hence we have AB  (a1b2 a2 b1 )2
examples in the pattern of Subjective Problems.
p1 cosec@. Therefore, from Eq. (2.17), the area of the par-  (2.18)
(a12 b12 )(a22 b22 )
allelogram  p1p2 cosec@.
Now the area of the parallelogram (by Problem 1) is
We have provided such problems within the
D C

p2
a

p1 p2 (cosec A ) 
d1 c1
–
d2 c2
–
(a12 b12 )(a22 b22 ) chapter, near to the concept.
p-a a12 b12 a22 b22 a1b2 a2 b1
a 90°
B
A N a
[from Eq. (2.18)]
p1 90° (d1 c1 )(d2 c2 )
M 
a1b2 a2 b1
FIGURE 2.18
3. Prove that the area of the parallelogram formed by the
2. Show that the area of parallelogram whose sides are lines 4y − 3x − a  0, 3y − 4x a  0, 4y − 3x − 3a  0 and
a1x b1y c1  0, a1x b1y d1  0, a2x b2y c2  0 and 3y − 4x 2a  0 is 2a2/7.
a2x b2y d2  0 is
Solution: Rewriting the equations of the sides of the
(d1 c1 )(d2 c2 ) parallelogram, we have
a1b2 a2 b1 3x − 4y a  0
4x − 3y − a  0
Solution: Consider Fig. 2.18. Let the equations of the
3x − 4y 3a  0
sides be a1x b1y c1  0, a1x b1y d1  0, a2x b2y
c2  0 and a2x b2y d2  0. Therefore, and 4x − 3y − 2a  0

d1 c1 Here, c1  a, d1  3a, c2  −a, d2  −2a, a1  3, b1  −4, a2 


p1  4 and b2  −3. Therefore, by Problem 2,
a12 b12
(d1 c1 )(d2 c2 )
d2 c2 Area 
p2  a1b2 a2 b1
a22 b22

SUMMARY
2.1. Slope of line: Let l be a non-vertical line (i.e., l is not 4. Intercept form: If a and b are x and y intercepts of a
parallel to y-axis) making an angle P with the posi- x y
At the end of every chapter, a summary is pre- tive direction of x-axis. Then, tan P is called the slope
of the line l. Generally, the slope of a line is denoted
line (ab x 0), then the line equation is  1.
a b
Note: Area of the triangle formed by the coordi-
sented that organizes the key formulae and by m.
Caution: The concept of slope is followed only for nate axis and the line
x y 1
 1 is ab sq. unit.
a b 2
non-vertical lines.
theorems in an easy to use layout. The related Note: Slope of a horizontal line (which is parallel to
5. Slope–intercept form: The equation of a non-
vertical line which is having slope m and
x-axis) is always zero.
topics are indicated so that one can quickly 2.2. If A(x1, y1) and B(x2, y2) are two points on a non-
y-intercept c is
y  mx c
FHHHE y y1
summarize a chapter. vertical line, then the slope of the line AB is 2
x2 x1
. Note: Equation of any line (except the y-axis)
passing through origin is the form y  mx.
6. Normal form: Let l be a line whose distance from
2.3. Intercepts on the axes: If a line l meets x-axis at
(a, 0) and y-axis at (0, b), then a is called x-intercept the origin is ON ( p) and ON make an angle @
and b is called y-intercept of the line l. with the positive direction of the x-axis. Then, the
equation of the line l is x cos @ y sin @  p.
2.4. Equations of the axis: The equation of x-axis is y  0
and the equation of y-axis is x  0. 2.6. Definition (first-degree equation): If a, b and c are
real and either a or b is not zero, then ax by c is
2.5. Various forms of straight line equations: called first-degree expression in x and y and ax by
c  0 is called first-degree equation in x and y.
1. Two-point form: Equation of the line passing
through two points (x1, y1) and (x2, y2) is 2.7. Theorem: Every first-degree equation in x and y rep-
(x – x1) (y1 – y2)  (y – y1) (x1 – x2) resents a straight line and the equation of any line in
the coordinate plane is a first-degree equation in x
2. Point–slope form: Equation of the line which is and y.
having slope m and passing through the point
(x1, y1) is 2.8. General equation of a straight line: First-degree
equation in x and y is called the general equation of
y – y1  m(x – x1) a straight line.
3. Symmetric form: If a non-vertical makes an angle
2.9. Various forms of ax by c  0, where abc x 0:
P with the positive direction of x-axis and passes
through a point (x1, y1), then its equation is 1. Slope–intercept form:
x x1 y y1 ¥ a ´ ¥ c ´
 y ¦ µ x ¦ µ
cos Q sin Q § b¶ § b¶
Note: In the above relation, if we consider that
2. Intercept form:
each ratio is equal to r (real number), then every
point on the line is of the form (x1 rcosP, y1 x y
rsinP). Also r gives the distance of the point (x, y) 1
( c / a) ( c / b)
on the given line from the fixed point (x1, y1).
B. WORKED-OUT PROBLEMS AND ASSESSMENT

Mere theory is not enough. It is also important to practice and test what
has been proved theoretically. The worked-out problems and exercise at
the end of each chapter will enhance the concept building of students. The
worked-out problems and exercises have been divided into:
1. Single Correct Choice Type Questions
2. Multiple Correct Choice Type Questions
3. Matrix-Match Type Questions
4. Comprehension-Type Questions
5. Integer Answer Type Questions

WORKED-OUT PROBLEMS
In-depth solutions are provided to all worked-out problems for students to understand the logic behind
and formula used.

WORKED-OUT PROBLEMS SINGLE CORRECT CHOICE


Single Correct Choice Type Questions
1. If the line 3ax 5y a 2  0 passes through the point 1 4c
– c  24 c2  36  c p6
TYPE QUESTIONS
( 1, 4), then value of a is 2 3

(A) 9 (B) 7 (C) −9 (D) −7 Since y-intercept is positive, the value c is 6 and the equa-
Solution:
have
Since the line passes through ( 1, 4), we
tion of the line is 3x 4y 24  0.
Answer: (B) These are the regular multiple choice ques-
3a( 1) 5(4) a 2  0 3. A non-horizontal line passing through the point (4, 2)
and whose distance from the origin is 2 units is
tions with four choices provided as asked in
 2a 18  0
Hence, a  9 and the line is 27x 5y 7  0.
(A) 3x 4y 10  0
(C) 4x 3y 10  0
(B) x y 2 0
(D) 2x 3y 2  0
JEE (Main & Advanced). Only one among
Answer: (A)

2. A line has slope 3 / 4, positive y-intercept and forms


Solution: Let the slope of the line be m. Now the
equation of the line, by Theorem 2.2, is
the four choices will be the correct answer.
a triangle of area 24 sq. units with coordinate axes.
Then, the equation of the line is y 2  m( x 4)

(A) 3x 4y 24  0 (B) 3x 4y 24  0 Therefore, by Theorem 2.14,


(C) 3x 4y 25  0 (D) 3x 4y 25  0 m(0 4) 0 2
2
Solution: Let the line be m2 1
¥ 3´  (2 m 1)2  m2 1
y  ¦ µ x c
§ 4¶
 3m 2 4 m  0
so that the intercepts on the x and y axes, respectively, are 4
4c/3 and c. Therefore, the area of the triangle (by Quick m  0,
3
Look 4) is

Multiple Correct Choice Type Questions


1. For the hyperbola 9 x 2 16 y2 18 x 32 y 151  0, 2. If the circle x2 y2  a2 cuts the hyperbola xy  c2 at
which of the following are true? four points (xk, yk) (where k  1, 2, 3 and 4), then
5 (A) x1 x2 x3 x4  0 (B) y1 y2 y3 y4  0
(A) Eccentricity is
4 (C) x1 x2 x3 x4  c4 (D) y1 y2 y3 y4  c4
(B) Foci are (−4, 1) and (6, 1)
Solution: The abscissa xk (where k  1, 2, 3 and 4) are
(C) Centre is (1, −1) the roots of the equation
9
(D) Length of the latus rectum is
2 c4
x2  a2
Solution: The given equation is x2
 x 4 a2 x 2 c4  0
9( x 2 2 x) 16( y2 2 y) 151  0
Therefore
 9( x 1)2 16( y 1)2  151 9 16  144
x1 x2 x3 x4  0
2 2
( x 1) ( y 1)
 1 Since the coefficient of x3 is zero, we have
16 9
¤ x1 x2  a 2 , ¤ x1 x2 x3  0, ¤ x1 x2 x3 x4  c 4
X2 Y2
 1
16 9 Now,

where X  x 1, Y  y 1. Here a 2  16, b2  9. The ¥ 1 1 1 1´


eccentricity e is given by y1 y2 y3 y4  c 2 ¦
§ x1 x2 x3 x4 ¶µ
9 25 c 2 ( ¤ x2 x3 x4 )
9  16(e 2 1) or e 2  1  
16 16 x1 x2 x3 x4

MULTIPLE CORRECT CHOICE so that

e
5

c 2 (0)
c4
0

4
TYPE QUESTIONS
Finally
The centre is given by c 2 c 2 c 2 c 2 c8
y1 y2 y3 y4  – – –   c4
( X  0, Y  0)  ( x 1  0, y 1  0)  (1, 1) x1 x2 x3 x4 c 4
The foci is given by Answers: (A), (B), (C), (D)

Multiple correct choice type questions have ( X  p ae, Y  0)  ( x 1  p 5, 1)  (6, 1) and ( 4, 1) 3. On the ellipse 4x2 9y2  1, the points at which the
tangents are parallel to the line 9y  8x are
The latus rectum is given by
four choices provided, but one or more of the 2b2 2(9) 9
 
¥ 2 1´
(A) ¦ , µ
§ 5 5¶
2 1
(B) ¥¦ , ´µ
§ 5 5¶
a 4 2
choices provided may be correct. Answers: (A), (C), (D)
MATRIX-MATCH TYPE QUESTIONS
Matrix-Match Type Questions These questions are the regular “Match the
1. Match items of Column I with those of Column II. a b
xx1 y(0) ( x x1 ) ( y 0)  x12 ax1
2 4 Following” variety. Two columns each con-
Column I Column II  4 xx1 2ax 2ax1 by  4 x12 4ax1
(A) If x-axis bisected (p) ( d, 2) ‡ (2, d)
each of two chords
 2(2 x1 a) x by 2ax1 4 x12  0 taining 4 subdivisions or first column with
drawn from the point
(a, b/2) on the circle
This passes through the point (a, b/2). This implies
that four subdivisions and second column with
2x(x a) y(2y b) 
0 (ab x 0), then a/b
belongs to (q) ( 2, 2)
¥ b´
2(2 x1 a)a b ¦ µ 2ax1 4 x12  0
§ 2¶ more subdivisions are given and the student
¥ b2 ´
(B) If the circles x2 y2
10x 16  0 and
 4 x12 6ax1 ¦ 2a 2 µ  0
§ 2¶ should match elements of column I to that of
x2 y2  r2 intersect
in two distinct
points, then r lies in
(which has two distinct real roots)
¥ b2 ´
column II. There can be one or more matches.
 4 x12 6ax1 ¦ 2a 2 µ  0
the interval (r) ( d, 2 ) ‡ ( 2, d) § 2¶
(C) If the line y x  0 (which has two distinct real roots)
bisects chords drawn
from the point (1
¥ b2 ´
 (6a)2  4(4) ¦ 2a 2 µ
a 2 /2, 1 a 2 /2) § 2¶
to the circle 2x2  9a 2  2(4a 2 b2 )
(s) ( 3, 3)
2y2 (1 a 2 )x
 a 2  2b2
(1 a 2 )y  0, then
a belongs to a
 2
(D) Point (2, K) lies b
inside the circle x2
y2  13 if and only if (t) (2, 8) Therefore
K belongs to a a
 2 or  2
b b
Solution:
Answer: (A) m (r)
(A) The given circle equation is
(B) O  (0, 0) and A  (5, 0) are the centres and r, 3 are
S y 2 x 2 2 y2 2ax by  0 the radii of the circles. The two circles intersect in
b two distinct points. So
S y x 2 y2 ax y  0
2 r 3  OA  r 3
Let (x1, 0) be the midpoint of a chord of the circle. š r 3  5r 3
Therefore, the equation of the chord is

COMPREHENSION-TYPE QUESTIONS
Comprehension-Type Questions
Comprehension-type questions consist of 1. Passage: Consider the straight line 3x y 4  0. An- (A) x 3 y 4  0 (B) x 3y 5  0
swer the following questions.
x 3y 4  0 (D) x 3 y 2  0
a small passage, followed by three multiple (i) The point on the line 3 x y 4  0 which is
equidistant from the points ( 5, 6) and (3, 2) is
(C)
(iii) If the line y 5  k( x 3) is parallel to the given
line then the area of the triangle formed by this
choice questions. The questions are of single (A) ( 1, 1) (B) ( 2, 2)
1
line and the coordinate axes (in sq. units) is
(C) ( 3, 5) (D) ( , 3) 8 16
correct answer type. 3
(ii) Equation of the line passing through the point
(A)
3
(B)
3
(C) 4 (D) 5
(1, 1) and perpendicular to the given line is

Solution:
(ii) Line perpendicular to the given line is of the form
(i) Let A  ( 5, 6) and B  (3, 2). The slope of AB is
1
6 2 1 y x c
 3
5 3 2
This line passes through (1, 1). It implies that
and the midpoint of AB  ( 1, 4). Hence, the per-
pendicular bisector of the segment AB is 1 2
1 c c
y 4  2( x 1) or 2 x y 6  0 . Solving this equa- 3 3
tion and the given line equations, we have x  2
Thus, the required line is
and y  2. Thus, ( 2, 2) is the point on the given
line which is equidistant from both A( 5, 6) and x 2
y or x 3 y 2  0
B(3, 2). 3 3
Answer: (B) Answer: (D)

Integer Answer Type Questions INTEGER-TYPE QUESTIONS


1. The area of the quadrilateral formed by the lines 3. The orthocentre of the triangle formed by the lines
x y  1 is __________ sq. unit. x y  1, 2x 3y  6 and 4x – y 4  0 lies in the quad-
rant whose number is __________.
Solution: The given quadrilateral is a square with ver-
tices (1, 0), (0, 1), ( 1, 1) and (0, 1), and hence its area
is ( 2 )2  2.
Solution: Solving the above equations taken two by
two, the vertices of the triangle are
The questions in this section are numerical
Answer: 2 ¥ 3 8 ´
§ 5 5¶
¥ 3 16 ´
A ¦ , µ , B  3, 4 , and C ¦ , µ
§ 7 7¶
problems for which no choices are provided.
2. Two rays in the first quadrant, x y  a and ax – y 
1, intersect each other in the interval a (a0 , d). The
value of a0 is __________. (IIT-JEE 2006)
The equation of the altitude drawn from A to the side The students are required to find the exact
BC is
Solution: Solving the given two equations, we have
y
8 3¥ 3´
 ¦x µ
answers to numerical problems and enter the
1 a a(1 a ) a a 1 5 2§ 5¶
x
1 a
and y  ax 1 
1 a
1
1 a  3 x 2 y  5 (2.126) same in OMR sheets. Answers can be one-
Since the two rays intersect each other in the first quad- Again the equation of the altitude from B onto CA is
rant, we have x  0 and y  0 which implies that 1
y 4  ( x 3)
digit or two-digit numerals.
1 a  0 and a a 1  0 4
Therefore, if 1  a  0, then the a( a) 1  0 which is not  x 4 y  13 (2.127)
sensible. Hence, a Ž( 1, 0). If a  0, then the lines x y  0 Solving Eqs. (2.126) and (2.127), the coordinates of the
and y  1 intersect in fourth quadrant. Thus, a x 0. orthocentre are
Hence, a  0 and a 2 1  0  a  1. Therefore, a0  1.
Answer: 1
EXERCISES
For self-assessment, each chapter has ad-
equate number of exercise problems where
EXERCISES
the questions have been subdivided into
Single Correct Choice Type Questions
1. Equation of the line through (0, 3) and having slope ¥ 19 5 ´
(A) ¦ , ¥ 19 5 ´
(B) ¦ , µ
various categories which include Multiple
2 is § 8 4 µ¶ § 8 4¶
(A) y 2 x 3  0 (B) y 2x 3  0
¥ 5 19 ´ ¥ 5 19 ´
Choice Questions as asked in JEE (Main &
(C) ¦ , µ (D) ¦ ,
(C) y 2x 3  0 (D) y 2 x 3  0 § 4 8¶ § 4 8 µ¶

2. Equation of the line passing through ( 5, 2) and (3, 2) 5. If the area of the triangle formed by the line 2x
Advanced).
is 3y c  0 with coordinate axes is 27 sq. units, then
c is equal to
(A) x 2  0 (B) y 2  0
(C) x 2  0 (D) y 2  0 (A) p16 (B) p15 (C) p8 (D) p18

Multiple Correct Choice Type Questions


1. If the distance of the line 8x 15y K  0 from the 3
point (2, 3) is equal to 5 units, then the value of L is (A) the slope of l 
2
(A) 24 (B) 24 (C) 146 (D) 146 (B) the line l passes through (0, 0)
(C) the intercept on the axes are 2, 3
2. If the line 3 x y – 9  0 is reduced to the form xcos@ (D) the line l forms a triangle of area 5 sq. units with
ysin @  p, then the coordinate axes
(A) A  60o (B) A  30o
4. If l is the line passing through the point (2, 3) and is
9 parallel to the line joining the points (4, 1) and ( 2, 3),
(C) p  (D) p  9
2 then

3. If l is the line passing through the point ( 2, 3) and (A) ( 10, 1) is a point on l
perpendicular to the line 2x 3y 6  0, then (B) the slope of l is 6

Matrix-Match Type Questions


In each of the following questions, statements are given in 1. Let S be the system of lines passing through the
two columns, which have to be matched. The statements in intersection of the lines x y – 1  0 and x – y – 1 
column I are labeled as (A), (B), (C) and (D), while those 0. Match the items of Column I with those of Col-
in column II are labeled as (p), (q), (r), (s) and (t). Any umn II.
given statement in column I can have correct matching with
one or more statements in column II. The appropriate bub- Column I Column II
bles corresponding to the answers to these questions have
(A) Equation of the line (p) 2x – y – 2  0
to be darkened as illustrated in the following example.
belonging to S and
Example: If the correct matches are (A) m (p), (s), (B) m passing through the
(q), (s), (t), (C) m (r), (D) m (r), (t), that is if the matches point (2, 3) is
are (A) m (p) and (s); (B) m (q), (s) and (t); (C) m (r); (B) Equation of the line (q) x y – 1  0
and (D) m (r), then the correct darkening of bubbles will belonging to S and
look as follows: parallel to the line y 
t 2 1i

Comprehension Type Questions


1. Passage: Let u y x y  0, A  (1, 2) and B  (3, –1). (i) Area of the triangle in square units is
Answer the following questions.
1 2 1 1
(A) (B) (C) (D)
(i) If M is a point on the line u  0 such that AM 3 3 2 3 2 2
BM is minimum, then the reflection of M on the
line y x is (ii) The gradients of the two sides AB and AC are
(A) (2, –2) (B) (–2, 2) 1 1
(A) 3, (B) 2,
(C) (1, –1) (D) (–1, 1) 3 2
(ii) If M is a point on u  0 such that AM BM is (C) 2 1, 2 1 (D) 2 3 , 2 3
maximum, then the distance between M and the (iii) The circumradius of the triangle is
point N(1, 1) is
1 2 1 1
(A) 3 5 (B) 5 2 (C) 7 (D) 10 (A) (B) (C) (D)
3 3 3 2
(iii) If M i i 0 h h AM BM i

Integer Answer Type Questions


The answer to each of the questions in this section is a 4. In $ABC, the equations of the madians AD and BE,
non-negative integer. The appropriate bubbles below the respectively, are 2x 3y – 6  0 and 3x – 2y– 10  0.
respective question numbers have to be darkened. For 1
If AD  6, BE  11, then (Area of $ABC ) is
example, as shown in the figure, if the correct answer to 11
________.
the question number Y is 246, then the bubbles under Y
labeled as 2, 4, 6 are to be darkened.
5. P(1, 2), Q(4, 6), R(5, 7) and S(a, b) are the vertices
X Y Z W of the parallogram PQRS. Then, a b is equal to
0 0 0 0 _______.
1 1 1 1
6. The area of the triangle formed by the line x
2 2 2
y  3 and the angle bisectors of the pair of lines
3 3 3 3
x 2 y2 2 y 1  0 is ______ sq. unit.
4 4 4
5 5 5 5 7. A straight line through the origin O meets the par-
ll l li 4 2 9 d2 6 i P d
ANSWERS
ANSWERS
The Answer key at the end of each chapter Single Correct Choice Type Questions
contains answers to all exercise problems. 1. (A) 11. (D)

2. (B) 12. (D)

3. (D) 13. (A)

4. (B) 14. (B)

5. (A) 15. (C)


6. (D) 16. (B)

7. (B) 17. (D)

8. (A) 18. (C)

9. (D) 19. (B)


10. (B) 20. (C)

Multiple Correct Choice Type Questions


1. (B), (C) 4. (A), (B), (C)
2. (A), (C) 5. (A), (B), (C)

3. (A), (C)

Matrix-Match Type Questions


1. (A) m (q); (B) m (t); (C) m (p), (r); (D) m (s) 3. (A) m (t); (B) m (p); (C) m (q); (D) m (r)

2. (A) m (r); (B) m (p); (C) m (s); (D) m (t)

Comprehension Type Questions


1. (i) m(A); (ii) m (C); (iii) m (A) 3. (i) m(C); (ii) m (B); (iii) m (A)

2. (i) m(D); (ii) m (A); (iii) m (D)

Integer Answer Type Questions


1. 4 4. 1

2. 5 5. 3

3. 3 6. 3
BOOK FEEDBACK FORM

WE WOULD LIKE TO HEAR FROM YOU

Please complete this form. Your feedback concerning “Mathematics for JEE (Main & Advanced)”
will be appreciated. If you do want to complete and submit the form online, you may visit us at www.
wileyindia.com/murti. You can also fill-in the form and send it as an attachment via E-mail or Fax it to
+91-11-23275895.
Our e-mail address is acadmktg@wiley.com and mailing address is Wiley India Pvt. Ltd, Attn:
Marketing Department, 4436/7, Ansari Road, Daryaganj, New Delhi- 110002, India.

* Fields marked are compulsory

* Name

* Email Address

Address
Cut from here

Phone Mobile
School/Institute/College

* Name of Course Instructor

* Competitive Exam(s) Studying for

How did you hear about this book? Bookseller Classmate Teacher Advertisement
Other
(please mention)

Would you be interested in further readings by the authors? Yes No

Would you be interested in our other bestsellers from the Wiley MAESTRO series? Yes No

Your Comments

May we use your comments in future promotions? Yes No



Contents
1 Rectangular Coordinates, Basic Formulae, Locus and Change of Axes 1
1.1 Rectangular Coordinates ................................................................................................................................. 2
1.2 Basic Formulae................................................................................................................................................. 3
1.3 Locus ................................................................................................................................................................ 6
1.4 Change of Axes................................................................................................................................................ 7
Worked-Out Problems ................................................................................................................................... 11
Summary ........................................................................................................................................................ 18
Exercises ........................................................................................................................................................ 19
Answers .......................................................................................................................................................... 20

2 Straight Line and Pair of Lines 21


2.1 Straight Line ................................................................................................................................................... 22
2.2 Pair of Lines .................................................................................................................................................... 42
Worked-Out Problems ................................................................................................................................... 72
Summary ...................................................................................................................................................... 115
Exercises ...................................................................................................................................................... 118
Answers ........................................................................................................................................................ 123

3 Circle 125
3.1 Introduction.................................................................................................................................................. 126
3.2 Relation Between a Circle and a Line in its Plane ........................................................................................ 128
3.3 Classification of Points in a Plane w.r.t. a Circle in the Same Plane ............................................................. 133
3.4 Relation Between Two Circles ...................................................................................................................... 150
3.5 Common Tangents to Two Circles ............................................................................................................... 159
Worked-Out Problems ................................................................................................................................. 170
Summary ...................................................................................................................................................... 205
Exercises ...................................................................................................................................................... 208
Answers ........................................................................................................................................................ 210

4 Parabola 211
4.1 Conic Section ............................................................................................................................................... 212
4.2 Parabola ....................................................................................................................................................... 213
Worked-Out Problems ................................................................................................................................. 249
Summary ...................................................................................................................................................... 276
Exercises ...................................................................................................................................................... 278
Answers ........................................................................................................................................................ 283
xx Contents

5 Ellipse and Hyperbola 285


5.1 Ellipse........................................................................................................................................................... 286
5.2 Inverted Ellipse ............................................................................................................................................ 287
5.3 Hyperbola .................................................................................................................................................... 322
Worked-Out Problems ................................................................................................................................. 345
Summary ................................................................................... ..................................................................373
Exercises ..................................................................................................................................................... 377
Answers ....................................................................................................................................................... 383

6 Three-Dimensional Geometry 385


6.1 Pre-Requisites .............................................................................................................................................. 386
6.2 Coordinates, Direction Cosines and Direction Ratios .................................................................................. 399
6.3 Plane ............................................................................................................................................................ 410
6.4 Line .............................................................................................................................................................. 421
Worked-Out Problems ................................................................................................................................. 430
Summary ................................................................................... ..................................................................454
Exercises ..................................................................................................................................................... 456
Answers ....................................................................................................................................................... 461

Index 463
Rectangular
Coordinates, Basic
Formulae, Locus and
Change of Axes
1
y Contents
1.1 Rectangular Coordinates
1.2 Basic Formulae
Basic Formulae, Locus and

1.3 Locus
−x x
Rectangular Coordinates,

1.4 Change of Axes

Worked-Out Problems
−y
Summary
Exercises
y
Answers

−z x

The locus of a point is the


−x
Change of Axes

z path traced out by the point


when it moves according to a
given rule (or rules). In other
−y
words, a locus is the path of
90°
a single moving point that
obeys certain conditions.

180° 0°

270°
Equiangular spiral
2 Chapter 1 Rectangular Coordinates, Basic Formulae, Locus and Change of Axes

1.1 Rectangular Coordinates


Geometry is a thought-provoking subject for any genuine mathematics student. Geometry was initially pursued by the
Indians and Greeks. That kind of geometry is called pure geometry. Even though pure geometry is very interesting,
sometimes the proofs needed constructions and also they were cumbersome. At this stage, the concept of studying
geometry by using algebra was introduced by Rene Descartes (1596 1650 AD). Thus, the modern analytic geometry
emerged and is called “Cartesian geometry” named after Rene Descartes. In the following section, we discuss the
rectangular Cartesian coordinates.

1.1.1 Rectangular Cartesian Coordinates


FHHHHHHE FHHHHHHE
Select a plane and in that plane, let X aOX and Y aOY be two perpendicularly intersecting lines (intersecting at O).
FHHHHHHE FHHHHHHE HHHHE HHHHE
X aOX is called
HHHHHE x-axis, YE aOY is called y-axis and O is called the origin. Further, OX and OY are called positive direc-
HHHHH
tions, and OX a and OY a are called negative directions. Let P be a point in the plane. From P, draw perpendicular PL
to x-axis and PM perpendicular to y-axis (see Fig. 1.1).
Y

M P

X O L X

Y

FIGURE 1.1

Let the magnitudes of OL and OM be x and y, respectively.


HHHE HHHHE HHHHE HHHHE
1. If OL and OM are in the directions of OX and OY , then we say that x is the x-coordinate of P and y is the
y-coordinate of P and we write P  (x, y).
HHHE HHHHHE HHHHE HHHHE
2. If OL is in the direction of OX a and OM is in the direction of OY , then we write P  ( x, y).
HHHE HHHHE HHHHHE HHHHHE
3. If both OL and OM are in the directions of OX a and OY a, then we write P  ( x, y).
HHHE HHHHE HHHHE HHHHHE
4. If OL is in the direction of OX and OM is in the direction of OY a, then we write P  (x, y).

QUICK LOOK 1

O  (0, 0), P (x, 0) lies on the x-axis, Q  (0, y) lies on the y-axis.

HHHHE HHHHE HHHHHE HHHHE HHHHHE HHHHHE HHHHE HHHHHE


DEFINITION 1.1 Quadrants The regions bounded by ( OX , OY ), ( OX a , OY ), ( OX a , OY a ) and ( OX , OY a )
are the first, second, third and fourth quadrants, respectively.
Sign of the Coordinates
1. (x, y) lies in the first quadrant šx 0, y 0.
2. (x, y) lies in the second quadrant šx  0, y  0.
3. (x, y) lies in the third quadrant šx  0, y  0.
4. (x, y) lies in the fourth quadrant šx 0, y 0.
From the above list one can conclude that ( 2 , 1), ( 3, 2 ), ( 2, 3/2) and (1/2, 3 ) belong to the first, second,
third and fourth quadrants, respectively.
1.2 | Basic Formulae 3

1.2 Basic Formulae


In this section, we recall some of the basic formulae which have been discussed in your earlier mathematics classes in
school. We state such formulae without proofs.

1.2.1 Distance Between Two Points


1. The distance between two points A(x1, y1) and B(x2, y2) is given by

AB  ( x1 x2 )2 ( y1 y2 )2

2. Distance between origin O and the point P(x, y) is

OP  ( x 0)2 ( y2 0)2  x 2 y2
3. If A  (x1, 0) and B  (x2, 0), then
AB  x1 x2
Also if A  (0, y1) and B  (0, y2) then

AB  y1 y2

1.2.2 Notation
Let AFHHH
E B be two points. Thus, the line segment
and HHHHE connecting A and B is denoted by AB and the line through A Hand
HHHE
B by AB . The ray from A to B is denoted by AB (readers please observe the arrowheads in all cases). In vectors AB
means a line segment AB having direction from A to B.

1.2.3 Section Formulae


1. Let A and B be two points and P be a point on AB lying between A and B. Then we say that P divides AB inter-
nally in the ratio AP:PB (see Fig. 1.2).

A P B

FIGURE 1.2

FHHHE
2. If P lies on the line AB (not in between A and B), then we say that P divides externally the segment AB and we
write the ratio as –(AP):PB or AP:–PB. The minus sign indicates external division.
3. The coordinates of a point P which divides the segment joining A(x1, y2) and B(x2, y2) in the ratio m:n (m n ≠ 0)
are

¥ mx2 nx1 my2 ny1 ´


P¦ , µ
§ m n m n ¶

If m/n is positive, then division is internal division and if m/n is negative, the division is external division.

QUICK LOOK 2

The coordinates of the midpoint of the segment joining A(x1, y1) and B(x2, y2) are

¥ x1 x2 y1 y2 ´
¦§ , µ
2 2 ¶
4 Chapter 1 Rectangular Coordinates, Basic Formulae, Locus and Change of Axes

Examples

1. The distance between the points A(2, 3) and B( 2, 2) 2 2 2


¥ 1´ ¥ 1´ 1 ¥ 1´
is 2 2
(2 2) (3 2)  17 . a 2 ¦ t 2 2 µ 4a 2 ¦ t µ  a t ¦t µ 4
§ t ¶ § t¶ t § t¶
2. The distance between the points P(cos @, cos A ) 2
and Q (sin @, sin A ) is 1 ¥ 1´
 a t ¦t µ
t § t¶
(cos A sin A )2 (cos B sin B )2  2 sin 2A sin 2B
2
¥ 1´
 a ¦t µ
3. The distance between the points A(at2, 2at) and § t¶
B(a/t 2 , 2a/t ) is 2
¥ 1´
 a¦t µ if a  0
§ t¶

Examples

1. The coordinates of the point which divides the seg- 2. If P divides AB in the ratio 2:1, then
ment joining A(2, 3) and B(3, 2) in the ratio 1:2
are ¥ 8 1´
P¦ , µ
¥ 2 s 2 1 s 3 2 s 3 1 s 2 ´ ¥ 7 4 ´ § 3 3¶
¦§ , µ¶  ¦§ , µ¶
1 2 1 2 3 3

Convention: If a point P divides the line joining A and B internally in the ratio 1:2 or 2:1, then P is called point of
trisection of AB .

1.2.4 Area of a Triangle


The area of the triangle whose vertices are A(x1, y1), B(x2, y2) and C(x3, y3) is
1 1
x1 ( y2 y3 ) x2 ( y3 y1 ) x3 ( y1 y2 )  det A
2 2

¨ x1 x2 x3 ·
where A  ©© y1 y2 y3 ¸¸ is a 3 × 3 matrix.
©ª 1 1 1 ¹¸

Generally, area of a triangle is denoted by $ (see Vol. 2).

QUICK LOOK 3

Three points A(x1, y1), B(x2, y2), C(x3, y3) are collinear if and only if
x1 x2 x3
y1 y2 y3  0
1 1 1

QUICK LOOK 4

Area of quadrilateral ABCD  Area of $ABC Area of $ACD


1.2 | Basic Formulae 5

1.2.5 Some Points and Circles Associated with a Triangle


In this section, we will discuss some important terms associated with a triangle. The vertices of the triangle would be
considered as A, B, C.

Centroid
The point of concurrence of the medians of a triangle is called the centroid of the triangle and is denoted by G. The
coordinates of the centroid of the triangle with vertices (x1, y1), (x2, y2) and (x3, y3) are

¥ x1 x2 x3 y1 y2 y3 ´
¦§ , µ¶
3 3

Note that centroid trisects each median.

Incentre
The internal bisectors of the angles of a triangle are concurrent. This point is called the incentre of the triangle and is
denoted by I. The incentre is equidistant from three sides and this equal distance r is called the inradius of the triangle.
If a circle is drawn with centre at I and radius r, then this circle touches the sides of the triangle internally. (For more
details, see Chapter 4, Vol. 2.)
Incentre formulae
Let A(x1, y1), B(x2, y2) and C(x3, y3) be the vertices of a triangle and BC a, CA b and AB c. Thus, the incentre is

¥ ax bx2 cx3 ay1 by2 cy3 ´


I¦ 1 , µ
§ a b c a b c ¶

Excentre
The point where the external bisectors of two angles and the internal bisector of one angle are concurrent is called the
excentre. Thus, there are three excentres, namely, excentre opposite to the vertex A (denoted by I1), excentre opposite
to the vertex B (denoted by I2) and excentre opposite to the vertex C (denote by I3). Also

¥ ax1 bx2 cx3 ay1 by2 cy3 ´


I1  ¦ , µ
§ a b c a b c ¶
¥ ax bx2 cx3 ay1 by2 cy3 ´
I2  ¦ 1 , µ
§ a b c a b c ¶
¥ ax bx2 cx3 ay1 by2 cy3 ´
I3  ¦ 1 , µ
§ a b c a b c ¶

Note: For inradius and three exradii, see Chapter 4, Vol. 2.

Circumcentre, Circumradius and Circumcircle


The point where the perpendicular bisectors of the sides of a triangle are concurrent is called the circumcentre of
the triangle. It is equidistant from the vertices of the triangle. This equal distance is denoted by R and is called the
circumradius. Thus, if a circle is drawn with the circumcentre as centre and circumradius R as radius, then that circle
will pass through the vertices of the triangle. Such a circle is called the circumcircle of the triangle.

Orthocentre
The point of concurrence of the three altitudes of a triangle is called the orthocentre of the triangle.

Note: The circumcentre and orthocentre lie inside the triangle if the triangle is acute.
6 Chapter 1 Rectangular Coordinates, Basic Formulae, Locus and Change of Axes

Nine-Point Circle and Nine-Point Centre


In a triangle, the feet of the altitudes, the midpoints of the three sides and the midpoints of the segments joining the
orthocentre and the vertices are concyclic. Such a circle is called the nine-point circle and its centre is called the nine-
point centre of the triangle. Also the nine-point centre N is the midpoint of the segment joining the circumcentre and
the orthocentre. The radius of the nine-point circle is half of the circumradius of the triangle because if $DEF is the
pedal triangle of $ABC, then the angles of $DEF are 180o 2A, 180o 2B and 180o 2C and the sides are a cos A,
b cos B and c cos C (see Theorem 4.23, page 223, Chapter 4, Vol. 2). If we use sine rule to $DEF, then we obtain that
R/2 is the circumradius of $DEF which is the radius of the nine-point circle.

QUICK LOOK 5

Nine-point circle of $ABC is the circumcircle of the pedal triangle of $ABC as well as the circle passing through
the midpoints of the sides.

IMPORTANT NOTE

In a triangle ABC, 2. The centroid G divides the segment joining the cir-
cumcentre and orthocentre in the ratio 1:2.
1. The circumcentre, the centroid, the nine-point cen-
tre and the orthocentre are collinear in the given or- 3. The nine-point centre is the midpoint of the segment
der (see Definition 4.7, page 228, Chapter 4, Vol. 2). joining the circumcentre and the orthocentre.

Pedal Line (or Simson’s Line)

T H E O R E M 1.1 The feet of the perpendiculars drawn from a point on the circumcircle of a triangle onto its sides
are collinear. This line is called Pedal line or Simson’s line of the triangle. The converse of this theo-
rem is also true. That is, if from any point in the plane of a triangle, the feet of the perpendiculars
onto the sides are collinear, then the point lies on the circumcircle.
We can prove these two results by using plane geometry or what is called pure geometry. The
line LMN is the Pedal line (see Fig. 1.3).

N
A
P
M

L
B

FIGURE 1.3

1.3 Locus
In pure geometry, using congruent triangles property, it was proved that all the points on a line bisecting perpendicu-
larly the segment joining two given points are equidistant from the two given points and this line is called the perpen-
dicular bisector of the segment joining the two points. Of course, any point which is equidistant from these two given
points lies on this line. Thus, describing a set of points satisfying a certain geometrical condition(s) is difficult in pure
1.4 | Change of Axes 7

geometry. That is why mathematicians introduced analytical geometry (a combination of algebra and pure geometry)
and described the locus by algebraic equations. In this section, we introduce the concept of locus, equation of a locus
and few examples.

DEFINITION 1.2 Locus Let P be a geometrical condition(s) and S be the set of all points in the plane which
satisfy P. Then S is called a locus.

QUICK LOOK 6

S is the locus of a geometrical condition(s) P š every point of S satisfies the condition P and every point
satisfying P belongs to S.

DEFINITION 1.3 Equation of the Locus Let S be a locus and f(x, y)  0 be an algebraic equation in x and y. If
every point P(x, y) belonging to S satisfies the equation f(x, y)  0 and any point in the plane
satisfying the equation f(x, y)  0 belongs to S, then f(x, y)  0 is called the equation of the
locus S.
Here afterwards, we will describe a locus by its algebraic equation.

1.4 Change of Axes


In analytical plane, selection of the rectangular coordinate axes is arbitrary. When the axes change, the coordinates of
the point also change. The study of the relations between the original coordinates and the changed coordinates is
called change of axes. This change of axes is sometimes necessary to make the equation of a curve as simple as possible
to prove certain properties. Not that the same properties could not be proved otherwise, but the working out of the
proof would be more complicated. It is by experience that the student will learn the best method for change of axes.
In the following, we discuss three types of change of axes. We begin with shifting of the origin.

1.4.1 Shifting of Origin Without Changing the Directions of the Axes

y Y

X O N X

x O (0, 0) L M x

Y
y

FIGURE 1.4
FHHHHE FHHHHE FHHHHHHHE FHHHHE
Let x aOx and y a Oy be the coordinate axes. Let Oa( x1 , y1 ) be a point. Through Oa draw lines X aOa X parallel to xa Ox
FHHHHHHE FHHHHE FHHHHHHE FHHHHE FHHHHE FHHHHE
and Y aOa Y parallel to ya Oy . Suppose Y aOaY meets x aOx in L. We call ( x aOx , y a Oy ) as the old system of axes and
FHHHHHHHE FHHHHHHE
( X aOa X , Y aOa Y ) as the new system of axes. Now, every point in the coordinate plane will have two systems of coor-
dinates, namely old coordinates (with respect to old axes) and new coordinates (with respect to new axes).
Suppose P is a point whose old and new coordinates are (x, y) and (X, Y), respectively. Draw PM perpendicular to
old x-axes meeting the new X-axes in N. Now
x  OM  OL LM  OL Oa N  x1 X  X x1
8 Chapter 1 Rectangular Coordinates, Basic Formulae, Locus and Change of Axes

and
y  PM  PN NM  PN Oa L  Y y1
Thus, the relations between old and new coordinates of the point P are
x  X x1
y  Y y1 (1.1)

QUICK LOOK 7

1. Shifting of the origin is also called TRANSLA- 2. Equation f(x, y)  0 of a curve will be changed to
TION of axes. The effect on the coordinates is f(X x1, Y y1)  0.
Old coordinate  New coordinate Corresponding
coordinate of the new origin

Note: The new origin Oa may be in any quadrant. Still the relation between old and new coordinates is the same.

1.4.2 Rotation of Axes (Without Changing the Origin)


FHHHHE FHHHHE HHHE
Let x aOx and y a Oy be original axes (old axes) (see Fig. 1.5). Rotate Ox about O through an angle P in the anticlock-
FHHHHHHE FHHHHHHE HHHHHE HHHHHE
wise sense. Let the new axes be X aOX and Y aOY ( OX a and OY a are not shown in the figure). Let P be a point in
FHHHHE
the plane and let its old and new coordinates be (x, y) and (X, Y), respectively. Draw PL perpendicular to x aOx (old
FHHHHHHE
x-axis) and PM perpendicular to X aOX (new X-axis). Draw MQ perpendicular to old x-axis and MN perpendicular
PL. Now
QOM  Q  NMO  PMN  90B Q

so that NPM  Q . Also

x  OL  OQ LQ MN  OQ MN (1.2)
From $MOQ,
OQ
cos Q 
OM

y
Y

P(x, y )
(X, Y ) X

q
N M

q
x O L Q x

y

FIGURE 1.5
1.4 | Change of Axes 9

Therefore
OQ  OM cos Q  X cos Q (1.3)
Also, from $PNM,

MN
sin Q 
PM
Therefore
MN  PM sin Q  Y sin Q (1.4)
Hence from Eqs. (1.2), (1.3) and (1.4), we have
x  X cos Q Y sin Q

Again,

y  PL
 PN NL
 PN MQ
 MQ PN
 X sin Q Y cos Q
Therefore
y  X sin Q Y cos Q
The above-mentioned relations (in color screen) can be written in the form of a matrix equation as follows:
¨cos Q sin Q · ¨ X · ¨ x ·

© sin Q
ª cos Q ¸¹ ©ªY ¸¹ ©ª y ¸¹

Notes:
1. If the origin is shifted to the point (x1, y1) and the axes are rotated through P in the anticlockwise sense, then the
old coordinates (x, y) will be transformed to
x  X cos Q Y sin Q x1 , y  X sin Q Y cos Q y1
2. If the rotation is clockwise, then we have to replace P with –P.

DEFINITION 1.4 If a, b, h are real and at least one of a, h, b is not zero, then ax 2 2hxy by2 is called second
degree homogeneous expression and ax 2 2 hxy by2  0 is called second degree homogeneous
equation.

DEFINITION 1.5 If a, h, b, g, f, c are real and at least one of a, h, b is not zero, then ax 2 2 hxy by2 2 gx 2 fy c
is called second degree general expression and ax 2 2 hxy by2 2 gx 2 fy c  0 is called
second degree general equation.

Examples

1. 2 x 2 xy y2 3. 2 x 2 3 xy y2 x y 1
2. x 2 2 xy y2 4. x 2 2 xy y2 x y 1

DEFINITION 1.6 In the second degree general expression, ax2, 2hxy, by2 are called second degree terms; gx, fy
are called first degree terms; and c is called constant term.
10 Chapter 1 Rectangular Coordinates, Basic Formulae, Locus and Change of Axes

T H E O R E M 1.2 If h2 ≠ ab, then to remove the first degree terms of the equation ax 2 2 hxy by2 2 gx 2 fy
¥ hf bg gh af ´
c = 0, the origin is to be shifted to the point ¦ , .
§ ab h2 ab h2 µ¶

PROOF Suppose that the origin is shifted to the point (x1, y1) and let x  X x1, y  Y y1. Therefore, the
given equation is transformed to
a( X x1 )2 2 h( X x1 )(Y y1 ) b(Y y1 )2 2 g( X x1 ) 2 f (Y y1 ) c  0
This implies
aX 2 2 hXY bY 2 2(ax1 hy1 g ) X 2(hx1 by1 f )Y
ax12 2 hx1 y1 by12 2 gx1 2 fy1 c  0
This further implies
aX 2 2 hXY bY 2 2(ax1 hy1 g ) X 2(hx1 by1 f )Y
(ax1 hy1 g ) x1 (hx1 by1 f ) y1 ( gx1 fy1 c)  0 (1.5)

Now, put ax1 hy1 g  0 and hx1 by1 f  0. Solving we get


hf bg gh af
x1  2
and y1 
ab h ab h2
¥ hf bg gh af ´
Therefore, the origin is to be shifted to the point ¦ , so that Eq. (1.5) will be
§ ab h2 ab h2 µ¶

aX 2 2 hXY bY 2 gx1 fy1 c  0


where x1 and y1 are as defined above.

IMPORTANT NOTE

1. Under shifting of origin, the second degree terms of 2. Under change of axes, only the coordinates of the
a second degree equation will not change. points will change, but the distance between two
points as well as the areas will not change.

T H E O R E M 1.3 To remove the xy-term from the equation ax 2 2 hxy by2  0, the axes are to be rotated through
the angle (1/2) Tan 1 [2 h/(a b)] when a x b and through the angle P /4 when a  b.
PROOF Suppose axes are rotated through an angle P in the anticlockwise sense. Let x  X cos Q Y sin Q
and y  X sin Q Y cos Q . Therefore, the given equation will be transformed to

a( X cos Q Y sin Q )2 2 h( X cos Q Y sin Q )( X sin Q Y cos Q ) b( X sin Q Y cos Q )2  0


In this equation, the coefficient of xy is
2a sin Q cos Q 2 h(cos2 Q sin 2 Q ) 2b sin Q  0
 (b a)sin 2Q 2 h cos 2Q  0
2h
 tan 2Q  , if a x b
a b
1 ¥ 2h ´
 Q  Tan 1 ¦ , if a x b
2 § a b µ¶

If a  b, then cos 2Q  0  2Q  P /2 or Q  P /4.


Worked-Out Problems 11

WORKED-OUT PROBLEMS
Since this chapter is only to recall what the students have 4. Show that the three points P(a, b c), Q(b, c a) and
learnt in their junior classes and practice the important R(c, a b) are collinear.
formulae, we give a combination of both subjective and
Solution: We have
objective type questions here. Students should practice
all questions. 1 a(c a a b) b(a b b c)
Area of $PQR 
2 c(b c c a)
1
Questions Based on Basic  ac ab ba bc cb ca
2
Formulae
1
1. Find the area of the triangle whose vertices are  0
2
A(2, 3), B(4, 2) and C( 5, 2). 0
Solution: We have Therefore P, Q, R are collinear (see Quick Look 3).
1
Area  x1 ( y2 y3 ) x2 ( y3 y1 ) x3 ( y1 y2 ) 5. If the three points A(3, 1), B(2K, 3K) and C(K, 2K) are
2
collinear, find the value of K.
1
 2[2 ( 2)] 4[ 2 ( 3)]] 5( 3 2)
2 Solution: A, B and C are collinear implies
1 Area of $ABC  0
 8 4 25
2
1
37  3(3L 2 L ) 2 L (2 L 1) L (1 3L )  0
 sq. units 2
2
1
 3L 4 L 2 2 L L 3L 2  0
2. Show that the area of the triangle with vertices 2
A( 3, 4), B(6, 2) and C(4, 3) is 24.5 sq. units.  L 2 2L  0
Solution: We have  L  0 or L  2
1 Now
Area  x1 ( y2 y3 ) x2 ( y3 y1 ) x3 ( y1 y2 )
2 K  0 B  C so that A, B and C are collinear.
1 K  2 A  (3, 1), B  ( 4, 6) and C  ( 2, 4)
 3(2 3) 6( 3 4) 4(4 2)
2
1 6. If the three points (a, 0), (0, b) and (2, 2) are collinear,
 15 42 8
2 then show that
1
 49 1 1 1
2 
a b 2
 24.5 sq. units
Solution: By hypothesis
3. Prove that the points A(x, x – 2), B(x 3, x) and C(x 2,
1
x 2) form a triangle whose area is independent of x. a(b 2) 0(2 0) 2(0 b)  0
2
Solution: We have  ab 2a 2b  0
1  2a 2b  ab
Area of $ABC  x[ x ( x 2)] ( x 3)[ x 2 ( x 2)]
2 1 1 1
 
( x 2)( x 2 x) a b 2
1
 2 x 4 x 12 2 x 4 7. The points (1, 2), (2, 4) and (t, 6) are collinear. Find t.
2
8 Solution: By hypothesis the area of the triangle is zero.

2 Therefore
 4 sq. units which is independent of x
12 Chapter 1 Rectangular Coordinates, Basic Formulae, Locus and Change of Axes

1 bisect each other. Hence, AC and BD bisect each other.


1(4 6) 2(6 2) t(2 4)  0 That is, AC and BD have the same point as their mid-
2
1 point.
 2 8 2t  0 Therefore
2
 6 2t  0 ¥ x1 x3 y1 y3 ´ ¥ x2 x y2 y ´
¦§ , µ ¦ , µ
t4 2 2 ¶ § 2 2 ¶
 x1 x3  x2 x, y1 y3  y2 y
8. If O is the origin and Q( 2, 4) is a point on OP such
 x  x1 x2 x3 , y  y1 y2 y3
that OQ  (1/3)OP, find the coordinates of P.
Solution: Let P  (x, y). See Fig. 1.6. Now, D C(x 3, y3)
OQ  (1/3)OP OP  3OQ
OQ:QP  1:2
Therefore A(x 1, y1) B(x 2, y2)
¥ x y´ FIGURE 1.7
( 2, 4)  Q  ¦ , µ
§ 3 3¶
x y Note: The student can remember it easily.
  2,  4
3 3
 x  6, y  12 11. If P(1, 2), Q(4, 6), R(5, 7) and S(a, b) are the verti-
ces of a parallelogram PQRS, then find the values of
 P  ( 6, 12) a and b. (IIT-JEE 1998)
1 2 Solution: According to the above formula (Problem
O (0, 0) Q (−2, −4) P (x , y ) 10), we have
FIGURE 1.6 a1 5–42
b2 7–63
9. If x1, x2, x3 as well as y1, y2, y3 are in GP with the same
common ratio, then show that the points (x1, y1), 12. Show that the four points A( a, b), O(0, 0), B(a, b)
(x2, y2) and (x3, y3) are collinear. (IIT-JEE 1999) and C(a2, ab) are collinear.
Solution: Let x2  x1k, x3  x1k2 and y2  y1k, y3  y1k2. Solution: Since O(0, 0) is the midpoint of AB it fol-
Then lows that
Points A, O and B are collinear (1.6)
x1 x2 x3 x1 x1k x1k 2
Now,
y1 y2 y3  y1 y1k y1k 2
1 1 1 1 1 1 a 0 a 2 1 0 a
b 0 ab  ab 1 0 a  0 (& two rows are identical)
1 k k2 1 1 1 1 1 1
 x1 y1 1 k k 2
Points A, O and C are collinear (1.7)
1 1 1
Statements (1.6) and (1.7) A, O, B, C are collinear.
 0 (sin
nce two rows are identical)
13. Let O(0, 0), P(3, 4), Q(6, 0) be the vertices of the tri-
Hence, the points are collinear (by Quick Look 3).
angle OPQ. The point R lies inside the $OPQ such
10. In a parallelogram ABCD if (x1, y1), (x2, y2) and
that the triangles OPR, PQR, OQR are of equal area.
(x3, y3) are the coordinates of A, B, C, respectively, The coordinates of R are
then show that the coordinates of D are (x1 x3 – x2, (A) (4/3, 3) (B) (3, 2/3)
y1 + y3 – y2).
(C) (3, 4/3) (D) (4/3, 2/3)
Solution: Suppose the coordinates of D are (x, y). See
(IIT-JEE 2007)
Fig. 1.7. It is known that in a parallelogram the diagonals
Worked-Out Problems 13

Solution: A point inside a triangle divides the triangle Solution: Suppose C  (x, y). Then
into three triangles of equal areas if and only if the point
is the centroid of the triangle. Hence, R must be the cen- ¥ 4 5 x 3 2 y ´
(0, 0)  ¦ , µ¶
troid of $OPQ. Therefore § 3 3
4 5 x 3 2 y
¥ 0 3 6 0 4 0´ ¥ 4´   0 and 0
R¦ , µ  ¦ 3, µ 3 3
§ 3 3 ¶ § 3¶  x  1, y  1
Answer: (C) So the coordinates of C are (1, 1).
14. An integral point means that both coordinates of 17. If A(a, b), B(a r cos @, b r sin @ ) and C(a r cos A,
the point are integers. The number of integral points b r sin A ) are the vertices of an equilateral triangle,
exactly in the interior of the triangle with vertices then
(0, 0), (0, 21) and (21, 0) (see Fig. 1.8) is
(A) A B  π/4 (B) A B  π/2
(A) 133 (B) 190 (C) 233 (D) 105
(C) A B  π/6 (D) A B  π/3
Solution: The integral points must be on the vertical
lines x  1, 2, 3, …, 20. The number of integral points on Solution: $ABC is an equilateral triangle implies
x  1 inside the triangle are (1, 1), (1, 2), (1, 3), …, (1, 19)
(total number is 19). Similarly, the number of points on AB  BC  CA
x  2 is 18, on x  3 is 17, etc. Finally, the number of points  r 2 (cos2 A sin 2 A )  r 2 (cos2 B sin 2 B ) 
on x  19 is 1 and on x  20 is 0.
r 2 (cos A cos B )2 r 2 (sin A sin B )2
Therefore, the total number of integral points inside
the triangle is  2r 2 [1 cos(A B )]  r 2
19 s 20 1
19 18 17 ! 1 0   190  cos(A B ) 
2 2
P
 A B 
P(0, 21) 3
(1, 20)
Answer: (D)

18. The centroid of $ABC is (2, 7). If A  (4, 8), B  (a, 0)


and C  (0, b), then

O(0, 0) (A) a  2, b  13 (B) a  2, b  13


x=1 Q(21, 0)
(C) a  13, b  2 (D) a  2, b  13
FIGURE 1.8
Solution: By hypothesis
Answer: (B)
4 a 0 8 0 b
 2 and 7
15. A line segment AB is of length 10 units and A  3 3
(2, 3). If the abscissa of B is 10, then there will be two
This implies a  2, b  13.
values for the coordinate of B whose sum is equal to
Answer: (B)
(A) 3 (B) –3 (C) 6 (D) –6
Solution: Suppose B  (10, y). Then 19. If the point P(x, y) is equidistant from the points
A (6, 1) and B(2, 3), then find a relation between
AB  10  (10 – 2)2 (y 3)2 102 x and y.
(y 3)2  36
Solution: By hypothesis
y 3  ±6
PA  PB
y  3 or –9
PA2  PB2
Therefore, the sum of the values of y  3 – 9  –6.
(x 6)2 (y 1)2  (x 2)2 (y 3)2
Answer: (D)
x2 12x 36 y2 2y 1  x2 4x 4 y2 6y 9
16. Origin is the centroid of a triangle ABC. If A  (4, –3) 8x 8y  24
and B  (–5, 2), then find the coordinates of C. x y  3
14 Chapter 1 Rectangular Coordinates, Basic Formulae, Locus and Change of Axes

20. If the area of the triangle whose vertices are (a, 0),  Absolute value of
(3, 4) and (5, 2) is 10, then m1 m2 m2 m1
a2
(A) a  1 or 22/3 (B) a  1 or 13/3 (m3 m1 )(m3 m2 ) m1 m2 1 1
2
(C) a  1 or 23/3 (D) a  2 or 23/3 1 0 0
Solution: By hypothesis 1 2
 a (m1 m2 )(m2 m3 )(m3 m1 )
6a 26  p 20 2
 a  46/6 or 6/6
23. If a, b, c are the roots of the equation x3 – 6x2 11x –
 a  1 or 23/3
6  0, then find the centroid of the triangle whose ver-
Answer: (C) tices are (ab, 1/ab), (bc, 1/bc), (ca, 1/ca).
Solution: By hypothesis
21. If the points (x, 2 2x), (1 x, 2x) and ( 4 x, 6 2x)
are collinear, find x. a b c6
ab bc ca  11
Solution: By hypothesis,
and abc  6
1 x(2 x 6 2 x) (1 x)(6 2 x 2 2 x)
0 Therefore
2 (4 x)(2 2 x 2 x)
 x(4 x 6) 4(1 x) (4 x)(4 x 2)  0 ab bc ca 11

3 3
 4 x 2 6 x 4 4 x 4 x 2 14 x 8  0
 8x2 4x 4  0 and

 2x2 x 1  0 1¥ 1 1 1 ´ a b c 6 1
¦ µ  
 (2 x 1)( x 1)  0 3 § ab bc ca ¶ 3abc 3s6 3
1 Hence, the centroid of the triangle is
 x  , 1
2 ¥ 1 1 1´

Note: ¦ ab bc ca ab bc ca µ ¥ 11 1 ´
¦ 3
,
3 µ  ¦§ 3 , 3 µ¶
1 ¥1 ´ ¦§ µ¶
x  ( x, 2 2 x)  (1 x, 2 x)  ¦ , 1µ
2 §2 ¶
Caution: First, we have to check whether a, b, c are real
22. Show that the area of the triangle whose vertices or not. In the present case, the roots of the given equa-
are (am1m2, a(m1 m2)), (am2m3, a(m2 m3)) and tion are 1, 2 and 3.
(am3m1, a(m3 m1)) is
24. Find the area of the triangle having midpoints of its
1 2
a (m1 m2 )(m2 m3 )(m3 m1 ) sides at (2, 1), ( 1, 3) and (4, 5).
2
Solution: Area of the triangle is four times the area of
Solution: Let $ be the area of the determinant so that
the triangle formed by the midpoints of its sides. There-
the value of $ is the numerical value (i.e., absolute value)
fore
of the determinant
1
am1 m2 am2 m3 am3 m1 Area of the triangle  4 s 2( 3 5) 1(5 1) 4(1 3)
1 2
a(m1 m2 ) a(m2 m3 ) a(m3 m1 )  2 16 4 16
2
1 1 1  8 sq. units
m1 m2 m2 m3 m3 m1
a2 25. O(0, 0) is one of the vertices of triangle whose cir-
 Absolute value of m1 m2 m2 m3 m3 m1
2 cumcentre is S(3, 4) and centroid G(6, 8). Then, the
1 1 1
 Absolute value of triangle
m1 m2 m2 (m3 m1 ) m1 (m3 m2 ) (A) is right angled
a2
m1 m2 m3 m1 m3 m2 (B) must be equilateral
2
1 0 0 (C) must be right-angled isosceles
(By C2 C1 and C3 C1) (D) is isosceles
Worked-Out Problems 15

Solution: Clearly S(3, 4) is the midpoint of OG (see Now,


Fig. 1.9). Hence OG is the median through as well as the ¥ ax bx2 cx3 ay1 by2 cy3 ´
perpendicular bisector of the side opposite to the vertex Incentre  ¦ 1 , µ
§ a b c a b c ¶
O. Hence the triangle is isosceles.
where
O(0, 0)
ax1 bx2 cx3 5 5 (2) 3 5( 2) 4 5( 4)

a b c 12 5
S(3, 4) 10 5 22 5
  1
12 5
ay1 by2 cy3 5 5 (3) 3 5 ( 5) 4 5 (6)
G(6, 8) 
a b c 12 5
FIGURE 1.9
39 5 15 5
Answer: (D)  2
12 5
26. Find the circumcentre and circumradius of the Hence, the incentre of the triangle is ( 1, 2).
triangle whose vertices are A(1, 1), B(2, 1) and
C(3, 2). 28. Find the ratio in which the point ( 2, 9) divides the
Solution: Let S(x, y) be the circumcentre of $ABC so segment joining the points A(1, 3) and B(2, 7).
that SA  SB  SC. Now Solution: Suppose P  ( 2, 9) and AP:PB  m:1. Then
using section formula we have
SA  SB  (x 1)2 (y 1)2  (x 2)2 (y 1)2
 2x 2y 2  4x 2y 5 ¥ 2m 1 7m 3 ´
( 2, 9)  ¦ ,
§ m 1 m 1 µ¶
 2x 4y  3 (1.8)
This implies
SB  SC  (x 2)2 (y 1)2  (x 3)2 (y 2)2
2m 1 7m 3
 4x 2y 5  6x 4y 13 2  and 9  (1.11)
m 1 m 1
 2x 6y  8 (1.9)
Solving the first equality in Eq. (1.11), we get
SC  SA  (x 3)2 (y 2)2  (x 1)2 (y 1)2
2m 2  2m 1
 6x 4y 13  2x 2y 2
 4 m  3
 4x 2y  11 (1.10)
 m  3 / 4
Solving Eqs. (1.8) and (1.9), we have x  5/2 and y  1/2
which also satisfy Eq. (1.10). Hence Solving the second equality in Eq. (1.11), we get

¥ 5 1´ 9m 9  7m 3
Circumcentre of the triangle  ¦ , µ
§ 2 2¶  16 m  12
2 2
 m  3/4
¥5 ´ ¥1 ´ 9 1 5
Circumradius, SA  ¦§ 1µ¶ ¦§ 1µ¶   We can see from both cases that the ratio is –3:4 or 3: –4.
2 2 4 4 2
The division is external division.
27. Find the incentre of the triangle whose vertices are
A (2, 3), B( 2, 5) and C( 4, 6). 29. Show that the points ( 3, 4), (2, 6) and ( 6, 10) form
a right-angled triangle.
Solution: We have
Solution: Let the vertices be A, B and C, respectively.
2 2 Then
a  BC  ( 2 4) ( 5 6)  4 121  5 5

b  CA  (2 4)2 (3 6)2  36 9  3 5 (AB)2  ( 3 2)2 ( 4 6)2  25 100  125


(BC)2  (2 6) 2 (6 – 10)2  64 16  80
c  AB  (2 2)2 (3 5)2  16 64  4 5
(CA)2  ( 6 3)2 (10 4)2  9 196  205
Therefore
Now, by Pythagoras theorem, we have
a b c  12 5
(AB)2 (BC)  (AC)2
16 Chapter 1 Rectangular Coordinates, Basic Formulae, Locus and Change of Axes

 B  90o 33. Find the equation of the locus of a point P which


moves such that its distance from the origin is twice
Hence, $ABC is a right-angled triangle. its distance from the point A(1, 2).
30. The circumcentre of a triangle lies at the origin and Solution: We have
the centroid is the midpoint of the segment joining
(2, 2) and (2, 2). Find the orthocentre. OP  2PA š(OP)2  4(PA)2
šx2 y2  4[(x 1)2 (y 2)2]
Solution: In a triangle, circumcentre, centroid and
orthocentre are collinear and the centroid divides the šx2 y2  4x2 4y2 – 8x 16y 20
line joining the circumcentre and orthocentre in the ra- š3x2 3y2 – 8x 16y 20  0
tio 1:2. Hence, the equation of the locus is 3x2 3y2 8x – 16y
Suppose H(x, y) is the orthocentre. Then 20  0.
OG:GH  1:2
34. Find the locus of the point P such that the distance
Now, O  (0, 0), G  (2, 0) and H  (x, y) (see Fig. 1.10). of P from the point A(4, 0) is twice the distance of P
Therefore from the x-axis.
OG:GH  1:2  (2, 0)  G  (x/3, y/3) Solution: Let P  (x, y). Distance of P from the x-axis
x  6, y  0 is y . Therefore
So H  (6, 0). AP  2 y š ( AP )2  4 y2
1 2 š ( x 4) 2 y 2  4 y 2
O (0, 0) G (2, 0) H (x, y ) š x 2 3 y2 8 x 16  0
FIGURE 1.10
Hence the equation of the locus is x2 3y2 8x 16  0.

Locus 35. Let A  (2, 3) and B  ( 3, 4). If P is a moving point


such that the area of $PAB is 17/2 sq. units, then find
31. Find the locus of the point which is equidistant from the locus of P.
the points ( 3, 1) and (7, 5).
Solution: Let P  (x, y). Then
Solution: Let A  ( 3, 1) and B  (7, 5). Let P  (x, y).
Then 17
Area of $PAB 
2
AP  PB š(AP)2  (PB)2 1 17
š(x 3)2 (y 1)2  (x 7)2 (y 5)2 š x(3 4) 2(4 y) 3( y 3) 
2 2
š6x 2y 10  14x 10y 74 š x 5 y 17  17
š20x 8y 64  0 š x 5 y 17  p17
š5x 2y – 16  0 Therefore the equation of the locus is
Hence, the equation of the locus is 5x 2y – 16  0. (x 5y) (x 5y – 34)  0
32. Let A(5, 4) and B(7, 6) be two points. Find the
36. Find the equation of the locus of the point which is
locus of the point P such that PA:PB  2:3. at a constant distance of 5 units from the fixed point
Solution: Let P be (x, y). Then ( 2, 3).

PA:PB  2:3 š3PA  2PB Solution: Let A  ( 2, 3) and let P  (x, y). Now
š9(PA)2  4(PB)2 AP  5 š(AP)2  25
š9[(x – 5)2
(y 4)2]
 4[(x – 7)2 š(x 2)2 (y – 3)2  25
(y – 6)2] šx2 y2 4x – 6y – 12  0
š5x2 5y2 34x 120y 29  0
The equation of the locus is x2 y2 4x – 6y – 12  0.
Hence, the equation of the locus is 5x2 5y2 34x 120y Later in Chapter 3, we will see that this equation repre-
29  0. This equation represents circle (which will be dis- sents circle with centre at the point ( 2, 3) and radius 5
cussed in Chapter 3). units.
Worked-Out Problems 17

Change of Axes X Y
and y  X sin( 45B ) Y cos(45B ) 
37. Suppose the origin is shifted to the point ( 1, 2). Find 2
the new coordinates of the point (2, 3). Now,
Solution: Let (X, Y) be the new coordinates. Therefore 2 2
by the formula [Eq. (1.1)], we have ¥ X Y´ ¥Y X´
x 2 y2  a 2  ¦ µ ¦ µ a
2
§ 2 ¶ § 2 ¶
2  x 1, 3  y 2
 4 XY  2a 2
 x  3, y  1
a2 a
Hence, (3, 1) are new coordinates of the point (2, 3).  XY  or XY  c 2 where c 
2 2
38. Find the transformed form of the equation 2x2 4xy Note: When we deal with hyperbola (conic section), we
3y2  0 if the origin is shifted to the point (1, 1). see that x 2 y2  a 2 is the standard equation of rectan-
Solution: Put x  X 1, y  Y 1 in the given equation. gular hyperbola and xy  c 2 is the rectangular hyperbola
We get in the simplest form.

2(X 1)2 4(X 1) (Y 1) 3(Y 1)2  0 41. Find the point to which origin is to be shifted so as
 2X 2 4XY 3Y 2 8X 10Y 9  0 to remove the first degree terms of the equation
2 x 2 4 xy 5 y2 20 x 22 y 14  0.

39. When the axes are rotated through 30o in the anti-
Solution: In the given equation, a  2, h  2, b  5,
clockwise sense without changing the origin, find the g  10, f  11, c  14. Therefore
new coordinates of the point ( 2, 4). ab – h2  10 – 4  14
hf – bg  2( 11) – ( 5)(10)  22 50  28
Solution: Let (x, y) be the old coordinates and (X, Y)
gh – af  10(2) – 2( 11)  42
be the new coordinates. Therefore
Therefore
x  X cos Q Y sin Q (1.12)
¥ hf bg gh af ´
and y  X sin Q Y cos Q (1.13) New origin  ¦ ,
§ ab h2 ab h2 µ¶
Solving Eq. (1.12), we get
¥ 28 42 ´
¦ ,
2  X cos 30B Y sin 30B § 14 14 µ¶
3X Y  (
2, 3)
 2 
2 42. Find the angle through which the axes are to be
 3 X Y  4 (1.14) rotated so as to remove the xy-term from the equation
Solving Eq. (1.13), we get x 2 2 3 xy y2 2a 2  0.

4  X sin 30B Y cos 30B Solution: In the given equation, a  1, h  3 and


b  1 so that a x b. Therefore
X 3Y
4
2 1 ¥ 2h ´
Required angle of rotation  Tan 1 ¦
 X 3Y  8 (1.15) 2 § a b µ¶

Solving Eqs. (1.14) and (1.15), we get X  2 3 and 1 ¥ 2 3´


 Tan 1 ¦ µ
Y  1 2 3 are the new coordinates of ( 2, 4). 2 § 2 ¶
1
40. If the axes are rotated through 45o in the clockwise  Tan 1 ( 3 )
2
sense, then find the transformed form of the equa-
1¥P´
tion x 2 y2  a 2.  ¦ µ
2 § 3¶
Solution: Let (x, y) and (X, Y) be the old and the new P
coordinates, respectively. Therefore 
6
X Y
x  X cos( 45B ) Y sin( 45B ) 
2
18 Chapter 1 Rectangular Coordinates, Basic Formulae, Locus and Change of Axes

SUMMARY
1.1 Distance between two points: If A(x1, y1) and (2) Excentre
B(x2, y2) are two points, then the distance ¥ ax1 bx2 cx3 ay1 by2 cy3 ´
between the two points is given by I1  ¦ , µ
§ a b c a b c ¶
AB  ( x2 x1 )2 ( y2 y1 )2 (3) Excentre
¥ ax bx2 cx3 ay1 by2 cy3 ´
1.2 Section formula: If A(x1, y1) and B(x2, y2) are two I2  ¦ 1 , µ
§ a b c a b c ¶
points and P(x, y) is a point on the line AB dividing
(4) Excentre
the segment AB in the ratio l:m, where l m x 0,
then ¥ ax bx2 cx3 ay1 by2 cy3 ´
I3  ¦ 1 , µ
§ a b c a b c ¶
lx2 mx1 ly my1
x and y  2
l m l m 1.8 Nine-point centre: In any triangle, the midpoints
of the sides, the feet of the altitudes and the
This formula is valid for both internal and external
midpoints of the segments joining the vertices with
divisions.
the orthocentre are concyclic. This circle is called
1.3 Midpoint: If A(x1, y1) and B(x2, y2) are two points, the nine-point circle whose centre is known as the
then the coordinates of the midpoint of AB are nine-point centre which is denoted by N.
given by 1.9 In any triangle, the circumcentre S, the centroid
¥ x1 x2 y1 y2 ´ G, the nine-point centre N and the orthocentre H
¦§ , µ are collinear in the given order (that is, SGNH).
2 2 ¶
Further, G divides SH in the ratio 1:2 and N is
1.4 Centroid coordinates: If G(x, y) is the centroid of the midpoint of SH. Further, the radius of the
a triangle whose vertices are (x1, y1), (x2, y2) and nine-point circle is half of the circumradius of the
(x3, y3), then triangle.
x1 x2 x3 y y2 y3 1.9* Pedal line: Let ABC be a triangle and P be a point
x and y  1 on the circumcircle of the triangle other than the
3 3
vertices. Then the feet of the perpendiculars drawn
1.5 Area of a triangle: The area of the triangle whose from P on to the sides of the triangle are collinear.
vertices are (x1, y1), (x2, y2) and (x3, y3) is the abso- This line is called the Pedal line of the point P or
lute value of the determinant Simson’s line.
x1 y1 1 1.10 If A(x1, y1), B(x2, y2) and C(x3, y3) are three
1 consecutive vertices of a parallelogram ABCD,
x2 y2 1
2 then the fourth vertex D is given by (x1 x3 – x2,
x3 y3 1
y1 y3 – y2).
1.6 Condition for collinearity of three points: Three 1.11 Locus: Let P be a geometrical condition(s) and S
points (x1, y1), (x2, y2) and (x3, y3) are collinear if and be the set of all points in the plane which satisfy P.
only if Then S is called a locus.
x1 y1 1 1.12 Equation of the locus: Let S be a locus and
x2 y2 1 0 f(x, y)  0 be an algebraic equation in x and y.
x3 y3 1 If every point (x, y) belonging to S satisfies the
equation f(x, y)  0, and any point in the plane
1.7 Coordinates of incentres and excentres: Let A(x1, satisfying the equation f(x, y)  0 belongs to S, then
y1), B(x2, y2) and C(x3, y3) be the vertices of a tri- f(x, y)  0 is called the equation of the locus S. The
angle and suppose the lengths BC, CA, and AB are locus is generally given by its equation.
a, b, c, respectively, then 1.13 Shifting of origin without changing the direction of
¥ ax bx2 cx3 ay1 by2 cy3 ´ the axes: Suppose the origin O (0, 0) is shifted to the
(1) Incentre I  ¦ 1 , µ point Oa(h, k ). Let the old and the new coordinates
§ a b c a b c ¶
of a point be (x, y) and (X, Y), respectively. Then x
 X h and y  Y k give the relations between the
old and the new coordinates.
Exercises 19

1.14 Rotation
FHHHHE of the Faxes
HHHHE without changing the origin: are called the first degree terms and c is called the
Let x aOx and y a Oy be the original axes. Rotate constant term.
these axes through an angle P in the anticlockwise
1.17 Theorem: To remove the first degree terms from
direction about the origin
FHHHHHHE O. Let the
E new position
FHHHHHH the second degree general equation, the origin is to
of the axes be X aOX and Y aOY . If (x, y) and
be shifted to the point
(X, Y) be the coordinates of a point with respect to
the old and the new axes, then ¥ hf bg gh af ´
¦§ , µ
x  X cos P – Y sin P and y  X sin P Y cos P ab h2 ab h2 ¶
If the rotation is in the clockwise sense, then in the provided h2 x ab.
above relations, replace P with –P.
1.18 To remove the xy term from the second degree
1.15 Second degree homogeneous and general general equation, the axes are to be rotated through
equations: the angle
(1) Homogeneous equation: If a, h, b are real and 1 ¥ 2h ´
Tan 1 ¦
atleast one of them is not zero, then ax2 2hxy 2 § a b µ¶
by2  0 is called second degree homogeneous
equation in x and y. provided a x b. When a  b, the angle of rotation is
P /4.
(2) General equation: If a, h, b, g, f and c are real and
atleast one of a, h, b is not zero, then the equa- 1.19 When the origin is shifted, the second degree terms of
tion ax2 2hxy by2 2gx 2fy c  0 is called the second degree general equation will not change.
second degree general equation in x and y.
1.20 In the change of axes, only the coordinates of a
1.16 In the second degree general equation, ax2, 2hxy, point will change, but the distance between two
by2 are called the second degree terms, 2gx, 2fy points and the areas will not change.

EXERCISES
1. Show that the points (1, 1), ( 3 , 3 ) and (0, 3 1 ) 10. Show that the four points (0, 1), (2, 1), (0, 3), ( 2, 1),
form the vertices of an isosceles right-angled triangle. taken in this order, form a square.

2. Let ABCD be a rectangle and P any point in the plane 11. Find the area of the triangle whose vertices are (5, 2)
of the rectangle. Then, prove that ( 9, 3) and ( 3, 5).

(PA)2 (PC)2  (PB)2 (PD)2 12. Show that the area of the triangle whose vertices are
HHHHE HHHHE (a cos A , bsin A ), (a cos B, bsin B ) and (a cos G ,
(Hint: Take A as origin, AB and AD as axes.) bsin G ), where a, b are positive, is
3. Prove that the points (3, 6), (2, 1) and (1, 4) are col- ¥ A B´ ¥ B G ´ ¥G A´
2ab sin ¦ µ sin ¦ µ sin ¦
linear. § 2 ¶ § 2 ¶ § 2 µ¶
4. Show that the points (1, 4), (3, 2) and ( 3, 16) are
13. O is the origin, P1  (x1, y1), P2  (x2, y2) and P1OP2  Q .
collinear. Show that OP1 – OP2 – cos Q  x1 x2 y1 y2 .
5. Show that the points (a, a), ( a, a) and ( a 3 , a 3)
14. Find the incentre of the triangle whose vertices are
are the vertices of an equilateral triangle. (7, 9), (3, 7) and ( 3, 3).
6. Let A  (3, 5), B  ( 5, 4), C  (7, 10) and D  (15, 9).
15. Find the centroid of the triangle with vertices (2, 7),
In the given order, show that the points form a paral- (3, 1) and ( 5, 6).
lelogram.
16. Find the incentre of the triangle whose vertices are
7. If A  (2, 3), B  (6, 5), C  ( 2, 1) and D  ( 6, 7),
(3, 2), (7, 2) and (7, 5).
then show that ABCD is a rhombus.
17. Let A  (2, 3) and B  ( 1, 5). If P is a variable point
8. Show that (1, 6), (5, 2), (12, 9), (8, 13), taken in this
such that the segment AB subtends right angle at P,
order, form a rectangle.
then find the equation of the locus of P.
9. If A  (5, 3), B  (11, 5) and C  (12, K) are such that
18. Let A  (1, 1) and B  ( 2, 3). If P is a variable point
ABC  90o, then show that K equals either 4 or 2.
such that the area of $PAB is 2 sq. units, then show
20 Chapter 1 Rectangular Coordinates, Basic Formulae, Locus and Change of Axes

that the equation of the locus of P is (2x 3y 1) 28. (2, 3), (3, 4) and (6, 8) are the vertices of a triangle.
(2x 3y 9)  0. Find its centroid, circumcentre and orthcentre.
19. O(0, 0), A(6, 0) and B(0, 4) are three points. P is a (Hint: To find the orthocentre, refer the note under
variable point such that the area of $POB is twice Quick Look 4.)
that of $POA. Show that the equation of the locus of 29. If the origin is shifted to the point (1, 1), then find
P is x2 9y2  0. the transformed equation of x2 y2 – 2x 2y – 4  0.
20. A(2, 3), B(1, 5) and C( 1, 2) are three points. P is 30. Show that the equation 2x2 y2 – 8x 4y 1  0 will
a variable point such that (PA)2 (PB)2  2(PC)2. be transferred to 2X 2 Y  11 if the origin is shifted
Show that the locus of P is 10x 8y 29  0. to the point (2, 2).
21. Let A  (2, 3) and B  (2, 3). Find the locus of the 31. Find the point to which the origin is to be shifted so
point P such that PA PB  8. as to remove the first degree terms of the following
22. Show that the equation of the locus of the point equations:
equidistant from the points (a b, a b) and (a b, (i) x2 y2 8x – 6y – 25  0
a b) is x y  0.
(ii) 4x2 9y2 – 8x 36y 4  0
23. Find the incentre of the triangle with vertices (1, 3 ), (iii) 14x2 4xy 11y2 – 36x 48y 41  0
(0, 0) and (2, 0).
32. When the axes are rotated through an angle 45o in
(Hint: The triangle is equilateral.)
the anticlockwise sense, then show that the equation
24. A point moves such that the sum of its distances 3x2 10xy 3y2 – 9  0 will be transformed to the
from two fixed points (ae, 0) and ( ae, 0) is always form 8x2 – 2y2 – 9  0.
2a. Prove that the equation of the locus is x2/a2
33. When the axes are rotated through an angle O /4 in
y2/b2  1 where b2  a2(1 – e2) or a2(e2 1) according
the anticlockwise sense, the transformed equation of
as 0 e 1 or e  1.
a curve is 17x2 – 16xy 17y2  225. Find the original
25. Show that the equation of the locus of the point equation of the curve.
which is equidistant from the points (a b, b – a) (Hint: Solve the equation x  X cos Q Y sin Q ,
and (a – b, a b) is bx – ay  0. y  X sin Q Y cos Q for X and Y after replacing P
26. A bar of length a b is moving such that its extremi- with O /4 and substitute the values of X and Y in the
ties lie on the coordinate axes. Show that the locus of given equation.)
the point dividing the bar in the ratio a:b (the direc- 34. Find the angle through which the axes are to be
tion is from y-axis tip towards x-axis tip) is x2/a2 rotated so as to remove the xy term of the equation
y2/b2  1. x2 y2 4xy – 2x 2y – 6  0.
27. (@, A ), ( x , y ) and (p, q) are the circumcentre, 35. Find the angle of rotation of the axes so as to remove
centroid and orthocentre of a triangle. Prove that the xy term from the equation x 2 2 3 xy y2  2a 2
3 x  2A p and 3 y  2B q . and also find the transformed form.
(Hint: See the note given under Quick Look 4.)

ANSWERS
11. 29 sq. units 28. G  (11/3, 5); Circumcentre  ( 27/2, 39/2); Orthocen-
tre  (38, 24)
14. ( 13 8 2 , 2 2 1 )
29. x2 y2  6
15. (0, 4)
31. (i) ( 4, 3); (ii) (1, 2); (iii) (1, 2)
16. (6, 3)
33. 25x2 9y2  225
17. x2 y2 – x 8y 13  0
34. 45o
21. 16x2 7y2 – 64x – 48  0
35. 30o, x2 – y2  a2
23. (1, 1/ 3 )
Straight Line and Pair
of Lines 2
Contents
2.1 Straight Line
2.2 Pair of Lines
Straight Line and Pair of Lines

y
14
Worked-Out Problems
Summary
12
Exercises
Answers
10

The simplest geometric figure is a


6
straight line which is a line segment
joining any two points.
4
Two lines, that is, pair of lines, can be
2 related each other by different ways
such as intersecting lines, perpendic-
ular lines and parallel lines.
−4 −3 −2 −1 0 1 2 3 4 x
−2

Parallel lines Perpendicular lines

Intersecting lines
22 Chapter 2 Straight Line and Pair of Lines

In this chapter, we discuss various types (forms) of equations of a straight line, angle between two lines, conditions for
two lines to be parallel and perpendicular, sides of straight lines, concurrency of lines, etc. ‘Subjective Problems’ sec-
tion provides subjective worked-out problems for the preceding sections. Students are advised to solve each and every
problem to grasp the topics.

2.1 Straight Line


When areas of coordinates are selected, any line parallel to x-axis (including x-axis) is called a horizontal line and any
line perpendicular to x-axis is called a vertical line. First, we discuss the concept of an ‘slope of a non-vertical line’.

DEFINITION 2.1 Slope If a non-vertical line l makes an angle P with the positive direction of the x-axis (that is
measured in counterclock sense), then the value of tanP is called the slope of line l. Generally,
slope of a line is denoted by m.

Note: Slope is defined for non-vertical lines only. We do not talk about the slope of a vertical line.

QUICK LOOK 1

1. Since the angle made by a horizontal line with 3. The slope of a line is positive š the line makes acute
x-axis is 0 or O, its slope is always zero. angle with the positive direction of the x-axis.
2. Two non-vertical parallel lines make the same angles 4. The slope of line is negative š the line makes ob-
with the x-axis implies that their slopes are equal; tuse angle with the positive direction of the x-axis.
however, if the slopes of two lines are equal, then
the lines are parallel lines.

T H E O R E M 2.1 The slope of line passing through two points A(x1, y1) and B(x2, y2) is
y2 y1 ¥ y1 y2 ´

x2 x1 ¦§ x1 x2 µ¶
PROOF Case 1: If the line is horizontal, then
y1 y2
y1  y2  0
x1 x2

and slope of the line is also zero.


Case 2: If the line is not horizontal, then we consider the following: Let the line make an angle P
with the positive direction of the x-axis (see Fig. 2.1). Draw AL and BM perpendicular to x-axis
and AN perpendicular to BM. Clearly, NAB  P. Therefore, it is clear that
BN BN BM MN
tan Q   
AN LM OM OL
BM AL y2 y1
 
OM OL x2 x1

y
B

A q N

q
O L M x

FIGURE 2.1
2.1 Straight Line 23

DEFINITION 2.2 Intercepts of a Line Suppose a line meets x-axis in the point (a, 0) and y-axis in the point
(0, b). Then a is called x-intercept of the line and b is called the y-intercept of the line.

QUICK LOOK 2

1. A vertical line has x-intercept only. 3. For a line through origin, both intercepts are zero.
2. A horizontal line has y-intercept only.

T H E O R E M 2.2 Equation of a line passing through point A(x1, y1) and having slope m (see Fig. 2.2) is
(P O I N T –S L O P E
y y1  m( x x1 )
FORM)
PROOF Suppose P(x, y) is any point on the given line. Then, by Theorem 2.1, the slope of the line is
y y1
m
x x1
Therefore,
y y1  m( x x1 )
Conversely, let Q(xa, ya) be a point such that
y a y1  m( x x1 )
y a y1
 m (slope of the line)
x x1
FHHHE
This implies that the line AQ coincides with the given line which, in turn, implies that Q lies on
the given line. Therefore, equation of the line is y y1  m( x x1 ).
y

A (x1, y1)

q
x

FIGURE 2.2

Example 2.1

Write the equation of a line passing through ( 2 , 2 ) y 2  1( x 2 )


and having slope 1.
 x y0
Solution: We have m  1 and x1  2  y1. Therefore,
the equation of the line is

Example 2.2

Write the equation of the line which is inclined at an Solution: We have


angle O/3 with the positive direction of the x-axis and ¥P´
passing through the point (1, −2). m  tan ¦ µ  3
§ 3¶
24 Chapter 2 Straight Line and Pair of Lines

Now given x1  1 and y1  2. Therefore, equation of the y 2  3 ( x 1)


line is
 3 x y (2 3 )  0

T H E O R E M 2.3 Equation of the line having slope m and y-intercept c is y  mx c.


(S L O P E –
INTERCEPT FORM)
PROOF Since c is the y-intercept of the line, the line passes through the point (0, c) and has slope m.
Therefore, by Theorem 2.2, the equation of the line is
y c  m( x 0)
 y  mx c

QUICK LOOK 3

Equation of any line passing through origin (excluding y-axis) is of the form y  mx (i.e., c  0).

Example 2.3

Write the equation of the line having slope 1/2 and 1


y-intercept 1. y x 1
2
Solution: We have m  1/2 and c  1 so that the equa-  x 2y 2  0
tion of the line is

Example 2.4

Write the equation of the line having slope 2 and at (3/2, 0) and (0, −3). Hence, the vertices of the triangle
y-intercept −3 and hence find the area of the triangle are (0, 0), (3/2, 0) and (0, −3). Therefore, area of the tri-
formed by this line and the two coordinate axes. angle is

Solution: We have m  2 and c  −3. Therefore, the 1 3 9


0 0 3  3 0 0 0 0  sq. unit
equation of the line is y  2 x 3 . The line meets the axes 2 2 4

T H E O R E M 2.4 If a and b are non-zero intercepts of a line on x- and y-axis, respectively, then the equation of these
(I N T E R C E P T two intercepts is
FORM)
x y
1
a b
PROOF This line passes through the points (a, 0) and (0, b) (see Fig. 2.3) so that its slope is
b 0 b

0 a a
Therefore, by Theorem 2.3, the equation of the line is
b
y ( x a)
a
2.1 Straight Line 25

y x
  1
b a
x y
 1
a b

B (0, b)

O A (a, 0) x

FIGURE 2.3

QUICK LOOK 4

Area of the triangle formed by the line and the coordinate axes is equal to
x y 1 1 1
1 (OA – OB)  a b  ab sq. unit
a b 2 2 2

Example 2.5

Find the equation of the line whose intercepts are numer- x y


or 1
ically equal. a a
That is,
Solution: Suppose the intercepts are a and −a or −a
and a. Then the equation is x ya

x y or x y  a
1
a a

Example 2.6

Find the equation of the line whose sum of the intercepts Now,
on the axes is 3 and their product is 2.
a b 3
Solution: The equation of a line is and ab  2

x y Solving these equations, we get a  2 and b  1 or a  1


1 and b  2. Therefore, equation of the lines is
a b
x y
1
2 1
26 Chapter 2 Straight Line and Pair of Lines

x y Therefore, the equation of the line is


or 1
1 2 x 2y 2  0
or 2x y 2  0

T H E O R E M 2.5 Equation of the line passing through two points A(x1, y1) and B(x2, y2) is
(T W O -P O I N T
( x x1 )( y1 y2 ) ( y y1 )( x1 x2 )  0
FORM)
FHHHE
PROOF Case 1: Suppose the line AB is vertical. Hence, x1  x2 and y1 x y2 . If P(x, y) is any point on
FHHHE
the line AB, then x1  x2  x so that
( x x1 )( y1 y2 )  ( y y1 )( x1 x2 )
FHHHE
Case 2: AB is not a vertical line. Therefore, by Theorems 2.1 and 2.2, its equation is
( y1 y2 )
y y1  ( x x1 )
( x1 x2 )

 ( x x1 )( y1 y2 )  ( y y1 )( x1 x2 )

Example 2.7

Write the equation of the line passing through the points [ x (a b)][a b (a b)]  [ y (a b)][a b (a b)]
(a b, a b) and (a b, a b).
2b[ x (a b)]  2b[ y (a b)]
Solution: We have x1  a b, y1  a b, x2  a b and x a b  y a b
y2  a b. Therefore, equation of the line is Therefore
x y  2a

Example 2.8

Find the equation of the line joining the points ¥ A B´ ¥ A B´


(a cos A , a sin A ) and (a cos B, a sin B ) where a  0.  ( x a cos A )2 sin ¦ cos ¦  ( y a sin A )
§ 2 µ¶ § 2 µ¶
Solution: Equation of the line is ¥ A B A B´
¦§ 2 sin sin µ
2 2 ¶
( x a cos A )(a sin A a sin B )  ( y a sin A )
(a cos A a cos B )
A B ¥ A B´ ¨ A B
 x cos y sin ¦  a ©cos A cos
2 § 2 µ¶ ª 2
 ( x a cos A )(sin A sin B )  ( y a sin A )(cos A cos B )
A B·
sin A sin
2 ¸¹
A B A B ¥A B´
 x cos y sin  a cos ¦
2 2 § 2 µ¶

Note: One can see in Example 2.8 that (a cos A , a sin A ) and (a cos B, a sin B ) are the points on a circle with centre at
origin and radius a. In the above chord equation, if B  A , then x cos A y sin A  a is the equation of the tangent to
the circle with centre (0, 0) and radius a.
2.1 Straight Line 27

Example 2.9

Show that the equation of the line joining two points 2at2 2at1 2
(at12 , 2at1 ) and (at22 , 2at2 ) is 
at22 at12 t1 t2
2
y 2at1  ( x at12 ) Hence, the equation of the line is
t1 t2
2
Solution: The slope of the line joining the two points is y 2at1  ( x at12 )
t1 t2

Note: In Chapter 4, we will discuss and show that for all real values of t, the locus of the point (at 2, 2at ) is the parabola
y2  4ax and the equation of the tangent at (at 2, 2at ) is ty  x at 2 which can be obtained by substituting t1  t2  t
in the equation of the line joining the points (at12, 2at ) and (at22, 2at ) which is also discussed in Example 4.3.

T H E O R E M 2.6 A E l is such that its perpendicular distance ON (O is the origin) from the origin is p and the ray
line
FHHH
(E Q U A T I O N O F ON makes angle @ with the positive direction of the x-axis (measured in counterclock sense).
STRAIGHT LINE Then the equation of the line l is
IN NORMAL
x cos A y sin A  p
FORM)
PROOF Case 1: Suppose that the given line l is a vertical line meeting positive x-axis at point N [see Fig.
2.4(a)] so that ON  p and @  0. Therefore, if p(x, y) is any point on l, then x  p and the equation
of the line l is x  p which is equivalent to
x cos A y sin A  p (& A  0)
Similarly, if l meets the negative x-axis at point N [see Fig. 2.4(b)], then the equation of the line l
is x  −p so that
x cos A y sin A  p (& A  P )

y l l y

O p N (p, 0) x N (−p, 0) O x

(a) (b)

FIGURE 2.4

Case 2: Suppose the line l is horizontal and meets positive y-axis at point N so that N  (0, p) and
@  O/2 [see Fig. 2.5(a)]. Since its equation is y  p, it is also given by
x cos A y sin A  p (& A  P /2)
Similarly, if N lies on the negative y-axis, then N  (0, −p) and @  −3O/2 [see Fig. 2.5(b)] so that its
equation is y  −p. The equation of the line can be written as
x cos A y sin A  p
28 Chapter 2 Straight Line and Pair of Lines

y y

a = 3p
l N (0, p) 2

p x
p
p =a
2
O x l N (0, −p)

(a) (b)

FIGURE 2.5

Case 3: Suppose l is an inclined line meeting positive coordinate axes at points A and B, respec-
tively [see Fig. 2.6(a)]. So the coordinates are A  (p sec@, 0) and B  (0, p cosec@). By Theorem 2.4,
the equation of the line l is
x y
1
p sec A p cosec A
Hence
x cos A y sin A  p
Similarly, for other cases, namely,
P 3P 3P
 A  P or P  A  or  A  3P
2 2 2
we can show that its equation is
x cos@ y sin@  p
These three cases are shown in Figs. 2.6(b), 2.6(c) and 2.6(d), respectively.

l y

N
p
a p a
O A x
p <a <p
l 2

(a) (b)
y y

l l

a
O x O p x
a
N N

p < a < 3p 3p < a < 2p


2 2

(c) (d)

FIGURE 2.6
2.1 Straight Line 29

Example 2.10

Find the equation of the line whose distance from origin Solution: We have p  2 at @  O /4. Hence, the equation
is 2 and the normal ray through origin makes an angle O /4 of the line is
with the positive direction of the x-axis measured in coun-
terclock sense. P P
x cos y sin  2 or x y  2 2
4 4

Example 2.11

Find the equation of the line whose distance from the 2P 2P ¥ 3´


¥ 1´
origin is 4 and the normal ray through origin makes an x cos y sin  4 or x¦ µ y¦ µ 4
3 3 § 2¶ § 2 ¶
angle (2O )/3 with the positive direction of the x-axis mea-
sured in counterclock sense. That is,
Solution: We have p  4 and @  (2O)/3. Hence the equa- x 3 y  8
tion of the line is

T H E O R E M 2.7 The equation of the straight line passing through (x1, y1) and making an angle P with the positive
(S Y M M E T R I C direction of the x-axis measured in counterclock sense is
FORM OF LINE)
( x x1 ) : cos Q  ( y y1 ) : sin Q

PROOF Suppose the line is L which passes through (x1, y1).


Case 1: The line L is vertical so that P  O /2. Since the equation of L is x  x1 which is written as
( x x1 ) : cos Q  ( y y1 ) : sin Q
because cosP  0 and sinP  1.
Case 2: Suppose L is non-vertical so that P xO /2. Hence by Theorem 2.2, its equation is
( y y1 )  tan Q ( x x1 )
 ( x x1 )sin Q  ( y y1 )cos Q
Thus
( x x1 ) : cos Q  ( y y1 ) : sin Q

If ( x x1 )  G cos Q and ( y y1 )  G sin Q , we have

( x x1 )2 ( y y1 )2  G

which shows that G represents the distance of the point (x, y) on the line from the given point (x1, y1). Therefore, if
(x, y) is any point on the line and F is any real parameter, then the locus of the point (x1 F cosP, y1 F sinP ) is the
straight line
( x x1 ) : cos Q  ( y y1 ) : sin Q
Also, F takes positive values for points on the line on one side of (x1, y1) and takes negative values for the points on the
other side of (x1, y1). For a given positive value of F , there will be two points on the line which are equidistant from (x1,
y1). Further, the equations
x  x1 F cosP and y  y1 F sinP
(P is a fixed and F is a parameter) are called the parametric equations of the line passing through (x1, y1) and making
angle P with the positive direction of the x-axis measured in counterclock sense.
30 Chapter 2 Straight Line and Pair of Lines

Example 2.12

Write the parametric equation of the line passing through ¥ 3´


the point (3, 2) and making an angle 120° with the posi- and y  y1 G sin Q  2 G ¦ µ
tive direction of the x-axis measured in counterclock § 2 ¶
sense and also find the coordinates of the points on the Solving these equations, we get
line which are at unit distance from the point (3, 2). x  3 −F /2

Solution: We have (x1, y1)  (3, 2), cosP  cos 120°  −1/2 and y  2 F( 3 /2)
and sinP  sin 120°  3 /2. Therefore, the parametric Substituting F  1 and F  −1 in the above coordinates,
equations of the line are the points on the line which are at a distance of 1 unit
from the point (3, 2) are obtained, respectively, as
¥ 1´
x  x1 G cos Q  3 G ¦ µ ¥ 5 4 3´ ¥ 7 4 3´
§ 2¶
¦ 2 , 2 µ and ¦ 2 , 2 µ
§ ¶ § ¶

DEFINITION 2.3 If a, b and c are real and either a or b is non-zero, then the expression ax by c is called first-
degree expression in x and y and the equation ax by c  0 is called first-degree equation in x
and y.

Examples

1. 2 x y 3  0 3. 2y 1  0 5. x cos A y sin A  p
2. x 2  0 4. 3x 4 y 1  0

T H E O R E M 2.8 Every first-degree equation in x and y represents a straight line and conversely, the equation of a
straight line is a first-degree equation in x and y.
PROOF Suppose ax by c  0 is a first-degree equation so that either a or b is non-zero. Now, either b  0
or b x 0. If b  0, then the equation reduces to ax c  0 where a x 0 which represents the vertical line
x  −c/a. If b x 0, then the equation can be written as y  (−a/b)x (−c/b) which represents straight
line having slope (−a/b) and y-intercept −c/b. In any case, ax by c  0 represents a straight line.
Conversely, let L be a straight line in the coordinate plane. Therefore, L is either a vertical line
or a non-vertical line. Suppose L is a vertical line meeting x-axis at the point (a, 0), the equation
of L is x−a  0 which is a first-degree equation in x and y. If L is non-vertical, then it meets y-axis
at some point, say (0, c), and has slope, say, m, then the equation of L is y  mx c (see Theorem
2.3) which is a first-degree equation as coefficient of y is −1 (x0).

In view of Theorem 2.8, the following definition is provided.

DEFINITION 2.4 General Equation of a Straight Line First-degree equation in x and y is called the general
equation of a straight line. Suppose ax by c  0 is a first-degree equation, then we have the
following conditions:
1. If c  0, then the line passes through (0, 0).
2. If b x 0, then the first-degree equation represents a line having slope (−a/b) and y-intercept (−c/b).
3. If b  0, ax by c  0 represents a vertical line.
4. If abc x 0, then the first-degree equation represents a line with x-intercept (−c/a) and y-intercept (−c/b).

T H E O R E M 2.9 Two first-degree equations a1x b1y c1  0 and a2x b2y c2  0 represent parallel lines if and
only if a1b2  a2b1 (i.e., a1:b1  a2:b2).
2.1 Straight Line 31

PROOF When two lines are parallel, both are either vertical or non-vertical. If both are vertical, then b1 
b2  0 [by point (3) below Definition 2.4] so that a1b2  a2b1  0. If both are non-vertical, their
slopes are equal [by point (2) below Definition 2.4]. Thus,
a1 a
 2
b1 b2
Hence, a1b2  a2b1or a1:b1  a2:b2
Conversely, when a1b2 a2b1 and if b1  0, then a1x 0 so that b2  0. Therefore, b1  0 šb2  0 and
hence if one of the lines is vertical, then the other is also vertical so that the given lines are parallel.
If both b1 and b2 are non-zero, then
a1 a2
a1b2  a2 b1  
b1 b2
a a
 1  2
b1 b2
Since the slopes are equal, the lines are parallel.

QUICK LOOK 5

1. Equation of any line parallel to the line ax by 2. In particular, equation of the line passing through
c  0 is of the form ax by ca 0. the point (x1, y1) and parallel to the line ax by
c  0 is a(x−x1) b(y−y1)  0.

T H E O R E M 2.10 Two first-degree equations a1x b1y c1  0 and a2x b2y c2  0 represent the same straight line
if and only if a1:b1:c1  a2:b2:c2.
PROOF Suppose the two equations represent the same straight line. Since every line is parallel to itself, by
Theorem 2.9, a1b2  a2b1 and hence a1:b1  a2:b2. If b1  0, then b2 0 so that a1 and a2 are non-zero.
This implies that
c1 c
 2
a1 a2
Therefore
a1 : b1 : c1  a2 : b2 : c2
If b1x 0, then b2x 0 (since a1b2  a2b1). Let
b1
L
b2

Therefore,
a2 b2 1
a1b2  a2 b1   
a1 b1 L
 a1  L a2 (2.1)
Also (0, −c1/b1) is a point on a1x b1y c1  0 which implies that (0, c1 /b1 ) also lies on
a2x b2y c2  0. Therefore
¥ c ´
b2 ¦ 1 µ c2  0
§ b1 ¶
c2 b2 1
   (2.2)
c1 b1 L
32 Chapter 2 Straight Line and Pair of Lines

Therefore, from Eqs. (2.1) and (2.2)


a1:b1:c1  a2:b2:c2
Conversely, suppose a1:b1:c1 a2:b2:c2. Therefore, for some real K x 0, we have a1  Ka2, b1  Kb2,
c1  Kc2. Hence
a1 x b1 y c1  0 š L (a2 x b2 y c2 )  0
š a2 x b2 y c2  0

Therefore, both equations represent the same straight line.

T H E O R E M 2.11 Let ax by c  0 be a straight line. Then

¥ a ´ ¥ b ´ c
1. ¦
2 2
µ x ¦ 2 2
µy is the normal form of the given line if c  0.
§ a b ¶ § a b ¶ a b2
2

¥ a ´ ¥ b ´ c
2. ¦ µ x ¦ µy is the normal form of the given line if c  0.
§ a 2 b2 ¶ § a 2 b2 ¶ a b2
2

PROOF Suppose x cos A y sin A  p is the normal form of ax by c  0. Therefore, by Theorem 2.10,
there exists a real K x 0 such that cos@  Ka, sin@  Kb and −p  Kc. Now,
cos2@ sin2@  1
1
implies K±
a b2
2

Also −Kc  p  0 (since p is the distance of the line from origin) implies that
« 1
® 2 if c  0
® a b2
L¬
® 1 if c  0
®­ a 2 b2

Therefore, if c  0, then the normal form of the line is


¥ a ´ ¥ b ´ c
x¦ µ y¦ µ
§ a 2 b2 ¶ § a 2 b2 ¶ a 2 b2
or if c  0, the normal form of the line is
¥ a ´ ¥ b ´ c
x¦ µ y¦ µ
§ a 2 b2 ¶ § a 2 b2 ¶ a b2
2

QUICK LOOK 6

1. To reduce ax by c  0 to the normal form, take c


2. p  p is the distance of the line ax by
constant c to the right-hand side (RHS), divide both a b2
2

sides with a 2 b2 and then make RHS positive. c  0 from the origin. That is,
c
p
a 2 b2
2.1 Straight Line 33

Example 2.13

Write 2x 3y 5  0 in the normal form. What is the dis- To make the RHS positive, we multiply both sides with
tance of the line from origin? (−1). Thus, the normal form is

Solution: In 2x 3y 5  0, the constant 5 should be ¥ 2 ´ ¥ 3 ´ 5


¦§ µ¶ x ¦§ µ¶ y 
taken to the RHS of the equation, that is 13 13 13
where
2x 3y  −5
2 3
2 2 2 2 cos A  and sin A 
Dividing both sides with a b  2 3  13 , we 13 13
get
Now, the distance of the line from the orgin is
2 3 5
x y c 5 5
13 13 13 p  
2 2
a b 13 13

Example 2.14

Find the normal form of the line 3x 4y − 10  0 and its where


distance from the origin.
3 4
cos A  and sin A 
Solution: The equation 3x 4y − 10  0 can be 5 5
written as 3x 4y  10. Dividing both sides with
Now the distance of the line from the origin is
a 2 b2  32 4 2  5, we get
10 10
3 4 p  2
x y 2 5 5
5 5

T HEOREM 2.12 If 0 P O/2 is the angle between the lines a1x b1y c1  0 and a2x b2y c2  0, then
(A N G L E
BETWEEN TWO
a1a2 b1b2
cos Q 
LINES) (a12 b12 )(a22 b22 )
or equivalently

¥ a1a2 b1b2 ´
Q  Cos 1 ¦ µ
¦§ (a 2 b2 )(a 2 b2 ) µ¶
1 1 2 2

PROOF Let the lines represented by the given equations (in the order written) be l1and l2, respectively. If
l1 and l2 are parallel lines, then we consider that Pis equal to 0 or O and also
a1a2 b1b2
1
(a12 b12 )(a22 b22 )

because a1:b1  a2:b2 (Theorem 2.9). Hence

¥ a1a2 b1b2 ´
Cos 1 ¦ µ  Cos 1 (1)  0
¦§ (a 2 b2 )(a 2 b2 ) µ¶
1 1 2 2

Therefore, without loss of generality, we assume that l1 and l2 are intersecting so that a1:b1 xa2:b2.
FHHHE FHHHHE
Let OL1 and OL2 be the lines through origin O and parallel to the lines l1 and l2, respectively
34 Chapter 2 Straight Line and Pair of Lines

FHHHE FHHHHE
(see Fig. 2.7). Therefore, from Quick Look 5, part (2), the equations of OL1 and OL2 are, respec-
FHHHE FHHHHE
tively, a1x b1y  0 and a2x b2y  0. Let P1 and P2 be the angles made by OL1 and OL2 with the
positive direction of the x-axis so that Q1 Q 2 is the measure of the angle between the lines. We
FHHHE FHHHHE
know that P(b1, −a1) lies on OL1 and Q(b2, −a2) lies on OL2 . Draw PL and QM perpendicular
to x-axis. Now,
cos Q  cos(Q1 Q 2 )  cos Q1 cos Q 2 sin Q1 sin Q 2
¥ b1 ´¥ b2 ´ ¥ a ´ ¥ a ´
1 2
¦ µ¦ µ ¦ µ¦ µ
¦§ a 2 b2 µ¶ ¦§ a 2 b2 µ¶ ¦§ a 2 b2 µ¶ ¦§ a 2 b2 µ¶
1 1 2 2 1 1 1 1

a1a2 b1b2

(a12 b12 )(a22 b22 )

P  Q1 Q 2 and 0 PO/2.
a1a2 b1b2
cos Q 
(a12 b12 )(a22 b22 )

y
L1

P(b1, −a1) L2

Q(b2, −a2)
q1
q2
O L M x

FIGURE 2.7

Note: If both lines are non-vertical and P is the angle between them, then

m1 m2
tan Q 
1 m1 m2

where m1 and m2 are their slopes.

QUICK LOOK 7

1. (a) The lines a1x b1y c1  0 and a2x b2y c2  0 2. The equation of any line perpendicular to the line
are at right angles š a1a2 b1b2  0 (since ax by c  0 is of the form bx−ay ca  0.
P  90°). 3. In particular, the equation of the line passing
(b) Product of the slopes is −1 provided the lines through the point (x1, y1) and perpendicular to the
are at right angles. line ax by c  0 is b (x−x1) − a (y−y1)  0.

DEFINITION 2.5 Image or a Reflection of Point Let l be a straight line and P be a point on the plane of the
line, but not on the line. A point Q in the same plane is called the image or reflection of P in
the line l if l is the perpendicular bisector of the segment PQ.
2.1 Straight Line 35

QUICK LOOK 8

To find the image of P on the line l, draw PM perpen- P


dicular to the line l and produce it to Q such that PM 
90°
MQ (see Fig. 2.8). l
M

FIGURE 2.8

T H E O R E M 2.13 Let l be a line whose equation is ax by c  0 and P(x1, y1) be a point on the plane of the line,
but not on l. Suppose M(h, k) is the foot of the perpendicular drawn from point P onto l and
Q(x1a, y1a) is the image of point P in the line. Then

h x1 k y1 (ax1 by1 c)
1.  
a b a 2 b2
x1a x1 y1a y1 (ax1 by1 c)
2.  2
a b a 2 b2
PROOF 1. See Fig. 2.9. The line PM is perpendicular to the line ax by c  0, so by Quick look 7, part
(2), we have
¥ k y1 ´ ¥ a ´
¦§ h x µ¶ ¦§ b µ¶  1
1
h x1 k y1
  L (say)
a b
 h  x1 L a, k  y1 L b (2.3)
Since M(h, k) lies on the line, we have ah bk c  0, which implies that
a( x1 L a) b( y1 L b) c  0
(ax1 by1 c)
L (2.4)
a 2 b2
Therefore, from Eqs. (2.3) and (2.4), we get
h x1 k y1 (ax1 by1 c)
 
a b a 2 b2
2. Since M is the midpoint of PQ, we can see that
x a1 x1 ya1 y1 (ax1 by1 c)
  2
a b a 2 b2
P (x1, y1)

ax by c 0 M (h, k) l

Q (x1, y1 )

FIGURE 2.9
36 Chapter 2 Straight Line and Pair of Lines

Note: Theorem 2.13 gives the formulae for the foot of the perpendicular drawn from a point onto the given line and
the image of a point on the given line.

Example 2.15

Find the foot of the perpendicular drawn from the point ¥ 1 2´


(1, 2) onto the line 3x 4y − 1  0 and also find the image ¦§ , µ¶
5 5
of (1, 2) on the given line.
2. Suppose (@, A ) is the image of (1, 2). From Theorem
Solution: We have (x1, y1)  (1, 2) and the line is 3x 2.13, we have
4y − 1  0.
A 1 B 2 [3(1) 4(2) 1] 20 4
1. Suppose (h, k) is the foot of the perpendicular. There-   2  
3 4 32 4 2 25 5
fore, from Theorem 2.13, we have
Hence
h 1 k 2 [3(1) 4(2) 1] 10 2
  2 2
  12 7
3 4 3 4 25 5 A  1 
5 5
Hence
16 6
B  2 
6 1 5 5
h  1 
5 5 Thus, the image is
8 2
k  2  ¥ 7 6´
5 5 ¦§ , µ¶
5 5
Therefore, foot of the perpendicular from (1, 2) is

T H E O R E M 2.14 The distance of the line ax by c  0 from the point P(x1, y1) is
ax1 by1 c
a 2 b2
PROOF See Fig. 2.10. Shift the origin to the point P(x1, y1) and let the new coordinates of any point be
denoted by (X, Y). Hence, by Section 1.4.1, we have x  X x1 and y  Y y1 so that P becomes
(0, 0) and the equation of the given line is a(X x1) b(Y y1) c  0. That is,
aX bY ax1 by1 c  0 (2.5)
Hence, by Quick Look 6, part (2), the distance of the line [Eq. (2.5)] from P(0, 0) is equal to
ax1 by1 c
a 2 b2
Y

y P X

ax
+b 90°
y+
c=
0

O x

FIGURE 2.10
2.1 Straight Line 37

T H E O R E M 2.15 The distance between the parallel lines ax by c  0 and ax by ca  0 is


c ca
a 2 b2
PROOF See Fig. 2.11. Let P(x1, y1) be a point on ax by ca  0 so that
ax1 by1 c a  0 (2.6)
Now, the distance between the two parallel lines is equal to the distance of the line ax by c  0
from the point P(x1, y1), which is given by
ax1 by1 c
(by Theorem 2.14)
a 2 b2
ca c
 [from Eq. (2.6)]
a 2 b2

ax  by  c   0 P(x1, y1)

ax  by  c  0
M

FIGURE 2.11

T H E O R E M 2.16 If a1x b1y c1  0 and a2x b2y c2  0 are two intersecting lines, then their point of intersection
is
¥ b1c2 b2 c1 c1a2 a1c2 ´
¦§ a b a b , a b a b µ¶
1 2 2 1 1 2 2 1

PROOF Since the lines are not parallel, we have a1b2 xa2b1. Let (@, A ) be the point of intersection of the
lines. Therefore,

a1A b1B  c1 (2.7)

and a2A b2 B  c2 (2.8)


Solving Eqs. (2.7) and (2.8) for @ and A, we get
b1c2 b2 c1
A
a1b2 a2 b1
c a a c
B 1 2 1 2
a1b2 a2 b1

Notation: The line ax by c  0 is denoted by L so that L  0 represents a straight line if either a or b is not zero.
L y ax by c
L11  ax1 by1 c
L22  ax2 by2 c

T H E O R E M 2.17 Let L yax by c be a line and A(x1, y1) and B(x2, y2) be the two points on the same plane, but
not on the line L  0, then L  0 divides the line segment AB in the ratio −L11:L22.
38 Chapter 2 Straight Line and Pair of Lines

FHHHE
PROOF Suppose L  0 meets the line AB in point P(x, y) and AP:PB  K:1. Therefore
x1 L x2
x
L 1
y1 L y2
y
L 1
Since P(x, y) lies on the line L  0, we have
( x1 L x2 ) ( y L y2 )
a b 1 c  0
L 1 L 1
That is, ax1 by1 c K(ax2 by2 c)  0. That is, L11 KL22  0.Therefore,
L11
L
L22

Hence, AP : PB  L11 :L22 .

QUICK LOOK 9

1. L  0 divides AB internally 2. Points A and B lie on the same side of L  0

š−L11:L22 is positive šthe division is external


L11
L11 š 0
š 0 L22
L22
šL11 and L22 are opposite signs L11
š 0
L22
Therefore, points A and B lie on the opposite sides
of line L  0 šL11 and L22 are of opposite signs. Therefore, L11 and L22 have the same sign.

DEFINITION 2.6 Origin and Non-Origin Sides Let L be a straight line in the coordinate plane which is not
passing through the origin. The side of the region in which the origin lies is called the origin
side of the line and the other is called the non-origin side of the line L (see Fig. 2.12).
y

L Non-origin
side

Origin side

O x

FIGURE 2.12

T H E O R E M 2.18 Let L yax by c  0 be a line which is not passing through the origin. Then a point A(x1, y1)
(not on the line L  0) lies
(a) on the non-origin side of L  0 šc and L11 are of opposite signs
(b) A(x1, y1) lies on the origin side of L  0 šc and L11 have the same sign
PROOF In Quick Look 9, take B  (x2, y2)  (0, 0). Hence, points A and B (which is equal to the origin)
lie on the opposite sides of L  0 so that L11 and L22  c have the opposite signs and points A and
(0, 0) are on the same side. Hence, L11 and L22  c have the same sign.
2.1 Straight Line 39

QUICK LOOK 10

Let ax by c  0 be a line and c x 0. 2. If c  0, then for all points on the origin side L11 0
and L11  0 for all points on the non-origin side.
1. If c  0, then for all points on the origin side L11  0
and for all points on the non-origin side L11  0.

Example 2.16

Check whether the point (2, 1) lies on the origin side or L11  2(2) − 3(1) 1  2
non-origin side of the line 2x − 3y 1  0.
and c1
Solution: We have Ly 2x − 3y 1, A(x1, y1)  (2, 1) and That is, L11 and c have the same origin. Hence, (2,1) lies
c  1. That is, on the origin side of L  0.

Example 2.17

Find the ratio in which the line L yx 2y − 3  0 divides and L22  3 2(2) − 3  4
the line joining the points A(1, 2) and B(3, 2).
Therefore, L  0 divides AB in the ratio
Solution: A(1, 2)  (x1, y1) and B(3, 2)  (x2, y2) so that −L11:L22  −2:4  −1:2
L11  1 2(2) − 3  2 That is, L  0 divides AB externally in the ratio 1:2.

T H E O R E M 2.19 Suppose U1 ya1x b1y c1  0 and U2 ya2x b2y c2  0 are two parallel lines. Then for any real
values of K1 and K2 such that L1 L 2 x 0 , the equation K1U1 K2U2  0 represents the line paral-
lel to each of the lines U1  0 and U2  0.
PROOF From Quick Look 5, U2  0 can be written as U2 ya1x b1y c1a  0. Therefore,
K1U1 K2U2 y (K1 K2) [a1x b1y] K1c1 K2c1a  0
which represents the line parallel to U1 ya1x b1y c1  0.

T H E O R E M 2.20 If U1 ya1x b1y c1  0 and U2 ya2x b2y c2  0 are intersecting lines, then the equation K1U1
K2U2  0 ( L1 L 2 x 0) represents the lines passing through the intersection of U1  0 and U2  0.
Conversely, the equation of any line passing through the intersection of U1  0 and U2  0 is of the
form K1U1 K2U2  0 for some K1 and K2 such that L1 L 2 x 0.
PROOF Suppose U1 ya1x b1y c1  0 and U2 ya2x b2y c2  0 are intersecting lines so that a1b2 − a2b1x 0.
Let p(@, A ) be the point of intersection of U1  0 and U2  0. Therefore,
a1A b1B c1  0 (2.9)
and a2A b2 B c2  0 (2.10)
Now, L1 L 2 x 0 implies that atleast one of K1 and K2 is not zero. Consider that
K1U1 K2U2  (K1a1 K2a2)x (K1b1 K2b2)y K1c1 K2c2  0
If both K1a1 K2a and K1b1 K2b2 are zero, then
a1 L b
 1  1
a2 L 2 b2
40 Chapter 2 Straight Line and Pair of Lines

which is a contradiction of the hypothesis a1b2 xa2b1. Therefore, K1U1 K2U2  0 is a first-degree
equation and hence it represents a straight line. Also, from Eqs. (2.9) and (2.10), it follows that
K1U1 K2U2  0 passes through (@, A ). Conversely, suppose U yax by c  0 is a line passing
through (@, A ), then
aA bB c  0 (2.11)
Since either a or b is not zero [i.e., (a, b) x (0, 0)] and a1b2−a2b1x 0, it follows that the equations
a1x a2y  a and b1x b2y  b
have unique solution, say, x  K1 and y  K2 and L1 L 2 x 0. Therefore,
K1a1 K2a2  a and K1b1 K2b2  0
Now, from Eq. (2.11), we get
c  aA B b  (L1a1 L 2 a2 )A (L1b1 L 2 b2 )B
 L1 (a1A b1B ) L 2 (a2A b2 B )
 L1 ( c1 ) L 2 ( c2 ) [from Eqs. (2.9) and (2.10)
 L1c1 L 2 c2
Therefore
ax by c  (L1a1 L 2 a2 ) x (L1b1 L 2 b2 ) y L1c1 L 2 c2
 L1 (a1 x b1 y c1 ) L 2 (a2 x b2 y c2 )
 L1U 1 L 2U 2
Thus, the equation of any line passing through the intersection of the lines U1  0 and U2  0 is of
the form K1U1 K2U2  0 where L1 L 2 x 0.

Note: If U1  0 and U2  0 are two intersecting lines, then K1U1 K2U2  0, where eitherK1 or K2 is non-zero, represents
all the lines passing through their point of intersection including U1  0 and U2  0. If K1 x0, then the equation can be
written as U1 (K2/K1) U2  0 which is of the form U1 KU2  0, where K is K2/K1. The equation U1 KU2  0 represents
all lines passing through the intersection of U1  0 and U2  0 including U1  0, but excluding U2  0. Hence, for all
practical purposes, we consider the equation U1 KU2  0, where K is a real parameter in solving the problems.

QUICK LOOK 11

Suppose U1  0 and U2  0 are two lines, then 2. If U1  0 and U2  0 are intersecting lines, then U1
KU2  0 represents a line passing through the inter-
1. If U1  0 and U2  0 are parallel lines, then U1 KU2
section of U1  0 and U2  0.
 0 represents a line parallel to each of U1  0 and
U2  0.

T H E O R E M 2.21 If U1  0 and U2  0 are two intersecting lines, then every line in the plane of U1  0 and U2  0 is
of the form K1U1 K2U2 K3  0.

PROOF Let U  0 be a line in the plane of U1  0 and U2  0. Therefore, either U  0 passes through the
intersection of U1  0 and U2  0 or it does not. In the former case, U  0 must be of the form K1U1
K2U2  0 for some real K1and K2, such that L1 L 2 x 0. In this case, K3  0. Suppose U  0 does
not pass through the intersection of U1  0 and U2  0. Hence, there is one and only one line L in
the plane passing through the intersection of U1  0 and U2  0 and parallel to the line L. Hence,
by Theorem 2.20, the equation of L must be of the form K1U1 K2U2  0. Since L is parallel to U
 0, then U  0 is of the form K1U1 K2U2 K3  0 for some real constant K3. Thus, the theorem is
proved.
2.1 Straight Line 41

Example 2.18

Find the equation of the line passing through the inter- (2x − y 5) − 5 (x y 1)  0
section of the lines 2x − y 5  0 and x y 1  0 and the
 3 x 6 y  0
origin.
 x 2y  0
Solution: By Theorem 2.20 and the Note mentioned Direct Method: Solving the equations 2x − y 5  0 and
below it, the required line is of the form x y 1  0, we get the point of intersection (−2, 1).
(2x − y 5  0) K(x y 1)  0 Therefore, the equation of the line joining (−2, 1) and
(0, 0) is
Since this also passes through (0, 0), we have K  −5.
Hence the required line is 0 1
y x or x 2y  0
0 2

Example 2.19

Find the equation of the line passing through the inter- (1 L ) 3


section of the lines x − 2y − 3  0 and x 3y − 6  0 and 
2 3L 4
parallel to the line 3x 4y − 7  0.
 4 4K  −6 9K K  2
Solution: We have U1 yx − 2y − 3  0 and U2 yx 3y Hence, the required line is 3x 4y− 15  0.
− 6  0. The required line is Direct Method: The point of intersection of the lines x−
U1 KU2  (1 K) x (−2 3K) y − 3 − 6K  0 2y − 3  0 and x 3y − 6  0 is (21/5, 3/5). Hence, from Quick
Look 5, part (2), the equation of the required line is
Since this line is parallel to 3x 4y − 7  0, the slopes
must be equal.Therefore ¥ 21´ ¥ 3´
3¦ x µ 4 ¦ y µ  0
§ 5¶ § 5¶
 3x 4y− 15  0

Example 2.20

Find the equation of the line passing through the inter- Thus, the required line equation is
section of the lines x 3y − 1  0 and x − 2y 4  0 and
¥ 11´ ¥ 22 ´ 44
perpendicular to the line 2x 3y  0. ¦§ 1 µ¶ x ¦§ 3 µ¶ y 1 0
4 4 4
Solution: We have U1 yx 3y − 1  0 and U2 yx − 2y  15 x 10 y 40  0
4  0. Equation of the required line is
 3x 2 y 8  0
U 1 LU 2 y (1+L )x (3 2L )y 1 4L  0
Direct Method: Let U1 yx 3y − 1  0 and U2 yx− 2y
Since this line is perpendicular to the line 2x 3y  0, we 4  0. Therefore, the point of intersection of the lines
have U1  0 and U2  0 is (−2, 1). Hence, from Quick Look 7,
part (3), the equation of the required line is
(1 L ) ¥ 2 ´
s ¦ µ  1
3 2L § 3 ¶ 3(x 2) − 2(y − 1)  0
 9 6 L  2 L 2  3x − 2y 8  0
11
L
4

T H E O R E M 2.22 Let U1 ya1x b1y c1  0, U2 ya2x b2y c2  0 and U3 ya3x b3y c3  0 be three lines such
that no two lines are parallel. Then, these lines are concurrent if and only if
42 Chapter 2 Straight Line and Pair of Lines

a1 b1 c1
a2 b2 c2  0
a3 b3 c3

PROOF Let P be the point of the intersection of the lines U2  0 and U3  0. Therefore, by Theorem 2.16,
we have
¥ b c b2 c3 c3 a2 c2 b3 ´
P¦ 3 2 ,
§ a3 b2 a2 b3 a3 b2 a2 b3 µ¶
Point P also lies on U1  0. So
¥ b c b2 c3 ´ ¥ c a c b ´
a1 ¦ 3 2 b1 ¦ 3 2 2 3 µ c1  0
§ a3 b2 a2 b3 µ¶ § a3 b2 a2 b3 ¶
š a1 (b3 c2 b2 c3 ) b1 (c3 a2 c2 a3 ) c1 (a3 b2 a2 b3 )  0
š a1 (b2 c3 b3 c2 ) b1 (a2 c3 a3 c2 ) c1 (a2 b3 a3 b2 )  0
a1 b1 c1
š a2 b2 c2  0
a3 b3 c3

T H E O R E M 2.23 Let U1 ya1x b1y c1  0, U2 ya2x b2y c2  0 and U3 ya3x b3y c3  0 be three lines such
that no two are parallel. If there exists non-zero real numbers K1, K2 and K3 such that K1U1 K2U2
K3U3  0, then the three lines U1  0, U2  0 and U3  0 are concurrent.
PROOF Suppose K1U1 K2U2 K3U3  0 where K1, K2 and K3 are non-zero real numbers. Therefore
¥ L ´ ¥ L ´
U3  ¦ 1 µ ¦ 2 µ U2
§ L3 ¶ § L3 ¶
which is of the form KU1 LU2  0. Hence, by Theorem 2.20, the line U3  0 passes through the
point of intersection of the lines U1  0 and U2  0. Therefore, the three lines are concurrent.

Note: Direct method of showing three lines to be concurrent: (a) Find the point of intersection of two of the three
given lines. (b) Verify whether the point lies on the remaining third line or not.

2.2 Pair of Lines


In this section, we obtain the equation of the angle bisectors of the angle between two intersecting lines. Also we will
study the condition for a second-degree homogeneous equation and general equation to represent pair of lines, and
the properties of these lines.

T H E O R E M 2.24 If a1x b1y c1  0 and a2x b2y c2  0 are intersecting lines, then the equations of their angle
bisectors are
a1 x + b1 y + c1 (a2 x + b2 y + c2 )
p
a12 b12 a22 b22

PROOF Let the lines represented by the given equations be L1 and L2, respectively (see Fig. 2.13) and
A be their point of intersection. Now
2.2 Pair of Lines 43

P(x, y) is a point on the bisector of the lines


š the perpendicular distances drawn from P on to the lines are equal

a1 x1 + b1 y1 + c1 a2 x1 + b2 y1 + c2
š  (by Theorem 2.14)
a12 b12 a12 b12
a1 x + b1 y + c1 (a2 x + b2 y + c2 )
š p
a12 b12 a22 b22

Therefore, the locus of P(x1, y1) is


a1 x + b1 y + c1 (a2 x + b2 y + c2 )
p
a12 b12 a22 b22

L2

L1
y

a1
0
P

x+
c2 =

b1
2y +

y+
c1
+b

=0
a2 x

O x

FIGURE 2.13

Note: Identification of Acute Angle Bisector Suppose L1 and L2 are not at right angles and let L1a and L2a be their
angle bisectors. Let P be the angle between L1 and L1a . If 0  tan Q  1, then L1a is the angle bisector; if tan Q  1 ,
then L2a is the acute angle bisector.
The following theorems are also useful in identifying acute or obtuse angle bisectors of two lines.

T H E O R E M 2.25 SupposeU1 ya1x b1y c1  0 and U2 ya2x b2y c2 0, where c1 x 0 and c2 x 0, be two intersecting
lines. If c1 and c2 are of same sign, then the equation
a1 x + b1 y + c1 (a2 x + b2 y + c2 )

a12 b12 a22 b22

gives the bisector of the angle in which the origin lies.


PROOF Suppose the lines represented by U1  0 and U2  0 are L1 and L2, respectively, and L1a and L2a are
their angle bisectors (see Fig. 2.14). Point P(x1, y1) lies on the origin angle bisector L1 š either
both the origin and P are on the same sides for both L1 and L2 or lie on opposite sides for both
L1 and L2. Since c1 and c2 are of same sign, it follows that (by Quick Look 10) L11  a1x
b1y c1 and L22  a2x b2y c2 are of same sign. Therefore
44 Chapter 2 Straight Line and Pair of Lines

a1 x + b1 y + c1 (a2 x + b2 y + c2 )

a12 b12 a22 b22

is the origin angle bisector.


y
L1
L2
L1

L2

P(x1, y1)
O x

FIGURE 2.14

T H E O R E M 2.26 Suppose U1 ya1x b1y c1  0 and U2 ya2x b2y c2 0 are two lines that do not pass through
origin and let c1 and let c2 have the same sign. Then
a1 x + b1 y + c1 (a2 x + b2 y + c2 )
1.  is the acute angle bisector if a1a2 b1b2  0.
a12 b12 a22 b22

a1 x + b1 y + c1 (a2 x + b2 y + c2 )
2.  if a1a2 b1b2  0.
a12 b12 a22 b22

PROOF Let L1 and L2 be the lines represented by U1  0 and U2  0, respectively. Draw OM and ON per-
pendicular to L1 and L2, respectively. Let NOM  @ so that MPN  O −@ (see Fig. 2.15). Now,
@ is acute or obtuse according to whetherO −@ is obtuse or acute. Also
a1a2 + b1b2
cos A   0 or > 0
(a12 b12 ) (a22 b22 )

according to whether @ is acute or obtuse. So,


a1a2 b1b2< 0 or > 0
š@ is acute or obtuse
Therefore,
1. If a1a2 b1b2  0, then the origin angle bisector is the acute angle bisector and hence its equa-
tion is
a1 x + b1 y + c1 (a2 x + b2 y + c2 )
 (by Theorem 2.25)
a12 b12 a22 b22

2. If a1a2 b1b2  0, then O – @ is obtuse and hence origin angle bisector is the obtuse angle bisec-
tor so that its equation is
a1 x + b1 y + c1 (a2 x + b2 y + c2 )
 (by Theorem 2.25)
a12 b12 a22 b22
2.2 Pair of Lines 45

y L2
L1
P a

a 2x + b 2
p-a

=0 M

y + c2 =
c 1
y+ N
b1 a
x+
a 1

0
O x

FIGURE 2.15

QUICK LOOK 12 IDENTIFYING ACUTE ANGLE BISECTOR

Let U1 ya1x b1y c1  0 and U2 ya2x b2y c2  0 be represents origin angle bisector (Theorem 2.25). The
intersecting lines and c1x 0 x c2. First, make both c1 and origin angle bisector (that is with sign) is the acute
c2 of same sign (or c1 and c2 are positive). Then angle or obtuse angle bisector according to whether
a1a2 b1b2 is negative or positive.
a1 x + b1 y + c1 (a2 x + b2 y + c2 )

a12 b12 a22 b22

Example 2.21

Find the angle bisectors of the angle between the lines x y+1 7x + y + 3
x−y 1 0 and 7x y 3  0 and identify the acute angle 
1 5
bisector.
 5 x 5 y 5  7 x y 3
Solution: We have U1 yx − y 1  0 and U2 y 7x y  12 x 4 y 8  0
3  0. Here, c1  1 and c2 3 are of the same sign.There-  3x y 2  0
fore, the angle bisectors are is the acute angle bisector.
x y+1 7x + y + 3 Aliter: Consider the line x − y 1  0 and the bisector 3x
p
2 2 − y 2  0 and let P be the angle between them.Hence
1 1 7 2 12
x y+1 7x + y + 3 m1 m2 1 3 1
 p tan Q    1
1 5 1 m1 m2 1 3 2
Since a1a2 b1b2  1(7) (−1)(1)  6  0, the bisector with Therefore, 3x − y 2  0 is the acute angle bisector.
sign is obtuse angle and hence

DEFINITION 2.7 Let a, h, b, g, f and c be real numbers and atleast one of the real numbers a, h and b be non-zero.
Then, the expression ax2 2hxy by2 is called second-degree homogeneous expression and ax2
2hxy by2 2gx 2fy c is called second-degree general expression in x and y. At the same
time, ax2 2hxy by2  0 is called second-degree homogeneous equation and ax2 2hxy
by2 2gx 2fy c  0 is called second-degree general equation.

Examples

1. 2x2 xy by2  0 3. x2 2 xy y2  0 5. x2 xy y2 − x y − 1  0
2. x2 − y2  0 4. 3x2 − 6xy y2 20x − y 3  0
46 Chapter 2 Straight Line and Pair of Lines

T H E O R E M 2.27 If h2 rab, then the locus represented by the equation ax2 2hxy by2  0 is a pair of lines passing
through the origin.
PROOF Since a is real, either a  0 or a x 0.
Case 1: a  0. In this case, the given equation becomes
y(2 hx by)  0 (2.12)
Since a  0, atleast one of the values of h and b is not zero so that 2hx by  0 is a first-degree
equation and hence it represents a straight line. Also, y  0 represents x-axis. Hence, Eq. (2.12)
represents pair of lines, namely, y  0 (i.e., x-axis) and the line 2hx by  0.
Case 2: a x 0. In this case, the equation is written as
a 2 x 2 2ahxy aby2  0
 (ax hy)2 (h2 ab) y2  0
Therefore

(ax hy h2 aby)(ax hy h2 aby)  0

The locus represented by the given equation is the pair of lines ax (h p h2 ab ) y  0 and both
these lines pass through origin. When h2  ab or h  b  0, these two lines are identical (coincide)
with each other.

Note: This is very useful in problem solving.


1. When h2 rab, let the lines represented by ax2 2hxy by2  0 be l1x m1y  0 and l2x m2y  0 so that ax2 2hxy
by2 y (l1x m1y) (l2x m2y). Equating the corresponding coefficients on both sides, we have
l1l2  a, l1m2 l2m1  2h
and m1m2  b
2. If both the lines represented by ax2 2hxy by2  0 are non-vertical having slopes m1 and m2, then
¥ 2h ´ ¥ a´
y2 ¦ µ xy ¦ µ x 2 y ( y m1 x)( y m2 x)
§ b¶ § b¶
so that m1m2  a/b and m1 m2 −(2h)/b.

QUICK LOOK 13

To find the lines represented by the second-degree so that the corresponding linear equations are the
homogeneous equation ax2 2hxy by2  0, factorise required equations.
the expression ax2 2hxy by2 into two linear factors

Example 2.22

Find the lines represented by the equation x2− 5xy pair of lines passing through origin. Further,
6y2  0.
x2− 5xy 6y2 y (x− 2y)(x− 3y)
Solution: We have a  1, h  −5/2 and b  6. Also h2   The lines are x − 2y  0 and x − 3y  0
25/4  6  ab. Therefore, x2− 5xy 6y2  0 represents a
2.2 Pair of Lines 47

T H E O R E M 2.28 If ax2 2hxy by2  0 represents a pair of lines and @ is the angle between them, then
a b
cos A 
(a b)2 4 h2

PROOF Suppose the lines are l1x m1y  0 and l2x m2y  0. Therefore, from Theorem 2.27, Note (1),
l1l2  a, l1m2 l2m1  2h and m1m2  b. From Theorem 2.12, we have

l1l2 m1 m2
cos A 
(l12 m12 )(l22 m22 )
a b

l12 l22 m12 m22 l12 m22 l22 m12
a b

(l1l2 m1 m2 )2 (l1 m2 l2 m1 )2
a b

(a b)2 4h 2

where @ is the acute or obtuse according to whethera b > 0 or < 0.

Note:
a b
1. gives the acute angle.
(a b)2 4 h2

2 h2 ab
2. tan A  also gives the acute angle between the lines.
a b
3. ax2 2hxy by2  0 represents a pair of perpendicular lines ša b  0 (i.e., coefficient of x2 coefficient of
y2  0).

T H E O R E M 2.29 Suppose ax2 2hxy by2  0 represents a pair of lines and (x1, y1) is a point in the plane. Then,
1. The equation of the pair of lines passing through (x1, y1) and parallel to these lines is a(x−x1)2
2h(x−x1) (y−y1) b(y−y1)2  0.
2. The equation of the pair of lines passing through (x1, y1) and perpendicular to the given lines
is b(x−x1)2− 2h(x−x1) (y−y1) a(y−y1)2  0.
PROOF Suppose the lines represented by ax2 2hxy by2  0 are l1x m1y  0 and l2x m2y  0. Hence,
from Theorem 2.27, Note (1), l1l2  a, l1m2 l2m1  2h and m1m2  b.
1. From Quick Look 5, part (2), the equations of the lines through (x1, y1) and parallel to the lines
l1x m1y  0 and l2x m2y  0 are l1 (x − x1) m1 (y − y1)  0 and l2 (x − x1) m2 (y − y1)  0.
Hence, their combined equation is
[l1 (x − x1) m1 (y − y1)][l2 (x − x1) m2 (y − y1)]  0
l1l2 (x − x1)2 (l1m2 l2m1) (x − x1) (y − y1) m1m2 (y − y1)2  0
a (x − x1)2 2h (x − x1) (y − y1) b (y − y1)2  0
2. From Quick Look 7, part (3), the equations of the lines through (x1, y1) and perpendicular to
the lines are m1 (x − x1) −l1 (y − y1)  0 and m2 (x − x1) −l2 (y − y1)  0. Hence, their combined
equation is
[m1 (x − x1) −l1 (y − y1)][m2 (x − x1) −l2 (y − y1)]  0
m1m2 (x − x1)2− (l1m2 l2m1) (x − x1) (y − y1) l1l2 (y − y1)2  0
b (x − x1)2− 2h (x − x1) (y − y1) a (y − y1)2  0
48 Chapter 2 Straight Line and Pair of Lines

Example 2.23

Find the equation of the pair of lines passing through the 1. Equation of the pair of lines passing through (1, 1) and
point (1, 1) and parallel to the lines x − 2y  0 and x − 3y  0 is
1. parallel to the lines x2 − 5xy 6y2  0. (x − 1)2 −5 (x − 1) (y − 1) 6(y − 1)2  0
2. perpendicular to the lines x2 − 5xy 6y2  0.  x2 − 5xy 6y2 3x − 7y 2  0
2. Equation of the pair of lines passing through (1, 1) and
Solution: The lines represented by x2 − 5xy 6y2  0 are
perpendicular to the lines is
x − 2y  0 and x − 3y  0. Therefore, from Theorem 2.29,
we get the following: 6(x − 1)2 5 (x − 1) (y − 1) (y − 1)2  0
 6x2 5xy y2− 17x − 7y 12  0

T H E O R E M 2.30 If ax2 2hxy by2  0 represents two intersecting lines, then the combined equation of the pair of
angle bisectors of the angle between the lines is

h( x 2 y2 )  (a b) xy
PROOF Suppose the lines are l1x m1y  0 and l2x m2y  0 so that l1l2  a, l1m2 l2m1  2h and m1m2  b.
By Theorem 2.24, the angle bisectors are
l1 x m1 y l2 x m2 y
p
l12 m12 l22 m22
 (l22 m22 )(l1 x m1 y)2  (l12 m12 )(l2 x m2 y)2
 (l12 m22 l22 m12 ) x 2 (l12 m22 l22 m12 ) y2  2 ¨ªl2 m2 (l12 m12 ) l1 m1 (l22 m22 )·¹ xy
 (l12 m22 l22 m12 )( x 2 y2 )  2[(l1 m2 l2 m1 )(l1l2 m1 m2 )]xy

Since the lines are intersecting, l1m2 − l2m1 x 0. Hence, cancelling l1m2 − l2m1 on both sides, we have
(l1 m2 l2 m1 )( x 2 y2 )  2 (l1l2 m1 m2 ) xy
 2 h( x 2 y2 )  2 (a b) xy
 h( x 2 y2 )  (a b) xy

T H E O R E M 2.31 The second-degree general equation S yax2 2hxy by2 2gx 2fy c  0 represents a pair of
lines if and only if
1. abc 2fgh − af 2 − bg2 − ch2  0.
2. h 2 rab, g 2 rac and f 2 rbc.
PROOF Suppose S  0 represents pair of lines and let the lines be l1x m1y n1  0 and l2x m2y n2  0.
Therefore
S y (l1x m1y n1) (l2x m2y n2)
Equating the corresponding coefficients on both sides, we have l1l2  a, l1m2 l2m1  2h and
m1m2  b, l1n2 l2n1  2g, m1n2 m2n1 2f, n1n2 c.
1. 8 fgh  (2 f )(2 g )(2 h)  (m1 n2 m2 n1 )(l1 n2 l2 n1 )(l1 m2 l2 m1 )
 l1l2 (m12 n22 m22 n12 ) m1 m2 (l12 n22 l22 n12 ) n1 n2 (l12 m22 l22 m12 ) 2l1l2 m1m2 n1 n2
 a [(m1 n2 m2 n1 )2 2 m1 m2 n1 n2 ] b[(l1 n2 l2 n1 )2 2l1l2 n1 n2 ]
c [(l1 m2 l2 m1 )2 2l1l2 m1 m2 ] 2abc
 a(4 f 2 2bc) b(4 g 2 2ca) c(4 h2 2ab) 2abc
 4(af 2 bg 2 ch2 abc)
2.2 Pair of Lines 49

Therefore

2 fgh  af 2 bg 2 ch2 abc or abc 2 fgh af 2 bg 2 ch2  0

Generally, the number of abc 2 fgh af 2 bg 2 ch2 is denoted by $. Therefore, $  0. Also


a h g
$ h b f
g f e

2. 4(h2 ab)  4 h2 4ab  (l1 m2 l2 m1 )2 4l1l2 m1 m2

 (l1 m2 l2 m1 )2 r 0 (equality holds if the lines are parallel).


Similarly, g2 rca
and f 2 rbc.

The proof of the converse part is a bit lengthy and beyond the scope of this book. Hence, we
assume the validity of the converse part.

T H E O R E M 2.32 If S yax2 2hxy by2 2gx 2fy c  0 represents a pair of lines, then the homogeneous equa-
tion ax2 2hxy by2  0 also represents pair of lines passing through origin and parallel to the
lines S  0.
PROOF Let the lines represented by S  0 be l1x m1y n1  0 and l2x m2y n2  0. Therefore
l1l2  a, l1m2 l2m1  2h, m1m2  b, l1n2 l2n1  2g, m1n2 m2n1  2f, n1n2  c, h2rab.
Now,

ax 2 2 hxy by2 y l1l2 x 2 (l1 m2 l2 m1 ) xy m1 m2 y2


y (l1 x m1 y)(l2 x m2 y)

Therefore, the lines represented by ax2 2hxy by2  0 are l1x m1y  0 and l2x m2y  0 which
are, respectively, parallel to the l1x m1y n1  0 and l2x m2y n2  0.

QUICK LOOK 14

1. If h2  ab, then ax2 2hxy by2  0 represents pair 2. If S yax2 2hxy by2 2gx 2fy c  0 represents
of coincidental lines so that in this case, S  ax2 a pair of distinct intersecting lines, this pair together
2hxy by2 2gx 2fy c  0 represents a pair of with the pair represented by ax2 2hxy by2  0 form
parallel lines. a parallelogram with the origin as one of the vertices.

Example 2.24

Prove that the equation S yx2 4xy 3y2 − 4x − 10y 3  0 2. h2  4, ab  3, h2 ab, g2  4, ac  3, g2 ac and f 2  25,
represents a pair of lines and find the equations of the lines. bc  9, f 2 bc.
Therefore, the given equation S yx2 4xy 3y2 − 4x −
Solution: We have a  1, h  2, b  3, g  −2, f  −5 and 10y 3  0 represents a pair of intersecting lines (by
c  3. Now, Theorem 2.31). Consider the equation x2 4xy
3y2  0. Therefore
1.
x 2 xy 3 xy 3 y2  0
a h g 1 2 2
x( x y) 3 y( x y)  0
$ h b f  2 3 5
( x y)( x 3 y)  0
g f e 2 5 3
 1 (9 25) 2 (6 10) 2 ( 10 6) Since S  0 represents lines parallel to x y  0 and x
 16 8 8  0 3y  0, we have
50 Chapter 2 Straight Line and Pair of Lines

x 2 4 xy 3 y2 4 x 10 y 3 y ( x y m)( x 3 y n) 3m n  10 (2.14)
and mn  3 (2.15)
Equating the coefficients of x and y and the constant
Solving Eqs. (2.13) and (2.14), we get m  −3, n  −1 so
terms, we have
that mn  3. Therefore, the lines represented by S  0
m n  4 (2.13) are x y− 3 0 and x 3y− 1  0.

2.2.1 Procedure to Find the Lines Represented by the Second-Degree General Equation
S y ax2 2hxy by2 2gx 2fy c  0
Step 1: Factorise the homogeneous part ax2 2hxy by2 and suppose
ax2 2hxy by2 (l1x m1y) (l2x m2y)
Step 2: S y(l1x m1y n1)(l2x m2y n2)
Step 3: Equate the corresponding coefficients of x and y and also the constant terms on both sides and solve
for n1 and n2.
Theorem 2.33 is the last result which is very useful in solving some locus problems. This theorem is called homogenis-
ing the second-degree curve equation with a straight line equation.

T H E O R E M 2.33 Suppose that the straight line lx my  1 meets the curve represented by the second-degree gen-
eral equation S yax2 2hxy by2 2gx 2fy
FHH E c  0 at E points A and B. If O is the origin, then
FHHtwo
the combined equation of the pair of lines OA and OB is

S a y ax 2 2 hxy by2 2 (gx fy)(lx my) c (lx my)2  0


PROOF Clearly the coordinates of both points A and B satisfy the line equation lx my 1 as well as
S  0 and hence points A and B satisfy Sa  0. Also (0, 0) satisfies Sa  0. That is, Sa  0 passes through
A, B and origin (see Fig. 2.16). On simplification, we can see that Sa  0 is a homogeneous equation
FHHE FHHE
of second degree representing pair of lines, which are nothing but the lines OA and OB.
y

B
2x
+
m
y=
1

O x

FIGURE 2.16

Example 2.25

Show that the lines joining the origin to the points of Solution: Suppose that the line [( x y)/ 2 ]  1 meets
intersection of the curve x2 − xy y2 3x 3y − 2  0 and the curve at points A and B (see Fig. 2.17). Therefore, the
the line x − y − 2  0 are mutually perpendicular. FHHE FHHE
combined equation of the pair of lines OA and OB is
2.2 Pair of Lines 51

2 y
2 2 ¥ x y´ ¥ x y´
x xy y (3 x 3 y) ¦ 2¦  0 (2.16)
§ 2 µ¶ § 2 µ¶
B

From Eq. (2.16), A

Coefficient of x2 Coefficient of y2  [1 (3/ 2 ) − 1] 90°


[1 –(3/ 2 ) – 1] O x
0
Therefore, from Theorem 2.28, Note (3), it implies that
AOB  90°. FIGURE 2.17

Subjective Problems
1. If p1 and p2 are the distances between the opposite Also
sides of a parallelogram and @ is one of its angles,
then show that the area of the parallelogram is p1p2 a1a2 b1b2
cos A 
cosec@. (a12 b12 )(a22 b22 )

Solution: ABCD is a parallelogram (see Fig. 2.18). AM Therefore


 p1, DN  p2 and BAD  @.
(a1a2 b1b2 )2
sin 2 A  1 cos2 A  1
Area of the parallelogram  (AB) p2 (2.17) (a12 b12 )(a22 b22 )
Now from $AMP, sin@  p1/AB and hence we have AB  (a1b2 a2 b1 )2
p1 cosec@. Therefore, from Eq. (2.17), the area of the par-  (2.18)
(a12 b12 )(a22 b22 )
allelogram  p1p2 cosec@.
Now the area of the parallelogram (by Problem 1) is
D C
a
p2 d1 c1 d2 c2 (a12 b12 )(a22 b22 )
p1 p2 (cosec A )  – –
p-a a12 b12 a22 b22 a1b2 a2 b1
a 90°
B
A N a
[from Eq. (2.18)]
p1 90° (d1 c1 )(d2 c2 )
M 
a1b2 a2 b1
FIGURE 2.18
3. Prove that the area of the parallelogram formed by the
2. Show that the area of parallelogram whose sides are lines 4y − 3x − a  0, 3y − 4x a  0, 4y − 3x − 3a  0 and
a1x b1y c1  0, a1x b1y d1  0, a2x b2y c2  0 and 3y − 4x 2a  0 is 2a2/7.
a2x b2y d2  0 is
Solution: Rewriting the equations of the sides of the
(d1 c1 )(d2 c2 ) parallelogram, we have
a1b2 a2 b1 3x − 4y a  0
4x − 3y − a  0
Solution: Consider Fig. 2.18. Let the equations of the
3x − 4y 3a  0
sides be a1x b1y c1  0, a1x b1y d1  0, a2x b2y
c2  0 and a2x b2y d2  0. Therefore, and 4x − 3y − 2a  0

d1 c1 Here, c1  a, d1  3a, c2  −a, d2  −2a, a1  3, b1  −4, a2 


p1  4 and b2  −3. Therefore, by Problem 2,
a12 b12
(d1 c1 )(d2 c2 )
d2 c2 Area 
p2  a1b2 a2 b1
a22 b22
52 Chapter 2 Straight Line and Pair of Lines

(3a a)( 2a a) 2a 2 Hence


 
3( 3) (4)( 4) 7 BD CE AF
– –  1
DC EA FB
4. Let p and q be non-zero real numbers and x1, x2 and
x3 be non-zero real roots of the equation x3 − 3px2 l A(x1, y1)
3qx − 1  0. Then show that the centroid of the tri-
angle whose vertices are [x1, (1/x1)], [x2, (1/x2)] and F
[x3, (1/x3)] is (p, q). E

Solution: Let y1  1/x1, y2  1/x2 and y3  1/x3. By


hypothesis,
x1 x2 x3  3p B(x2, y2) C(x3, y3) D
x1x2 x2x3 x3x1  3q
FIGURE 2.19
x1x2x3  1
Therefore Note: Either the line l meets all three sides externally or
two sides internally and one side externally. That is why
x1 x2 x3 3 p product of the ratios is equal to −1.
 p (2.19)
3 3
Now, Try it out The converse of Menelaus’ theorem
is also true.
y1 y2 y3 1 ¥ 1 1 1´
 ¦
3 3 § x1 x2 x3 µ¶
6. (Ceva’s Theorem) In the plane of $ABC, let O be a
1 ¥ x x x3 x1 x1 x2 ´ point (not on any side). If the lines AO, BO and CO
 ¦ 2 3 µ¶
3§ x1 x2 x3 meet the opposite sides BC, CA and AB at points D,
E and F, respectively, then show that
1 ¥ 3q ´
 ¦ µ q (2.20)
3§ 1 ¶ BD CE AF
– –  1
DC EA FB
From Eqs. (2.19) and (2.20), the centroid of the triangle is
(p, q). Solution: Without loss of generality, we may assume
that O is the origin and points A, B and C are (x1, y1), (x2,
5. (Menelaus’ Theorem) Suppose a straight line meets
y2) and (x3, y3), respectively. See Fig. 2.20. Observe that ei-
the sides BC, CA and AB of a triangle at points D, E FHHHE FHHHE FHHE
and F, respectively. Then show that ther all the three lines AO , BO and CO divide the sides
BC, CA and AB internally or two of them divide two sides
BD CE AF externally and one dividesFthe
– –  1 HHHE third
FHHHE side internally.
FHHE Now,
DC EA FB the equations of the lines AO , BO and CO are, respec-
tively, xy1−x1y  0, xy2−x2y  0 and xy3 − x3y  0. Therefore,
Solution: Let the coordinates of A, B and C be (x1, y1), by Theorem 2.17, we get
(x2, y2) and (x3, y3). See Fig. 2.19. Suppose L is the line
whose equation is L yax by c  0 meeting the sides in BD ( x y x1 y2 )
 2 1
D, E and F. Therefore, by Theorem 2.17, we get DC x3 y1 x1 y3
BD L22 CE ( x y x2 y3 )
  3 2
DC L33 EA x1 y2 x2 y1
CE L33 AF ( x y x3 y1 )
  1 3
EA L11 FB x2 y3 x3 y2
AF L11 Hence

FB L22 BD CE AF
– –  1
DC EA FB
2.2 Pair of Lines 53

A(x1, y1) B A (3, 4)

90°
M 2, 3
I
F

45°
O E
C (1, −1) D

FIGURE 2.21
B(x2, y2) D C (x3, y3)
Aliter (Using Complex Numbers): Let A  3 4i, C  1
FIGURE 2.20 − i, M  midpoint of AC  2 3/2i. Let B denote Z. Rotate
HHHHE
MA about M through 90° in the anticlockwise sense so
that
Try it out Converse of Ceva’s theorem is also
true. Z ; 2 (3 / 2)i =
i
(3 4i) [2 (3 / 2)i]

7. If A(3, 4) and C(1, −1) are the ends of a diagonal of 2Z 4 3i


 i
a square ABCD, then find the equations of the sides 2 5i
of the square. 1 5i
Z
2
Solution: See Fig. 2.21. Equation of the line AC is
¥ 1 5´
 B  ¦ , µ
4 1 § 2 2¶
y 4  ( x 3)
3 1
Similarly,
 2y − 8 5x − 15
¥ 1 5´ ¥ 9 1´
 5x − 2y − 7  0 D  ¦ 3 1 ,4 1 µ =¦ , µ
§ 2 2¶ § 2 2¶
Its slope is 5/2. Let the slope of side CD be m. Then
Now, we can write the sides equations because the verti-
m (5 / 2) ces are obtained as A(3, 4), B(−1/2, 5/2), C(1, −1), D(9/2,
DAC  45o  1  tan 45o  1/2).
1 (5m / 2)

2m 5 8. Find the equations of the lines passing through the


1
5m 2 point (2, 3) and making an angle 45° with the line 3x −
y 5  0.
 5m 2  p(2 m 5)
7 3 Solution: See Fig. 2.22. Let m be the slope of a side.
m or ABC  45° so that we have
3 7
Therefore, equations of CB and CD are m 3
1  tan 45o 
1 3m
7
y 1 ( x 1)  7 x 3 y 4  0
3 Therefore,
3
y 1  ( x 1)  3 x 7 y 10  0 3m 1  p(m 3)
7
1
Similarly, the equations of AB and BD, respectively, are m  2 or
2
3 Therefore, the equations of the lines are
y 4  ( x 3)  3 x 7 y 19  0
7
y − 3  − 2(x− 2) x y − 7  0
7
y 4  ( x 3)  7 x 3 y 33  0 1
3 y 3 ( x 2)  x 2 y 4  0
2
Therefore, the equations of the sides are 3x − 7y − 10  0,
7x 3y − 4  0, 3x − 7y − 19  0 and 7x 3y − 33  0.
54 Chapter 2 Straight Line and Pair of Lines

A(2, 3) vertex of the rectangle, find the equations of the oth-


er three sides.

Solution: Let ABCD be the rectangle (see Fig. 2.24)


where A is (−3, 1) and the equation of AD is 3x 7y
45° 2  0. Since the line joining (−3, 1) and (1, 1) is hori-
B 3x − y + 5 = 0 C zontal, the point (1, 1) must be the end of the diagonal
through A (−3, 1). Therefore, C is (1, 1) and the equation
FIGURE 2.22 of BC is

9. Two adjacent sides of a parallelogram are 4x 5y  0 3


y 1 ( x 1)
and 7x 2y  0 and one diagonal is 11x 7y − 9  0. 7
Find the equations of the other sides and the second  3 x 7 y 10  0
diagonal.
Equation of AB is
Solution: Solving the equations 4x 5y  0 and 11x 7
7y − 9  0, we have x  5/3, y  −4/3. Let A  (5/3, −4/3). y 1 ( x 3)
3
Solving the equations 7x 2y  0 and 11x 7y− 9  0, we  7 x 3 y 24  0
have x  −2/3, y  7/3. Let C  (−2/3, 7/3). Therefore,
Equation of the side CD is
¥ 2 7´ ¥ 5 4´
C ¦ , µ , O(0, 0) and A¦ , µ 7
§ 3 3¶ § 3 3¶ y 1 ( x 1)  7 x 3 y 4  0
3
are the three consecutive vertices of the parallelogram. Therefore, the other three sides are 3x 7y − 10  0, 7x
So its fourth vertex B (see Fig. 2.23) is − 3y 24  0 and 7x − 3y − 4  0.
¥ 2 5 7 4´
¦§ , µ¶  (1, 1) C(1, 1)
B
3 3 3 3
Therefore, the vertices of the parallelogram OABC are
O(0, 0), A(5/3, −4/3), B(1, 1) and C(−2/3, 7/3). Since the D A(−3, 1)
side BC is parallel to OA and passes through (1, 1), its 3x + 7y + 2 = 0
equation is
FIGURE 2.24
4
y 1 ( x 1) or 4 x 5 y 9  0
5 11. The three lines x 2y 3  0, x 2y − 7  0 and 2x −
y − 4  0 form the three sides of two squares. Find the
Also, the equation of the side AB is
equations of fourth side of each square.
7
y 1 ( x 1) or 7 x 2 y 9  0
2 Solution: Let the squares be ABCD and ADEF
with AD as common side (see Fig. 2.25). Solving x
and the second diagonal is x−y  0.
2y 3  0 and 2x − y − 4  0, we have A  (1, −2). Solv-
y ing the equations x 2y − 7  0 and 2x − y − 4  0,
B
we have D  (3, 2). The length of the sides of squares
C
 AD  (3 1)2 (2 2)2  4 16  2 5 . Let B (h, −
0

(3 h)/2) be a point on the line x 2y 3  0 such that


=
2y
+

A AB  2 5. Therefore,
7x

= 0
4x + 5y 2
¥ 3 h ´
O x (h 1)2 ¦ 2µ  20
§ 2 ¶
4(h 1)2 (h 1)2  80
FIGURE 2.23 (h 1)2  16
h 1  p4
10. One side of a rectangle lies along the line 3x 7y h  5 or 3
2  0 and (−3, 1) is a vertex on it. If (1, 1) is another
2.2 Pair of Lines 55

Therefore, B  (5, 4) and F  ( 3, 0). Hence, the equa- 13. The vertices B and C of $ABC lie on the line 4x − 3y
tion of the side BC is y 4  2(x − 5) or 2x − y − 14  0 and  0 and x-axis, respectively, BC passes through (2/3,
the equation of the side FE is y − 0  2(x 3) or 2x − y − 2/3) and ABOC is a rhombus where O is the origin.
6  0. Equations of the fourth side of the square are 2x − Find the equation of the line BC and the coordinates
y − 14  0 and 2x − y − 6  0. of the vertex A.
x + 2y − 7 = 0
E C Solution: See Fig. 2.27. OB  OC  CA  AB and the di-
D agonals OA and BC are at right angles. Let OC  a. Sup-
2x − y = 4 pose B  [x1, 4x1/3]. Equation of the side AB is y  (4x1)/3
and the coordinates of A are [a x1, (4x1)/3]. Hence
F B
A (CA)2  a 2
x + 2y + 3 = 0
16 x12
FIGURE 2.25
 x12  a2
9
5x
12. Each side of a square is of length 4. The centre of the ap 1
square is (3, 7) and one of its diagonals is parallel to the 3
line y  x. Find the coordinate of the vertices of square. y

Solution: See Fig. 2.26. ABCD is the square. M(3, 7) is


the centre of the square. AC is parallel to the line y  x. B
2,2 A
Therefore, the equation of the diagonal AC is 3 3

x
y − 7  1(x − 3)

=4
3y
 x y 4  0 (2.21)
Hence, the equation of the diagonal BD is O C x

y − 7  −1 (x − 3)
 x y 10  0 (2.22) FIGURE 2.27
Since the length of the side is 4, the lengths of the diago-
Case 1: If a  5 x1 /3, we have
nals are 4 2. Let A  (h, h 4) and MA  2 2. This
implies that ¥ 5x ´ ¥ 8x 4x ´ ¥ 4 x1 ´
C¦ 1, 0µ , A  ¦ 1 , 1 µ , B  ¦ x1 , µ and O  (0, 0)
§ 3 ¶ § 3 3 ¶ § 3 ¶
(h 3)2 (h 4 7)2  8
By hypothesis, points B, (2/3, 2/3) and C are collinear.
 (h 3)2  4
This implies that
 h  3p 2
4 x1
 h  5 or 1 x1 1
3
Hence, A  (5, 9) and C  (1, 5). Let B  (k, 10 −k) and 2 2
1 0
MD  2 2. This implies that 3 3
(k 3)2 (10 k 7)2  8 5 x1
0 1
 k 3  p2 3
 k  5 or 1 5 x1 ¥ 4 x1 2 ´ ¥ 2x 8x ´
 ¦§ µ 1¦ 1 1 µ  0
3 3 3¶ § 3 9 ¶
Hence, B  (5, 5) and D  (1, 9). Therefore, the vertices of
the square are (5, 9), (5, 5), (1, 5), (1, 9).  5 x1 (4 x1 2) (6 x1 8 x1 )  0
D C  20 x12 12 x1  0
3
 x1  0,
5
(3, 7)
Now, x1  0 B  (0, 0), which is actually not the origin.
Hence, x1  3 / 5. Therefore,
A 4 B
¥ 3 4´ ¥ 8 4´
O  (0, 0), B  ¦ , µ , A  ¦ , µ and C  (1, 0)
FIGURE 2.26 § 5 5¶ § 5 5¶
56 Chapter 2 Straight Line and Pair of Lines

5 x1 Note: To check whether our answers are correct or not,


Case 2: a  we need to verify the following:
3
1. The incentre I(1, 0) is inside $ABC.
2. The distance of I(1, 0) from the three sides must be
Try it out Try case 2 mentioned above, that is, equal.
a 5 x1 /3.
15. Determine all values of @ for which the point (@, @ 2)
lies inside the triangle formed by the lines 2x 3y −
14. In $ABC, A  (−4, 1). The internal bisectors the 1  0, x 2y − 3  0 and 5x − 6y − 1  0.
angles B and C are, respectively, x − 1  0 and x − (IIT-JEE 1992)
y − 1  0. Find the coordinates of B and C and the
equations of the sides AB and AC. Solution: Let L y 2x 3y − 1  0, Layx 2y − 3  0 and Lq y
5x − 6y − 1  0. Vertices are A (5/4, 7/8), B (1/3, 1/9) and
Solution: See Fig. 2.28. Let BE and CF be the bisectors C (−7, 5). See Fig. 2.29.
of the angles B and C whose equations are, respective-
ly, x − 1  0 and x – y − 1  0. Suppose M and N are the 1. For the side BC (L  0), the points A(5/4, 7/8) and
reflections of the vertex A in the bisectors BE and CF, (@, @ 2) must be on the same side (by Quick Look 9).
respectively. Hence, M and N lie on the line BC. Let M  ¥ 5´ ¥ 7´ 20 21 8
(h, k).Therefore, by Theorem 2.13, we have L11  2 ¦ µ 3 ¦ µ 1   0.
§ 4¶ § 8¶ 8
h 4 k 1 (4 1) ¥ k 1 ´
  2 ¦§ here means k  1µ  L22  2(A ) 3A 2 1  0
1 0 1 0 ¶
 h  2 and k  1  3A 2 2A 1  0

Hence, M  ( 2, 1). Let N  (ha, ka).Therefore,  (3A 1)(A 1)  0

ha 4 k a 1 (4 1 1) Therefore,
  2 2  4
1 1 1 12 1
 h a  0 and k a  3 A  1 or A  (2.26)
3
Hence, N  (0, 3) . Therefore, equation of the side BC is 2. For the side CA (La  0), the points B(1/3, 1/9) and (@,
@ 2) are on the same side. Therefore
¥ 3 1´
y 3 ¦ ( x 0)
§ 0 2 µ¶ 1 2 3 2 27
a 
L11 3 0
 2x − y 3  0 3 9 9

Equations of BE is and a  A 2A 2 3  0
L22 (by Quick Look 9)

x1 (2.23)  (2A 3)(A 1)  0

Equation of CF is 3
 A 1 (2.27)
x y1 (2.24) 2

Equation of BC is 3. For the side AB (Lq  0), the points C and (@, @ 2) are
on the same side. Therefore
2 x y  3 (2.25)
aa  5( 7) 6( 5) 1  0
L11
Solving Eqs. (2.23)–(2.25), we have B  (1, 5) and C 
(−4, −5). and aa  5A 6A 2 1  0
L22
A(4, −1)  6A 2 5A 1  0
 (3A 1)(2A 1)  0
F
E 1 1 (2.28)
A  or A 
x=1 3 2

B N M C From Eqs. (2.26)–(2.28), @  (−3/2, −1) ‡ (1/2, 1).

FIGURE 2.28
2.2 Pair of Lines 57

A
5,7 Therefore
4 8
4 m 16
x
4m 3

x
=

+
1

2y


6y

m(4 m 16)

3

=
x (a, a 2) and y 5 m
5x

0
4m 3
B
1,1 2x + 3y − 1 = 0 C (−7, 5) 4 m2 16 m 4 m2 17 m 15
3 9 
4m 3
FIGURE 2.29 m 15

4m 3
16. If the sum of the distances of a point from two per-
pendicular lines in a plane is 1, then prove that its Hence
locus is a square. (IIT-JEE 1992) ¥ 4 m 16 m 15 ´
Q¦ ,
§ 4 m 3 4 m 3 µ¶
Solution: Take the two perpendicular lines as coordi-
nate axes. P(x, y) is a point on the locus š|x| |y|  1. This Since M(1, 5) is the midpoint of PQ, we have
implies that
1 ¥ 9 m 4 m 16 ´
1 ¦ µ (2.30)
x y1 2 § 5 m 4m 3 ¶
x−y  1
1 ¥ 25 m m 15 ´
−x y  1 and 5 ¦ µ (2.31)
2 § 5 m 4m 3 ¶
−x−y  1
From Eq. (2.31), we get
These lines form a square.
2(5 m)(4 m 3)  (9 m)(4 m 3) (5 m)(4 m 16)
17. A line meets the straight lines 5x − y − 4  0 and 3x
 2( 4 m2 17 m 15)  ( 4 m2 33m 27) ( 4 m2
4y − 4  0 at points P and Q. If (1, 5) is the midpoint
of PQ, find the equation of the line PQ. 36 m 80)
 8m2 34 m 30  8m2 69m 53
Solution: See Fig. 2.30. Let the line PQ be  35m  83
y 5  m( x 1) (2.29) 83
m
Substituting y  mx 5 m in the equation 5 x y 4  0, 35
we have Substituting the value of m  83/35 in Eq. (2.29), equa-
5 x mx 5 m 4  0 tion of line PQ is obtained as
 (5 m) x m 9  0 83
y 5 ( x 1)
Therefore 35
 83 x 35 y 92  0
9 m ¥ 9 m´ 25 m
x and y  m¦ 5 m  The value of m obtained from Eq. (2.31) is also equal to
5 m § 5 m µ¶ 5 m
83/35.
Hence
¥ 9 m 25 m ´
P¦ ,
§ 5 m 5 m µ¶ Q
=4

Substituting y  mx 5 m in the equation 3x 4y 4  0, M(1, 5)


4y

we have
+

4
y= P
3x

3 x 4(mx 5 m) 4  0 5x −

 (3 4 m) x 16 4 m  0 20, 8
23 23

FIGURE 2.30
58 Chapter 2 Straight Line and Pair of Lines

19. A line through A(−5, −4) meets the lines x 3y


Try it out
2  0, 2x y 4  0 and x − y − 5  0 at the points B,
We can take the equation of the line PQ as C and D, respectively. If
x 1 y 5 2 2 2
 G ¥ 15 ´ ¥ 10 ´ ¥ 6 ´
cos Q sin
i Q ¦§ µ ¦§ µ  ¦§ µ
AB ¶ AC ¶ AD ¶
and substitute x  1 F cosP and y  5 F sinP in both then find the equation of line.
the line equations and obtain the coordinates of P (IIT-JEE 1993)
and Q. Finally, use that (1, 5) is the midpoint of PQ.
Solution: Let the line through A(−5, −4) be

18. The line joining two points A(2, 0) and B(3, 1) is rotat- x 5 y 4
 G (by Theorem 2.7)
ed about A through an angle 15° in the counterclock cos Q sin Q
sense. Find the equation of the line in the new posi- Therefore, every point on the line is of the form x  −5
tion and the coordinates of the new position of B. F cosP, y  −4 F sinP. Let AB  F1, AC  F2 and AD  F3.
Since B  (−5 F1 cosP, −4 F1 sinP ), we have
Solution: See Fig. 2.31. Let XAB  @ so that
( 5 G 1 cos Q ) 3 ( 4 G 1 sin Q ) 2  0
1 0
tan A   1 or A  45o ( & B lies on x 3 y 2  0)
3 2
Therefore
Since XAC  @ 15°  60°, the equation of the line AC
(point C is the new position of point B) is 15
G1 
cos Q 3 sin Q
y 0  tan 60o( x 2)  3 ( x 2) Hence
Therefore, equation of the line AB in its new position is 15
 cos Q 3 sin Q (& G 1  AB) (2.32)
3x y 2 3  0 AB
Similarly, points C and D lie on 2x y 4  0 and x − y
Since C  (x, 3 (x− 2)) and AC  AB  2, we have − 5  0, respectively. We have
( x 2)2 3( x 2)2  ( AC )2  2 10
 2 cos Q sin Q (2.33)
1 AC
 ( x 2)  p
2 6
1 and  cos Q sin Q (2.34)
 x  2p AD
2
Now, by hypothesis,
Hence 2 2 2
¥ 15 ´ ¥ 10 ´ ¥ 6 ´
¥ ¦§ µ¶ ¦§ µ¶  ¦§ µ
1 3´ AB AC AD ¶
C  ¦2 , µ
§ 2 2¶
Hence, from Eqs. (2.32)–(2.34), we have
The value of x  2 − 1/ 2 gives the position of point B,
(cos Q 3 sin Q )2 (2 cos Q sin Q )2  (cos Q sin Q )2
when AB is rotated about point A through angle 15° in
clockwise sense.  5 cos2 Q 10 sin 2 Q 10 sin Q cos Q  cos2 Q sin 2 Q
y 2 sin Q cos Q
C  4 cos2 Q 9 sin 2 Q 12 sin Q cos Q  0
B(3, 1)
 (2 cos Q 3 sin Q )2  0

15

 Therefore,
O A(2, 0) x
2
2 cos Q 3 sin Q  0 or tan Q 
3
FIGURE 2.31 Hence, the equation of the line is
2.2 Pair of Lines 59

2 x 2 y 2
y 4  ( x 5) or 2 x 3 y 22  0  G (by Theorem 2.7)
3 cos Q sin Q
See Fig. 2.33. Let BP  F 1, BQ  F 2 and BR  F . Therefore
20. One diagonal of a square is the portion of the line 7x
5y  35 intercepted between the axes. Determine P  (2 G 1 cos Q , 5 G 1 sin Q )
the extremities of the other diagonal.
and Q  (2 G 2 cos Q , 5 G 2 sin Q )
Solution: See Fig. 2.32. A(5, 0) and B(0, 7) are the ex- Since point P lies on x − 2y  0, we have
tremities of the given diagonal. Therefore, the slope of
(2 G 1 cos Q ) 2(5 G 1 sin Q )  0
the diagonal AB is −7/5. Hence, the slope of the other
diagonal, say, CD is 5/7. Therefore 8
G1  (2.35)
5 cos Q 2 sin Q
tan Q 
7 Similarly, Q lies on 2x − y  0, we have
so that 1
7 5
G2  (2.36)
cos Q  and sin Q  2 cos Q sin Q
74 74 By hypothesis,
The equation of the diagonal CD is 1 1 2

x (5 / 2) y (7 / 2) G1 G2 G
 G (say)
7 / 74 5 / 74
Therefore, from Eqs. (2.35) and (2.36), we get
Since, G  74 / 2, the other vertices of the diagonal are
cos Q 2 sin Q 2
(2 cos Q sin Q ) 
¥5 7 74 7 5 74 ´ 8 G
C¦ s , s  (6, 6)
§2 74 2 2 74 2 µ¶ 16
 17 cos Q 10 sin Q  (2.37)
G
¥5 74 7 7 74 5 ´
and D¦ s , s µ Let R  (x, y) so that
§2 2 74 2 2 74 ¶
x  2 F cosP  F cosP  x − 2
¥ 5 7 7 5´ y  5 F sinP  F sinP  y – 5
 ¦ , µ  ( 1, 1)
§ 2 2 2 2¶
Therefore, from Eq. (2.37),
y 16  17G cos Q 10G sin Q
B(0, 7)  17( x 2) 10( y 5)
The locus of R is 17x− 10y  0.
5,7
M y
2 2
Q

B(2, 5)
A(5, 0)

0 P
2y =
FIGURE 2.32 x−

21. A variable line l passing the point B(2, 5) intersects O x


the lines 2x2 − 5xy 2y2  0 at P and Q. Find the locus
of the point R such that the distances BP, BR and FIGURE 2.33
BQ are in harmonic progression (HP).
22. ABC is an equilateral triangle in which B  (1, 3) and
Solution: The given equation 2x2 − 5xy 2y2  0 rep- C  (−2, 7). Find the coordinates of the vertex A.
resents the pair of lines 2x − y  0 and x − 2y  0. Let the
equation of the line through B(2, 5) be
60 Chapter 2 Straight Line and Pair of Lines

Solution: See Fig. 2.34. The slope of BC is 3 3


y  5
7 3 4 2

2 1 3 Hence
Therefore, the slope of perpendicular bisector of BC is ¥ 1 3 3 ´ ¥ 1 3 3´
3/4 and its equation is A  ¦2 3 , 5µ or ¦ 2 3 , 5
§ 2 2 ¶ § 2 2 µ¶
3¥ 1´
y 5 ¦ x µ¶
4§ 2 A

where M(−1/2, 5) is the midpoint of BC. Since


BC  (7 3)2 ( 2 1)2  5, we have

5 3 60° 60°
Altitude AM = (sin 60o) AB  C(−2, 7)
2 B(1, 3) −1,
M 5
2
The equation of the altitude AM is
FIGURE 2.34
3
y  (2 x 1) 5 (2.38)
8 Aliter (Using Complex Numbers): B  1 3i and C  −2
Now, 7i. Rotate CB about C through an angle 60° in anti-
clockwise sense (see Fig. 2.35). Thus
75
 ( AM )2
4 Z ( 2 7i)
 cos 60o i sin 60o
¥ 1´
2 (1 3i) ( 2 7i)
 ¦ x µ ( y 5)2 [where A  ( x, y)]
§ 2¶ Therefore
2
¥ 1´ ¨3 · ¥ 1 i 3´ 3 4 3 i(3 3 4)
 ¦ x µ © (2 x 1) 5 5¸ [by Eq. (2.38)] Z 2 7i  ¦
§ 2 ¶ ª8 ¹ 2 µ ( 3 4i )  2
§ ¶
(2 x 1)2 9 This gives
 (2 x 1)2
4 64
3 4 3 i(3 3 4)
25(2 x 1) 2 Z 2 7i
 2
64 3 4 3 i(3 3 4) 4 14i

Therefore 2
(2 x 1)2  48 4 3 i(3 3 10)

2
 2 x 1  p4 3
¥ 1´ ¥ 3 3 ´
1 p 4 3  ¦2 3 µ i¦ 5µ
x § 2 ¶ § 2 ¶
2
Therefore
1 ¥ 1´
x2 3 and ¦ 2 3 µ ¥ 1 3 3 ´
2 § 2¶ A  ¦2 3 , 5µ
§ 2 2 ¶
When x  2 3 (1/2) we have Similarly, if we rotate CB in clockwise sense about C
3¥ 1´ through an angle 60°, we get the second position of A.
y  ¦x µ 5
4§ 2¶ A( Z )
3¥ 1 1´
 ¦2 3 µ 5 60°
4 § 2 2¶

3 3
 5 60° 60°
2
C(−2, 7) B(1, 3)
and when x  2 3 (1/ 2) we have
FIGURE 2.35
2.2 Pair of Lines 61

y D
Try it out Solve the same problem by taking
the equation of the altitude AM
M as
1
x
2  y 5  G A(0, k)
cos Q sini Q
where cosP  4/5 and sinP  3/5. (1,
2)
C

Note: With regard to geometric problems concerning O


equilateral triangles, squares, etc., the method of using x
complex numbers is easier than the method of using
coordinates. B

23. The sides of a rhombus are parallel to the lines y


FIGURE 2.36
2x  3 and y  7x 2. The diagonals intersect at (1, 2).
If one vertex lies on the y-axis, then find the coordi-
24. A ray of light is sent along the line x − 2y 5  0. Upon
nates of this vertex.
reaching the line 3x − 2y 7  0, the ray is reflected
Solution: See Fig. 2.36. Let ABCD be the rhombus from it. Find the equation of the reflected ray.
where A  (0, k). AB and CD are parallel to y  7x 2
whereas BC and AD are parallel to y 2x  3. Hence, the Solution: See Fig. 2.37. We have
equation of AB is x 2y 5  0 (Incident ray) (2.41)
y  7x k (2.39) 3x 2 y 7  0 (Surface line) (2.42)
Since (1, 2) is the midpoint of AC and A (0, k), it fol- P  (−1, 2) is the point of incidence of the lines given in
lows that C  (2, 4 − k). Also, BC is parallel to 2x 3 and Eqs. (2.41) and (2.42). Hence, the equation of the normal
passes through C (2, 4 − k). Hence, the equation of BC is at P(−1, 2) is
y (4 k )  2( x 2) 2
y 2  ( x 1)
 y  2x k (2.40) 3
Solving Eqs. (2.39) and (2.40), we have  2 x 3y 4  0 (2.43)

¥ 2k 9k ´ Let m be the slope of the reflected ray. Since the normal


B  ¦ , µ at P makes equal angles with the incident line and
§ 5 5¶
reflected line, we have
¥ 2k 9k ´
Since A  (0, k ), B  ¦ , µ , C  (2, 4 k ) and AB 2 2 1
§ 5 5¶ m
 BC, we have 3  3 2
2m ¥ 2 ´ ¥ 1 ´
2 2 2 2 1 1 ¦ µ ¦ µ
¥ 2k ´ ¥ 9k ´ ¥ 2k ´ ¥ 9k ´ 3 § 3 ¶ § 2¶
¦§ µ¶ ¦§ k µ¶  ¦§ 2 µ¶ ¦§ 4 k µ¶
5 5 5 5 3m 2 7
2 2
 
 10k  k 5 3 2m 4
 k 5  p 10k 3m 2 7
 p
5 5 5 3 2m 4
k  ( 10 1), ( 10 1)
10 1 9 9 1 29
m ,
Therefore 2 2

¥ 5 ´ ¥ 5 ´ However, m x 1/2. So that the reflected line is


A  ¦ 0, ( 10 1)µ , ¦ 0, ( 10 1)µ
§ 9 ¶ § 9 ¶ 29
y 2  ( x 1)
2
62 Chapter 2 Straight Line and Pair of Lines

Normal Solution: See Fig. 2.39. The distance between the par-
allel lines is

=0
+5
12 7 19


2y
2 2 13

x−
q q 2 3
3x − 2y + 7 = 0 Let P  (3, −5). Let p1 be the distance from the first line,
P which is given by

FIGURE 2.37 2(3) 3( 5) 7 16



2 2 13
2 3
25. Straight lines 3x 4y  5 and 4x − 3y  15 intersect at
the point A. Points B and C are chosen on those lines and p2 be the distance from the second line, which is
such that AB  AC. Determine the possible equa- given by
tions of the line BC passing through the point BC.
2(3) 3( 5) 12 3

Solution: See Fig. 2.38. The two lines intersect at A(−3, 2
2 3 2 13
1). Clearly, the lines 3x 4y  5 and 4x − 3y  15 are at right
angles to each other and AB  AC implies that $ABC is a Therefore
right-angled isosceles triangle with its right angle at ver- 19
tex A. Let m be the slope of the line BC. Hence p1 p2 
13
3 This implies that P(3, 5) lies in between the two lines. Let
m
4 4m 3 m be the slope of a line through P (3, −5). Therefore, by
1  tan 45o  
3m 4 3m hypothesis, we have
1
4
2
 4 m 3  p(4 3m) m
1  tan 45o  3
Two cases arise: 2m
1
3
1
Case 1: 4 m 3  (4 3m)  m  .
7 3m 2
  p1
Case 2: 4 m 3  4 3m  m  7. 3 2m

Therefore, the equation of the line BC is Two cases arise:


1
1 Case 1: 3m 2  3 2 m  m  .
y 2  ( x 1)  x 7 y 13  0 5
7 Case 2: 3m 2  (3 2 m)  m  5.
or y 2  7( x 1)  7 x y 9  0
Therefore, the required lines are
C 1
y 5  ( x 3)  x 5 y 28  0
45° 5
4x − 3y = 15

and y 5  5( x 3)  5 x y 10  0
(1, 2)
2x + 3y = 7 45°

90° 45°
A 3x + 4y = 5 B (3, −5)

45°
FIGURE 2.38
2x + 3y = −12

26. Show that the point (3, −5) lies between the parallel FIGURE 2.39
lines 2x 3y  7 and 2x 3y  −12 and find the equa-
tion of the lines through (3, −5) cutting the above 27. A(10, 0) and B( 5, 5) are two vertices of a triangle
lines at an angle 45°. whose incentre is the origin. Find the coordinates of
the third vertex.
2.2 Pair of Lines 63

Solution: See Fig. 2.40. Let C(h, k) be the third vertex. Two cases arise:
Since O(0, 0) is the incentre, BO and AO are the bisec-
Case 1: 4 m 3  3m 4  m  7.
tors of the angles B and A, respectively. The equation of
BO is 1
Case 2: 4 m 3  (3m 4)  m  .
7
x y0 (2.43)
Therefore, the equations of the sides AB and AC are
The equation of AO is
y 2  7( x 2)  7 x y 16  0
y0 (2.44)
1
and y 2  ( x 2)  x 7 y 12  0
Equation of the side AB is 7
0 5 C
y ( x 10)
10 5
45°

3x
 x 3 y 10  0 (2.45)

+
4y
Since BO is the angle bisector of B, the image of C(h, k)

=
4
in the line BO lies on the line AB. Since the equation of
45°
BO is y  x, the image of C(h, k) in BO is (k, h) and this
A(2, 2) B
lies on AB. Therefore
FIGURE 2.41
k 3h  10 (2.46)
The image of C(h, k) in the angle bisector AO lies on the 29. The points (1, 3) and (5, 1) are two opposite vertices
side AB. That is, (h, k) lies on the side AB. Therefore, of a rectangle. The other two vertices lie on the line
from Eq. (2.45), we get y  2x c. Find c and the other remaining vertices.
h 3( k )  10 (2.47)
Solution: See Fig. 2.42. ABCD is the rectangle in which
Solving Eqs. (2.46) and (2.47), we get h  2 and k  4. A  (1, 3), C  (5, 1). Points B and D lie on the line y 
Hence, the third vertex is ( 2, 4). 7x c. The diagonals intersect in (3, 2) which lies on the
y line y  7x c. Therefore, 2  2(3) c or c  4. That is, the
equation of the diagonal BD is
y  2x 4 (2.48)
C
Suppose M  (3, 2) is the midpoint of the diagonals.
Therefore,

MD  MB  (1/ 2) AC  5
O A(10, 0) x
Let B be (x, 2x 4). Therefore

B(−5, −5) MB  5
 (MB)2  5
FIGURE 2.40  (3 x)2 (2 2 x 4)2  5
 ( 3 x ) 2 (6 2 x ) 2  5
28. Find the equations of the sides of a right-angled isos-
celes triangle whose hypotenuse is the line 3x 4y  5 x 2 30 x 40  0
4  0 and the right angle vertex is the point (2, 2).  x2 6x 8  0
 x  2, 4
Solution: See Fig. 2.41. Let the slope of AB be m.
Therefore Hence, B  (2, 0) and D  (4, 4).
CBA  45o D C(5, 1)

¥ 3 ´ M(3, 2)
m ¦ µ
§ 4¶
 1  tan 45o 
¥ 3 ´
1 m¦ µ
§ 4¶ A(1, 3) B

 4 m 3  p(3m 4) FIGURE 2.42


64 Chapter 2 Straight Line and Pair of Lines

30. The ends of a straight line segment AB of constant 31. A variable straight line through the point of intersec-
length c move on two perpendicular lines OX and tion of the lines
OY which are the coordinate axes. If the rectangle x y x y
OAPB is completed, then show that the locus of the  1 and  1
a b b a
foot of the perpendicular drawn from P on to AB is meets the coordinate axes at A and B. Show that the
x 2/3 y2/3  c 2/3. locus of the midpoint of AB is the curve

Solution: See Fig. 2.43. Let A  (a, 0) and B  (0, b) so 2 (a b) xy  ab(x y)


that the equation of the line AB is
Solution: Equation of any line through the intersec-
x y tion of the given lines is of the form
1 (2.49)
a b
x y ¥x y ´
and also 1 L ¦ 1µ  0
a b §b a ¶
a 2 b2  c 2 (2.50)
This line meets the x-axis at
Let M(x1, y1) be the foot of the perpendicular drawn
from P onto the line AB. Since M(x1, y1) lies on AB, we ¥ ab(1 L ) ´ ¥ ab(1 L ) ´
A¦ , 0µ and B  ¦ 0, µ
have § b aL ¶ § a bL ¶
x1 y1
1 (2.52) Let M(x1, y1) be the midpoint of AB. Therefore
a b
Since P  (a, b) and PM is perpendicular to AB, we ab(1 L ) ab(1 L )
2 x1  and 2 y1 
have b aL a bL

Slope of PM s Slope of AB  1 Therefore

Therefore, 1 1 (b aL ) (a bL ) (a b)(1 L ) a b
  
2 x1 2 y1 ab(1 L ) ab(1 L ) ab
¥ b y1 ´ ¥ b 0 ´
¦§ a x µ¶ ¦§ 0 a µ¶  1 Hence
1

 ax1 by1  a 2 b2 (2.53) (a b)2 x1 y1  ab( x1 y1 )

From Eqs. (2.52) and (2.53), we get Therefore, the locus of (x1, y1) is

a3 b3 2(a b) xy  (ab)( x y)
x1  , y1 
a2 b 2
a 2 b2 32. The equations of the perpendicular bisectors of the
Therefore sides AB and AC of the triangle ABC are x y 5 
0 and x 2y  0, respectively. If the point A is (1, 2),
a 2 b2 then find the equation of the side BC.
x12/3 y12/3  2 2 2/3
 (a 2 b2 )1/3  c 2/3
(a b )
Solution: See Fig. 2.44. Perpendicular bisectors of the
[by Eq. (2.50)]
sides AB and AC, respectively, are
Therefore, the locus of M(x1, y1) is
x y 5  0 (2.54)
x 2/3 y2/3  c 2/3
and x 2y  0 (2.55)
y
Solving Eqs. (2.54) and (2.55), the circumcentres of
B P (a, b) $ABC is  10/ 3, 5/ 3 . Also, the equation of AB is
y 2  1( x 1)
M
 x y 1 0
Suppose B  (x, x 1). We have
O A x SA  SB
2 2 2 2
¥ 10 ´ ¥5 ´ ¥ 10 ´ ¥ 5´
¦ 1µ ¦ 2µ  ¦ x µ ¦ x 1 µ
FIGURE 2.43 § 3 ¶ §3 ¶ § 3¶ § 3¶
2.2 Pair of Lines 65

A(1, −2)
 132 112  (3 x 10)2 (3 x 8)2
 18 x 2 108 x 126  0 x−y y =0
+5= +2
0
 2 x 2 12 x 14  0 x

 x2 6x 7  0 −10 , 5
S
 ( x 7)( x 1)  0 3 3

 x  1, 7 B C
Hence FIGURE 2.44
x  1  ( x, x 1)  (1, 2)  A Aliter: A  (1, 2) and the perpendicular bisector of AB
and x  7  B  ( 7, 6) is

Therefore x y 5  0 (2.56)
B  ( 7, 6) Hence, B is the image of A in the given line in Eq. (2.56).
If B  (h, k), then by Theorem 2.13, we have
Similarly, equation of AC is
h 1 k 2 2[1 ( 2) 5]
y 2  2 ( x 1)    8
1 1 12 ( 1)2
 y  2x 4
Suppose C  (h, 2h 4), we have Therefore

SC  SA h  7, k  6
2 2 2 2 Hence
¥ 10 ´ ¥ 5´ ¥ 10 ´ ¥5 ´
¦ hµ ¦ 2 h 4 µ  ¦ 1µ ¦ 2µ
§ 3 ¶ § 3¶ § 3 ¶ §3 ¶ B  ( 7, 6)
2
  3h 10 (6 h 17)2  132 112 Similarly, C (x1, y1) is the image of A in the line x 2y  0
which implies that
 45h2 144 h 99  0
x1 1 y1 2 2(1 4) 6
 5h2 16 h 11  0   2 
1 2 1 22 5
 5h2 5h 11h 11  0 6 11
 5h(h 1) 11(h 1)  0  x1  1
5 5
11 12 2
 h  1, and y1  2 
5 5 5
Now
Therefore
h  1  the point (h, 2h 4)  (1, 2)  A
¥ 11 2 ´
11 ¥ 11 2 ´ C¦ , µ
h C  ¦ , µ § 5 5¶
5 § 5 5¶
Thus, the equation of the side BC is
Hence
14 x 23 y 40  0
¥ 11 2 ´
B  ( 7, 6) and C  ¦ , µ
§ 5 5¶ 33. A line cuts x-axis at A(7, 0) and y-axis at B(0, 5). A
variable line PQ is drawn perpendicular to AB cut-
This implies that the equation of the side BC is
ting the x-axis at P and the y-axis at Q. If AQ and BP
6 (2 / 5) intersect at R, then find the locus of R.
y 6  ( x 7)
7 (11/ 5) (IIT-JEE 1990)
28 FHHHE
 ( x 7) Solution: See Fig. 2.45. The slope of AB is
46
14 5 0 5
 ( x 7) 
23 0 7 7
 14 x 23 y 40  0
66 Chapter 2 Straight Line and Pair of Lines

Therefore, the slope of PQ is –7/5. Consider $ABQ in Similarly,


which QP is the altitude from Q onto AB and AP is the gx2 fy2  0 (2.59)
altitude from A onto BQ. These two intersect at P. Hence,
BP is the third altitude of $ABQ. Therefore, BR is per- Therefore, g  f  0 because x1 y2 x2 y1 x 0 or the
pendicular to AR. Hence, if R  (h, k), then matrix

Slope of BR s Slope of AR  1 ¨ x1 y1 ·
©x y2 ¸¹
¥ k 5´ ¥ k ´ ª 2
¦  1
§ h µ¶ ¦§ h 7 µ¶ is a non-singular matrix. Equations (2.58) and (2.95)
 h2 k 2 7 h 5k  0 have zero solution only so that g  0 and f  0.
Therefore, the locus of R(h, k) is
35. If S y a x 2 2 hxy by2 2 gx 2 fy c  0 represents
2 2
x y 7 x 5y  0 a pair of distinct lines (h2 ab), then prove that their
point of intersection satisfies the equations ax hy g
y  0, hx by f  0 and gx fy c  0, and the point of
Q intersection is
¥ hf bg gh af ´
¦§ , µ
ab h2 ab h2 ¶
R
Solution: Since S  0 represents a pair of lines, we
have
P
O A (7, 0) x $  abc 2 fgh af 2 bg 2 ch2  0 (by Theorem 2.31)
90°
and h2 ab implies that the lines are distinct intersecting
lines. Let P(x1, y1) be the point of intersection (see Fig.
2.46). Shift the origin to the point P(x1, y1) and let the
B (0, −5) new coordinates be (X, Y) so that by Section 1.4.1, x  X
x1, y  Y y1. Therefore, S  0 is transformed to

S y a ( X x1 )2 2 h ( X x1 )(Y y1 ) b (Y y1 )2
FIGURE 2.45 2 g ( X x1 ) 2 f (Y y1 ) c  0
 S y aX 2 2hXY bY 2 2 (ax1 hy1 g )X
34. Let S yax2 2hxy by2 2gx 2fy c  0 where h2
ab represent a pair of lines both passing through 2 (hx1 by1 f )Y ax12 2 hx1 y1 by12
origin. Prove that g  f  c  0. 2 gx1 2 fy1 c  0 (2.60)

Solution: Since S  0 passes through (0, 0) we have Since Eq. (2.60) represents a pair of lines through origin
c  0. Let P(x1, y1) and Q(x2, y2) be points on one of the P, by Problem 34, we have
lines other than the origin so that area of $OPQ x 0. This ax1 hy1 g  0 (2.61)
implies that
hx1 by1 f  0 (2.62)
x1 y2 x2 y1 x 0
and S11  ax12 2 hx1 y1 by12 2 gx1 2 fy1 c  0 (2.63)
 x1 y2 x2 y1 x 0 (2.57)
Solving Eqs. (2.61) and (2.62) for x1 and y1, we get
P(x1, y1) lies on one line which passes through origin.
This implies that ( x1, y1) also lies on the line. There- hf bg
x1 
fore, ab h2
gh af
ax12 2 hx1 y1 by12 2 gx1 2 fy1  0 y1 
ab h2
and ax12 2 hx1 y1 by12 2 gx1 2 fy1  0 Therefore, the point of intersection is
imply that ¥ hf bg gh af ´
¦§ , µ
gx1 fy1  0 (2.58) ab h2 ab h2 ¶
2.2 Pair of Lines 67

Also, from Eq. (2.63), we have Hence, the point of intersection is


(ax1 hy1 g ) x1 (hx1 by1 f ) y1 gx1 fy1 c  0 ¥ 12 2 ´
¦§ , µ
Since ax1 hy1 g  0 and hx1 by1 f  0, we have 7 7¶

gx1 fy1 c  0
37. Show that the equation of the pair of lines passing
The point of intersection of the lines represented by S  0 through origin and perpendicular to the lines repre-
satisfy the following three equations: sented by the equation ax2 2hxy by2  0 is bx2
2hxy ay2  0.
ax1 hy1 g  0 º
®
hx1 by1 f  0» (2.64) Solution: Suppose the lines represented by ax2 2hxy
gx1 fy1 c  0 ®¼ by2  0 are l1x m1y  0 and l2x m2y  0 so that we
have
Y l1l2  a º
®
y l1 m2 l2 m1  2 h» (2.65)
and m1 m2  b ®¼

We know that [see Quick Look 7, part (3)] the equations


P X of the lines passing through the origin and perpendicular
to the lines l1x m1y  0 and l2x m2y  0, respectively,
are m1x l1y  0 and m2x l2y  0 and hence their com-
bined equation is
(m1 x l1 y)(m2 x l2 y)  0
O x
 (m1m2) x2 (l1m2 l2m1) xy (l1l2) y2  0
From Eq. (2.65), we have

FIGURE 2.46 bx2 2hxy ay2  0

38. Show that the product of the perpendicular distances


QUICK LOOK 15 of the lines ax2 2hxy by2  0 from a point (x0, y0)
Solving any two equations given in Eq. (2.64), we get is
the point of intersection of lines represented by
S  0. ax02 2 hx0 y0 by02
(a b)2 4 h2
36. Find the point of intersection of the lines represent-
ed by S y3x2 xy 4y2 10x 4y 8  0.
Solution: Suppose the given lines are l1x m1y  0 and
l2x m2y  0 so that
Solution: Comparing the given equation with the
second-degree general equation, we have a = 3, h = 1/2, l1l2  a
b  4, g  5, f  2 and c  8. So
l1 m2 l2 m1  2 h
1 and m1 m2  b
hf bg  (2) ( 4)5  21
2 See Fig. 2. 47. Let d1 and d2 be the distances of the lines
¥ 1´ 7 from P(x0, y0). Therefore, by Theorem 2.14, we have
gh af  5 ¦ µ (3)2 
§ 2¶ 2
l1 x0 m1 y0
Therefore d1 
l12 m12
hf bg 21 21 s 4 12
2
  
ab h (49 / 4) 49 7 l2 x0 m2 y0
and d2 
gh af 7 4 2
 s  l22 m22
ab h2 2 49 7
68 Chapter 2 Straight Line and Pair of Lines

Hence Therefore, the area of $OAB is

(l1 x0 m1 y0 )(l2 x0 m2 y0 ) 1 l1 m2 l2 m1
d1d2  (by Section 1.2.4)
2 (lm1 l1 m)(lm2 l2 m)
(l12 m12 )(l22 m22 )

l1l2 x02 (l1m2 l2 m1 ) x0 y0 m1 m2 y02 1 (l1 m2 l2 m1 )2 4l1l2 m1 m2


 
(l1l2 m1 m2 )2 (l1 m2 l2 m1 )2 2 l 2 m1m2 (l1 m2 l2 m1 )lm l1l2 m2

ax02 2 hx0 y0 by02 1 4 h2 4ab


 
(a b)2 4 h2 2 bl 2 2hlm am2

y h2 ab

bl 2 2 hlm am2
P(x0, y0)
l2 x 0
d1
+
m d2 y= y
2y m1
=
0 x+
l1
lx + my
B =1
O x

O x

FIGURE 2.47 FIGURE 2.48

39. Find the area of the triangle formed by the lines Note: If the given line is lx my n  0 (n ≠ 0), then
ax2 2hxy by2  0 and lx my  1 write

Solution: Let the lines be l1x m1y  0 and l2x m2y ¥ l ´ ¥ m ´


¦§ µ¶ x ¦§ µy1
 0 so that n n ¶

l1l2  a and use the above formula so that the area is


l1 m2 l2 m1  2 h n2 h2 ab
and m1 m2  b bl 2 2 hlm am2
Suppose the line lx my  1 meets these lines at A and B
(see Fig. 2.48). Substituting y  l1 / m1 in lx my  1, we 40. If n≠ 0, then show that the triangle formed by the
have lines (lx my)2 3(mx ly)2 and lx my n  0 is
equilateral and find its area.
m1 l1
x and y 
lm1 l1 m lm1 l1 m Solution: See Fig. 2.49. The equation of the sides pass-
Thus, ing through origin is

¥ m1 l1 ´ (l 2 3m2 ) x 2 8 lmxy (m2 3l 2 ) y2  0 (2.66)


A¦ ,
§ lm1 l1 m lm1 l1 m µ¶ Let P be the angle between these sides. Therefore, by
and similarly, Theorem 2.28, Note (3), we have

¥ m2 l2 ´ 2 16l 2 m2 (l 2 3m2 )(m2 3l 2 )


B¦ , tan Q 
§ lm2 l2 m lm2 l2 m µ¶ (l 2 3m2 ) (m2 3l 2 )
2.2 Pair of Lines 69

41. Show that the equation of the diagonal not pass-


2 16l 2 m2 10l 2 m2 3(l 4 m4 ) ing through the origin of the parallelogram formed

2(l 2 m2 ) by the lines ax 2 2 hxy by2 2 gx 2 fy c  0
and ax 2 2 hxy by2  0 is 2 gx 2 fy c  0
3 l 4 m4 2l 2 m2 and the diagonal passing through the origin is

l 2 m2 y(hf bg )  x( gh af ).
3 (l 2 m 2 )
 Solution: Suppose the lines
l 2 m2
 3 S y ax 2 2 hxy by2 2 gx 2 fy c  0 (2.67)

That is, the angle between the sides represented by the and the lines
equation of the sides provided in Eq. (2.66) is 60o. Also, ax 2 2 hxy by2  0 (2.68)
the combined equation of the pair of angle bisectors of
equation of the sides passing through origin given in Eq. intersect in A(x1, y1) and B(x2, y2). Hence, both A and B
(2.66) is satisfy Eqs. (2.67) and (2.68) and hence they satisfy their
difference 2gx 2fy c  0 which is first-degree equation
4lm( x 2 y2 )  [l 2 3m2 (m2 3l 2 )]xy in x and y. Hence, 2gx 2fy c  0 represents a straight
(by Theorem 2.30) line passing through points A and B. Thus, AB is the
2 2 2
 lm (x y )  (l m ) xy2 diagonal represented by 2gx 2fy c  0. Since
 (lx my) (mx ly)  0 ¥ hf bg gh af ´
(0, 0) and ¦§ , µ
The angle bisectors of the angle at the vertex origin are lx ab h2 ab h2 ¶
my  0 and mx ly  0 which, respectively, are parallel
are the ends of the diagonal passing through origin, its
and perpendicular to the base line lx my n  0. Hence,
equation is
the triangle formed by the lines is equilateral. Suppose a
is the length of the sides and p is the length of the altitude ¥ gh af ´
from origin onto the base lx my n  0 so that y¦ x
§ hf bg µ¶
n  y  hf bg   gh af x
p (by Theorem 2.14)
2 2
l m
42. Show that the area of parallelogram formed by the lines
and
3
 sin 60o 
p ax 2 2 hxy by2 2 gx 2 fy c  0 and ax2 2hxy
2 a by2  0 is
2p 2 n c
a 
3 3 2 2
l m 2 h2 ab
Therefore, area of the triangle is
Solution: Suppose the sides of the parallelogram are
3 2 3 4n2 (by Theorem 2. 32)
a 
4 4 3(l 2 m2 )
l1x m1y n1  0
n2
 l2x m2y n2  0
3 (l 2 m 2 )
l1x m1y  0
and l2x m2y  0
y
Therefore, l1l2  a, l1m2 l2m1  2h, m1m2  b, l1n2 l2n1 
2g, m1n2 m2n1  2f and n1n2  c. Now, by Problem 2, the
lx

area of the parallelogram is


+
m
y+

(n1 0)(n2 0) n1 n2
n


=
0

l1 m2 l2 m1 (l1 m2 l2 m1 )2 4l1l2 m1 m2
O x
c c
 
4 h2 4ab 2 h2 ab
FIGURE 2.49
70 Chapter 2 Straight Line and Pair of Lines

43. Show that the triangle formed by the pair of lines x2


g 2 ac
4xy y2  0 and the line x y 3  0 is an equilat- 2
eral triangle and find its area. a(a b)

Solution: The equation of the lines x2 4xy y2  0 can If we use n1 n2  2 f /m2 , then the distance between the
be written as parallel lines is

(x y)2 3(x y)2  0 f 2 bc


2
Hence, by Problem 40, the pair of lines x2 4xy y2  0 b a b
and the line x y 3  0 form an equilateral triangle
whose area is 45. Show that the pair of lines a 2 x 2 2 h(a b) xy
b2y2 = 0 are equally inclined to the pair of lines
n2 3 3 ax 2 2 hxy by2  0.
 sq. units
3 (l 2 m 2 ) 2 FHHE FHHE FHHE FHHE
Solution: Two pairs of lines ( PA, PB ) and ( PC , PD )
44. If S y ax 2 2 hxy by2 2 gx 2 fy c  0 represents are said to be equally inclined to each other if both pairs
a pair of lines, then show that h2  ab, af 2  bg2 and have the same angle bisectors at point P. Now, for the
the distance between these lines is given pairs of lines, origin is the common point. By Theo-
rem 2.30, the equation of the pair of angle bisectors of
g 2 ac
2 the lines a 2 x 2 2 h(a b) xy b2 y2  0 is
a(a b)
h(a b)( x 2 y2 )  (a 2 b2 ) xy
which is also equal to
 h( x 2 y2 )  (a b) xy
f 2 bc
2 which also represents the pair of angle bisectors of the
b(a b)
line ax 2 2 hxy by2  0. Hence, both pairs are equally
inclined to each other.
Solution: Suppose the pair of parallel lines are lx my
n1  0 and lx my n2  0. Therefore,
46. Find the equation of the pair of lines passing through
l 2  a, 2lm  2h, m2  b origin which are at a distance d units from a point
(x1, y1) x (0, 0).
l(n1 n2)  2g, m(n1 n2)  2f, c  n1n2
Solution: See Fig. 2.50. Let P be (x1, y1) and y  mx the
Now, h2  l 2 m2  ab line whose distance from P is equal to d. That is,

¨ m2 (n1 n2 )2 · y1 mx1
and af 2  l 2 © d (by Theorem 2.14)
¸ 1 m2
ª 4 ¹
2
¨ l(n n2 )2 ·  ( y1 mx1 )2  d 2(1 m2 )
 m2 © 1 ¸  bg
2
2
ª 2 ¹ ¥ y ´ ¥ y2 ´ ¥ y´
 ¦ y1 x1 µ  d 2 ¦ 1 2 µ ¦§& m  µ¶
§ x ¶ § x ¶ x
Also, the distance between the two parallel lines is (by
Theorem 2.15)  ( xy1 x1 y)2  d 2( x 2 y2 )

n1 n2 (n1 n2 )2 4 n1 n2 y

2 2 2 2
l n l m
N d P(x1, y1)
4g 2
4c d
l2

M
l 2 m2
O x

g 2 ac
2
a(a b)
FIGURE 2.50
2.2 Pair of Lines 71

47. If the equation x2 5xy 4y2 3x 2y c  0 repre- and y(hf bg)  x(gh af)
sents a pair of lines, then find the value of c and also
The parallelogram is a rhombus if the diagonals are at
the angle between the lines.
right angles (see Fig. 2.51). That is, if the product of their
slopes is equal to 1:
Solution: By Theorem 2.31, we have
g ¥ gh af ´
a h g  1
2 2 2
f ¦§ hf bg µ¶
$  abc 2 fgh af bg ch  h b f 0
g f c  g 2 h afg  f 2 h bfg

This implies  h( f 2 g 2 ) fg(a b)  0

5 3
1 y
2 2
5
4 1 0 90°
2
3
1 c
2
2 5 3 O x
 5 8 2 0
3 2 2c
FIGURE 2.51
 2(16c 4) 5(10c 6) 3(10 24)  0
 18c 20  0 49. Show that the four lines given by the equations 2x2
3xy 2y2  0 and 2x2 3xy 2y2 3x y 1  0 form
10 a square.
c
9
If @ is the angle between the lines, then Solution: We know that the lines represented by the
equations are
a b 2x y  0
cos A  (by Theorem 2.28)
2 2
(a b) 4 h x 2y  0
2x y 1  0
1 4 5 and 2x 2y 1  0
 
¥ 25 ´ 9 25 Solving 2x y  0 and x 2y 1  0, we have
(1 4)2 4 ¦ µ
§ 4¶
3 1
x  and y 
Hence 5 5
¥ 5 ´ so that the slope of diagonal through origin is 1/3. Since
A  Cos 1 ¦
§ 34 µ¶ the other diagonal is 3x y 1  0, the diagonals are at
right angles. Therefore, the parallelogram is a square.
48. Show that the straight lines ax 2 2 hxy by2  0 and
the straight lines ax 2 2 hxy by2 2 gx 2 fy c  0 50. Find the centroid of the triangle formed by the lines
form a rhombus if (a b)fg h(f 2 g2)  0. 12x2 20xy 7y2  0 and 2x 3y 4  0.

Solution: By Theorem 2.32, the given pairs of lines Solution: We have


form a parallelogram. Also, by Problem 41, the diagonals 12 x 2 20 xy 7 y2 y (2 x y)(6 x 7 y)
of the parallelogram are
Therefore, the sides of the triangle are
2gx 2fy c  0
2x y  0 (2.69)
72 Chapter 2 Straight Line and Pair of Lines

6x 7y  0 (2.70) l1 n2 l2 n1  2 g
2 x 3 y  4 (2.71) m1 n2 m2 n1  2 f
Solving Eqs. (2.69) and (2.71), we get (1, 2) as the n1 n2  c
vertex. Solving Eqs. (2.70) and (2.71), we obtain (7, 6) as
another vertex.By hypotheses, (0, 0) is the third vertex. By hypothesis,
Hence, the centroid of the triangle is
n1 n2
¥ 0 1 7 0 2 6´ ¥ 8 8´ 
¦§ , µ ¦ , µ l12 m12 l22 m22
3 3 ¶ § 3 3¶
 n12 (l22 m22 )  n22 (l12 m12 )
2 2
51. If S y ax 2 hxy by 2 gx 2 fy c  0 represents
a pair of lines equidistant from the origin, then show (squaring and crross- multiplying)

that f 4 g 4  c(bf 2 ag 2 ).  l12 n22 l22 m12  m22 n12 m12 n22
 (l1n2 l2 n1 )(l1 n2 l2 n1 )  (m2 n1 m1 n2 )(m2 n1 m1 n2 )
Solution: Suppose the lines are l1x m1y n1  0 and
l2x m2y n2  0. Therefore  2 g(l1 n2 l2 n1 )  2 f (m2 n1 m1 n2 )
l1l2  a  g 2(4 g 2 4ac)  f 2(4 f 2 4bc)
l1 m2 l2 m1  2 h  f 4 g 4  bcf 2 cag 2  c(bf 2 ag 2 )
m1 m2  b

WORKED-OUT PROBLEMS
Single Correct Choice Type Questions
1. If the line 3ax 5y a 2  0 passes through the point 1 4c
( 1, 4), then value of a is – c  24 c2  36  c p6
2 3

(A) 9 (B) 7 (C) −9 (D) −7 Since y-intercept is positive, the value c is 6 and the equa-
tion of the line is 3x 4y 24  0.
Solution: Since the line passes through ( 1, 4), we
Answer: (B)
have
3a( 1) 5(4) a 2  0 3. A non-horizontal line passing through the point (4, 2)
and whose distance from the origin is 2 units is
 2a 18  0
(A) 3x 4y 10  0 (B) x y 2 0
Hence, a  9 and the line is 27x 5y 7  0.
Answer: (A) (C) 4x 3y 10  0 (D) 2x 3y 2  0
Solution: Let the slope of the line be m. Now the
2. A line has slope 3 / 4, positive y-intercept and forms equation of the line, by Theorem 2.2, is
a triangle of area 24 sq. units with coordinate axes.
Then, the equation of the line is y 2  m( x 4)

(A) 3x 4y 24  0 (B) 3x 4y 24  0 Therefore, by Theorem 2.14,


(C) 3x 4y 25  0 (D) 3x 4y 25  0 m(0 4) 0 2
2
Solution: Let the line be m2 1
¥ 3´  (2 m 1)2  m2 1
y  ¦ µ x c
§ 4¶
 3m 2 4 m  0
so that the intercepts on the x and y axes, respectively, are 4
4c/3 and c. Therefore, the area of the triangle (by Quick m  0,
3
Look 4) is
Worked-Out Problems 73

When m  0, the line is 7. Let A(h, k), B(1, 1) and C(2, 1) be the vertices of a

Worked-Out Problems
right-angled triangle with AC as its hypotenuse. If the
y 20
area of the triangle is 1, then the set of values of k is
which is horizontal. When m  4/3, the line is given by
4 (A) {0, 2} (B) { 1, 3} (C) { 3, 2} (D) {1, 3}
y 2  (x 4)
3
4x 3y 10 0 Solution: See Fig. 2.52. Since BC is horizontal line (y
 1) and AB is perpendicular to BC, it follows that AB
Answer: (C) is a vertical line passing through B(1, 1). Hence, h  1.
Therefore
4. The positive value of k such that the distance of the
line 8x 15y k  0 from the point (2, 3) is 5 units is 1  Area of $ABC  ( AB)( BC )
1
(A) 12 (B) 6 (C) 8 (D) 24  k 1 (1)
2
Solution: By Theorem 2.14, we have
So, k 1 p2 or k  –1, 3.
8(2) 15(3) k
5 A(h, k)
8 2 152
 61 k p(5 s 17) p85
k  24 or 146
Since k  0, its value is 24.
Answer: (D)

5. Equation of the line passing through the point (2, 3)


B(1, 1) C(2, 1)
and parallel to the line joining the points (4, 1) and
( 2, 2) is FIGURE 2.52

(A) x 6y 12  0 (B) x 6y 12  0 Answer: (B)


(C) x 6y 16  0 (D) x 6y 16  0 8. If the perpendicular bisector of the line segment join-
Solution: Slope of the line joining the points (4, 1) and ing P(1, 4) and Q(@, 3) has y-intercept 4, then a pos-
(−2, 2) is sible value of k is
2 1 ¥ 1´ (A) 2 (B) 4 (C) 1 (D) 2
¦  µ¶
2 4 § 6 Solution: Since every point (x, y) on the perpendicu-
Hence, by Theorem 2.2, the equation of the given line is lar bisector of PQ is equidistant from both P and Q we
have
1
y 3  ( x 2) ( x 1)2 ( y 4)2  ( x A )2 ( y 3)2
6
 2 x 8 y 17  2A x 6 y A 2 9
x 6y 16  0
Answer: (D) Therefore, equation of the perpendicular bisector of PQ is
2(A 1) x 8  2 y A 2
6. Equation of the line passing through the point ( 2, 3)
and perpendicular to the line 2x 3y 6  0 is Hence

(A) 3x 2y  0 (B) 2x 3y 0 8 A2


4  y-intercept 
(C) 3x 2y  0 (D) 2x 3y  0 2
Solution: By Quick Look 7, part (3), the equation of Therefore, @ 2  16 or @p4.
the line is Answer: (B)
−3(x 2) −2(y − 3)  0
9. The lines
 3x 2y  0
K(K2 1)x y b  0
Answer: (C)
74 Chapter 2 Straight Line and Pair of Lines

and (K2 1)2x (K2 1)y 2b  0 Solving the equations which are taken pairwise, the ver-
tices of the triangle are
are perpendicular to a common line for
A( p, 0), B( q, 0) and C[pq, (p 1)(q 1)]
(A) exactly one value of K
(B) exactly two values of K Since the side AB is along the x-axis, altitude CN, drawn
from C to the side, is a vertical line so that the equation
(C) more than two values of K
of CN is
(D) no value of K
x  pq (2.75)
Solution: By hypothesis, the given lines are parallel.
Hence Equation of altitude AM is
( pq q)
(L 2 1)2 y 0 ( x p)
L (L 2 1)  ( p 1)(q 1)
L2 1
q (2.76)
Therefore K 1. y ( x p)
q 1
Answer: (A)
Solving Eqs. (2.75) and (2.76), we get the orthocentre as
10. The number of integral values of m for which the q
x-coordinate of the point of intersection of the lines y ( pq p)  pq
q 1
3x 4y  9 and y mx 1 is also an integer is
If (x, y) is the orthocentre, then we have x pq and y  pq
(A) 2 (B) 0 (C) 4 (D) 1
so that the locus of the orthocentre is y  x or x y  0
(IIT-JEE 2001)
C (pq, (p + 1)(q + 1))
Solution: Solving the given equations, the x-coordinate
of their point of intersection is
5
 5, 1
3 4m M
90°
when m  1 and 2, respectively. That is for m  1 and
2, the x-coordinate is also an integer. Hence, the number
of integral values is 2.
Answer: (A) 90°
A(−p, 0) N B(−q, 0)
11. The locus of the orthocentre of the triangle formed
FIGURE 2.53
by the lines y  0 (i.e., x-axis) and the lines
Answer: (A)
(1 p)x py p(1 p)  0
(q 1)x qy q(1 q)  0 12. Two adjacent sides of a parallelogram are 4x 5y  0
and 7x 2y  0. If the equation to one diagonal is
where pxq, is
11x 7y 9  0, then the equation of the other di-
(A) the line x y  0 agonal is
(B) the line x y  0 (A) x y  0 (B) x y  0
(C) the curve y2  4pq(p q)( pq)q (C) 2x 11y 9  0 (D) x y 9  0

(D) the curve x2 y2 Solution: See Fig. 2.54. We have


1
p( p q) q( p q) 4x 5y  0 (2.77)
(IIT-JEE 2009) 7x 2y  0 (2.78)
Solution: The given lines are 11x 7y  9 (2.79)
y0 (2.72) Solving Eqs. (2.77) and (2.79), we get
(1 p) x py p(1 p)  0 (2.73) ¥ 5 4´
A¦ , µ
(1 q) x qy q(1 q)  0 (2.74) § 3 3¶
Worked-Out Problems 75

Also solving Eqs. (2.78) and (2.79), we get Also


¥ 2
C  ¦ ,
7´ OB2  OC 2
§ 3 µ
3¶ 9x2
 x2  h2
Now 16
25x 2
¥ 5 4´ ¥ 2
A¦ , µ , O(0, 0) and C ¦ ,
7´   h2
§ 3 3¶ § 3 µ 16

25 400 h2
are three consecutive vertices of a parallelogram. The  –  h2
16 (20 3h)2
fourth vertex B is
 252  (20 3h)2
¥5 2 4 7´  20 3h  p25
¦§ 0 , 0 µ¶  (1, 1)
3 3 3 3
5
Hence, the other diagonal is y x.  h  , 15
3
−2 , 7
C
3 3 When h  5 / 3
B
¥ 5´
20 ¦ µ
20 h § 3¶ 100 4
x   
0

11
y=

20 3h 5
¥ ´ 75 3
x+

20 3 ¦ µ
+2

§ 3¶
7y
7x

=9

When h  15
20( 15) 300
5 , −4 x   12
O 4x + 5y = 0
A
3 3 20 3( 15) 25

FIGURE 2.54
Hence, the coordinates of B and C, respectively, are
4 5
Answer: (B) either ¥¦ , 1´µ and ¥¦ , 0´µ or (12, 9) and ( 15, 0).
§3 ¶ §3 ¶
13. The vertices B and C of $ABC lie on the lines 4y Two cases arise:
3x  0 and y  0, respectively, and the side BC passes 4 ¥5 ´
through the point P(0, 5). If ABOC is a rhombus Case 1: B  ¥¦ , 1´µ , P  (0, 5) and C  ¦ , 0µ
§3 ¶ §3 ¶
where O is the origin and P is an internal point to
the rhombus, then the vertex A is P is an inside point and B, P and C are collinear.
This implies
(A) (3, 9) (B) (9, 3)
(C) (3, 9) (D) ( 3, 9) BP PC BC
16 25
Solution: See Fig. 2.55. Let B  ( x, 3 x /4) and C  (h, 0). BP PC  16 25
9 9
Since B, P and C are collinear, from Quick Look 2,
Chapter 1, we have 4 10 5 10
  3 10
3 3
x 0 h
1 10
3x and BC  1 
5 0 0 9 3
4
1 1 1 ¥4 ´ ¥5 ´
Therefore, B x ¦ , 1µ and C x ¦ , 0µ
§3 ¶ §3 ¶
¥ 3x ´
 5x h ¦ 5µ  0
§ 4 ¶ Case 2: B  (12, 9), P  (0, 5) and C  ( 15, 0). Now
 20 x h(3 x 20)  0 BP  12 2 4 2  160  4 10
 x(20 3h)  20h PC  225 25  250  5 10
20 h
x (2.80) BC  27 2 9 2  729 81  810  9 10
20 3h
76 Chapter 2 Straight Line and Pair of Lines

Therefore, Therefore, from R3 R1sinP R2cosP, we have

BP PC  9 10  BC x1 y1 1 x1 y1 1
Hence x2 y2 1  x2 y2 1
x3 y3 1 0 0 1 (sin Q cos Q )
A (12 0 15, 9 0 0)  ( 3, 9)
 (1 sin Q cos Q )( x1 y2 x2 y1 )  0
O
This implies that x1y2 x2y1 0 or sinP cosP  1. Since
0 P  P / 4, sinP cosP x 1. Therefore, x1y2 x2y1  0.
This implies that

sin(B A )sin B cos B cos(B A )  0


C
P(0, 5)
B  cos(B A B )  0

Hence, 2A @  P /2 which is impossible because 0 @


and A  P /4. Therefore
2B A x P / 2

Thus x1y2 x2y1 x 0. Hence, the points P, Q and R are


A
non-collinear.
FIGURE 2.55 Answer: (D)
Answer: (D)
15. Let O(0, 0), P(3, 4) and Q(6, 0) be the vertices of a
14. Consider the points triangle OPQ. A point R is inside the triangle such
that the triangles OPR, PQR and OQR are of equal
P  [ sin(A @), cosA ]
areas. Then, the coordinates of R are
Q  [cos(A @ ), sinA ]
¥4 ´ ¥ 2´
and R  [cos(A @ P ), sin(A P )] (A) ¦ , 3µ (B) ¦ 3, µ
§3 ¶ § 3¶
where 0 @, A, P  P /4. Then 4´
¥ ¥2 4´
(C) ¦ 3, µ (D) ¦ , µ
(A) P lies on the segment RQ § 3¶ §3 3¶
(B) Q lies on the segment PR (IIT-JEE 2007)
(C) R lies in the segment QP Solution: Let $ABC be any triangle. P is an inside
(D) P, Q and R are non-collinear point such that the triangles PBC, PCA and PAB are
(IIT-JEE 2008) of equal areas if and only if P is the centroid of ΔABC.
Hence, R must be the centroid of ΔOPQ and so
Solution: It is known that three points (x1, y1), (x2, y2)
and (x3, y3) are collinear if and only if ¥ 4´
R  ¦ 3, µ
x1 y1 1 § 3¶
x2 y2 1  0 Answer: (C)
x3 y3 1
16. If none of two lines among the following three lines
Take P  (x1, y1), Q  (x2, y2) and R  (x3, y3). Therefore,
ax by (a b)  0
x3  cos(A @ P )  cosP cos(A @) sinP sin(A @) bx (a b)y a  0
x2cosP x1sinP and (a b) x ay b  0

and are parallel then the lines


y3  sin(A P)  sin A cosP sinP cosA (A) are concurrent
y2cosP y1sinP (B) form a right-angled triangle
Worked-Out Problems 77

(C) form an equilateral triangle 19. Sum of the slopes of the lines which make 45o with
(D) form a triangle with circumcentre (a b, a b) the line 3x y 5  0 is
3 3 1
Solution: By adding C1 and C2 to C3, we get (A) (B) (C) 2 (D)
2 2 2
a b (a b) a b 0 Solution: Let m be the slope of the required line.
b (a b) a b (a b) 0  0 Therefore
a b a b a b a 0
m 3
1  tan 45o 
Hence by Theorem 2.22, the lines are concurrent. 1 m(3)
Answer: (A)  3m 1 p(m 3)

17. If the line 2x 3y 4 K(6x y 12)  0 is perpendicu- Hence, m  2, 1/ 2 so that their sum is
lar to the line 7x 5y 4  0, then the value of K is
1 3
29 25 25 29 2 
(A) (B) (C) (D) 2 2
37 27 27 37
Answer: (B)
Solution: Slope of the given line is
(2 6 L ) 20. Let PS be the median of the triangle with vertices

3 L P(2, 2), Q(6, 1) and R(7, 3). The equation of the line
passing through (1, 1) and parallel to PS is
This line is perpendicular to the line 7x 5y 4  0. This
implies that (A) 2x 9y 7  0 (B) 2x 9y 11  0
(C) 2x 9y 11  0 (D) 2x 9y 7  0
(2 6 L ) ¥ 7 ´
s ¦ µ  1 (IIT-JEE 2000)
3 L § 5¶
Solution: See Fig. 2.57. P  (2, 2) and S  (13 / 2, 1).
 14 42 L  15 5L
Therefore, the slope of the median PS is
 37 L  29
2 1 2
29 
L 2 (13 / 2) 9
37
Answer: (D) Hence, the equation of the line through the point (1, 1)
and having slope 2 / 9 is
18. The points A(1, 3) and C(5, 1) are extremities of a di- 2
agonal of a rectangle ABCD. The other two vertices B y 1 ( x 1)
9
and D lie on the line y  2x c. Then, the value of c is
 2x 9y 7  0
(A) 2 (B) 4 (C) 4 (D) 2
Solution: See Fig. 2.56. Let M  (3, 2) be the intersec- P(2, 2)
tion of the diagonals. Since M(3, 2) lies on the diagonal
BD whose equation is given as y  2x c, we have
2  2(3) c
 c −4
D C(5, 1)

Q(6, −1) S
13 , 1 R(7, 3)
2
A(1, 3) B(h, k)
FIGURE 2.57
FIGURE 2.56 Answer: (D)
Answer: (C)
78 Chapter 2 Straight Line and Pair of Lines

21. The orthocentre of the triangle formed by the lines Thus, from Eq. (2.82), we have
xy  0 and x y  1 is
3  cotP
¥1 1´ ¥ 1 1´
(A) ¦ , µ (B) ¦ , µ ¥P ´
§2 2¶ § 3 3¶  tan ¦ Q µ
§2 ¶
¥ 1 1´  tan( D E) [by Eq.(2.83)]
(C) (0, 0) (D) ¦ , µ
§ 4 4¶
tan D tan E

(IIT-JEE 1995) 1 tan D tan E
Solution: See Fig. 2.58. By hypothesis, the sides of the ( AC / DC ) ( BC / EC )

triangle are y  0, x  0 and x y  1. Therefore, the trian- 1 ( AC / DC ) – ( BC / EC )
gle is a right-angled triangle at the origin. Now for a right- (2b / a) (2a / b)
angled triangle, the right-angled vertex is its orthocentre. 
Hence, origin is the orthocentre of the given triangle. (1 4)
2(a 2 b2 ) 2(60)2
y   (2.84)
3ab 3ab
Therefore, from Eq. (2.84), the area of $ABC is

1 60 2
ab   400
x+
2 9
1 y=
1 A

O 1 x

60
FIGURE 2.58 E
Answer: (C) G
q
22. In $ABC, C  90o and the side AB is of length 60
units. The equations of the medians AD and BE, re-
spectively, are y x 3 and y  2x 4. Then the area 90°
of $ABC (in square units) is B D C

(A) 200 (B) 300 (C) 400 (D) 500 FIGURE 2.59

Solution: See Fig. 2.59. By hypothesis, we have Answer: (C)

a2 b2  602 (2.81) 23. A straight line L with negative slope passes through
the point (8, 2) and cuts the positive coordinate
Let P be the acute angle between the medians AD and axes at points P and Q. Then the absolute mini-
BE. Therefore mum value of OP OQ as L varies (where O is the
2 1 1 origin) is
tan Q   (2.82)
1 2(1) 3 (A) 6 (B) 9 (C) 12 (D) 18
(IIT-JEE 2002)
From the quadrilateral DGEC, we have
Solution: Let the equation of the line L, by hypothesis,
DGE GDC 90o GEC  360o be
y 2 m(x 8)
Hence
P where m  0. Therefore
(O P) GDC GEC  2O
2 ¥ 2 ´
p  ¦ 8 , 0µ and Q  (0, 2 8m)
P § m ¶
 GDC GEC  P (2.83)
2
Worked-Out Problems 79

Now, 25. The equations of a pair of opposite sides of a paral-


lelogram are x2 5x 6  0 and y2 6y 5  0. The
¥ 2´ equations of its diagonals are
OP OQ  ¦ 8 µ (2 8 m) (& m  0)
§ m¶
(A) x 4y  13, y  4x 17
¥2 ´ 2
 10 ¦ 8 mµ r 10 2 s ( 8 m) (B) 4x y  13, 4y x 7
§m ¶ m
(C) 4x y  13, y  4x 7
 10 8 (& AM r GM )
(D) y 4x  13, y 4x  7
and equality occurs if and only if Solution: See Fig. 2.61. We have
2 1 x2 − 5x 6 y (x − 2)(x− 3)
 8m or m 
m 2
y2 − 6y 5 y (y − 1)(y − 5)
Hence, the absolute minimum of OP OQ is 12 6  18.
Therefore, the sides of the parallelogram are
Answer: (D)
x  2, x  3 and y  1, y  5
24. A straight line through the origin O meets the par- Therefore, the vertices of the rectangle are A(2, 1), B(3, 1),
allel lines 4x 2y  9 and 2x y 6  0 at points C(3, 5) and D(2, 5). Hence, the equation of AC is
P and Q, respectively. Then the point O divides the
segment PQ in the ratio 5 1
y 1 ( x 2)  4( x 2)
3 2
(A) 1:2 (B) 3:4 (C) 2:1 (D) 4:3
 4x y 7  0
Solution: See Fig. 2.60. The line 4x 2y  9 has positive
interception coordinates while the line 2x y 6  0 has Equation of BD is
negative interception on the axes. Hence, origin O lies in 5 1
between the axes. Suppose OM and ON, respectively, are y 1 ( x 3)  4( x 3)
2 3
drawn perpendicular to the given two lines. Observe O,
M and N are collinear (see Fig. 2.60). Now,  4x y  13
9 6
OM  and ON 
2 5 5
If any line through O meets the parallel lines in P and Q, D C(3, 5) y=5
then by pure geometry, we have
9 6
OP : OQ  OH : ON  :  3:4
2 5 5
y=1
A(2, 1) B
4x
+
2y
x=2

x=3

=
9
M
FIGURE 2.61
Answer: (C)
2x O
+
y= 26. The line x y 2  0 divides the segment joining the
−6
points A(3, 1) and B(8, 9) in the ratio
N
(A) 2:3 (B) 2:1 (C) 1:2 (D) 3:4
Solution: See Fig. 2.62. Suppose the line x − y − 2  0
meets the line joining A and B in P and AP:PB  K:1.
Therefore,
FIGURE 2.60
Answer: (B) ¥ 8 L 3 9 L 1´
P¦ ,
§ L 1 L 1 µ¶
80 Chapter 2 Straight Line and Pair of Lines

Since P lies on the line x y 2  0, we have diameter 4y − x − 7  0. If O is the centre of the circle,
then we know that
8L 3 9L 1
20
L 1 L 1 AD  2(OM) (2.85)
FHHHE FHHHE
 3L 3 1 2  0 Since AB is a horizontal
FHHHEline, OM is a vertical line and
2 hence the equation of OM is x  1. Therefore, the line OM
L and the diameter intersect at O and we have O  (1, 2).
3
Now
Hence, AP:PB  2:3.
AD  2(OM)  2(2)  4
By hypothesis, AB is 8. Thus, the area of the rectangle
ABCD  8 s 4  32.

l 1
A(3, −1) P B(8, 9)
D C

x−y−2=0

0
7=
− x−
FIGURE 2.62 4y O(1, 2)

Answer: (A) x=1

Note: Using Theorem 2.17, the ratio AP:PB is A(−3, 4)


L11:L22where M(1, 4) B(5, 4)

L yx − y − 2  0, A  (3, 1) and B  (8, 9)


Now,
FIGURE 2.63
L11 3 ( 1) 2  Answer: (B)
and L22 8 9 2  3
29. The vertices of a triangle are A( 1, 7), B(5, 1) and
so that C(1, 4). The equation of the bisector of the angle
AP:PB  L11:L22 ABC is
AP:PB  ( 2):( 3)  2:3 (A) 2y x 7 (B) 7y x 2
(C) 3y x 7 (D) 7y x 3
27. The ratio in which the line 3x y 6  0 divides the
line joining the points A(3, 4) and B( 2, 1) is Solution: We have
(A) 2:11 (B) 9:2 (C) 11:1 (D) 1:12
AB  (5 1)2 (1 7)2  100  10
Solution: L y 3x − y 6  0, A  (3, 4), B  (−2, 1)
L11 3(3) 4 6  11, L22 3( 2) 1 6  1 and BC  (5 1)2 (1 4)2  25  5

Hence, by Theorem 2.17, the ratio AP:PB is Suppose the bisector of ABC meets the side AC at D
L11:L22  ( 11):( 1)  11:1 (see Fig. 2.64). Therefore, CD:DA  BC:BA  5:10  1:2.
Hence
Answer: (C)
¥ 2 1 7 8 ´ ¥ 1 1 ´
28. One of the diameters of the circle circumscribing the D¦ , µ ¦ , µ
§ 3 3 ¶ § 3 3¶
rectangle ABCD is 4y  x 7. If A and B are the
points ( 3, 4) and (5, 4), respectively, then the area of
Since B  (5, 1) and D  (1/ 3, 1/ 3), the equation of the
the rectangle in square units is
bisector BD of ABC is
(A) 22 (B) 32 (C) 42 (D) 26 1 (1 / 3)
y 1 ( x 5)
Solution: See Fig. 2.63. Let M be the midpoint of AB 5 (1 / 3)
so that M  (1, 4). Clearly, A and B do not lie on the
Worked-Out Problems 81

2 1 of the acute angle between L1 and L2 intersects the


 y 1 ( x 5)  ( x 5) line L3 at R. Consider the following two statements:
14 7
Hence, 7y x 2. 1. S1: The ratio PR:RQ  2 2: 5 .
2. S2: In any triangle, bisector of an angle divides
A(−1, −7) the triangle into two similar triangles.
Then
(A) both S1 and S2 are true
(B) both S1 and S2 are false
10 2 (C) S1 is true while S2 is false
(D) S2 is true while S1 is before
D (IIT-JEE 2007)

1 Solution: Solving L1 0 and L3  0, we get P  (−2, −2).


Solving L2 0 and L3  0, we get that Q  (1, −2). There-
fore, OP  2 2 and OQ  5 where O is the origin. In
B(5, 1) 5 C(1, 4)
any triangle, the internal angle bisector of an angle di-
FIGURE 2.64 vides the opposite side internally in the ratio of the other
Answer: (B) two sides. Hence
PR:RQ  OP :OQ  2 2 : 5
30. Let the algebraic sum of the perpendicular distances
from the points (2, 0), (0, 2) and (1, 1) onto a variable Hence S1 is true. S2 is false because in a triangle whose
line be zero. Then, the line passes through a fixed angles are 90o, 60o and 30o, the bisector of 90o cannot
point whose coordinates are divide the triangle into two similar triangles.
Answer: (C)
(A) (1, 1) (B) ( 1, 1)
¥2 1´ ¥1 2´ 32. If the lines 2x 3y k  0, 3x 4y 13  0 and
(C) ¦ , µ (D) ¦ , µ 8x 11y 33  0 are concurrent, then the value of k is
§3 3¶ §3 3¶
(A) 7 (B) 6 (C) 7 (D) 6
Solution: Algebraic distance of a point (x1, y1) from a
line ax by c  0 is Solution: By Theorem 2.22, we have
ax1 by1 c 2 3 k
a 2 b2 3 4 13  0
Suppose the variable line is ax by c  0. Therefore, by 8 11 33
hypothesis,  2(132 143) 3( 99 104) k( 33 32)  0
2a 0 c 0 2b c a b c  22 15 k  0
0
2 2 2 2 2 2
a b a b a b  k  7
 3a 3b 3c  0 Direct Method: Solving 3x 4y 13  0 and 8x 11y
 a b c  0 33  0, we obtain x  11 and y  5 so that (11, 5) is the
intersection of these lines. The third line 2x 3y k  0
Hence, the line ax by c  0 passes through the point
also passes through (11, 5). This implies (11.5) satisfying
(1, 1).
the equation of the third line. So
Answer: (A)
2(11) 3(5) k  0 k  7
Note: If the algebraic distances of the points (xr, yr) Answer: (C)
(r  1, 2, ..., n) from a straight line l are zero, then the line
l passes through the point ( x , y ) where 33. If the lines ax by c  0, bx cy a  0 and
cx ay b  0 are concurrent and a b c x 0, then
x1 x2 ––– xn y y2 ––– yn
x and y  1 the quadratic equation ax2 bx c  0 has
n n
(A) equal roots (B) rational roots
31. Lines L1: y x  0 and L2: 2x y  0 intersect the line (C) irrational roots (D) no real roots
L3: y 2  0 at P and Q, respectively. The bisector
82 Chapter 2 Straight Line and Pair of Lines

Solution: By Theorem 2.22, we have 35. The straight line 2x 3y 1  0 bisects the angle be-
tween two straight lines of which one line is 3x 2y
a b c
4  0. Then, the equation of the other line is
b c a 0
(A) 9x 26y  48 (B) 9x 46y  28
c a b
(C) 9x 26y  28 (D) 3x 26y  48
 3abc a3 b3 c3  0
Solution: See Fig. 2.65. Let L1 y 3x 2y 4  0, L y 2x
 (a b c)(a2 b2 c2 ab bc ca)  0 3y 1  0 and L2 y 0 be the required lines. Since the line
The condition a b c x 0 implies a2 b2 c2 ab bc ca L2  0 passes through the intersection of the lines L1 0
 0 which in turn implies that and L  0, L2  0 is of the form (by Theorem 2.20)
1 L2 y (3x 2y 4) K(2x 2y 1)  0
[(a b)2 (b c)2 (c a)2]  0 a b c
2 Suppose (@, A ) (x the intersection of L1  0 and L  0) is
Hence, the quadratic equation is x2 x 1  0 whose a point on the line L  0 so that
roots are
2@ 3A 1  0 (2.85)
1 p i 3
But L  0 is an angular bisection of L1  0 and L2  0
2 which implies that
Answer: (D)
(3A 2B 4) L (2A 3B 1) 3A 2B 1
34. The equation of the line passing through the inter- 
2 2
section of the lines 2x 5y 1  0 and 3x 2y 8  0, (2 L 3) (3L 2) 32 2 2
and having equal non-zero intercepts on the axes, is  13L 2 24 L 13  13 [from Eq. (2.85)]
(A) x y  3 (B) x y  2 24
 L  0 or
(C) x y  1 (D) x y  3 13
Solution: By Theorem 2.20, any line passing through Therefore, K  0 gives the line L1  0 and K  24 / 13 gives
the intersection of the given lines is of the form
the line
(2x 5y 1) K(3x 2y 8)  0
24
On simplification we get L2 y (3x 2y 4) (2x 3y 1)  0
13
(2 3K)x (2K )y 1 8K 0
y 9x 46y 28  0
whose intercepts on the axes are
Therefore, the required line is L2 y 9x 46y 28  0.
8L 1 8L 1
,
2 3L 2 L 5
Therefore,
8L 1 8L 1

0

2 3L 2 L 5

1
y
2 0

 (8K 1)(2 3K 2K 5)  0


3
2x

1
L

L

K  or 7 0
8 
4
2y
If K  1 / 8, then the line is x 2y  0 which is not the x
3
L1
case. Hence x  7 and the required line is 19x 19y
q
57  0 or x y 3  0. q

Direct Method: The point of intersection of the given


lines is (2, 1). Any line having equal intercepts on the FIGURE 2.65
coordinates axes is of the form x y a. This line passes Answer: (B)
through the point (2, 1) which implies that a  3. Hence
the equation of the required line is x y  3. 36. A variable straight line passes through the point
Answer: (A) P(@, A ) and cuts the axes of coordinates in points
Worked-Out Problems 83

A and B, respectively. If the parallelogram OACB is (n n)(n n)


completed, then the locus of the vertex C (O is the 2
origin) is lm lm

x y B A  n2  lm
(A) 1 (B) 1
B A x y Answer: (B)
A B
(C) 1 (D) @ x A y  (@ A )xy 38. Consider the family of lines 2x y 4 K(x 2y
x y
3)  0 where K is a parameter. Then, the number of
Solution: See Fig. 2.66. Let the line AB be lines belonging to this family whose distance from
the point (2, 3) is 10 is
x y
1
a b (A) 4 (B) 2 (C) 1 (D) 0

where A  (a, 0) and B  (0, b). This line passes through Solution: The given family of lines can be written as
P(@, A ) which implies that (2 K )x (1 2K)y (4 3K)  0
A B By hypothesis, the distance of this line from (2, 3) is
1 (2.86)
a b
Let C(h, k) be the fourth vertex. Therefore, h a and k b. (2 L )(2) (1 2 L )( 3) (4 3L )
 10
Hence, from Eq. (2.86), we have (2 L )2 (1 2 L )2
A B
1  (5K 5)2  10(5K2 5)
h k
(K 1)2  2(K2 1)
Therefore, the locus of C is
K2 2K 1  0
A B
1 K  1
x y
Hence, the required line is 3x y 1  0.
y Answer: (C)

39. The point (2, 1) is shifted through a distance 3 2


B C(h, k) units measured parallel to the line x y  1 in the
decreasing direction of ordinates to reach a point B.
The image of the point B in the line x y  1 is
P(a, b ) (A) (0, 0) (B) (0, 1)
(C) ( 3, 2) (D) (3, 4)
Solution: See Fig. 2.67. Let A  (2, 1). By hypothesis, A
is translated along the line
O A x
y − 1 −1(x − 2)
 x y  3 (2.87)
FIGURE 2.66 Therefore abscissa of B is given by the equation
Answer: (C)
(2 x)2 (2 x)2  18
37. If the area of the rhombus enclosed by the lines lx  2 x p3
my n  0, lx my n  0, lx my n  0 and x  1, 5
lx my n  0 is 2 sq. unit, then
Hence, the point B  ( 1, 4) or (5, 2). However, A(2,
(A) m2 ln (B) n2 lm 1) is translated in the decreasing sense of ordinates.
(C) m ln (D) n lm Thus, B  (5, 2). Hence, the image of B(5, 2) in the
line x 5  1 is Ba(3, 4) because the slope of BBa is 1
Solution: From Problem 2 of the section ‘Subjective and the midpoint of BBa is (4, 3) which lies on the line
Problems’, the area of the rhombus is x y 1  0.
84 Chapter 2 Straight Line and Pair of Lines

y lies on the altitude through A(5, 12) and parallel to the


line y x, whose equation is
y − 12  1(x − 5)
A(2, 1)
(0, 1)
x x−y 70
+
y Answer: (C)
=
1
3p 3p
4 4 42. Consider the following two families of lines repre-
O (1, 0) sented by the equations (x y 6) K(2x y 3)  0
B(5, −2)
and (x 2y 4) L(3x 2y 4)  0. If these families
of lines are at right angles to each other, then their
points of intersection lie on the curve
FIGURE 2.67
(A) x2 y2 3x 4y 3  0
Answer: (D) (B) x2 y2 5x 2y 3  0
40. If a, b and c are real such that a2 9b2 4c2 6ab 
(C) x2 y2 3x 4y 3  0
0, then the line ax by c  0 always passes through (D) x2 y2 3x 4y 3  0
the point with negative ordinate Solution: By Theorem 2.20, the equation (x − y − 6)
¥ 1 3´ ¥ 1 3´ K(2x y − 3)  0 represents the family of lines passing
(A) ¦ , µ (B) ¦ , µ through the intersection of the lines x − y− 6  0 and 2x
§ 2 2¶ § 2 2¶
y − 6  0 which is (3, −3). Similarly, the second equation
¥ 1 3´ ¥ 1 3´ represents a family of concurrent lines which is concur-
(C) ¦ , µ (D) ¦ , µ
§ 2 2¶ § 2 2¶ rent at the point of intersection of the lines x 2y −4 
0 and 3x − 2y − 4  0 which is (2, 1). Suppose the line
Solution: We have through the point (3, −3) is
a 2 9b2 4 c2 6 ab  0 y 3 m(x 3) (2.88)
2 2
 (a 3b) 4c  0 Then the line through the point (2, 1) and perpendicular
 (a 3b 2c)(a 3b 2c)  0 to the line given in Eq. (2.88) is
1
¥a 3 ´ ¥ a 3b ´ y 1 ( x 2) (2.89)
 ¦ b cµ ¦ cµ  0 m
§2 2 ¶§ 2 2 ¶
From Eqs. (2.88) and (2.89), we have
Therefore, the line ax by c  0 passes through the
points y 3 ( x 2)
m
x 3 y 1
¥ 1 3´ ¥ 1 3´
¦§ , µ¶ and ¦§ , µ  ( x 3)( x 2) ( y 1)( y 3)  0
2 2 2 2¶
Since the ordinate is negative, the point is  x 2 y2 5 x 2 y 3  0
Answer: (B)
¥ 1 3´
¦§ , µ¶
2 2
Note: In Chapter 3, we will see that the equation
Answer: (C) x2 y2 3x 4y 3  0 represents a circle describable
on the segment joining (3, 3) and (2, 1) as ends of a
41. Let A(5, 12), B( 13cosP, 13sinP ) and C(13sinP, diameter.
13cosP ) where P is real be the vertices of $ABC.
Then, the orthocentre of $ABC lies on the line 43. The line 3x 2y 24  0 meets x-axis at A and
(A) x y 7  0 (B) x y 7  0 y-axis at B. The perpendicular bisector of AB meets
the line through (0, 1) parallel to x-axis at C. Then,
(C) x y 7  0 (D) x y 7  0
the area of the $ABC is
Solution: Observe that B and C are images of each
(A) 91 sq. unit (B) 12 sq. unit
other on the line y x and hence the side BC varies and
is perpendicular to the line y x. Hence, the orthocentre (C) 36 sq. unit (D) 48 sq. unit
Worked-Out Problems 85

Solution: The given line 3x 2y  24 in the intercept Therefore, the equation of AD is


form is
5 (23 / 2)
x y y 5 ( x 8)
1 8 5
8 12
33
 y 5 ( x 8)
so that A  (8, 0) and B  (0, 12). Equation of the perpen- 6
dicular bisector of the segments AB is 11
 y 5  ( x 8)
2 2
y 6  ( x 4) Therefore, p r  11 78  89.
3
 2x 3y 10  0 A(−8, 5)

This line meets the line y 1  0 at the point C( 13/2, 1).


A A
Therefore, the area of the triangle ABC is 2 2

8 0 1 8 0 1
1 1
0 12 1  0 12 1
2 4
13 13 2 2
1 1
2
5 3
8 0 1 B(−15, −19) D C(1, −7)
1
 0 12 1 FIGURE 2.68
4
13 2 2 Answer: (D)
1
 8(24 2) 1(0 156) 45. The equations of the sides AB, BC and CA of $ABC
4 are, respectively, 2x y  0, x by  c and x y  3.
364 If G(2, 3) is the centroid of $ABC, then the value of
  91
4 b c is
Answer: (A) (A) 50 (B) 47 (C) 74 (D) 57

44. The vertices of a triangle are A( 8, 5), B( 15, 19)


Solution: See Fig. 2.69. We have
and C(1, 7). The internal bisector of the angle A has 2x y  0 (2.90)
the equation px 2y r  0. Then, p r is equal to
x by  c (2.91)
(A) 78 (B) 88 (C) 98 (D) 89
x–y3 (2.92)
Solution: See Fig. 2.68. We have
Solving Eqs. (2.90) and (2.92), we get A (1, 2). Solving
2 2
AB  7 24  625  25 Eqs. (2.90) and (2.91), we get
¥ c 2c ´
BC  16 2 12 2  400  20 B¦ ,
§ 1 2b 1 2b µ¶
CA  9 2 12 2  225  15 Solving Eqs. (2.91) and (2.93), we get
Suppose the internal bisector of the angle A meets the ¥ 3b c c 3 ´
C¦ ,
side BC at D so that BD:DC  AB:AC  25:15  5:3. § 1 b 1 b µ¶
Therefore
Suppose D(x, y) is the midpoint of BC. Then
¥ 15 s 3 5(1) 19 s 3 5 s ( 7) ´
D¦ , µ¶ 2x 1 2y 2
§ 5 3 5 3 AG:GD  2:1   2 and 3
3 3
¥ 92 ´ ¥ 23 ´
 ¦ 5, µ  ¦ 5, µ ¥ 5 11´
§ 8 ¶ § 2 ¶ D¦ , µ
§2 2 ¶
86 Chapter 2 Straight Line and Pair of Lines

D lies on BC. So 6 56
and k  2 
5 ¥ 11´ 25 25
b¦ µ  c
2 § 2¶ Hence
 11b 2c  5 (2.93) ¥ 17 56 ´
Ha  ¦ , µ
Again, if E is the midpoint of AC and BG:GE 2:1, then § 25 25 ¶

¥ 6 12b c 9 18b 2c ´ Therefore, the circumradius is


E¦ ,
§ 2(1 2b) 2(1 2b) µ¶ R  OH a
Since E lies on the side CA, we have 2 2
¥ 17 ´ ¥ 56 ´
6 12b c 9 18b 2c  ¦2 µ ¦3 µ
3 § 25 ¶ § 25 ¶
2(1 2b) 2(1 2b)
 3 6b 3c  6 12b 332 19 2

 6b c  3 (2.94) 25

Solving Eqs. (2.93) and (2.94), we get b  11 and c  63. 1450


Hence, b c  74. 1450 25 58
  
25 5 5
A(1, −2)
A

2
=0

x−

O(2, 3) H(1, 2)
+y

y=
2x

G(2, 3)
4x − 3y + 3 = 0
1 B C

x + by = c D
B C HÄ

FIGURE 2.69 FIGURE 2.70

Answer: (C) Answer: (B)

46. In an acute-angled triangle ABC, equation of the 47. A ray of light emanating from the point (3, 4) meets
side BC is 4x – 3y 3  0, orthocentre H is (1, 2) and the y-axis at (0, b) and reflects towards the x-axis
the circumcentre O is (2, 3). Then the value of the which meets it at (a, 0) and again reflects from
circumradius R is x-axis to pass through the point (8, 2). Then the val-
ue of a is
28 58 23
(A) (B) (C) 23 (D) 13 9 17 14
5 5 5 (A) (B) (C) (D)
3 2 3 3
Solution: See Fig. 2.70. Suppose the image of the or-
thocentre H(1, 2) on the side BC is Ha. It is known that the Solution: Suppose P  (3, 4), B  (0, b), A(a, 0) and
images of the orthocentre of a triangle in the sides lie on the Q  (8, 2) (see Fig. 2.71). From the figure, we can see
circumcircle. Hence, Ha lies on the circumcircle. Suppose that
Ha (h, k). Therefore, by Theorem 2.13, part (2) we have
2 4 b b 4 b b 4
   
h 1 k 2 2(4 6 3) 2 8 a 3 0 a 3 a 3 a
  
4 3 4 2 32 25  2(3 a)  4(8 a)
Therefore  6a  26
8 17 13
h  1  a
25 25 3
Worked-Out Problems 87

y 49. In $ABC, A  (4, 1). If x 1  0 and x y 1  0


P(3, 4)
are the interior angle bisectors of angles B and C,
respectively, then the equation of the side BC is
(A) 2 x y 3  0 (B) 2 x y 3  0
B(0, b)
(C) x 2y 3  0 (D) x 2 y 3  0
Q(8, 2) Solution: Let Aa and Aaa be the images of A(4, −1)
in the bisectors x − 1  0 and x y 1  0, respectively.
Therefore, Aa and Aaa lie on the line BC (see Problem 27
q
O A(a, 0) x of the section ‘Subjective Problems’). Hence, the equa-
tion of BC is the equation of AaAaa. Now, Aa (−2, −1)
and Aaa (0, 3). Therefore, the equation of the side BC is
FIGURE 2.71
3 1
Answer: (A) y 3 ( x 0)
0 2
48. Let P be the point (3, 2). Q is the image of P in the x-  2x – y 3  0
axis and R is the image of Q in the line y  x. Finally, Answer: (B)
S is the image of R through the origin and PQRS is a
convex quadrilateral. Then, the area of PQRS is Note: The main tool used here is that if L  0 is the angle
(A) 15 (B) 16 (C) 18 (D) 5 bisector of ABC , then the image of A on the line L  0
lies on the line BC (see Fig. 2.73).
Solution: See Fig. 2.72. By hypothesis, P  (3, 2), Q  (3,
−2), R  (2, −3) and S  (−2, 3).
0
L=
Area of PQRS  Area of DPQR Area of DPRS A
90°
1
 3( 2 3) 3( 3 2) 2(2 2)
2
B AÄ C
1
3( 3 3) 2(3 2) 2(2 3)
2 FIGURE 2.73

1 1
 3 15 8 18 2 10 50. The equations of the sides AB and AC of $ABC,
2 2 respectively, are 2x – y  0 and x y  3 and the
 2 13  15 centroid G is (2, 3). Then, the slope of the side BC is

S(−2, 3) (A) 4 (B) 3 (C) 2 (D) 5


y
Solution: See Fig. 2.74. Solving the equations 2 x y  0
and x y  3, we have A  (1, 2). Equation of the median
2 P(3, 2)
through A(1, 2) is

1 2 3
y 2  ( x 1)  x 1
1 2
x – y  –1 (2.95)
O 1 2 3 x
Let B  (h, 2h) and C  (k, 3 – k). Therefore, the midpoint
Area of ΔPRS −1 of BC is
y=
−x

−2 Q(3, −2) ¥ h k 2h k 3 ´
¦§ , µ¶ (2.96)
2 2
Also, let D be the midpoint of BC. Since AG:GD 2:1, we
have
R(2, −3)
¥ 5 7´
FIGURE 2.72 D¦ , µ (2.97)
§ 2 2¶
Answer: (A)
From Eqs. (2.95) and (2.97), we have
88 Chapter 2 Straight Line and Pair of Lines

h k 5 2h k 3 7 Solution: Solving x y – 2  0 and 2x – y 1  0, we get


 and  that the point of intersection of these two lines as
2 2 2 2
which implies that ¥ 1 5´
¦§ , µ¶
3 3
h k  5 and 2 h k  4
The line ax by c  0 also passes through the point
Solving these equations, we get h  3 and k  2. Hence, (1/ 3, 5 / 3) implies that
B  (3, 6) and C  (2, 1). Thus, the slope of BC is
¥ 1´ ¥ 5´
6 1 a ¦ µ b¦ µ c  0
5 § 3¶ § 3¶
3 2
¥ 1´ ¥ 5´
A(1, 2)  2a ¦ µ 3b ¦ µ c  0
§ 6¶ § 9¶

x+ Hence, the line 2ax 3by c  0 passes through the point


y= ¥ 1 5´
0

3 ¦§ , µ¶ .
y=

6 9
2x −

G(2, 3)
Answer: (D)

C 53. If a, b and c are real and satisfy the equation 4a2 9b2
12ab – 9c2  0, then ax by c  0 represents two
families of concurrent lines. The distance between
B
their points of concurrence is
FIGURE 2.74 2
(A) 1 (B) 3 3 (C) 13 (D) 2 13
Answer: (D) 3
Solution: From the given relation we have
51. If a, b and c are real and a b c  0, then the line
3ax 4by c  0 passes through the point whose (2a 3b)2 (3c)2  0
coordinates are
 (2a 3b 3c)(2a 3b 3c)  0
¥ 1 1´  2a 3b 3c  0 or 2a 3b 3c  0
(A) (1, 1) (B) ¦ , µ
§ 3 4¶
¥ 2´ ¥ 2 ´
¥ 1 1´  a ¦ µ b(1) c  0 or a ¦ µ b( 1) c  0
(C) (3, 4) (D) ¦ , µ § 3¶ § 3¶
§ 4 3¶
which implies that ax by c  0 passes through
Solution: We have
¥ 2 ´ ¥ 2 ´
¥ 1´ ¥ 1´ ¦§ , 1µ¶ or ¦§ , 1µ¶
a b c  0  3a ¦ µ 4b ¦ µ c  0 3 3
§ 3¶ § 4¶
whose distance between them is
which implies that the line 3ax 4by c  0 passes through
the point 2 13
3
¥ 1 1´
¦§ , µ¶ Answer: (C)
3 4
Answer: (B) 54. Let A be (1, 1). A line through A meets the x-axis in
B. A line through A and perpendicular to AB meets
52. If the lines ax by c  0 (a, b, c are non-zero and the y-axis in C. Then the locus of the midpoint of the
real), x y – 2  0 and 2x – y 1  0 are concurrent, segment BC is
then the lines 2ax 3by c  0 are passing through
a fixed point given by (A) x y  2 (B) x y  2 xy
1
¥ 1 1´ (C) x y  (D) x y  1
(A) ¦ , µ (B) (2, 3) 2
§ 2 3¶
¥ 2 7 ´ ¥ 1 5´ Solution: See Fig. 2.75. Let the line passing through the
(C) ¦ , µ (D) ¦ , µ point A(1, 1) be y – 1  m(x – 1). Therefore
§3 5 ¶ § 6 9¶
Worked-Out Problems 89

¥ m 1 ´ l m
B¦ , 0µ h 2 2
and k  2
§ m ¶ l m l m2
The equation of the line through the point A(1, 1) and l 2 m2 1
 h2 k 2  2 2 2
 2 (2.98)
perpendicular to AB is (l m ) (l m 2 ) 2
1 Also
y 1 ( x 1)
m
l 2 m2 1
Therefore, h2 k 2   (2.99)
(l 2 m 2 ) 2 (l 2 m 2 )
¥ 1 m´ From Eqs. (2.98) and (2.99), we have
C  ¦ 0, µ
§ m ¶
h 2 k 2  ( h 2 k 2 )2
Let m(h, k) be the midpoint of BC. Therefore,
Hence (h, k) lies on the curve we get
m 1 m 1
h and k
2m 2m x 2 y 2  ( x 2 y 2 )2
This gives Answer: (B)

(m 1) 56. The vertices B and C forming the base of an isosceles


h 1  k h k
2m triangle ABC are (2, 0) and (0, 1), respectively. The
Hence, the locus of M(h, k) is the line x y  1. vertex A lies on the line x  2. The orthocentre of
$ABC is
y
¥5 ´ ¥3 ´
(A) ¦ , 1µ (B) ¦ , 1µ
§4 ¶ §4 ¶
A(1, 1)
¥ 3 3´ ¥ 4 5´
(C) ¦ , µ (D) ¦ , µ
90° § 2 2¶ § 3 3¶

Solution: See Fig. 2.76. A  (2, k), B  (2, 0), C  (0, 1).
O B x Now
M
C
AB  AC
 2 2 (k 1)2  k 2
 5 2k  0
FIGURE 2.75 5
Answer: (D) k
2

55. Two lines x y  0 and x – y  0 are fixed sides of a Thus,


triangle and the third side is a variable line whose ¥ 5´
equation is lx my  1 where l2 m2  1. Then the A  ¦ 2, µ
§ 2¶
circumcentre of the triangle lies on the curve
Now, the equation of the altitude through A  2, 5 / 2 is
(A) x 2 y2  ( x 2 y2 )2 (B) x 2 y2  ( x 2 y2 )2
5 ¥ 1´
(C) x 2 y2  x 2 y2 (D) x 2 y2  4 y  2( x 2) ¦&slope of BC  µ
2 § 2¶
Solution: The vertices of the triangle are
 2 y 5  4( x 2)
¥ 1 1 ´ ¥ 1 1 ´
O(0, 0), A ¦ , and B ¦ ,  4x 2y  3 (2.100)
§ l m l m µ¶ § l m l m µ¶
Altitude through C(0, 1) is
Since the lines y x and y  x are at right angles, AOB 
90o. Therefore, the circumcentre of $OAB is the midpoint y1 (2.101)
of the hypotenuse AB. Let S(h, k) be the circumcentre of Therefore, from Eqs. (2.100) and (2.101), the orthocentre
$OAB. Therefore, of $ABC is
1¥ 1 1 ´ 1¥ 1 1 ´ ¥5 ´
h ¦§ µ¶ and k  ¦§ µ ¦§ , 1µ¶
2 l m l m 2 l m l m¶ 4
90 Chapter 2 Straight Line and Pair of Lines

y
A(2, K)
a cos A b sin A 0
 sin A cos A 0 0
a b p

 p cos A (a cos A ) sin A (b sin A )  0

x=2
C(0, 1)
 a cos A b sin A 1  0 (2.102)
Since the angle between xcos@ ysin@  p and xsin@
ycos@ is P /4, by Theorem 2.12, we have
O B(2, 0) x
1 P a cos A b sin A
 cos 
2 4 cos A sin 2 A a 2 b2
2

1
FIGURE 2.76  [by Eq . (2.102)]
Answer: (A) a b2
2

 a 2 b2  2
57. Let A (3, 4), B  (1, 2) and P  (2 L 1, 2 L 1). If Answer: (A)
the sum PA PB is minimum, then the value of K is
7 7 3 59. P is a point on the line 2x y – 1  0. Points Q and R
(A) (B) 1 (C) (D) are on the line 2x y – 2  0 such that $PQR is equi-
8 9 2
lateral. Then, the length of the side of the triangle is
Solution: It is known that PA PB ≥ AB and equality
occurs if and only if A, P and B are collinear. Hence PA 2
(A) 4 5 (B) 3 5 (C) (D) 5
PB is minimum which implies that 15

3 4 1 Solution: See Fig. 2.77. We have p as the altitude


through P. So
1 2 1 0
1 2 1
2L 1 2L 1 1 p 
2 2 5
2 1
 3(2 2 L 1) 4(1 2 L 1) 1(2 L 1 4 L 2)  0
Suppose a is the length of the side. Therefore
 16 L 3 8 3  0
3 p 1
7  sin 60o  
L 2 a a 5
8
Answer: (A) Hence
2
58. The angle between the lines xcos@ ysin@  p and a
15
ax by p  0 is P / 4 where p  0. If these two lines
together with the line x sin @ y cos @ = 0 are concur- 2x y 1 0 P
rent, then
1
(A) a 2 b2  2 (B) a 2 b2 
2
1
(C) a 2 b2  1 (D) a 2 b2  1 5
p

Solution: Since the lines are concurrent, by Theorem


2.22, we have 2x y 2 0 60 60
Q R
cos A sin A p
FIGURE 2.77
sin A cos A 0 0
a b p Answer: (C)
Worked-Out Problems 91

60. If a, b and c are real and are in AP, then the family of 63. A  ( 4, 0) and B  (4, 0). The points M and N are
lines ax by c  0 are concurrent at the point variable points on the y-axis such that N lies above
M and MN  4. The lines AM and BN intersect at P.
(A) (1, 1) (B) (1, 2)
Then, the locus of P is
(C) (2, 1) (D) (1, 2)
(A) x 2 2 xy 16  0 (B) x 2  2 xy 16
Solution: Suppose
(C) x 2 16  2 xy (D) x 2 2 xy 16  0
a c
b Solution: See Fig. 2.78. We have A  (−4, 0) and B  (4,
2
0) so that y-axis is the perpendicular bisector of AB. Since
Then MN  4, we can take N  (0, a 4) and M  (0, a). Equation
ax by c  0  2ax (a c) y 2c  0 of the line AM is
x y
 (2 x y)a ( y 2)c  0 1 (2.103)
4 a
which represents family of concurrent lines concurrent at
the point C(1, 2) which is the intersection of the lines and the equation of BN is
2 x y  0 and y 2  0. x y
1 (2.104)
Answer: (D) 4 a 4
From Eq. (2.103), we have
61. If A(1, p2), B(1, 0) and C(p, 0) where p x are the
vertices of a triangle, then the value of p for which 4y
a (2.105)
the area of $ABC is minimum is 4 x
1 1 From Eq. (2.104), we get
(A) (B)
2 3 4y
a 4  (2.106)
1 4 x
(C) (D) None
2 Therefore, from Eqs. (2.105) and (2.106), we have
Solution: Let S be the area of ΔABC so that y y
1
4 x 4 x
1 p2 1
1 1  2 xy  16 x 2
S 0 1 1  [1(1 0) p2 (0 p) 1(0 p)]
2 2
p 0 1  x 2 2 xy 16  0
1 3 y
 p p 1
2
1
Real value of p  exists such that 3 p2 1  0. Hence N
3 1
minimum value of S exists at p  . 4
3 P
Answer: (B)
M
62. The locus of the point which moves such that its dis-
tance from the point (4, 5) is equal to its distance
from the line x – y 1  0 is
(A) a straight line A(−4, 0) O B(4, 0) x

(B) a circle with centre at (4, 5)


(C) ( y 5)2  4( x 4)
(D) ( x 4)2  y 5 FIGURE 2.78
Solution: The point (4, 5) lies on the line x – y 1  0. Answer: (A)
Hence, the required locus is the line y – 5  –1(x – 4) or
x y – 9  0. 64. The vertices of a parallelogram are described in the
Answer: (A) order A(3, 1), B(13, 6), C(13, 21) and D(3, 16). If a
92 Chapter 2 Straight Line and Pair of Lines

line through origin divides the parallelogram into A


two congruent parts, then the slope of the line is
13 11 8 11 M E F N
(A) (B) (C) (D)
8 12 11 8
L
Solution: See Fig. 2.79. It can be observed that AD and
BC are vertical lines. Suppose the line through the origin
meets AD at P and BC at Q in such a way that CQ  B C
PA, DP BQ. Also PQCD and PQBA are the congruent
parts. Suppose AP k. FIGURE 2.80
Therefore, P  (3, k 1) and Q  (13, 21 k ). Hence, Answer: (C)
the slope of the line is
k 1 21 k 66. The equations of two adjacent sides of a rhombus are y
 x and y  7x. The diagonals of the rhombus intersect
3 13
at the point (1, 2). Thus, the area of the rhombus is
 13k 13  63 3k
 16k  50 50 20 10 40
(A) (B) (C) (D)
25 3 3 3 3
k
8 Solution: See Fig. 2.81. Let M be the point (1, 2). Since
Thus, the slope of the line is the diagonal AC is perpendicular to the diagonal OB, the
equation of the diagonal AC is
(25 / 8) 1 33 11
  1
3 3s8 8 y 2  ( x 1)
2
y
C(13, 21)  x 2y 5  0 (2.107)
k Solving Eq. (2.107) and the equation y x, we get
D(3, 16)
Q
¥ 5 5´
A¦ , µ
§ 3 3¶

B(13, 6) Solving Eq. (2.107) and the equation y  7x, we get


P
k ¥ 1 7´
C¦ , µ
§ 3 3¶
A(3, 1)

O x Therefore, area of the rhombus is 2$OAC which is given


by
FIGURE 2.79 1 5 7 5 1 30 10
2s s s  
Answer: (D) 2 3 3 3 3 9 3
B
65. The number of straight lines that are equidistant
from the vertices of a triangle is
(A) 1 (B) 2
(C) 3 (D) infinite
Solution: See Fig. 2.80. Let ABC be a triangle. Let
C A
E and F be the midpoints of AB and AC, respectively.
FHHE M(1, 2)
Draw AL, BM and CN Fperpendicular
HHE FHHEline EF . It
to the
is known that the line EF is parallel to BC and EF 
y=

(1/2)BC. It is clear that


y=
7x

$BEM y $AEL and $CFN y $AFL


FHHE
Therefore, BM  AL  CN . Thus the line EF is equi- O(0, 0)
distant from A, B and C. Similarly, we have two more FIGURE 2.81
lines. Thus, the numbers of lines equidistant from A, B
Answer: (C)
and C is 3.
Worked-Out Problems 93

67. A point P on the line 3x 5y – 15  0 is equidistant 10 m 5(10 m)


from the coordinates axes. P can lie in Case 1: 
m 1 m 7
(A) first quadrant  (10 m)[m 7 5(m 1)]  0
(B) first or second quadrant  m  10 or m  3
(C) first or third quadrant
10 m 5(10 m)
(D) any quadrant Case 2: 
m 1 m 7
Solution: The point (x, y) is equidistant from both axes
 (10 m)[m 7 5(m 1)]  0
š x  y  y  p x.
2 1
15  m  10 or m  
1. y x and (x, y) lies on the line 3x 5y  15  x  , 6 3
8
15 If m  10, then the third side passes through the origin
y  P lies in the first quadrant.
8 (0, 0), which is false in this case. Thus
15 15
2. y  x and (x,y) lies on 3x 5y  15  x  , y  P 1
2 2 m  3 or
lies in the second quadrant. 3
Answer: (B) Hence, the third side is
3x y 7  0
68. Two equal sides of an isosceles triangle are given by
the equations y  x and y  7x and its third side  x 3 y 31  0
passes through the point (1, 10). Then, the equa-
tions of the third side is

7x
(A) x 3 y 31  0 or 3x y 13  0

y=
(B) x 3 y 29  0 or 3 x y 13  0
y
=
−x

(C) 3 x y 7  0 or x 3 y 31  0
(D) 3 x y 7  0 or x 3 y 29  0
Solution: See Fig. 2.82. Let
O
y 10  m( x 1) (2.108)
be the equation of the third side. Substituting y  x in
A
Eq. (2.108), we have
B (1, −10)
10 m (10 m)
x and y
1 m 1 m
Let FIGURE 2.82

¥ 10 m (10 m) ´ Answer: (C)


A¦ , µ
§ 1 m 1 m ¶
69. The equation of the line which passes through the
Substituting y  7x in Eq. (2.108), we have intersection of the lines x 2y 3  0 and 3x 4y
7 0 and is perpendicular the line x – y 8  0 is
¥ 10 m 7(10 m) ´
B¦ , µ
§ m 7 m 7 ¶ (A) x 2 y 3  0 (B) x y 2  0
Now, (C) x y 2  0 (D) 3 x 3 y 4  0

OA  OB Solution:
Direct Method: Intersection of the lines x 2y 3  0 and
(10 m)2 (10 m)2 (10 m)2 49(10 m)2
  3x 4y 7 0 is ( 1, 1). Slope of the line x – y 8  0
(m 1)2 ( m 7) 2 is 1. Hence, the equation of the required line is
2 (10 m) 5 2 (10 m) y 1  1( x 1) or x y 2  0
 p
m 1 m 7 Answer: (C)
10 m 5(10 m)
 p
m 1 m 7
94 Chapter 2 Straight Line and Pair of Lines

70. Let A be the point (t, 2) and B be the point on the The three families have a common line if
y-axis such that the slope of AB is t. Then, the locus
of the midpoint of AB, as t varies over all real num- ¥ a a 2 12 ´
¦§ 2 , µ , (1, 1) and (3, 3)
bers, is a 6 a2 6 ¶
(A) y  2 2 x 2 (B) x 2 y 1  0 are collinear. Since the equation of the line joining (1, 1)
(C) y  1 x 2 2
(D) 2 x y 2  0 and (3, 3) is y x, we have

Solution: See Fig. 2.83. Equation of the line AB is a a 2 12



a2 6 a2 6
y 2  t( x t )
 a 2 a 12  0
Now,
 (a 4)(a 3)  0
x  0  y  2 t2
 a  4, 3
 B  (0, 2 t 2 )
a4 (& a  0)
Let M(x, y) be the midpoint of AB. Therefore
Answer: (D)
t 0 2 (2 t 2 )
x ,y
2 2 72. If the length of the intercept made on the line y ax
by the lines y  2 and y  6 is less than 5, then
 t  2 x, 2 y  4 t 2  4 4 x 2
4 4 4 4
Thus, the locus of M is y  2 2 x 2. (A) a (B) a  or a 
3 3 3 3
y 3 4
(C) a (D) a ( d, d)
4 3
A (t, 2) y=2 Solution: The line y ax intersects the lines y  2 and y
 6 at points A  2 /a, 2 and B  6 /a, 6 . Now
B
2
¥ 2 6´
AB  5 š ¦ µ (2 6)2  52
§ a a¶
O x
16
š  25 16  9
a2
16
FIGURE 2.83 š a2 
9
Answer: (D)
4 4
ša or a 
71. If the three families of lines 3 3
(y – 2x 1) K1(2y – x – 1)  0 Answer: (B)

(3y – x – 6) K2(y – 3x 6)  0 73. The equation of the obtuse angle bisector of the an-
and (ax + y 2) + K3(6x + ay a) = 0 (a > 0) gle between the lines x – 2y 4  0 and 4x – 3y 2 
0 is
have a common line, then the value of a is
(A) (4 5 ) x (2 5 3) y (4 5 2)  0
(A) 3 (B) 2 (C) 6 (D) 4
(B) (4 5 ) x (2 5 3) y 4 5  0
Solution: The first and second families of lines are con-
current at (1, 1) and (3, 3), respectively, while the third (C) (4 5 ) x (2 5 3) y 4 5 2  0
family of lines are concurrent at
(D) (4 5 ) x (2 5 3) y 4 5  0
¥ a a 2 12 ´
¦§ 2 , 2 µ Solution: c1  4, c2  2 are positive. Since a1a2 b1b2 
a 6 a 6 ¶
4 6  0, by Theorem 2.26,
Worked-Out Problems 95

x 2y 4 ¥ 4 x 3y 2 ´  ( x y 1)( x y 1)  0
 ¦
2
1 2 2 § 4 2 32 µ¶ Therefore, the pair of lines is x y 1  0 and x y 1  0
and their angle bisectors are
is the obtuse angle bisector. So we have
x y 1 ¥ x y 1´
5 ( x 2 y 4)  4x 3y 2  p¦ µ
2 § 2 ¶
 (4 5 ) x (2 5 3) y (4 5 2)  0 That is y  1 and x  0. Therefore, the sides of the triangle
Answer: (A) are x  0, y  1 and x y  3. The vertices are (0, 1), (0, 3)
and (2, 1). Hence the area of the triangle is
74. Two vertices of a triangle are (5, 1) and ( 2, 3). If or-
1 4
igin is the orthocentre of the triangle, then the third 0(3 1) 0(1 1) 2(1 3)   2
vertex is 2 2
Answer: (A)
(A) (4, 7) (B) ( 4, 7)
(C) (4, 7) (D) ( 4, 7) 76. The slope of one of the lines ax2 2hxy by2  0 is
Solution: See Fig. 2. 84. Let B  (5, −1) and C  (−2, 3), twice that of the other. Then
and the third vertex A  (h, k). The orthocentre is O(0, 0). (A) 8 h2  9ab (B) 4 h2  3ab
Since AO is perpendicular to BC, we have
(C) h2  4ab (D) 9 h2  8ab
¥ k´ ¥ 3 1 ´
¦§ µ¶ ¦§ µ  1 Solution: Let y  mx and y  2mx be the two lines.
h 2 5 ¶
Therefore
 4k  7 h (2.109)
ax 2 2hxy 6 y2 y b( y mx)( y 2mx)
Now BO is perpendicular to AC
Equating the corresponding coefficients, we have
¥ 1´ ¥ 3 k ´
¦ µ¦  1 2h a
§ 5 ¶ § 2 h µ¶ 3m  , 2 m2 
b b
 3 k  5(2 h)
2
a ¥ 2 h ´ 8h2
 5h k  13 (2.110)   2 m2  2 ¦ µ  2
b § 3b ¶ 9b
Solving Eqs. (2.109) and (2.110), we have h  4 and 2
 9ab  8 h
k  7. Hence, A  ( 4, 7).
Answer: (A)
A (h, k)
77. A is a point on the x-axis. Through the point A, a
line is drawn parallel to y-axis so as to meet the lines
ax2 2hxy by2  0 in B and C. If AB BC, then

O(0, 0) (A) 9 h2  8ab (B) 8 h2  9ab


(C) 4 h2  3ab (D) 3h2  8ab
B (5, −1) C (−2, 3) Solution: See Fig. 2.85. Let y  m1 x and y  m2 x be
FIGURE 2.84 the lines represented by ax 2 2 hxy by2  0. Therefore
Answer: (D) 2 h
m1 m2 
b
75. Area of the triangle formed by the line x y  3 and a
the angle bisectors of the pair of lines x2 – y2 2y 1 m1 m2 
b
 0 is
Suppose A  (A , 0) and the line x @ meets y  m1 x at
(A) 2 (B) 4 (C) 6 (D) 8
B and y  m2 x at C. Thus, B  (A , m1A ) and C  (A , m2A ).
(IIT-JEE 2004) Now,
Solution: m2A 0
AB  BC   m1A
x 2 y2 2 y 1  0  x 2 ( y 1)2  0 2
 m2  2 m1
96 Chapter 2 Straight Line and Pair of Lines

Hence, from Worked-Out Problem 76, we get 8 h2  9ab. 80. Let PQR be right-angled isosceles triangle right an-
gled at P(2, 1). If the equation of the line QR is 2x
y y – 3  0 then the combined equation of the pair of
x
m2 lines PQ and PR is
=
y
C
(A) 3 x 2 3 y2 8 xy 20 x 10 y 25  0
x
y = m1 (B) 3 x 2 3 y2 8 xy 20 x 10 y 25  0
B (C) 3 x 2 3 y2 8 xy 10 x 15 y 20  0
O A(a, 0) x (D) 3 x 2 3 y2 8 xy 15 y 20  0
(IIT-JEE 1999)
FIGURE 2.85 Solution: See Fig. 2.86. Let m be the slope of the line
Answer: (B) PQ. Since PQR  45°, from the ‘Note’ of Theorem 2.12,
we have
78. If one of the lines represented by ax2 2hxy by2  m 2
0 bisects the angle between the axes, then 1  tan 45o 
1 m( 2)
(A) 4 h2  ab (B) 8 h2  9ab  m 2  p(1 2 m)
(C) (a b)2  4 h2 (D) (a b)2  2 h2 1
 m  3,
Solution: Suppose y x is one of the lines. That is, 3
Therefore, the combined equation of the lines PQ and
ax 2 2 hx 2 bx 2  0 PR is
 a b  2 h
¨ 1 ·
[ y 1 3( x 2)] © y 1 ( x 2)
 (a b)2  4 h2 ª 3 ¹̧
Suppose y  x is one of the lines. Then  (3 x y 5)( x 3 y 5)  0

a 2h b  0  3x2 3y2 8xy 20 x 10 y 25  0


R
 (a b)2  4 h2
Answer: (C) 45° 2x
+y
−3
79. If the equation 12x2 7xy ky2 13x – y + 3  0 rep- =0
resents a pair of lines, then the value of k is
90° 45°
(A) 5 (B) 5 (C) 10 (D) 10
P (2, 1) Q
Solution: Comparing the given equation with the second-
degree general equation, we have FIGURE 2.86
Answer: (B)
7 13 1
a  12, h  , b  k, g  , f  and c  3
2 2 2 81. Area of the parallelogram formed by the lines y 
Since the equation represents pair of lines, by Theorem mx, y  mx 1, y  nx and y  nx 1, where m ≠ n,
2.31, we get is equal to
m n 2
$  abc 2 fgh af 2 bg 2 ch 2  0 (A) (B)
(m n)2 m n
That is,
1 1
(C) (D)
¥ 1´ ¥ 13 ´ ¥ 7 ´ ¥ 1´ ¥ 169 ´ m n m n
(12)(k )(3) 2 ¦ µ ¦ µ ¦ µ 12 ¦ µ k ¦
§ 2 ¶ § 2 ¶ § 2¶ § 4¶ § 4 µ¶
(IIT-JEE 2001)
¥ 49 ´
3¦ µ  0 Solution: From Problem 2 of the section ‘Subjective
§ 4¶
Problems’, the area of the parallelogram is
 144k 91 12 169k 147  0
(1 0)(1 0) 1
 25k  250 
m n m n
 k  10
Answer: (B)
Answer: (D)
Worked-Out Problems 97

82. All chords of the curve 3x2 – y2 – 2x 4y  0 which n1 3n2  14 (2.112)


subtend a right angle at the origin pass through a
fixed point whose coordinates are 2 n1 4 n2  22 (2.113)

(A) (0, 0) (B) (1, 2) Solving Eqs. (2.112) and (2.113), we have n1  5 and
n2  3. Therefore, the sides of the parallelogram are
¥ 1 10 ´
(C) ¦ , 1µ (D) (2, 1) 3x 4 y  0
§ 3 ¶
3x 4 y 5  0
(IIT-JEE 1982)
x 2y  0
Solution: See Fig. 2.87. Let lx my  1 be the chord and x 2y 3  0
of the given curve subtending right angle at the origin:
Suppose the line meets the curve at A and B. Hence, by Hence, from Problem 2 of the section ‘Subjective Prob-
Theorem 2.33, the combined equation of the pair of lines lems’, the area of the parallelogram is
OA and OB is (5 0)(3 0) 15
2 2
 sq. unit
3 x y (2 x 4 y)(lx my)  0 3(2) (4) 2
Since AOB  90o, from the above equation and from Answer: (C)
Theorem 2.28, we have
84. If the second-degree general equation S y ax2 2hmy
Coefficient of x 2 Coefficient of y2  0 by2 2gx 2fy c  0 represents pair of intersect-
 (3 2l ) ( 1 4 m)  0 ing lines, then the area of the parallelogram formed
by the lines S  0 and ax2 2hxy by2  0 is
 l 2m 1  0
 l m( 2)  1 C C
(A) (B)
(a b)2 4 h2 2 (a b)2 4ab
Hence the line lx my  1 passes through the point (1, 2).
C C
y (C) (D)
2 2
2 (a b) 4 h 2 h2 ab
B
Solution: Suppose the lines represented by S  0 are
lx + m l1 x m1 y n1  0 and l2 x m2 y n2  0 so that from
y=1
A
Theorem 2.32, the equation ax 2 2 hxy by2  0 repre-
90°
sents l1 x m1 y  0 and l2 x m2 y  0. Also we have
O x
l1l2  a
l1 m2 l2 m1  2 h
FIGURE 2.87 m1 m2  b
Answer: (B) l1 n2 l2 n1  2 g

83. The area of the parallelogram formed by the lines m1 n2 m2 n1  2 f


3x2 10xy 8y2 14x 22y 15  0 and 3x2 10xy and n1 n2  c
8y2  0 is
Hence the sides of the parallelogram are
15
(A) 5 (B) 10 (C) (D) 15 l1x m1y 
2
Solution: We have l1 x m1 y n1  0
l2 x m2 y  0
3 x 2 10 xy 8 y2 y (3 x 4 y)( x 2 y)
l2x n2y n2  0
Therefore, for some values of n1 and n2
Therefore, from Problem 2 of the section ‘Subjective
3 x 2 10 xy 8 y2 14 x 22 y 15 y Problems’, the area of the parallelogram is
(3 x 4 y n1 )( x 2 y n2 ) (2.111) (n1 0)(n2 0) n1 n2

Equating the coefficients of x and y on both sides of Eq. l1 m2 l2 m1 (l1 m2 l2 m1 )2 4l1l2 m1m2
(2.111), we have
98 Chapter 2 Straight Line and Pair of Lines

c c 1 m2 2
   
4 h2 4ab 2 h2 ab (1 m)2 (1 m)2 3
Answer: (D)  3m2 3  2(1 2 m m2 )

Note: By using this formula, for the parallelogram in  m2 4 m 1  0


Worked-Out Problem 83, the area is equal to 4 p 16 4
m  2p 3
15 15 2

2 25 24 2 Now
P
85. If x2 – 2pxy – y2  0 and x2 – 2qxy – y2  0 represent 0Q b  tan Q  m  2 3
3
a pair of lines (p xq) such that each pair bisects the
angle between the other, then so that P  15o.
(A) pq  1 (B) pq  1 Answer: (C)

1 1 1 1 1 87. The equation 16x4 – y4  0 represents


(C) 1 (D) 
p q p q 2
(A) a single straight line (B) a pair of lines
Solution: By Theorem 2.30, the equation of the pair of (C) a single point (D) xy  2
angle bisector of the lines x2 – 2pxy – y2  0 is
Solution: The given equation is
p( x 2 y2 )  (1 1) xy
(4x2 y2)(2x y) (2 x y)  0
2 2
 px 2 xy py  0
The equation 4 x 2 y2  0 represents the single point (0,
However, by hypothesis, x 2 2qxy y2  0 is the pair of 0). Hence, the equation represents the pair of lines
angle bisectors. Therefore, 2 x y  0 and 2 x y  0 which pass through the origin.
p 2 p Answer: (B)
   pq  1
1 2q 1
88. If the lines x 2y  9, 3x – 5y  5 and ax by  1
Answer: (B)
are concurrent, then the line 5x 2y – 1  0 passes
through the point
86. A straight line is drawn through the point (1, 2) mak-
ing an angle Q (0  Q b P / 3) with the positive direc- (A) (b, a) (B) (a, b)
tion of the x-axis to intersect the line x y – 4  0 at (C) ( a, b) (D) ( a, b)
a point P so that the distance between P and (1, 2) is
Solution: The point of intersection of the lines x 2y
2 / 3 . Then, the value of P is
– 9  0 and 3x – 5y 5  0 is (5, 2) which also lies on the
P P P P line ax by  1. Hence,
(A) (B) (C) (D)
3 10 12 18 a(5) b(2)  1
Solution: Let A be (1, 2) and the line through A(1, 2)  5a 2b  1
be y 2  m( x 1). This line meets the line x y  4 at
point P. So Therefore, the line 5x 2y  1 passes through the point
(a, b).
4 x 2  m( x 1)
Answer: (B)
 x(1 m)  2 m
2 m 2 m 3m 2 89. Straight lines are drawn from the point A(3, 2) to
x and y  4 x  4 
1 m 1 m 1 m meet the line 6x 7y – 30  0 at point P. Then, the
locus of the midpoints of the segment AP is
Therefore,
(A) x 2 y2  30 (B) 6 x 7 y  31
¥ 2 m 3m 2 ´
P¦ ,
§ 1 m 1 m µ¶ (C) (6 x 3)2 (7 y 2)2  30 2 (D) 6 x 7 y  32

Now, Solution: Let the line that passes through A(3, 2) be

2 2 x 3 y 2
2 ¥ 2 m´ ¥ 3m 2 ´ 2  r (say)
AP   ¦1 µ ¦§ 2 µ  cos Q sin Q
3 § 1 m¶ 1 m ¶ 3
Worked-Out Problems 99

Every point on this line is of the form P(3 r cos Q , 2 r Solution: The given relation is
sinP) and this lies on the line 6x + 7y = 30. It implies that
(5a 4b)2 c 2  0
r(6 cos Q 7 sin Q )  2 (2.114)  (5a 4b c)(5a 4b c)  0
Suppose M(h, k) be the midpoint of AP. Therefore, ¨ ¥ 5´ · ¨ ¥ 5´ ·
 © 2a ¦ µ b( 4) c ¸ © 2a ¦ µ b(4) c ¸  0
2 h  6 r cos Q and 2k  4 r sin Q ª § 2 ¶ ¹ª § 2 ¶ ¹
 6(2 h 6) 7(2k 4)  r(6 cos Q 7 sin Q )  2  2ax by c  0

[from Eq. (2.114)] It passes through

 12 h 14k 62  0 ¥5 ´ ¥ 5 ´
¦§ , 4µ¶ and ¦§ , 4µ¶
 6 h 7k 31  0 2 2
Answer: (B)
Hence, the locus of M(h, k) is the line 6 x 7 y 31  0.
Answer: (B) 92. The area of the triangle formed by the lines x y
 3 and angle bisectors of the pair of straight lines
90. If no two lines of the three lines x2 – y2 2y  1 is
(m – 2)x (2m – 5)y  0 (A) 2 sq. unit (B) 4 sq. unit
(m – 1)x (m2 – 7)y – 5  0 (C) 6 sq. unit (D) 8 sq. unit
and x y – 1  10 (IIT-JEE 2004)
are parallel, then the three lines are concurrent Solution: The given lines are
(A) for three values of m (B) for two values of m x 2 ( y 1)2  0
(C) for one value of m (D) no real value of m  ( x y 1)( x y 1)  0
Solution: By Theorem 2.22, the three lines are concur- Therefore, the given lines are x y 1  0 and x + y 1 = 0
rent if whose angular bisectors are
1 1 1 x y 1 x y 1
p
m 2 2m 5 0  0 2 2
m 1 m2 7 5 That is, y 1  0 and x  0. Thus, the vertices of the tri-
2 angle are (0, 1), (0, 3) and (2, 1). Hence, the area is
 1[(m 2)(m 7) (m 1)(2 m 5)]
5 ;2m 5 m 2=  0 1
2
;0(3 1) 0(1 1) 2(1 3)=  2 sq. unit
 m3 2 m2 7 m 14 2 m2 5m 2 m 5 Answer: (A)
10 m 25 5m 10  0
93. A straight line is drawn through (1, 0) to the curve
 m 3 4 m 2 5m 6  0 x2 y2 6x – 10y 1  0 such that the intercept made
 (m 3)(m2 m 2)  0 on it by the curve subtends a right angle at the origin.
Then, the slope of the line is
Since (m2 m 2)  0 has no real solutions, it follows
that m  3. However, m  3 implies that the three lines 1 1
are parallel which is in contradiction of the hypothesis. (A) 1 or (B) 1 or
9 9
Hence, if m x 3, then the lines cannot be concurrent.
1 1
Answer: (D) (C) 1 or (D) 1 or
9 9
91. If 25a2 16b2 – 4ab – c2  0, then the line 2ax by c  Solution: Let y  m(x – 1) be the line meeting the
0 passes through a fixed point whose coordinates are curve in the point A and B. Hence, by Theorem 2.33, the
combined equation of the pair of lines OA and OB (‘O’
¥5 ´ ¥5 ´ is the origin) is
(A) ¦ , 4µ (B) ¦ , 4µ
§2 ¶ §2 ¶
¥ 5 (mx y) (mx y)2
´
(C) ¦ , 4µ (D) (5, 4) x 2 y2 (6 x 10 y) 1 0
§ 2 ¶ m m
100 Chapter 2 Straight Line and Pair of Lines

Since AOB  90o, from the above equation, we have The lines represented by x2 4xy 4y2 5x 10y
4  0 are parallel lines and they are x 2 y 4  0 and
Coefficient of x 2 Coefficent of y2  0 x 2y 1  0 .
¥ 10 1 ´ Therefore, from Problem 2 of the section ‘Subjective
 (1 6 1) ¦ 1 µ 0
§ m m2 ¶ Problems’, the area is
 9 m2 10 m 1  0 ( 1 3)( 1 4)
6
 (9 m 1)(m 1)  0 1 0
1 Answer: (B)
m or m  
9
96. A straight line L passing through the point (3, 2) is
Answer: (B)
inclined at an angle 60° to the line y 3 x  1. If L
intersects the x-axis, then the equation of L is
94. The area bounded by the curves y  x 1 and
y  x 1 is (A) y 3 x 2 3 3  0
(A) 1 (B) 2 (C) 2 2 (D) 4 (B) y 3x 2 3 3  0
Solution: The given lines are (C) 3y x 3 2 3  0
y  x 1, y  x 1 and y  x 1, y  x 1 (D) 3y x 3 2 3  0
That is, the lines are (IIT-JEE 2011)
x y 1  0, x y 1  0, x y 1  0 and x y 1  0 Solution: Let the equation of the line L be y 2 
Therefore, from Problem 2 of the section ‘Subjective m(x – 3). Therefore, by hypothesis
Problems’, the area of the parallelogram is
m 3
3
(1 1)(1 1) 1 m 3
2
1 ( 1)
 m 3  p 3 (1 m 3 )
Answer: (B)
 m  0 or m  3
95. The area of the parallelogram formed by the pairs of
If m  0, then L should be horizontal, which is not true in
lines x2 4xy 4y2 – 5x – 10y 4  0 and y2 – 4y 3 
this case. Hence, m  3 and the equation of L is
0 sq. units is
(A) 4 (B) 6 (C) 8 (D) 12 y 2  3 ( x 3)

Solution: y2 4 y 3  0 represent the lines y 1  0  y 3x 2 3 3  0


and y 3  0. Answer: (B)

Multiple Correct Choice Type Questions


1. Let L1 be a straight line passing through the origin made by the circle on L1 and L2 , it follows that the two
and L2 be the straight line x y  1. If the intercepts chords are equidistant from the centre. Therefore
made by the circle x2 y2 – x 3y  0 on L1 and L2 are
equal, then L1 may be represented by the equations ¥ 1´ 3 1 3
m¦ µ 1
§ 2¶ 2
(A) x y  0 (B) x y0  2 2
1 m2 12 12
(C) x 7y  0 (D) x 7 y  0
Solution: The equation of the circle can be written as
(m 3)2
2 2  2
¥ 1´ ¥ 3´ 5 4(1 m2 )
¦§ x µ¶ ¦§ y µ¶ 
2 2 2  m2 6 m 9  8 8 m2
so that its centre is at 1/ 2, 3 / 2 and radius is 5 / 2. Let  7 m2 6 m 1  0
y  mx be the equation of L1 . Since the intercepts are  (7 m 1)(m 1)  0
Worked-Out Problems 101

1 Solution: See Fig. 2. 88. The point


 m  1,
7 ¥1 ´
M  ¦ , 1µ
§2 ¶
Hence, the equations of L1 are y  x and y  x / 7. That
is, x y  0 and x 7 y  0. is the intersection of the diagonals so that
Answer: (B), (C) 1 5
DM  MB  AC 
2 2
2. The lines ax by c  0, bx cy a  0 and cx ay
b  0 are concurrent if Since the slope of AC is 4 / 3, the equation of the diago-
nal BD is
(A) a b c  0
3¥ 1´
(B) a 2 b2 c 2  ab bc ca y 1 ¦§ x µ¶
4 2
(C) a 3 b3 c 3  3abc
3 6 x 11
(D) a 2 b2 c 2  2(ab bc ca) y (2 x 1) 1 
8 8
Solution: The three lines are concurrent if
Let
a b c
¥ 6 x 11´
b c a  0 (By Theorem 2.22) B  ¦ x, µ
§ 8 ¶
c a b
Now,
 3abc a 3 b3 c 3  0 2 2
5 ¥ 1´ ¥ 6 x 11 ´ 25
3 3 3
 a b c 3abc  0 MB  ¦x µ ¦ 1µ 
2 § 2 ¶ § 8 ¶ 4
Also
 16(2 x 1)2 (6 x 3)2  25 s 16
3 3 3
a b c 3abc  0
 100 x 2 100 x 375  0
 (a b c)(a 2 b2 c 2 ab bc ca)  0
 a b c  0 or a 2 b2 c 2  ab bc ca  4 x 2 4 x 15  0
Answer: (A), (B), (C)  (2 x 5)(2 x 3)  0
5 3
3. The area of a triangle ABC is 20 sq. unit. The coordi- x ,
nates of A are ( 5, 0) and those of B are (3, 0) and the 2 2
vertex C lies on the line x – y – 2  0. The coordinates Therefore
of C are
5 6 x 11 6(5 / 2) 11 1
(A) ( 3, 5) (B) ( 5, 7) x y  
2 8 8 2
(C) (5, 3) (D) (7, 5) 3 6( 3 / 2) 11 5
and x y 
Solution: Let C be ( x, x 2). Therefore 2 8 2
1 Therefore, other vertices are
20  5(0 x 2) 3( x 2 0) x(0 0) |
2 ¥ 5 1´ ¥ 3 5 ´
¦§ , µ¶ and ¦§ , µ¶
 40  8 x 16 | 2 2 2 2
 x 2  p5 D C (−1, 1)
Hence, x  7, 3 . Therefore, C  (7, 5), (–3, –5). 5
2
Answer: (A), (D)
M
4. If A(2, 3) and C( 1, 1) are the ends of a diagonal of 5
2
a squares ABCD, then the other vertices are
A (2, −3) B
¥ 1 5´ ¥ 5 1´
(A) ¦ , µ (B) ¦ , µ
§ 2 2¶ § 2 2¶ FIGURE 2.88

¥ 3 5 ´ ¥ 1 5 ´ Aliter (Using Complex Numbers): For complex numbers,


(C) ¦ , µ (D) ¦ , µ
§ 2 2¶ § 2 2¶ see Vol. 1. The points are
102 Chapter 2 Straight Line and Pair of Lines

¥1 ´ 7. A line l is drawn through the point (1, 2) so that it in-


M  ¦ , 1µ and C  ( 1, 1) tersects the line x y 4  0 at a point whose distance
§2 ¶
from the point (1, 2) is 2 / 3. Then the angle made by
Replace the point D by Z. Therefore we have l with the x-axis is
Z [(1 / 2) i] (A) 15o (B) 75o (C) 105o (D) 60o
i
( 1 i) [(1 / 2) i]
Solution: Let y 2  m(x 1) be the equation of the
1 ¥ 3 ´ 3i line l. That is,
 Z i  ¦ 2iµ i  2
2 § 2 ¶ 2 y  mx 2 m (2.115)
3 5
z i y  x 4 (2.116)
2 2
¥ 3 5´ Solving Eqs. (2.115) and (2.116), we have
 D  ¦ , µ
§ 2 2¶ m 2 3m 2
x and y 
Hence m 1 m 1

¥ 3 5´ ¥ 5 1´ Since the distance between


B  ¦2 1 ,3 1 µ  ¦ , µ
§ 2 2¶ § 2 2¶ ¥ m 2 3m 2 ´
¦§ , µ and (1, 2)
Answer: (B), (C) m 1 m 1 ¶

5. Equation of a line passing through (1, 1) which is at a is 2 / 3 , we have


distance of 1 unit from the origin is 2 2
¥ m 2 ´ ¥ 3m 2 ´ 2
(A) x  1 (B) y  1 ¦§ 1µ ¦ 2µ 
m 1 ¶ § m 1 ¶ 3
(C) x y  1 (D) x y  2 2
¥ 1 ´ ¥ m2 ´ 2
Solution: It is clear that the vertical line x  1 passes ¦ µ ¦ µ
§ m 1¶ § (m 1)2 ¶ 3
through (1, 1) and its distance from the origin is 1. Hence,
y  1 is the other line.  3 3m2  2(m 1)2
Answer: (A), (B)
 m2 4 m 1  0
6. The equation of the lines passing through the point (1, 1) 4 p 16 4
whose distance from origin is 2 is m  2p 3
2
1 10 Now, tan Q  2 p 3 implies Q  75o or 15o.
(A) y 1  ( x 1)
3
Answer: (A), (B)
1 3
(B) y 1 ( x 1)
3 8. Let A(4, 3), B( 4, 3) and C(0, 5) be the vertices of a
1 3 triangle and P  (5, 0). Let L, M and N be the feet of
(C) y 1 ( x 1) the perpendiculars drawn from P onto the sides BC,
3
CA and AB, respectively. Then
1 10
(D) y 1  ( x 1) (A) the centroid of $LMN does not exist
3
(B) the orthocentre of $LMN is the origin
Solution: Let y  m(x 1) be the line so that by hy-
(C) the area of $LMN is 5 sq. unit
pothesis
(D) L, M and N are collinear
m( 1) 0 1 Solution: See Fig. 2.89. The sides BC, CA and AB are
2
1 m2 represented by the equations 2 x y 5  0, 2 x y 5  0
and y 3  0, respectively. We can see that L  ( 1, 3),
 (m 1)2  4(1 m2 )
M = (3, 1) and N  (5, 3). Now
 3m2 2 m 3  0
Slope of LM  Slope of MN  1
2 p 4 36 1 p 10
m  so that L, M and N are collinear.
6 3
Answer: (A), (D)
Worked-Out Problems 103

y Now, from Eq. (2.118)


2a 2b  ab  2A
B (−4, 3) A(4, 3) N
Therefore
90°
a bA (2.119)
Hence, from Eqs. (2.118) and (2.119), a and b are the
M roots of the equation x 2 A x 2A  0. Therefore
90° 2a 2b  2A  a b  A
O
P(5, 0) Hence a and b are roots of the equation
L
90°
x 2 A x 2A  0
Answer: (C), (D)
C

10. The points A(2, 1)and B(3, 2) are the two vertices
FIGURE 2.89 of $ABC and C lies on the line x – y 3  0. If the
area of $ABC is 5 sq. units, then the coordinates of
Note: See Theorem 1.1. the third vertex C are
Answer: (A), (D) ¥ 3 3 ´
(A) ¦ , µ (B) (0, 0)
§ 2 2¶
9. A line through (2, 2) and the axes form a triangle of
area A units. Then, the intercepts on the axes made ¥ 5 5´ ¥ 7 13 ´
(C) ¦ , µ (D) ¦ , µ
by the line are roots of the equation § 2 2¶ §2 2 ¶
(A) x 2 A x A  0 (B) x 2 A x A  0 Solution: Let C be (x, x 3). By hypothesis, we have
2 2
(C) x A x 2A  0 (D) x A x 2A 1
5 2( 2 x 3) 3( x 3 1) x(1 2) |
Solution: Let 2
x y  10  10 2 x 3 x 6 3 x |
1
a b  4 x 4  p10
be the equation of the line which forms a triangle with  4 x  14 or 6
the coordinate axes of area A sq. unit. Since the line 7 3
passes through (2, 2), we have  x  or
2 2
2 2 Therefore, the coordinates of the third vertex C are
1 (2.117)
a b
¥ 7 13 ´ ¥ 3 3 ´
¦§ , µ¶ or ¦§ , µ¶
1 2 2 2 2
We have ab  A so that
2 Answer: (A), (D)
ab  p2A (2.118)

Matrix-Match Type Questions


1. A( 2, 1), B(5, 4) and C(2, 3) are the vertices of
Column I Column II
$ABC. AD, BE and CF are the altitudes of the tri-
angle and M is the midpoint of BC. Match the items (C) Equation of the (r) 7 x 3 y 5  0
of Column I with those of Column II. median AM is
(s) x 11y 11  0
(D) Equation of the alti-
(t) 3 x 7 y 1  0
Column I Column II tude CF is
(A) Equation of AD is (p) x y 1  0
(B) Equation of BE is (q) x 11y 9  0
(Continued)
104 Chapter 2 Straight Line and Pair of Lines

Solution:
Column I Column II
(A) Slope of BC is (B) If A(1, 2), B(4, 6), C(5, 7) and (q) 1
4 3 7 S(a, b) are the vertices of a par-
 allelogram in the given order,
5 2 3
then the value of a b is
Therefore, the equation of the altitude AD is
¥ p r´
3 (C) If ¦ , µ is the centroid of
y 1 ( x 2) § q s¶
7 (r) 4
 3x 7 y 1  0 $ABC given in (B), then the
p r
Answer: (A) m (t) value of is
q s 1
(B) Slope of CA is
(D) Let p  lim lim cos2 n m P x (s) 3
1 3 nmd mmd
 1
2 2 where x rational and q  lim
nmd
Therefore, the equation of the altitude BE is lim cos2 m n x, where x is irra-
mmd
y 4  1( x 5) tional. Then the area of the tri-
 x y 1 0 angle with vertices (p, q), (2, 1)
(t) 5
and ( 2, 1) is
Answer: (B) m (p)
(C) The midpoint of BC is Solution:
(A) Since PRQ  90o, in general, the locus represented
¥ 7 1´
¦§ , µ¶ by R is a circle with P and Q as ends of the diam-
2 2
eter. Because area of $PQR is 2 sq. unit, there will
be four positions for R (two each in the two semi-
and the slope of the median AM is 1/ 11 so that
circles for which PQ is a diameter).
the equation of the median AM is
Answer: (A) m (r)
1
y 1 ( x 2) (B) It is known that a  1 5 4  2 and b  2 7 6  3.
11
Therefore a b  5.
 x 11y 9  0
Answer: (B) m (t)
Answer: (C) m (q)
(D) Lastly, the slope of AB is (C) Centroid ¥¦ 10 , 15 ´µ  p r  25  5
§ 3 3¶ q s 1 5
4 1 3 Answer: (C) m (t)

5 2 7
(D) We have
and hence the equation of the altitude CF is
p x (& m is even and cos m P  1 )
7
y 3 ( x 2) Similarly, q x. Since p x is rational and q x is
3 irrational, we have p q  0. Therefore, (p, q)  (0, 0).
 7 x 3y 5  0 Hence the area of the triangle is
Answer: (D) m (r) 1
| 2(1) ( 2)(1) |  2
2
2. Match the items of Column I with those of Column II. Answer: (D) m (p)

Column I Column II 3. In Column I, a family of concurrent lines is given and


their points of concurrence are given in Column II.
(A) If the line segment joining the (p) 2 Match the items of Column I with those of Column II.
points P(1, 3) and Q(5, 7) sub-
tends a right angle at a point R,
Column I Column II
such that the area of $PQR is
2 sq. unit, then the number of (A) If a, b, c are real as 2a 3b c  ¥ 2 3´
such points R is 0, then the lines ax by c  0 (p) ¦§ 5 , 5 µ¶
are concurrent at
(Continued)
(Continued)
Worked-Out Problems 105

Column I Column II Column I Column II


(B) If a is a parameter, then the (q) (2, 3) (B) The equation of the (q) x 4 y 24  0
family of lines (1 a)x (2 a) line passing through
y 5  0 is concurrent at the intersection of the
(C) The lines (a d)x ay (a d) (r) (1, 2) lines 3x 2y 10  0
 0 for different values of d are and 4x 3y 7  0 and
concurrent at also passing through
¥ 5 5 ´ the point (2, 1) is
(D) For different values m and n, (s) ¦§ 3 , 3 µ¶ (r) x 2 y 6  0
the lines (m 2n)x (m 3n)y (C) Equation of the line
m n  0 are concurrent at which passes through
(t) (1, 2) the point ( 2, 4) and
has sum of its inter-
Solution: cepts equal to 3 is
(s) x 3 y 5  0
(A) 2a 3b c  0  ax by c  0 passes through (D) A  (1, 2), B  (3, 4)
(2, 3). and C  (2, 7). Equa-
Answer: (A) m (q) tion of the line pass-
ing through A  (1, 2)
(B) The equation (1 a) x (2 a) y 5  0 is written as and perpendicular to
( x 2 y 5) a( x y)  0 , Hence, by Theorem 2.12, the line BC is (t) 4 x y 4  0
the line passes through the intersection of the lines
x 2 y 5  0 and x y  0 and the point of inter- Solution:
section is
(A) Any line passing through the intersection of the
¥ 5 5 ´ lines 2 x 5 y 3  0 and x 3 y 7  0 is of the
¦§ , µ
3 3¶ form
Answer: (B) m (s) (2 x 5 y 3) L ( x 3 y 7)  0 (see Theorem 2.20)
(C) The equation (a d) x ay (a d)  0 is written  ( 2 L ) x ( 5 3L ) y 3 7 L  0
as a( x y 1) d(1 x)  0 so that this line passes
through the intersection of the lines 1 x  0 and This line is perpendicular to the line
x y 1  0 which is given by (1, 2). 4x y 1  0
Answer: (C) m (t)
¥ 2 L ´
¦ ( 4)  1
(D) The equation (m 2 n) x (m 3n) y m n  0 is § 5 3L µ¶
written as m( x y 1) n(2 x 3 y 1)  0 . Hence,  8 4 L  5 3L
the line passes through the intersection of the lines
 L  3
x y 1  0 and 2 x 3 y 1  0 which is given by
Hence, the required line is
¥2 3´
¦§ , µ
5 5¶ x ( 4 y) 3 21  0
Answer: (C) m (p)  ( x 4 y) 24  0
Answer: (A) m (q)
3. Match the items of Column I with those of Column II.
(B) The required line equation is of the form
(3 x 2 y 10) L (4 x 3 y 7)  0. This passes
Column I Column II
through the point (2, 1) which implies that
(A) The equation of the (p) 22 x 25 y 69  0
[3(2) 2(1) 10] L (8 3 7)  0
line perpendicular to
4x y 1  0 and  14 4 L
passing through the 7
intersection of the L
2
lines 2x 5y 3  0
and x 3y 7  0 is Hence the required line is

(Continued)
106 Chapter 2 Straight Line and Pair of Lines

7 Column I Column II
(3 x 2 y 10) (4 x 3 y 7)  0
2
(B) If A( 5, 6), B( 1, 4) (q) 9 x y 3  0
 22 x 25 y 69  0
and C(3, 2) are the
 22 x 25 y 69  0 vertices of a triangle,
Answer: (B) m (q) then the equation of
the line passing
(C) Let the line be
through the centroid
x y and the circumcentre (r) 28 x 21y 12  0
1
a b of $ABC is
Therefore (C) The equation of the
line passing through
2 4 the point (1, 6),
1
a b whose product of the
(s) x 2 y 8  0
or 4a 2b  ab (2.120) intercepts on the axes
is 1, is
and a b 3 (2.121)
(D) The equation of the
Solving Eqs. (2.120) and (2.121), we have a  1, line whose x-intercept
b = 4 or a  6, b  3. is 3 / 7, and is per-
Case 1: a  1, b  4. The line is pendicular to the line (t) 4 x y 2  0
3x 4y – 10  0, is
x y
1
1 4 Solution:
 4x y 4  0 (A) Let the line be
Case 2: a  6, b  3. The line is x y
1
x y 2a a
1
6 3 Since it passes through the point (2, 3), we have
 x 2y 6  0 2 3
1
Answer: (C) m (r), (t) 2a a
(D) The slope of BC is  2 6  2a
a4
7 4
 3 Hence the required line equation is
2 3
x y
Hence, the equation of the line passing through 1
8 4
A(1, 2) and perpendicular to the line BC is
 x 2y 8  0
1
y 2  ( x 1) Answer: (A) m (s)
3
 x 3y 5  0 (B) We have G as the cenrtroid of $ABC which is
Answer: (D) m (s) given by ( 1, 4 / 3). Equation of the perpendicular
bisector of the side BC is
4. Match the items of Column I with those of Column II. 2 x 3y 1  0 (2.122)
Equation of the perpendicular bisector of the side
Column I Column II AB is
(A) Equation of the line (p) 2 x 33 y 46  0 2 x 5 y 11  0 (2.123)
through the point (2,
3), such that its Solving Eqs. (2.122) and (2.123), the circumcentre
x-intercept is twice its of $ABC is given by
y intercept, is ¥ 19 5 ´
¦§ , µ
(Continued) 8 4¶
Worked-Out Problems 107

¥ 19 5 ´ 4´ 6. Match the items of Column I with those of Column II.


S¦ , ¥
or and G  ¦ 1, µ
§ 8 4 µ¶ § 3¶
Column I Column II
Then the slope of the line SG is
(A) Equation of the line with (p) x 4y 8  0
(5 / 4) (4 / 3) 1 8 2 x-intercept 4 and passing
 s 
( 19 / 8) 1 12 11 33 through the point (2, 3) is
Hence, the equation of the line SG is (B) Equation of the line pass- (q) 3 x 2 y  12
ing through (4, 1) and form-
4 2
y  ( x 1) ing a triangle with positive
3 33 coordinate axes whose area
 2 x 33 y 46  0 is 8 sq. unit is
Answer: (B) m (p) (C) Equation of the line with (r) x y 7  0
equal intercepts on the
(C) Let the equation of the line be
axes and is passing through
x the point (2, 5) is
ay  1
a (D) Equation of the line which (s) 2 x y 1  0
This passes through (1, 6). This implies that makes an angle of 135o
with the positive direction
1 of the axis and makes an
6a  1
a intercept of 8 on y-axis is
 6a 2 a 1  0 (t) x y 8  0

 (3a 1)(2a 1)  0 Solution:


1 1 (A) Equation of the line is
a ,
2 3
x y
1
Therefore, the required lines are 4 b
1 1 It passes through (2, 3). This implies that
2 x y  1 and 3 x y  1
2 3
2 3 3 1
 4 x y 2  0 or 9 x y 3  0  1   b  6
4 b b 2
Answer: (C) m (q), (t)
Hence, the equation of the line is
(D) Let the required line be
x y
x y 1
1 7 x y 4 6
 1
( 3 / 7) b 3 b  3 x 2 y  12  0
Since this line is perpendicular to the line Answer: (A) m (q)
3 x 4 y 10  0, we have (B) Let
¥ 7b ´ ¥ 3 ´ 4 x y
¦§ µ ¦ µ  1  b  1
3 ¶§ 4 ¶ 7 a b
Therefore, the required line is It passes through (4, 1) and forms a triangle with
7 x 7 y positive axes having area 8. Therefore
1
3 4 4 1
1 (2.124)
 28 x 21y 12  0 a b
Answer: (D) m (r) 1
and (ab)  8 (2.125)
2
From Eqs. (2.124) and (2.125), a  8 and b  2.
Hence, the equation of the line is
108 Chapter 2 Straight Line and Pair of Lines

x y (B) By Theorem 2.31, we have


 1 or x 4 y 8  0
8 2 1
6 h
Answer: (B) m (p) 2
(C) Equation of the line with equal intercepts on the 5
h 6 0
axes is 2
x y 1 5
 1 or x y  a 1
a a 2 2
This passes through the point (2, 5) implies that a  7. ¥ 25 ´ ¥ 5 ´ 1 ¥ 5h ´
 6 ¦ 6 µ h ¦ h µ ¦ 3µ  0
Hence, the line is § 4¶ § 4¶ 2 § 2 ¶
x y 7  0 6 5h 5h 3
 h2 0
Answer: (C) m (r) 4 4 4 2
(D) Let the line be y  (tan 135o) x c where c  8. That  4 h2 10 h  0
is, y  x 8 or x y 8  0.  2 h(2 h 5)  0
Answer: (D) m (t)
5
 h  0 or h 
7. Match the items of Column I with those of Column II. 2
Since h x 0, we have 2 h  5.
Column I Column II Answer: (B) m (s)
(A) If 6x2 5xy 6y2 9x 20y c  (p) 8
(C) We have
0 represents a pair of lines, then
the value of c is equal to 7 13
12
(B) If 6x2 2hxy 6y2 x 5y 1  0 (q) 6 2 2
represent a pair of lines, then 2h 7 1
value (h x 0) is k 0
2 2
(C) If 12x2 7xy ky2 13x y 3  (r) 6 13 1
0 represents a pair of lines, then 3
2 2
the value of k is
(D) If d is the distance between the (s) –5 ¥ 1 ´ 7 ¥ 21 13 ´ 13 ¥ 7 13k ´
 12 ¦ 3k µ ¦ µ ¦ µ 0
parallel lines represented by the § 4¶ 2 § 2 4¶ 2 § 4 2 ¶
equation 9x2 24xy 16y2 12x 12(12k 1) 7 s 55 91 169k
16y 12  0, then the value of 5d is (t) 10  0
4 8 8 4
 288k 24 385 91 338k  0
Solution:
 50k  500
(A) The given equation represents pair of lines. By
 k  10
Theorem 2.31, we have
Answer: (C) m (t)
5 9
6 (D) If ax 2 2 hxy by2 2 gx 2 xy c  0 represents
2 2
a pair of parallel lines, then the distance between
5
6 10  0 them (see Problem 44 of the section ‘Subjective
2 Problems’) is
9
10 c
2 g 2 ac
2
5 ¥ 5c ´ 9 a(a b)
 6( 6c 100) ¦ 45µ (25 27)  0
2§ 2 ¶ 2
We have a  9, b  16, g  6 and c  12. Therefore
 24( 6c 100) 5(5c 90) 18 s 52  0
 169c 2400 450 936  0 g 2 ac 36 108 2 s 12 8
d2 2  
 169c  1014 a(a b) 9 s 25 3s 5 5
 c  6  5d  8
Answer: (A) m (r) Answer: (D) m (p)
Worked-Out Problems 109

8. Match the items of Column I with those of Column II. (B) If the angle between the lines is P, then

Column I Column II 2 h2 ab (49 / 4) 120 529 23


tan Q  2  
(A) The angle between the lines a b 12 10 4 2
P
joining the origin to the (p) 2 Hence
points of intersection of the
line 3 x y  2 and 7x2 4xy ¥ 23 ´
Q  Tan 1 ¦ µ
8y2 2x 4y 8  0 is § 2¶
(B) The angle between the lines (q) 1 Answer: (B) m (s)
represented by the equation (C) Suppose the line x 2 y 6  0 meets the given
12x2 7xy 10y2 13x y curve at points A and B. Hence, the combined
3  0 is equation of the pair of lines OA and OB is
(C) The angle between the lines P 2
joining the origin to the points (r) 3 ¥ x 2y´
2 x 2 6 xy 3 y2 (4 x 2 y) ¦
¥ x 2y´
36 ¦ 0
of intersection of the curve § 6 µ¶ § 6 µ¶
2x2 6xy 3y2 4x 2y 36 
In this equation, the coefficient of x2 the coeffi-
0 and the line x 2 y 6  0 is
cient of y2 is
(D) If the lines joining the origin (s) Tan 1 ¥¦ 23 ´µ
to the points of intersection § 2¶ ¥ 2 ´ ¥ 2 ´ ¥ 2´ ¥ 2 ´
¦§ 2 1µ¶ ¦§ 3 4µ¶  ¦§ 1 µ¶ ¦§ 1µ¶  0
of the curve 2x2 2xy 3y2 3 3 3 3
2x y   0 and the line
Hence
x 2 y  k are at right angles,
then the value of k is (t) 1 P
AOB 
2
Solution:
Answer: (C) m (p)
(A) Suppose the line 3 x y  2 meets the curve at the
points A and B. Then, by Theorem 2.33, the com- (D) The combined equation of the pair of lines is
bined equation of the pair of lines OA and OB is 2
¥ x 2y´ ¥ x 2y´
2 x 2 2 xy 3 y2 (2 x y) ¦ 0
¥ 3x y ´
2
¥ 3x y ´ § k µ¶ ¦§ k µ¶
7 x 2 4 xy 8 y2 (2 x 4 y) ¦ 8¦ 0
§ 2 µ¶ § 2 µ¶
Since the given two lines are at right angles, in the
In this equation, above equation, the coefficient of x2 the coeffi-
cient of y2  0. This implies
Coefficient of x2 Coefficient of y2
 (7 3 18) (8 2 2) ¥ 2 1´ ¥ 2 4´
¦§ 2 2 µ¶ ¦§ 3 2 µ¶  0
 8 8  0 k k k k
Hence  5k 2 5  0
P  k  p1
AOB 
2
Answer: (D) m (q), (t)
Answer: (A) m (p)

Comprehension-Type Questions
1. Passage: Consider the straight line 3x y 4  0. An- (A) x 3 y 4  0 (B) x 3y 5  0
swer the following questions.
(C) x 3y 4  0 (D) x 3 y 2  0
(i) The point on the line 3 x y 4  0 which is
equidistant from the points ( 5, 6) and (3, 2) is (iii) If the line y 5  k( x 3) is parallel to the given
line then the area of the triangle formed by this
(A) ( 1, 1) (B) ( 2, 2) line and the coordinate axes (in sq. units) is
1
(C) ( 3, 5) (D) ( , 3) 8 16
3 (A) (B) (C) 4 (D) 5
(ii) Equation of the line passing through the point 3 3
(1, 1) and perpendicular to the given line is
110 Chapter 2 Straight Line and Pair of Lines

Solution: (A) x y 1  0 (B) x y 3 0


(i) Let A  ( 5, 6) and B  (3, 2). The slope of AB is
(C) 2 x y 4  0 (D) 3 x y 1  0
6 2 1
 (iii) The locus of the feet of the perpendiculars from
5 3 2 the origin on each of the lines of the members of
and the midpoint of AB  ( 1, 4). Hence, the per- the family is
pendicular bisector of the segment AB is (A) (2 x 1)2 4( y 1)2  5
y 4  2( x 1) or 2 x y 6  0 . Solving this equa-
tion and the given line equations, we have x  2 (B) (2 x 1)2 ( y 1)2  5
and y  2. Thus, ( 2, 2) is the point on the given (C) (2 x 1)2 4( y 1)2  5
line which is equidistant from both A( 5, 6) and
B(3, 2). (D) (2 x 1)2 4( y 1)2  5
Answer: (B) Solution:
(ii) Line perpendicular to the given line is of the form (i) The slope of a line belonging to the given family is
1 3 4L
y x c
3 1 L
This line passes through (1, 1). It implies that and the slope of the line 3 x 4 y 2  0 is 3/4.
Therefore, by hypothesis, we have
1 2
1 c c
3 3 3 4L 3

1 L 4 13L 9
Thus, the required line is 1  tan 45o  
1 [3(3 4 L ) / 4(1 L )] 16 L 13
x 2
y or x 3 y 2  0
3 3 Therefore, 16 L 13  p(13L 9). Hence
Answer: (D)
4 22
L ,
(iii) The line y 5  k(n 3) is parallel to the given line 3 29
 k  3. That is,
Case 1: When we have
3x y  4
4
x y L
or 1 3
(4 / 3) 4
the slope is
Hence, the area of the triangle is
3 4 L 3 (16 / 3)
1 ¥ 4´ 8  7
1 L 1 (4 / 3)
¦§ µ¶ (4) 
2 3 3
Hence, the required equation is
Answer: (A)
4
(3 x y 1) (4 x y 2)  0
2. Passage: Consider the family of concurrent lines 3
which are concurrent at (1, 2) represented by the  7 x y 5  0
equation (3x – y – 1) K(4x – y – 2)  0, where L is a
parameter. Answer the following questions.  7x y 5  0

(i) A member of the family with positive slope which Case 2: When we have
makes an angle 45o with the line (3 x 4 y 2)  0 22
is L
29
(A) 7 x y 5  0 (B) 4 x 3 y 2  0 the slope is
(C) x 7 y 15  0 (D) 5 x 3 y 4  0 3 (88 / 29) 1

(ii) Equation of the line belonging to the given fami- 1 (22 / 29) 7
ly which is perpendicular to the line x y 1  0 Thus, the required line is 7 x y 5  0.
is
Answer: (A)
Worked-Out Problems 111

(ii) By hypothesis, we have x 1 y 3 2(1 3 2)


   Aa  ( 1, 1)
¥ 3 4L ´ 1 1 12 12
¦§ µ ( 1)  1
1 L ¶ Since the line BC is same as the line AaC, its equa-
2 tion is
L
3 1 (2 / 5)
y 1 ( x 1)
Hence, the required line is 1 (2 / 5)
2 7
(3 x y 1) (4 x y 2)  0  y 1 ( x 1)
3 3
 x y 1 0  7 x 3y 4  0
Answer: (A) Answer: (B)
(iii) Let A  (1, 2) at which the given family of lines are (ii) Solving the equations of the line BC and the angle
concurrent and O be the origin. If P is the foot of bisector of ABC , we have
the perpendicular from origin O onto any line of
¥ 5 9 ´
the family then P lies on the circle drawn on OA as B ¦ , µ
§ 2 2¶
diameter because APO is equal to 90o. The circle
with (0, 0) and (1, 2) as ends of a diameter is Answer: (D)

x( x 1) y( y 2)  0 (see Chapter 3) (iii) Since A  (1, 3) and B  ( 5 / 2, 9 / 2), the equation of


2 2
 x y x 2y  0 AB is 3 x 7 y 24  0.
2 2 Answer: (C)
 (2 x 1) 4( y 1)  5
Answer: (D)
4. Passage: Suppose a line lx my  1 meets a second-
degree curve ax2 2hxy by2 2gx 2fy c  0 at
3. Passage: In ΔABC, A  (1, 3) and C  ( 2 / 5, 2 / 5) two points A and B. Then, the combined equation of
are two vertices and x y – 2  0 is the equation of the pair of lines OA and OB is ax2 2hxy by2 (2gx
the internal bisector of ABC . Answer the following 2fy)(lx my) c(lx my)2  0 which is a second-
questions. degree homogeneous equation. Answer the following
(i) Equation of the side BC is questions.

(A) 7 x 3 y 4  0 (B) 7 x 3 y 4  0 (i) All chords of the curve 3 x 2 y2 2 x 4 y  0


which subtend right angle at the origin will pass
(C) 7 x 3 y 4  0 (D) 7 x 3 y 4  0
through a fixed point whose coordinates are
(ii) The coordinates of B are
(A) ( 1, 2) (B) (1, 2)
¥ 17 3 ´ (C) (1, 2) (D) ( 1, 2)
(A) ¦ , µ (B) (1, 1)
§ 10 10 ¶
(IIT-JEE 1991)
¥ 3 17 ´ ¥ 5 9 ´ (ii) If the intercept of the line lx my  1 made by
(C) ¦ , µ (D) ¦ , µ
§ 10 10 ¶ § 2 2¶ the curve x 2 y2 a 2  0 subtends right angle at
(iii) Equation of AB is the origin, then l 2 m2 is equal to

(A) 3 x 7 y 24  0 2 1
(A) (B) (C) 2a 2 (D) 3a 2
(B) 13 x 7 y 8  0 a2 a2
(C) 3 x 7 y 24  0 (iii) The line y  mx c makes an intercept on the
(D) 13 x 7 y 8  0 curve y 4ax  0 which subtends angle at the
origin. Then, the line y  mx c passes through
Solution: a fixed point whose coordinates are
(i) The image of A(1, 3) on the bisector of ABC lies
on the line BC. Therefore, if Aa is the image of (A) (2a, 0) (B) (a, 0)
A(1, 3) in the line x y 2  0, then (C) (3a, 0) (D) (4a, 0)
(IIT-JEE 1994)
112 Chapter 2 Straight Line and Pair of Lines

Solution:  a 2 (l 2 m 2 )  2
(i) Suppose lx my  1 is a line meeting the curve
3 x 2 y2 2 x 4 y  0 at points A and B. Therefore, 2
 l 2 m2 
the combined equation of the pair of lines OA and a2
OB is Answer: (A)

3 x 2 y2 (2 x 4 y)(lx my)  0 (iii) Suppose the line y  mx c, where c x 0. meets the


curve y2 4ax  0 at two points A and B. The com-
Since AOB  90o, in the above equation, the coef- bined equation of the pair of lines OA and OB is
ficient of x 2 the coefficient of y2  0. Therefore,
(3 2l ) ( 1 4m)  0 ¥ y mx ´
y2 4ax ¦ 0
§ c µ¶
 l 2m 1  0
Now, AOB  90o  the coefficient of x 2 the
Hence, the line lx my 1  0 passes through the
coefficient of y2  0. This means
point (1, 2).
Answer: (B) 4am
1 0
c
(ii) Suppose the line lx my  1 intersects x 2 y2  a 2
at points A and B. Therefore, the combined equa-  c  4am
tion of the pair of lines OA and OB is Therefore, the equation of the line is
2 2 2 2
x y  a (lx my)  0 y  mx c  mx 4am

Since AOB  90o , in the above equation, the coef-  y  m( x 4a)

ficient of x 2 the coefficient of y2  0. That is, which passes through the fixed point (4a, 0).
(1 a 2 l 2 ) (1 a 2 m2 )  0 Answer: (D)

Integer Answer Type Questions


1. The area of the quadrilateral formed by the lines 3. The orthocentre of the triangle formed by the lines
x y  1 is __________ sq. unit. x y  1, 2x 3y  6 and 4x – y 4  0 lies in the quad-
rant whose number is __________.
Solution: The given quadrilateral is a square with ver-
tices (1, 0), (0, 1), ( 1, 1) and (0, 1), and hence its area Solution: Solving the above equations taken two by
is ( 2 )2  2. two, the vertices of the triangle are
Answer: 2 ¥ 3 8 ´ ¥ 3 16 ´
A ¦ , µ , B  3, 4 , and C ¦ , µ
§ 5 5¶ § 7 7¶
2. Two rays in the first quadrant, x y  a and ax – y 
1, intersect each other in the interval a (a0 , d). The The equation of the altitude drawn from A to the side
value of a0 is __________. (IIT-JEE 2006) BC is
Solution: Solving the given two equations, we have 8 3¥ 3´
y  ¦x µ
1 a a(1 a ) a a 1 5 2§ 5¶
x and y  ax 1  1
1 a 1 a 1 a  3 x 2 y  5 (2.126)
Since the two rays intersect each other in the first quad- Again the equation of the altitude from B onto CA is
rant, we have x  0 and y  0 which implies that 1
y 4  ( x 3)
1 a  0 and a a 1  0 4
Therefore, if 1  a  0, then the a( a) 1  0 which is not  x 4 y  13 (2.127)
sensible. Hence, a Ž( 1, 0). If a  0, then the lines x y  0 Solving Eqs. (2.126) and (2.127), the coordinates of the
and y  1 intersect in fourth quadrant. Thus, a x 0. orthocentre are
Hence, a  0 and a 2 1  0  a  1. Therefore, a0  1.
Answer: 1
Worked-Out Problems 113

¥ 3 22 ´ 33 45
¦§ , µ¶ h and k 
7 7 7 14
which lies in quadrant number 1. Hence
Answer: 1 ¥ 33 45 ´
( x1 , y1 )  ¦ , µ
§ 7 14 ¶
4. If the lines 3x – 5y 9  0, 4x ky – 28  0 and 13x
8y 1  0 are concurrent, then the value of k is Solving Eqs. (2.128) and (2.129), we have
__________. 12 15
h and k 
Solution: Since the lines are concurrent, by Theorem 7 28
2.22, we have Thus
3 5 9 ¥ 12 15 ´
( x2 , y2 )  ¦ ,
4 k 28  0 § 7 28 µ¶
13 8 1 Therefore
 3( k 224) 5( 4 364) 9( 32 13k )  0 33 12 21
x1 x2   3
 120k 672 1800 288  0 7 7 7
 120k  840 Answer: 3
k7
7. The points A(0, 4), B(5, 1) and C(1, 3) are the ver-
Answer: 7 tices of a triangle. If h is the altitude from A to BC
and $ is the area of the triangle, then h2 /$ is equal to
5. If the slope of the line ax (3 – a)y 7  0 is 7, then __________.
the value of integral part of a is __________.
Solution: See Fig. 2.90. We have
Solution: By hypothesis, we have
1
$ 0(1 3) 5( 3 4) 1(4 1)  16
a 2
7
3 a
Length BC  (5 1)2 (1 3)2  4 2 . Therefore
 a  21 7a
1 1
7 h( BC )  $  (h)(4 2 )  16
a 2 2
2
Hence, h  (4 2 ) and h2  32. So
 ;a =  3
Answer: 3 h2 32
 2
$ 16
6. If (x1, y1) and (x2, y2) are two points on the line 5x
– 12y 5  0 that lie at a distance of 3 units from the A(0, 4)
line 3x 4y – 12  0, then the value of is x1 x2 is
__________.
Solution: Let (h, k) be a point on the line 5x 12y
h
15  0 whose distance from the line 3 x 4 y 12  0 is 3
units. Then

5h 12k  15 (2.128)
B(5, 1) C
3h 4k 12 (1, −3)
and 3
32 4 2 FIGURE 2.90

which gives that Answer: 2

3h 4k  27 (2.129) 8. If one of the lines represented by the equation mx2 –


or 3h 4k  3 (2.130) (1 – m2)xy – my2  0 bisects angles between the coor-
dinate axes, then m is equal to __________.
Solving Eqs. (2.128) and (2.129), we have
114 Chapter 2 Straight Line and Pair of Lines

Solution: The given equation is x2 m2xy xy my2 Since equation of BC is the equation of BAa, we have
 0. That is, mx( x my) y( x my)  0. Therefore, the the equation of BC as
lines are x my  0 and mx y  0 . The angle bisectors
of the coordinates are y  x and y  x . Hence, m  p1 1 (2 / 5)
y 1 ( x 2)
or m  1. 2 (9 / 5)
Answer: 1  y 1  3( x 2)
 3x y 5  0
9. Consider the lines ax y 1  0, x by 1  0 and
x y c  0, where a, b and c are distinct and differ- so that a  3, b  1. Hence, a b  4.
ent from 1. Then Answer: 4
1 1 1
11. If the equation of the bisector of the acute angle be-
1 a 1 b 1 c
tween the lines 2x – y 4  0 and x – 2y – 1  0 is
is equal to __________. ax by 1  0, then the value of a – b is equal to
Solution: Since the lines are concurrent, we have __________.

a 1 1 Solution: The lines are 2 x y 4  0 and x 2 y 1  0


in which c1 and c2 are positive and
1 b 1 0
1 1 c a1a2 b1b2  2 2  4  0

Using C 2 C1 and C 3 C1 (where C1 , C 2 and C 3 are Hence, by Theorem 2.26, the acute angle bisector is
used to denote the three columns), we have 2x y 4 ( x 2 y 1)

a 1 a 1 a 5 5
1 b 1 0 0  3x 3 y 3  0
1 0 c 1  x y 1 0
 a(b 1)(c 1) (1 a);c 1 0 = (1 a); 0 (b 1)=  0 Hence,
 a(1 b)(1 c) (1 a)(1 c) (1 a)(1 b)  0 a b  1 ( 1)  2
Dividing by (1 a)(1 b)(1 c), we get Answer: 2

a 1 1 12. The number of possible straight lines passing through


0
1 a 1 b 1 c (2, 3) and forming a triangle with coordinate axes
Adding both sides by 1, we get whose area is 12 sq. unit is __________.

1 1 1 Solution: Let
1
1 a 1 b 1 c x y
1
Answer: 1 a b
be the line. Therefore
10. In $ABC, the vertex A  (1, 2), y  x is the perpen-
dicular bisector of the side AB and x – 2y 1  0 is 2 3
1 (2.131)
the equation of the internal angle bisector of C. If a b
the equation of the side BC is ax by – 5  0, then the
1
value of a – b is __________. and ab  12
2
Solution: Since y  x is the perpendicular bisector of
or ab  p24 (2.132)
the side AB and A  (1, 2), we have B  (2, 1). Since the
image Aa( x, y) of A in the angular bisector x 2 y 1  0 Case 1: When ab  24, from Eq. (2.131), we have
lies on the line BC, we have 3a 2b  ab  24. Hence
x 1 y 2 2(1 2(2) 1) 4 ¥ 24 ´
   3a 2 ¦ µ  24
1 2 12 2 2 5 § a¶
Therefore,  3a 2 24a 48  0
¥ 9 2´  a 2 8a 16  0
Aa  ¦ , µ
§ 5 5¶  a  4, b  6.
Summary 115

Case 2: When ab  24, from Eq. (2.131), we have  a 2 8a 16  0


3a 2b  24 8 p 64 64
a  4 p 4 2
¥ 24 ´ 2
 3a 2 ¦ µ  24
§ a¶
Therefore, b will have two values corresponding to a.
2
 3a 24a 48  0 Hence, the number of lines is 3.
Answer: 3

SUMMARY
2.1. Slope of line: Let l be a non-vertical line (i.e., l is not 4. Intercept form: If a and b are x and y intercepts of a
parallel to y-axis) making an angle P with the posi- x y
tive direction of x-axis. Then, tan P is called the slope line (ab x 0), then the line equation is  1.
a b
of the line l. Generally, the slope of a line is denoted
by m. Note: Area of the triangle formed by the coordi-
x y 1
Caution: The concept of slope is followed only for nate axis and the line  1 is ab sq. unit.
a b 2
non-vertical lines.
5. Slope–intercept form: The equation of a non-
Note: Slope of a horizontal line (which is parallel to vertical line which is having slope m and
x-axis) is always zero. y-intercept c is
2.2. If A(x1, y1) and B(x2, y2) are two points on a non- y  mx c
FHHHE y y1
vertical line, then the slope of the line AB is 2 . Note: Equation of any line (except the y-axis)
x2 x1 passing through origin is the form y  mx.
6. Normal form: Let l be a line whose distance from
2.3. Intercepts on the axes: If a line l meets x-axis at
(a, 0) and y-axis at (0, b), then a is called x-intercept the origin is ON ( p) and ON make an angle @
and b is called y-intercept of the line l. with the positive direction of the x-axis. Then, the
equation of the line l is x cos @ y sin @  p.
2.4. Equations of the axis: The equation of x-axis is y  0
and the equation of y-axis is x  0. 2.6. Definition (first-degree equation): If a, b and c are
real and either a or b is not zero, then ax by c is
2.5. Various forms of straight line equations: called first-degree expression in x and y and ax by
c  0 is called first-degree equation in x and y.
1. Two-point form: Equation of the line passing
through two points (x1, y1) and (x2, y2) is 2.7. Theorem: Every first-degree equation in x and y rep-
(x – x1) (y1 – y2)  (y – y1) (x1 – x2) resents a straight line and the equation of any line in
the coordinate plane is a first-degree equation in x
2. Point–slope form: Equation of the line which is and y.
having slope m and passing through the point
(x1, y1) is 2.8. General equation of a straight line: First-degree
equation in x and y is called the general equation of
y – y1  m(x – x1) a straight line.
3. Symmetric form: If a non-vertical makes an angle
2.9. Various forms of ax by c  0, where abc x 0:
P with the positive direction of x-axis and passes
through a point (x1, y1), then its equation is 1. Slope–intercept form:
x x1 y y1 ¥ a ´ ¥ c ´
 y ¦ µ x ¦ µ
cos Q sin Q § b¶ § b¶
Note: In the above relation, if we consider that
2. Intercept form:
each ratio is equal to r (real number), then every
point on the line is of the form (x1 rcosP, y1 x y
rsinP). Also r gives the distance of the point (x, y) 1
( c / a) ( c / b)
on the given line from the fixed point (x1, y1).
116 Chapter 2 Straight Line and Pair of Lines

3. Normal form: m1 m2
(a) When c  0: tan Q 
1 m1 m2
¥ a ´ ¥ b ´ c
x¦ µ y¦ µ and the acute angle is given by
§ a 2 b2 ¶ § a 2 b2 ¶ a b2
2
m1 m2
tan Q 
(b) When c  0: 1 m1 m2
¥ a ´ ¥ b ´ c

2 2
µ y¦ 2 2
µ 2.15. Condition for parallel and perpendicular: Let
§ a b ¶ § a b ¶ a b2
2
a1x b1y c1  0 and a2x b2y c2  0 be two
lines. Then the following two conditions are
2.10. The line ax by c  0 in normal form: Write applicable:
ax by  –c. Divide by a 2 b2 both sides of the 1. The lines are parallel š a1b2  a2b1.
equation and then make the right-hand side (RHS) 2. The lines are perpendicular to each other if
positive. and only if a1a2 b1b2  0.
2.11. Distance of a line from a point: Suppose (x1, y1) 2.16. Condition for parallel and perpendicular in
is not a point on the line ax by c  0. Then, the terms of slopes: Let m1 and m2 be the slopes of
perpendicular distance drawn from (x1, y1) onto two lines. Then, the following two conditions are
the given line is applicable:
ax1 by1 c 1. The lines are parallel š m1  m2.
2. The lines are at right angles. š m1 m2  –1.
a 2 b2
2.17. Equation of the line parallel to the line ax by
In particular, if c x 0, then the distance of the origin c  0 and passing through the point (x1, y1) is
from the line is
a(x – x1) b(y – y1)  0
c
2.18. Equation of the line passing through the point
a 2 b2
(x1, y1) and perpendicular to the line ax by c
2.12. Distance between two parallel lines: The distance
 0 is
between the parallel lines ax by c  0 and b(x – x1) – a(y – y1)  0
ax by c a  0  0 is
(2.18)*. The area of the parallelogram formed by the
c ca lines a1x b1y c1  0, a1x b1y d1  0, a2x
2
a b 2 b2y c2  0 and a2x b2y d2  0 is

(c1 d1 )(c2 d2 )
2.13. Angle between two lines: If P is an angle between
the lines a1x b1y c1  0 and a2x b2y c2  0, a1b2 a2 b1
then
2.19. Notation: L y ax by c, L11  ax1 by1 c and
a1a2 b1b2 L22  ax2 by2 c.
cos Q 
a12 b12 a22 b22 2.20. Theorem: Let L  ax by c  0 be a straight
line and A(x1, y1) and B(x2, y2) be two points in
where P is acute or obtuse according to the condi- the plane of L  0. Suppose points A and B are
tions a1a2 b2b2  0 or  0. To determine the acute FHHHE
angle, we take not on the line and the line AB is not parallel
to the line L  0. Then, the line L  0 divides the
a1a2 b1b2 segment AB in the ratio –L11:L22.
cos Q 
a12 b12 a22 b22 1. Points A and B are on the opposite sides of
L  0 š the division is internal division š
2 14. Angle between the lines in terms of their slopes: If –L11:L22 is positive š L11 and L22 are of
P is an angle between the lines whose slopes are m1 opposite sign.
and m2, then 2. Points A and B are on the same side of
the line L  0 š the division is external
Summary 117

š – L11:L22 is negative š L11 and L22 are exist non-zero real numbers such that K1u1 K2u2
of same sign. K3u3  0, then the lines u1  0, u2  0 and u3  0 are
concurrent.
2.21. Origin and non-origin sides of a line: Suppose l is a
straight line which is not passing through the origin. 2.29. Theorem (Foot of the perpendicular): Let (x, y) be
Then, l divides the entire plane into two regions. the foot of the perpendicular drawn from the point
Origin region means, the region in which the origin (x1, y1) on to the line ax by c  0, then
lies. The other is called non-origin region.
x x1 y y1 (ax1 by1 c)
 
2.22. To determine the position of a point: Let L y ax a b a 2 b2
by c be a straight line where c x 0 and A(x1, y1)
x (0, 0) be a point which does not lie on the line 2.30. Theorem (Image): If (x, y) is the image of (x1, y1) in
L  0. Then the mirror line ax by c  0, then
1. A(x1, y1) lies on the origin side of L  0 š c and x x1 y y1 2(ax1 by1 c)
L11 has the same sign.  
a b a 2 b2
2. A(x1, y1) lies on the non-origin side of L  0 š
c and L11 has the opposite sign. Note: From 2.29 and 2.30, we can write the foot of
the perpendicular and the image of a point with
2.23. Let L y ax by c  0 be a line where c x 0. Then
respect to a line.
1. if c  0, then L11  0 for all points on the origin
side and L11  0 for all points on non-origin side. 2.31. Angle bisectors: If a1x b1y c1  0 and a2x b2y
2. if c  0, then L11  0 for all points on the origin c2  0 are two intersecting lines, then the equations
side and L11  0 for all points in the non-origin of their angle bisectors are
side.
a1 x b1 y c1 (a2 x b2 y c2 )
p
2.24. Theorem: Suppose u1  0 and u2  0 are two parallel a12 b12 a22 b22
lines. Then, K1u1 K2u2  0 represents lines parallel
to the lines u1  0 and u2  0 for all real values of K1
2.32. Acute angle bisector: Suppose a1x b1y c1  0
and K2 such that L1 L 2 x 0.
and a2x b2y c2  0 where c1c2 x 0 and c1, c2 are
2.25. Theorem: If u1  0 and u2  0 are two intersecting of the same sign. If a1a2 b1b2  0, then the acute
lines, then for all K1 and K2 ( L1 L 2 x 0), the angle bisector is
equation K1u1 K2u2  0 represents the lines
a1 x b1 y c1 ¥a x b y c ´
passing through the intersection of u1  0 and u2
 ¦ 2 2 2
µ
 0. Conversely, the equation of any line passing a12 b12 ¦§ a2 b2 µ¶
2 2
through the intersection of u1  0 and u2  0 is K1u1
K2u2  0 for some K1 and K2.
2.33. Theorem: The second-degree homogeneous
Note: Instead of K1u1 K2u2  0, we consider the equation ax2 2hxy by2  0 represents a pair
equation u1 Ku2  0 which is practically more of lines passing through the origin, where h2 r ab.
useful. If h2  ab, both lines coincide, otherwise they are
distinct lines.
2.26. Corollary: Suppose u1  0 and u2  0 are the two
intersecting lines. Then, the equation of any line in 2.34. Identities:
the plane of u1  0 and u2  0 is of the form K1u1 1. Suppose the lines represented by ax2 2hxy
K2u2 K3 0. by2  0 are l1x m1y  0 and l2x m2y  0. Then
2.27. Theorem: When the lines a1x b1y c1  0, a2x l1l2  a, l1m2 l2m1  2h, m1m2  b
b2y c2  0 and a3x b3y c3  0 are concurrent
2. If we consider the lines as y  m1x and y  m2x,
(assuming that no two lines are parallel), then
then
a1 b1 c1
2 h a
a2 b2 c2  0 m1 m2  and m1 m2 
b b
a3 b3 c3
2.35. Angle between the lines ax2 2hxy by2  0: If @
2.28. Theorem: Suppose u1  0, u2  0 and u3  0 are is the angle between the lines represented by the
three lines such that no two are parallel. If there equation ax2 2hxy by2  0, then
118 Chapter 2 Straight Line and Pair of Lines

a b lines parallel to the lines represented by ax2 2hxy


cos A  by2  0 if and only if
(a b)2 4 h2 (i) $  abc 2fgh – af 2 – bg2 – ch2  0
(ii) h2 r ab, g2 r ca and f 2 r bc
2 h2 ab
or tan A 
a b 2.40. Formula: The area of the parallelogram formed by
the pairs of lines ax2 2hxy by2 2gx 2fy c 
To determine the acute angle, take the absolute
0 and ax2 2hxy by2  0 is
value.
c
2.36. Condition for orthogonal lines: Suppose ax2 sq. unit
2
2hxy by2  0 represents pair of straight lines. 2 h ab
Then, they are at right angles if and only if a b  FHHE FHHE
0 (that is, coefficient of x2 coefficient of y2  0). 2.41. Equally
FHHEinclined
FHHE pairs: The pair of lines ( PA, PB)
and ( PC , PD) are said to be equally inclined to
2.37. Equation of the angle bisectors of the lines each other if both pairs have the same pair of angle
ax2 2hxy by2  0: If ax2 2hxy by2  0 bisectors.
represents a pair of distinct lines, then the
combined equation of the pair of angle bisectors 2.42. Theorem: Suppose the line lx my  1 intersects
of the lines is h(x2 – y2)  (a – b)xy. the second-degree curve ax2 2hxy by2 2gx
2fy c  0 at points A and B.FThen,
HHE theFHH
combined
E
2.38. Theorem: The area of the triangle formed by equation of the pair of lines OA and OB, where
the pair of lines ax 2 2hxy by2  0 and the line O is the origin, is given by the equation
lx my  1 is
ax2 2hxy by2 (2gx 2fy) (lx my)
h2 ab c(lx my)2  0
bl 2 2 hlm am2 In particular, OA and OB are at right angles if
Coefficient of x2 Coefficient of y2  0
2.39. Theorem: The second-degree general equation ax2
in the above equation.
2hxy by2 2gx 2fy c  0 represents a pair of

EXERCISES
Single Correct Choice Type Questions
1. Equation of the line through (0, 3) and having slope ¥ 19 5 ´
, ¥ 19 5 ´
(A) ¦ (B) ¦ , µ
2 is § 8 4 µ¶ § 8 4¶
(A) y 2 x 3  0 (B) y 2x 3  0
¥ 5 19 ´ ¥ 5 19 ´
(C) ¦ , µ (D) ¦ ,
(C) y 2x 3  0 (D) y 2 x 3  0 § 4 8¶ § 4 8 µ¶

2. Equation of the line passing through ( 5, 2) and (3, 2) 5. If the area of the triangle formed by the line 2x
is 3y c  0 with coordinate axes is 27 sq. units, then
c is equal to
(A) x 2  0 (B) y 2  0
(C) x 2  0 (D) y 2  0 (A) p16 (B) p15 (C) p8 (D) p18

6. If the line 2x 3by – 13  0 passes through the point


3. The points A( 5, 6), B( 1, 4) and C(3, 2) are three
non-collinear points. Then, the equation the median ( 2, 4), then the value of b is
through point C of $ABC is 5 4 17 19
(A) (B) (C) (D)
(A) 7 x 6 y 1  0 (B) x 6y 9  0 4 3 12 12
(C) x 6 y 9  0 (D) x 6 y 9  0 7. In the straight line equation xcos @ ysin @  p, if
p  6 or A  30o, then the equation is
4. The circumcentre of the triangle with vertices A( 5, (A) 3 x y 12  0 (B) 3 x y 12  0
6), B( 1, 4) and C(3, 2) is
(C) 2 x 3 y 12  0 (D) 2 x 3 y 12  0
Exercises 119

8. If the line 3x – by – 8  0 makes angle 45o with the (A) 2 x y 5  0 (B) 2 x y 5  0


positive directions of the x-axis, then b is equal to (C) 2 x y 5  0 (D) 2 x y 5  0
9 9
(A) 7, (B) 7, 16. B(2, 0) and C(0, 1) are the ends of the base of an
7 7
isosceles triangle for which the line x  2 is one side.
9 9
(C) 7, (D) 7, Then, the orthocentre of the triangle is
7 7
¥3 ´ ¥4 7 ´
9. The incentre of the triangle whose sides are y  0, 3x – (A) ¦ , 1µ (B) ¦ , µ
§4 ¶ § 3 12 ¶
4y  0 and 4x 3y – 50  0 is
¥ 3 3´ ¥5 ´
¥ 15 5 ´ ¥ 15 5 ´ (C) ¦ , µ (D) ¦ , 1µ
(A) ¦ , µ (B) ¦ , µ § 2 2¶ §4 ¶
§ 2 2¶ § 2 2¶

¥ 15 5 ´ ¥ 15 5 ´ 17. In $ABC , A  (1, 10), circumcentre ( 1/ 3, 2 / 3) and


(C) ¦ , (D) ¦ ,
§ 2 2 µ¶ § 2 2 µ¶ orthocentre (11/ 3, 4 / 3). Then, the coordinates of the
midpoint of BC are
10. The incentre of the triangle formed by the lines 15x –
¥ 11´
8y 25  0, 3x – 4y –10  0 and 5x 12y – 30  0 is (A) (1, 5) (B) ¦ 1, µ
§ 3 ¶
¥ 4 1´ ¥ 4 1 ´ (C) (1, 6) (D) (1, 3)
(A) ¦ , µ (B) ¦ , µ
§ 7 4¶ § 7 4¶
Hint: The centroid G divides the line joining the cir-
¥ 4 1´ ¥ 4 1´ cumcentre and orthocentre in the ratio 1:2.
(C) ¦ , µ (D) ¦ , µ
§7 4 ¶ § 7 4¶
18. A rhombus is situated in the first quadrant with x – y
11. The equation of the lines which are parallel to the  0 and 7x – y  0 as two of its adjust sides. Then the
line 8x – 15y 34  0 and whose distance from the slope of the longer diagonal of the rhombus is
point ( 2, 3) is equal to 3 are 8x – 15y c1 and 8x – 1 1
15y c2  0. Then c1 c2 is equal to (A) 2 (B) (C) (D) 2
2 2
(A) 112 (B) 10 (C) 122 (D) 102
19. The distance of the line 2x – 3y – 4  0 from the
12. The equation of the line which passes through the point (1, 1) measured in the direction of the line x
point of intersection of the lines 3x – 5y 9  0 and y – 1  0 is
4x 7y – 28  0 and the point (4, 2) is 1
(A) (B) 2 (C) 5 2 (D) 2 2
2
(A) 3 x 2 y 16  0 (B) 38 x 87 y 326  0
20. The line parallel to x-axis and passing through the
(C) 4 x 3 y 22  0 (D) 38 x 87 y 22  0 intersection of the lines ax 2by 3b  0 and bx –
2by – 3a  0 where (a, b) ≠ (0, 0) is
13. A( 1, 4), B(1, 4) and C (5, 4) are the vertices of a tri-
angle. Then, the length of the altitude from A onto (A) above the x-axis at a distance of 3/2 units
BC is from it
12 12 12 (B) above the x-axis at a distance of 2/3 units
(A) (B) (C) (D) 3 from it
5 5 5 5
(C) below the x-axis at a distance of 3/2 units
14. A point moves such that its distance from the point from it
( 1, 2) is always equal to its distance from the line (D) below the x-axis at a distance of 2/3 units
3x 4y 2 = 0. Then the locus of the point is from it
(4x 3)2 ax by 121  0
21. The L1 y4x 3y – 12  0 intersect x-axis at A and
where a b is equal to y-axis at B. A variable line L2 perpendicular to L1
(A) 178 (B) 116 (C) 54 (D) 121 intersects x-axis at P as y-axis at Q. Then, the perpen-
dicular circumcentre of $ABC lies on
15. The equation of the line through the point of inter-
(A) 4 x 3 y 7  0 (B) 6 x 8 y 7  0
section of the lines x – 3y 1  0 and 2x 5y – 9  0,
and whose distance from the origin is 5 is (C) 3 x 4 y 2  0 (D) 3 x 4 y 2  0
120 Chapter 2 Straight Line and Pair of Lines

22. If the lines ax – y 4  0, 3x – y 5  0 and x y 8  0 the origin onto the line lies on the curve whose equa-
are concurrent, then the value of a is tion is given by
35 5 15 (A) x 2 y2 A x B y  0
(A) (B) (C) 35 (D)
13 13 13
(B) x 2 y2 A x B y  0
23. The lines x2 4xy y2  0 and x – y – 4  0 form a
(C) x 2 y2 A x B y  0
triangle which is
(D) x 2 y2 A x B y  0
(A) equilateral
(B) right angled 27. 3x2 – 8xy – 3y2 10x 20y – 25  0 are the bisectors
(C) an isosceles of the angles between two lines of which one line is
(D) isosceles right angled passing through the origin. Then, the equation of the
other line is
24. Equation of the line which is parallel to the common (A) x 2 y  0 (B) 2 x y  0
line of the pair of lines 6x2 – xy – 12y2 and 15x2 14xy
(C) x 2 y 5  0 (D) x 2 y 5  0
– 8y2  0 and whose distance from this common line
is 7 units is
28. The straight line 2x 3y 1  0 bisects the angle be-
(A) 3 x 4 y  p35 (B) 5 x 2 y  p7 tween two straight lines of which one line is 3x 2y
(C) 3 x 4 y  p35 (D) 2 x 3 y  p7 4  0. Then, the equation of the other line is
(A) 3 x 23 y  28 (B) 9 x 4 y  28
25. The point (4, 1) undergoes the following three trans-
(C) 9 x 46 y  28 (D) 9 x 46 y  28
formations successively:
I. Reflection about the line y x. 29. A(1, 3) and B(5, 2) are two points. If P is a variable
II. translation through a distance 2 units along the points on the line y x, then the minimum value of
positive direction of the x-axis. PA PB is
III. Rotation through an angle O/4 about the origin
in the counterclockwise direction. (A) 2 5 (B) 3 5 (C) 4 5 (D) 5
Then, the final position of the point is given by
Hint: If Aa is the image of A on the line y x, then
¥ 1 7 ´ AaB is the value.
(A) ¦ , (B) ( 2 , 7 2 )
§ 2 2 µ¶
30. The equation of the line passing through the point of
1 7 ´
(C) ¥¦ , µ (D) ( 2 , 7 2 ) intersection of the lines x y 1  0 and 3x y 5 
§ 2 2¶ 0 and is perpendicular to the line x 3y 1  0 is
26. A straight line is passing through a fixed point (A) x 3y 1  0 (B) 3x y 1  0
(@, B ). Then, the foot of the perpendicular drawn (C) x 3y 1  0 (D) 3x y 1  0

Multiple Correct Choice Type Questions


1. If the distance of the line 8x 15y K  0 from the 3
point (2, 3) is equal to 5 units, then the value of L is (A) the slope of l 
2
(A) 24 (B) 24 (C) 146 (D) 146 (B) the line l passes through (0, 0)
(C) the intercept on the axes are 2, 3
2. If the line 3 x y – 9  0 is reduced to the form xcos@ (D) the line l forms a triangle of area 5 sq. units with
ysin @  p, then the coordinate axes
(A) A  60o (B) A  30o
4. If l is the line passing through the point (2, 3) and is
9 parallel to the line joining the points (4, 1) and ( 2, 3),
(C) p  (D) p  9
2 then

3. If l is the line passing through the point ( 2, 3) and (A) ( 10, 1) is a point on l
perpendicular to the line 2x 3y 6  0, then (B) the slope of l is 6
Exercises 121

(C) the area of the triangle formed by the line l and (D) a straight line parallel to y-axis
the coordinate axes is 64/3
8. Two points A(–1, –1) and B(4, 5) and the third vertex
(D) the orthocentre of the triangle formed by l and
C lie on the line 5x – y – 15  0. If the area of the tri-
the axes is (1, 1)
angle is 19/2, then the coordinates of the vertex C are
5. If S is the family of lines passing through the point (A) (2, –5) (B) (5, 4) (C) (3, 0) (D) (5, 10)
(5, 0), then
9. The line
(A) the line belonging to S and having slope 2 is
2x y 5 0 x y
1
(B) the line belonging to S and having y-intercept 5 c d
is x y 5  0 passes through the intersection of the lines
(C) the line belonging to S and perpendicular to the x y x y
line x y 1  0 is x y 5 0  1 and  1
a b b a
(D) the line belonging to S and perpendicular to the
and the lengths of the perpendicular drawn from
line x y 1  0 is x y 5 0
the origin onto these lines are equal. Then
6. A line l is passing through the point (1, –6). If the 1 1 1 1
(A) 2
2  2 2
product of the intercepts of l on the axes is 1, then the a b c d
equation of l is 1 1 1 1
(B) 
(A) 2x y 5  0 (B) 2x y 4  0 a b c d
(C) 4x y 2  0 (D) 9x y 3  0 1 1 1 1
(C) 2
2  2 2
a b c d
7. The graph of the function 1 1 1 1
(D) 2 2 2 2  2
y  cosx cos(x 2) – cos2(x 1) is a b c d
(A) a straight line passing through (0, sin 2 1) 10. The straight lines 3x y – 4  0, x 3y – 4  0 and x
¥P ´ y  0 form a triangle which is
(B) a straight line passing through ¦ , sin 2 1µ
§2 ¶ (A) obtuse angled (B) equilateral
(C) a straight line parallel to x-axis (C) isosceles (D) right-angled

Matrix-Match Type Questions


In each of the following questions, statements are given in 1. Let S be the system of lines passing through the
two columns, which have to be matched. The statements in intersection of the lines x y – 1  0 and x – y – 1 
column I are labeled as (A), (B), (C) and (D), while those 0. Match the items of Column I with those of Col-
in column II are labeled as (p), (q), (r), (s) and (t). Any umn II.
given statement in column I can have correct matching with
one or more statements in column II. The appropriate bub- Column I Column II
bles corresponding to the answers to these questions have
(A) Equation of the line (p) 2x – y – 2  0
to be darkened as illustrated in the following example.
belonging to S and
Example: If the correct matches are (A) m (p), (s), (B) m passing through the
(q), (s), (t), (C) m (r), (D) m (r), (t), that is if the matches point (2, 3) is
are (A) m (p) and (s); (B) m (q), (s) and (t); (C) m (r); (B) Equation of the line (q) x y – 1  0
and (D) m (r), then the correct darkening of bubbles will belonging to S and
look as follows: parallel to the line y 
p q r s t 2x 1 is
A
(r) x y 1  0
(C) Equation of the line
B belonging to S and
having equal inter-
C
cept (absolutely) is
D
(Continued)
122 Chapter 2 Straight Line and Pair of Lines

Column I Column II 3. Match the items of Column I with those of Column II.
(D) Equation of the line (s) 3x – y – 3  0
belonging to S and Column I Column II
perpendicular to the (t) x – y – 1  0 (A) Equation of the line whose (p) x y 8 
line x y – 1  0 is x-intercept is 4 and passes 0
though the point (2, –3)
2. Match the items of Column I with those of Column II.
(B) Equation of the line having (q) x y  7
equal intercept on the axes
Column I Column II
and passing through the
(A) If the lines x 2ay a  0, (p) GP point (2, 5) is
x 3by b  0 and x 4cy
(C) Equation of the line which (r) 3x 2y  12
c  0 are concurrent, then
makes an angle of 135o with
a, b and c are in
the positive direction of the
(B) If the lines ax by (ak b) (q) HP x-axis and which cuts the
= 0, bx cy (bk c)  0 and y-axis at a distance of 8 units
(ak + b)x + (bk + c)y = 0 are from the origin is (s) x 4y  8
concurrent and b2 x ac, (D) Equation of the line through
then the point (4, 1) which forms
(C) If the lines ax 2y 1  0, (r) k is a root of a triangle of 8 sq. unit with
bx 3y 1  0 and cx 4y ax2 2bx c  positive axes is (t) x y 8  0
1  0 pass through a fixed 0
point, then a, b and c form
(D) If a, b and c are distinct (s) AP
positive and the lines a(x
y) c  0, x 1  0 and (t) k is a root of
c(x y) b  0, then a, c ax2 2bx c 
and b are in 0

Comprehension Type Questions


1. Passage: Let u y x y  0, A  (1, 2) and B  (3, –1). (i) Area of the triangle in square units is
Answer the following questions.
1 2 1 1
(A) (B) (C) (D)
(i) If M is a point on the line u  0 such that AM 3 3 2 3 2 2
BM is minimum, then the reflection of M on the
line y x is (ii) The gradients of the two sides AB and AC are
(A) (2, –2) (B) (–2, 2) 1 1
(A) 3, (B) 2,
(C) (1, –1) (D) (–1, 1) 3 2
(ii) If M is a point on u  0 such that AM BM is (C) 2 1, 2 1 (D) 2 3 , 2 3
maximum, then the distance between M and the (iii) The circumradius of the triangle is
point N(1, 1) is
1 2 1 1
(A) 3 5 (B) 5 2 (C) 7 (D) 10 (A) (B) (C) (D)
3 3 3 2
(iii) If M is a point on u  0 such that AM BM is
minimum, then the area of $ABM is equal to 3. Passage: If u 0 and u2 0 are two intersecting lines,
then for all values of K and L, the equation Ku Lu2
13 13 13 13
(A) (B) (C) (D) represents straight lines passing through the intersec-
8 6 2 4 tion of the lines u  0 and u2  0. In particular, the
equation u Ku2  0 represents all lines (except u2 
2. Passage: ABC is an equilateral triangle with vertex 0) passing through the intersection of u  0 and u2 
A  (1, 1) and the equation of the side BC is x y  1. 0. The converse of these are also true. Answer the fol-
Answer the following questions. lowing questions.
Answers 123

(i) The line (K 1)2x Ky – 2K2 – 2  0 passes through distance from the point Q(2, –3) is 10 is
a fixed point. The equation of the line passing
through this fixed point and having slope 2 is (A) d (B) 1 (C) 2 (D) 0
(A) 2x – y 8  0 (B) 2x – y – 5  0 (iii) In (ii), if L is the required line, then the image of
(C) 2x – y – 8  0 (D) 2x – y – 4  0 the point Q(2, –3) in the line is

(ii) Consider the family of lines p(2x y 4) q(x (A) (4, 1) (B) (–4, 1)
– 2y – 3)  0 (p and q are parameters). The num- (C) (4, –1) (D) (–4, –1)
ber of lines belonging to this family and whose

Integer Answer Type Questions


The answer to each of the questions in this section is a 4. In $ABC, the equations of the madians AD and BE,
non-negative integer. The appropriate bubbles below the respectively, are 2x 3y – 6  0 and 3x – 2y– 10  0.
respective question numbers have to be darkened. For 1
If AD  6, BE  11, then (Area of $ABC ) is
example, as shown in the figure, if the correct answer to 11
________.
the question number Y is 246, then the bubbles under Y
labeled as 2, 4, 6 are to be darkened.
5. P(1, 2), Q(4, 6), R(5, 7) and S(a, b) are the vertices
X Y Z W of the parallogram PQRS. Then, a b is equal to
0 0 0 0 _______.
1 1 1 1
6. The area of the triangle formed by the line x
2 2 2
y  3 and the angle bisectors of the pair of lines
3 3 3 3
x 2 y2 2 y 1  0 is ______ sq. unit.
4 4 4
5 5 5 5 7. A straight line through the origin O meets the par-
6 6 6 allel lines 4x 2y  9 and 2x y  6 at points P and
7 7 7 7 Q, respectively. If O divides the segment PQ in the
8 8 8 8 ratio p:q, then the value of p q is _____.
9 9 9 9
8. If a, b and c are real such that 3a 2b 4c  0, then
the line ax by c  0 passes through a fixed point (h,
1. If (a, b) is the image of the point (2, –3) on the line k) where [h k] ([–] is the usual symbol) is _______.
3x y – 1  0, then b – a is equal to _____.
9. The number of integral values of m, for which the
2. If L is the line belonging to the family of lines repre- x-coordinate of the point of intersection of the lines
sented by the equation (2x y 4) L (x 2y 3)  3x 4y 9  0 and y  mx 1 is also an integer is
0 (where L is a parameter) whose distance from the _______.
point (2, –3) is 10 units, then the slope of the line L
is ______. 10. P(m, n) is an interior point (where m and n are posi-
tive integers) of a quadrilateral formed by the lines
3. If the lines ax by 5  0 and px qy 1  0 are the di- y  0, x  0, 2x y 2  0 and 4x 5y 20  0. The
agonals of the parallogram whose sides are 2x y 7  possible number of positions of P is _____.
1
0, 3x 2y – 5  0 and 3x 2y 4  0, then the value of
8
(a b p q) is ________.

ANSWERS
Single Correct Choice Type Questions
1. (C) 3. (C)

2. (B) 4. (A)
124 Chapter 2 Straight Line and Pair of Lines

5. (D) 18. (A)


6. (C) 19. (B)

7. (A) 20. (C)

8. (D) 21. (B)

9. (B) 22. (A)

10. (A) 23. (A)

11. (C) 24. (C)

12. (B) 25. (C)


26. (A)
13. (B)
27. (C)
14. (A)
28. (D)
15. (C) 29. (D)
16. (D) 30. (B)

17. (B)

Multiple Correct Choice Type Questions


1. (A), (D) 6. (C), (D)
2. (B), (C) 7. (A), (B), (C)
3. (A), (B) 8. (C), (D)
4. (A), (C) 9. (A), (B)
5. (B), (D) 10. (A), (C)

Matrix-Match Type Questions


1. (A) m (s); (B) m (p); (C) m (q), (t); (D) m (t) 3. (A) m (r); (B) m (q); (C) m (p), (t); (D) m (s)

2. (A) m (q); (B) m (r); (C) m (s), (t); (D) m (p)

Comprehension Type Questions


1. (i) (D); (ii) (D); (iii) (D) 3. (i) (C); (ii) (B); (iii) (D)
2. (i) (C); (ii) (D); (iii) (B)

Integer Answer Type Questions


1. 3 6. 2
2. 3 7. 7
3. 5 8. 2
4. 4 9. 2
5. 5 10. 5
Circle 3
Contents
3.1 Introduction
3.2 Relation Between a Circle and a
Line in its Plane
3.3 Classification of Points in a
Plane w.r.t. a Circle in the Same
Plane
d 3.4 Relation Between Two Circles
Chor
3.5 Common Tangents to Two
Circles

r Worked-Out Problems
Diamete
Summary
Centre Exercises
T
Tangent
Answers

Arc

A circle is a simple shape of Euclid-


ean geometry consisting of those
points in a plane that are equidistant
Arc from a given point, the centre. A
circle can be defined as the curve
traced out by a point that moves so
Sector that its distance from a given point is
constant.
t
en
gm
Se
Circle
126 Chapter 3 Circle

In the previous chapter, we discussed about the straight line and pair of lines. It is known that a straight line is repre-
sented by first-degree equation in x and y and hence it is called first-degree curve. Curves represented by second-
degree equation in x and y are called second-degree curves. Some of the second-degree curves are pair of lines (studied
in the previous chapter), circle and conics. Among these second-degree curves, the circle has been known since ancient
times and has some special properties. In this chapter, we study the general equation of a circle, equation of the tangent
at a point, chord equation interval of its midpoint, chord of contact, orthogonal circles, etc. Subjective Problems have
been provided for the preceding sections. Students are advised to solve each and every problem to grasp the topics.

3.1 Introduction
We begin with the following definition.

DEFINITION 3.1 Circle Let A be a fixed point in a plane and r  0 a given real number. Then the locus of the point
P such that the distance AP is equal to r is called a circle with centre A and radius r (Fig. 3.1).

r P
A

FIGURE 3.1 A circle.

We translate the definition of the circle, and obtain the equations of a circle.

T H E O R E M 3.1 The equation of the circle with centre at the point A(h, k) and radius r is

( )2 ( )2  r 2

PROOF P(x, y) is a point on the given circle š(AP)2  r2 š(x − h)2 (y − k)2  r2, because the distance

AP  ( x h)2 ( y k )2

Note: If h  0, k  0 (i.e., origin is the centre), then the equation of the circle is x2 y2  r2.

QUICK LOOK 1

1. Equation of the circle with centre at origin and ra- is in the form x2 y2 2gx 2fy c  0, where g, f
dius r is x2 y2  r2. and c are real numbers.
2. Equation of the circle with centre at (h, k) and ra-
dius r is x2 y2 − 2hx − 2ky h2 k2 − r2  0, which

T H E O R E M 3.2 If g, f, c are real numbers and g2 f 2 – c  0, then the equation x2 y2 2gx 2fy c  0 represents
circle with centre at (−g, −f ) and radius g2 f 2 c.
2
PROOF x2 y2 2gx 2fy c  0 can be written as ( )2 ( )2   g f c which represents
circle with centre (−g, −f ) and radius g2 f 2 c according to Theorem 3.1.
3.1 Introduction 127

Note:
1. The locus represented by the equation (x − h)2 (y − k)2  0 is the single point (h, k) which is called point circle. In
fact, if the radius of a circle is zero, then it is called point circle.
2. If g2 f 2 – c  0, then the locus represented by x2 y2 2gx 2fy c  0 is the empty set and it represents point
circle if g2 f 2 – c  0. Further, the equation of the circle with centre (h, k) and radius r is of the form x2 y2
2gx 2fy c  0 (see Quick Look 1). With this understanding we refer the equation x2 y2 2gx 2fy c  0
as the general equation of a circle. Usually, when g2 f 2 – c  0, then we call the circle as imaginary circle.
3. If a ≠ 0, then the equation ax2 ay2 2gx 2fy c  0 can be written as x2 y2 2gax 2f ay ca  0, where ga  g/a, f a 
f/a, ca  c/a which represents circle in the broad perspective as per point (2). The equation ax2 ay2 2gx 2fy c  0
is called universal equation of the circle.
4. The second degree general equation S y ax2 2hxy by2 2gx 2fy c  0 represents a circle with positive radius
if and only if a  b  0 and g2 f 2  ac. The proof is not necessary.

T H E O R E M 3.3 If A(x1, y1) and B(x2, y2) are extremities of a diameter of a circle, then the equation of the circle is
(x − x1)(x − x2) (y − y1)(y − y2)  0.

PROOF Consider Fig. 3.2. Let P be any point on the circle whose coordinates are (x, y). It is clear that both
the points A and B satisfy the equation
(x − x1)(x − x2) (y − y1)(y − y2)  0 (3.1)

P (x,
x y)

90°

A(x
x1, y1) x2, y2)
B (x

FIGURE 3.2

Hence, we may assume P is not both A and B. From the elementary plane geometry, it is known that
angle in a semicircle is a right angle. Therefore, APB is a right angle. That is, the segments AP and
BP are at right angle to each other so that the product of their slopes is equal to −1. Therefore
¥ y y1 ´ ¥ y y2 ´
¦§ x x µ¶ ¦§ x x µ¶  1
1 2

Hence
(x − x1)(x − x2) (y − y1)(y − y2)  0
That is, every point on the given circle satisfies Eq. (3.1) and conversely, if any point Q(x, y) satisfies
Eq. (3.1), then we know that AQ and BQ are at right angles and hence Q must lie on the circle.
Hence, Eq. (3.1) represents the circle for which A(x1, y1) and B(x2, y2) are ends of a diameter.

T H E O R E M 3.4 If P(x, y) is a point on the circle x2 y2  r2, then there exists P such that x  r cos P and y 
r sin P and conversely the point (r cos P, r sin P ) lies on the circle x2 y2  r2 for all P. The
equations x  r cos P, y  r sin P are called parametric equations of the circle x2 y2  r2.
PROOF Let P(x, y) be a point on the circle x2 y2  r2 (see Fig. 3.3). Draw PM perpendicular to the x-axis
and let MOP  P. From ΔMOP, we have that
x y
cos P  and sin P 
r r
128 Chapter 3 Circle

Therefore, x  r cos P, y  r sin P. Also


r2 cos2 P r2 sin2 P  r2
shows that (r cos P, r sin P ) lies on the circle x2 y2  r2.
y

P (x,
x y)
r
y
q
O x M x

FIGURE 3.3

Note:
1. By shifting the origin to the centre (h, k) of the circle (x − h)2 (y − k)2  r2 and using Theorem 3.4 we can see
that
x  h r cos P
and y  k r sin P
are the parametric equations of the circle (x − h)2 (y − k)2  r2.
2. Since (−g, −f ) and r  g 2 f 2 c are, respectively, the centre and radius of the circle x2 y2 2gx 2fy c  0, it
follows that
x  −g r cos P and y  −f r sin P
are its parametric equations.

3.2 Relation Between a Circle and a Line in its Plane


Let C be a circle with centre A and radius r and l be a straight line in the plane of the circle. Draw AM perpendicular
to the line l. Then
1. AM  r š the line l and the circle C have no common points [see Fig. 3.4(a)].
2. AM  r šthe line touches the circle [see Fig. 3.4(b)].
3. AM  r šthe line l intersects the circle C in two distinct points [see Fig. 3.4(c)].

A
r
A
r

90° l M
l M l M

(a) (b) (c)

FIGURE 3.4
3.2 Relation Between a Circle and a Line in its Plane 129

General Note: In the proofs of theorems, we consider the circle x2 y2  r2 only to avoid tediousness of the proofs.

T H E O R E M 3.5 The perpendicular bisector of a chord of a circle passes through the centre of the circle.
PROOF Let A(x1, y1) and B(x2, y2) be ends of a chord of the circle x2 y2  r2 whose centre is O(0, 0) (see
Fig. 3.5). Let M be the midpoint of the chord AB so that
¥ x x2 y1 y2 ´
M¦ 1 , µ
§ 2 2 ¶

Now, A and B are points on the circle implies


x12 y12  r2

and x22 y22  r2


Therefore
2 2 2 2
1 2) ( 1 2) 0
( 1 2 )( 1 2) ( 1 2 )( 1 2) 0
y1 y2 ( x1 x2 )
 
x1 x2 y1 y2

Now,
y1 y2
0
2 y y2 ( x1 x2 )
Slope of line OM   1 
x1 x2 x1 x2 y1 y2
0
2

Therefore
¥y y2 ´ ¥ y1 y2 ´
(Slope of chord AB)(Slope of OM )  ¦ 1  1
§ x1 x2 µ¶ ¦§ x1 x2 µ¶

and hence OM is perpendicular to AB.

O
90°
A B
M

FIGURE 3.5

Notation: Here onwards, we use the following notation.

S x 2 y2 2 gx
gx fy c
S1 x 1 yy1 g( x x1 ) f ( y y1 ) c
xx
S2 x 2 yy2 g(xx x2 ) f ( y y2 ) c
xx
S21 S12  x1 x2 y1 y2 g( x1 x2 ) f ( y1 y2 ) c
S11 x12 y12 2 gx1 2f 1 c
130 Chapter 3 Circle

In general

Sii xi2 yi2 2 gx


gxi ffyyi c

In particular if S y x2 y2 − a2, then 1 xx1 yy1 a 2, S11 x12 y12 a 2, etc .

T H E O R E M 3.6 If S  0 is a circle and (x1, y1) is a point in the plane of the circle (not the centre), then the equation
S1  0 represents a straight line which is perpendicular to the line joining the centre and the point
(x1, y1).

PROOF Let
S y x2 y2 2gx 2fy c  0
Therefore
S1 xxx1 yy1 g( x x1 ) f ( y y1 ) c
y ( g x1 ) x ( f y1 ) y gx1 fy1 c

Since (x1, y1) ≠ (−g, −f ), it follows that


S1 y (g x1)x (f y1)y gx1 fy1 + c  0
is a first-degree equation in x and y and hence it represents a straight line. Also, since the slope of
the line S1  0 is −(g x1)/(f y1), it follows that it is perpendicular to the line joining (−g, −f ) and
(x1, y1).

T H E O R E M 3.7 The equation of the chord joining two points P(x1, y1) and Q(x2, y2) on a circle S  0 is S1
S2  S12 and hence the equation of the tangent at (x1, y1) is S1  0.
PROOF Let S y x2 y2 2gx 2fy c  0. Since P(x1, y1) and Q(x2, y2) lie on the circle (see Fig. 3.6), we have

11 x12 y12 2gx


2 gx
gx1 ffyy1 c  0
S22 x22 y22 2gx
2 gx
gx2 fy2 c  0
fy

¥ x x2 y1 y2 ´
Let C  (−g, −f ) (centre) and M  ¦ 1 , µ (the midpoint of AB ). From Theorem 3.5,
§ 2 2 ¶
AB is perpendicular to CM so that the equation of the chord AB is
¥ x1 x2 ´ ¥ y1 y2 ´
( 1)¦ g ( 1)¦ fµ  0
§ 2 ¶ § 2 ¶
( )(xx1
1 )( x2 2gg ) ( y y1 )( y1 y2 f)  0
 x(x
( 1 x2 ) y(y( 1 y2 ) 2 gx 2 fy  x12 y12 x1 x2 y1 y2 2 gx1 2 fy1
y(y
 x( x1 x2 ) y( y1 y2 ) 2 gx 2 fy  x1 x2 y1 y2 c (& & S11  0)
 [ xx1 yy1 g( x x1 ) f ( y y1 ) c] [ xx2 yy2 g( x x2 ) f ( y y2 ) c]
 x1 x2 y1 y2 gx1 fy1 gx2 fy2 2c c

C (−g, −f )

A(x
x1, y1) B (x
x2, y2)
M

FIGURE 3.6
3.2 Relation Between a Circle and a Line in its Plane 131

Therefore

1 S2  x1 x2 y1 y2 g( x1 x2 ) f ( y1 y2 ) c S12

That is, equation of the chord AB is S1 S2  S12.


Since the tangent at P(x1, y1) to the circle is the limiting position of the chord PQ as Q
approaches P along the circle (see Chapter 3, Vol. 3), the equation of the tangent is
S1 S1  S11 [ P( x1 , y1 ) lies on the circle]
Hence, S1 y xx1 yy1 g(x x1) f(y y1) c  0 is the tangent at (x1, y1).

QUICK LOOK 2

If S  x2 y2 − a2  0 is the circle, then the tangent at (x1, y1) is S1y xx1 yy1 − a2  0.

Example 3.1

Find the equation of the tangent to the circle x2 y2 − which implies that (2, 3) lies on the circle S  x2 y2 −
2x − 4y 3  0 at the point (2, 3). 2x − 4y 3  0. Here, (x1, y1)  (2, 3) so that the equation
of the tangent at (2, 3) is
Solution: We have
S1  x(2) y(3) − (x 2) − 2(y 3) 3  0
22 32 − 2(2) − 4(3) 3  16 − 16  0 that is, S1  x y − 5  0.

Example 3.2

Find the equation of the tangent to the circle 3x2 3y2 − Here, g  −2/3, f  −1, c  0 and (x1, y1)  (0, 0). The equa-
4x − 6y  0 at (0, 0). tion of the tangent at (0, 0) is
2
Solution: The equation of the circle is in the universal S1 x(0) y(0) ( x 0) 1( y 0) 0
form so that its general form is 3

4 That is, S1 y 2x 3y  0.
S y x2 y2 − ¥ ´ x − 2y  0
§ 3¶

Example 3.3

Find the equation of the tangent to the circle the tangent at (a, 0) is
x2 y2 2ay cot @ − a2  0 x(a) y(0) a cot @ (y 0) – a2  0
at (a, 0).  ax (c cot @)y − a2  0
 x y cot @ − a  0
Solution: Clearly, (a, 0) lies on the circle. Equation of

Example 3.4

Find the equation of the tangent to the circle x2 y2  a2 and (x1, y1)  (a cos P, a sin P )
at (a cos P, a sin P ).
The equation of the tangent at (a cos P, a sin P) is
Solution: We have in ) a 2  0
x(a cos Q ) y(a sin
S y x2 y2 − a2  0  x cos P y sin P − a  0
132 Chapter 3 Circle

Example 3.5

Find the equation of the tangent to the circle S y x2 y2  2 x y 6  0


2x − 2y − 3  0 at the point (1, 2) and also find the tan-  2x y 6  0
gent to the circle parallel to this tangent.

Solution: The equation of the tangent at A(1, 2) (see B (−3, 0) 2x + y + 6 = 0


Fig. 3.7) is

S1 y x(1) y(2) 1(x 1) − 1(y 2) − 3  0


(−1, 1)
 S1 y 2x y − 4  0 (3.2)
The tangent to the circle parallel to the tangent given by
Eq. (3.2) must be at the other end B(−3, 0) of the diam-
eter through the point A(1, 2). Therefore, the tangent at A (1, 2) 2x + y − 4 = 0
B(−3, 0) is
FIGURE 3.7
x( ) y( ) 1(x
(x
(x (y
) 1(y
(y ) 3  0

T H E O R E M 3.8 The condition for the line y  mx c, c ≠ 0 to touch the circle x2 y2  a2 is that c2  a2(1 m2) and
in such a case the point of contact is (−a2m/c, a2/c).
PROOF Suppose the line y  mx c touches the circle at the point P(x1, y1). Hence, by Theorem 3.7, the
equation of the tangent at P(x1, y1) is

S1 xxx1 yy1 a 2  0

That is, S1  0 and y  mx c represent the same line. Therefore

x1 y1 a 2
 
m 1 c
Therefore

a2 m a2
x1  and y1 
c c
Since (xl, y1) lies on the circle, we have

a4 m a4
 a2
c2 c2
 c 2  a 2 (1 m2 )

Conversely, suppose c2  a2(1 m2). Therefore

m( ) 0 c
a
1 m2

That is, the length of the perpendicular drawn from (0, 0) (i.e. the centre of the circle) onto the line
y  mx c is equal to the radius a. Hence, by point (2) in Section 3.2, the line y  mx c touches
the circle.

Note:
1. For any m ≠ 0, the two lines y  mx p a 1 m2 are parallel tangents to the circle x2 y2  a2.
3.3 Classification of Points in a Plane w.r.t. a Circle in the Same Plane 133

2. In general, to show that a line touches a circle, it is enough if we show that the length of the perpendicular drawn
from the centre onto the line is equal to the radius of the circle.

3.3 Classification of Points in a Plane w.r.t. a Circle in the Same Plane


Let C be a circle with centre at the point A and radius r and P be any point in the plane (Fig. 3.8). Then
1. P lies outside the circle š AP  r.
2. P lies on the circle (on the circumference) šAP  r.
3. P lies inside the circle šAP  r.

R (On the circle)


r
A

Q (Inter
nte nal point)

P (External point)

FIGURE 3.8

T H E O R E M 3.9 Let S  0 be a circle and P(x1, y1) be a point in the plane of the circle. Then P(x1, y1) lies outside
or inside or on the circle according as S11    0.

PROOF Let S  x2 y2 2gx 2fy c  0, A  (−g, −f) and r  g 2 f 2 c . Now


P(x1, y1) lies outside S  0 šAP  r
š( )2  r 2
2 2 2 2
š( 1 ) ( 1 )  g f c
š x12 y12 2 gx1 2 fy1 c  0
š S11  0

Similarly, P(x1, y1) lies inside S  0 šS11  0.

T H E O R E M 3.10 Through every external point, two distinct tangents can be drawn to a circle.
PROOF Let S y x2 y2 − a2  0 be a circle and P(x1, y1) be an external point to the circle, so that

11 x12 y12 a2  0

By Note (1) under Theorem 3.8, we know that y  mx a 1 m2 touches the circle S y x2 y2

− a2  0. This line passes through P(x1, y1)

š y1  mx1 a 1 m2
2
š( 1 1)  a2 ( 2
)
2 2
š( 1 )m 2 x1 y1 m y12 a 2  0
2
(3.4)
134 Chapter 3 Circle

Equation (3.4) is a quadratic equation in m whose discriminant is

4 x12 y12 4( x12 a 2 )( y12 a 2 )  4a 2 ( x12 y12 a2 )


 4a 2 S11 0 (& S11  0)
0

Therefore, the quadratic equation [Eq. (3.4)] in m has two distinct roots, say m1 and m2, so that
there are two tangents through (x1, y1) with slopes m1 and m2 and

y12 a2
m1 m2 
x12 a2

QUICK LOOK 3

The two tangents through (x 1, y 1) to the circle Therefore, the locus of (x1, y1) is the circle x2 y2  2a2,
S y x 2 y 2 − a 2  0 are at right angles implies and is which is a circle concentric with S  0 and having radius
implied by (š) 2 times the radius of S  0. That is, the locus of the
point through perpendicular tangents drawn to a circle
y12 a2 S  0 is also a circle concentric with S  0 and radius
m1 m2  1 š 1
x12 a2 equal to 2 times the radius of S  0.
š x12 y12  2a 2

DEFINITION 3.2 Director Circle The locus of the point through which perpendicular tangents are drawn to a
given circle S  0 is a circle called the director circle of S  0.

QUICK LOOK 4

If the centre and radius of a circle are A and r, respectively, then the centre and radius of its director circle are A
and r 2, respectively.

Example 3.6

Find the locus of the point of intersection of perpendicu- director circle of S  0 is


lar tangents to the circle S y x2 y2 − 2x 2y − 2  0. That
is, find the director circle of S  0. ( )2 ( )2  ( )2  8

Solution: The centre and radius of the circle S  0 are That is,
(1, −1) and 2, respectively. Hence, the equation of the x2 y2 − 2x 2y − 6  0

T H E O R E M 3.11 The length of the tangent drawn from an external point P(xl, y1) to the circle S  0 is S11 .

PROOF Let S y x2 y2 2gx 2fy c  0, centre A  (−g, −f ) and radius r  g 2 f 2 c . Let T be the
point of contact of the tangent from P to the circle. See Fig. 3.9. From Pythogoras theorem, we
have
(AP)2  (PT)2 (AT)2
 (x1 g)2 (y1 f )2  (PT)2 (g2 f 2 − c)
 (PT )2  x12 y12 2gx1 2fy1 c  S11  0
3.3 Classification of Points in a Plane w.r.t. a Circle in the Same Plane 135

because P(x1, y1) is an external point (see Theorem 3.9). Therefore


PT  S11

A (−g, −f )

P (x
x1, y1) T

FIGURE 3.9

DEFINITION 3.3 Chord of Contact Let C be a circle and P be an external point to C. Let A and B be the points
of contact of the tangents drawn from P to C. Then the chord AB is called the chord of con-
tact of the point P with respect to the circle C.

T H E O R E M 3.12 The equation of the chord of contact of a point P(x1, y1) with respect to the circle S  0 is S1  0.

PROOF Let AB be the chord of contact of P (see Fig. 3.10). Suppose A  (x2, y2) and B  (x3, y3). The
equation of the tangent at A(x2, y2) is S2  0.
This tangent passes through ( x1 , y1 ) S21 S12 . Therefore, the point A(x2, y2) satisfies
the first-degree equation S1  0. Similarly, S13  0 implies that the point B(x3, y3) satisfies the first-
degree equation S1  0. Hence, the equation of the chord AB is S1  0.

P (x
x1, y1) A

FIGURE 3.10

QUICK LOOK 5

The chord of contact AB of P is perpendicular to the line joining P with the centre of the circle (see Theorem 3.6).

T H E O R E M 3.13 The equation of the chord of the circle S  0 whose midpoint is M(x1, y1) is S1  S11 (see Fig. 3.11).

C (−g, −f )

A B
M (xx1, y1)

FIGURE 3.11
136 Chapter 3 Circle

PROOF Let C(−g, −f ) be the centre of the circle S y x2 y2 2gx 2fy c  0. Let M(x1, y1) be the
midpoint of the chord AB. Since AB is a chord perpendicular to CM, the equation of AB is
( 1 )( 1 ) ( 1 )( 1 )0
 xx1 yy1 gx fy  x12 y12 gx1 fy1
 xx1 yy1 g(x x1) f(y y1) c  x12 y12 2gx1 2fy1 c
 S1  S11

Subjective Problems (Sections 3.1 till 3.3)


1. If the circle S y x2 y2 2gx 2fy c  0 intersects the Solution: Let the circle intersect the line in A and B
x-axis in two points, then show that the length of the and M be the midpoint of AB. If C is the centre of the
circle (see Fig. 3.13) then p  CM. Using Pythagoras the-
intercept is 2 g 2 c.
orem, we have
Solution: Let the circle meet the x-axis in A(x1, 0) and A  2 r2
AB  2 AM p2
B(x2, 0) (see Fig. 3.12) so that x1, x2 are the roots of the
equation x2 2gx c  0 (since the x-axis equation is y  0).
Therefore
x1 x2  −2g and x1x2  c C
Hence r
p

(AB)2  (x1 − x2)2  (x1 x2)2 − 4x1x2  4g2 − 4c  4(g2 − c) A M B

 AB  2 g 2 c
FIGURE 3.13
Similarly, the length of the intercept made by the circle
3. Two rods of lengths 2a and 2b slide along the coor-
S  0 on y-axis is 2 f 2 c.
dinate axes such that their ends are always concyclic.
y Find the locus of the centre of the circle.
y

N P (h, k )

O A B x C

O A M B x
FIGURE 3.12

QUICK LOOK 6
FIGURE 3.14
1. x-axis touches the circle S  0 š g2  c.
y-axis touches the circle S  0 šf 2  c. Solution: P(h, k) is the centre of the circle passing
2. The circle S  0 touches both the axes šg2  c  through points A, B, C and D (Fig. 3.14) where AB  2a and
f2 š g c f . DC  2b. This implies and is implied by (š)
3. The equation of the circle, with radius a, which
touches both axes, is given by x2 y2 ± 2ax ± 2ay PA  PC  radiuss of the circle
a2  0 (one in each of the quadrants). š ( PA)2  ( PC )2
š ( AM )2 ( PM )2  ( PA)2  ( PC
P )2 N )2
( PN (CN )2
2. Find the length of the intercept on a straight line by a
circle with radius r and the length of the perpendicu- where M and N are the midpoints of AB and CD, respec-
lar from the centre of the circle onto the line being p. tively. From the above, we have
3.3 Classification of Points in a Plane w.r.t. a Circle in the Same Plane 137

6. Find the equation of the circle passing through the


a2 k 2  h2 b2
points P(−1, 2), Q(3, −2) and whose centre lies on
šk 2 2
h b 2
a 2 the line x  2y.

(h, k ) is y2
š Locus of (h x 2  b2 a2 Solution: Let S y x2 y2 2gx 2fy c  0 be the
required circle. Since its centre (−g, −f ) lies on the line
4. Find the equation of the circle passing through the x  2y, we have
points A(0, 1), B(2, 3) and C(−2, 5). g − 2f  0 (3.6)
Solution: Let S y x2 y2 2gx 2fy c  0 be the circle The circle passes through the points P(−1, 2) and
passing through points A, B and C. Therefore Q(3, −2). Hence
2f c 1 −2g 4f c  −5 (3.7)

4 g 6 f c  13 6g 4f c  −13 (3.8)
44 g 10 f 29 (3.5) Solving Eqs. (3.6), (3.7) and (3.8), we obtain g  −2, f 
−1 and c  −5. Therefore, the equation of the circle is
Solving the system of equations provided in Eq. (3.5), we
given by S y x2 y2 − 4x − 2y − 5  0.
get g  1/3, f  −10/3 and c  17/3 so that the equation of
the circle is
7. The line x  y is tangent at (0, 0) to a circle of radius 1.
2 20 17 Find the centre of the circle.
x2 y2 x y 0
3 3 3
Solution: Let C(x1, y1) be the centre of the circle (see
3xx 2 y2 2 x 220y
0 y 17 0 Fig. 3.16). The line joining C(x1, y1) and O(0, 0) is perpen-
dicular to the line y  x. Therefore
Note: Under the given hypothesis, to find the equation
of the circle, it is sufficient if we find its centre and radius y1
 1  y1 x1
or assume the circle as x2 y2 2gx 2fy c  0 and find x1
the values of g, f and c.
Hence
5. Find the equation of a circle with the centre at point
1
(6, 1) and touching the line 5x 12y − 3  0. 1  x12 y12  2x12  x1  p
2
Solution: Let A be (6, 1). Since the line 5x 12y − 3  0
touches the circle (say, at M as shown in Fig. 3.15), the 1
and y1  p
distance of the line from the centre is equal to the radius. 2
Therefore, the radius is given by
Therefore
5(6) 12(1) 3 39
r  3 ¥ 1 1 ´ ¥ 1 1 ´
52 12 2 12 (x1, y1)  ¦ ,
§ 2 µ¶ or ¦§ , µ
2 2 2¶
Therefore, the equation of the circle is (x − 6)2 (y − 1)2  9.
That is, x2 y2 − 12x − 2y 28  0.

1
A (6, 1)

r O (0, 0) y =x

FIGURE 3.16
M 5x + 12y − 3 = 0
8. Determine the position of the point (−1, −2) relative
FIGURE 3.15
to the circle S y x2 y2 4x 6y 9  0.
138 Chapter 3 Circle

Solution: We have Solution: The line lx my  1 touches the circle x2


y2  a2. This implies that the length of the perpendicular
S11  ( 1)2 ( 2)2 4( 1) 6( 2) 9 drawn from the centre (0, 0) onto the line is equal to the
 14 16  2 0 radius a. Therefore,
Therefore, by Theorem 3.9, the point (−1, −2) lies inside 1
the circle. a
l m2
2

9. From each point on the line 2x y − 4  0, a pair of 1


 l 2 m2 
tangents are drawn to the circle x2 y2  1. Prove that a2
the chords of contact pass through a fixed point.
Therefore, the point (l, m) lies on the circle x2 y2  1/ 2 .
Solution: Let P(x1, y1) be a point on the line 2x y −
4  0. Therefore
12. Show that the line 3x − 4y − 1  0 touches the circle
2x1 y1 − 4  0 (3.9) x2 y2 − 2x 4y 1  0 and find the coordinates of
By Theorem 3.12, the equation of the chord of contact of the point of contact.
(x1, y1) with respect to the circle
Solution: The centre of the circle is (1, −2) and its ra-
S y x2 y2 − 1  0 dius is 12 2 2 1 2. The distance of the line from the
is given by centre (1, −2) is given by
S1 y xx1 yy1 − 1  0 (3.10) 3(1) 4( 2) 1 10
 2
From Eq. (3.9), we get y1  4 − 2x1. Substituting the value 3 2
4 2 5
of y1 in Eq. (3.10), we have
which is equal to the radius of the circle. Therefore, the
(x − 2y)x1 + (4y − 1)  0
line touches the circle. Let (x1, y1) be the point of contact.
so that the line passes through the point of intersection That is, at (x1, y1), the line
of the lines x − 2y  0 and 4y − 1  0 which is given by
3x – 4y − 1  0 (3.11)
¥ 1 1´
, is the tangent. But, by Theorem 3.7, the equation of the
§ 2 4¶
tangent at (x1, y1) is
10. Find the equations of the tangent to the circle x2 S1 x 1 yy1 ( x x1 ) 2( y y1 ) 1  0
xx
y2  9 which is perpendicular to the line 2x 3y
That is,
7  0.
S1 ( x1 1) x ( y1 2) y x1 2 y1 1  0 (3.12)
Solution: Any line perpendicular to the line 2x 3y
7  0 is of the form 3x − 2y c  0. This line touches the Equations (3.11) and (3.12) represent the same straight
circle x2 y2  9 if and only if the perpendicular drawn line. Therefore,
onto the line 3x − 2y c  0 from the centre (0, 0) is equal x1 1 y1 2 x1 2 y1 1
to the radius 3. That is,   t( )
3 4 1
c Hence
3
32 22 x1 3t 1 y1 4t 2

Therefore and t  x1 2 y1 1  ( 3t 1) (8t 4) 1  11t 4


c  ±3 13 2
t
Hence, the equations of the required tangent are 5
3 x y 3 13  0 Therefore,

and 3 x y 3 13  0 6 1
x1 3t 1  1
5 5
11. If the line lx my  1 touches the circle x2 y2  a2, 8 2
then show that the point (l, m) lies on the circle and y1 4t 2  2 
2 5 5
x2 y2  / .
3.3 Classification of Points in a Plane w.r.t. a Circle in the Same Plane 139

Hence passing through the point (2, 3) is the circle x2 y2 −


2x − 3y  0.
¥ 1 2 ´ (IIT-JEE 1989)
( 1 , y1 ) ,
§ 5 5¶
Solution: Let M(h, k) be the foot of the perpendicular
13. Find the length of the chord of the circle x2 y2 − drawn from the origin O(0, 0) to a line passing through
10x − 20y − 44  0 on the line 3x − 4y  0. the point (2, 3) as shown in Fig. 3.18. Therefore, OM is the
perpendicular drawn to the line so that
Solution: C  (5, 10) is the centre and r  13 is the ra-
(Slope of OM ) (Slope of the line)  1
dius of the given circle. Suppose the line 3x − 4y  0 cuts
the circle at points A and B and M is the midpoint of AB ¥ k ´ ¥ k 3´
  1
(see Fig. 3.17). Therefore, CM is the perpendicular drawn § h¶ § h 2¶
to AB (see Theorem 3.5) which is given by  h(h 2) k(k 3)  0
3(5) 4(10) Therefore, the locus of the point (h, k) is x2 y2 − 2x −
CM  5
32 42 3y  0 which is equivalent to the circle described on the
line joining the points (0, 0) and (2, 3) as diameter.
Therefore,
AB  2 AM
y (2, 3)
2 2
 2 CA CM
M (h, k )
 2 1132 52  2 s 12  24

O x

C (5, 10)

90° 3x − 4y = 0 FIGURE 3.18


A M B
15. Find the equation of the circle passing through the
FIGURE 3.17 origin which is cutting the chord of equal length 2
on the lines y  x and y  −x.
Aliter: Put y  3x/4 in the equation of the circle so that
we have Solution: Let S y x2 y2 2gx 2fy c  0 be the re-
quired circle. Since it passes through the origin (0, 0), c  0.
9x2 ¥ 3x ´ Put y  x in S  0. Then x2 (g f )x  0. Therefore, x  0, x
x2 10 x 20 44  0
16 § 4¶  −(g f ) so that the points of intersection are A(0, 0) and
That is, B[−(f g), −(f g)]. Now

25 x 2
400 x 704  0 AB  2
 (5 x 8)(5 x 88) 0  f g  p1

Therefore, Similarly,

88 8 g − f  ±1
x ,
5 5 Therefore, the centres of circles are given by (1, 0), (−1,
Hence, the points are A  (88/5, 66/5) and B  (−8/5, −6/5) 0), (0, 1) and (0, −1) and the equations of the circle are
and the length is given by given by x2 y2 ± 2x  0 and x2 y2 ± 2y  0.

2 2 16. The angle between the pair of tangents from a point


¥ 88 8 ´ ¥ 66 6 ´
AB   576  24 P to the circle S y x2 y2 4x − 6y 9 4cos2 @  0
§ 5 5¶ § 5 5¶
is 2@. Show that the point P lies on the circle x2 y2
14. Show that the equation of the locus of the foot of the 4x − 6y 9  0 and hence find the equation of the
perpendicular drawn from the origin upon any line director circle of S  0.
140 Chapter 3 Circle

Solution: The centre of the circle S  0 is (−2, 3) and its Solution: Let S y x2 y2 − 2ax  0 and P  (h, k) be a
radius is 2sin @ (note that 2@ being the angle between point on S  0. Therefore
the tangents, we have 0  @  O/2). Let P  (h, k) as shown
in Fig. 3.19. Then h2 k 2 2ah
h 0 (3.13)
CT 2sin A Now, Q (h / 2, k / 2) is the centre of the circle drawn on
sin @  
CP 2 2 OP as the diameter. From Eq. (3.13), we have
( ) ( )
2 2
¥ h´ ¥ k´ ¥ h´
Therefore a 0
§ 2¶ § 2¶ § 2¶
(h 2)2 (k − 3)2  4
Therefore, the locus of Q is x2 y2 − ax  0.
Hence, (h, k) lies on the circle
(x 2)2 (y − 3)2  4 or x2 y2 4x − 6y 9  0 19. Find the point for which the line 9x y − 28  0 is the
chord of contact with respect to the circle 2x2 2y2 −
If @  O/4, then 2@  O/2 and therefore the equation of 3x 5y − 7  0.
the director circle of S y x2 y2 4x − 6y 11  0 is given
by x2 y2 4x − 6y 9  0. Solution: The given circle is
T 3 5 7
S x 2 y2 x y  0
2sin a 2 2 2
C (2, 3) Let P(x1, y1) be the point whose chord of contact with
respect to S  0 is
2sin a
a
9x y − 28  0 (3.14)
a
P T But, in fact, the chord of contact of P(x1, y1) with respect
to S  0 is given by
FIGURE 3.19
3 5 7
xx1 yy1 ( x x1 ) ( y y1 )  0
17. Prove that the locus of the point whose chord of con- 4 4 2
tact with respect to a circle subtends a right angle at That is,
the centre of the circle is its director circle.
( 1 ))x ( 1 ))yy 3 1 5 y1 14  0 (3.15)
Solution: P is a point and AB its chord of contact with Equations (3.14) and (3.15) represent the same line.
respect to a circle with centre C (see Fig. 3.20) such that Therefore,
BCA  90°. Since CA and CB are at right angles to the
tangents PA and PB, respectively. It follows that APB 4 x1 3 4 1 5 33 x1 5 y1 14
  t ( )
is a right-angled triangle. Hence, point P lies on the di- 9 1 28
rector circle of the given circle.
Hence
3 9t
x1 
4
t 5
B C and y1 
90° 4
so that
90°° ¥ 3 9t ´ ¥ t 5´
P A 28t  3 5 14
§ 4 ¶ § 4 ¶
FIGURE 3.20
 112t  99 2277t 5t 25 56
18. Point P is on the circle x2 y2 − 2ax  0. A circle is  90 90
drawn on OP as diameter where O is the origin. As
P moves on the circle, find the locus of the centre of t1
the circle.
3.3 Classification of Points in a Plane w.r.t. a Circle in the Same Plane 141

Therefore Since AOB  90°, from the above equation,


3 + 9t 12 Coefficient of x2 Coefficient of y2  0
x1   3
4 4
2 gh 2ch2 2 fk 2ck 2
t 5 4  1 1 0
and y1    1 (h 2 k 2 ) ( h 2 k 2 )2 (h 2 k 2 ) ( h 2 k 2 )2
4 4
2 2 2 2 2 2 2
Hence, P  (3, −1).  2( ) 2( )( ) 2 ( )0

 h2 k 2 gh kf c  0
20. Show that the circle x2 y2 4x − 4y 4  0 touches
both axes and find the points of contact. Hence, the locus of (h, k) is given by

Solution: The centre is (−2, 2) and the radius is 2. The x2 y2 gx fy c  0


distance of the centre (−2, 2) from the coordinate axes is y
equal to 2. Hence, the circle touches both coordinate axes,
and the points of contact are given by (−2, 0) and (0, 2).
B N (h, k )
21. Find the equations of the circle touching both axes
90° A
and passing through the point (2, 1).
90°
Solution: Since the circle touches both axes and passes O x
through the point (2, 1), the centre of the circle must be
in the first quadrant. Hence, its equation should be of the
form x2 y2 − 2ax − 2ay a2  0. It passes through the
point (2, 1) which implies that FIGURE 3.21
5 4a 2a 2 0
23. Find the equation of the circle which touches the
 ( 1)( 5)  0
x-axis at (a, 0) and cuts of chord of length l on the
 1 5 positive y-axis. Determine the equation of the circle
The equations of the circle are x2 y2 − 2x − 2y 1  0 when a  12 and l  10.
and x2 y2 − 10x − 10y 25  0.
Solution: Let S y x2 y2 2gx 2fy c  0. Since the
22. The chords of the circle x2 y2
2gx 2fy 2c  0 circle touches x-axis (see Fig. 3.22), we have
subtend right angle at the origin. Show that the locus
of the foot of the perpendiculars from the origin to g2  c (Quick Look 6)
these chords is the circle x2 y2 gx fy c  0. Therefore

Solution: Let AB be a chord of the circle S y x2 y2 a2  c (Problem 1)


2gx 2fy 2c  0 subtending right angle at the origin Also l is the y-intercept. This implies that
( AOB  90°) and N(h, k) be the foot of the perpendicu-
lar drawn from the origin O to the chord AB (see Fig. 2 f2 cl ( )
3.21). Since the slope of ON is k/h, the equation of the
chord AB is l2
 f2 c
4
h
y k  ( x h)
k l2 l2
 f2  c  a2
hx ky  h2 k2 4 4

Therefore, the combined equation of the pair of lines


F E F E l2 4a 2
f p
OA and OB is given by 2
2
¥ hx ky ´ ¥ hx ky ´ Since the intercept is on the positive y-axis, we have
x2 y2 (2gx 2ffy) 2c 2 0
§ h2 k 2 ¶ § h k2 ¶
l2 a2
(by Theorem 2.33) f±
2
142 Chapter 3 Circle

Hence, the required equation of the circle is Since n ≠ 1, the locus is the circle (n2 − 1)(x2 y2)
2a(n2 1)x (n2 − 1)a2  0.
x2 y2 2ax
ax ( l 2 a2 )y a2  0
2 Note: In the above problem, if n  1, then the locus of P
¥ l 2 a2 ´ l 2 a2 is the perpendicular bisector of the segment AB .
 ( )2 ¦ y µ 
§ 2 ¶ 4
26. Prove that from a point (a, b) of the circle x(x − a)
y y(y − b)  0, two chords, each bisected by the x-axis,
Q can be drawn if and only if a2  8b2.

Solution: If P  (a, b), then the equation x(x − a) y(y


− b)  0 represents the circle with O and P as extremities
l
of a diameter (see Fig. 3.23). Let M(h, 0) be the midpoint
of a chord of the circle. Equation of this chord is

P a b
x
xh y( ) ( x h) ( y ) h2 ah
O A (a, 0) 2 2
(see Theorem 3.13)
That is,
FIGURE 3.22 ¥ a´ b ah
h x y h2
§ 2¶ 2 2
24. A point moves such that the length of the tangent
from it to the circle x2 y2 4x − 5y 6  0 is double This chord is passing through (a, b). This implies
the length of the tangent to the circle x2 y2  4.
¥ a´ b2 ah
Show that the locus is a circle. Find its centre and h a  h2
radius. § 2¶ 2 2
š 2 h a 2 b 2  2 h 2 ah
x2 y2
Solution: Let S y 4x − 5y 6  0 and Sa y x2
y2 − 4  0. Let P(x, y) be a point from which the tangent š 2 h2 3ah a 2 b2  0 (which has two distinct root
o s)
to S  0 is double the tangent to Sa  0. By hypothesis, 2 2 2
š9  8( )
2 2
S11 a
S11 (see Theorem 3.11) š a  8b
Therefore y

S11  4Sa11 P (a, b )

 x12 y12 4 x1 5 y1 6  4( x12 y12 4)


 3 x12 3 y12 4 x1 5 y1 22  0

Hence, the locus is a circle whose equation is given by O M x


3x2 3y2 − 4x 5y − 22  0.

25. If A and B are two fixed points and P is a variable


point such that PA:PB  n:1, then show that the locus FIGURE 3.23
of P is a circle if n ≠ 1.
27. Find the points on the line x − y 1  0, the tan-
Solution: Without loss of generality, we take A  (a, 0) gents from which to the circle x2 y2 − 3x  0 are of
and B(−a, 0). Let P be (x1, y1). So length 2.
PA  nPB Solution: Let P(x1, y1) be a point on the line x − y
š (PA)2  n2(PB)2 1  0 from which the length of the tangents to the
circle S y x2 y2 − 3x  0 is of length 2. Therefore
š( 1 )2 y12  n2 [( x1 a)2 2
1]
x1 y1 1 0 and S11  2
2 2 2 2
š( ))(( 1 1) a( ) x1 (n2 1)
1 2
0
x1 y1 1 0 a d x12 y12 3x
3 x1 4
3.3 Classification of Points in a Plane w.r.t. a Circle in the Same Plane 143

Therefore a( x12 y12 a 2 ) /2 3/ 2


aS11
 
x12 ( x1 1)2 3 x1  4 x12 y12 (OP )2
2 12 x1
 2x 30
B
 2 x12
2 x1 3x
2x 3 x1 3  0
 (2x
2 1 3)( x1 1) 0 a

Hence O (0, 0)
M
3 P (x1, y1) a
x1  1,
2
x1 y1  x1 1 A

1 0 FIGURE 3.24
3 5
2 2 29. Tangents are drawn to the circles x2 y2  a2 and x2
y2  b2 at right angles to one another. Find the locus
Therefore, the points on the line are given by (−1, 0) and of their point of intersection.
(3/2, 5/2).
Solution: Both circles are concentric at the origin (see
28. Show that the area of the triangle formed by the two Fig. 3.25). From the Note given under Theorem 3.8, the
tangents from (x1, y1) to the circle x2 y2  a2 and lines
their chord of contact is

a( x12 y12 a 2 )3 /2 y  mx a 1 m2
x12 y12 1 1
y x b a 2
m m
Solution: Let P  (x1, y1), O  (0, 0) and AB be the
are tangents to the given circles and are at right angles.
chord of contact (see Fig. 3.24). Suppose OP meets the
Therefore,
chord AB at point M so that AM  MD and OM is per-
pendicular to AB. The equation of chord AB is given by ( )2 ( )2  ( 2 2
)( 2
)
S1 y xx1 yy1 − a2  0 (see Theorem 3.12) ( 2 2
)( 2
) ( 2 2
)( 2
)
Therefore That is, x2 y2  a2 b2 is the required locus.
a2
OM 
x12 y12
and

a4 x12 y12 a 2 O (0, 0)


AM  OA2 OM 2  a 2 a
x12 y12 x12 y12

Also

x12 y12 a 2
PM 
x12 y12 P

x12 y12 a 2 FIGURE 3.25


 (& P lies outsidee thee circle)
x12 y12 Note: In the problem, if b  a, then the equation of the
Now, the area of $PAB is given by director circle of x2 y2  a2 is x2 y2  2a2.

1 a x12 y12 a 2 ( x12 y12 a 2 ) 30. Find the equations of the tangents to the circle x2
AB – PM  AM – PM  s y2 − 6x − 4y 5  0 which make an angle of 45° with
2 x12 y12 x12 y12 positive direction of the x-axis.
144 Chapter 3 Circle

Solution: In the circle S y x2 y2 − 6x − 4y 5  0, the 32. Find the equations of the tangents to the circle S y
centre is (3, 2) and radius is 2 2. The given line is x2 y2 8x 4y − 5  0 from the point (3, −3).
y  x c (& tan 45° 1)
Solution: The centre and the radius of the circle S  0
The line touches the given circle if and only if the dis- are (−4, −2) and 5, respectively. We have S11  32 (−3)2
tance of the line from the centre is equal to the radius. 8(3) 4(−3) − 5  25  0. Hence, (3, −3) is external to S
Therefore  0. Let y 3  m(x − 3) be a line passing through (3, −3).
3 2 c This line touches the circle S  0. This implies that
2 2
12 12 m( ) 2 3
2
5
 ( 1) 16 2
m 1
 1 4 ( m )2 25(m2 )
 3 5
 24 m2 14 m 24  0
Hence, the required equations of the tangents are x − y  12 m2 7 m 12 0
3  0 and x − y − 5  0. 2
 12 16 m 9 12 0
31. Prove that the tangent to the circle x2
 5 at the y2  4 (3 4) 3(3 4)  0
point (1, −2) also touches the circle x2 y2 − 8x − 6y − 4 3
20  0 and find the coordinates of the point of contact.   ,
3 4
Solution: Tangent to the circle S y x2 y2 − 5  0 at Therefore, the tangents from (3, −3) to the circle S  0
(−1, −2) is given by are

S1 x( 1) y( 2) 5 0 4
y 3 ( x 3)
S1 x 2y
2y 5  0 3
S1 x 2y 5  0 (3.16) 3
and y 3 ( x 3)
4
Now, this line [Eq. (3.16)] also touches the circle x2 y2
− 8x − 6y − 20  0. This implies that the distance of its That is,
centre (4, 3) from the line [Eq. (3.16)] is equal to the 4 x 3 y 21  0
radius 3 5. So
4y 3 0
3 x 4y
4 6 5
3 5 33. A circle passes through the points (−1, 1), (0, 6) and
12 22
(5, 5). On this circle, find the points at which the tan-
Hence, the tangent at (−1, −2) to x2 y2  5 also touches gents are parallel to the line joining origin to the cen-
the circle x2 y2 − 8x − 6y − 20  0 at say (x1, y1). But, the tre of the circle.
tangent at (x1, y1) is given by
Solution: Let S y x2 y2 2gx 2fy c  0 be the
xx1 yy1 4( x x1 ) 3( y y1 ) 20  0
circle passing through the points (−1, 1), (0, 6) and (5, 5).
 ( x1 4) x ( y1 3) y (4x
4 x1 3 y1 20)  0 Therefore,
x 4 y1 3 (4 x1 3 y1 20)
 1    t (say) 2 g 2 f c  2 º
1 2 5 ®
 x1  t 4 1 2t 3 12 f c  36» (3.17)
and 5t  ( 1 )  [ (t 4) (2 3)) ] 10 g 10 f c  50®¼
1
 5  10 45 Solving the set of equations given in Eq. (3.17), we obtain
 t  3 g  −2, f  −3 and c  0. Therefore, S y x2 y2 − 4x − 6y 
0 is the required circle. Let P(x1, y1) be a point on the
Therefore, circle S  0 at which the tangent to the circle is parallel to
( 1 , y1 ) ( 3 4, ( 3) ) ( , 3) the line joining origin to the centre (2, 3). Therefore, the
slope of the tangent S1 y xxl yy1 − 2(x x1) − 3(y y1)
Note: When a line touches a circle, then the point of con-  0 is equal to 3/2 and the slope of the line S1 y (x1 − 2)x
tact is the foot of the perpendicular drawn from the (y1 − 3)y − 2x1 − 3y1  0 is 3/2. Hence
centre of the circle onto the line.
3.3 Classification of Points in a Plane w.r.t. a Circle in the Same Plane 145

¥ x 2´ 3 Also
¦ 1 µ
§ y1 3 ¶ 2 F ( a, 0)

2 x1 y1 3 Now, the equation of EF is


  t (say)
3 2 3
y ( x a)
Therefore, x1  2 − 3t and y1  3 2t. Sine (x1, y1) lies on the 4
circle S  0, we have  3 x 4 y 6a  0
(2 − 3t)2 (3 2t)2 − 4(2 − 3t) − 6(3 2t)  0 Therefore, the distance of the centre (a, a) from the line
Therefore, EF is equal to

13t2 − 13  0  t  ±1 4a 6a
3a 4a
a
Hence, the point (x1, y1)  (−1, 5), (5, 1). 32 42
Hence, the line EF touches the circle. In Chapter 2, we
34. Find the locus of the midpoints of the portions of the
discussed the following result: If (h, k) is the foot of the
tangents to the circle x2 y2  a2 terminated by the
perpendicular from (x1, y1) onto a line lx my n  0,
coordinate axes.
then
Solution: Tangent at P(x1, y1) to the circle S y x2 y2 − h x1 k y1 (lx my1 n)
  12
a2  0 is S1 y xx1 yy1 − a2  0. Therefore, the intercepts of l m l m2
the tangent S1  0 on the axes are A(a2/x1, 0) and B(0, a2/ [seee Theorem 2.13(1)]
y1). M(h, k) is the midpoint of AB. This means there
Now, equation for the line EF is 3x − 4y 6a  0 and
a2 a2 (x1, y1)  (a, a). Therefore,
h ,k 
2x1 2y1
h a k a ¥ 3 4a 6a ´ a
a2 a2   
š x1  , y1  3 4 § 32 4 2 ¶ 5
2h 2k
This gives
a4 ¥ 1 1´
š a 2  x12 y12  µ 3a 2a 4a 9a
4 § h2 k 2 ¶ h a  and k  a 
4 1 1 5 5 5 5
š 2  2 2
a h k Therefore, the point of contact is given by

Therefore, the locus of M(h, k) is given by ¥ 2a 9a ´


,
§ 5 5¶
1 1 4
2
2

x y a2 D (0, 2a )
E C (2a , 2a )

35. ABCD is a square of side 2a units. Taking AB and a


AD as coordinate axes, find the equation of the circle
which touches all the four sides of the square. If E is O (a, a)
a point on DC such that 3DE  DC and F is a point
on BA produced such that FA  AB, prove that EF
touches the circle and also find the coordinates of
F (2a , 0) B (2a , 0)
the point of contact. A (0, 0)

Solution: By hypothesis, A is (0, 0), B  (2a, 0), C  (2a, FIGURE 3.26


2a) and D  (0, 2a). Therefore, O  (a, a) is the centre of
the circle so that a is its radius (see Fig. 3.26). Hence, the 36. Find the point of intersection of the tangents to the
equation of the circle is (x − a)2 (y − a)2  a2. Further, circle passing through the points (4, 7), (5, 6) and (1, 8)
E divides the segment DC in the ratio 1:2, that is, DE:EC at the point where it is cut by the line 5x y 17  0.
 1:2. Therefore,
Solution: Let A  (4, 7), B  (5, 6) and C  (1, 8).
¥ 2a ´ Let S(x1, y1) be the circumcentre of $ABC. Therefore,
E , 2a
§ 3 ¶ SA  SB  SC  R (circumradius). Hence
146 Chapter 3 Circle

SA  SB  ( x1 )2 (y
( 1 )2  ( x1 )2 (y
( y1 )2 Solution: We know that y-axis touches the circle x2
y2 2gx 2fy c  0 if f 2  c. For the circle mentioned
 2 x1 2 y1  4 in this problem also, we have q2  f 2  c  q2. Therefore,
 x1 y1  2 (3.18) y-axis touches the circle. Since the tangents drawn from
the origin are at right angles, the other tangent touches
SB  SC  ( x1 )2 (y( 1 )2  ( x1 )2 (y
( y1 )2 the x-axis. Therefore,
 8 x1 4 y1  4
p2  q2 (see Quick Look 6 and Problem 1)
 2 x1 y1  1 (3.19)
Aliter: Since the tangents drawn from (0, 0) are at right
Solving Eqs. (3.18) and (3.19), we have x1  1 and y1  3. angles, origin must lie on the director circle of the given
Therefore, the centre of the circle is (1, 3) and the radius circle (see Definition 3.2 and Quick Look 4). Therefore,
of the circle is R  SA  5. Therefore, the equation of the (0, 0) lies on the circle (x − p)2 (y − q)2  2p2.
circle is (x − 1)2 (y − 3)2  52. That is,
38. If the points
x2 y2 2 x 6 y 15  0
Let us consider that the line ¥ 1´
¦§ mi , m ¶ (i  1, 2, 3, 4)
5x y 17  0 (3.20) i

meets the circle at points P and Q and the tangents at are concyclic, then show that m1m2m3m4  1.
points P and Q meet at T(x1, y1) as shown in Fig. 3.27. Then, (IIT-JEE 1989)
Eq. (3.20) is the equation of the chord of contact of T(x1,
y1). However, the chord of contact of T(x1, y1) is given by Solution: Let S y x2 y2 2gx 2fy c  0 be a circle
passing through the four points
S1 xxx1 yy1 ( x x1 ) 3( y y1 ) 15 15 0
 S1 ( x1 1) x ((yy1 3) y ( x1 3 y1 115)  0 (3.21) ¥ 1´
¦§ mi , m µ¶
i
That is, Eqs. (3.20) and (3.21) represent the same line.
Therefore, by Theorem 2.10, we have Therefore,

x1 1 y1 3 ¥ x 3y
3 y1 15 ´ 1 2f
  ¦ 1 mi2 2 g(mi ) c  0
§ µ¶  t (say) mi2 mi
5 1 17
This gives Hence, mi, where i  1, 2, 3 and 4, are the roots of the
equation
x1 1 5t; y1  3 t
m4 gm 3
gm cm2 f 1
fm 0
and 17t  (1 5t ) 3(3 t ) 15 Therefore, the product of the roots is 1. That is,
m1 m2 m3 m4  1.
Solving, we get
25t  25  t  1 39. Find the values of K for which the point (K − 1, K 1)
( 4,, ). lies in the larger segment of the circle x2 y2 − x −
Therefore, 1 , y1 )
y − 6  0 made by the chord whose equation is
x y − 2  0. (IIT-JEE 1989)
Q
Solution: Let S y x2 y2 − x − y − 6  0. Let C be the
5x

centre of the circle which is given by (1/2, 1/2) and its


+y

(1, 3) radius is given by


+1
7

1 1 13
=0

6 
4 4 2
T (xx1, y1) P Draw CM perpendicular to the line x y − 2  0 (see Fig.
3.28). Therefore,
FIGURE 3.27 ( /2) (1// ) 2 1 13
CM    (radius)
12 12 2 2
37. If the tangents drawn from the origin to the circle
x2 y2 − 2px − 2qy q2   (q ≠ 0) are at right Therefore, x y − 2  0 intersects the circle S  0 in two
angles, then show that p2  q2. distinct points, say, points A and B. Now,
3.3 Classification of Points in a Plane w.r.t. a Circle in the Same Plane 147

P(K − 1, K 1) lies inside the circle S  0 Second, L  0 does not have common point with S  0 š
C1M1  r1  1, where C1M1 is the perpendicular drawn
š(K − 1)2 (K 1)2 − (K − 1) − (K 1) − 6  0
from C1 onto the line. This implies
š 2K 2 2K 4  0 4(1) L
3(1) 4(
1
š (K 1)(K 2)  0 32 42
š 1  K  2 (3.22) š( 7) 2 25
2
Let L  x y − 2  0. Now, P(K − 1, K 1) lies inside the š L 14 L 24  0
circle, in the larger segment of the circle if −1  K  2 and š( 2)(
)( 12) 0
P and C lie on the same side of the chord AB. P(K − 1, K š L  2 or L  12 (3.25)
1) and C(1/2, 1/2) lie on the same side of L  0, if L11 
(K − 1) (K 1) − 2 and L22  (1/2) (1/2) − 2 have the Similarly
same sign. But L22  −1  0. Therefore,
C2 M2  2 š L 2 62 L 861  0
62
L11  ( 1) ( 1) 2 0
š (L 21)  L 41  0
 2K 2  0 š L  21 L  41 (3.26)
K 1 (3.23) The given line L  0 satisfies the specified condition if all
From Eqs. (3.22) and (3.23), we have −1  K  1. the three conditions given in Eqs. (3.24)–(3.26) simulta-
neously hold. Therefore, 12  L  21.

P (l − 1, l + 1)
1, 1
C
2 2 C 2(9, 1)
x +y −2=0
A M B 2

M2
FIGURE 3.28
3x

40. Find the range of K for which the line 3x 4y − K  0 lies


+4
y

in between the circles S y x2 y2 − 2x − 2y 1  0 and


−l

Sa y x2 y2 − 18x − 2y 78  0 without intersecting


=0

and without touching the circles.

Solution: Let L y 3x 4y − K  0.
M1
2
S x y 2x 2y 1  0 1
2 2
Sa x y 18 x 2 y 78  0 C 1(1, 1)

C1  (1, 1) and C2  (9, 1) are the centres and r1  1 and r2


 2 are the radii of S  0 and Sa  0, respectively. First, the
line L  0 lies in between the circles. The centres C1 and
FIGURE 3.29
C2 lie on the opposite sides of L  0 (see Fig. 3.29). Now
L11  3 4 L 41. ABCD is a rectangle. A circle passing through the
vertex C touches the sides AB and AD at M and N,
and L22  3(9) 4(1) L respectively. If the distance of the line MN from the
have opposite signs vertex C is p units, then show that the area of the
rectangle ABCD is p2.
š (7 L )(31 L )  0
Solution: Take A as origin. Let AB  a and AD  b so
š( )( )0 that C  (a, b). Let r be the radius of the circle so that its
š 7  L  31 (3.24) centre O  (r, r) (see Fig. 3.29). Equation of the chord
148 Chapter 3 Circle

MN is x/r y/r  1 or x y  r. Now, C(a, b) is a point on


43. A circle S y x2 y2 4x − 2 2 c  0 is the direc-
the circle (x − r)2 (y − r)2  r2, which implies
tor circle of Sl, Sl is the director circle of S2, S2 is the
2
b2 2r(a b) r 2 0 (3.27) director circle of S3 and so on. If the sum of all the
radii of these circles is 2, then find C.
The distance of C(a, b) from the chord MN is given by
a b r Solution: According to Quick Look 4, the radius of the
p director circle of a given circle is 2 times the radius of
2 the given circle. Therefore, the radius of S  0 is given by
 (a b r ) 2 2 p2
22 ( 2 )2 c  6 c  a (say)
 (a b)2 2r(a b) r 2  2 p2
 a 2 b2 2r(a b) r 2 aab
b p2 Therefore, the radius of S1 is a// , the radius of S2 is a/2
2 and so on. Therefore,
 2ab
b  2p
2ab
a a a
b  p2
 ab [ (3.27)] 2  a d
2 2 2 2
Therefore, the area of ABCD is given by p2.
¥ 1 1 1 ´
 a 1 dµ
C (a, b ) § 2 2 2 2 ¶
D

a 2a
 
N p 1 1/ 2 2 1
O (r,
r r)

This gives
4( 2 1)2 2a 2  2(6
(6 )
A M B

FIGURE 3.30  2(2


2(2 1 2 2 ) 6 c

42. A is a point on the circle x2 y2  2a2. From A, two  4 2


tangents are drawn to the circle x2 y2  a2 whose
44. Find the point of intersection of tangents to the circle
points of contact are B and C. As A moves on the
x2 y2  a2 which are inclined at angles @ and A with
circle x2 y2  2a2, show that the locus of the circum-
the positive direction of the x-axis such that cot @
centre of $ABC is the circle x2 y2  a2/2.
cot A  0.
Solution: According to Quick Look 4, A lies on the di-
Solution: Suppose P(h, k) is the point of intersection
rector circle of x2 y2  a2. Hence, BAC  90° (see Fig.
of the tangents which are inclined at angles @ and A with
3.31). Therefore, the circumcentre of $ABC  midpoint
of BC  midpoint of OA because ABOC is a square, x-axis. y  mx a 1 m2 always touches x2 y2  a2 (k
where O is (0, 0). If A  (h, k), the circumcentre is (h/2, − mh)2  a2(1 m2) which has two distinct roots, say, m1
k/2) and hence h2 k2  2a2. This implies (h/2)2 (k/2)2 
and m2. Let m1  tan @, m2  tan A. Therefore,
a2/2. Therefore, the locus is given by

a2 cot A cot B  0
x2 y2  1 1
2  0
tan A tan B
m m2
 1 0
m1 m2
 2 hk  0
a O (0, 0) Therefore, locus of (h, k) is xy  0.
C

a 45. Find the midpoint of the chord of the circle x2 y2 


25 intercepted on the line x − 2y − 2  0.
A B
Solution: Let M(x1, y1) be the midpoint of the chord.
Therefore, equation of the chord is S1 S11 . That is,
FIGURE 3.31
3.3 Classification of Points in a Plane w.r.t. a Circle in the Same Plane 149

xx1 yy1 25  x12 y12 25 From Eqs. (3.31) and (3.32), we get

xx1 yy1  x12 y12 (3.28) pn


A
lp mq
However, by hypothesis, qn
and B
x − 2y − 2  0 (3.29) lp mq
is the chord. From Eqs. (3.28) and (3.29), we have Therefore
x1 y1 x12 y12 OP – OQ  k 2 A2 B2 p2 q 2 k2
   t (say)
1 2 2
x1 t y1 2t n 2 ( p2 q 2 )
 – p2 q 2  k 2
and 2tt x12 y12 2
t  t  0 or
2 (lp mq)2
5
 n( p2 q2 ) 2
((llp mq)
Here, t  0 implies that (x1, y1) = (0, 0) is not on the line
x 2 y 2  0. Therefore, n the curve n( x 2 y2 )
Hence, Q( p, q) lies on k 2 (lx my).
¥ 2 4 ´
( 1 , y1 ) ,

lx
§5 5 ¶

+
m
y+
46. Tangent at any point on the circle x2 y2  a2 meets

n
=
0
the circle x2 y2  b2 at points P and Q. If the tan-
gents drawn at points P and Q of this circle intersect
at right angles, then show that b2  2a2.

Solution: Let S y x2 y2 − a2  0 and Sa y x2 y2 − b2  0. P


By hypothesis, b  a. Suppose that the tangents at points O Q
P and Q to Sa  0 meet in T(h, k). Since PTQ  90°, T
must lie on the director circle of Sa  0. Therefore, FIGURE 3.32

h2 k 2  2b2 (3.30) 48. For all values of the parameter @, show that the locus of
Now, equation of PQ is hx ky − b2  0 and it touches the the point of intersection of the lines x cos @ y sin @ 
circle S  0 so that p and x sin @ − y cos @  q is the circle x2 y2  p2 q2.

b2 Solution: Squaring and adding the given equations, we


a have x2 y2  p2 q2 which represents circle with centre
h2 k 2
as origin and radius p2 q2 .
and hence b4  a2(h2 k2). Therefore, from Eq. (3.30), we
get b2 a 2. 49. A circle touches the line y  x at a point P whose

47. Let us consider that the line lx my n  0 does not distance from the origin is 4 2. The point (−10, 2) is
pass through the origin O and P is a point on the an interior point of the circle. The length of the chord
line. On the segment OP, let Q be a point such that on the line x y  0 is 6 2. Find the equation, of the
OP·OQ  k2, where k is a fixed number. Then show circle. (IIT-JEE 1990)
that Q lies on the curve n2(x2 y2)  k2(lx my).
Solution: Let C(h, k) be the centre of the circle. Now,
Solution: Let P  (@, A ). Since P lies on the line lx P  (x, x) and OP  4 2  2 2  32  4. Let M
my n  0, we have be the midpoint of AB (see Fig. 3.33) where AB  6 2.
Therefore
l@ mA n  0 (3.31)
AM  MB  3 2
Let Q  (p, q). Since the equation of OP is A x − @ y  0
and Q(p, q) lies on OP (Fig. 3.32), we have Also OPCM is a rectangle šCM  OP  4 2. There-
fore
pB A q  0 (3.32)
150 Chapter 3 Circle

AC 2  ( AM )2 (CM )2  (3 2 )2 (4 2 )2  50 bisector of the side BC follows A  (O, k), where k  0,


we have
Hence, the radius (r) of the circle is AC  5 2. Now
OA k
h k tan 60o  
4 2  CM   h k  p8 (3.33) OC 1
2
k 3
h k
5 2  PC   h k  p10 (3.34) Therefore, A  ( , 3 ). The equation of the perpendicu-
2 lar bisector of AB is given by
From Eqs. (3.33) and (3.34), we have (h, k)  (9, −1), (1, −9), 3 1 ¥ 1´
(−1, 9) or (−9, 1). Therefore, equation of the circle is given y x
2 3 § 2 ¶
by
which meets y-axis at
S ( x )2 (y
( y )2  50
¥ 3 1 ´ ¥ 1 ´
or S ( x )2 ((yy )2  50 ¦ 0, 2 µ  ¦ 0, µ
§ 2 3¶ § 3¶
or S ( x )2 (y
( y )2  50 Therefore, centre is given by
2 2
or S ( x ) (y
( y )  50 ¥ 1 ´
¦§ 0, µ
However, (−10, 2) is an interior point of the circle for 3¶
which S11  0 which is satisfied by the equation
Also
S y (x 9)2 (y − 1)2  50
BC 2 4 2
2R    R
sin A sin 60o 3 3

Therefore, circumradius is given by 22/ 3 . Hence, the


equation of the circumcircle is given by
(−10, 2) C (h, k)
k 2 2
2¥ 1 ´ ¥ 2 ´
x y µ¶  ¦§ µ
§ 3 3¶
B P
6 2
3 2 2 1 4
4 2  x2 y2 y 0
M 3 3 3 3
2 9 °
90
A
O (0, 0)  3( x 2 y2 ) 2 y 3 0
FIGURE 3.33 y

50. Two vertices of an equilateral triangle are (−1, A


0) and (1, 0) and its third vertex lies above the
x-axis. Find the equation of the circumcircle of the
triangle.
60° 60°
Solution: Let B  (−1, 0) and C  (1, 0) be two ver- B (−1, 0) O C (1, 0)
tices of an equilateral triangle ABC (see Fig. 3.34).
Now, BC  AC  AB  2 and OC  OB  1. Since the
FIGURE 3.34
third vertex lies above the x-axis on the perpendicular

3.4 Relation Between Two Circles


Let C1 and C2 be two circles with centres A and B and radii r1 and r2, respectively.
1. C1 and C2 do not have any common point š AB  r1 r2 [see Fig. 3.35(a)].
3.4 Relation Between Two Circles 151

2. C1 and C2 touch each other externally šAB  r1 r2 [see Fig. 3.35(b)].


3. C1 and C2 intersect in two distinct points š 1 r2  AB  r1 r2 [see Fig. 3.35(c)].
4. C1 and C2 touch each other internally š AB  r1 r2 [see Fig. 3.35(d)].
5. One circle lies completely within the other without having common point š AB  r1 r2 [see Fig. 3.35(e)].

C2

C1
C1 C2

r1 r2 r1 r2
A B A P B

(a) (b)

C2
C2
C1
C1

P
B A
A B

(c) (d)

C2
C1

A
B

(e)

FIGURE 3.35

QUICK LOOK 7

1. If the two circles touch externally at a point P, then 3. Whether the two circles touch internally or exter-
A, P and B are collinear and P divides AB inter- nally, the point of contact is the foot of the perpen-
nally in the ratio r1: r2. dicular drawn from either of the centres onto the
2. If the two circles touch internally, then P divides common tangent.
AB externally in the ratio r1: r2.
152 Chapter 3 Circle

T H E O R E M 3.14 If S  0 and Sa  0 are non-concentric circles in the standard form, then the equation S − Sa  0
represents a straight line perpendicular to the line joining the centres.
PROOF Let

S x 2 y2 2 gx
gx fy c  0
2 2
Sa x y 2g
g ax
ax f ay c a  0
and ( , f ) ( g a, ). Therefore, S − Sa y 2(g − ga)x 2(f − f a)y c − ca  0 is a first-degree
equation and hence it represents a straight line. Since the slope of the line joining the centres is
f − f a/g − ga and the slope of S − Sa  0 is −(g − ga)/f − f a, S − Sa  0 is perpendicular to the line join-
ing the centres.

QUICK LOOK 8

1. If the circles S  0 and Sa  0 cut in points P and Q, 2. If the circles S  0 and Sa  0 touch each other at
then S − Sa  0 passes through both P and Q and point P, then S − Sa  0 is the common tangent of
hence S − Sa  0 is the common chord PQ of the the two circles at point P and P divides the line
circles. joining the centres in the ratio of their radii.

T H E O R E M 3.15 Let S y x2 y2 2gx 2fy ca  0 and L y ax by c  0 be a line. Then for all values of K, the
equation S KL y x2 y2 2gx 2fy ca K(ax by c)  0 represents a family of circles.
PROOF S KL y x2 y2 (2g Ka)x (2f Kb)y Kc ca  0 which represents circle in the sense of the
Note given under Theorem 3.2.

QUICK LOOK 9

1. If the line L  0 intersects the circle S  0 in two dif- S KL  0 where L y S − Sa  0 represents a circle.
ferent points P and Q, then S KL  0 represents a In particular, if S  0 and Sa  0 intersect, then S
circle passing through points P and Q. KL  0 represents a circle passing through their
2. If L  0 touches S  0 at point P, then S KL  0 points of intersection. Further, if S  0 and Sa  0
represents a circle touching S  0 at point P and L  touch each other at point P then S KL  0 where
0 is the common tangent at point P. L y S − Sa  0 represents circle touching both S  0
3. If S  0 and Sa  0 are two non-concentric circles and Sa  0 at point P.
in the standard form and L y S − Sa  0, then

Example 3.7

Show that the two circles x2 y2 − 4x 6y 8  0 and distance between the centres (see Fig. 3.36) is given by
x2 y2 − 10x − 6y 14  0 touch each other and find the
coordinates of the point of contact. AB  ( )2 ( )2  45  3 5
Therefore
Solution: Let us consider that
3 5  AB  r1 r2
S x 2 y2 4x 6y 8  0
So, the two circles touch each other externally. Suppose
Sa x 2 y2 10 x 6 y 14  0 P(x, y) is the point of contact. Then
Centre and radius of S  0 are A(2, −3) and r1  5 ; AP : PB 5 : 2 5  1: 2
centre and radius of Sa  0 are B(5, 3) and r2  2 5 . The
3.4 Relation Between Two Circles 153

Hence Therefore, P  ( , 1).


1(5) 2(2) 1 2
x 3
1 2
A (2, −3) P B (5, 3)
1(3) 2( 3)
and y  1
1 2 FIGURE 3.36

Example 3.8

Show that the two circles S y x2 y2 4y − 1  0 and Sa y AP : PB r1 : r2  2 : 1


x2 y2 6x y 8  0 touch each other. Find the common
tangent at the point of contact and the point of contact. So
1(0) 2( 3)
Solution: A ( , 2) r1  , respectively, are the x  2
2 1
centre and the radius of S  0. Similarly B  (−3, −1/2)
and 2 5/ 2, respectively, are the centre and the radius ¥ 1´
1( 2
) 2
of Sa  0. The distance between the centres (see Fig. 3.37) § 2¶
and y
is given by 2 1
2 Therefore, P  ( , 1).
¥ 1 ´ 9 3 5
AB  ( )2 ( )  9   r1 r2
§ 2 ¶ 4 2 2 1
A (0, −2) P −1
B −3,
Therefore, S  0, Sa  0 touch each other externally. The 2
common tangent is S − Sa y 2x − y 3  0. Suppose P(x, y)
FIGURE 3.37
is the point of contact. Therefore

Example 3.9

Find the equation of the circle whose diameter is the which lies on the common chord L y S − Sa  ax − by  0.
common chord of the circles (x − a)2 y2  a2 and x2 Therefore
(y − b)2  b2.
¥ 2a L a ´ ¥ Lb´
a b 0
Solution: Let us consider that § 2 ¶ § 2 ¶

S ( x a)2 y2 a 2 x 2 y2 2aax  0 2a 2 L a 2
 2a L b2 0

Sa x 2 ( y b)2 b2 x 2 y2 by  0 2a 2
L
The common chord equation is L y ax − by  0 which is a2 b2
S − Sa  0. Equation of the circle described on L  0 as Hence, equation of the given circle is given by
diameter is of the form
2a 2
S  KL  x2 y2 − 2ax K(ax − by)  0 S LL y S L0
a2 b2
whose centre is given by
2a 2
 x 2 y2 2ax (ax by)  0
¥ 2a L a L b ´ a2 b2
,
§ 2 2 ¶
 (a 2 b 2 ) ( x 2 y 2 ) b2 x 2a 2 by  0
aab

Example 3.10

Find the equation of a circle passing through the point Solution: L y S −Sa y −5x 5y  0 is the common chord
(2, 1) and the points of intersection of the circles S y x2 of S  0 and Sa  0. Any circle passing through the inter-
y2 − 2x 3y − 1  0 and Sa y x2 y2 3x − 2y − 1  0. section of S  0 and Sa  0 is of the form
154 Chapter 3 Circle

S KL  x2 y2 −2x + 3y −1 K(x − y)  0 Therefore, the required equation is x2 y2 − 2x 3y −


The given circle passes through the point (2, 1). This 1 − 3(x − y)  0. That is,
implies that x2 y2 5 x 6 y 1  0
2 2 12 2(2) 3(1) 1 L (2 1)  0  L  3

Example 3.11

The line 2x 3y  1 cuts the circle x2 y2  4 at points Since AB is the diameter of S L L  0 and ( , L/ )
A and B. Show that the equation of the circle described lies on L  0, we have
on AB as diameter is 13(x2 y2) − 4x − 6y − 50  0.
¥ 3L ´
2( ) 3 1
§ 2 ¶
Solution: Let S y x2 y2 − 4  0 and L y 2x 3y − 1  0.
The required circle equation is 2
L
13
S KL x2 y2 − 4 K(2x 3y − 1)  0
Therefore
The centre of the circle is
2
¥ 3L ´ S L L y x 2 y2 4 (2 x 3 y 1)  0
L, 13
§ 2 ¶ 2 2
 13( ) 4 6 y 50  0

Example 3.12

The condition for the circle S y x2 y2 2gx 2fy c  0 š Common chord L y S − Sa  0 is diameter of the
which bisects the circumference of the circle Sa y x2 y2 circle Sa  0
2gax 2f ay ca  0 is that 2(g − ga)ga 2(f − f a)  c − ca.
š The centre of S a  lies li on L  0
Solution: We have š 2(( a)( ( a) ( a)( ( a) c c a  0
S  0 bisects the circumference of Sa  0 š 2(( g g a)g a ( f f a) f a  c c a.

DEFINITION 3.4 Angle of Intersection Suppose point P is a common point of two circles C1 and C2. Then the
angle between the tangents drawn to the circles at point P is called the angle of intersection of
C1 and C2 (see Chapter 3, Vol. 3).

DEFINITION 3.5 Orthogonal Circles If the angle of intersection of two circles is a right angle, then the two
circles are said to intersect orthogonally.

QUICK LOOK 10

Two circles C1 and C2 with centres A and B, respec-


tively, intersect each other orthogonally at P if and
only if AP is tangent to C2 and BP is tangent to C1 (see P
Fig. 3.38). That is, if and only if $APB is right-angled r1 90° r2
with right angle at P (see Fig. 3.38)
A B

FIGURE 3.38
3.4 Relation Between Two Circles 155

T H E O R E M 3.16 Let C1 and C2 be two circles with centres at points A and B, respectively, which intersect at point
P. If P is the angle of intersection of the circles, then

( )2 ( )2 ( )2 ( )2 r12 r22
cos Q  
2( )( ) 2r1r2

where r1 AP
A P d r2  BP.

PROOF Suppose the tangent to circle C2 at point P meets the line AB at point T1 and the tangent to circle
C1 at point P meets AB at point T2 (see Fig. 3.39). Therefore, PT2 is at right angles at AP and PT1
is at right angles to PB. It is given that T1 P
PTT2  P. Now,
APT
T1  90° − P and BPT
T2  90° − P  APB  90° (90° − P)  180° − P
Therefore, using cosine rule to $PAB, we get
( )2  ( )2 ( )2 2( )( )cos(180o Q )
2 2
( ) ( ) 2( )(BP )cos Q
2
( AB) ( AP ) ( BP ) 2
( AB)2 r12 2
r22
 cos Q  
2( AP
A )( BP ) 2r1r2

C2

C1

r1 q r2

A T1 T2 B

FIGURE 3.39

QUICK LOOK 11

The circles intersect orthogonally if and only if P  90°. That is, if and only if (AB)2  r12 r22.

Theorem 3.17 gives algebraic condition for two circles to cut orthogonally.

T H E O R E M 3.17 The condition for two circles S y x2 y2 2gx 2fy c  0 and Sa y x2 y2 2ga(x) 2f ay ca  0
to cut orthogonally is that 2gga 2ffa  c ca.

PROOF Let A ( g, f ), r1  g 2 f 2 c , B  ( g a, f a) and r2 g a2 f a2 c a . The two circles cut


each other orthogonally
2 2
š  AB  1 r2 2 Quick Look 11
š ( g g a )2 ( f f a )2  ( g 2 f 2 c ) ( g a 2 f 2
c a)

š 2 gg a 2 ff a  c c a
156 Chapter 3 Circle

Example 3.13

Show that the circles S y x2 y2 − 8x − 6y 21  0 and Now,


Sa y x2 y2 − 2y − 15  0 cut orthogonally.
(AB)2  42 22  20  rl2 r22
Solution: For S y0, the centre and radius are According to Quick Look 11, the two circles cut orthogo-
2 2
nally. Also, we haveg  −4, f  −3, c  21 and ga  0, f a  −1,
A ( , 3), r1  4 3 21  2 ca  −15. Therefore,
For Sa y0, the centre and radius are
2gga 2ff a  2(−4)(0) 2(−3)(−1)  6  c ca
B ( , 1), r2  1 15  4 Hence, by Theorem 3.17, the two circles cut orthogo-
nally.

Example 3.14

Find the angle of intersection of the circles S y x2 y2 − ( )2 r12 r22 5 ( ) ( ) 5


16  0 and Sa y x2 y2 − 4x − 2y − 4  0. cos Q   
2r1r2 2(( )( ) 6
Solution: Let P be the angle of intersection of the cir- 1 ¥ 5 ´ ¥ 5´
Q  C  P Cos 1
cles. For S y 0, centre and radius are A  (0, 0), r1  4. § 6¶ § 6¶
For Sa y0, centre and radius are B ( , 1) r2  . By
Theorem 3.16, we have

Note: One may think that we can find the angle of intersection by using calculus method. The difficulty in using the
calculus method is that we should find the coordinates of the point of intersection which is a cumbersome process. This
is not necessary in geometrical method.

T H E O R E M 3.18 Let S  0 be a circle and A(x1, y1) be a point in the plane of the circle. Let L is a line through A
meeting the circle S  0 in P and Q. Then AP·AQ is constant. That is, AP·AQ is independent of the
line L.
PROOF Let S y x2 y2 2gx 2fy c  0. Suppose the parametric equations of the line L  0 be
x  x1 r cos P and y  y1 r sin P
Put x  x1 r cos P and y  y1 r sin P in S  0. Therefore,
( 1 cos Q )2 in )2 2 g((
( y1 r sin 1 ccos ) ( y1 r sin
in ) c  0
That is,

r 2 2[( x1 g) ( y1 f ) n ]r S11 0 (3.35)

Since Eq. (3.35) is a quadratic equation in r, it will have two roots, say, r1 and r2. L  0 meeting the
circle implies that r1 and r2 are real and AP, AQ are equal to r1 and r2. Therefore, AP·AQ  r1r2  S11,
which is constant. S11 value depends on A(x1, y1), but not on L  0.

QUICK LOOK 12

If A(x1, y1) is an external point, then we know that from A(x1, y1) (see Theorem 3.11). Hence, A(x1, y1) is
S11  0 and S11 is the length of the tangent drawn external point. This implies AP·AQ  S11  Square of
the tangent from point A.
3.4 Relation Between Two Circles 157

DEFINITION 3.6 Power of a Point Let S  0 be a circle and A(x1, y1) a point in the plane of the circle. If a line
L through point A meets the circle at points P and Q, then the power of A with respect to the
circle is defined as
1. AP·AQ if A is external point.
2. − (AP·AQ) if A is internal point.
3. 0 if A lies on the circle.

QUICK LOOK 13

According to Theorem 3.18, the power of a point power is positive or negative or zero when the point
A(x1, y1) with respect to the circle S  0 is S11. The lies outside or inside or on the circle, respectively.

Examples

1. The power of the point A(2, 3) with respect to the 2. The power of the point (1, 1) with respect to the circle
circle S y x2 y2 − 4x − 2y − 6  0 is S11  22 32 − Sy x2 y2 4x 2y − 1  0 is S11  12 12 4(1) −
4(2) − 2(3) − 6  −7. 2(1) − 1  7.

T H E O R E M 3.19 Let S  0 and Sa  0 be two non-concentric circles. Then, the locus of the point whose powers
with respect to the two circles are equal is a straight line perpendicular to the line joining the
centres.
PROOF Let S y x2 y2 2gx 2fy c  0 and Sa y x2 y2 2gax 2fay ca  0 be two non-concentric
circles [i.e. (−g, −f) x (−ga, −fa)]. A(x1, y1) is a point on the locus š S11  Sa11. This implies

x12 y12 2 gx1 2 fy


f 1 c  x12 y12 2g
g axx1 f ay1 c a
š 2( g g a) x1 ( f f a) y1 c c a  0

Therefore, the locus of A(x1, y1) is the line


S S a y 2( g ggaa) x ( f f a) y c c a  0

Note: Since the circles are non-concentric, S − Sa  0 is a first degree equation and hence it is a straight line. Also, one
can see that the line S − Sa  0 is perpendicular to the line joining the centres.

DEFINITION 3.7 Radical Axis The locus of the point whose powers with respect to two non-concentric
circles are equal is a straight line called the radical axis of the two circles.

QUICK LOOK 14

1. The radical axis of the circles S  0, Sa  0 is S − Sa  0. 3. If the two circles touch, then the common target at
2. S − Sa  0 is the common chord if they intersect. the point of contact is the radical axis.
That is, if the two circles intersect, then their radical
axis is their common chord.

T H E O R E M 3.20 The radical axes of three circles (see Fig. 3.40) with non-collinear centres taken two by two are
concurrent and this point is called the radical centre of the three circles.
158 Chapter 3 Circle

C1
L1

C2

A B

Radical
L3 centre L2

C3

FIGURE 3.40
PROOF Let S  0, Sa  0 and Sq 0 be those circles (in the standard form) with non-collinear centres. Then
by Theorem 3.19, L1  S − Sa  0, L2  Sa − Sq  0 and L3  Sq − S  0 are the radical axes of the pairs
(C1, C2), (C2, C3) and (C3, C1), respectively. Now L1 L2 L3  0 implies that the three lines
L1  0, L2  0, and L3  0 are concurrent (see Theorem 2.14) where C1, C2 and C3 are S  0, Sa  0
and Sq  0, respectively.

Since the radical centre lies on all the three radical axes, its powers with respect to all the three circles are equal. Hence
the radical centre lies outside the three circles or lies inside the three circles.
If the radical centre lies outside the circles, then the length of the tangent drawn from it to the circles is same and
hence if a circle is drawn with the centre at the radical centre and length of the tangent drawn from it to the circles as
radius, then this circle cuts all the three circles orthogonally.

Example 3.15

Find the radical centre of the circles circles is (33/4, 7/3) with respect to S  0. The power of
(33/4, 7/3) is equal to
S x 2 y2 16 x 60  0
2 2
Sa x 2 3 y2 36 x 81  0 ¥ 33 ´ ¥ 7´ ¥ 33 ´ ¥ 1089 ´ ¥ 49 ´
¦§ µ¶ ¦§ µ¶ 16 ¦§ µ¶ 60  ¦§ µ ¦ µ 72
4 3 4 16 ¶ § 9 ¶
S aa x 2 y2 16 x 12 y 88  0
(10585 10368)

Solution: We have 144
217
L1 S Sa x 33 0 
144
L2 S a S aa 4 x 12 y 61
61 0
L3 S aa S 1 y 28  0 Therefore, the required circle is
2 2
From L1  0 and L3  0, we have x  33/4, y  7/3. Also ¥ 33 ´ ¥ 7´ 217
x y 
(33/4, 7/3) lies on L2  0. Hence, the radical centre of the § 4¶ § 3¶ 144

Example 3.16

Find the equation of the circle which cuts orthogo- Sa y x2 y2 2x 4y − 4  0 and Sq y x2 y2 − 7x −


nally all the three circles S y x2 y2 x 2y 3  0, 8y − 9  0.
3.5 Common Tangents of Two Circles 159

Solution: We have 2 2
¥ 47 ´ ¥ 34 ´ 47 ¥ 34 ´ 3455
S11  2 3
L1 S Sa x 2y
2y 7  0 § 3¶ § 3¶ 3 § 3¶ 9
L2 S a S aa 9 x 12 y 5  0
So, the required circle is
L3 S aa S 8 x 10 y 1122 4 x 5y 6  0
2 2
¥ 47 ´ ¥ 34 ´ 3455
Solving L1  0 and L3  0, we have x  (−47)/3 and x y 
§ 3¶ § 3¶ 9
y  34/3. Therefore, the radical centre is given by
( / 3,, / 3). Now

3.5 Common Tangents of Two Circles


Let C1 and C2 be two non-concentric circles. Suppose A and B are their centres and r1 and r2 are their radii, respectively.
Case 1: Suppose circles C1 and C2 are such that one circle completely lies outside the other. That is, AB  r1 r2 [see
Fig. 3.41(a)].In this case, there are four common tangents. Two are called direct tangents and the other two are
called transverse tangents. A tangent is called direct common tangent if the centres of the circles lie on the
same side of the target. Transverse common tangent means that the centres lie on the opposite sides. If the radii
are different, then the direct common tangents intersect in a point which is called the external centre of simili-
tude denoted by S2. S2 divides externally the line joining the centres in the ratio of their radii. The two trans-
verse common tangents intersect in a point is called the internal centre of similitude denoted by S1. S1 divides
the line joining the centres internally in the ratio of their radii [see Fig. 3.41(b)].
C1
r1 C2 C1
r2 C2
r1
A S2
B
A B
S1 r2

AB > r 1 + r 2
(b)

FIGURE 3.41 (a) Direct common tangents. (b) Transverse common tangents.

Case 2: Suppose circles C1 and C2 touch each other externally. Then the number of common tangents is three. Two
tangents are direct and one tangent is transverse (see Fig. 3.42).
C1

A r1 r2
B C2

AB = r 1 + r 2

FIGURE 3.42

Case 3: Suppose circles C1 and C2 intersect in two distinct points. Then the number of common tangents is two which
are direct. In this case, r1 r2  AB  r1 r2.
Case 4: Suppose circles C1 and C2 touch each other internally. That is, AB  r1 r2 . In this case, there is only one
common tangent.
Case 5: If one of the two circles C1 and C2 completely lies within the other, there is no common tangent.
160 Chapter 3 Circle

Example 3.17

Consider S y x2 y2 − 14x 6y 33  0 and Sa y x2 y2 Hence, the direct common tangents are


30x − 2y 1  0. Find the number of common tangents
and their equations. ¥ 3 ´
y 5  ¦ µ ( x 18)
§ 4¶
Solution: The centre A and radius r1 of S  0 are (7, −3) ¥ 7´
and y 5  ¦ µ ( x 18)
and 5. The centre B and radius r2 of Sa  0 are (−15, 1) and § 24 ¶
15. The distance between the centres is given by
Suppose S1 is the internal centre of similitude so that S1
2 2 2 2 divides BA internally in the ratio 15:5  3:1. Therefore,
AB  ( ) (( )  22 4  500
 10 5  5 15 ¥3 ´
S1  , 2
§2 ¶
Therefore, the two circles do not have any common point.
Suppose S2 is the external centre of similitude. Then S2 Now, y 2  m[x – (3/2)] touches S  0 which implies
divides BA in the ratio 15:5  3:1 externally. Therefore that
¥ 11( 15) 3(7) 1(1) 3( 3) ´ 3
S2  ,  (18
(18, 5) m( ) 3 2
§ 2 2 ¶ 2 5
Let y 5  m(x − 18) touch S  0. Therefore 1 m2

m( ) 3 5 ( m )2 100 m2 100
5
1 m2  21m2 44 m 96  0
( m )2 25(( m2 )  21m2 72 m 28 m 96  0
 3m(7m 24) 4(7 m 24)  0
 96 m2 44 m 21  0
4 24
m ,
 96 m2 72 m 28 m 21  0 3 7
 24 (4 3) 7(4 m 3)  0 Therefore, the transverse common tangents are
 (24 7)(4 m 3)  0 4¥ 3´
y 2 x
3 7 3§ 2¶
  ,
4 24 24 ¥ 3´
and y 2 x
7 § 2¶

Though the following theorem is not of much practical usage, we state it without proof.

T H E O R E M 3.21 If (x1, y1) is an external point to a circle S  0, then the combined equation of the pair of tangents
(P A I R O F drawn from (x1, y1) to the circle is
TANGENTS)
S12  SS11

Note: If S y x2 y2 2gx 2fy c  0 is a circle and (x1, y1) lies outside the circle, then the combined equation of the
pair of tangents is

[ 1 1 ( x x1 ) ( y y1 ) ]2 ( 2 2
)(xx12 y12 2 gx1 2 f
)( 1 c)

Subjective Problems (Sections 3.4 and 3.5)


1. Find the equations of the tangents to the circle S y x2 Solution: Let y 3  m(x − 3) be a tangent to S  0. The
y2 8x 4y − 5  0 from the point (3, −3) and also centre of the circle is (−4, −2) and its radius is 5. There-
write their combined equation. fore
3.5 Common Tangents of Two Circles 161

(See point 3 of Quick Look 9). This circle passes through


m( ) 2 3
5 origin š − a2 − L a2  0 š L  −1. The required circle is
1 m2
x2 y2 − 2cy − a2 − (bx − cy − a2)  0
( m )2 25( m2 )  x2 y2 − bx − cy  0
 24 m2 14 m 24  0 which also passes through the centres (0, c) and (b, 0) of
the circles S  0 and Sa  0, respectively.
 12 m2 7 m 12 0
4. Suppose the lengths of the tangents drawn from A(x1,
2
 12 16 m 9 12 0 y1) and B(x2, y2) to the circle x2 y2  a2 are of lengths
l1 and l2, respectively, and x1x2 y1y2  a2. Then show
 4 (3 4) 3(3 4)  0
that (AB)2 is equal to l12 l22.
3 4
  , Solution: We have
4 3
Therefore, the tangents are given by y 3  (3/4)(x − 3) or l12 S11  x12 y12 a 2 l22  x22 y22 a 2
3x − 4y − 21  0 and y 3  (−4/3)(x − 3) or 4x 3y − 3 
Therefore
0. Combined equation is (3x − 4y − 21)(4x 3y − 3)  0.
( )2  ( 1 2)
2
( 1 2)
2

Note: The two tangents from (3, −3) are at right angles
2 2 2 2
and hence (3, −3) lies on the director circle of S  0. One ( 1 1 ) ( 2 2 ) 2( 1 2 1 2)
can check this fact.
 ( x12 y12 ) ( x22 y22 ) 2a 2 (by hypothesis)
2. Find the equations of the tangents drawn from origin
to the circle S y x2 y2 − 6x − 2y 8  0.  ( x12 y12 a 2 ) ( x22 y22 a 2 )

Solution: S11  8  0  (0, 0) is external point to S  0.  l12 l22


The centre of the circle is (3, 1) and its radius is 2. We
know that y mx touches the circle. So 5. Find the equation of a circle which bisects the circum-
ferences of the circles x2 y2  1, x2 y2 2x  3 and
m( ) 1 x2 y2 2y  3.
 2
1 m2
Solution: Let the given circles be C1, C2 and C3, re-
š( m )2 2(( m2 ) spectively. Let C be the required circle and its equa-
tion be S y x2 y2 2gx 2fy c  0. Let Sa y x2
7 2 6m 1
š 7m 0 y2 1  0. Since C bisects the circumference of C1, the
line S − Sa y 2gx 2fy c 1  0 passes through the
š (m
( )( m ) 0 centre of C1  (0, 0). Therefore,
Therefore c 1
1 Now, C bisects the circumference of C2  the line S −
m  1,
7 Sq y 2(g − 1)x 2fy c 3  0 passes through the centre
(−1, 0) of C2. Therefore
Hence, the tangents are y  x a d x y  .
2( 1)( 1) 1 3 0
3. Let the circles S y x2− 2cy − y2  0 and Sa y a2
 2 g 2 1 3  0
x2 y2 − 2bx a2  0 whose centres are A and B, re-
spectively, intersect at points P and Q. Show that the g2
points P, Q, A, B and origin are concyclic.
Similarly, since the circle C bisects the circumference of
Solution: Let L y S − Sa y bx – cy − a2  0. The re- circle C3, we have f  2. Therefore, equation of the
quired circle equation is of the form required circle C is

S L L y x 2 y2 cyy a 2 L (bx cy a 2 )  0
b cy S y x2 y2 4x 4y − 1  0
162 Chapter 3 Circle

6. Find the equation of a circle that passes through the Solution: Let S y x2 y2 2gx 2fy c  0 be the re-
point (1, 2) which bisects the circumference of the quired circle. Now S  0 touches the line x  2a implies
circle x2 y2  9 and is orthogonal to the circle x2
y2 − 2x 8y − 7  0. g 2a  g 2 f 2 c (3.41)
S  0 cuts orthogonally x2 + y2 a2  0 implies
Solution: Let
2g(0) + 2 f (0) c a  c  a2
2
(3.42)
2 2
Sa y x y 9  0 From Eqs. (3.41) and (3.42),
2 2
S aa y x y 2 x 8 y 7  0 (g 2a)2  g2 f2 − a2
Let S y x2 y2 2gx 2fy c  0 be the required circle.  y2 g 2 4aa 2 ag  g 2 f 2 a 2
Since S  0 passes through (1, 2), we have f 2 a( gg ) a 2  0
2 g 4 f c  5 (3.36) Therefore, the locus of the centre (−g, −f) is given by
S  0 bisects the circumference of Sa  0  S − Sa  0
y2 a 5a 2  0
ax
passes through (0, 0). This gives
¥ 5a ´
c 9  0  y 2  4a x
§ 4¶
 c  9 (3.37)
S  0 and Sq  0 cut orthogonally which implies that 9. Show that the equation of a straight line meeting
the circle x2 y2  a2 in two points at equal dis-
2 g( 1) 2 f (4)  c 7 tance d from a point (x1, y1) on the circumference is
2 2
 2 g 8 f  c 7 (3.38) 2( 1 1 ) 0.
From Eqs. (3.36)–(3.38), we have
Solution: Let S y x2 y2 − a2  0. See Fig. 3.43. The
2g 4 f  9 5  4 required line is the common chord of the circle S  0 and
and 22 g 8 f  9 7  16 Sa y (x − x1)2 (y − y1)2  d2. Therefore, the line equation
is given by
Therefore, g f  1, and c  99. Hence the required
circle is S S a y a 2 2 xx1 2 yy1 ( x12 y12 ) d 2  0

S y x2 y2 8x − 2y − 9  0  2 xx1 2 yy1 2a 2 d2  0 ( x12 y12 a2 )

7. Find the equation of the circle which passes through


origin, has its centre on the line x y  4 and cuts or-
thogonally the circle x2 y2 − 4x 2y 4  0.

Solution: Let S y x2 y2 2gx 2fy c  0 be the


O (0, 0) P
required circle. S p h gh ( , )  c  0. The
centre (−g, −f) lies on the line x y  4 implies that
d
g f  4 (3.39)
S  0 cuts the circle Sa y x2 y2 − 4x 2y 4  0 implies d A (xx1, y1)
that
Q
2 g( 2) 2 f (1)  c 4
 4 g 2 f  c 4 FIGURE 3.43

 4 g 2 f  4 (3.40) ( )
10. Prove that the two circles x2 y2 2ax c  0 and x2
Solving Eqs. (3.39) and (3.40), we have g  −2 and f  −2. y2 2by c  0 touch each other if and only if
Therefore 1 1 1

S x y2 2
4x 4y  0 a2 b2 c

Solution: Let
8. Find the equation of the locus of the centres of all cir-
cles which touch the line x  2a and cut orthogonally S x 2 y2 2aax c  0
the circle x2 y2  a2.
Sa x 2 y2 by c  0
3.5 Common Tangents of Two Circles 163

The two circles touch each other š S − Sa  0 is the common as y-axis. Their centres lie on the x-axis. Therefore, we can
tangent at the point of contact. This implies that the dis- write their equations as
tance of the centre of one of the circles from the line S − Sa
 0 is equal to the radius of the circle. Therefore S x 2 y2 2ax
a c  0

a( a) b( ) and Sa x 2 y2 bx c a  0
b
 a2 c ( S S ax by )
a2 b2 Since the origin lies on the radical axis of the circles C1
4 2 2 2 and C2, its powers with respect to the circles are equal.
 a  (a
(a b ))(a
(a c)
Hence, C  C1. Therefore, the equations of C1 and C2 are
4 4
 a  a c(a b ) a 2 b2 2 2
S y x2 y2 2ax c  0 and Sa y x2 y2 2bx c  0.
2
 ( b2 )  a 2 b2
14. Find the equation of the circle whose centre lies on
a 2 b2
1
  the line x y − 11  0 and which passes through the
a 2 b2 c intersection of the circle x2 y2 − 3x 2y − 4  0 and
1 1 1 the line 2x 5y 2  0.
 2 2 
a b c
Solution: Let S y x2 y2 − 3x 2y − 4  0 and L y 2x
11. Find the equation of the circle which cuts orthogo- 5y 2  0. The required circle equation is of the form
nally each of the circles
S L L y x 2 y2 x y L (2 x 5 y 2)  0
Sa x 2 y2 2x 17y 4  0
S aa x 2 y2 7 x 6 y 11  0 y x 2 y2 (3 2 L ) x ( L ) y 4 2 L  0
S aaa x 2 y2 x 22 y 3  0 The centre [(3 − 2K)/2, −(2 5K)/2] lies on the line
x y 11  0
Solution: Radical axis of Sa  0 and Sq  0 is Sa − Sq y
5x − 11y 7  0 and the radical axis of Sq  0 and Saq  0 is 3 2 L 2 5L
 11  0
Sq − Saq y 8x − 16y 8  0. That is, 2 2
x 111y 7  7 L 21  0
and x 2y 1  L  3
Solving these equations, we obtain the radical centre as Therefore, the required circle is
(3, 2). If t is the length of the tangent from (3, 2) to the
circle Sa  0, we have x2 y2 − 3x 2y − 4 − 3(2x 5y 2)  0

t Sa11  9 4 6 34 4  57  x 2 y2 9 x 13 y 10  0
Therefore, the required circle is (x − 3)2 (y − 2)2  57.
15. A circle touches both axes and also touches the line
4x 3y − 6  0 and lies in the first quadrant. Find the
12. Prove that the locus of the midpoint of a system of
equation of the circle.
parallel chords of a circle is a diameter of the circle.
Solution: (x − h)2 (y − h)2  h2 (h  0) touches both
Solution: Let S y x2 y2 − a2  0. Let M(x1, y1) be the
positive axes. This circle also touches the line 4x 3y −
midpoint of a chord parallel to a line lx my n  0. Equa-
6  0. Therefore
tion of the chord in terms of (x1, y1) as its midpoint is
xx1 yy1  x12 y12 4( ) 3h 6
h
But this is parallel to lx my n  0. This implies lx1 − 42 32
my1  0. Therefore, the locus of M(x1, y1) is lx − my  0
 (7
(7 6)2 255h2
which passes through the centre (0, 0).
 (7
(7 6 5 )( 7 6 5 )0
13. Prove that any two circles with different centres can be
in the form x2 y2 2ax c  0 and x2 y2 2bx c  0,  (12
(12 6) ( )0
c is same for both the equations.
1 3
h ,
Solution: Let C1 and C2 be two circles with A and B as 2 2
their centres. Take the line AB as x-axis, their radical axis
164 Chapter 3 Circle

Therefore, the equations of the circles are given by Hence, the required circle is


2
1´ 1
2 (a2 b2) (x2 y2)  2ab(bx ay)
¥ ¥
x y  (3.43)
§ 2¶ § 2¶ 4 18. On the line x 5y − 22 − (x − 8y 30)  0, the circle
¥ 3´
2
¥ 3´ 9
2 S y x2 y2 − 2x 2y − 14  0 makes an interrupt of
and x y  (3.44) length 2 3. Find the value(s) of L and the line(s)
§ 2¶ § 2¶ 4
equations.
Note: If the line 4x 3y − 6  0 cuts the axes in A and B,
then Eq. (3.43) represents the incircle of OAB and Eq. Solution: Centre and radius of the circle are (1, −1)
(3.44) represents the excircle opposite to the vertex O. and 4 (see Fig. 3.44). The given line equation is

( )x ( ) y 22 30 L 0
16. Determine the equation of the circle which touches
the line y  x at the origin and bisects the circumfer- ( )(( ) ( ))( 22 30 L
) 22
ence of the circle x2 y2 2y − 3  0.   16 3
( )2 ( 5 8 L )2
Solution: Let S y x2 y2 2gx 2fy c  0 be the (by hypothesis)
required circle. It passes through (0, 0). This implies c  0. 39 L 26
Now S  0 touches the line x y  0. Therefore   13
65L 2 78 L 26
g f
 g2 f  ( 3L 2) 2  5L 2 6 L 2
2
 4L 2 6L 2  0
 ( g f )2  0
 2 L 2 3L 1 0
 g  f
 (2 1)( 1) 0
If S  0 bisects the circumference of Sa y x2 y2 2y − 1
3  0, then S Sa  0 passes through the centre (0, 1) of  L  1,
2
S a  0. This implies that
When K 1, the equation of the line is 2x 3y + 8 = 0
2 gx 2( g 1) y 3  0
and when K 1/2, the equation of the line is 3x 2y
passes through (0, 1) (& f g) 14 = 0.
 0 2( g 1)( 1) 3 0
 2g
2 5  2 f
Therefore, S x 2 y2 5 x 5 y  0.
(1, −1)
4
17. Find the equation of the described on the common
chord of the circles (x − a)2 y2  a2 and x2 (y − b)2
3
 b2 as diameter.

Solution: Let S y x2 y2 − 2ax  0 and Sa x2 y2 − 2by FIGURE 3.44


 0. Common chord equation is
19. Find the equation of the circle which passes through
L y S − Sa y ax − by  0 the point (1, 1) and which touches the circle
Equation of the circle passing through the intersection of x2 y2 4x − 6y − 3  0
S  0 and Sa  0 is of the form
at the point (2, 3) on it.
S KL y x2 y2 − 2ax K(ax − by)  0
whose centre is (a − Ka/2, Kb/2). Since the common chord Solution: Let S y x2 y2 4x − 6y − 3  0. The equa-
is a diameter, we have a(a − Ka/2) −b(Kb/2)  0. There- tion of the tangent to S  0 at (2, 3) is
fore S1 y x(2) y(3) 2(x 2) − 3(y 3) − 3  0
2 2 2
L( )  2a  S1 y x 8  0  2  0
2
2a Any circle touching the circle S  0 and having x − 2  0
L
a 2
b2 as a common tangent is of the form S L (x − 2)  0 is
3.5 Common Tangents of Two Circles 165

x2 y2 4x − 6y − 3 L (x − 2)  0 1
and y ( x 2)  x 3 y 2  0
This circle passes through (1, 1) implies that 3

1 1 4 6 3 ( 2)  0
(1 Therefore, T1 is given by
 3 L  0 x 3y 2  0
 L  3 and T2 is given by
Therefore, the required circle is x 3y 2  0
x2 y2 4x − 6y − 3 − 3(x − 2)  0 Any circle touching C and the tangents T1 and T2 must
2 2
 x y x 6y 3  0 have its centre on the x-axis and also touch C at (1, 0)
and (−1, 0) only (see Fig. 3.45). Equation of the tangent
20. Find the equation of the circle which touches the line to C at (1, 0) is x − 1  0. Equation of the circle touching
3x − y − 6  0 at the point (1, −3) and having radius C at (1, 0) is of the form x2 y2 − 1 L (x − 1)  0 whose
2 10. centre is (−K/2, 0) and radius ( 2 / ) . If this
circle also touches x − 3 y 2  0, we have
Solution: S y (x − 1)2 (y 3)2  0 is a point circle and
L y 3x − y − 6  0. Therefore, required circle is of the L
form 3 0 2
2 L2
 L 1
S L L y x 2 y2 x y L (3 x y 6)  0 1 3 4
y x 2 y2 (2 3L ) x (L 6) y 10 6 L  0 4 L 1
  ( L 2) 2
4 2
Radius of the circle  2 10 implies that
 4 L  p ( L 2)
2 2
¥ 2 3L ´ ¥ L 6´  4 L  L 4 L  0
10 6 L 40
§ 2 ¶ § 2 ¶
which corresponds to C. Therefore,
 (2 3K)2 (K 6)2 40 24K  160
4 L 22(L 2)  L 8
10K 2  160
K  p4 and the circle is
Hence, the equations of the circle are given by x2 y2 1 8(
8 x 1) 0
x2 y2 10 x 2y
2 y 14 0 (  4)  x 2 y2 8 x 7  0
x2 y2 14 x 10 y 34  0 ( 4) Similarly, any circle touching C at (−1, 0) is of the form x2
y2 − 1 L (x − 1)  0. The circle also touches the tangent
21. Let T1 and T2 be two tangents drawn from (−2, 0) to T1  L  8/3 and the circle is [x (4/3)]2 y2  1/9.
the circle C: x2 y2  1. Determine the circles touch-
ing C and having T1 and T2 as their pair of tangents. y
Also find the equations of all possible common tan-
gents to those circles when taken two by two.
(IIT-JEE 1999)

(−2, 0) O
Solution: Any line through (−2, 0) is of the form y 
m(x 2)  mx 2m. This touches the circle C. Now
x2 y2  1
 (2m)2 ( m2 ) (by Theorem 3.8)
1 FIGURE 3.45
mp
3
The remaining part is left as an exercise for the students.
The tangents are obtained as
22. C1 and C2 are two concentric circles. The radius of C2
1 is twice that of C1. From a point P on C2, tangents PA
y ( x 2)  x 3 y 2  0
3 and PB are drawn to C1. Prove that the centroid of
166 Chapter 3 Circle

$ PAB lies on C1. (IIT-JEE 1998) C2


C1
Solution: Let the circle C1 be x2 y2  a2. So the equa-
B
tion of C2 must be x2 y2  4a2. Let P(h, k) be on C2 so
that

h2 k 2  4a 2 (3.45)
P A
Equation of AB is

hx ky  a 2 (3.46) FIGURE 3.46


Substituting y  (a2 – hx)/k in x2 y2  a2, we get
23. The chord of contact of a point on the circle x2 y2  a2
2 2
¥ a hx ´ with respect to the circle x2 y2  b2 touches the circle
x2 ¦ µ a
2
x2 y2  c2. Show that b2  ac (i.e. a, b, c are in GP).
§ k ¶
 (h 2 k 2 ) x 2 2a 2 hx
h (a 2 k 2 )a 2 0 Solution: By hypothesis, we have a  b  c. Let P(x1, y1)
 4a 2 2 2
4a x 2a hx (a k )a  0 2 2 2
[from Eq. (3.45)] be a point on x2 y2  a2 so that

Therefore, 4 x 2 2 hx a 2 k 2  0 has two distinct roots, x12 y12  a 2 (3.47)


say x1 and x2. Hence The equation of the chord of contact of P(x1, y1) with
2h h respect to the circle x2 y2  b2 is
x1 x2  
4 2 xx1 yy1 b2  0 (3.48)
a k2
2
x1 x2  The line given in Eq. (3.48) touches the circle x2 y2  c2.
4 This gives
Therefore
0 1 0 y1 b2
a 2
hx1 a hx2 2 c
y1 y2  x12 y12
k k
2
2a h( x1 x2 )  b2  x12 y12 – c  ac [from Eq. (3.47)]

k
2a (h2 /2)
2 24. Prove that the equation x2 y2 − 2x − 2ay − 8  0 rep-
 resents a family of circles intersecting in two points P
k
and Q. Determine a member of this family of circles
4a 2 h 2 such that tangents drawn to this circle at P and Q

2k intersect on the line x 2y 5  0.
h k 2 h2 k
2
  [from Eq. (3.45)] Solution: The given equation is (x2 y2 − 2x − 8) − (2a)y
2k 2
 0 which is of the form S L L  0, where S  0 is the circle
Suppose G(x, y) is the centroid of $ PAB (see Fig. 3.46). x2 y2 − 2x − 8  0 and L  0 is the line y  0 (x-axis). The line
In such case y  0 intersects the circle x2 y2 − 2x − 8  0 at two points
P(−2, 0) and Q(4, 0). Therefore, the given equation rep-
x1 x2 h (h/2) h h
x   resents a family of circles all passing through P(2, 0) and
3 3 2 Q(4, 0).
y1 y2 k k Since the tangents at points P and Q intersect at (x1, y1)
and y 
3 2 which lies on the line x 2y 5  0, we get
Therefore x1 2y
2 y1 50 (3.49)

1 2 1 Also the equation of the line PQ is given by


x2 y2  (h k2 )  (4a 2 ) [from Eq. (3.45)]
4 4 xx1 yy1 ( x x1 ) a( y y1 ) 8 0
Hence, the centroid G( x, y) li C1 .  ( x1 1) x ( y1 a) y x1 ay1 8  0 (3.50)
3.5 Common Tangents of Two Circles 167

However, actually the equation of PQ is ( 3 1)sin 45o ( 3 1)2 2


O2 A    2  r2  2
y0 (3.51) sin 105o 2 ( 3 1)
From Eqs. (3.50) and (3.51), we get x1 − 1  0 and x1 ay1
8  0. Therefore x1  1 and since x1 2y1 5  0 [from
A
Eq. (3.49)], we have y1  −3. Substituting x1  1 and y1 
−3 in x1 ay1 8  0, we have r1 r2

90
1 − 3a 8  0  a  3 4 °
45 30°
O1 O2
M
Hence, the required member of the family is

x2 y2 2 x 6 y 8 0 B

25. Prove that the centre of a circle which cuts orthogo- FIGURE 3.47
nally two given circles lies on the radical axes of the
two circles. 27. Find the centre of the smallest circle which cuts orthog-
onally the two circles x2 y2  1 and x2 y2 8x 8y −
Solution: Let 33  0.

S a y x 2 y2 2 g ax f ay c a  0 Solution: Suppose x2 y2 2gx 2fy c  0 is the re-


quired circle. Since this circle cuts orthogonally the given
S aa y x 2 y2 2 g aaax f aaay c aa  0
two circles, we have c  11 and g f  −4. The radius of
be the two given circles. Suppose a circle S y x2 y2 the circle is
2gx 2fy c  0 cuts orthogonally Sa  0 and Sq  0.
Therefore, by Theorem 3.17, we have g2 f 2 c  g ( g 4) 2 1

2 gg a 2 ff a  c c a  2 g 2 8 g 155  2( g 2)2 7
and 2 gg aa 2 ff aa  c c aa which is minimum if and only if g  −2. Hence, the centre
is (2, 2).
Therefore, 2(ga gq)g 2(f a f q)f  ca − cq. That is, the
radical axis Sa − Sq  0 of Sa  0 and Sq  0 passes through
28. The tangents to a circle at two points P and Q meet
(−g, −f).
at point T. The lines joining points P and Q to one
extremity of the diameter parallel to PQ meet the di-
26. The common chord of two intersecting circles sub-
ameter perpendicular to PQ at points R and S. Prove
tends angles 90° and 60°, respectively, at the centres
that RT  ST.
of the circles and the distance between the centres is
3 1. Find the radii of the circles.
Solution: Without loss of generality, let us consider
that the circle as x2 y2  a2 and let P(h, k) and Q(h, −k)
Solution: Let the common chord AB meet the line
be the ends of the chord PQ (see Fig. 3.48). Equation of
joining the centres O1 and O2 of the circles at point M
the tangent at P(h, k) is hx ky − a2  0 so that T  (a2/h,
(see Fig. 3.47). Then
0). The equation of the line joining P(h, k) and B(0, a) is
AO1 B AO2 B  60o
AO
a k
y a (x ) (3.52)
 AO1O2  5 , O2O1  30o
AO h

O2  105o. Using sine value for $O1 AO


Therefore, O1 AO O2 , Substituting y  0 in Eq. (3.52) we get R  (ah/a – k, 0).
we get Similarly, the line QB meets the x-axis at S(ah/a k, 0).
Let M  (x, 0) be the midpoint of RS. Then
O1O2 OA O A
 1  2 [ah/(a k )] [ah/(a k )]
sin 105o sin 30o sin 45o x
2
si 30o
( 3 1) sin a2 h a2 h
O1 A   2  2 ( h2 k 2  a 2 )
sin 105o a k2 h
( 3 1)() 1// ) ( )2 2 a2
   2  r1 2 
cos 15o 2(( ) h
168 Chapter 3 Circle

Therefore, M T and hence RT  ST . is an altitude of $AB


$ C which is also the radical axis of
the two circles with AB and AC as diameters. Similarly,
y
the other altitudes BE and CF are the radical axes of the
B (0, a) pairs described on AB, BC and AC, BC. Thus, the alti-
P tudes of $AB
$ C are radical axes. Hence, the orthocentre
is their radical centre.
A

R T S O x
B C
D
Q
B  (0, a) FIGURE 3.49

FIGURE 3.48 31. Show that two direct common tangents and the
transverse common tangent to the two circles S y x2
29. Suppose two circles pass through the points (0, a) and y2 − 6x  0 and Sa y x2 y2 2x  0 form an equi-
(0, −a) and touch the line y  mx c. If the two circles lateral triangle.
cut orthogonally, then show that c2  a2(2 m2).
Solution: A(3, 0) and 3 are the centre and radius of S 
Solution: The centre of a circle passing through the 0 (see Fig. 3.50). B(−1, 0) and 1 are the centre and radius
points (0, −a) and (0, a) must lie on x-axis. Let the equa- of Sa  0. Now AB  4  3 1  S  0 and Sa touch each
tion of any such circle be other externally at (0, 0) and x  0 (i.e., y-axis) is the com-
mon tangent at the origin. Let T be the external centre of
S x 2 y2 2 ggxx a 2  0 similitude. Therefore, T divides the line joining the cen-
tres A and B externally in the ratio 3:1. That is,
S  0 touches the line y  mx c implies that
AT : TB 3 : 1
m  g 0 c 2 2
 g a
¥ 3(( ) ( )3 ´
1 m2 or T , (( 3, )
2 2 2 2 2 2
§ 3 1 ¶
 m g 2cmg
cmg c  ( g a )(1 m )
Let y  m(x 3) be a tangent from T(−3, 0) to Sa  0.
 g 2 2cmg a 2 (11 2
) 2
0 Therefore
Let g1 and g2 be two the roots of this quadratic equation. m( ) 0
Hence 1
1 m2
g1 g2  2cm
m2  1 m2
 4m
g1 g 2  a 2 ( a m 2 ) c 2
1
Now, the circles x2 y2 2g1x − a2 and x2 y2 2g2x − mp
3
a2  0 cut orthogonally implies that
Hence, the tangents from T(−3, 0) are x − 3 y 3  0
g1 2 2 f1 f2  c1 c2 and x 3 y 3  0. Therefore, the vertices of the trian-
 2 g1 g2  aa 2 a2 gle are P(0, 3 ), T(−3, 0) and Q(0, − 3 ), which form an
equilateral triangle.
 g1 g 2  a 2
y
 a 2 (1 m2 ) c 2 a2
P
 a2 ( 2
)  c2

30. Prove that the radical centre of the circles described B 3 A


on the sides of a triangle as diameters is the orthocen- T (−1, 0) O (3, 0) x
tre of the triangle.
Q
Solution: The circles described on AB and AC as diam-
eters (see Fig. 3.49) will intersect at a point D on the side
BC and because ADB ADC  90o. We have that AD FIGURE 3.50
3.5 Common Tangents of Two Circles 169

32. Tangents PQ and PR are drawn to the circle S y x2 The point of intersection of the lines (−3, 3/2), which is
y2 − a2  0 from the point P(x1, y1) to touch the the centre of the required circle. Also the centre and
circle at Q and R. Determine the equation of the cir- radius of the circle x(x − 4) y(y − 3)  1 are (2, 3/2) and
cumcircle of $PQR. 5/2, respectively.
If the required circle is to just contain the circle with centre
Solution: QR being the chord of contact of P(x1, y1) with (2, 3/2) and radius 5/2, then this circle should touch the
respect to the circle S  0, its equation is S1 y xx1 yy1 − a2  0 required circle internally. If r is the radius of the required
Any circle passing through Q and R is of the form S KL circle the distance between their centres (−3, 3/2) and (2,
 0 where S y x2 y2 − a2  0 and L y xx1 yy1 − a2  0. 3/2) must be equal to r − 5/2 (since r  5/2). Therefore
Therefore,
5
r  (2 3)2 0  5
S KL y x2 y2 − a2 K(xx1 yy1 − a2)  0 2
which is the circumcircle of $PQR, with the condition 5 15
 r  5 
that it passes through P(x1, y1). Therefore 2 2
Hence, the equation of the required circle is
x12 y12 a 2 L ( x12 y12 a2 )  0
2 2
L  1 ¥ 3´ ¥ 15 ´
( )2 y 
§ 2¶ § 2¶
Therefore, the equation of the circumcircle of $PQR is
x2 y2 − xx1 − yy1  0. 225 9
 x 2 y2 6 x 3 y  9
4 4
33. If 4l2 − 5m2 6l 1  0, then show that the line lx
2 2 180
my 1  0 touches a fixed circle. Find the equation of  x y 6 x 3y   45
the circle. 4
 x 2 y2 6 x 3 y 45  0
Solution: Suppose the line lx my 1  0 touches cir-
cle (x − h)2 (y − k)2  a2. So 35. From each point on the line 2x y  4, tangents are
drawn to the circle x2 y2  1. Find the point through
lh mk 1
a which those chords of contact pass. (IIT-JEE 1997)
l2 m2
 (hl km 1)2  a 2 (l 2 m2 ) Solution: Let P(h, k ) x y .
Therefore,
Therefore
2h k 4 (3.55)
l 2 (h 2 a 2 ) m 2 (k 2 a 2 ) 2hklm
hklm
l hl
hl km 1  0 The chord of contact of P(h, k) with respect to the circle
(3.53) x2 y2  1 is hx ky − 1  0. That is,

However, by hypothesis, hx (4 2 h) y 1  0
4l2 − 5m2 6l 1  0 (3.54) From Eq. (3.55), we have k  4 − 2h. Therefore

From Eqs. (3.53) and (3.54), we have h( x y) 4 y 1  0

h2 a2 k2 a 2 2 hk 2 h k 1 Hence these lines are concurrent at the point of intersec-


     tion of the lines x − 2y  0 and 4y − 1  0 which is (1/2, 1/4).
4 5 0 6 0 1
Therefore, k , h  3 and a  5 . The circle equation is 36. Consider a curve ax2 2hxy by2  1 and a point P
obtained as (x − 3)2 y2  5. not on it. A line drawn from the point P intersects
the curve at points Q and R. If the product PQ·PR is
34. Find the equation of the circle having the pair of independent of the slope of the line, then show that
lines x2 2xy 3x 6y  0 as its normals and having the curve is a circle. (IIT-JEE 1997)
the size just sufficient to contain the circle x(x − 4)
y(y − 3)  0. Solution: Let P be (x1, y1) and line through P(x1, y1)
(Fig. 3.51) be
Solution: We have
x x1 y y1
x2 2xy 3x 6y y x(x 2y) 3(x 2y) (x 2y)(x 3)   r (say)
cos Q i Q
sin
170 Chapter 3 Circle

Substitute x  x1 r cos Q and y  y1 r sin Q in the equa- ax12 2 hx


h 1 y1 by12 1
tion ax2 2hxy by2  0, so that we have PQ – PR  r1r2  2
a Q h s Q cos Q b sin 2 Q
( x1 r c Q )2 ))( 1 r sin Q )
h( x1 r coss )(y Hence r1r2 is independent of the slope tan Q if a  b and
2
b( y1 r sin Q ) 1
b 0 h  0. In such a case, the given curve becomes ax2 ay2 
1, which is a circle,
That is,
r 2 (a cos2 Q 2 h Q Q b in 2 )
bsin
P
2[(ax1 hy1 )cos Q (hx
2[( (hx
h 1 by i Q ]r ax12
by1 ) sin
h 1 y1 by12 1  0
2 hx (3.56) Q
If PQ  r1 and PR  r2, then r1 and r2 are the roots of Eq.
(3.56). Hence
R

FIGURE 3.51

WORKED-OUT PROBLEMS

Single Correct Choice Type Questions


5 5 5
1. The angle between the tangents drawn from the ori- (A) (B) (C) (D) 5 2
2 2 2
gin to the circle x2 + y2 − 14x + 2y + 25  0 is
Solution: The radius (r) is equal to the distance of the
P P 5P P
(A) (B) (C) (D) centre (2, 1) from the line 3x + y  0, which is given by
3 4 12 2
Solution: Centre and radius of the circle are (7, −1) and 3(2) 1 5

5, respectively. Now y  mx touches the circle 32 1 2

m( ) 1 Answer: (B)
š 5
1 m2 3. The angle at which the circle x2 + y2  16 can be seen
š( )2  25( m2 ) from the point (8, 0) is

2
P P P P
š 24 14 m 24  0 (A) (B) (C) (D)
2 4 3 6
2
š 12 7 m 12  0 Solution: The required angle is the angle between
the tangents drawn from the point (8, 0) to the circle
1 m2 16m
š 12 166 m 9 12 0 x2 + y2  16. Let y  m(x − 8) be a line through (8, 0). This
š 4 (3m
3 4) 3(3 4)  0 line touches the circle. So
m( ) 0
3 4 š 4
šm , (3.57) 1 m2
4 3
2
š 64  16( m2 )
Since the product of the slopes is −1, the tangents from
2
(0, 0) are at right angles. š3 1
1
Note: From Eq. (3.57), the product of the roots is −1 š p
3
which shows that the product of the slopes is −1.
Answer: (D) Therefore, the tangents are inclined at angles of 30° and 150°
with the positive direction of the x-axis. Hence, the acute
2. The line 3x + y  0 is a tangent to the circle which has angle between tangents to the circle can be seen is 60°.
its centre at the point (2, −1). Then, the radius is Answer: (C)
Worked-Out Problems 171

4. The radius of the circle passing through the points 5 ¥ 7´


( 1, 1), (0, 6) and (5, 5) is (A) 2 Tan 1 ¥¦ ´µ (B) Tan 1 ¦ µ
§ 12 ¶ § 12 ¶
(A) 2 3 (B) 2 (C) 2 2 (D) 13
P ¥ 7´
Solution: Let S y x2 y2 2gx 2fy c  0 be the re- (C) (D) 2 Tan 1 ¦ µ
3 § 12 ¶
quired circle. Therefore
Solution: Let y  m(x 13) be a line through (13, 0).
2 g 2 f c  2 (3.58) This line touches the circle
12 f c  36 (3.59)
m(0 13) 0
10 g 10 f c  50 (3.60) š 5
1 m2
Solving Eqs. (3.58)–(3.60), we have g  2, f  3 and c  š 169 m2  25(1 m2 )
0. Therefore, the radius of the circle is
š 144 m2  25
2 2
( 2) ( 3) c  13. 5
šmp
Answer: (D) 12

5. The radius of the circle which passes through the point


Therefore, the angle between the tangents is
(2, 3) and touches the line 2x 3y 13  0 at the point ¥ 5´
(2, 3) is 2 Tan 1 ¦ µ
§ 12 ¶
(A) 2 2 (B) 8 (C) 13 (D) 2 3 Answer: (A)
Solution: Let A  (2, 3) and B  (2, 3). Let M be the 7. The tangent to the circle x2 y2  5 at the point (1, 2)
midpoint of AB  (2, 0) and C be the centre of the circle also touches the circle x2 y2 8x 6y 20  0 at the
(see Fig. 3.54). Therefore, C is the intersection of the lines point
CB and CM. Equation of the line CM is
(A) (3, 1) (B) (2, 2)
y0 (3.61)
(C) (2, 4) (D) (4, 2)
Equation of the line CB is
Solution: Tangent to the circle x2 y2  5 at the point
3 (1, 2) is
y 3  ( x 2)
2
x(1) y( 2) 5  0
That is,
 x 2y 5  0 (3.63)
3x 2y  0 (3.62)
Equations (3.61) and (3.62) imply that the centre of the Point of contact of the tangent given in Eq. (3.63) with
circle is (0, 0). Therefore, the radius of the circle is the circle x2 y2 8x 6y 20  0 is the foot of the per-
pendicular drawn from the centre (4, 3) onto the tan-
CB  (2 0)2 ( 3 0)2  13 gent given in Eq. (3.63). Suppose (x1, y1) is the foot of the
perpendicular from (4, 3) onto the line x 2y 5  0
A (2, 3)
(see Fig. 3.53). Therefore, by Theorem 2.13(1), we have

x1  4
; 4 2( 3) 5=  4 1  3
C M (2, 0) 12 2 2

and y1  3 [( 2)(1)]  1

B (2, −3) Therefore, the point of contact is (3, 1).


3 =0
y −1 (4, −3)
−3
2x

FIGURE 3.52
Answer: (C) x − 2y − 5 = 0 (x1, y1)

FIGURE 3.53
6. The angle between the tangents drawn from the point
(13, 0) to the circle x2 y2  25 is Answer: (A)
172 Chapter 3 Circle

8. The number of common tangents to the circles 10. Let C1: x2 y2  5 and C2: x2 y2  9. The tangent at (1,
x2 y2 2x 6y 9  0 and x2 y2 6x 2y 1  0 2) to C1 meets the circle C2 at A and B. The tangents
is drawn at A and B to C2 meets in T. Then the coordi-
nates of T are
(A) 4 (B) 3 (C) 2 (D) 1
Solution: A  (1, 3) and r1  1, respectively, are the centre ¥ 9 18 ´
(A) (4, 5) (B) ¦ , µ
and the radius of the first circle. B  ( 3, 1) and r2  3, re- § 15 15 ¶
spectively, are the centre and radius of the second circle. So ¥ 9 18 ´
(C) (4, 5) (D) ¦ , µ
2 2 §5 5 ¶
AB  (1 3) (3 1)  20  2 5 > r1 r2
Solution: Equation of the tangent at (1, 2) to C1 is
Therefore, the two circles do not have a common point.
Hence, the number of common tangents is 4. x 2y 5  0 (3.64)
Answer: (A) Suppose T(x1, y1) is the point where the tangents to C2 at
points A and B meet (see Fig. 3.55). Hence, the equation
9. Any tangent to the circle x2 y2  a2 meets the axes of the chord AB is
at points A and B, respectively. The rectangle OACB
xx1 yy1 9  0 (3.65)
is completed. The locus of the vertex C of the rect-
angle is Equations (3.64) and (3.65) represent the same line AB.
Therefore
1
(A) x 2 y2  (B) x 2 y 2  a 2 x1 y1 9
a2  
1 2 5
a2
(C) x 2 y2  (D) x2 y2  2a2 9 18
2  x1  , y1 
5 5
Solution: Tangent at (x1, y1) to the circle (see Fig. 3.54) So
is xx1 yy1  a2. Therefore
¥ 9 18 ´
¥ a2 ´ T ¦ , µ
A  ¦ , 0µ §5 5 ¶
§ x1 ¶ C2
¥ a2 ´ 3 A
B  ¦ 0, µ C1
§ y1 ¶
Hence 5

¥ a2 a2 ´ O (0, 0) T
C¦ , µ
§ x1 y1 ¶
If C  (x, y), then B

1 1 x12 y12 x12 y12 a 2


1 FIGURE 3.55
2
2
 4
4
 4
 4

x y a a a a a2 Answer: (D)
Therefore, the locus of C is x 2 y 2  a 2 .
11. Tangents are drawn from any point on the circle
y
x2 y2  R2 to the circle x2 y2  r2. If the line joining
B C the points of intersection of these tangents with the
first circle also touches the second, then R:r is
(A) 2:1 (B) 1:2 (C) 3:1 (D) 2 :1
Solution: Let P(x1, y1) be a point on x2 y2  R2.
Suppose the tangents from point P to the circle x2 y2
O A x
 r2 meet the circle with radius R in A and B such that
AB touches the circle with centre r. Thus, for $PAB,
x2 y2  R2 is the circumcircle and x2 y2  r2 is the in-
circle and hence the circumcentre and incentre are the
same. Therefore, the triangle is equilateral so that R  2r.
FIGURE 3.54
Answer: (B) Answer: (A)
Worked-Out Problems 173

12. If the circles (x 1)2 (y 3)2  r2 and x2 y2 8x Solution: Let S y x2 y2 2gx 2fy c  0 be the
2y 8  0 intersect at two distinct points, then required circle which passes through origin. This implies
that
(A) 2 < r < 8 (B) r < 2
c0 (3.66)
(C) r  2 (D) r > 2
It passes through (2, 0) which implies that
Solution: For the first circle A  (1, 3) and radius  r,
and for second circle B  (4, 1) and radius r  3. Since 4 g c  4 (3.67)
the two circles intersect in two distinct points, we have
It touches the line y  x which implies that
r a r  AB  r r a
g f
2 2  g2 f 2 c
 3 r  3 4  3 r 2
Therefore, 2  r  8.  ( g f )2  2( g 2 f 2 c)
Answer: (A)  2 gf  g 2 f 2 c

13. If the line x cos Q y sin Q  2 is the equation of a  ( g f )2  0 [from Eq. (3.66)]
transverse common tangent to the circles x2 y2  4  g  f
and x2 y2 6 3 x 6y 20  0, then the value of
From Eq. (3.67), we get g  1 and f  1. Therefore,
Q is
equation of the circle is S y x2 y2 2x 2y  0.
P P 2P 5P Answer: (A)
(A) (B) (C) (D)
3 6 3 6
15. C1 is a circle with centre at A and radius 2. C2 is a
Solution: A  (0, 0) and r1  2, respectively, are the cen-
circle with centre at B and radius 3. The distance AB
tre and radius of the first circle. B  (3 3 , 3) and r2  4,
is 7. If P is the point of intersection of a transverse
respectively, are the centre and the radius of the second
common tangent with line AB, then the distance AP
circle. So
is
AB  27 9  6  r1 r2
14 7 9 8
(A) (B) (C) (D)
Therefore, the two circles touches each other externally, 5 3 4 3
and the transverse common tangent is
Solution: It is known that point P is the internal centre
( x 2 y2 6 3 x 6 y 20) ( x 2 y2 4)  0 of similitude and AP:PB  2:3 (see Section 3.5). There-
 6 3 x 6 y 24  0 fore,

 3x y 4  0 3 AP  2 PB  2( AB AP )  2(7 AP )
 5 AP  14
However, x cos Q y sin Q 2  0 is the transverse common
tangent. Therefore 14
 AP 
5
cos Q sin Q 2 1
   Answer: (A)
3 1 4 2
Hence 16. Two circles with centres at A(2, 3) and B(5, 6) and
having equal radii are intersecting orthogonally.
3 1 Then the radius of the circles is
cos Q  and sin Q 
2 2
(A) 3 2 (B) 2 2 (C) 3 (D) 2
Therefore
Soution: Two circles intersect orthogonally. There-
P fore, the square of the distance between the two circles
Q is equal to the sum of the squares of the radii (see Quick
6
Answer: (B) Look 11). Therefore, if r is the radius of the two equal
circles, then
14. The equation of the circle passing through the point ( AB)2  2r 2
(2, 0) and touching the line y  x at the origin is
2
(A) x2 y2 2x 2y  0 (B) x2 y2 2x 2y  0   (5 2)2 (6 3)2  2r 2
(C) x2 y2 2x 2y  0 (D) x2 y2 2x 2y  0
174 Chapter 3 Circle

From Eq. (3.68), we have


 32 32  2 r 2
r3 x 2 y2  9
Answer: (C)  h2 k 2  1

Therefore, the locus of the centroid G is x 2 y2  1.


17. A square ABCD has area 1 sq. unit. A circle touches
the sides AB and AD and passes through the vertex Answer: (D)
C. Then, the radius of the circle is
19. A circle is given by x2 (y 1)2  1. Another circle
1 2 C touches it externally and also the x-axis. Then the
(A) (B)
2 2 locus of its centre is

(C) 2 1 (D) 2
2 (A) {(x, y): x2  4y} ‡ {(x, y): y b 0}
HHHHE (B) {(x, y): x2 (y 1)2  4} ‡ {(x, y): y b 0}
Solution: HLet HHHE us consider A as origin, AB as positive
x-axis and AD as positive y-axis (see Fig. 3.56). Sup- (C) {(x, y): x2  y} ‡ {(0, y): y b 0}
pose S is the centre of the circle and r its radius. We have (D) {(x, y): x2  4y} ‡ {(0, y): y b 0}
S  (r, r ). Therefore (IIT-JEE 2005)
r  SC  (r 1)2 (r 1)2 Solution: Let P(h, k) be the centre of C so that k is
the radius of C because it touches x-axis (see Fig. 3.57).
 r 2  2(r 1)2 Also, since C touches the circle x2 (y 1)2  1 exter-
 r 2 4r 2  0 nally, we have

4 p 16 8 (h 0)2 (k 1)2  1 k
r  2 2 (& r  1)
2  h2 (k 1)2  1 2 k k 2
D C (1, 1)  h 2 2k  2 k
r  h 2  4k
r
1 S where k > 0 or h2  0 when k b0. Therefore, the locus of
r the centre P(h, k) is {(x, y): x2  4y} ‡ {(0, y): y b 0}.
y
A 1 B

FIGURE 3.56
Answer: (D)
(0, 1) P (h, k )
18. In $ ABC, B  (3, 0) and C  ( 3, 0) and A is a vari-
able vertex such that BAC  90°. Then the locus of
the centroid of $ ABC is the circle
O x
4
(A) x2 y2  2 (B) x2 y2 
9
1 FIGURE 3.57
(C) x2 y2  (D) x2 y2  1
9 Answer: (D)
Solution: It is clear that A moves on the circle with BC 20. The equation of the circle touching the line 2x 3y
as diameter and its equation is 1  0 at the point (1, 1) and cutting orthogonally the
( x 3)( x 3) y  0 2 circle x2 y2 2x 2y 3  0 is

x 2 y2  9 (3.68) 5 1
(A) x2 y2 5x y 0
2 2
Let G(h, k) be the centroid of $ ABC. Therefore (B) x2 y2 10x 5y 3  0
x 3 3 y 0 0 (C) x2 y2 20x 10y 8  0
h , k
3 3 (D) x2 y2 5x 5y 2  0
 3h  x, 3k  y
Worked-Out Problems 175

Solution: The required circle is of the form This implies CA  3

( x 1)2 ( y 1)2 L (2 x 3 y 1)  0
That is, A M B
2 (1, 3) 2
x2 y2 2( L 1)x (3 L 2)y 2 L  0 (3.69)
x2 y2
This circle cuts the circle 2x 2y 3  0 orthogo-
nally. From this and Theorem 3.17 we get C (2, 1)

¨ 3L 2 ·
2( L 1)(1) 2 © ( 1)  2 L 3
ª 2 ¸¹
Therefore
FIGURE 3.58
2 L 2 3L 2  2 L 3
Answer: (C)
 2 L  3
3 23. If the tangent at the point P on the circle x2 y2
L
2 6x 6y 2  0 meets the straight line 5x 2y 6  0 at
Substituting the value of K in Eq. (3.69), we get the a point Q on the y-axis, then the length of PQ is
required circle as (A) 4 (B) 2 5 (C) 5 (D) 3 5
5 1 Solution: The line 5x 2y 6  0 meets y-axis in (0, 3)
x2 y2 5x y 0
2 2 so that Q  (0, 3). Hence, QP is the length of the tangent
Answer: (A) drawn from Q. It is given by

S11  0 9 6(0) 6(3) 2  25  5


21. The centre of the circle inscribed in a square formed
by the lines x2 8x 12  0 and y2 14y 45  0 is Therefore, PQ  5.
Answer: (C)
(A) (4, 7) (B) (7, 4) (C) (9, 4) (D) (4, 9)
(IIT-JEE 2003) 24. Let AB be a chord of the circle x2 y2  r2 subtend-
Solution: The lines are x  2, x  6 and y  5, y  9. ing a right angle at the centre. Then, the locus of the
Therefore, the vertices of the square are (2, 5), (2, 9), (6, centroid of $ PAB as P moves on the circle is
9) and (6, 5). (A) a parabola (B) a circle
Centre of the circle  Centre of the square (C) an ellipse (D) a pair of lines
¥ 2 6 9 5´ Solution: Let A  (r cos A , r sin A ) and
¦ , µ
§ 2 2 ¶
¥ ¥P ´ ¥ P ´´
 (4, 7) B  ¦ r cos ¦ A µ , r sin ¦ A µ µ
§ §2 ¶ § 2¶¶
Answer: (A)
 ( r sin A , r cos A )
22. If one of the diameters of the circle x2 y2 2x Let P(h, k) be a point on the circle (see Fig. 3.59). There-
6y 6  0 is a chord to the circle with centre (2, 1), fore
then the radius of the circle is
h2 k 2  r 2 (3.70)
(A) 3 (B) 2 (C) 3 (D) 2
Let G(x, y) be the centroid of $ PAB. Therefore
(IIT-JEE 2004)
r cos A r sin A h
Solution: Let AB be a diameter of the circle x2 y2 x
3
2x 6y 6  0 (see Fig. 3.58). Centre of this circle is M 
r sin A r cos A k
(1, 3) and radius 12 32 6  2. C(1, 2) is the centre of and y
the required circle. Therefore 3
Therefore
(CA)2  ( AM )2 (CM )2
r 2  h2 k 2
 4 (2 1)2 (3 1)2
 4 1 4  9  [3 x r(cos A sin A )]2 [3 y r(sin A cos A )]2
176 Chapter 3 Circle

That is, the equation of the circumcircle is (x 1)(x 3) (y 8)


2 2 2 (y 2)  0. That is,
¨ r · ¨ r · ¥ r´
©ª x 3 cos A sin A ¸¹ ©ª y 3 sin A cos A ¸¹  ¦§ 3 µ¶ x 2 y2 4 x 10 y 19  0
Answer: (B)
which is a circle.
26. A line is touching two circles at T1 and T2, respec-
y
tively, such that the length T1T2 is 36. The minimum
B distance between the circles is 14. If the radius of the
larger circle is four times the radius of the smaller
A
circle, then the radius of the smaller circle is
x (A) 5 (B) 4 (C) 3 (D) 10
O
45° Solution: Points A and B are the centres of the two
circles. PQ  14 and T1T2  36. Draw T1M parallel to
P
AB to BT2 (see Fig. 3.60). Let r and 4r be the radii.
From $ T1T2M, we have (T1M)2  (T1T2)2 (T2M)2.
FIGURE 3.59
Therefore,
Answer: (B)
(14 5r )2  36 2 9r 2
25. Tangents drawn from the point P(1, 8) to the circle  16r 2 140r 50 s 22  0
x2 y2
6x 4y 11  0 to touch it at the points
A and B. Then, the equation of the circumcircle of  4r 2 35r 275  0
$ PAB is  4r 2 20r 55r 275  0
(A) x2 y2 4x 6y 19  0  4r(r 5) 55(r 5)  0
(B) x2 y2 4x 10y 19  0 r5
(C) x2 y2 2x 6y 29  0
(D) x2 y2 6x 4y 19  0
Solution: Equation of AB is
A r 14 4r B
S1 y x(1) y(8) 3(x 1) 2(y 8) 11  0 P Q
r r
because AB is the chord of contact. That is,
M
S1 y 2 x 6 y 30  0 T1 90° 3r
36
x 3 y 15  0 T2
Therefore, any circle passing through A and B is of the FIGURE 3.60
form
Answer: (A)
x 2 y2 6 x 4 y 11 L ( x 3 y 15)  0
27. Let PQ and RS be tangents at the extremities of the
This represents circumcircle of $ PAB š it passes
diameter PR of a circle of radius r. If PS and RQ in-
through P(1, 8). Therefore
tersect at point X on the circumference of the circle,
1 64 6 32 11 L (1 24 15)  0 then 2r is equal to
8 L  16
(A) PQ – RS (B) (PQ RS)/2
L2
(C) [2(PQ)(RS)]/PR RS (D) ( PQ2 RS 2 )/2
Therefore, the circumcircle of $ PAB is
Solution: In $PQR, QPR  90o (see Fig. 3.61). The
x2 y2 6x 4y 11 2(x 3y 15)  0 angle in the semicircle is given by PXR  90o. There-
That is, fore, $ PQX and $ PXR are similar. Hence

x 2 y2 4 x 10 y 19  0 PQ PX
 (3.71)
PR RX
Aliter: Suppose C(3, 2) is the centre of the circle. Since
Similarly, $ PRX and $ RXS are similar. Therefore
PAC  90o  PBC , the circumcircle of the quadrilat-
eral PACB is also the circumcircle of $PAB. Therefore, PR PX
 (3.72)
PC is a diameter of the circumcircle of $ PAB. Hence, RS RX
Worked-Out Problems 177

From Eqs. (3.71) and (3.72), we have Solution: Let S2 be the external centre of similitude
(see Fig. 3.63). Therefore, S2 divides the line joining O(0, 0)
PQ PR
 and A(6, 0) externally in the ratio 2:1. Hence S2  (12, 0).
PR RS Therefore, equation of the chord of contact of S2(12, 0)
Therefore with respect to x 2 y2  16 is

( PR)2  PQ – RS L y 3x 4  0 (3.73)
2r  PR  PQ – RS Equation of the chord of contact of S2(12, 0) with respect
to x 2 y2 12 x 32  0 is
R S
L a y 3 x 20  0 (3.74)
Now, equation of the circle passing through the points of
90° x intersection of x2 y2 16  0 and L  0 is
r x 2 y2 16 L (3 x 4)  0 (3.75)
Equation of the circle passing through the intersection of
P Q
x2 y2 12x 32  0 and La  0 is
FIGURE 3.61
x 2 y2 12 x 32 M(3 x 20)  0 (3.76)
Answer: (A)
Equations (3.75) and (3.76) represent the same circle
28. $PQR is inscribed in the x2 y2
 25. If Q and R š 3K  3 M 12 and 4 L 16  20 M 32
have coordinates (3, 4) and ( 4, 3), respectively, then
PQR is equal to š L M  4 and L 5 M  12
Therefore, L  2 and M  2, so that the equation of the
P P P P required circle is x2 y2 6x 8  0.
(A) (B) (C) (D)
2 3 4 6
Solution: From Fig. 3.62, we have the following: L
L
4 3 O (0, 0) S2 (12, 0)
Slope of OQ × Slope of OR  s  1 A(6, 0)
3 4
where O is the origin. Therefore QOR  90o. Hence

P FIGURE 3.63
QPR 
4 Answer: (B)
y
30. In $ABC, if sin2 A sin2 B sin2 C  1, then the
R (− 4, 3)
angle of intersection of the circumcircle of $ABC
and the circumcircle of its pedal triangle is
Q (3, 4)
90° 2P P P P
x
(A) (B) (C) (D)
O 5 3 2 3 4
p
4 Solution: For pedal triangle and its properties, see
Chapter 4, Vol. 2. Let O and H be the circumcentre and
P
orthocentre, respectively, of $ABC. Take O as origin
HHHE E HHHE E HHHHE HE
of reference and OA  a, OB  b and OC  C . Hence
FIGURE 3.62 E E E
a  b  c  R (which is the circumradius). Accord-
Answer: (C) ing to Example 5.2, part (2), Chapter 5, Vol. 2, we have
HHHHE E E E
OH  a b c. Therefore
29. The equation of the circle passing through the point
of contact of the direct common tangents of the cir- HHHHE 2
cles x2 y2  16 and x2 y2 12x 32  0 is OH  3R 2 2 R 2 (cos 2 A cos 2 B cos 2C )

(A) x2 y2 6x 8  0 (B) x2 y2 6x 8  0  3R 2 2 R 2 [3 2(sin 2 A sin 2 B sin 2 C )]


(C) x2 y2 4x 6  0 (D) x2 y2 4x 6  0  3R 2 2 R 2 (3 2)  5R 2 (3.77)
178 Chapter 3 Circle

If N is the centre of the circumcircle of the pedal triangle, (C) greater than 2r (D) less than 2r
then N is the midpoint of OH and the circumradius of
Solution: The two given lines are parallel (see Fig.
the pedal triangle is R/2. Therefore
3.65).Therefore, the circle completely lies within the lines.
E E E 2 2 This implies that
2 ¥ a b c´ 5R 2 2 ¥ R´
(ON )  ¦   R ¦ µ
§ 2 µ¶ 4 § 2¶ a bp 2
r
That is, square of the distance between the centres of the 12 2 2
circles is equal to the sum of the squares of their radii.
 a bp 2  r 2
Therefore, the circumcircle of $ABC and the circum-
circle of its pedal triangle intersect orthogonally (see
Quick Look 11).
Answer: (B)
(a, b)
31. Two equal circles of radius 2 1 touch each other
externally. Another circle of radius 1 touches both
the circles externally. Then, the perimeter of the re-
gion bounded between the three circles is
P P
(A) (2 2 ) (B) (2 2 ) FIGURE 3.65
2 4
Answer: (A)
P P
(C) (2 2 ) (D) (2 2 1)
4 4 33. If A is a point on the circle x2 y2  1 and B is a point
Solution: Let A and B be the centres of the circles on the circle x2 y2 10x 21  0, then the minimum
of equal radius 2 1 and C be the circle with unit ra- value of AB is
dius (see Fig. 3.64). Therefore, AC  BC  2 2 and (A) 1 (B) 2 (C) 4 (D) 8
AB  2( 2 1). Now
Solution: Suppose the centres of the two circles are
2 2 2
( AC ) ( BC )  (2 2 ) (2 2 ) 2 O(0, 0) and C(5, 0) (Fig. 3.66). The radii of the circles are
1 and 2. Suppose the line OC meets the circles at points P
 4( 2 1)2  ( AB)2 and Q, respectively, so that P  (1, 0) and Q  (3, 0). Mini-
Therefore, C  90o. The perimeter of the shaded region mum value of AB occurs when A  P and B  Q. Therefore,
is given by the minimum value of AB  5 (1 2)  2.

P P P
2( 2 1)  (2 2 )
4 2 2
1 2
O (0, 0) P Q C (5, 0)

A B
45° 45°
2+1 2+1
FIGURE 3.66
Answer: (B)
1 90° 1

C 34. Three circles with radii a, b and c touch each other


externally. The tangents drawn at their points of con-
tact meet at a point P whose distance from the points
FIGURE 3.64 of contact is 4. Then (abc)/(a b c) is equal to
Answer: (A)
(A) 8 (B) 16 (C) 24 (D) 32
32. If a circle having centre at (a, b) and radius r complete- Solution: P is equidistant from the sides of $ABC and
ly lies within the two lines x y 2  0 and x y – 2  0, that distance is 4 (Fig. 3.67). Therefore, 4 is the inradius of
then the minimum value of { a b 2 , a b 2 } is $ABC . The semiperimeter s of $ABC is given by
(A) greater than r 2 (B) less than r 2 (a b) (b c) (c a)
 a b c
2
Worked-Out Problems 179

Now $ is given by For this circle, the chord y  2x is a diameter and if the
center of this circle lies on y  2x, then
(a b c)(a b c b c)(a b c c a)(a b c a b)
¥ L ´
 abc(a b c) 2(5 L ) ¦ 0
§ 2 µ¶
Therefore L
 10 2 L 0
2
$ abc abc
4r  or  16 L4
s a b c a b c
Therefore, the required circle is x2 y2 2x 4y  0.
Answer: (A)

A
a b
B
37. The centre of the circle passing through the points
(0, 0) and (1, 0) and touching the circle x2 y2  9 in
a
P
b the first quardrant is
¥ 3 1´ ¥ 1 3´
c c (A) ¦ , µ (B) ¦ , µ
§ 2 2¶ § 2 2¶
C ¥ 1 1´ ¥1 ´
(C) ¦ , µ (D) ¦ , 2 µ
§ 2 2¶ §2 ¶

Solution: Let S y x2 y2 2gx 2fy c  0 be the


FIGURE 3.67 required circle. If it passes through (0, 0), we have c 
Answer: (B) 0. If it passes through (1, 0), we have 2g  1. Touching
internally the circle x2 y2  9 implies that
35. Three circles of equal radius r touch each other ex-
ternally. Then the radius of the circle having internal g2 f 2  3 g2 f 2 c
contact with all the three circles is
 4( g 2 f 2 )  9
(A) ( 2 3 )r (B) ( 2 3 )r
¥1 ´
 4¦ f 2µ  9
¥ 2 3´ §4 ¶
(C) ( 2 2 )r (D) ¦ µr
§ 3 ¶  4f 2  8
Solution: Let A, B and C be the centres of three circles.  f p 2
Therefore, $ABC is an equilateral triangle with side But the centre lies in the first quadrant. Therefore
length 2r. If R is the circumradius of $ABC, then
1
2r 2r g ,f  2
R  2
2 sin 60o 3
so that the centre is
Therefore, the radius of the required circle is obtained as ¥1 ´
¦§ , 2 µ¶
2r ¥ 2 3´ 2
R r  r  ¦ µr Answer: (D)
3 § 3 ¶
Answer: (D) 38. From the point A(0, 3) on the circle x2 4x
(y 3)2  0 a chord AB is drawn and extended to a
36. If y  2x is a chord of the circle S y x2 y2 10x  0, point M such that AM  2AB. Then, the equation of
then the equation of the circle described on this chord the locus of M is
as diameter is (A) x2 y2 8x 4y 3  0
(A) x2 y2
2x 4y  0 (B) 14x 2y  0 x2 y2 (B) x2 y2 8x 6y 27  0
(C) x y 6x 2y  0 (D) x y2 18x 14y  0
2 2 2
(C) x2 y2 8x 6y 9  0
Solution: The required circle of the form S K (2x (D) x2 y2 8x 9y 18  0
y)  0 is Solution: The given circle is
x2 y2 10x K (2x y)  0 (x 2)2 (y 3)2  4
180 Chapter 3 Circle

which touches the y-axis at A(0, 3). Let M  (h, k) so that 2


2 2 ¥ x cos A y sin A ´
2
B  [h/2, (k 3)/2] is the midpoint of AM. Since B lies on x y a ¦ µ¶  0 (see Theorem 2.33)
the circle § p
(h/2 2)2 (k 3/2 3)2  4 Since AOB  90o, in the above equation, the sum of the
we have coefficient of x2 and the coefficient of y2 is 0. That is,

(h 4)2 (k 3)2  16 ¥ a 2 cos2 A ´ ¥ a 2 sin 2 A ´


¦1 2 µ ¦1 µ 0
§ p ¶ § p2 ¶
Therefore, locus of M is x2 y2 8x 6y 9  0.
Answer: (C) 2 p2  a 2
y
39. If the chord of contact of tangents from a point P to
a given circle passes through Q, then the circle de- A
scribed on PQ as diameter
B
90°
(A) touches the given circle externally.
x
(B) touches the given circle internally. O

(C) cuts the given circle orthogonally.


(D) does not have common points with the given
circle.
FIGURE 3.68
Solution: Let S y x2 y2 a2  0 and P(x1, y1) be a
point whose chord of contact is S1 y xx1 yy1 a2  0 Answer: (A)
which passes through Q(x2, y2). Therefore
41. If the circle x2 y2 2gx 2fy c  0 is touched by
x2 x1 y2 y1  a 2 (3.78) the line y  x at P such that OP  6 2, then the
Equation of the circle with PQ as diameter is value of c is

( x x1 )( x x2 ) ( y y1 )( y y2 )  0 (A) 144 (B) 72 (C) 36 (D) 26

x 2 y2 ( x1 x2 ) x ( y1 y2 ) y x1 x2 y1 y2  0 (3.79) Solution: By Pythagoras theorem we have (OC)2 


(OP)2 (CP)2 where C  ( g, f) (Fig. 3.69). Therefore
The original circle is
g 2 f 2  72 ( g 2 f 2 c)
x 2 y2 a 2  0 (3.80)
 c  72
From Eqs. (3.79) and (3.80), we get
y
¨ ( x1 x2 ) · ¨ ( y1 y2 ) ·
2 gg a 2 ff a  2(0) ©
ª 2 ¸
¹
2(0) ©
ª 2 ¸¹  0
C (−g , −f )
and from Eq. (3.78), we have
x
c c a  ( x1 x2 y1 y2 ) a 2  0 y=
P
Hence, the circle described on PQ as diameter cuts the
original circle orthogonally. 6 2

Answer: (C) x
O

40. The condition that the chord x cos A y cos A  p


subtends a right angle at the centre of the circle x2
y2  a2 is
FIGURE 3.69
(A) a2  2p2 (B) p2  2a2
Answer: (B)
(C) a  2p (D) p  2a
Solution: Suppose the chord x cos A y cos A  p 42. The equation of the circle passing through the
meets the circle at points A and B (see Fig. 3.68). There- points of intersection of the circle x2 y2  4 and
fore, the combined equation of the pair of lines OA and the line 2x y  1 and having minimum radius is
OB is
Worked-Out Problems 181

(A) 5(x2 y2) 18x 6y 5  0 44. From point P on the line x y 25  0, tangents PA
(B) 5(x2 y2) 9x 8y 15  0 and PB are drawn to the circle x2 y2  9. As point
P moves on the line, the locus of the midpoint of the
(C) 5(x2 y2) 4x 9y 5  0
chord AB is
(D) 5(x2 y2) 4x 2y 18  0
(A) 5(x2 y2)  9(x y) (B) 5(x2 y2)  3(x y)
Solution: Equation of the required circle is of the
(C) 25(x2 y2)  9(x y) (D) 25(x2 y2)  3(x y)
form
Solution: Let P(h, k) be a point on the line x y 25  0.
x 2 y2 4 L (2 x y 1)  0 (3.81) Therefore
Radius of this circle is
h k  25 (3.82)
L2 1 Since AB being the chord of contact of P with respect to
L2 L 4  5L 2 4 L 16
4 2 the circle x2 y2  9, its equation is
5 4 L 16 hx ky  9 (3.83)
 L2
2 5 5
If M(x1, y1) is the midpoint of the chord AB, its equation
2
5 ¥ 2´ 16 4 is
 ¦§ L µ¶
2 5 5 25
xx1 yy1  x12 y12 (3.84)
2
5 ¥ 2´ 76 Both Eqs. (3.83) and (3.84) represent the chord AB.
 ¦ L µ¶
2 § 5 5 Therefore
This value is minimum when h k 9
  2
2 x1 y1 x1 y12
L
5 Therefore
Substituting the value of K in Eq. (3.81), we have the
9 x1
required circle equation as h
x12 y12
2
x 2 y2 4 (2 x y 1)  0 9 y1
5 k
2 2 x12 y12
 5( x y ) 4 x 2 y 18  0
Answer: (D) Hence, from Eq. (3.82), we have
9( x1 y1 )
43. If the angle of intersection of the circles x2 y2 x 25  h k 
x12 y12
y  0 and x2 y2 x y  0 is A , then the equation
of the line passing through (1, 2) and inclined at an  25( x12 y12 )  9( x1 y1 )
angle A with the positive direction of the x-axis is
Therefore, the locus of M(x1, y1) is
(A) x y 3  0 (B) y  2
25( x 2 y2 )  9( x y)
(C) x y 3  0 (D) x  1
Answer: (C)
Solution: S y x2 y2 x y  0 so that g  1/2, f  1/2
and c  0. 45. If the circles x2 y2 2x 2ky 6  0 and x2 y2
Sa y x2 y2 x y  0 so that ga  1/2, f a  1/2 and ca  0. 2ky k  0 intersect orthogonally, then k is
(IIT-JEE 2000)
Now
¥ 1´ ¥ 1´ ¥ 1 ´ ¥ 1´ 3 3
2 gg a 2 ff a  2 ¦ µ ¦ µ 2 ¦ µ ¦ µ  0  c c a (A) 2 or (B) 2 or
§ 2¶ § 2¶ § 2¶ § 2 ¶ 2 2
3 3
Therefore, S 0 and Sa  0 intersect orthogonally so that (C) 2 or (D) 2 or
2 2
P
A Solution: We have g  1, f  k, c  6 and ga  0, f a  k,
2
ca  k. Therefore,
Therefore, equation of the line is x  1.
2 gg a 2 ff a  c c a
Answer: (D)
182 Chapter 3 Circle

 2(1)(0) 2(k )(k )  6 k Tangents are drawn to the circle S y x2 y2


12  0 at points P and Q. The point of intersection
 2k 2 k 6  0 of these tangents is
 2k 2 3k 4k 6  0
¥ 18 ´ ¥ 18 ´
 k(2k 3) 2(2k 3)  0 (A) ¦ 6, µ (B) ¦ 6, µ
§ 5¶ § 5¶
 (k 2)(2k 3)  0
¥ 18 ´ ¥ 18 ´
Therefore (C) ¦ 6, µ (D) ¦ 6, µ
§ 5¶ § 5¶
3
k  2 or Solution: The common chord of the circles is L y S
2 Sa y 5x 3y 10  0. Note that PQ is L  0. Suppose the
Answer: (A) tangents at points P and Q meet in T(h, k). Therefore, the
equation of the chord PQ is
46. A circle passes through origin and has its centre on
hx ky  12 (3.88)
the line y  x. If the circle cuts the circle x2 y2
4x 6y 10  0 orthogonally, then its equation is However,
(A) x2 y2
2x 2y  0 (B) x2 y2
2x 2y  0 L y 5 x 3 y 10  0 (3.89)
(C) x2 y 2x 2y  0 (D) x y2 2x 2y  0
2 2
is PQ. Therefore, from Eqs. (3.88) and (3.89), we get
x2 y2
Solution: Let S y 2gx 2fy c  0 be the re- h/5  k/ 3  12/10  6/5. Hence
quired circle which passes through (0, 0). This implies that 18
h  6, k 
c0 (3.85) 5
The circle has the centre on the line y  x which implies Therefore
that
¥ 18 ´
g  f T (h, k )  ¦ 6, µ
(3.86) § 5 ¶
g f
Answer: (A)
The circle cuts the circle x2 y2 4x 6y 10  0 orthog-
onally implies that
49. If a > 2b > 0, then the positive value of m for which y
2( g )( 2) 2 f ( 3)  c 10
 mx b 1 m2 is a common tangent to the circles
4 g 6 f  c 10 (3.87)
x2 y2  b2 and (x a)2 y2  b2 is
From Eqs. (3.85)–(3.87), we have g  f  1 and c  0.
Therefore 2b a 2 4b2
(A) (B)
S y x 2 y2 2 x 2 y  0 a 2 4b2 2b

Answer: (D) 2b b
(C) (D)
a 2b a 2b
47. The number of common tangents of the circles S y
x2 y2 4  0 and Sa y x2 y2 6x 8y 24  0 is (IIT-JEE 2002)
2
(A) 0 (B) 1 (C) 3 (D) 4 Solution: We have y  mx b 1 m which is a com-
mon tangent. This implies that
Solution: O  (0, 0) and r1  2, respectively, are the cen-
tre and the radius of S  0. A  (3, 4) and r2  7, respec- m(0) 0 b 1 m2 ma 0 b 1 m2
tively, are the centre and the radius of Sa  0. Now b
OA  2
3 4 2
 5  7 2  r 2 r1 1 m2 1 m2

Thus, S  0 and Sa  0 touch each other internally. There-  b2 (1 m2 )  (ma b 1 m2 )2  m2 a 2


fore, there is only one common tangent for the given 2abm 1 m2 b2 (1 m2 )
circles.
Answer: (B)  m2 a 2 2abm 1 m2  0
 ma  2b 1 m2
48. Let the circles S y x2 y2 12  0 and Sa y x2
y2 5x 3y 2  0 intersect at points P and Q.  m2 a 2  4b2 (1 m2 )
Worked-Out Problems 183

so that m  1 or –1/7. Here m  1 implies that the equa-


 m 2 (a 2 4 b 2 )  4 b 2
tions of L1 is y  x or x – y  0.
2b
m (& m  0) Answer: (B)
a 2 4 b2
Answer: (A) 52. The equation of the circle passing through the inter-
section of the circles x2 y2  4 and x2 y2 2x 4y
50. One of the diameters of the circle circumscribing the 4  0 and touching line x 2y  0 is
rectangle ABCD is x 4y 7  0. If A  ( 3, 4) and (A) x2 y2 x 2y  0 (B) x2 y2 x 2y  0
B  (5, 4), then the area of ABCD is
(C) x2 y2 x 2y  0 (D) x2 y2 x 2y  0
(A) 72 (B) 64 (C) 32 (D) 36
Solution: The circle passing through the intersection
Solution: Let P(h, k) be the centre and Q(1, 4) be the of the given circles is
midpoint of AB (see Fig. 3.70) Since AB is a horizontal
segment, it follows that PQ is vertical. Therefore, h  1 x2 y2 4 L (x 2y 4)  0
and h 4k 7  0  k  2. Hence, the centre of the circle where x 2y 4  0 is the common chord. The centre of
is (1, 2). It is clear that BC  2PQ which implies that this circle is
BC  2(2)  4 ¥ L ´
¦§ , L µ
Now AB  8 and BC  4 which implies that 2 ¶

Area of ABCD  8 s 4  32 and the radius is

L2
L 2 4L 4
D C 4
P (h , k )
This circle touches the line x 2y  0. This implies that
A B
Q
(L / 2) 2( L ) L2
 L 2 4L 4
12 2 2 4
FIGURE 3.70
5L 1
Answer: (C)   5L 2 16 L 16
2 5 2
51. Let L1 be a straight line passing through the origin and  5L 2  5L 2 16L 16
L2 be the line x y  1. If the intercepts made by the  L  1
circle x2 y2 x 3y  0 on L1 and L2 are equal, then
which of the following equations can represent by L1. Hence, the required circle is x 2 y2 x 2 y  0.
(A) x y  0 (B) x y  0 Answer: (A)
(C) 2x 7y  0 (D) x 7y  0 53. If the line 3x 4y k  0 meets the circle x2 y2
(IIT-JEE 1999) 4x 8y 5  0 in two distinct points, then
Solution: Centre of the circle is (1/2, 3/2) and the ra- (A) 35 < k < 15
dius of the circle is 5 / 2 . Let P be the length of the (B) k < 35
perpendicular from the centre (1/2, 3/2) onto the line
L2. Therefore (C) k > 15
(D) k ( d, 35)‡ (15, d)
(1 / 2) (3 / 2) 1 2
P  Solution: Centre of the circle is (2, 4) and the radius
2 2
is 5. The line meets the circle in two distinct points. This
So, the intercept is implies that

5 4 2 3(2) 4(4) k
2 r 2 p2  2   2 5
2 2 2 32 4 2
š 10 k  25
Let y  mx be the equation of L1. Hence, we have
š 25  10 k  25
(m 3)2 š 35  k  15
4
2(1 m2 ) Answer: (A)
184 Chapter 3 Circle

54. The circumference of the circle x2 y2 2x 8y a Hence, S y x 2 y2 5 x 5 y  0.


 0 is bisected by the circle x2 y2 4x 22y b  0.
Answer: (D)
Then a b equals
(A) 25 (B) 35 (C) 45 (D) 50 56. A circle of radius 4 is drawn passing through ori-
gin and whose diameter is along the x-axis. The line
Solution: S y x2 y2
2x 8y a  0 and Sa y x2 y2
y  2x is a chord of this circle. Then the equation of
4x 22y b  0. Since Sa  0 bisects the circumference of
the circle whose diameter is this chord is
S  0, the centre of S  0 lies on the common chord
S Sa  0. Therefore (A) 5(x2 y2) 8x 16y  0
S S a y 6 x 14 y a b  0 (B) 5(x2 y2) 8x 16y  0
a b (C) 5(x2 y2) 8x 16y  0
 S S a y 3x 7 y 0 (D) 5(x2 y2) 8x 16y  0
2
The centre (1, 4) of S  0 lies on S Sa  0. This implies Solution: The given circle equation is
that S y ( x 4)2 y2 16  0
a b  S y x 2 y2 8 x  0
3(1) 7( 4) 0
2
 a b  50 The required circle is of the form
Answer: (D) S L (2 x y)  0
 x 2 y2 8 x L (2 x y)  0
55. The equation of the circle which touches the line x  y
at the origin and bisects the circumference of the circle The centre of this circle (see Fig. 3.71) lies on the line
x2 y2 2y 3  0 is 2 x y  0. This means
(A) x2 y2 5x 5y  0 L
2(4 L ) 0
(B) 2x2 2y2
5x 5y  0 2
2 2
(C) 2x 2y 5x 5y  0  16 5L  0
(D) x2 y2 5x 5y  0 16
L
5
Solution: Let S y x2 y2 2gx 2fy c  0 be the re-
quired circle. It passes through the origin. This implies Therefore, the required circle is
that x2 y2 8x 16/5 (2x y)  0
c0 (3.90)  5(x2 y2) 8x 16y  0
It touches the line x – y  0. This implies that y

g f
 g2 f 2 c
2
 ( g f )2  2( g 2 f 2 ) (& c  0) 2x
=
y
 g 2 f 2 2 gf  0 x
O (4, 0)
2
 (g f )  0
g f 0 (3.91)

It bisects the circumference of the circle x2 y2 2y 3  0.


The centre (0, 1) lies on the common chord 2gx 2(f 1)
y 3  0. Therefore
FIGURE 3.71
2( f 1)( 1) 3  0
Answer: (D)
 2 f  5
5 57. Two circles x2 y2 2ax c2  0 and x2 y2
f  2bx c2  0 meet at points P and Q. If R is a point
2
on the first circle and S is a point on the second circle
Worked-Out Problems 185

such that PR and QS are parallel, then the locus of ( x 4)( x 6) ( y 4)( y 1)  0
the midpoint of RS is
 x 2 y2 2 x 3 y 28  0 (3.92)
(A) x2 y2 (a b)x  0
From Problem 1 of the section ‘Subjective Problems
(B) x2 y2 (a b)x  0 (Sections 3.1 till 3.3)’, the length of the y-intercept of the
(C) x2 y2 (a b)x c2  0 circle provided in Eq. (3.92) is
(D) x2 y2 (a b)x c2  0
9
2 f2 c  2 28  11
Solution: The two circles intersect in P(0, c) and Q(0, 4
c). The lines through P and Q which are parallel to each
other are Therefore, AB  11. The ordinates of A and B are given
by
y  mx c and y  mx c
y2 3 y 28  0
Now y  mx c meets the first circle in R whose coordi-
 ( y 7)( y 4)  0
nates are
 y  7, 4
¥ 2(a mc) 2m(a mc) ´
¦§ , cµ
1 m2 1 m2 ¶ Therefore, A  (0, 7) and B  (0, 4). Equation of the
tangent at A(0, 7) is
and the line y  mx c meets the second circle in S whose
coordinates are 3
x(0) y(7) ( x 0) ( y 7) 28  0
¥ 2(b mc) 2m(b mc) ´ 2
¦§ , cµ  2 x 11y 77  0
1 m2 1 m2 ¶
 2 x 11y 77  0 (3.93)
Let M(x, y) be the midpoint of RS. Therefore
Equation of the tangent at B(0, 4) is
(a b) m(a b)
x and y  3
1 m2 1 m2 x(0) y( 4) ( x 0) ( y 4) 28  0
Now y/x  m. Substituting in 2
 2 x 11y 44  0
x(1 m2 )  (a b)
 2 x 11y 44  0 (3.94)
we get
Solving Eqs. (3.93) and (3.94), we have C  ( 121/4, 3/2)
¥ y2 ´ which is the intersection of the tangents at A and B.
¦ 1 µ  (a b) x Therefore, area of $ABC is
§ x2 ¶
3
x( x 2 y2 ) 1 1 121 ¥ 11´
 (a b) AB s Height  s 11 s ¦ µ
x2 2 2 4 § 2¶
x 2 y2 (a b) x  0 Answer: (C)
Answer: (B)
59. The locus of the centre of the circle whose intercept
on x-axis is of constant length 2a and which passes
58. The extremities of a diagonal of a rectangle are
through a fixed point (0, b) is
( 4, 4) and (6, 1). A circle circumscribes the rect-
angle and cuts an intercept AB on the y-axis. Then (A) x2 2by b2 a2  0
the area of the triangle formed by AB and the two (B) x2 2by b2 a2  0
tangents at A and B is
(C) x2 2by (a b)  0
2
¥ 11´ (11) 2 (D) x2 2by a2 b2  0
(A) ¦ µ (B)
§ 2¶ 2 Solution: See Fig. 3.72.
3 4
¥ 11´ ¥ 11´ k 2 a 2  (Radius)2  h (k b)2
(C) ¦ µ (D) ¦ µ
§ 2¶ § 2¶
where C(h, k) is the centre. Therefore, h2 2bk b2 a2 
Solution: The circumcircle of the rectangle is 0. Hence, the locus of C(h, k) is
x2 2by b2 a2  0
186 Chapter 3 Circle

y
CQ  2 bx (3.95)
(0, b ) CR  2 ax (3.96)
Also AB  b a, BP  b a and APB  90o which
C (h , k )
implies that

x ( AB)2  ( AP )2 ( BP )2
O a a
 (b a)2  ( AP )2 (b a)2
 AP  2 ab (3.97)
FIGURE 3.72 Further AP  RQ  LM. Therefore, from Eqs. (3.95)–
Answer: (A) (3.97), we get
AP  RQ  CR CQ
60. Two circles of radii a and b touch externally. If x is
the radius of a third circle which is between them  2 ab  2 ax 2 bx
and touches them externally and also touching their  ab  ax bx 2 ab ( x)
direct common tangent, then 1/x is equal to
 x(a b 2 ab )
1 1 1 1 1
(A) (B) Therefore
a b a b ab
1 1 2 1 1 1 1 a b 2 ab 1 1 2
(C) (D)  
a b ab a b ab x ab b a ab

Solution: See Fig. 3.73. Points A and B are the centres


with radii a and b, respectively. LM is a direct common tan-
a A b
gent. AP is drawn perpendicular to BM so that APML is B
a rectangle. Through the centre C of the circle with radius R
x P
x, draw a line parallel to AP meeting the line AL at R and L C
Q
the line BM at Q. Now, $ BCQ is right-angled triangle in
which BC is the hypotenuse. We have BC  b x and BQ  M
b x. By Pythagoras theorem, we have
FIGURE 3.73
( BC )2  ( BQ)2 (CQ)2  (b x)2  (b x)2 (CQ)2
Answer: (C)

Multiple Correct Choice Type Questions


1. The centres of the circles passing through (0, 0) and (1, 0) Therefore, the centres of the circles are given by
and touching the circle x2 y2  9 are
¥1 ´
¥1 ´ ¥ 1 ´ ( g, f )  ¦ , p 2 µ
(A) ¦ , 2 µ (B) ¦ , 2 µ §2 ¶
§2 ¶ § 2 ¶
Answers: (A), (C)
¥1 ´ ¥ 1 ´
(C) ¦ , 2 µ (D) ¦ , 2 µ
§2 ¶ § 2 ¶ 2. The equations of the chords of length 5 and passing
through the point (3, 4) on the circle 4x2 4y2 24x
Solution: Suppose S y x2 y2 2gx 2fy c  0 is the 7y  0 are
required circle. It passes through (0, 0)  c  0. It passes (A) 4x 3y  0 (B) 4x 3y  0
through (1, 0)  g  1/2. It is clear that S  0 touches
(C) 4x 3y 24  0 (D) 4x 3y 12  0
internally the circle x2 y2  9. Therefore
Solution: We have that the centre is C  (3, 7/8) and
g2 f 2  3 g2 f 2 c  3 g2 f 2 (& c  0) the radius 25/8 (see Fig. 3.74). Any line through (3, 4)
 4( g 2 f 2 )  9 of the form x 3/cos P  y 4/sin P  r (say) so that the
other point of the extremity of the chord is (3 r cos P,
¥1 ´ ¥ 1´ 4 r sinP ). Since the length of the chord is 5, we have
 4¦ f 2µ  9 ¦§& g  µ¶
§4 ¶ 2 r  5 and
f p 2
Worked-Out Problems 187

2 2 Let y  m(x 2) be a line through A(2, 0). This line makes


2 ¥ 7´ ¥ 25 ´
(3 5 cos Q 3) ¦ 4 5 sin Q µ  ¦ µ an angle of 45o with the line [given in Eq. (3.98)]. So we
§ 8¶ § 8¶
have
Therefore m 4/3
1
25 ´
2 2 1 4m / 3
¥ ¥ 25 ´
25 cos2 Q ¦ 5 sin Q µ  ¦ µ  3m 4  3 4 m
§ 8¶ § 8¶
125  3m 4  p 3 4 m
 25 sin Q  0
4 Therefore
Hence 1
m  7 or m 
4 7
sin Q 
5 Hence, the lines are x 7y 2  0 and 7x y 14  0.
3 Since the centres of the circles are at a distance of 5 2
cos Q  p
5 from A(2, 0), we have
4
tan Q  p x 2 y
3   p5 2
( 1 / 50 ) (7 / 50 )
Therefore, the chord equation is given by x 2 y
and   p5 2
4 (7 / 50 ) (1 / 50 )
y 4  p  x 3
3 Therefore, the centres of the circles are (1, 7), (3, 7), (9,
Hence 1) and ( 5, 1).
4 x 3 y  0 and 4 x 3 y 24  0 Answers: (A), (B), (C), (D)

4. The point(s) on the line x  2 from which the tangents


drawn to the circle x2 y2  16 are at right angles is
(are)
7
C 3, (A) (2, 2 5 ) (B) (2, 2 5 )
8

P (3, 4) Q
(C) (2, 2 7 ) (D) (2, 2 7 )
Solution: If the tangents drawn are at right angles, the
points must be the intersection of the line x  2 with the
FIGURE 3.74 director circle x2 y2  32 of the circle x2 y2  16. Sub-
Answers: (B), (C) stituting x  2 in x2 y2  32, we have
y2  28 or y  p2 7
3. A(2, 0) is a point on the circle (x 2)2 (y 3)2  25.
A line through A(2, 0) making an angle of 45° with the Therefore, the points are (2, 2 7 ) and (2, 2 7 ).
tangent to the circle at A is drawn. Then the equations Answers: (C), (D)
of the circles with the centres on these lines which are
at a distance of 5 2 units from point A and of radius 5. If the circles S y x2 y2 2x 2 L y 6  0 and Sa y
3 are x2 y2 2 L y L  0 cut each other orthogonally,
(A) (x 1)2 (y 7)2  9 then K is equal to
(B) (x 3)2 (y 7)2  9 3 3
(A) (B) 2 (C) 2 (D)
(C) (x 9)2 (y 1)2  9 2 2
(D) (x 9)2 (y 1)2  9 Solution: S  0 and Sa  0 cut orthogonally. This implies
that
Solution: The given circle is x2 y2 4x 6y 12 
0. Therefore, the equation of the tangent to this circle at 2(1)(0) 2(L )(L )  6 L
A(2, 0) is
 2L 2 L 6  0
x(2) y(0) 2(x 2) 3(y 0) 12  0
 2 L 2 4 L 3L 6  0
 4 x 3y 8  0 (3.98)  2 L (L 2) 3(L 2)  0
188 Chapter 3 Circle

Therefore (iii) Let p be the perpendicular from (1, 2) onto the


line L3 y 3x y  0. Therefore
3
L  2,
2 3(1) ( 1) 5

Answers: (A), (C) 10 2
Hence the intercept is
6. The equations of the circles of radius 1 unit and touch-
ing the circles x2 y2 2x  0 and x2 y2 2x  0 are 5
2 5  10
(A) x2 y2 2 3y 20 2

(B) x2 y2 2 2 x  0 (iv) Let p be the perpendicular from (1, 2) onto L4 y


x 3y 10  0. Therefore
(C) x2 y2 2 2 x 1  0
1 3( 2) 10 5
(D) x2 y2 2 3 y 2  0 
10 2
Solution: The two given circles are of unit radius
and having centres at ( 1, 0) and (1, 0), respectively. Hence the intercept is
The required circle is also of unit radius. Hence, the 5
required circle must touch these two given circles ex- 2 5  10
2
ternally. Also, the given circles x2 y2 2x  0 and
x2 y2 2x  0 touch each other externally at origin. Note: You can guess that the chords are of equal length
Hence, the centre of the required circle from the origin if they are located at equal distance from the centre.
is 3. Therefore, the required circles are Answers: (A), (B), (C), (D)
x2 (y 3 )2  1
8. The equations of the tangents drawn from the origin
and x2 (y 3 )2  1 to the circle S y x2 y2 2rx 2hy h2  0 are
Answers: (A), (D) (A) x  0 (B) y  0
(C) (h r )x 2rhy  0 (D) (h2 r2)x 2rhy  0
2 2
7. On which of the following lines, the circle S y x2
y2 2x 4y  0 makes equal lengths of intercepts? Solution:

(A) 3x y 10  0 (B) x 3y  0 x0

(C) 3x y  0 (D) x 3y 10  0  y2 2 hy h2  0
 ( y h)2  0
Solution: A(1, 2) and 5 are centre and radius of the
circle S  0. Therefore, y-axis touches the circle at (0, h). That is, x  0
(i) Let p be the length of the perpendicular from the is a tangent. Suppose y  mx is a tangent to S  0. Then
centre onto the line L1 y 3x y 10  0. Therefore m(r ) h
 r 2 h2 h2
3(1) ( 2)10 5 2
1 m

2
3 1 2 2  (h mr )2  r 2 (1 m2 )
Hence, the length of the intercept is  h2 2 mrh r 2  0
h2 r 2
5 m
2 5  10 2 hr
2
Therefore
(ii) Let p be the perpendicular from (1, 2) onto the
line L2 y x 3y  0. Therefore ¥ h2 r 2 ´ 2 2
y¦ µ x or (h r ) x 2rhy  0
1 3( 2) 5 § 2 hr ¶

10 2 Answers: (A), (C)

Hence, the intercept is 9. A straight line through the vertex P of a triangle PQR,
intersects the side QR at the point S and the circum-
5
2 5  10 circle of the triangle PQR at the point T. If S is not the
2 circumcentre of the triangle, then
Worked-Out Problems 189

1 1 2 Note: In choice (B), equality may occur when S is the


(A)  midpoint of PT. This can happen if we consider a right-
PS ST QS s SR
angled triangle PQR, where P  (a, b)(a > 0, b > 0), Q 
1 1 2 (0, 0) and R  (a, 0) such that a2 > 8b2 [see Problem 26 in
(B)  the section ‘Subjective Problems (Sections 3.1 till 3.3)’].
PS ST QS s SR
In particular, take P  (3, 1), Q  (0, 0), R  (3, 0) which is
1 1 4 right-angled triangle right angled at R and take T  (1,
(C) 
PS ST QR 1) so that the midpoint of PT lies on QR.
1 1 4 Answers: (B), (D)
(D) 
PS ST QR
10. The circles x2 y2  400 and x2 y2 10x 24y 120
(IIT-JEE 2008) 0
Solution: Since S is not the circumcentre, the two (A) touch each other externally.
chords PT and QR cannot bisect each other (see Fig. (B) touch each other internally.
3.75), but QR may bisect PT (see the Note at the end of
(C) point of contact is (100/13, 240/13).
the solution). Using AM GM inequality, the equality
(D) point of contact is (100/13, 240/13).
1 1 1 1
r2 – Solution: The centres of the circles are O(0, 0) and
PS ST PS ST
A(5, 12), respectively. The radii are r1  20 and r2  7, re-
occurs if PS  ST. However, spectively. OA  13  r1 r2 implies that the two circles
touch internally. Let P be the point of contact so that
PS – ST  QS – SR (by similar triangle) OP:PA  20: 7. Therefore
Therefore
¥ 100 240 ´
P¦ ,
1 1 2 § 13 13 µ¶
r (3.99)
PS ST QS – SR Answers: (B), (C)

Also the equality 11. If the circle x2 y2 2gx 2fy c  0 cuts the curve
xy  1 in four points (xi, yi) (i  1, 2, 3, 4), then
QR QS SR 1 1 1 1
  r2 –
QS – SR QS – SR SR QS QS SR (A) x1 x2 x3 x4  1
(B) y1 y2 y3 y4  1
The equality or occurs if QS  SR. Therefore
(C) x1 x2 x3 x4  2g
QR
r QS – SR (D) y1 y2 y3 y4  2f
2
4 1 Solution: Substituting y  1/x in the circle equation, we
b get
QR QS – SR (3.100)
1 2f
From Eqs. (3.99) and (3.100), we have x2 2
2 gx c  0
x x
1 1 4 Therefore, x4 2gx3 cx2 2fx 1  0 whose roots are

PS ST QR x1, x2, x3 and x4. Hence,
because PS  ST and QS  SR cannot hold simultane- x1 x2 x3 x4  2g,
ously (by the hypothesis, S is not the circumcentre). ¤ x1x2x3  2f
P and x1x2x3x4  1
Now
1
y  y1 y2 y3 y4  1
x
Q R
S and

T y1 y2 y3 y4 
1

1

1

1

x1 x2 x3 ¤
 2 f
x1 x2 x3 x4 x1 x2 x3 x4
FIGURE 3.75
Answers: (A), (B), (C), (D)
190 Chapter 3 Circle

12. Let C1: x2 y2  4 and C2: x2 y2 6x 8y 24  0.  c + 1  p4


Then
Therefore, c  3, 5. Hence, the required tangents are
(A) C1 and C2 touch each other internally
yx 3 and y  x 5
(B) C1 and C2 do not intersect
Answers: (A), (C)
(C) the number of common tangents is 2
(D) the number of common tangents is 1 15. The centre of a circle C lies on the line 2x 2y 9  0
and the circle cuts orthogonally the circle C1: x2 y2 
Solution: We have
4. Then C passes through
A  (0, 0) and r1  2
(A) ( 3, 3) (B) ( 1/2, 1/2)
B  (3, 4) and r2  7
(C) ( 4, 4) (D) ( 2, 2)
Now, AB  5  7 2  r2 r1. Therefore, the two circles
C1 and C2 touch each other internally and hence they Solution: Let the equation of C be S y x2 y2
have only one common tangent. 2gx 2fy c  0. The centre ( g, f ) lies on 2x 2y 9  0
implies that
Answers: (A), (D)
2 g 2 f  9 (3.101)
13. The equation of the circle with centre (4, 3) and
touching circle x2 y2  1 is The circle C cuts the circle C1 orthogonally. This implies
that
(A) x2 y2 8x 6y 11  0
2 g (0 ) 2 f (0 )  c 4
(B) x2 y2 8x 6y 9  0
(C) x2 y2 8x 6y 9  0 c4 (3.102)
(D) x2 y2 8x 6y 11  0 From Eqs. (3.101) and (3.102), the equation of the circle
C is given by
Solution: Since (3, 4) lies outside the circle x2 y2  1,
one circle has the external contact with x2 y2  1 and S y x 2 y2 2 gx (2 g 9) y 4  0
the other circle has the internal contact. O  (0, 0) and
r1  1. Let A  (4, 3) and r2 be the radius of the required y x 2 y2 9 y 4 2 g( x y)  0
circle OA  5. The circle C passes through the intersection of the circle
Case 1: r2  5 1  4 (i.e. external contact). Hence, the x2 y2 9y 4  0 and the line x y  0 and these points
required circle is are ( 1/2, 1/2) and ( 4, 4).
Answers: (B), (C)
(x 4)2 (y 3)2  16
x2 y2 8x 6y 9  0 16. The equation of the circle passing through origin and
Case 2: r2  5 1  6 (i.e. internal contact). Hence, the touching the lines x 2  0 and 3x 4y 50  0 is
required circle is (A) x2 y2 6x 8y  0
(x 4)2
(y  363)2 (B) x2 y2 6x 8y  0
2 2
 x y 8x 6y 11  0 (C) x2 y2 16x 12y  0
Answers: (B), (D) (D) x2 y2 8x 12y  0

14. The equation of the tangents to the circle x2 y2 Solution: The centre of the required circle lies on the
6x 4y 5  0 which is inclined at an angle of 45o line 2x y 10  0 which is an angular bisector of the
with the x-axis is lines x 2  0 and 3x 4y 50  0. Let C(@, 10 2@) be
the centre of the required circle. Hence
(A) x y 3  0 (B) x y 3  0
(C) x y 5  0 (D) x y 5  0 A 2 (10 2A )2  A 2
Solution: Required tangent equation is of the form  5A 2 40A 100  A 2 4A 4
y  x c. A  (3, 2) and r  2 2, respectively, are the cen-
tre and the radius of the given circle. Therefore, the line  4A 2 44A 96  0
y  x c touches the given circle implies that  A 2 11A 24  0
3 2 c  (A 3)(A 8)  0
2 2
2 2
1 1  A  3, 8
Worked-Out Problems 191

Therefore, the required centre of the circles are (3, 4) and y


(8, 6), respectively, and their radii are the distances of
their centre from the origin which are 5 and 10, respec-
tively. Hence, the circles are
(a , 10 − 2a )
(x 3)2 (y 4)2  25 and (x 8)2 (y 6)2  100
That is, x
−2 O
x2 y2 6x 8y  0 and x2 y2 16x 12y  0

FIGURE 3.77

Answers: (A), (C)

Matrix-Match Type Questions


1. Match items of Column I with those of Column II. a b
xx1 y(0) ( x x1 ) ( y 0)  x12 ax1
2 4
Column I Column II  4 xx1 2ax 2ax1 by  4 x12 4ax1
(A) If x-axis bisected (p) ( d, 2) ‡ (2, d)  2(2 x1 a) x by 2ax1 4 x12  0
each of two chords
drawn from the point This passes through the point (a, b/2). This implies
(a, b/2) on the circle that
2x(x a) y(2y b) 
¥ b´
0 (ab x 0), then a/b 2(2 x1 a)a b ¦ µ 2ax1 4 x12  0
§ 2¶
belongs to (q) ( 2, 2)
(B) If the circles x2 y2 ¥ b2 ´
 4 x12 6ax1 ¦ 2a 2 µ  0
10x 16  0 and § 2¶
x2 y2  r2 intersect (which has two distinct real roots)
in two distinct
points, then r lies in ¥ b2 ´
 4 x12 6ax1 ¦ 2a 2 µ  0
the interval (r) ( d, 2 ) ‡ ( 2, d) § 2¶
(C) If the line y x  0 (which has two distinct real roots)
bisects chords drawn
from the point (1
¥ b2 ´
 (6a)2  4(4) ¦ 2a 2 µ
a 2 /2, 1 a 2 /2) § 2¶
to the circle 2x2  9a 2  2(4a 2 b2 )
(s) ( 3, 3)
2y2 (1 a 2 )x
 a 2  2b2
(1 a 2 )y  0, then
a belongs to a
 2
(D) Point (2, K) lies b
inside the circle x2
y2  13 if and only if (t) (2, 8) Therefore
K belongs to a a
 2 or  2
b b
Solution:
Answer: (A) m (r)
(A) The given circle equation is
(B) O  (0, 0) and A  (5, 0) are the centres and r, 3 are
S y 2 x 2 2 y2 2ax by  0 the radii of the circles. The two circles intersect in
b two distinct points. So
S y x 2 y2 ax y  0
2 r 3  OA  r 3
Let (x1, 0) be the midpoint of a chord of the circle. š r 3  5r 3
Therefore, the equation of the chord is
192 Chapter 3 Circle

5  r 3  2  r and r 3  5  r  8 2. Match the items of Column I with those of Column II.


š2r8
Answer: (B)m(p), (r), (t) Column I Column II

(C) Let (A) The intercept on the (p) x2 y2 18x 6y


line y  x by the circle 120  0
1 2a x2 y2 2x  0 is AB.
A Equation of the circle
2
described on AB as
so that
diameter is
1 2a (B) The equation of the (q) 25(x2 y2) 20x
A
2 circle with radius 2y – 60  0
5 and touching the
Therefore, the given circle equation is S y x2 y2 circle x2 y2 2x
A x A y  0 which passes through origin. Let 4y – 20  0 at the
(x1, x1) be the midpoint of the chord. Therefore, point (5, 5) is
the chord equation is S1  S11 . That is,
(C) The equation of a (r) x2 y2 3x 3y – 8
A A circle which passes 0
xx1 y  x1 ( x x1 ) ( y x1 )  x12 x12
2 2 through the point (2,
A x1 A x1 0) whose centre is
the limit of the point
 2xx1 2 x1 y A x A x1 A y A x1  4 x12 2A x1 of intersection of the
2A x1 lines 3x 5y  1 and
(2 c)x 5c2y  1 as
 (2 x1 A ) x (2 x1 A ) y A x1 A x1 4 x1  0 c tends to 1 is
This passes through (A , A ). This implies that (D) The equation of (s) x2 y2 – x – y  0
the circle passing
(2 x1 A )A (2 x1 A )A (A A ) x1 4 x12  0 through the points
 4 x12 3(A A ) x1 (A 2 A 2 )  0 of intersections of
the circles x2 y2
 4 x12 3(A A ) x1 (A 2 A 2 )  0
4x 2y – 8  0 and
(has two distinct real roots in x1 ) x2 y2 2x 4y –
 9(A A )2  16(A 2 A 2 ) 8  0 at the point
( 1, 4) is
¥ 1 2a 2 ´
 9(2a 2 )  16 ¦ µ
§ 2 ¶ Solution:
¨ 1 a 2 1 a 2 · (A) Substitute y  x in the circle equation. Therefore
©& A  and A  ¸
ª 2 2 ¹
2x2 2x  0
2 2
 9a  4(1 2a )  x  0, 1
 a2  4 Hence, A(0, 0) and B  (1, 1). Therefore, the equa-
 a( d, 2) ‡ (2, d) tion of the circle with AB as diameter is given by
Answer: (C) m (p), (r) x( x 1) y( y 1)  0
(D) O  (0, 0) and r  13 . Let P  ( 2, L ).  x 2 y2 x y  0
P lies inside the circle
Answer: (A) m (s)
š OP  r (B) The equation of the tangent at (5, 5) to the given
š L 2 2 2  13 circle is
š L 3 x(5) y(5) ( x 5) 2( y 5) 20  0
š 3  L  3 That is, 4x 3y 35  0. The required circle is of the
Answer: (D) m (s) form x2 y2 2x 4y 20 L (4x 3y 35)  0. The
radius of the circle is 5. This implies that
Worked-Out Problems 193

2 That is,
¥ 3L ´
(1 2 L ) ¦ 2 µ 35L 20  25
§ 2¶ 25(x2 y2) 20x 2y 60  0
 4(1 2 L )2 (4 3L )2 140 L 20  0 Answer: (C)m (q)

 25L 2 100 L  0 (D) Let


 5L 2 20 L  0 S y x2 y2 4x 2y 8  0
 L  0 or 4 and Sa y x2 y2 2x 4y 8  0
L  0 gives the circle x2 y2 2x 4y 20  0 and Let
L  4 gives the required circle which is x2 y2
18x 16y 120  0. L y S Sa y 2x 2y  0
Aliter: Since the radius of the circle x2 y2 2x 4y Hence, the required circle is of the form
20  0 is 5 and the radius of the required circle is
S L L y x2 y2 4x 2y 8 L (x y)  0
also 5, it follows that the contact must be external
contact and the point (5, 5) must be the midpoint of This passes through ( 1, 4). This implies
the segment joining the centres. Hence, if (h, k) is
1 16 4 8 8 L ( 1 4)  0  L  1
the centre of the required circle, we have
Therefore, the required circle is
1 h 2 k
 5 and 5
2 2 x2 y2 3x 3y 8  0
 h  9, k  8 Answer: (D) m (r)
Hence, the equation of the required circle is
3. Match the items of Column I with those of Column II.
2 2
( x 9) ( y 8)  25
 x 2 y2 18 x 16 y 120  0 Column I Column II
Answer: (B)m(p) (A) The equation of (p) x2 y2 4x 2y
the circle passing 40
(C) We have the equation of the two lines as through the points
3x 5 y  1 (1, 1), (2, 2) and
having radius 1 is
and (2 c) x 5c 2 y  1
(B) If the two lines
2 2
On solving these equations, we get 3x 2y – 8  0 and (q) x y 2x 4y – 11
2x – y – 5  0 lie 0
1 c2 along two diameters
x
2 c 3c 2 of a circle which
c 1 touches the x-axis,
y
5(2 c 3c 2 ) then the equation of
the circle is (r) x2 y2 4x 2y
Therefore 40
(C) The equation of the
1 c 2 circle which touches
lim x  lim 
c m1 c m1 2 3c 5 the lines 4x 3y
10  0 and 4x 3y –
1 1
and lim y  lim  30  0 and whose
c m1 c m1 5(2 3c) 25 (s) 3x2 3y2 10x 6y
centre lies on the
– 45  0
Hence, the centre of the circle  (2/5, 1/25). Since line 2x y  0 is
the circle passes through (2, 0), its radius is (D) Equation of the
circle which passes
2
¥ 2´ ¥ 1´ 1601 through the points
¦§ 2 µ¶ ¦§ µ¶  (0, 5) (6, 1) whose (t) x2 y2 2x 4y
5 25 25
centre lies on the 40
Therefore, equation of the required circle is line 12x 5y  25 is
2 2
¥ 2´ ¥ 1´ 1601
¦§ x µ¶ ¦§ y µ¶  (IIT-JEE 1999)
5 25 625
194 Chapter 3 Circle

Solution: 12g 5f  25 (3.108)


(A) Let S y x2 y2 2gx 2fy c  0 be the required Solving Eqs. (3.106)–(3.108), we get g  5/3, f  1
circle. Therefore and c  15. Therefore, the required circle is
2 g 2 f c  2 (3.103) 10 x
S y x2 y2 2y 15  0
and 4 g 4 f c  8 (3.104) 3
 3x2 3y2 10x 6y 45  0
The radius is 1. This implies that
Answer: (D)m(s)
g2 f 2 c  1 (3.105)
4. Match the items of Column I with those of Column II.
Solving Eqs. (3.103)–(3.105), we have g  1, f  2
and c  4 or g  2, f  1 and c  4. Therefore, there
are two circles Column I Column II
x2 y2 2x 4y 4  0 (A) Equations of the cir- (p) x2 y2 9x 9y
cle circumscribing 36  0
and x2 y2 4x 2y 4  0 the rectangle whose
which satisfy the given conditions. sides are x 3y –
Answer: (A)m(p), (t) 4  0, 3x y – 22  0,
x 3y – 14  0 and
(B) The point of intersection of the lines 3x 2y 8  0 3x y – 62  0 is (q) 3 (x2 y2) 2y
and 2x y 5  0 is (2, 1) which is the centre of the
(B) Two vertices of an 3 0
required circle. Since the circle touches the x-axis,
equilateral triangle
its radius must be the absolute value of the ordinate
are ( 1, 0) and (1, 0)
of the centre. Hence, the radius is 1. Therefore, the
and its third vertex
required circle is x2 y2 4x 2y 4  0.
lies above the x-axis.
Answer: (B) m(r) The equation of its (r) x2 y2 – 9  0
(C) The two lines 4x 3y 10  0, 4x 3y 30  0 are circumcircle is
parallel tangents to the required circle. Therefore, (C) The equations of a
the length of the diameter of the required circle is circle with origin at
the distance between the parallel tangents which is centre and passing
2 2
equal to through the vertices of (s) x y 27x 3y
an equilateral triangle 142  0
30 10 whose median is of
8
4 2 32 length 6 units is
Hence, the radius of the required circle is 4. Also the (D) The vertices of a
line 2x y  0 intersects the two parallel tangents in triangle are (6, 0),
(t) x2 y2 – 16  0
the points A( 1, 2) and B(3, 6) (these points A and B (0, 6) and (7, 7).
are obtained on substituting y  2x in the tangents The equation of
equations). Therefore, the centre of the required the incircle of the
circle is (1, 2) and the circle equation is given by triangle is

( x 1)2 ( y 2)2  4 2 Solution:


 x 2 y2 2 x 4 y 11  0 (A) Solving the given equations, the two opposite verti-
Answer: (C)m(q) ces of the rectangle are obtained as (7, 1) and (20, 2)
and hence the equation of the circle is
x2 y2
(D) Let S y 2gx 2fy c  0 be the required
circle. It passes through (0, 5). This implies that (x 7)(x 20) (y 1)(y 2)  0

10f c  25 (3.106)  x 2 y2 27 x 3 y 142  0


Answer: (A)m(s)
It passes through (6, 1). This implies that
(B) Suppose A  ( 1, 0) and B  (1, 0) which lies on x-
12g 2f c  37 (3.107)
axis such that the origin is the midpoint of the seg-
The centre lies on the line 12x 5y  25. This implies ment AB (see Fig. 3.77). Hence, the third vertex
that lies on the positive y-axis. Suppose the third vertex
Worked-Out Problems 195

is C(0, k). Since CO is the median and CO is per- We know that if r is the radius of the incircle, then
pendicular to the side AB, it is the distance of the centre from any side of the
triangle (see Fig. 3.78). One side of the triangle is
3 CO x y  6. Therefore
 sin 60°  (& AB  BC  CA  2)
2 2
9 9
6
Therefore 2 2 3
r 
2 2 2
CO  3 1 1
k 3 Therefore, the incircle equation is
2 2
So C  (0, 3 ). In an equilateral triangle, the cir- ¥ 9´ ¥ 9´ 9
cumcentre and the centroid are the same. If G is the ¦§ x µ¶ ¦§ y µ¶ 
2 2 2
centroid of $ ABC, then
 x 2 y2 9 x 9 y 36  0
¥ k´ ¥ 1 ´
CG : GO  2 : 1  G  ¦ 0, µ  ¦ 0, µ (0, 6)
§ 3¶ § 3¶
Therefore, the radius of the circumcircle is 1/ 3 .
Hence, the circumcircle equation is given by
5√2 6√2
2 I
¥ 1 ´ 4 r r
( x 0) 2 ¦ y µ 
§ 3¶ 3
r
2y 1 4
 x 2 y2  (7, 7) 5√2 (6, 0)
3 3 3
 3 ( x 2 y2 ) 2 y 3  0 FIGURE 3.78
Answer: (D)m(p)
y
C (0, k) 5. Match the items of Column I with those of Column II.

2 2
Column I Column II
G
(A) The equation of the (p) 16(x2 y2) 48x
locus of the midpoints 16y 31  0
60° 60° of the chords of the
A (−1, 0) 0 B (1, 0) x circle 4x2 4y2 12x
4y 1  0 that subtend
an angle 2π/3 at its
centre is
(B) The locus of the
FIGURE 3.77 midpoint of chords (q) x2 y2 2x 2y
Answer: (B)m(q) of the circle x2 y2 10
2x 2y – 2  0 which
(C) In an equilateral triangle, the circumcentre and the subtend an angle
centroid are the same and hence the circumcentre 2π/3 at the centre is
is (0, 0). Since the centroid divides the median in
(C) The locus of the
the ratio 2:1 (from vertex to the base) it follows that
midpoint of the chords
the radius is (2/3)6  4. Hence, the circle equation is
of the circle x2 y2 4
x2 y2  16. 2 2
such that the segment (r) x y 2x 2y  2
Answer: (C) m(t) intercepted by the
(D) Perimeter of the triangle is 16 2. Using the incen- chord on the curve x2
tre formula, the incentre is obtained as 2x 2y  0 subtends
a right angle at the
¥ 72 2 72 2 ´ ¥ 9 9 ´ origin is
¦ , µ ¦ , µ
§ 16 2 16 2 ¶ § 2 2 ¶ (Continued)
196 Chapter 3 Circle

(B) A  (1, 1) is the centre and r  2 is the radius. Pro-


Column I Column II
ceeding as above, we get 1/2  AM/2. Therefore
(D) Through the point (s) x2 y2 2x 3y  0
(2, 3), secants are ( AM )2  1
drawn to the circle  ( x1 1)2 ( y1 1)2  1
x2 y2  4. Then,
the locus of the Therefore, the locus is x2 y2 2x 2y 1  0.
midpoints of these (t) x2 y2 3x 2y Answer: (B)m(q)
secants intercepted 40
by the circle is (C) Let M(x1, y1) be the midpoint of a chord so that its
equation is xx1 yy1  x12 y12. Suppose the chord
Solution: intersects the curve x2  2(x y) in two points P and
Q. Therefore, the combined equation of the pair of
(A) Given circle is S y x2 y2 3x y (1/4)  0. A  (3/2, lines OP and OQ is obtained as
1/2) is the centre and r  3/2 is the radius. Let M(x1,
y1) be the midpoint of a chord BC. Therefore, AM is ¥ xx yy1 ´
x 2  2( x y) ¦ 21 µ
perpendicular to BC and from Fig. 3.79, we have § x1 y12 ¶
BAM  CAM  60°
 ( x12 y12 ) x 2 2( x y)( xx1 yy1 )  0 (3.109)
Now
Since POQ  90°, from Eq. (3.109), we have
AM 1 AM
cos 60o    Coefficient of x2 Coefficient of y2  0
AB 2 AB
3 Therefore
  2 AM
2
( x12 y12 2 x1 ) 2 y1  0
¨¥ 2 2
3´ ¥ 1´ ·
 9  16 ©¦ x1 µ ¦ y1 µ ¸ So, the locus of (x1, y1) is
©ª§ 2¶ § 2 ¶ ¸¹
x 2 y2 2 x 2 y  0
¨ 9 1·
 9  16 © x12 y12 3 x1 y1 ¸ Answer: (C)m(r)
ª 4 4¹
(D) Let M(x1, y1) be the midpoint of a secant chord
Therefore, the locus is through (2, 3) of x2 y2  4 so that its equation is
16(x2 y2) – 48x 16y 31  0 xx1 yy1  x12 y12
This passes through the point (2, 3). This implies
3 , −1
A 2 x1 3 y1  x12 y12
2 2
3/2 60° 60° 3/2 Therefore, the locus of (x1, y1) is

B C x2 y2 2x 3y  0
M(x1,y1)
Answer: (D)m(s)

FIGURE 3.79
Answer: (A)m(p)

Comprehension Type Questions


1. Passage: If S  0 is a circle in standard form and L  0 is the line x y 11  0 and which passes through
a straight line in the plane of the circle, then, in general, the intersection of the circle x2 y2 3x 2y
the equation S L L  0 represents a circle. If L  0 4  0 and the line 2x 5y 2  0 is
cuts S  0 in two points P and Q, then S L L  0 rep-
(A) x2 y2 9x 13y 10  0
resents circle passing through P and Q. In particular,
if L  0 touches the circle S  0, then S L L  0 also (B) x2 y2 9x 13y 10  0
touches L  0. Answer the following questions. (C) x2 y2 9x 13y 10  0
(i) The equation of the circle whose centre lies on (D) x2 y2 9x 13y 10  0
Worked-Out Problems 197

(ii) The equation of the circle touching the line  L 4  L 1


2x 3y 1  0 at (1, 1) and cutting orthogonally 3
the circle having the line segment joining (0, 3) L
and ( 2, 1) as diameter is 2
Therefore, the required circle is
(A) 2x2 2y2 – 10x 5y – 1  0
(B) x2 y2 – 10x 5y 1  0 5 3
x 2 y2 5 x y 2  0
(C) 2(x2 y2) 10x 5y – 1  0 2 2
(D) 2(x2 y2) 10x 5y 1  0  2x2 2y2 10x 5y 1  0
Answer: (D)
(iii) The equation of the circle passing through the
intersection of circles x2 y2  4 and x2 y2 2x (iii) Let S y x2 y2 4  0 and Sa y x2 y2 2x 4y 4 
4y 4  0 and touching the line x 2y 5  0 is 0. Let L y S Sa y 2x 4y 8  0. That is,
(A) x2 y2 – x 2y  0 L y x 2y 4  0
(B) x2 y2
– x 2y  0 Now, the required circle is
(C) x y2 x 2y  0
2
S L L y x 2 y 2 4 L ( x 2 y 4)  0
(D) x2 y2 x 2y  0
¥ L´
(IIT-JEE 2004) y x 2 y2 2 ¦ µ x 2 L y 4 4 L  0
§ 2¶
Solution:
Therefore, the centre is
(i) Let S y x2 y2 3x 2y – 4  0 and L y 2x 5y
¥ L ´
2  0. The required circle is of the form ¦§ , L µ
2 ¶
S KL y x2 y2 3x 2y – 4 K(2x 5y 2)  0
and the radius is
whose centre is
L2
¥ 3 2 L (5L 2) ´ L 2 4 4L
¦§ , µ¶ 4
2 2
S L L  0 touches the line x 2y 5  0. This
which lies on the line x y – 11  0. That is, implies that
3 2 L 5L 2 22  0 L
2( L ) 5
 7 L 21  0 2 L2
 L 2 4 4L
 L  3 12 2 2 4
Therefore, the required circle is x2 y2 – 9x – 13y – 5L 10 1
10  0.   5L 2 16 L 16
2 5 2
Answer: (C)
(ii) The circle described on the line joining the points  5(L 2)2  5L 2 16 L 16
(0, 3) and ( 2, 1) as diameter is  4L 4  0
x( x 2) ( y 3)( y 1)  0  L  1
L  1 gives the circle x 2 y2 x 2 y  0.
2 2
 x y 2x 2y 3  0 (3.110) Answer: (A)
The required circle is of the form
2. Passage: ABCD is a square of side length 2 units. C1
( x 1)2 ( y 1)2 L (2 x 3 y 1)  0 is the circle inscribed in ABCD and C2 is the circum-
circle of ABCD. L is a fixed line in the same plane
¥ 3L ´
 x 2 y2 2(L 1) x 2 ¦ 1µ y 2 L  0 (3.111) through the point A. Answer the following questions:
§ 2 ¶
(i) If P is any point on C1and Q is any point on C2,
The circle given in Eq. (3.111) cuts orthogonally the then
circle given in Eq. (3.110). This implies that
( PA)2 ( PB)2 ( PC )2 ( PD)2
¥ 3L ´ 
2(L 1)(1) 2 ¦ ( 1)  2 L 3 (QA)2 (QB)2 (QC )2 (QD)2
§ 2 + 1µ¶
(A) 0.75 (B) 1.25 (C) 1 (D) 0.5
198 Chapter 3 Circle

(ii) A circle touches the line L and C1 externally (ii) Suppose L1 is a line parallel to L at a unit distance
such that both circles are on the same side of L. from L such that L1 and the circle C1 are on opposite
Then the locus of the centre of C is sides to the line L. Therefore, the centre of the circle
C is equidistant from the line L1 and the centre of
(A) an ellipse (B) a parabola
the circle C2. Hence, the centre of C is a parabola
(C) hyperbola (D) a pair of lines with focus at the centre of C2 and directrix L1.
(iii) A line M through A is drawn parallel to BD. Answer: (B)
Point S moves such that its distance from the
(iii) Clearly, the locus S is a parabola with A as a focus
line BD and vertex are equal. If locus of S cuts
and BD as directrix (see Fig. 3.81).
M at T2 and T3 and AC at T1. Then the area of
$ T1T2T3 is 1 2 1
AT1  OT1  OA  
2 2 2
1 2
(A) sq. unit (B) sq. unit T2 T3 is the latus rectum which is given by
2 3
(C) 1 sq. unit (D) 2 sq. unit 1
4s 2 2
2
Solution:
Therefore, the area of $T1T2T3 is
(i) Let O be the centre of the square so that O is the
centre of both C1 and C2 (see Fig. 3.80). Take O as 1 1 1
HHHE HE HHHE HE HHHHE T2T3 – AT1  s 2 2 s 1
origin. Let OA  A and OB  B so that OC  2 2 2
HE HHHHE HE HHHE HE HE
A , OD  B and OA  A  2 and B  2.
HHHE HE HHHHE E HE T3
Let OP  B and OC  q so that B  1 and
E
q  2. Now A
B
( PA)2 ( PB)2 ( PC )2 ( PD)2
HE HE HE E HE HE HE HE T1
p T2
 (A p) (B q) (A p) (B p)2
O
S
 2(A 2 p2 ) 2(B 2 p2 )
90
°
 2(2 1) 2(2 1)  12
C D
Also
FIGURE 3.81
(QA)2 (QB)2 (QC)2 (QD)2
Answer: (C)
 2(2 2) 2(2 2)  16
Therefore 3. Passage: A circle C of radius 1 is inscribed in a equi-
lateral PQR. The points of control of C with the side
¤ ( PA)2

12 3
  0.75
PQ, QR and RP are D, E and F, respectively. The line
PQ is given by the equation 3 x y 6  0 and the
¤ (QA) 2 16 4
point D is (3 3 / 2, 3 / 2). Further, it is given that the
origin and the centre of C are on the same side of the
D C side PQ. Answer the following questions.
C1 (i) The equation of the circle C is
P C2
(A) (x 2 3 )2 (y 1)2  1
C
2
¥ 1´
(B) (x 2 3 )2 ¦ y µ  1
√2 § 2¶
T
Q
(C) (x 3 )2 (y 1)2  1
B
A 1 (D) (x 3 )2 (y 1)2  1
(ii) Points E and F are given by
FIGURE 3.80
Answer: (A) ¥ 3 3´
(A) ¦ , µ , ( 3 ,0)
§ 2 2¶
Worked-Out Problems 199

¥ 3 1´ is parallel to the x-axis (see Fig. 3.82). Also the dis-


(B) ¦ , µ , ( 3 ,0) tance of I from the side PR is 1 implies that PR is
§ 2 2¶ the x-axis and R is the origin. Now, the equation of
¥ 3 PR is y  0 and that of QR is y  3 x. Therefore
3´ ¥ 3 1´
(C) ¦ , , ,
§ 2 2 µ¶ ¦§ 2 2 µ¶ F  ( 3 , 0)
and
¥ 3 3 ´ ¥ 3 1´
(D) ¦ , µ,¦ , µ ¥ 3 3´
§ 2 2 ¶ § 2 2¶ E  ( 3 cos 60o , 3 sin 60o )  ¦ , µ
§ 2 2¶
(iii) Equation of the sides QR and RP are
Answer: (A)
2 2 (iii) See Fig. 3.83. R  (0, 0) and F  ( 3 ,0). This im-
(A) y  x 1, y  x 1
3 3 plies that the equation PR is y  0. Again R  (0, 0)
x ¥ 3 3´
(B) y ,y  0 and E  ¦ , µ . Therefore, the equation of QR is
3 § 2 2¶
y  3 x.
3 3
(C) y x 1, y  x 1 Answer: (D)
2 2
Q
(D) y  3 x, y  0
30°
Solution: 30°

(i) C is the incircle of $PQR (see Fig. 3.82) where its


E D 3√3 , 3
incentre is I and its radius is 1 (by hypothesis). In 1 2 2
an equilateral triangle, the altitudes are the angle I
bisector. The incentre I lies on the altitude DR
whose equation is
°
3 1 ¥ 3 3´ 30
°
y  ¦x 2 µ 30 60°
2 3§ ¶ R F P

x 3y FIGURE 3.82

Let I be (x1, y1) so that x1  3 y1 . Therefore,


4. Passage: Let ¤ be a family of circles passing through
2 2 the two points P(3, 7) and Q(6, 5). Answer the follow-
¥ 3 3´ ¥ 3´
¦ x1 2 µ ¦§ y1 2 µ¶  1 (& ID  1) ing questions.
§ ¶
2
(i) The number of circles belonging to ¤ and touch-
2
¥ 3 3´ ¥ 3´ ing the x-axis is
¦ 3 y y
¦§ 1 µ 1 (& x1  3 y1 )
2 µ¶
1
§ 2¶ (A) 1 (B) 2
Therefore (C) infinite (D) 0
(2 3 y1 3 3 )2 (2 y1 3)2  4 (ii) If each of the circles of 3 cuts the circle x2 y2
4x 6y 3  0, then all these chords pass through
 16 y12 48 y1 32  0 a fixed point whose coordinates are
 y12 3 y1 2  0 ¥ 23 ´ ¥ 23 ´
(A) ¦ 2 µ (B) ¦ 3 µ
 y1  1, 2 § 3¶ § 3¶
Hence, y1  2  x1  2 3 and y1  1  x1  3 . (C) (1, 23) (D) (23, 1)
Incentre I and the origin lie on the same side of PQ (iii) The centre of the circle belonging to ¤ and cut-
 I  ( 3 , 1 ). Therefore, equation of the circle C is ting orthogonally the circle x2 y2  29 is
( x 3 )2 ( y 1)2  1 ¥ 1 3´ ¥ 7 9´
(A) ¦ , µ (B) ¦ , µ
Answer: (D) § 2 2¶ § 2 2¶
(ii) Since PQ makes angle 120° with the positive direc- ¥ 7´ ¥ 65 14 ´
tion of the x-axis and QPR 60°, it is clear that PR (C) ¦ 3, µ (D) ¦ , µ
§ 9¶ § 18 3 ¶
200 Chapter 3 Circle

(i) Equation of PQ is 2x 3y 27  0. Also the circle This chord passes through the intersection of 5x
described on PQ is a diameter is S y x2 y2 9x 6y 56  0 and 2x 3y 27  0 which is (2, 23/3).
12y 53  0. Any circle passing through P and Q is Answer: (A)
of the form
(iii) The circle given in Eq. (3.112) cuts orthogonally the
x 2 y2 9 x 12 y 53 L (2 x 3 y 27)  0 circle
 x 2 y2 (9 2 L ) x (12 3L ) y 53 27 L  0 x 2 y2  9
(3.112) 2(2 L 9) 2(2 L 12)
This touches the x-axis. That is  (0) (0)
2 2
12 3L
2  53 27 L 29
 53 27 L  27 L  24
4
 9 L 2 72 L 144  212 108 L 8
L
9
 9 L 2 36 L 68  0
Therefore, the circle is
The discriminant of the quadratic is positive so that
it has two distant roots. Hence, there are two circles 8
x2 y2 9x 12y 53 (2 x 3 y 27)  0
belonging to ¤ which touch the x-axis. 9
Answer: (B) ¥ 16 ´ ¥ 24 ´
 x 2 y2 ¦ 9µ x ¦ 12µ y 53 24  0
§ 9 ¶ § 9 ¶
(ii) Let
65 28
Sa y S LL  x 2 y2 x y 29  0
9 3
 x 2 y2 9 x 12 y 53 L (2 x 3 y 27)  0 Therefore, the centre is
S aa y x 2 y2 4 x 6 y 3  0
¥ 65 14 ´
Common chord of Sa  0 and Sq  0 is Sa Sq  ¦§ , µ¶
18 3
5x 6y 50 K(2x 3y 27)  0.
Answer: (D)

Integer Answer Type Questions


1. If x2 y2 4x 4y k  0 is the equation of the locus Solution: The equation
of the point from which perpendicular tangents can
be drawn to the circle x2 y2 4x 4y  0, then the x 2y 12  0 (3.114)
value k is _____. is chord of contract of (h, k) with respect to the circle
Solution: The equation S y x2 y2 4x 2y 11  0. But the chord of contact is
xh yk 2( x h) ( y k ) 11  0
x2 y2 4x 4y  0 (3.113)
 (h 2) x (k 1) y (2 h k 11)  0 (3.115)
represents a circle with centre (2, 2) and radius 8 
2 2 . Then the locus of point from which perpendicular Equations (3.114) and (3.115) represent the same straight
tangents can be drawn to the circle given in Eq. (3.113) is line. Therefore
a concentric circle whose radius is 2 times the radius h 2 k 1 (2 h k 11)
of the circle [this circle is called the director circle of the   L (say)
1 2 12
circle given in Eq. (3.113)]. Hence, the radius of the direc-
tor circle is 2 (2 2)  4. Thus k  8. Hence
Answer: 8 h L 2
k  2L 1
2. If the equation of the chord of contact of (h, k)
Now
with respect to the circle x2 y2 4x 2y – 11  0 is
x 2y 12  0, then h k is equal to _____. 12 L  2(L 2) (1 2 L ) 11
Worked-Out Problems 201

so that $
 5
8 L  16  L  2 s
Therefore Therefore, [k]  2.
h L 2 4 Answer: 2
k  1 2L  5
6. A and B are two fixed points in a plane and k > 0,
so that Then the locus of P such that PA:PB  k:1 is a circle,
h k  9 provided k is not equal to _____.
Answer: 9 Solution: If k  1, then PA  PB so that the locus of P
is the perpendicular bisector of AB.
3. The line Kx y 1  0 cuts the coordinate axes at Answer: 1
points P and Q. The line x 2y 3  0 intersects the
coordinate axes at points R and S. If P, Q, R and S are 7. Two parallel chords of a circle of radius 2 are at a dis-
concyclic, then the value of K is _____. tance 3 1 apart. If the chords subtend at the cen-
Solution: Equation of the circle passing through points ter, angles π/k and 2π/k where k > 0, then [k] (which is
P, Q, R and S is of the form the integer part of k) is _____.
(IIT-JEE 2010)
(L x y 1)( x 2 y 3) M( xy)  0
Solution: Since the distance between the chords is
This equation represents a circle if the coefficient of x2 is
3 1 > 2, the chords must be on the opposite sides of
equal to the coefficient of y2 and coefficient of xy  0.
the origin (see Fig. 3.83). 3 1 is the distance between
Therefore,
PQ and RS which is given by
K  2 and M = 5
P P
Hence, the value of K  2. 2 cos 2 cos
k 2k
Answer: 2
This holds when
4. A rational point in an analytical plane means that
both the coordinates of the point are rational P P

numbers. Then the maximum number of rational k 3
points on a circle C with centre (0, 2 ) is _____.
so that k  3.
Solution: Suppose, there are three rational points on
circle C. Now consider that the triangle with those ratio-
nal point vertices whose circumcenter is (0, 2 ). Since
the vertices of this triangle are rational points, its equa-
tions of the perpendicular bisectors are first-degree equa-
tions in x and y with rational coefficients and hence the P Q
2p
circumcentre must be a rational point, but here it is not a k
rational point because (0, 2 ) is the circumcentre. Hence, O
maximum number of rational points on the circle is 2. p
k
Answer: 2 R S

5. Three circles of radii 3, 4 and 5 units touch each other


externally. The tangents drawn at the points of contact
are concurrent at point P. If k is the distance of P from
the points of contact, then [k] (which is the integer FIGURE 3.83
part of k) is _____.
Answer: 3
Solution: Let A, B and C be the centres of the circles
with radii 3, 4 and 5, respectively. Hence, the sides of the 8. The centres of two circles C1 and C2 each of unit radius
$ ABC are AB  7, BC  9 and CA  8. Therefore, s  are at a distance of 6 units from each other. Let P be
12 and $  12 s 3 s 4 s 5  12 5 k is the inradius of the midpoint of the line segment joining the centres
$ABC which is given by of C1 and C2 and C be a circle touching the circles C1
and C2 externally. If a common tangent to C1, which is
202 Chapter 3 Circle

passing through P, is also a common tangent to C1 and Therefore


C2, then the radius of C is _____.
[ $]  [2 3 ]  3
(IIT-JEE 2009)
Solution: We have y

MN  MP PN  32 12 32 12  4 2
P(1, √3)
However, MN  BC  OB2 OC 2 , where BC is drawn
2
parallel to MN meeting OA in C (see Fig. 3.84). Therefore

(R 1)2 (R 1)2 0 T x

where R is the radius of the circle. Hence


16 s 2  4 R
R8
FIGURE 3.85
C1 C2
N Answer: 3

1 10. Let x2 y2 4x 2y 11  0 be a circle. A pair of


P tangents from (4, 5) with a pair of radii form a quad-
A B rilateral of area ______.
1 Solution: See Fig. 3.86. Let A be (4, 5) and AP be a
M tangent given by
1
90°
R S11  4 2 52 4(4) 2(5) 1  2
C
C Area of the quadrilateral is given by
R
¥1 ´
O 2(Area of $APC )  2 ¦ s 2 s 2µ  4
§2 ¶

P
2

5)
2 4,
A(

FIGURE 3.84 C
2
(2,1)
Answer: 8 2

Q
9. If $ is the area of the triangle formed by the positive
x-axis and the normal and the tangent to the circle
x2 y2  4 at (1, 3 ), then ; $ = (which is integer part FIGURE 3.86
of $ is _____. Answer: 4
Solution: Tangent at (1, 3 ) is x(1) y( 3 )  4 (see
11. If the points of intersection of the line 4x 3y
Fig. 3.85). y  0  x  4. The tangent meets the x-axis at
10  0 and the circle x2 y2 2x 4y 20  0 are (a,
T(4, 0). Therefore
b) and (c, d) where a and b are positive and c and d
are negative, then a c b d is _____.
PT  (4 1)2 3  2 3
Solution: Substituting y  (4x – 10)/3 in the circle equa-
Hence tion, we get
1 1 2
$  Area of $OTP  s PT s OP  (2 3 ) s 2  2 3 ¥ 4 x 10 ´ (4 x 10)
x2 ¦ 2x 4 20  0
2 2 § 3 µ¶ 3
Worked-Out Problems 203

 9 x 2 16 x 2 80 x 100 18 x 48 x 120 180  0 Therefore


c4 (3.117)
 25 x 2 50 x 200  0
From Eqs. (3.116) and (3.117), we get
 x2 2x 8  0
 ( x 4)( x 2)  0 2g 4f  9
 x  4, 2 Therefore
Calculation 2( g) 4( f )  9
4 x 10 Hence, the locus of ( g, f ) is 2x 4y 9  0.Therefore,
x y
3 a  2, b  4 and c  9 so that
4 2 c (a b)  9 6  3
2 6 Answer: 3
Hence
14. If ax by c  0 is the equation of the common
(a, b)  (4, 2) and (c, d)  ( 2, 6) chord of the circles 3x2 3y2 2x 12y – 9  0 and
Therefore 3x2 3y2 2x 12y – 9  0 then b a c is equal to
__________
a c b d  4 2 2 6  2
Solution: The given circles are
Answer: 2
S y x2 y2 – (2/3)x 4y – 3  0
12. The line joining the points A(3, 4) and B(1, 0) cuts
and Sa y x2 y2 6x 2y 15  0
the circle x2 y2  4 at points P and Q. If AP/PQ  K
and AQ/QB  L, then the value of LM is ________. Therefore, the common chord is
Solution: The point dividing the segment AB in the ¥ 2 x ´
S Sa y ¦ 6 xµ (4 y 2 y) 3 15  0
ratio k:1 is § 3 ¶

¥k 3 4 ´  20 x 6 y 36  0
¦§ , µ
k 1 k 1¶  10 x 3 y 18  0
This lies on the circle Therefore

x2 y2  4 a  10, b  3 and c  18
 (k 3)2 16  4(k 1)2 and hence
 3k2 2k – 21  0 b a c  13 18  5
for which K and L are the roots. Therefore Answer: 5
21
LM  7 15. The lines 3x 4y 4  0 and 6x 8y 22  0 are
3 the tangents to the same circle whose diameters is
Answer: 7 _____.
Solution: Since the lines 3x 4y 4  0 and 6x
13. If ax by c  0 is the locus of the centre of the 8y 22  0 are parallel lines, the distance between them
circle which passes through the point (1, 2) and cuts is the diameter of the circle. Therefore the diameter of
orthogonally the circle x2 y2  4 then – c (a b) the circle is
value is _____.
4 11 15
Solution: Let S y x2 y2 2gx 2fy c  0 be the  3
2
3 4 2 5
required circle. S  0 passes through (1, 2). This implies
that Answer: 3
2g 4f c  5 (3.116)
16. The value of r such that the area of the triangle
S  0 cuts orthogonally x2 y2  4 implies that formed by the pair of tangents drawn to the circle
2g(0) 2f(0)  c – 4 x2 y2  r2 from the point P(6, 8) and its chord of
contact is maximum is ____. (IIT-JEE 2003)
204 Chapter 3 Circle

Solution: PA  PB which is equal to the length of the


 f  4 g (3.119)
tangent (see Fig. 3.87). This is given by
Therefore, from Eqs. (3.118) and (3.119), the radius is
6 2 8 2 r 2  100 r 2
In Problem 28 in the section ‘Subjective Problems g 2 f 2 c  g 2 ( 4 g )2 1
(Sections 3.1 till 3.3)’, it is worked out that the area of
 2 g 2 8 g 15
ΔPAB is equal to
 2[( g 2)2 4] 15
aS113 / 2
x12 y12  2( g 2)2 7

Therefore, the area of $PAB is given by Therefore, the radius is minimum if g  2. In such a case,
f  2. Hence
r(100 r 2 )3 / 2
(h, k)  Centre  ( g, f )  (2, 2)
62 82
Therefore, h k  4.
r(100 r 2 )3 /2
Let $ Answer: 4
100
d$ 1 ¨ 3 · 18. The number of integer values of k for which the
 (100 r 2 )3 /2 r(100 r 2 )1/ 2 (2r )¸
dr 100 ©ª 2 ¹ chord of the circle x2 y2  125 passing through the
point P(8, k) and is bisected at the point P(8, k) with
1
 [100 r 2 ]1/ 2 (100 r 2 3r 2 ) integer slope is ______.
100
Solution: Since P(8, k) is the midpoint of the chord,
d$
 0  100 4r 2  0 (& r x 10) the chord is perpendicular to OP. Slope of OP  k/8.
dr Therefore, the slope of the chord is 8/k which is an in-
r5 teger if k  p1, p2, p4 and p 8. If k  p8, then the point P
lies outside the circle so that it cannot be the midpoint of
Also dΔ/dr > 0 for r < 5 and is negative for r > 5. There- any chord. Therefore
fore, Δ is maximum when r  5.
k x p8
B k  p1, p 2, p 4
r
so that the integer values of k is 6.
O (0, 0)
Answer: 6

r 19. A light ray gets reflected from the line x  2. If the


reflected ray touches the circle x2 y2  4 at the point
P (6,8)
of incidence on line is ( 2, 4), then the equation of
A
the incident ray is 3x 4y k  0 where [k/7] (which
FIGURE 3.87 is the integer part of k/7) is _____.
Answer: 5

17. If (h, k) is the center of smallest circle cutting orthog- Solution: Let y mx 2 1 m2 be a tangent to the
x2
onally the circles  1 and y2 x2 y2 8x 8y – circle x2 y2  4 [see Note (1) of Theorem 3.8] (See Fig.
33  0, then h k is equal to ______. 3.88). This passes through ( 2, 4). This implies
Solution: Let S y x 2 y2 2 gx 2 fy c  0 be the (2 m 4)2  4(1 m2 )
required circle. Therefore, 2 g(0) 2 f (0)  c 1. This
 16 m  12
implies that
3
m
c1 (3.118) 4
Also Hence, the slope of the reflected ray is 3/4. Thus, the slope
2 g(4) 2 f (4)  c 33  1 33  32 of the incident ray is 3/4. Therefore, the equation of the
incident ray is 3x 4y 22  0.
 g f  4
Answer: 3
Summary 205

y 20. The distance of the common chord of the circles


S y x2 y2 5x 8y 1  0 and Sq y x2 y2 3x 7y –
25  0 from the centre of the circle x2 y2 2x  0 is
______.
Solution: Clearly, the circles S  0 and Sa  0 intersect
and hence the equation of the common chord is
–2 0 2 x
S S a y 8 x 15 y 26  0 (3.120)
Centre of the circle x2 y2 2x is (1, 0). Therefore, the
distance of (1, 0) from the line S Sa  0 [i.e. Eq. (3.120)]
is

(–2, –4) 8(1) 15(0) 26 34


 2
FIGURE 3.88
2
8 15 2 17

Answer: 2

SUMMARY
3.1 Theorem: Equation of the circle with centre at the (ii) g2 f 2 – c  0  the circle is a point circle.
point (x1, y1) and radius r > 0 is (iii) g2 f 2 – c < 0  the circle is imaginary.
(x – x1)2 (y – y1)2  r2 3.6 Universal equation of a circle: The equation ax2
ay2 2gx 2fy c  0 (a x 0) is called univer-
If x1  y1  0, then the equation of the circle with
sal equation of the circle with centre at the point
centre at the point origin with radius r is x2 y2  r2.
g2 f2 c
3.2 Theorem: Equation of the circle with (x1, y1) and ( g /a, f /a) and radius .
2 2
(x2, y2) as extremities of a diameter is a a a
3.7 The second-degree general equation ax2 2hxy
(x – x1)(x – x2) (y – y1)(y –y2)  0
by2 2gx 2fy c  0 represents a circle if and only
3.3 Parametric equations of the circle: Suppose P(x, y) if a  b, h  0 and g2 f 2 > ac.
is a point on the circle with A(x 1,Ey1) as its centre and
FHHH 3.8 Theorem: The perpendicular bisector of a chord of
r as radius. Suppose the line AP makes an angle P
a circle passes through the centre of the circle.
with positive direction of the x-axis. Then x  x1
r cos P and y  y1 r sin P are the coordinates of the 3.9 Relation between a circle and a line in the same
point P. Conversely, (x1 r cos P, y1 r sin P) lies on plane: Let C be a circle with centre at A and radius
the circle with centre at A(x1, y1) and radius r for all r. Let l be a line in the plane of the circle C. Then
real values of P. Hence, x  x1 r cos P and y  y1 (i) The circle C and the line l have no common
r sin P (P being parameter) are called the paramet- points š The perpendicular distance of the
ric equations of the circle (x – x1)2 (y – y1)2  r2. centre A from the line l is greater than the
In particular,the parametric equations of the circle x2 radius r.
y2  r2 are x  r cos P, y  r sin P. (ii) The line touches the circle š The perpen-
dicular distance drawn from A onto the line
3.4 Point circle: Circle having radius zero is called point
is equal to the radius r.
circle. That is, (x – x1)2 (y – y1)2  0 is the point
(iii) The line l intersects the circle in two distinct
circle with centre at (x1, y1).
points š The perpendicular distance of the
3.5 General equation of the circle: If g, f and c are real line from the centre is less than the radius r.
numbers, then the equation x2 y2 2gx 2fy c  0
3.10 Notation: S is denoted by x2 y2 2gx 2fy c.
represents a circle with centre at (–g, –f ) and radius
That is, S y x2 y2 2gx 2fy c  0 is a circle.
2 2
g f c with an understanding of the following Then
conditions: (i) S1 means that xx1 yy1 g(x x1) f(y y1) c
(i) g2 f 2 – c > 0  the circle is a real circle. That is, S1 y xx1 yy1 g(x x1) f(y y1) c
(ii) S2 y xx2 yy2 g(x x2) f(y y2) c
206 Chapter 3 Circle

(iii) S12  x1x2 y1y2 g(x1 x2) f(y1 y2) c  S21 ¥ a 2 m a 2 ´


(iv) S11  x12 y12 2 gx1 2 fy1 c ¦ c , cµ
§ ¶
3.11 Property of S1  0: If (x1, y1) is not the centre of 3.18 Director circle: The locus of the point through
the circle S  0, then which perpendicular tangents can be drawn to a
S1 y (g x1)x (f y1)y gx1 fy1 c  0 circle is again concentric circle whose radius equals
2 times the radius of the given circle and is called
is a first-degree equation in x and y and hence it the director circle of the given circle. The director
represents a straight line and is perpendicular to circle of the circle (x – h)2 (y – k)2  r 2 is (x – h)2
the line joining the centre of the circle and the (y – k)2  2r2.
point (x1, y1).
3.19 Equation of the chord in terms of its midpoint:
3.12 Theorem (Equation of a chord): If P(x1, y1) and Equation of the chord of a circle S  0 whose
Q(x2, y2) are two points on a circle S  0, then the midpoint is (x1, y1) is S1  S11.
equation of the chord joining points P and Q is
S1 S2  S12. In particular, equation of the tangent 3.20 Definition (chord of contact): Let C be a circle
at (x1, y1) is S1  0. and P be an external point to C. Suppose the two
tangents drawn from P to C touch the circle at A
Note: If S y x2 y2 – a2  0, then the equation of and B. Then the chord AB is called chord of contact
the chord joining P(x1, y1) and Q(x2, y2) is of P with respect to the circle C.
(xx1 yy1 – a2) (xx2 yy2 – a2)  x1x2 y1y2 – a2 3.21 Equation of the chord of contact: Equation of the
In particular, the equation of the chord joining chord of contact of a point (x1, y1) with respect to a
(a cos @, a sin @ ) and (a cos A, a cos A ) is circle S  0 is S1  0.

(A B ) (A B ) (A B ) 3.22 Length of the chord: Suppose PQ is a chord of a


x cos y sin  a cos circle with centre A and radius r. AM is the line
2 2 2
drawn perpendicular to the chord PQ so that M is
3.13 Equation of a tangent: the midpoint of PQ. Then the length of the chord
(i) Cartesian form: The equation of the tangent
PQ is 2 r 2 ( AM )2 .
to the circle x2 y2  a2 at the point (x1, y1) is
3.23 Length of the tangent: The length of the tangent
xx1 yy1 – a2  0
drawn from an external point (x1, y1) to a circle
(ii) Parametric form: Equation of the tangent to S  0 is S11 .
the circle x2 y2  a2 at the point (a cos @,
3.24 Relation between two circles: Let C1 and C2 be
a sin @) is
two circles with centres A1 and A2, respectively,
x cos @ y sin @  a and radii r1 and r2, respectively. Then
(i) C1 and C2 do not have a common point and
3.14 Position of a point with respect a circle: Let C be a
neither is being completely within the other
circle with centre at point A and radius r. Let P be
š A1 A2 > r1 r2.
a point in the plane of the circle, then
(ii) C1 and C2 touch each other externally
(i) P lies outside the circle if and only if AP > r.
š A1 A2  r1 r2.
(ii) P lies inside the circle if and only if AP < r.
(iii) C1 and C2 intersect in two distinct points
(iii) P lies on the circle if and only if AP  r.
š r1 r2 < A1 A2 < r1 r2.
3.15 Condition for a point to lie outside or inside on the (iv) C1 and C2 touch each other internally
circle. Let S  0 be a circle and P(x1, y1) a point š A1 A2  r1 r2 .
in the plane of the circle, then (v) One lies completely within the other without
(i) P(x1, y1) lies outside the circle š S11 > 0. touching š A1 A2 < r1 r2 .
(ii) P(x1, y1) lies inside the circle š S11 < 0.
3.25 Angle of intersection of circles: Let C1 and C2 be two
(iii) P(x1, y1) lies on the circle š S11  0.
circles intersecting at points A and B. Then the angle
3.16 Theorem: From any external point of a circle, two between the tangents drawn to the circles at either of
tangents can be drawn to the circle. the points A and B is same and this angle is called the
angle of intersection of the two circles C1 and C2.
3.17 Condition for line to be tangent: The line y  mx c,
where c x 0, touches the circle x2 y2  a2 if and 3.26 Definition (Orthogonal circles): If the angle of
only if c2  a2(1 m2) and in such a case, the point intersection of two circles is a right angle, then
of contact is the two circles are said to intersect each other
orthogonally.
Summary 207

3.27 Angle of intersection formula: Let C1 and C2 be (ii) If the two circles touch each other, then their
two circles with centres A1 and A2, respectively, radical axis S – Sa  0 is a common tangent to
and radii r1 and r2, respectively. If @ is the angle of the circles at their point of contact.
intersection of the circles, then
3.36 Theorem: Let C1 and C2 be two non-concentric
( A A )2 ( A1 P )2 ( A2 P )2 circles. If a circle C cuts both C1 and C2 orthogonally,
cos A  1 2 then the centre of C lies on the radical axis of C1
2( A1 P )( A2 P )
and C2.
where P is one of the points of intersection of C1
3.37 Radical centre: If S  0, Sa  0 and Sq  0 are three
and C2.
circles with non-collinear centres, then their radical
3.28 Theorem (Two circles intersecting orthogonally): axes taken two by two are concurrent and this
Two circles x2 y2 2gx 2fy c  0 and point of concurrence is called the radical centre of
x 2 y2 2 g ax 2 f ay c a  0 cut each other the three circles.
orthogonally if and only if 2 gg a 2 ff a  c c a.
Note: (1) The tangents drawn from the radical
3.29 Suppose S  0 and Sa  0 are two non-concentric centre to each of the three circles are of equal
circles in the standard form (i.e., general form). length. (2) Taking the radical centre as the centre
Then, the equation S S a  0 is a first-degree and the length of the tangent drawn from the radi-
equation in x and y. Hence it represents a line cal centre to the circle as radius, if a circle is drawn,
which is also perpendicular to the line joining the then this circle cuts all the circles orthogonally.
centres of the circles.
3.38 Common tangents to two circles: Let C1 and C2
3.30 About the line S – Sa  0: Let S  0 and S a  0 be be two circles with centres A1 and A2, respectively,
two circles with different centres in the standard and radii r1 and r2, respectively. The following table
form. Then shows the number of common tangent(s) to the
(i) S Sa  0 represents the common chord of the two circles:
circles if the circles intersect.
(ii) If the circles touch each other at a point P, Nature of the Num- Number Number of
then S – Sa  0 represents the common tan- circles ber of of direct transverse
gent to the circles at P. common common common
3.31 Definition (Power of a point): Let C be a circle tangents tangents tangents
and P be a point in the plane of C. Suppose a line Circles do not 4 2 2
through P meets the circle C at two points A and have common
B. Then the value PA·PB is independent of the line points
through P. The power of P with respect to the circle (A1 A2 > r1
C is defined as follows: r 2)
(i) The value PA·PB if P is an external point to C.
Circles touch 3 2 1
(ii) The value –(PA·PB) if P lies inside C.
externally
(iii) The value zero if P lies on the circle.
(A1 A2  r1 r2)
3.32 Power of a point formula: The power of P(x1, y1) Circles 2 2 0
with respect to a circle S  0 is S11. intersect in
3.33 Theorem (radical axis): The locus of the two district
point whose powers with respect to two non- points
concentric circles are equal is a straight line and is ( r1 r2 <
perpendicular to the line joining the centres of the A1 A2 < r1 r2)
circles. This line is called the radical axis of the two Circles touch 1 — —
circles. internally
(A1 A2 
3.34 Equation of the radical axis: If S  0 and Sa  0 are
r1 r2 )
two non-concentric circles, then their radical axis
equation is S Sa  0 One circle 0 0 0
completely
3.35 About the radical axis:
within the
(i) If the circles S  0 and Sa  0 intersect, then
other (A1 A2
their radical axis S – Sa  0 is also a common
< r1 r2 )
chord.
208 Chapter 3 Circle

3.39 Centres of similitude: direct common tangents. Perform same with


(i) Suppose that the two circles C1 and C2 (dis- regards to transverse common tangents by finding
cussed in 3.38) do not intersect and the radius the internal centre of similitude.
r1 is not equal to r2. Then the two direct com-
3.42 Second method to find the common tangents:
mon tangents intersect on the line of centres
and this point is called external centre of si- Suppose the circles are (x – x1)2 (y – y1)2  r12
militude. This external centre of similitude and (x – x2)2 (y – y2)2  r22 . We know that the line
divides the segment A1 A2 in the ratio r1:r2 (x – x1)cosP (y – y1)sinP  r1 is a tangent to the
externally.
first circle. Now, impose the condition that this line
(ii) The two transverse common tangents also
also touches the second circle and find the values
intersect the line of centres in between the
of sinP and cosP.
centres. This point is called the internal cen-
tre of similitude and this point divides A1 A2 Note: In this second method, we get the common
internally in the ratio r1:r2. targets, but we do not know their nature unless we
(iii) If the circles touch each other externally, then use further test.
the internal centre of similitude is their point
3.43 S KL  0: Let S y x2 y2  2gn 2fy c  0 be a
of contact and the external centre of simili-
circle and L y ax by c  0 a straight line. Then,
tude exits as usual.
the equation S KL  0 represents a family of
(iv) If the two circles intersect, there is no internal
circles for all real values of K.
centre of similitude, but external centre of si-
militude exits. Note:
(i) If the line L  0 intersects the circle S  0 at
3.40 Theorem: Combined equation of the pair of
two points P and Q, then S KL  0 repre-
tangents drawn from an external point (x1, y1) to a
sents circles, all passing through points P and
circle S  0 is S12  S11.
Q.
3.41 Finding the common tangents to two circles: (ii) If L  0 touches the circle S  0 at the point P,
Step1: Determine the nature of the circles. then S KL  0 represents circles, all touching
Step2: If external centre of similitude exists, then at point P. In such cases, L  0 is a common
find its coordinates [using 3.39(i)]. tangent to all these circles at point P.
Step3: Consider a line through the external centre (iii) If S  0 and S  0 are two non-concentric cir-
of similitude and impose the condition that it cles, then S KL  0, where S – S  0 repre-
touches one of the circles. Then that line automati- sents a family of circles.
cally touches the other circle also. Thus, we get

EXERCISES
1. Find the equation of the circumcircle of the triangle 6. Show that x2 y2 10x 6y 9  0 touches y-axis.
whose vertices are (0, 0), (a, 0) and (0, b).
7. Find the equation of the circle inscribed in the triangle
2. Find the equation of the circumcircle of the quadrilat- whose sides are x-axis, y-axis and the line x y  1.
eral whose sides are 5x 3y 9  0, y  x/3, 2x  y and
8. The line x cos @ y sin @ – Q  0 cuts the circle
x 4y  2  0 in the given order.
x2 y2  a2 in A and B. Find the equation of the circle
3. If x1 and x2 are the roots of x2 b2
2ax  0 and y1 and described on AB as diameter.
y2 are the roots of x2 2px q2  0, then find the equa-
9. Show that the condition that the circle circumscrib-
tion of the circle described on the segment joining the
ing the triangle formed by the three lines a1x b1y
points A(x1, y1) and B(x2, y2) as diameter.
c1  0, a2 x b2 y c2  0 and a3 x b3 y c3  0 should
4. If (h, k) is the centre of the circle whose parametric have its centre lying on the x-axis so that
equations are x  1 2 cos P and y  3 2 sin P, then
find (h, k). ¥ a2 a3 b2 b3 a3 a1 b3 b1 a1a2 b1b2 ´
¦a b a b a1b3 b3 b1 a2 b1 a1b2 µ  0
5. Find the length of the intercept made by the circle ¦ 3 2 2 3
µ
x2 y2 10x 6y 9  0 on x-axis. § b3 c2 b2 c3 b1c3 b3 c1 b2 c1 b1c2 ¶
Exercises 209

10. Prove that the circles x2 y2 2gx 2fy c  0 and 25. The straight line x 2y 1  0 intersects the circle
x2 y2 2g ax 2f ay c a  0 should touch if x2 y2  25 at points A and B. Find the coordinates
of the point of intersection of the tangents drawn to
(2 gg a 2 ff a c c a)2  4( g 2 f 2 c)( g a 2 f a 2 c a)
the circle at points A and B.
11. Find the centre and radius of the circle which is 26. Find the area of the triangle formed by the chord of
inscribed in the triangle formed by the lines y  0, contact of the point P(4, 3) with respect to the circle
12x 5y  0 and 3x 4y 7 0. x2 y2  9 and the two tangents drawn from P(4, 3)
to the circle.
12. Find the equation of the circle passing through the
points (0, a) and (b, h) and having its centre on the [Hint: use the formula that the area is equal to
axis of x. 3/2)/(x 2 y 2).]
(rS11 1 1

13. Show that the equation of the circle passing through 27. A circle passes through origin and has its centre on
the points (a, b), (a, b) and (a b, a b) is b(x2 the line y  x. If it cuts x2 y2 4x 6y 10  0 or-
y2) (a2 b2)x (a b)(a2 b2)  0. thogonally, then find its equation.
14. Show that the equation of the circle passing through 28. Let C be the circle x2 y2 2x 4y 2 3 1  0. Let
origin and having intercepts of lengths 3 and 4 on Ca be a variable circle touching internally C and the
positive x-and y-axis, respectively, is x2 y2 3x tangents drawn from the point (1, 2) to Ca include an
4y  0. angle of 60o. Show that the center of Ca lies on the
circle (x 1)2 (y 2)2  3.
15. For all values of K and L, prove that the circles x2
y2 2Kx c  0 and x2 y2 2Ly – c  0 cut each other 29. A variable circle passes through the point A(p, q)
orthogonally. and touches the x-axis. Show that the other end of
the diameter through A lies on the curve (x p)2 
16. For two distinct values of K if the circle x2 y2
4qy.
2Kx c  0 cuts the circle x2 y2 2gx 2fy ca  0
orthogonally, then show that g  0 and ca  c. 30. If the tangents drawn from the origin to the circle
x2 y2 – 2ax – 2by 2  0 are at right angles, then
17. Prove that the radical axis of two circles bisects their
show that a2 b2  4.
common tangent.
31. Let 1 2r, 1 r and 1 be the radii of three concentric
18. If c2 > 1/2(a b)2, then show that the length of the
circles with centres at origin (r > 0). If the line y  x
common chord of the circles and (x a)2 (y b)2  c2
1 cuts the circle x2 y2  (1 2r)2 in two distinct
is 4c 2 2(a b)2 and (x b)2 + (y a)2  c2. points, then show that 0 < r < 2 1/2 2.
19. Find the equations to the circles which intersect the 32. Consider a family of circles passing through two fixed
circles x2 y2 6y 1  0 and x2 y2 4y 1  0 or- points A(3, 7) and B(6, 5). Show that the chords in
thogonally and touch the line 3x 4y 5  0. which the circle x2 y2 4x – 6y 3  0 cuts the mem-
20. Find the equation to the circle which passes through bers of the family are concurrent at a point. Find the
the origin and cuts orthogonally each of the circles coordinates of this point.
x2 y2 6x 8  0 and x2 y2 2x 2y – 7  0. (Hint: Equation of the given family by of circles S
21. Find the equation to the circle cutting orthogonally KL  0 where L  0 is the equation of AB and S  0 in
the three circles x2 y2 2x 17y 4  0, x2 y2 the equation of the circle described on AB as diam-
7x 6y 11  0 and x2 y2 x 22y 3  0. eter.)

22. If the lines a1x b1y c1  0 and a2 x b2 y c2  0 33. The equation (x 5y 22) K( x 8y 30)  0
cut the coordinate axes in concyclic points, then show (K is a parameter) represents a family of lines. Find
that a12a22  b12 b22. the lines belonging to the family on which the cir-
cle x2 y2 2x 2y – 14  0 cuts chords of length
23. If the line y  mx 2 cuts the circle x2 y2  1 in two 2 3.
distinct points, then show that either m < 3 or
m > 3. 34. Find the centres of the circles passing through the
points (0, 0) and (1, 0) and touching the circle x2
24. Suppose K1, K2 and K3 are positive real numbers that y2  9.
are in GP. If t1, t2 and t3 are the tangents drawn from
any point on the circle x2 y2  a2 to the circles, 35. A circle of radius 2 units rolls on the outside of the
x2 y2 2Kix a2  0 (i  1, 2, 3), then prove that t1, circle x2 y2 4x  0, touching it externally. Find the
t2 and t3 are in GP. locus of the centre of this outer circle.
210 Chapter 3 Circle

1. x2 y2 – ax – by  0 20. 3x2 3y2 8x – 29y  0

2. 9x2 9y2 20x 15y  0 21. x2 y2 4x 6y – 44  0

3. x2 y2 2ax 2py b2 q2  0 25. ( 25, 50)

4. ( 1, 3) 192
26. sq. unit
5. 8 25
2 2 2 27. x2 y2 – x – y  0
¥ 1 ´ ¥ 1 ´ ¥ 1 ´
7. ¦ x 1 µ ¦ y 1 µ  ¦§ 1 µ ¥ 23 ´
§ 2¶ § 2¶ 2¶ 32. ¦ 2, µ
§ 3¶
8. x2 y2 a2 2p(x cos @ y sin @ p)  0
33. 2x 3y 8  0 and 3x 2y 14  0
¥ 7 14 ´ 14
11. Centre is ¦ , µ and radius is ¥1 ´
§ 9 27 ¶ 27 34. ¦ , p 2 µ
§ 2 ¶
12. b(x2 y2 a2)  x(h2 b2 a2)
35. x2 y2 4x – 12  0
19. x2 y2  1 and 4(x2 y2) 15x – 4  0
Parabola 4
Contents
4.1 Conic Section
4.2 Parabola

Worked-Out Problems
Summary
Exercises
Answers

y
Axis of symmetry

The locus of the point which moves


in such a way that it is equidistant
from a fixed point and a fixed line is
known as a parabola.

Focus P
(h, k)
Vertex

P Directrix
Parabola
212 Chapter 4 Parabola

In the previous chapter, we discussed about circle, general equation of a circle, equation of the tangent at a point,
chord equation in terms of its midpoint, chord of contact, orthogonal circles, etc. In this chapter, we will discuss
broadly about one of the conics, namely, parabola. The chapter discusses about cone, base curve or guiding curve, cir-
cular cone and right circular cone, general conic, classification of conics, double ordinate and latus rectum, parametric
equations of y2  4ax, normal, normal chord, chord of contact, diameter of a parabola, focal chord and focal radius and
conormal points. ‘Subjective Problems’ section provides subjective worked-out problems for the preceding sections.
Students are advised to solve each and every problem to grasp the topics.

4.1 Conic Section


Before we discuss the properties of the conics, namely, parabola, ellipse and hyperbola, let us understand the term
conic or conic section. Conic sections are the curves obtained by projecting a circle through a fixed point V not in
the plane of the circle onto another plane. In fact, conics are sections of a cone by a plane cutting the cone in various
ways (of course, the base of the cone is a circle). In this context, we used the word cone and base of the cone. While
studying the elementary geometry and elementary solid geometry, a student visualises the shape of the cone as
given in his or her respective textbooks. In this section, we introduce the concept of a cone in a more general way.

DEFINITION 4.1 Cone Let S be a non-empty set of Fpoints


HHE in the space. Then, S is called a cone if there ex-
ists a point V S such that the line VP is Fcontained
HHE in S for all points P in S. This point V is
called the vertex of the cone and the line VP where P S is called generator of the cone S.

Examples

(1) Every line is a cone with every point on the line as (3) Two intersecting planes form a cone with every point
vertex and the line is the only generator. on their line of intersection as vertex.
(2) Every plane is a cone with all of its points as vertices.

DEFINITION 4.2 Degenerate and Non-degenerate Cones The cones described in the examples of Definition 4.1
are called degenerate cones. Generally, cones that are having more than one vertex are called
degenerate cones. Cones which do not degenerate are called non-degenerate cones. Using the
three-dimensional analytic geometry (Chapter 6), we can verify that the locus represented by the
equation x 2 y2 z2  0 is a cone with origin as the vertex.

DEFINITION 4.3 Base Curve or Guiding Curve If a plane is not passing through the vertex and intersects all
the generators of a cone, then the intersection of the plane and the cone are called base curve
or guiding curve.

DEFINITION 4.4 Circular Cone and Right Circular Cone If the base curve is a circle, then it is called a circular
cone (see Fig. 4.1). If the base curve is a circle and the line connecting the centre of the base
and the vertex of the cone is perpendicular to the plane of the circle, then the cone is called
right circular cone.
V

V q

Circular cone Right circular cone


O

(a) (b)

FIGURE 4.1
4.2 Parabola 213

Note: In a right circular cone, the line passing through the vertex and the centre of the base circle is called the axis of
the cone and it makes a constant angle with each of the generators and this constant angle is called semi-vertical angle
of the cone [P in Fig. 4.1(b)]. In a right circular cone, a plane which is not passing through the vertex and perpendicu-
lar to the axis cuts the cone in a circle. Definition 4.5 explains the geometrical concept of a parabola.

DEFINITION 4.5 Let V be the vertex of a right circular cone with semi-vertical angle P whose axis is l. Suppose
a plane which is not passing through the vertex makes an angle P with l. Then, the cone sec-
tion is a plane curve called parabola (see Fig. 4.2).

FIGURE 4.2

Now, let us define a general conic using two-dimensional geometry.

DEFINITION 4.6 General Conic Let l be a fixed line, S be a fixed point which does not lie on l and e be a
positive real number. Then, the locus of the point P, such that the distance SP is equal to
e times the perpendicular distance of l from P, is called a conic. For this conic, the fixed
line l is called directrix, the fixed point S is called focus and the number e is called the
eccentricity.

DEFINITION 4.7 Classification of Conics Conics are classified according to their eccentricity e.
1. If e  1, then the conic is called parabola.
2. If 0  e  1, then the conic is called ellipse.
3. If e 1, then the conic is called hyperbola.
In this chapter, we will discuss about parabola.

4.2 Parabola

T H E O R E M 4.1 Standard equation of a parabola is y2  4ax where a  0.


PROOF Let the line l be the directrix and point S be the focus. From point S, draw SZ perpendicular HHHE to
the directrix l and let
HHHEO be the midpoint of
HHHE SZ. Let ZO  OS  a (see Fig. 4.3). Consider Ox as
positive x-axis and Oy perpendicular to Ox as y-axis so that Z  (−a, 0) and S  (a, 0).
P(x, y) is a point on the parabola
š SP  PM where PM is the perpendicular distance of the directrix l (x a  0) from P
2
š ( x a) 2 y 2  x a

š y2  4ax
214 Chapter 4 Parabola

l y
P
M
L

Z O S (a, 0) x

L

FIGURE 4.3

QUICK LOOK 1

1. The focus and the directrix of y2  4ax are (a, 0) 3. Other forms of parabola:
and x a  0, respectively, and O(0, 0) is called the a. If the focus is to the left of the directrix, then its
vertex of the parabola. equation is y2  4ax.
2. The curve y2  4ax is symmetrix about the x-axis b. If the focus is on the y-axis, then the equation of
(also called the axis of the parabola). the parabola is x 2  p4ay.

T H E O R E M 4.2 Equation of the parabola with the vertex at (h, k), the axis parallel to the x-axis, the focus at a
distance a to the right of the vertex and the directrix parallel to the y-axis which is at a distance of
2a to the left focus is ( y k )2  4a( x h) (see Fig. 4.4).
PROOF Since the directrix equation is x  h − a and the focus is (h a, k), we have
SP  PM
2
š ( x h a)2 ( y k )2  x h a
š ( y k )2  4a( x h)

y l
M P
L

Z (h, k) S (a  h, k)

L
O x

FIGURE 4.4

QUICK LOOK 2

Other forms of the equations of parabola are as fol- 2. ( x h)2  4a( y k )


lows:
3. ( x h)2  4a( y k )
1. ( y k )2  4a( x h)
4.2 Parabola 215

DEFINITION 4.8 Double Ordinate and Latus Rectum Suppose a line perpendicular to the axis of parabola
meets the curve at P and Q. Then, PQ is called double ordinate of the curve. In particular,
double ordinate passing through the focus is called latus rectum. Usually, the ends of the latus
rectum are denoted by L and La (see Fig. 4.5).

T H E O R E M 4.3 The length of the latus rectum of the parabola y2  4ax is 4a.

PROOF Let L and La be the ends of the latus rectum. Since S  (a, 0), let L be (a, y).

L(a, y) lies on the curve  y2  4a(a)


 y  p 2a
Therefore, L  (2a, 0) and La  (−2a, 0) so that LLa  4a.
y
P

L Double ordinate

O S x

Latus rectum

L
Q

FIGURE 4.5

QUICK LOOK 3 (SOME PROPERTIES OF y2  4ax)

1. For any point (x, y) on the curve, x ≥ 0 and 3. The point on the curve where the axis meets the
y  p 4ax . That is, to each value of x ≥ 0, there are curve is called the vertex.
two corresponding values for y. In this case we say 4. For y2  4ax, (0, 0) is the vertex and y-axis is the
that the curve is symmetric about x-axis. tangent at the vertex.
2. x m d  y m ± d.

DEFINITION 4.9 Parametric Equations of y2  4ax For all real values t, the point (at2, 2at) lies on the parabola
y2  4ax. Conversely, if (x1, y1) is a point on the curve y2  4ax, then take t  y1 / 2a so that
x1  at2. That is, if (x1, y1) is a point on the curve y2  4ax, there corresponds a real t such that
x1  at2, y1  2at. Hence, the equations x  at2, y  2at, t  R are called the parametric equations
of the parabola y2  4ax.
Notation: S denotes the expression y2 − 4ax. That is,
S y y2 − 4ax
S1 y yy1 − 2a (x x1)
S2 y yy2 − 2a (x x2)
S12  S21  y1y2 − 2a (x1 x2)
S11  y12 4ax1

4.2.1 Classification of Points


Let P(x, y) be a point in the plane of the parabola. If x  0, then P is called external point to the curve. Suppose x  0.
Then, draw PM perpendicular to the axis (i.e., x-axis) meeting the curve at point Q. Point P is called external or inter-
nal point to the curve according as PM  QM or PM  QM (see Fig. 4.6).
216 Chapter 4 Parabola

y P
Q

Q
P

O M M x

FIGURE 4.6

T H E O R E M 4.4 Let P (x1, y1) be a point in the plane of the parabola S y y2 − 4ax  0. Then
1. P lies outside the parabola š S11 0.
2. P lies inside the parabola š S11 0.
PROOF Draw PM perpendicular to the axis of the parabola meeting the curve at Q (x1, y2) (see Fig. 4.6).
Point P(x1, y1) lies outside the parabola
š PM  QM
š y1  y2

š y12  y22  4ax1 (& Q lies on the curve)

š y12 4ax1  0
š S11  0
In the above argument, when the point lies inside the curve, change ‘greater than’ symbol to ‘less
than’ symbol.

Example 4.1

Find the equation of the parabola whose focus is (1, − 1) š (SP )2  ( PM )2


and directive is the line x y 7  0.
2
2 2 x y 7
š ( x 1) ( y 1) 
Solution: We have 2
P(x, y) is a point on the curve š 2 x 2 2 y2 4 x 4 y 4  x 2 y2 2xxy 14 x 14 y 1 9
šSP  PM
š x 2 2 xy y2 18 x 10 y 45  0
where S  (1, − 1) and PM is the distance of P from the
directrix.

Note: In general, when the focus, directrix and the eccentricity of the conic are given, its equation is a second-degree
general equation in x and y. The converse can be proved under certain conditions. Discussion of second-degree general
equation is out of scope of this book and generally it is not taught in Class 12 level.

Example 4.2

Determine the coordinates of the focus, the equation of the Solution: The equation of the curve is
directrix and the length of the latus rectum of the parabola
¥ 8´ ¥ 2´
y2  ¦ µ x  4 ¦ µ x
2 ¥ 8´ § 3¶ § 3¶
y  ¦ µ x.
§ 3¶
4.2 Parabola 217

so that a  2 / 3. Hence, S  (a, 0)  (2 / 3, 0) is the focus, is the equation of the directrix and 4a  8 / 3 is the length
of the latus rectum.
2
x  0 or 3x 2  0
3

Example 4.3

Find the equation of the parabola with its vertex at (3, 2) and the focus is 2 so that a  2. Hence, the equation of
and focus at (5, 2). the parabola is

Solution: The points (3, 2) and (5, 2) lie on the hori- ( y k )2  4a( x h)
zontal line y  2. Therefore, the axis of the parabola is  ( y 2)2  4(2)( x 3)
the horizontal line y − 2  0. Distance between the vertex
 ( y 2)2  8( x 3)

Example 4.4

Show that y  ax2 bx c, a ≠ 0 represents a parabola Therefore, the vertex (X  0, Y  0) is given by


and find its vertex, focus, directrix and latus rectum.
¥ b 4ac b2 ´
¦ 2a , 4a µ
Solution: We have § ¶

y  ax 2 bx c The focus ( X  0, Y  1/ 4a) is given by


y b c ¥ b 4ac b2 1 ´ ¥ b 1 4ac b2 ´
  x2 x , y   ,
a a a ¦ 2a 4a 4a µ¶ ¦§ 2a 4a 4a µ¶
§
2
y ¥ b´ 4ac b2
  ¦x µ The directrix equation is given by
a § 2a ¶ 4a 2
2 1 4ac b2 1
¥ b´ 1¥ 4ac b2 ´ Y or y 
 ¦x µ  ¦ y 4a 4a 4a
§ 2a ¶ a§ 4a µ¶
The latus rectum is given by
1 2
¥ b 4ac b2 ´
X  Y ¦ where X  x and Y  y ¥ 1´ 1
a § 2a 4a µ¶ 4¦ µ 
§ 4a ¶ a

Example 4.5
Find the vertex, focus, directrix and latus rectum of the Therefore, the vertex is given by
parabola y2 4 x 2 y 3  0.
¥ 1 ´
( X  0, Y  0)  ¦ , 1µ
Solution: The given equation is written as § 2 ¶

¥ 1´ Since a  1, the focus (X  −1, Y  0) is


( y 1)2  4 ¦ x µ
§ 2¶ ¥ 3 ´
¦§ , 1µ¶
Let 2
1 The directrix is given by
X  x and Y  y − 1
2 1 1
Then we have X  a or x  1 or x  0
2 2
Y 2  4X
The latus rectum is given by 4a  4.
218 Chapter 4 Parabola

Example 4.6

Find the vertex, focus, directrix and latus rectum of the 1. The vertex (X  0, Y  0) is (2, −1)
parabola x2 4x 5y 1  0. 5´ ¥ 1´
¥
2. The focus ( X  0, Y  a) is ¦ x  2, y 1  µ  ¦ 2, µ
§ 4¶ § 4¶
Solution: The given equation is written as
3. The directrix equation is
¥ 5´
( x 2)2  5( y 1)  4 ¦ µ ( y 1)
§ 4¶ 5
Y  − a or y 1  or 4y 9  0
4
Let X  x − 2 and Y  y 1 so that
¥ 5´
X 2  4aY 4. The latus rectum is 4a  4 ¦ µ  5.
§ 4¶
where a  5/4. Therefore

T H E O R E M 4.5 The equation of the chord joining two points P(x1, y1) and Q(x2, y2) on the parabola S y y2 − 4ax  0
(E Q U A T I O N O F is S1 S2  S12 and in particular, equation of the tangent at (x1, y1) is S1 y yy1 − 2a (x x1)  0.
THE CHORD)

PROOF Since P(x1, y1) and Q(x2, y2) are the points on the curve, we have y12  4ax1 and y22  4ax2 . There-
fore

y12 y22  4a( x1 x2 )


4a y y2
  1  Slope of the chord PQ
y1 y2 x1 x2

Hence, the equation of the chord PQ is


4a
y y1  ( x x1 )
y1 y2
 (y y1) (y1 y2)  4a(x x1)
 yy1 yy2 y12 y1 y2  4ax 4ax1
 yy1 yy2 4ax  y1 y2 (& y12  4ax1 )
 ; yy1 2a( x x1 )= [ yy2 2a ( x x2 )]  y1 y2 2ax1 2ax2
S1 S2  S12
Now, as the point Q approaches point P along the curve, in the limiting case, the position of the
chord PQ takes the position of the tangent at P so that the equation of the tangent at point P is
S1 S1  S11  0 [& ( x1 , y1 ) lies on the curve]
Thus, S1 y yy1 − 2a (x x1)  0 is the equation of the tangent at (x1, y1).

QUICK LOOK 4

In the equation S1 y yy1 − 2a (x x1)  0, if we replace tangent at (at2, 2at) is ty  x at2. Also note that the
x1 with at2 and y1 with 2at, then the equation of the slope of the tangent at (at2, 2at) is 1/t.

T H E O R E M 4.6 The point of intersection of the tangents to the parabola y2  4ax at the points (at12 , 2at1 ) and
(I N T E R S E C T I O N (at22 , 2at2 ) is [at1t2, a(t1 t2)].
OF THE
TANGENTS AT
t 1 A N D t 2)
4.2 Parabola 219

PROOF By Quick Look 4, the tangents at t1 and t2, respectively, are given by

t1 y  x at12 (4.1)

and t2 y  x at22 (4.2)


Solving Eqs. (4.1) and (4.2), we get x  at1t2 and y  a(t1 t2).

QUICK LOOK 5

If the tangents are at right angles, the product of their so that their point of intersection lies on the directrix
slopes is equal to 1. This implies x a0
1 1
s  1  t1t2  1
t1 t2

T H E O R E M 4.7 Condition for the line y  mx c to touch the parabola is


a
c
m
and in such case, the point of contact is
¥ a 2a ´
¦§ 2 , µ¶
m m
PROOF 1 Suppose the line y  mx c touches the parabola S y y2 − 4ax  0 at (x1, y1). By Theorem 4.5, the
equation of the tangent at point (x1, y1) is S1 y yy1 − 2a (x x1)  0. That is, the equations y  mx c
and S1 y yy1 − 2a (x x1)  0 represent the same straight line. Hence,
2a y1 2ax1
 
m 1 c
Therefore
c 2a
x1  and y1 
m m
Now, (x1, y1) lies on the parabola y2 4ax. So we have

y12  4ax1
4a 2 ¥ c´
  4a ¦ µ
m2 § m¶
a
 c
m
This gives
a 2a
x1  2
, y1 
m m
That is, if the line y  mx c touches the parabola y2  4ax, then
a
c
m
and the point of contact is
¥ a 2a ´
¦§ 2 , µ¶
m m
220 Chapter 4 Parabola

Conversely, suppose
a
c
m
then the point of intersection of the line
a
y  mx and y2  4ax
m
given by the equations
2 2
¥ a´ ¥ a´
¦§ mx µ¶  4ax and ¦§ mx µ¶  0
m m
a
x
m2
is a double root. Hence, the line
a
y  mx
m
intersects the parabola y2  4ax at the coincident point
¥ a 2a ´
¦§ 2 , µ¶
m m

a
Thus, y  mx touches the parabola.
m

PROOF 2 Line y  mx c touches the parabola y2  4ax


š The quadratic equation (mx c)2 4ax  0 has equal roots
š m2 x 2 2(cm 2a) x c 2  0 has equal roots
š 4(cm 2a)2 4c 2 m2  0
š 4cam 4a 2  0
a
šc
m

Note: Difference between Proof 1 and Proof 2: In Proof 1, we obtain both the condition and the coordinates of the
point of contact, but in Proof 2 we obtain only the condition
a
c
m

QUICK LOOK 6

For all real values of m ≠ 0, the line touches the parabola y2  4ax at the point
a ¥ a 2a ´
y  mx ¦§ 2 , µ¶
m m m
4.2 Parabola 221

T H E O R E M 4.8 From any external point in the plane of a parabola, two tangents can be drawn to the parabola.
PROOF Let S y y2 − 4ax  0 be a parabola and P(x1, y1) be a point located in the plane of the parabola and
external to it. Therefore, by Theorem 4.4, we get
S11  y12 4ax1  0
Also, by Theorem 4.7, the line
a
y  mx
m
touches the parabola. Now, this line passes through the point
a
P ( x1 , y1 ) š y1  mx1
m
š x1 m2 y1 m a  0 has distinct real roots
š y12 4ax1  0 which is true because (x1, y1) is an external point.
Hence, through (x1, y1), there are two tangents whose slopes, say m1 and m2 , are the roots of the
quadratic equation x1 m2 y1 m a  0.

In the quadratic equation obtained in Theorem 4.8, the sum of the roots is
y1
m1 m2 
x1
and the product of the roots is
a
m1 m2 
x1
Now, the tangents through (x1, y1) are at right angles
a
š 1  m1 m2 
x1
š x1 a  0
š Locus of (x1, y1) is the directrix x a  0
Theorem 4.9 provides the clear interpretation of the intersection of perpendicular tangents.

T H E O R E M 4.9 The locus of the point of intersection of perpendicular tangents to a parabola is the directrix of
(D I R E C T O R the parabola. This locus is called director circle even though it is a line.
CIRCLE)
PROOF According to Quick Look 6, the lines
a x
y  mx and y  am
m m
are tangents to the parabola and they intersect at right angles. Their point of intersection satisfies
the equation
¥ 1´ ¥ 1´
x ¦ m µ  a ¦ m µ
§ m ¶ § m¶
Hence, the locus of the point of intersection is the line x  −a which is the directrix.
222 Chapter 4 Parabola

Example 4.7

Find the locus of the point of intersection of perpendicu- where


lar tangents to the parabola y2 4 x 2 y 3  0.
1
a  1, X  x and Y  y − 1
Solution: The given equation is written as 2

¥ 1´ Directrix equation is
( y 1)2  4 ¦ x µ
§ 2¶ 1
X  a or x 1
 Y 2  4aX 2
Thus, the required locus is
2x − 1  0

Example 4.8

Find the director circle of the parabola x 2 2 y  4 x 3. X 2  4aY

Solution: The given equation is written as where a  1/2.


¥ 1´ The directrix equation Y  a or y − 1  0.
( x 2)2  1 2 y  2 ¦ y µ Therefore, the director circle is the directrix y − 1  0.
§ 2¶
Let X  x 2 and Y  y (1/2). Substituting in above
equation we get

DEFINITION 4.10 Normal As defined in Vol. 3, the normal to a parabola at point P is defined to be the line per-
pendicular to the tangent to the parabola at point P and passing through point P (see Fig. 4.7).
nt
ge
n

y
Ta

P
90°

Normal

O x

FIGURE 4.7

T H E O R E M 4.10 Equation of the normal at t is


[N O R M A L
tx y  2at at3
EQUATION AT
(at 2 , 2at)]
PROOF Tangent at (at2, 2at) is ty  x at2 (see Quick Look 4). Thus the slope of the tangent at t is 1/t.
Hence, the equation of the normal at (at2, 2at) is

y 2at  t( x at 2 )
 tx y  2at at3
4.2 Parabola 223

QUICK LOOK 7

Since the slope of the normal at (at2, 2at) is −t, replac- slope m at the point (am2, − 2am) is
ing −t by m, the equation of the normal in terms of its
y  mx − 2am − am3

T H E O R E M 4.11 The point of intersection of the normals at t1 and t2 is


(P O I N T O F
INTERSECTION [2a a(t12 t1t2 t22 ), at1t2 (t1 t2 )]
OF NORMALS
A T t 1 A N D t 2)

PROOF According to Theorem 4.10, the two equations

t1 x y  2at1 at13 (4.3)

t2 x y  2at2 at23 (4.4)


are normals at t1 and t2. Solving Eqs. (4.3) and (4.4), we get

x  2a a(t12 t1t2 t22 ) and y  at1t2 (t1 t2 )

T H E O R E M 4.12 If the normal to the parabola at t1 meets the curve again at t2, then
2
t2  t1
t1

PROOF Equation of the normal at (at12 , 2at2 ) is

t1 x y  2at1 at13

This line meets the curve again at (at22 , 2at2 ). So we have

t1 (at22 ) 2at2  2at1 at13

 t1 (t22 t12 )  2(t1 t2 )

 t1 (t2 t1 )  2 (& t1 x t2 )

2
 t2  t1
t1

T H E O R E M 4.13 If the normals at P(at12 , 2at1 ) and Q(at22 , 2at2 ) intersect at a point R on the curve again, then
1. t1t2  2
2. The product of the abscissa of P and Q is 4a2
3. The product of the ordinates of P and Q is 8a2
PROOF 1. Let R be (at32 , 2at3 ). Then, by Theorem 4.12, we have
2 2
t1  t3  t2
t1 t2
¥ 1 1´
 t2 t1  2 ¦ µ
§ t1 t2 ¶
 t1t2  2
224 Chapter 4 Parabola

2. The product of the abscissa  (at12 )(at22 )  a 2 (t1t2 )2  a 2 (2 2 )  4a 2 .


3. The product of the ordinates  (2at1)(2at2)  4a2(t1t2)  8a2.

DEFINITION 4.11 Normal Chord A chord of a parabola is called a normal chord if it is normal at one of its
extremities.

T H E O R E M 4.14 If the line lx my n  0 is a normal to the parabola y2  4ax, then al3 2alm2 m2 n  0.
PROOF Suppose the line
lx my n  0 (4.5)
is normal at (at2, 2at). However, at (at2, 2at), the equation of the normal is
tx y  2at at3 (4.6)
Equations (4.5) and (4.6) represent the same straight line. Therefore

t 1 (2at at 3 )
 
l m n
l n ¥ l´ ¥ l3 ´
t and  2at at 3  2a ¦ µ a ¦ 3 µ
m m § m¶ §m ¶
 m2 n  2alm2 al 3
 al 3 2alm2 m2 n  0

DEFINITION 4.12 Chord of Contact Let P be an external point to a parabola and the tangents drawn from P
to the parabola touch the curve at A and B. Then, the chord AB is called the chord of contact
of P with respect to the given parabola.

T H E O R E M 4.15 Equation of the chord of contact of the point P(x1, y1) with respect to the parabola S y y2 4ax  0
is S1  0.
PROOF See Fig. 4.8. Suppose A(x2, y2) and B(x3, y3) are the points of contact of the tangents drawn from
P(x1, y1) to the parabola. Hence, by Theorem 4.5, the equation of the tangent at A(x2, y2) is
S2 y yy2 2a( x x2 )  0
This tangent passes through the point P(x1, y1). Therefore,
y1 y2 2a( x1 x2 )  0
Thus, A(x2, y2) satisfies the equation
S1 y yy1 2a( x x1 )  0
Similarly, B(x3, y3) satisfies the equation S1  0. Hence, the equation of the line AB is S1  0.

A (x2, y2)

P (x1, y1)

B (x3, y3)

FIGURE 4.8
4.2 Parabola 225

T H E O R E M 4.16 The combined equation of the pair of tangents drawn from an external point P(x1, y1) to the
(P A I R O F parabola S y y2 4ax  0 is S12  SS11 .
TANGENTS)
PROOF See Fig. 4.9. Let Q(x2, y2) ( ≠ P) be a point on the pair of tangents drawn through P. According to
Theorem 4.8, two tangents can be drawn from P. Suppose point R divides the segment PQ in the
ratio l:K (K ≠ − 1). Hence, the coordinates of R are
¥ L x1 x2 L y1 y2 ´
¦§ , µ
L 1 L 1 ¶
R lies on the curve. So we have
2
¥ L y1 y2 ´ ¥ L x1 x2 ´
¦§ µ¶ 4a ¦§ µ 0
L 1 L 1 ¶
š L 2 S11 2 L S12 S22  0 (4.7)
Now
FHHE
PQ touches the parabola
š Equation (4.7) has equal roots in K
2
š 4S12  4S11S22
2
š S12  S11S22

Therefore, the locus of Q(x2, y2) is S12  S11S or S12  SS11 .

P (x1, y1) O x

Q (x2, y2)

FIGURE 4.9

QUICK LOOK 8

Note
S21  SS11

 [ yy1 2a( x x1 )]2  ( y2 4ax)( y12 4ax1 )

Note: The equation of the pair of tangents in Quick Look 8 is generally discussed only in theory and it is not useful in
solving problems. In most of the problems, we use that
a
y  mx
m
is a tangent to the parabola y2  4ax.
226 Chapter 4 Parabola

T H E O R E M 4.17 The equation of the chord of S y y2 4ax  0 whose midpoint is (x1, y1) is
(E Q U A T I O N O F
S1  S11
THE CHORD
USING ITS  yy1 2a( x x1 )  y12 4ax1
MIDPOINT)

PROOF Suppose A(x2, y2) and B(x3, y3) are the ends of the chord whose midpoint is M(x1, y1). By
FHHHE
Theorem 4.5, the equation of the line AB is
S2 S3  S23.
That is,
[ yy2 2a( x x2 )] [ yy3 2a( x x3 )]  y2 y3 2a( x2 x3 ).
y( y2 y3 ) 2a( x2 x3 ) 4ax  y2 y3 2a( x2 x3 ).
Since x2 x3  2x1 and y2 y3  2y1, we have
2 yy1 4ax1 4ax  y2 y3 4ax1
That is,
y2 y3
yy1 2ax  (4.8)
2
Since the chord AB is passing through (x1, y1), from Eq. (4.8), we have
y2 y3
 y12 2ax1 (4.9)
2
Therefore, from Eqs. (4.8) and (4.9), the chord equation is

yy1 2ax  y12 2ax1

 yy1 2a( x x1 )  y12 4ax1


Hence, S1  S11 is the equation of the chord.

DEFINITION 4.14 Diameter of a Parabola A line intersecting the parabola and parallel to the axis of the
parabola is called a diameter of the parabola.

QUICK LOOK 9

The line y  k (i.e., a diameter) meets the parabola at the point

¥ k2 ´
¦ 4a , k µ
§ ¶

T H E O R E M 4.18 In a parabola, the midpoints of parallel chords lie on a line which is a diameter of the parabola.
PROOF See Fig. 4.10. Suppose M(x1, y1) be the midpoint of a chord which is parallel to the line lx my
n  0. By Theorem 4.17, equation of the chord whose midpoint is (x1, y1) is
S1  S11

 yy1 2a( x x1 )  y12 4ax1


 2ax y1 y y12 2ax1  0
4.2 Parabola 227

This chord is parallel to the line


lx my n  0  2am ly1  0
This implies that (x1, y1) lies on the line 2am ly  0, which is horizontal, and hence it is a diam-
eter.
y

O x

FIGURE 4.10

Note: If the chords are vertical, then their midpoints lie on the axis of the parabola.

Example 4.9

Find the coordinates of the point whose chord of contact S1 y yy1 2( x x1 )  0 (4.11)
with respect to the parabola y2  4x is 2 x 7 y 2  0.
Therefore, from Eqs. (4.10) and (4.11), we get
Solution: Suppose (x1, y1) is the point whose chord of 2 y1 2 x1
contact with respect to y2  4x is  
2 7 2
2x 7y 2  0 (4.10) Hence, x1  1 and y1  7. Thus, the required point is
However, the chord of contact of (x1, y1) is (1, 7).

Example 4.10

Show that the chords of contacts of points on the line yy1 2a( x x1 )  0 (4.13)
2x 3y 4  0 with respect to the parabola y2  4ax pass
through a fixed point. From Eqs. (4.12) and (4.13), we get
yy1 2ax a(3 y1 4)  0
Solution: Let P(x1, y1) be a point on the line 2x 3y
4  0. Therefore  y1 ( y 3a) 2a( x 2)  0 (4.14)
2 x1 3 y1 4  0 (4.12) By Theorem 2.20, Eq. (4.14) represents the lines passing
Now, the chord of contact of (x1, y1) with respect to through the fixed point which is the intersection of the
y2  4ax is lines x  2 and y  3a. Hence, the fixed point is (2, 3a).

Example 4.11

The normals at P and Q of the parabola y2  4ax meet at Solution: Let P  (at21, 2at1) and Q  (at22 , 2at2 ). Sup-
a point (x1, y1) on the curve. Show that pose (x1, y1)  (at32 , 2at3 ). Therefore, by Theorems 4.12
( PQ)2  ( x1 4a)( x1 8a) and 4.13, we have
228 Chapter 4 Parabola

2 2  [a(t12 t22 ) 2at1t2 ][a(t12 t22 ) 2at1t2 4a] (4.16)


t1  t3  t2
t1 t2
Also by Theorem 4.11, we have

and t1t2  2 (4.15) x1  2a a(t12 t1t2 t22 ) and y1  at1t2 (t1 t2 )

Now, Therefore
a(t12 t22 )  x1 2a at1t2 (4.17)
( PQ)2  a 2 (t12 t22 )2 4a 2 (t1 t2 )2
 a 2 (t1 t2 )2 ¨ª(t1 t2 )2 4 ·¹ Using Eqs. (4.15) and (4.17) in Eq. (4.16), we have
( PQ)2  ( x1 2a 2a 4a)( x1 2a 2a 8a)
 a 2 [t12 t22 2t1t2 ][t12 t22 2t1t2 4]
 ( x1 8a)( x1 4a)

DEFINITION 4.15 Focal Chord and Focal Radius The chord passing through the focus of a parabola is called the
focal chord. If P is a point on a parabola with focus S, then SP is called the focal radius of P.

T H E O R E M 4.19 Let y2  4ax be a parabola with focus S(a, 0) and directrix x a  0. Then, the following properties
(P R O P E R T I E S hold good.
OF FOCAL
1. If P(at12 , 2at1 ) is one end of the focal chord, then the other end is
CHORDS)
¥ a 2a ´
¦ 2, t µ
§ t1 1 ¶

That is, if t is the parameter of one end of the focal chord, then the parameter of the other end
is –1/t.
2. Length of the focal chord whose one end is (at2, 2at) is
2
¥ 1´
a¦t µ
§ t¶
3. If PSQ is a focal chord, then
1 1 1 2
 
SP SQ a 2a
In other words, the semi-latus rectum is the harmonic mean between the focal radii of the ends
of a focal chord.
4. The circle described on a focal chord touches the directrix.
5. The circle described on a focal radius of a point on the parabola touches the tangent at the
vertex.
6. The circle described on a focal radius of a point P(at2, 2at) cuts an intercept of length a 1 t 2
on the normal at P.
7. The tangents drawn at the extremities of a focal chord are at right angles and hence intersect
on the directrix of the parabola.

PROOF See Fig. 4.11.


1. Let Q(at22 , 2at2 ) be the other end of the focal chord for which P(at12 , 2at1 ) is one end. Equa-
tion of the focal chord PQ is
2
y 2at1  ( x at12 )
t1 t2
This chord passes through S(a, 0). So we have
2
0 2at1  (a at12 )
t1 t2
4.2 Parabola 229

 at12 at1t2  a at12


 t1t2  1
1
 t2 
t1

2. Let P(at2, 2at) be one end of the focal chord so that by point (1), the other end is
¥ a 2a ´
Q¦ 2 , µ
§t t ¶
Therefore,
2 2
¥ a´ ¥ 2a ´
( PQ)2  ¦ at 2 2 µ ¦ 2at µ
§ t ¶ § t ¶
2 2
¥ 1´ ¨ ¥ 1´ ·
 a2 ¦ t µ ©¦ t µ 4¸
§ t ¶ ©§ t ¶ ¸¹
ª
2 2
¥ 1´ ¥ 1´
 a2 ¦ t µ ¦ t µ
§ t¶ § t¶

4
¥ 1´
 a2 ¦ t µ
§ t¶
Hence,
2
¥ 1´
PQ  a ¦ t µ
§ t¶

¥ a 2a ´
3. Let P  (at2, 2at) and Q  ¦ 2 , µ . Then, we know that
§t t ¶

a
SP  at 2 a, SQ  a
t2
Therefore,

1 1 1 t2 1 2
  
SP SQ a(t 1) a(1 t 2 ) a 2a
2

and 2a is the semi-latus rectum. Hence, SP, semi-latus rectum, and SQ are in HP.
4. P(at2, 2at) and Q(a / t 2, 2a / t ) are the ends of a focal chord. It is known that the centre of the
circle described on PQ as diameter is
¥ 1 ¥ 2 a ´ ¥ 1´ ´
¦§ 2 ¦§ at 2 µ¶ , a ¦§ t t µ¶ µ¶
t
and radius [by point (2)] is
2
PQ a ¥ 1´
 ¦t µ
2 2§ t¶
Now, the distance of the centre of the circle from the directrix is
1¥ 2 a ´
¦ at 2 µ¶ a
2§ t
230 Chapter 4 Parabola

1 ¥ 2 1 ´
 a ¦ t 2 2µ
2 § t ¶
2
a ¥ 1´ PQ
 ¦§ t µ¶ 
2 t 2

Hence, the circle described on PQ as diameter touches the directrix.


5. The circle with SP as diameter is ( x a)( x at 2 ) y( y 2at )  0. The centre is

¥ a at 2 2at ´ ¥ a 2 ´
¦ 2 , 2 µ  ¦§ 2 (1 t ), at µ¶
§ ¶
and the radius is
1 1
SP  (at 2 a)2 4a 2 t 2
2 2
a a
 (t 2 1)2 4t 2  (t 2 1)
2 2
Distance of the centre from the y-axis is the radius, which is given by
a
(1 t 2 )
2
Thus, the circle described on SP as diameter touches the tangent at the vertex.
6. Let the circle described on SP meet the normal at P in N. Draw SM perpendicular to the tan-
gent at P. Since SNPM is a rectangle, we have (see Fig. 4.11)

a t(0) at 2
PN  SM  (& equation of the tangent at P is ty  x at 2 )
2
1 t

 a 1 t2

7. Let P(at2, 2at) and Q(a /t 2, 2a /t ). The tangent at P is


ty  x at2
whose slope is 1/t. The tangent at Q is
y a
 x 2
t t
whose slope is –t. Therefore, the tangents at P and Q intersect at right angles.

y
P

90°
N
O S x
Nor
m
al

FIGURE 4.11
4.2 Parabola 231

QUICK LOOK 10

Extremities of focal chords are the only points where angles on the direction and conversely.
the tangents drawn to the parabola intersect at right

T H E O R E M 4.20 In general, three normals can be drawn through a point in the plane of a parabola to the curve.
(N U M B E R O F
NORMALS FROM
A POINT)

PROOF Let P(h, k) be a point in the plane of y2  4ax. It is known that the normal to parabola at (at2, 2at)
(see Theorem 4.10) is tx y  2at at3. This normal passes through P(h, k )

š th k  2at at 3
š at 3 (2a h)t k  0 (4.18)
Equation (4.18) is a cubic equation in t and hence, in general, it has three roots and hence, there are
three points on the curve at which normals drawn to the parabola are concurrent at P(h, k).

QUICK LOOK 11

If t1, t2 and t3 are the roots of Eq. (4.18), then we have k


3. t1t2 t3 
1. t1 t2 t3  0 a
Also note that all the three roots t1, t2 and t3 need not
2a h
2. t1t2 t2 t3 t3 t1  necessarily be real and distinct.
a

DEFINITION 4.16 Conormal Points Points on a parabola are called conormal points if the normals drawn at
them are concurrent.
Section 4.2.2 provides a procedure to determine the number of normals from a given point to a parabola. Since the
proof involves Cardon’s method of solving a cubic equation, generally, the proof is avoided. Students are advised to
adopt only the procedure.

4.2.2 Procedure to Determine Number of Normals


Let P(x1, y1) be a point in the plane of the parabola. From Theorem 4.20, the normal at (at2, 2at) passes through
P(x1, y1) if and only if
tx1 y1  2at at 3 (4.19)
so that the number of normals through is equal to the number of real roots of Eq. (4.19). Therefore, Eq. (4.19) is written as
(2a x1 ) y
t3 t 1 0
a a
Let
2a x1 y
H , G  1 and $  G 2 4 H 3
3a a
Then we have
1. Only one normal if Δ  0.
2. Two normals if Δ  0.
3. Three normals if Δ  0.
232 Chapter 4 Parabola

Example 4.12

Find the number of normals to the parabola y2  4ax from Hence, there are three normals to the parabola from the
the point (6a, 0) and also find the feet of the normals. point (6a, 0) and the feet of the normals are (0, 0), (4a,
4a) and (4a, −4a). Also, according to the procedure given
Solution: Normal tx y  2at at3 passes through in Section 4.2.2, we have
(6a, 0). So we have
2a x1 2a 6a 4
3
x1  6a, y1  0, H   
6at  2at at 3a 3a 3

 t 3 4t  0 and
3
 t ( t 2 4)  0 y1 ¥ 4 ´
G  0 and $  G 2 4 H 3  0 4 ¦ µ  0
a § 3¶
 t  0, p 2
Hence, there are three normals to the given parabola.

Example 4.13

Find the number of normals to y2  4ax from the point Hence, from (a, a), there is only one normal. Also it is clear
(a, a). that the cubic equation in t is
P (t ) y t 3 t 1  0
Solution: We have
which has only one real root because if it has two real
2a x1 2a a 1
x1  a, y1  a, H    roots, then by Rolle’s theorem (Theorem 3.4, Vol. 3)
3a 3a 3
Pa(t)  3t2 1  0
y a
and G 1   1 has a root in between them which is absurd. Thus, the
a a
cubic equation
Now
P (t ) y t 3 t 1  0
3
¥ 1´ has only one real root and hence it has only one normal
$  G 2 4 H 3  ( 1)2 4 ¦ µ  0
§ 3¶ through (a, a).

Example 4.14

Let (atr2 , 2atr ) for r  1, 2 and 3 be conormal points on the y1


parabola y2  4ax. If the normals at those points to the t1t2 t3  (4.20c)
a
curve meet the axis of the parabola in points whose dis-
tances from the vertex are in AP, then prove that the point The normal at t1, namely, t1 x y  2at1 at13 meets the
of concurrence of the three normals lies on the curve. axis at the point (2a at12 , 0) and similarly the normals at
27ay2  2( x 2a)3 . t2 and t3 meet the axis at (2a at22 , 0) and (2a at32 , 0). By
hypothesis,
Solution: Let P(x1, y1) be the point where the normals
(2a at12 ) (2a at32 )  4a 2at22
at t1, t2 and t3 are concurrent. Hence, by Theorem 4.20, t1,
t2 and t3 are roots of the equation  t12 t32  2t22 (4.20d)
3
at (2a x1 )t y1  0 (4.20) Also, from Eq. (4.20a), we have
Therefore, by Quick Look 11, we have (t1 t3 )2  t22  t12 2t1t3 t32  t22
t1 t2 t3  0 (4.20a) Therefore, from Eq. (4.20d), we get
2a x1
t1t2 t2 t3 t3 t1  (4.20b) 2t22 2t1t3  t22
a
4.2 Parabola 233

and from Eq. (4.20c), we get 1/ 3


¥ 2y ´
 3 y1  (2a x1 ) ¦ 1 µ (4.21)
¥ y ´ § a ¶
t22  2t1t3  2 ¦ 1 µ
§ at2 ¶ Taking cube on both sides of Eq. (4.21), we get
2 y1
 t23  2 y1
a 27 y13  (2a x1 )3
a
Since t2 is a root of Eq. (4.20), we have
 27ay12  2( x1 2a)3
1/ 3
¥ 2 y1 ´ Hence, the point (x1, y1) lies on the curve
2 y1 (2a x1 ) ¦ y1  0
§ a µ¶
27ay2  2( x 2a)3

T H E O R E M 4.21 In general, a circle and parabola will intersect in four points such that the algebraic sum of the
ordinates of the common points is zero.

PROOF Suppose S y x 2 y2 2 gx 2 fy c  0 is a circle and y2  4ax is a parabola. Every point on the


parabola is of the form (at2, 2at), t Z. Substituting x  at2 and y  2at in the circle equation
S  0, we have

a 2 t 4 2a( g 2a)t 2 4aft c  0 (4.22)


Equation (4.22), being a fourth-degree equation in t, has four real roots, at the most. If t1, t2, t3 and
t4 are the roots of Eq. (4.22), then
t1 t2 t3 t4  0

so that
2at1 2at2 2at3 2at4  0
or y1 y2 y3 y4  0
where yr  2atr (r  1, 2, 3, 4). Thus, the algebraic sum of the ordinates of the common points is
zero.

QUICK LOOK 12

If t1, t2, t3 and t4 are the parameters of the common 2( g 2a)


points of the parabola y2  4ax and the circle ¤t t 1 2 
a
(4.23b)
S y x 2 y2 2 gx 2 fy c  0, then tr, where r  1, 2, 3
and 4, are roots of the equation 4 f
2 4 2
¤t t t 1 2 3 
a
(4.23c)
a t 2a( g 2a) t 4aft c  0
c
and hence by the relation between the roots and the t1t2 t3 t4  (4.23d)
a2
coefficients, we have
t1 t2 t3 t4  0 (4.23a)

Subjective Problems
1. Prove that the foot of the perpendicular drawn from tangent at the vertex (see Fig. 4.12).
the focus onto any tangent to a parabola falls on the
234 Chapter 4 Parabola

y Now,
P
¥ [a(t 2 1) / t ] 0 ´ ¥ 2at 0 ´
T 90° Slopeof SQ s Slope of SP  ¦ µ ¦§ 2 µ
§ a a ¶ at a ¶
Normal
(t 2 1) 2t
O S(a, 0 ) x  s 2  1
2t t 1
Hence, PSQ  90o.

3. Prove that the circumcircle of a triangle formed by


three tangents to a parabola passes through the focus
of the parabola.
FIGURE 4.12
Solution: Let the tangents be
y2
Solution: Let  4ax be a parabola and P(at2, 2at)
be
tr y  x atr2
a point on the curve. The tangent at P(at2, 2at) is given
by where r  1, 2 and 3. By Problem 1, the feet of the per-
pendiculars drawn from the focus of the parabola onto
ty  x at2
the three tangents (which are the sides of a triangle) are
This meets the tangent at the vertex (i.e., y-axis) in the collinear on the tangent at the vertex. Hence, from the
point T(0, at). Since the focus is S(a, 0), the slope of ST is section ‘Pedal Line (or Simson’s Line)’ , Chapter 1, the
given by circumcircle of the triangle passes through the focus.
at 0 4. Prove that the orthocentre of a triangle formed by
 t
0 a three tangents to a parabola lies on the directrix.
However, the slope of the tangent at P  1/t. So
Solution: Let tr y  x atr2 , r  1, 2, 3 be three tangents
¥ 1´ to a parabola so that, from Theorem 4.6, the vertices of
Slope of ST s Slope of the tangent at P  t ¦ µ  1
§ t¶ the triangle formed by the three are given by
Therefore, ST is perpendicular to the tangent at P.
A[at1t2 , a(t1 t2 )]
2. Prove that the portion of the tangent to a parabola in- B[at2 t3 , a(t2 t3 )]
tercepted between the point of contact and the direc-
trix subtends a right angle at the focus of the parabola C[at3 t1 , a(t3 t1 )]
(see Fig. 4.13). So
y a(t1 t2 ) 1
P Slope of BC  
at3 (t1 t2 ) t3
Therefore, the equation of the altitude through A of
Q
$ABC is
90°
y a(t1 t2 )  t3 ( x at1t2 ) (4.24)
Z O S x
Similarly, the equation of the altitude through B is
y a(t2 t3 )  t1 ( x at2 t3 ) (4.25)

x+a=0 Subtracting Eq. (4.25) from Eq. (4.24), we get


a(t3 t1 )  (t1 t3 ) x
FIGURE 4.13  x  a

Solution: The tangent at P(at2, 2at) is ty  x at2. This That is, the abscissa of the orthocentre is –a and hence
meets the directrix at the point the orthocentre lies on the directrix.

5. If P( x1 , y1 ), Q( x2 , y2 ) and R( x3 , y3 ) are the three


¥ a(t 2 1) ´
Q ¦ a, µ points on the parabola y2  4ax, then show that the
§ t ¶ area of the triangle PQR is the absolute value of
4.2 Parabola 235

1 1
( y1 y2 )( y2 y3 )( y3 y1 )  (Area of $PQR) (By Quick Look 13)
8a 2

Solution: The area of ΔPQR is 7. Prove that the area of the triangle formed by the three
normals to the parabola y2  4ax at the points t1 , t2 and
x1 y1 1 ¥ 4ax1 y1 1´ t3 is
1 1 ¦
x2 y2 1  absolute value of 4ax2 y2 1µ 1 2
2 8a ¦¦ µ a (t1 t2 )(t2 t3 )(t3 t1 ) (t1 t2 t3 )2
x3 y3 1 § 4ax3 y3 1 µ¶ 2
y12 y1 1 Solution: By Theorem 4.11, the vertices of the triangle
1 2 are
 Absolute value of y2 y2 1
8a
y32 y3 1 [2a a(t12 t1t2 t22 ), at1t2 (t1 t2 )]

1 [2a a(t22 t2 t3 t32 ), at2 t3 (t2 t3 )]


 ( y1 y2 )(( y2 y3 )( y3 y1 )
8a
and [2a a(t32 t3 t1 t12 ), at3 t1 (t3 t1 )]

QUICK LOOK 13
Let these points be A, B and C, respectively. Therefore
Area of the $ABC
If we replace y1, y2 and y3, respectively, by 2at1, 2at2
and 2at3, then the area of the triangle is equal to  Absolute value of

a 2 (t1 t2 )(t2 t3 )(t3 t1 ) 2a a(t12 t1t2 t22 ) at1t2 (t1 t2 ) 1


1
2a a(t22 t2 t3 t32 ) at2 t3 (t2 t3 ) 1
2
2a a(t32 t3 t1 t12 ) at3 t1 (t3 t1 ) 1
6. Prove that the area of the triangle formed by three
tangents to a parabola is half of the area of the tri-  Absolute value of
angle whose vertices are the points of contact of the
tangents. (IIT-JEE 1996) 2 t12 t1t2 t22 t1t2 (t1 t2 ) 1
a2
(t3 t1 )(t2 t1 t3 ) t2 (t3 t2 )(t2 t1 t3 ) 0
2
Solution: Let P(at12 , 2at1 ), Q(at22 , 2at2 ) and R(at32 , 2at3 ) (t3 t2 )(t1 t2 t3 ) t1 (t3 t2 )(t1 t2 t3 ) 0
be the three points on y2  4ax and the three points A,
B, and C be the points of intersection of the tangents (By R2 R1 and R3 R1)
drawn at points P, Q and R of the triangle. Therefore, by  Absolute value of
Theorem 4.6, we have
¨ a2 2·
A[at1t2 , a(t1 t2 )] © (t3 t1 )(t3 t1 )(t1 t2 t3 ) ¸
ª2 ¹
B[at2 t3 , a(t2 t3 )] 2 t12 t1t2 t22 t1t2 (t1 t2 ) 1
C[at3 t1 , a(t3 t1 )] s 1 t2 0
Therefore 1 t1 0

Area of $ABC a2
 (t1 t2 )(t2 t3 )(t3 t1 ) (t1 t2 t3 )2
at1t2 a(t1 t2 ) 1 2
1
 Absolute value of the determinant at2 t3 a(t2 t3 ) 1
2 8. Show that the length of the side of an equilateral tri-
at3 t1 a(t3 t1 ) 1 angle inscribed in the parabola y2  4ax with one ver-
at1t2 a(t1 t2 ) 1 tex at the vertex of the parabola is 8a 3 .
a2
 Absolute value of (t3 t1 )(t1 t2 ) t2 1 0 Solution: Let OPQ be the equilateral triangle in-
2
t3 1 0 scribed in the parabola (see Fig. 4.14), where O(0, 0) is
(By R3 R2 and then R2 R1) the vertex so that POM  QOM  30o. Let OP 
OQ  r. Therefore
a2
 (t1 t2 )(t2 t3 )(t3 t1 ) ¥r 3 r´
2 P  (r cos 30o, r sin 30o)  ¦ , µ
§ 2 2¶
236 Chapter 4 Parabola

Now, P lies on the parabola. So Therefore, the equation of the chord PQ is


r2 ¥ r 3´ y  mx 4am
 4a ¦ µ
4 § 2 ¶  m(x 4a) y  0
 r  8a 3 Certainly, this line passes through the fixed point (4a, 0).

y y
P
P

30° 90° x
O 30° M x O

Q Q

FIGURE 4.14 FIGURE 4.15

9. Show that the locus of the midpoints of chords of a 11. Show that the intersection of two perpendicular nor-
parabola passing through the vertex is in turn a pa- mals to the parabola lies on the curve y2  a(x 3a).
rabola whose latus rectum is half of the latus rectum
of the original. Solution: The normals at P(at12 , 2at1 ) and Q(at22 , 2at2 )
are
Solution: By Theorem 4.17, the equation of the chord
t1 x y  2at1 at13
in terms of its midpoint M(x1, y1) is
S1  S11 and t2 x y  2at2 at23

 yy1 2a( x x1 )  y12 4ax1 respectively. Therefore, their point of intersection is

 yy1 2ax  y12 2ax1 which passes through


h (0, 0) [2a a(t12 t1t2 t2 ), at1t2 (t1 t2 )]

 y12 2ax1  0 Suppose this point of intersection is R. Since the normals


are at right angles, we have (−t1)(−t2)  −1 or t1t2  1.
Hence, the locus of M(x1, y1) is the parabola y2  2ax. Let
10. Prove that the chords of a parabola subtending right x1  2a a(t12 t1t2 t22 )  2a a ¨ª(t1 t2 )2 t1t2 ·¹ (4.26)
angle at the vertex pass through a fixed on the axis.
(IIT-JEE 1994) y1
and y1  at1t2 (t1 t2 )  a( 1)(t1 t2 ) or t1 t2 
a
Solution: See Fig. 4. 15. Let y  mx c be a chord (say, Therefore, from Eq. (4.26), we have
PQ) subtending a right angle at the vertex. Therefore, by
Theorem
FHHE F2.33,
HHHE the combined equation of the pair of lines x1  2a a ¨ª(t1 t2 )2 t1t2 ·¹
OP and OQ is
¨ y2 · y2
¥ y mx ´  2a a © 12 1¸  3a 1
y2 4ax ¦ 0 ªa a
§ c µ¶ ¹
or y12  a( x1 3a)
Since POQ  90o, in the above equation,
Hence, R(x1, y1) lies on y2  a(x – 3a).
Coefficient of x 2 Coefficient of y2  0
4am 12. Let P be a point on the parabola y2  4ax and N be the
 1 0 foot of the perpendicular drawn from point P onto
c
the axis. If T is the intersection of the tangent at P with
 c  4am the axis of the parabola, then show that vertex is the
midpoint of TN and SP  ST, where S is the focus.
4.2 Parabola 237

Solution: Let P be (at2, 2at) so that the equation of the 14. Prove that the area of the triangle formed by pair
tangent at P is ty  x at2 and it meets the axis (y  0) at of tangents drawn from (x1, y1) to the parabola
point T( at2, 0). However, N  (at2, 0). Hence, the vertex S y y2 4ax  0 and the chord of contact of (x1, y1)
is the midpoint of TN. Also, is
ST  SO OT  a(1 t2)  SP S 3/ 2
2a
13. Prove that the locus of the midpoints of chords of a
parabola which subtend right angle at the vertex is
Solution: See Fig. 4.17. The equation of the chord of
another parabola whose latus rectum is half that of
contact of point P(x1, y1) is
the original parabola. Further, all such chords pass
through a fixed point on the axis of the parabola. S1  0
 yy1 2a( x x1 )  0 (4.29)
Solution: See Fig. 4. 16. Let M(x1, y1) be the midpoint
of a chord of S y y2 4ax  0 which subtends a right Suppose Q(h, k) and R(l, m) are the ends of the chord of
angle at the vertex. Hence, by Theorem 4.17, equation of contact of P. Hence, k and m are the roots of the equa-
the chord is tion. By Eq. (4.29), and y2  4ax, we have
S1  S11 ¥ y2 ´
yy1  2a ¦ x1 µ
 yy1 2a( x x1 )  y12 4ax1 § 4a ¶
so that
 yy1 2ax  y12 2ax1
k m  2y1 (4.30)
yy1 2ax
 1 (4.27) and km  4ax1 (4.31)
y12 2ax1
Also, Q and R lie on the line provided in Eq. (4.29). So
Suppose the chord provided in Eq. (4.27) cuts the parab-
ola at points P and Q. Therefore, by Theorem 2.33, ky1  2ah 2ax1 and my1  2al 2ax1
FHHE FHHHE the
combined equation of the pair of lines OP and OQ is
 (k m) y1  2a(h l )
¥ yy 2ax ´ h l y
y 2 4a ¦ 2 1 µ 0 (4.28)   1 (4.32)
§ y1 2ax1 ¶ k m 2a
Since POQ  90o, in Eq. (4.28) Therefore

Coefficient of x2 Coefficient of y2  0 y12


2
RQ  (h l )2 (k m)2  2
(k m)2 (k m)2
8a 4a
 1 0
y12 2ax1 k m y12 4a 2
(k m)2 4k
km y12 4a 2
 
 y12  2a( x1 4a) 2a 2a

Hence, the locus of (x1, y1) is the parabola y2  2a( x 4a). Therefore, from Eqs. (4.30) and (4.31), we get
If we put y12  2ax1 8a 2 in Eq. (4.27), we get the equa-
tion of the chord as yy1 2ax  8a 2 which passes 4 y12 16ax1 y12 4a 2
RQ 
through (4a, 0). 2a
1 2
y
P
 y1 4ax1 y12 4a 2 (4.33)
a
Now,
PM  Perpendicular distance from P onto the chord QR

90° x
y12 2a( x1 x1 ) y12
O M (x1, y1)  
y12 4a 2 y12 4a 2

[& ( x1 , y1 ) lies outside] (4.34)

Q Hence, from Eqs. (4.33) and (4.34), the area of $PQR is

FIGURE 4.16
238 Chapter 4 Parabola

¥ ( y2 4ax1 )3 / 2 ´ S11
3/ 2 By hypothesis, the length of the segment PQ is
1
(QR)( PM )  ¦ 1 µ 
2 § 2a ¶ 2a a t1 t2 (t1 t2 )2 4  2l
Thus,
Q
( PQ)2  a 2 (t1 t2 )2 [(t1 t2 )2 4]  4l 2

 a 2 [(t1 t2 )2 4t1t2 ][(t1 t2 )2 4]  4l 2 (4.37)


Now, from Eqs. (4.35) and (4.36), we get
M
P (x1, y1) y1
t1 t2 
a
2 x1
R and t12 t22 
a
FIGURE 4.17 which imply that

15. Show that the locus of the midpoint of chords of y12 2 x1


 (t1 t2 )2  t12 t22 2t1t2  2t1t2
a parabola which pass through a fixed point is in a 2
a
turn a parabola.
Thus,
Solution: Let M(x1, y1) be the midpoint of a chord of y12 2ax1
S y y2 4ax  0 which passes through a fixed point, say, t1t2  (4.38)
2a 2
(h, k). So
Substituting the value of t1t2 from Eq. (4.38) and
yy1 2a( x x1 ) ( y12 4ax1 )  0 t1 t2  y1 /a in Eq. (4.37), we have
(S1  S11) passes through (h, k). Therefore we have ¨ y2 4( y12 2ax1 ) · ¨ y12 ·
a 2 © 12 2 ¸ © 2
4 ¸  4l
2
ky1 2a(h x1 ) y12 4ax1  0 ªa 2a ¹ ªa ¹
 ky1 2ah y12 2ax1  0  (4ax1 y12 )( y12 4a 2 )  4a 2 l 2

 y12 ky1  2a( x1 h) Hence, the locus of (x1, t1) is

¥ k´
2
k 2 (4ax y2 )( y2 4a 2 )  4a 2 l 2
 ¦ y1 µ  2a( x1 h)
§ 2¶ 4
2
17. Find the locus of the midpoints of focal chords of
¥ k´ ¨ k2 · S  y2 4ax  0.
 ¦ y1 µ  2a © x1 h ¸
§ 2¶ ª 8a ¹
Solution: Let M(x1, y1) be the midpoint of a focal
Therefore, the locus of (x1, y1) is the parabola chord. Then
2
¥ k´ ¥ k2 ´ yy1 2a( x x1 )  y12 4ax1
¦§ y µ¶  2a ¦ x h µ
2 § 8a ¶ It passes through (a, 0). So
16. Show that the locus of the midpoints of chords of the 0 2a(a x1 )  y12 4ax1
y2
parabola  4ax which are of constant length 2l is
 y12  2ax1 2a 2
(4ax y2 )( y2 4a 2 )  4a 2 l 2.
 y12  2a( x1 a)
Solution: Let P(at12 ,
2at1 ) and Q(at22 ,
2at2 ) be the
extremities of a chord of length 2l and M(x1, y1) be the Thus, the locus of M(x1, y1) is the parabola y2  2a( x a)
midpoint of PQ so that whose vertex is S (a, 0) and latus rectum is 2a.

a(t12 t22 ) 18. Show that the midpoints of normal chords of the pa-
x1  (4.35) rabola S y y2 4ax  0 lies on the curve
2
2a(t1 t2 ) y 2 4a 3
and y1   a(t1 t2 ) (4.36)  x 2a
2 2a y 2
4.2 Parabola 239

Solution: Let M(x1, y1) be the midpoint of a normal 20. P(at12 , 2at1 ) and Q(at22 , 2at2 ) are two points on the
chord, which is normal at the point (at2, 2at). Hence, the parabola S  y2 4ax  0 such that t1 t2  4. Show
two equations yy1 2ax  y12 2ax1 and tx y  2at at 3
that the circle described on PQ as diameter touches
represent the same normal. Therefore,
the parabola.
t 1 2at at 3
  2 Solution: The equation of the circle described as PQ
2a y1 y1 2ax1
as diameter is
2a y12 2ax1
t and 2at at 3  ( x at12 )( x at22 ) ( y 2at1 )( y 2at2 )  0 (4.39)
y1 y1
3 and the equation of the parabola y2  4ax in parametric
¥ 2a ´ ¥ 2a ´ y12 2ax1 form is
 2a ¦ a 
§ y µ¶ 1
¦§ y µ¶ y
1 1
x  at2 and y  2at
4a 2
8a 4
y12 2ax1 The circle provided in Eq. (4.39) touches the parabola if
 3 
y1 y1 y1 the equation obtained by substituting x  at2 and y  2at
in the circle has equal roots. The common points of circle
8a 4
 4a 2  y12 2ax1 and the parabola are given by
y12
(at 2 at12 )(at 2 at22 ) (2at 2at1 )(2at 2at2 )  0
4a 3 y2
 2a  1 x1  (t t1 )(t t2 ) 4  0
y12 2a
 t 2 (t1 t2 )t t1t2 4  0
y 2 4a 3
 1 2  x1 2a
2a y1 whose discriminant is

Hence, (x1, y1) lies on the curve (t1 t12 )2 4(t1t2 4)  (t1 t2 )2 16

y 2 4a 3  4 2 16 (by hypothesis t1 t2  4)
 x 2a
2a y 2 0
Hence, the circle touches the parabola.
19. Prove that tangent to a parabola bisects the angle
between its focal radius and the line drawn parallel
to the axis through the point of contact. QUICK LOOK 14

Problem 20 is also stated as follows: Prove that the


Solution: See Fig. 4.18. Suppose the tangent at point P
circle described on a chord of a parabola as diameter,
meets the axis at point T. Tangent at P(at 2, 2at ) is
such that the difference of the ordinates of the ends
ty  x at2 of the chord is equal to twice the latus rectum,
so that touches the parabola.

T  ( at2, 0)
21. Prove that the locus of the point, through which only
Hence, ST  a at2  SP. Therefore, $SPT is isosceles
and hence two normals can be drawn to the parabola y2  4ax,
is the curve 27ay2  4( x 2a)3 .
SPT  STP  TPT a
y
Solution: Let P(x1, y1) be a point through which the
two normals are drawn to the parabola. Therefore, if t1, t2
T P and t3 are the parameters of the feet of the normals, then
by Theorem 4.20, t1, t2 and t3 are the roots of the cubic
equation

at 3 (2a x1 )t y1  0
T O S (a, 0 ) x
and hence we have
t1 t2 t3  0 (4.40)
2a x1
t1t2 t2 t3 t3 t1  (4.41)
a
FIGURE 4.18
240 Chapter 4 Parabola

y1 23. There are three points A, B and C on a parabola at


t1t2 t3  (4.42) which the normals drawn are concurrent at a point
a
(h, k). Show that the circumcircle of the triangle
From point (x1, y1), only two normals exist štwo of the ABC passes through the vertex of the parabola and
parameters t1, t2 and t3 are equal. Suppose t1  t2. There- also find its equation.
fore, from Eq. (4.40),
(4.43) Solution: Suppose A  (a12 , 2at1 ), B  (at22 , 2at2 ) and
2t1  t3
C  (at32 , 2at3 ). Therefore, by Theorem 4.20, t1, t2 and t3
From Eqs. (4.42) and (4.43), we get are the roots of the cubic equation

y at 3 (2a h)t k  0 (4.46)


2t13  1 (4.44)
a where
Also from Eq. (4.41), t1 t2 t3  0 (4.46a)
2a x1 2a h
t12 2t1t3  t1t2 t2 t3 t3 t1  (4.46b)
a a
2a x1 k
 t12 2t1 ( 2t1 )  (& 2t1  t3 ) t1t2 t3  (4.46c)
a a
x1 2a Suppose the equation of the circumcircle of $ABC is
 3t12  (4.45)
a
x 2 y2 2 gx 2 fy c  0 (4.47)
From Eqs. (4.44) and (4.45), we get
Since A, B and C are the common points of the circle
¥ y1 ´
2
3 2 2 3 ¥ x1 2a ´
3
provided in Eq. (4.46) and the parabola, let D(at42 , 2at4 )
¦§ µ  (t1 )  (t1 )  ¦§ µ be the fourth point (by Theorem 4.21). Further, t1, t2, t3
2a ¶ 3a ¶
and t4 are roots of the equation
y12 ( x1 2a)3
  a 2 t 4 2a( g 2a)t 2 4aft c  0
4a 2 27a 3
So we have
 27ay12  4( x1 2a)3
t1 t2 t3 t4  0
Hence, the locus of the point is
2( g 2a)
27ay2  4( x 2a)3 ¤t t 1 2 
a
22. Show that the locus of the point through which two 4f
of the three normals drawn to the parabola  4ax y2 ¤t t t 1 2 3 
a
are at right angles to the curve is y2  a( x 3a). c
t1t2 t3 t4 
Solution: Suppose t2t3  1 so that a2
Now, t1 t2 t3 t4  0 and t1 t2 t3  0 [from Eq. (4.46a)]
y1 y
t1t2 t3   t1  1 imply that t4  0. Hence, the fourth point D is the vertex.
a a Also
Since t1 is a root of the cubic equation at 3 (2a x1 )t 2( g 2a)
y1  0, we have
a
 ¤ t1t2  t1t2 t1t3 t1t4 t2 t3 t2 t4 t3 t4
3
¥ y ´ ¥ y ´  t1t2 t2 t3 t3 t1 (& t4  0)
a ¦ 1 µ (2a x1 ) ¦ 1 µ y1  0
§ a ¶ § a ¶ Therefore, from Eq. (4.46b), we get
y12 2a x1 2( g 2a) 2a h
 1 0 
2 a a
a a
 2 g  2a h (4.48)
 y12 a(3a x1 )  0
Further,
Hence, the locus of (x1, y1) is
4 f
y2  a( x 3a) a
 ¤t t t
1 2 3  t1t2 t3 (& t4  0)
4.2 Parabola 241

Therefore, from Eq. (4.46c), we have The equation of the chord QR is


4 f k 2
a
 ¤t t t1 2 3  t1t2 t3 
a
y 2at1 
t1 t2
( x at12 ) (4.55)

k Substituting y  2at in Eq. (4.55), we get


 2f  (4.49)
2
A  (att1 att2 at1t2 , 2at )
Moreover,
Substituting y  2at in Eqs. (4.53) and (4.54), we get
c
0  t1t2 t3 t4  c0 (4.50) B  (2att1 at12 , 2at )
a2
Substituting the values of 2g, 2f and c from Eqs. (4.48), and C  (2att2 at22 , 2at )
(4.49) and (4.50), respectively, in Eq. (4.47), the equation Therefore
of the circumcircle of $ABC is obtained as
PB  at 2 2att1 at12  a(t t1 )2 (4.56)
2 2 ¥ k´
x y (2a h) x ¦ µ y  0
§ 2¶ and PC  a(t t2 )2 (4.57)
2 2
 2 x 2 y 2(2a h) x ky  0 Also

24. Suppose QR is the chord of contact of a point P with ( PA)2  (att1 att2 at1t2 at 2 )2
respect to the parabola S y y2 4ax  0. If QR is a
 a 2 [t(t1 t ) t2 (t1 t )]2
normal chord, normal at Q, then show that the direc-
trix bisects PQ.  a 2 (t1 t )2 (t2 t )2
Solution: Let Q  (at12 , 2at1 ) and R  (at22, 2at2) so that Therefore, from Eqs. (4.56) and (4.57), we have
P  [at1t2 , a(t1 t2 )]. Since QR is normal at Q, from The-
orem 4.8, we have ( PA)2  PB – PC

2 Q
t2  t1
t1

2 C B A y = 2at
or t1 t2  (4.51)
t1 P
O
Let (x, y) be the midpoint of PQ so that, from Eq. (4.51), x
we get

at12 at1t2 at1 (t1 t2 ) at1 ¥ 2 ´ R


x  
2 2 2 ¦§ t1 µ¶

That is, x  a and hence the directrix bisects PQ. FIGURE 4.19

25. A diameter, which is through the point P on a pa- 26. The normal at point P of a parabola meets the same
rabola, meets a chord QR of the parabola in point A. curve at Q. Show that the tangents at P and Q inter-
The tangents at Q and R meet the diameter through sect on the diameter passing through the other end
P in B and C. Show that PA is geometric mean be- of the focal chord passing through P.
tween PB and PC.
Solution: See Fig. 4.20. PQ is a normal chord normal at
Solution: See Fig. 4.19. Let P  (at2, 2at), Q  (at12 , 2at1 ), P. Let P  (at12 , 2at1 ) and Q  (at22 , 2at2 ) so that by Theo-
R  (at22 , 2at2 ). The diameter through P is rem 4.12, we get
y  2at (4.52) 2
t2  t1 (4.58)
Tangents at points Q and R, respectively, are t1
t1 y  x at12 (4.53) Let R be the point of intersection of the tangents at P
and Q so that by Theorem 4.6, we have
and t2 y  x at22 (4.54)
242 Chapter 4 Parabola

R  ;(at1t2 , a(t1 t2 )= Now,

Therefore, from Eq. (4.58), we get SP – SQ – SR  (a at12 )(a at22 )(a at32 )

¥ ¥ 2 ´ ´  a 3 (1 t12 )(1 t22 )(1 t32 )


R  ¦ at1t2 , a ¦ µ µ
§ § t1 ¶ ¶  a 3 ¨ª1 (t12 t22 t32 ) (t12 t22 t22 t32 t32 t12 )

¥ 2a ´ t12 t22 t32 ·¹


 ¦ at1t2 , (4.59)
§ t1 µ¶
Now, if Pa is the other end of the focal chord through P,
 a 3 ¨1 (t1 t2 t3 )2 2
ª ¤ t t (¤ t t )
1 2 1 2
2

then by Theorem 4.19, part (1), we have 2t1t2 t3 (t1 t2 t3 ) t12 t22 t32 ·¹
¨ 2
¥ a 2a ´ ¥ 2a h ´ ¥ 2a h ´ k2 ·
Pa  ¦ 2 ,  a 3 ©1 0 2 ¦ µ ¦ µ 0 2 ¸
§ t1 t1 µ¶ ©ª § a ¶ § a ¶ a ¸¹

Therefore, from Eq. (4.59), it is clear that the point R lies  a[a 2 4a 2 2ah 4a 2 4ah h 2 k 2 ]
on the diameter passing through Pa.
 a[a 2 2ah h2 k 2 ]
 a[(a h)2 k 2 ]
P
90°  a( AS)2

28. Show that the locus of the point from which the pair
of tangents drawn to the parabola y2  4ax including
S constant angle @ is cot 2 A ( y2 4ax)  ( x a)2 .
R P
Solution: Let P(x1, y1) be the point of intersection of
tangents drawn at Q(at12 , 2at1 ) and R(at22 , 2at2 ) so that
Q x1  at1t2 (4.60a)
and y1  a(t1 t2) (4.60b)
It is known that 1/t1 and 1/t2 are the slopes of the tangent
FIGURE 4.20 at Q and R so that by hypothesis, we have
2 2
27. If the normals at three points P, Q and R on the pa- ¥ (1/t ) (1/t2 ) ´ ¥ t t ´ (t t )2 4t1t2
tan 2 A  ¦ 1 µ ¦ 2 1 µ  1 2
rabola y2  4ax meet at point A, then show that § 1 (1/t1t2 ) ¶ § t1t2 1¶ (1 t1t2 )2
SP · SQ · SR  a(AS)2
Therefore, from Eqs. (4.60a) and (4.60b), we get
where S is the focus.
( y12 /a 2 ) (4 x1 /a)
tan 2 A 
Solution: Let A  (h, k) and P  (at12 , 2at1 ), 1 ( x1 /a)2
Q  (at22 , 2at2 ) and R  (at32 , 2at3 ) so that t1, t2 and t3
are roots of the equation y12 4ax1

( x1 a)2
at 3 (2a h)t k  0
Since t1, t2 and t3 are the roots, we have Therefore, the locus of (x1, y1) is
t1 t2 t3  0 (x a)2  cot2@ (y2 − 4ax)

2a h
t1t2 t2 t3 t3 t1 
a QUICK LOOK 15

k If @  90°, then the tangents from (x1, y1) are at


and t1t2 t3 
a right angles so that x1 a  0 and hence (x1, y1) lies
on the directrix.
4.2 Parabola 243

2 2 2
29. A straight line touches both x y  2a and y2  Therefore, from Eqs. (4.61a) and (4.61b), we get
8ax. Show that its equation is y  p( x 2a). y1
tan Q   ( x1 a)tan Q y1  0
x1 a
Solution: We have
This implies that (x1, y1) lies on the line (x − a) tanP − y  0
2a which passes through the focus (a, 0).
y  mx
m
32. Show that the point from which a tangent to y2 
is a tangent to y2  8ax for all m x 0. This line also touches 4a(x a) and a tangent to y2  4b(x a) are at right
the circle x2 y2  2a2. So angles lies on the line x a b  0.
4a 2
 ( 2a )2 (1 m2 ) Solution: The parabola y2  4a(x a) is written as Y2 
m2 4aX where Y  y and X  x a. For Y2  4aX, a tangent is
4 of the form
  2(1 m2 )
m2 a
 m4 m2 2  0 Y  mX
m
 (m2 2)(m2 1)  0 a
 y  m( x a)
 m2  1 or m  p1 m
Therefore
Therefore, the tangent equation is y  p( x 2a).
a
y  m1 ( x a)
30. If two tangents to a parabola make complementary m1
angles with the axis, then show that their point of in- is a tangent to y2  4a(x a) and
tersection lies on the line x  a.
b
y  m2 ( x b)
Solution: Let the tangents at t1 and t2 make comple- m2
mentary angles, say @ and A, with the axis of the pa-
is a tangent to y2  4b(x b). These tangents are perpen-
rabola. Therefore
dicular to each other, which implies that m1m2  1. Now,
1 the abscissa of the point of intersection satisfies the
tan A  equation
t1
a b
1 m1 ( x a) m2 ( x b) 0
and tan B  m1 m2
t2
a b
and suppose A B  90o  t1t2  1. Since the abscissa of  (m1 m2 ) x am1 bm2 0
m1 m2
the point of intersection of the tangent at t1 and t2 is
at1t2  a, the point of intersection lies on the line x  a.  (m1 m2 ) x am1 am2 bm2 bm1 (& m1 m2  1)
 (m1 m2 ) x a(m1 m2 ) b(m1 m2 )  0
31. A pair of tangents is drawn to a parabola such that
the sum of the angles made by them with the x-axis is Therefore, the point of intersection of the tangent lies on
constant. Prove that the point lies on a line through the line x a b  0.
the focus.
33. In a parabola, prove that the tangents drawn at the
extremities of a chord intersect on the diameter bi-
Solution: Let the tangents at t1 and t2 intersect at (x1,
secting the chord.
y1) so that
x1  at1t2 (4.61a) Solution: Let P(at12 , 2at1 ) and Q(at22 , 2at2 ) be two
points on the parabola y2  4ax. Therefore, the point of
and y1  a(t1 t2) (4.61b) the intersection of these tangents is [at1t2 , a(t1 t2 )] and
1 1 the midpoint of the chord PQ is
Let tan A  at tan B  and suppose @ A  P, where
t1 t2 ¥a 2 2 2a(t1 t2 ) ´ ¥a 2 2 ´
P is constant, we have ¦§ (t1 , t2 ), µ¶  ¦§ (t1 t2 ), a(t1 t2 )µ¶
2 2 2
(1/t1 ) (1/t2 ) t t y1 /a Hence, the point of intersection of the tangents at P and
tan Q  tan(A B )   1 2 
1 [(1/t1 ) s (1/t2 )] (t1t2 ) 1 ( x1 /a) 1 Q lies on the diameter y  a(t1 t2) which bisects the
chord PQ.
244 Chapter 4 Parabola

34. If r1 and r2 are the lengths of two perpendicular which is of the form Y2  4X where X  x − 1 and Y 
chords passing through the vertex of the parabola y − 1 and a  1. Therefore, the vertex is (1, 1), focus at
y2  4ax, then show that (X  1, Y  0) is
16a 2 (r12 / 3 r22 / 3 )  (r1r2 )4 / 3 ( x 1  1, y 1  0)  (2, 0)
The directrix equation is X 1  0 or x  0. Every point
Solution: See Fig. 4.21. Let OP and OQ be perpendicu- on the given parabola is of the form
lar chords such that OP  r1 and OQ  r2. Suppose OP
makes angle P with the axis so that OQ makes 90° – ( X  t 2 , Y  2t )  (1 t 2 , 1 2t )
P with the axis. Therefore, P  (r1cos P, r1sin P) and Q 
Hence, the equation of the tangent at P(1 t 2 , 1 2t ) is
(r2sin P, r2cos P). Therefore,
t( y 1)  ( x 1) t 2 (4.62)
r12 sin 2 Q  4a(r1 cos Q )
Since the directrix is x  0, by substituting x  0 in Eq.
4a cos Q (4.62), we have
 r1 
sin 2 Q
¥ t 2 t 1´
Similarly, Q  ¦ 0, µ
§ t ¶
4a sin Q
r2 
cos2 Q Therefore, we have

¨ cos2 / 3 Q sin 2 / 3 Q · ( 4 a) 2 / 3 ¥ t 2 t 1´
 r12 / 3 r22 / 3  (4a)2 / 3 © 4 / 3  P  (1 t 2, 1 2t ) and Q  ¦ 0,
4/3 ¸ 4/3 t µ
ª sin Q cos Q ¹ (sin Q cos Q ) § ¶
Let R be (x1, y1) and PR:RQ  1:−2 (see Fig. 4.22.) There-
(16a 2 )(16a 2 )1/ 3 (16a 2 )4 / 3

 16a 2 r12 / 3 r22 / 3 
(sin Q cos Q )4 / 3

(sin Q cos Q )4 / 3
fore

2(1 t 2 ) 1(0)
Also x1   2(1 t 2 ) (4.63)
2 1
4/3 4/3
¥ 4a cos Q 4a sin Q ´ ¥ 16a 2 ´ 2(1 2t ) [(t 2 t 1)/t ]
(r1r2 )4 / 3  ¦ s µ ¦ µ and y1 
§ sin Q cos2 Q ¶ § sin Q cos Q ¶ 2 1

 16a 2 (r12 / 3 r22 / 3 )  (r1r2 )4 / 3 3t 2 t 1


 (4.64)
t
y
P From Eq. (4.64), we have

ty1  3t 2 t 1  3(1 t 2 ) t 2
r1
From Eq. (4.63), we get
q
3 x1
O 90°− q x ty1  t 2
2
r2
3x 4
 t( y1 , 1)  1
2
Q 3 x1 4
t
2( y1 1)
FIGURE 4.21
Substituting the value of t in Eq. (4.63), we have
35. The tangent at a point P to the parabola − y2 ¨ ¥ 3x 4 ´ 2 ·
2y − 4x 5  0 intersects the directrix at Q. Find 2
x1  2[1 t ]  2 ©1 ¦ 1
µ ¸
the locus of the point R which divides the chord PQ ©ª § 2( y1 1) ¶ ¸¹
externally in the ratio 1/ 2 : 1. (IIT-JEE 2004)
 4 x1 ( y1 1)2  2 ¨ª 4( y1 1)2 (3x1 4)2 ·¹

Solution: The given parabola equation can be written as  2 x1 ( y1 1)2  4( y1 1)2 (3x1 4)2
 2( x1 2)( y1 1)2  (3 x1 4)2
( y 1)2  4( x 1)
4.2 Parabola 245

Therefore, the locus of R(x1, x2) is 2


4 x1 ¥ 8´ 64
  ¦ y1 µ 8
2( x 2)( y 1)2  (3 x 4)2 9 § 9¶ 9
2
1 −2 4 x1 ¥ 8´ 8
  ¦ y1 µ
P Q R 9 § 9 ¶ 9
2
FIGURE 4.22 ¥ 8´ 4
 ¦ y1 µ  ( x1 2)
§ 9¶ 9
36. Show that the locus of the point that divides a chord,
whose slope is 2, of the parabola y2  4x internally Hence, the locus of (x1, y1) is the parabola
in the ratio 1:2 is a parabola. Find the vertex of this 2
¥ 8´ 4
parabola. (IIT-JEE 1995) ¦§ y µ¶  ( x 2)
9 9
Solution: Let P(t12 , 2t1 ) and Q(t22 , 2t2 ) be the ends of whose vertex is (2, 8 / 9).
a chord whose slope is 2. Therefore
1 2
2 P R Q
2
t1 t2
FIGURE 4.23
 t1 t2  1 (4.65)
37. The angle between a pair of tangents drawn from
Let R (x1, y1) divide PQ internally in the ratio 1:2 (see point P to the parabola y2  4ax is 45o. Show that
Fig. 4.23). Hence the locus of point P is a hyperbola. (IIT-JEE 1998)
3 x1  2t12 t22 (4.66)
Solution: From Problem 28, the locus of the point P is
and 3 y1  4t1 2t2  2(t1 t2 ) 2t1
cot 2 45o( y2 4ax)  ( x a)2
Therefore, from Eq. (4.65), we get
3 y1  2 2t1  ( y2 4ax)  ( x a)2

Therefore,  x 2 6ax a 2 y2  0
 (x 3a)2 − 8a2 − y2  0
3y 2
t1  1 ( x 3a)2 y2
2  1
8a 2 8a 2
3 y1 2 4 3 y1
and t2  1 t1  1  which is a hyperbola, in fact, a rectangular hyperbola.
2 2
Substituting the values of t1 and t2 in Eq. (4.66), we have 38. Suppose the normals at three different points on the
2 2 parabola y2  4x pass through the point (h, 0). Show that
¥ 3y 2 ´ ¥ 4 3 y1 ´ h  2. (IIT-JEE 1984)
3 x1  2 ¦ 1 ¦
§ 2 µ¶ § 2 µ¶
Solution: Let t1, t2 and t3 be the parameters of the feet of
9 y12 12 y1 4 16 24 y1 9 y12
 the normals drawn from (h, 0). Hence, t1, t2 and t3 are the
2 4 roots of the cubic equation t 3 (2 h)t  0 (see Theorem
27 y12 48 y1 24 4.20). That is, t1, t2 and t3 are the roots of t(t 2 2 h)  0.

4 Since the equation has three roots, h must be greater
than 2.
 12 x1  27 y12 48 y1 24
 4 x1  9 y12 16 y1 8 39. Three normals with slopes m1, m2 and m3 are drawn
¥ 16 8´ from point P, which is not located on the axis of the
 4 x1  9 ¦ y12 y1 µ parabola y2  4x. If m1m2  @ results in the locus of P
§ 9 9¶
being part of the parabola, then find the value of @.
¨¥ 2
8´ 8 64 · (IIT-JEE 2003)
 4 x1  9 ©¦ y1 µ ¸
©ª§ 9¶ 9 81 ¸¹
Solution: Let P  (h, k ), where(k x 0), and (tr2 , 2tr ),
where r  1, 2 and 3, be the points on the parabola at
246 Chapter 4 Parabola

which the normals pass through P(h, k). Since the equa- 4 4 2t 2
tion of normal at (at2, 2at) to the parabola y2  4ax is tx t and 1 
y1 x1 1
y  2at at3, the slope of the normal is –t. We have m1 
−t1, m2  −t2 and m3  −t3, so by hypothesis we get ¥ 16 ´
 ( x1 1)  4 2 ¦ 2 µ
m1 m2  A § y1 ¶
 t1t2  A (4.67) 32
 x1 3 
y12
Since t1, t2 and t3 are the roots of
 y12 ( x1 3) 32  0
t3 (2 − h)t − k  0 (4.68)
we have Hence, the locus of (x1, y1) is

t1 t2 t3  0 y2 ( x 3) 32  0

t1t2 t2 t3 t3 t1  2 h 41. Find the locus of the point of intersection of perpen-


and t1t2 t3  k dicular normals drawn to the parabola x2  8y.
Since t1t2  @ [from Eq. (4.67)], we have
Solution: Let P(4t1 , 2t12 ) and Q(4t2 , 2t22 ) be the points
k on x2  8y. Equation of the normals at P and Q are
t3  (& t1 t2 t3  k )
A
1
Since t3 is a root of Eq. (4.68), we have y 2t12  ( x 4t12 ) (4.71)
t1
3
¥ k´ k 1
¦§ µ¶ (2 h) k  0 and y 2t22  ( x 4t22 ) (4.72)
A A t2
 k 2 A 2 (2 h) A 3  0 Since the normals are at right angles, we have
 k 2  A 2 (A 2 h) ¥ 1´ ¥ 1´
¦§ t µ¶ ¦§ t µ¶  1
Hence, the locus of (h, k) is the curve  y2 @ 2(x
@ − 2) 1 2
and this is identical with y2  4x only when @  2.
 t1t2  1 (4.73)
40. Show that the locus of the point of intersection of tan- Solving Eqs. (4.71) and (4.72) and from Eq. (4.73), we
gents drawn at the ends of normal chords to the parabola have
y2  8(x − 1) is y2(x 3) 32  0.
x  2t1t2 (t1 t2 )  2(t1 t2 )
Solution: The given parabola equation is Y2  8X where
Y  y and X  x − 1. Every point on this parabola is and y  2(t12 t1t2 t22 2)  2(t12 t22 1) (& t1t2  1)

(x  2t2, y  4t)  (2t2 1, 4t) ¨ x2 ·


 2[(t1 t2 )2 2t1t2 1]  2 © 3¸
ª 4 ¹
Normal at (2t2 1, 4t) is
tX Y  2(2t) 2t3 ¥ x´
¦§& t1t2  1 and t1 t2  µ¶
2
 t( x 1) y  4t 2t 3 (4.69)
 2 y  x 2 12
Suppose the tangents at the ends of normal chord inter-
sect at P(x1, y1). Then, the normal chord is the chord of which is the required locus.
contact of P(x1, y1) and hence its equation is
42. Two tangents of the parabola y2  8x meet the tan-
Yy1 4( X x1 )  0 gent at the vertex in P and Q. If PQ  4, prove that
 yy1 4( x 1 x1 )  0 the locus of the point of intersections of the tangents
is y2  8(x 2).
 yy1 4 x 4 4 x1  0 (4.70)
Equations (4.69) and (4.70) represent the same straight Solution: Let the two tangents be
line. Therefore
t1 y  x 2t12
t 1 4t 2t 3 and t2 y  x 2t22
 
4 y1 4 x1 4
4.2 Parabola 247

so that their point of intersection is [2t1t2 , 2(t1 t2 )]. 44. From the vertex of a parabola if a pair of chords
Now, P  (0, 2t1) and Q  (0, 2t2). Since PQ  4, we have is drawn at right angles to one another and with
these chords as adjacent sides a rectangle be
t1 t2  2 formed, prove that the locus of the fourth vertex is
y2  4a( x 8a).
Suppose x1  2t1t2 and y1  2(t1 t2). Therefore
Solution: Let y2  4ax be the parabola, P  (at12 , 2at1 )
y12 and Q  (at22 , 2at2 ) such that POQ  90o (see Fig. 4.24).
 (t1 t2 )2  (t1 t2 )2 4t1t2  4 2 x1
4 Now
Thus, the locus of (x1, y1) is ¥ 2at ´ ¥ 2at ´
POQ  90o  ¦ 21 µ ¦ 22 µ  1
y2  8(x 2) § at1 ¶ § at2 ¶

43. A chord of the parabola y2  4ax subtends right angle


 t1t2  4 (4.76)
at the vertex. Show that the locus of the point of in- Let R  (h, k) be the fourth vertex of the rectangle
tersection of the normals drawn at the extremities of POQR. Therefore (by Worked-Out Problem 10,
the chords is y2  16a(x 6a). Chapter 1), we have

Solution: Suppose Q(at12 , 2at1 ) at R(at22 , 2at2 ) are the h  at12 at22 0
extremities of a chord subtending right angle at the ver- and k  2at1 2at2 0
tex. Therefore
Therefore
Slope of QO s Slope of RO  − 1 (where O is the vertex)
2
¥ k´ 2 2 2 h
¥ 2at ´ ¥ 2at ´ ¦§ µ¶  (t1 t2 )  t1 t2 2t1t2  8 (& t1t2  4)
 ¦ 21 µ ¦ 22 µ  1 2a a
§ at1 ¶ § at2 ¶
 k 2  4a(h 8a)
 t1t2  4 (4.74)
Suppose (x1, y1) is the point of intersection of the nor- Hence, the locus of (h, k) is
mals drawn at points Q and R so that
y2  4a( x 8a)
x1  2a a(t12 t1t2 t22 ) y
and y1  at1t2 (t1 t2 )
Therefore, from Eq. (4.74), we get P

x1  2a a(t12 t22 4) (4.75a)


and y1  a( 4)(t1 t2 ) (4.75b) O x
R
which implies that
x1 2a 2 2 y1 Q
 t1 t2 and  t1 t2
a 4a
Therefore FIGURE 4.24

y12 x1 2a
 (t1 t2 )2  t12 t22 2t1t2  8 45. Let PSPa and QSQa be two local chords of the parab-
16a 2 a
ola y2  4ax where S is the focus. If the line PQ passes
(& t1t2  4) through a fixed point (@, A ), then show that PaQa also
passes through the fixed point (a 2 /A , aB /A )
 y12  16a( x1 6a)
Hence, the locus of the point of intersection of the nor- Solution: Let P  (at12 , 2at1 ) and Q  (at22 , 2at2 ) so that
mals drawn at the extremities of the chords subtending by Theorem 4.19, part (1), we have
right angle at the vertex is
¥ a 2a ´
y2  16a( x 6a) Pa  ¦ 2 , µ
§ t1 t1 ¶
248 Chapter 4 Parabola

2
¥ a 2a ´ ¥ p´ 2
and Qa  ¦ 2 , µ ¦§ x µ¶ 2 px  p
§ t2 t2 ¶ 2
Equation of PQ is 3 p2
 x 2 px 0
2 4
y 2at1  ( x at12 )
t1 t2  4 x 2 4 px 3 p2  0
 y(t1 t2 )  2 x 2at1t2  4 x 2 6 px 2 px 3 p2  0
Since this chord passes through (@, A ), we have  2 x(2 x 3 p) p(22 x 3 p)  0
B(t1 t2 )  2A 2at1t2 (4.77) p 3 p
x ,
The equation of line PaQa is 2 2
¥ 1 1 ´ ¥ 1´ ¥ 1´ Since p  0 and (x, y) a point on the parabola, x cannot be
y ¦ µ  2 x 2a ¦ µ ¦ µ
§ t1 t2 ¶ § t1 ¶ § t2 ¶ negative. Hence
 (t1 t2 ) y  (2t1t2 ) x 2a p
x and y  p p
2
Now, PaQa passes through (a2/@, aA/@)
Thus, the points of intersection are ( p / 2, p) and ( p / 2, p).
¥ aB ´ ¥ a2 ´
š (t1 t2 ) ¦ µ  (2t1t2 ) ¦ µ 2a
§ A ¶ §A¶ 47. The normals at P and Q on a parabola y2  4ax inter-
sect at a point R on the curve. If M is the midpoint of
š (t1 t2 )aB  2a 2 t1t2 2aA PQ and N is the midpoint of MR, then show that the
š B(t1 t2 )  2at1t2 2A locus of the point N is a parabola.

which is true according to Eq. (4.77). Thus PaQa passes Solution: Let P  (at12 , 2at1 ), Q  (at22 , 2at2 ) and R  (at32 ,
through (a2/@, aA/@). 2at3). Hence, by Theorem 4.12, we have

46. Circle with centre at the focus of the parabola y2  2 2


t1  t3  t2
2px touches the directrix of the parabola. Show that t1 t2
the points of intersection of the circle and the pa-
 t1t2  2 (4.79)
rabola are ( p / 2, p) and ( p / 2, p). (IIT-JEE 1995)
Let M(x1, y1) be the midpoint of PQ. Therefore
Solution: Focus S  ( p / 2, 0). Let the radius be r so
that the equation of the circle is 2 x1  a(t12 t22 ) (4.80a)

2 and y1  a(t1 t2 ) (4.80b)


¥ p´ 2 2
¦§ x µ y r By Theorem 4.11, if R  (h, k), then

This touches the directrix. So h  2a a(t12 t1t2 t22 )

p p and k  at1t2 (t1 t2 )


r
2 2
Therefore, from Eq. (4.79), we get
šr p
h  4a a(t12 t22 ) (4.81a)
Hence, the equation of the circle is
and k  2a(t1 t2 ) (4.81b)
2
¥ p´ 2 2
¦§ x µ y  p (4.78) Let (@, A ) be the midpoint of MR. Therefore,

2A  x1 h and 2B  y1 k
The abscissa of the point of intersection of the circle Q
with the parabola are the roots of which are written [from Eqs. (4.80a), (4.80b), (4.81a) and
(4.81c)] as follows:
Worked-Out Problems 249

49. If P(x1, y1), Q(x2, y2) and R(x3, y3) are three points
a(t12 t22 )
2A  4a a(t12 t22 ) on y2  4ax and the normals at P, Q and R meet at a
2 point, then prove that
3a 2 2
A  (t1 t2 ) 2a x1 x2 x2 x3 x3 x1
4 0
y3 y1 y2
and 2B  a(t1 t2 ) 2a(t1 t2 )
a Solution: Suppose xr  atr2 and yr  2atr , for r  1, 2
B  (t1 t2 )
2 and 3. The normals at P, Q and R meet at point A(h, k).
Hence, by Theorem 4.20 and Quick Look 11, t1, t2 and t3
a2 a 2 ¨ 4A 8a ·
Now B2  (t1 t2 )2  4¸ are the roots of the equation
4 4 ©ª 3a ¹
at 3 (2a h) t k  0
a
 [A a] (& t1t2  2)
3 Also t1 t2 t3  0
a 2a h
 ( x a) t1t2 t2 t3 t3 t1 
3 a
k
Therefore, the locus of the midpoint (@, A ) of MR is y2  t1t2 t3 
a
a
(@ 2a).
4 Now,

48. Prove that the locus of the midpoint of the portion of x1 x2 a(t12 t22 ) (t1 t2 )(t1 t2 ) t3 (t1 t2 )
a normal to y2  4ax intercepted between the curve   
y3 2at3 2 t3 2
and the axis is another parabola.
(& t1 t2 t3  0)
Solution: Normal at P(at2, 2at) is
Similarly,
tx y  2at at3
x2 x3 t1 (t2 t2 )
Substituting y  0 in the above equation, we have 
y1 2
x  2a at2
x3 x1 t2 (t3 t1 )
Hence, the normal meets the axis at N(2a at2,0). Let (@, and 
y2 2
A ) be the midpoint of PN so that @  a at2 and A  at.
Therefore Hence
2
A a 2 ¥ B´ x1 x2 x2 x3 x3 x1 1
t ¦ µ  [t3 (t1 t2 ) t1 (t2 t3 )
a § a¶ y3 y1 y2 2
 B 2  a(A a) t2 (t3 t1 )]
Hence, the locus of the midpoint of PN is 1
 (0)  0
2
y2  a(x a)
which is a parabola.

WORKED-OUT PROBLEMS
Single Correct Choice Type Questions
1. L is the normal to the parabola y2  4x and passes Solution: Observe that (9, 6) lies on the parabola
through the point (9, 6). If the slope of the normal is y2  4x. Normal at (t2, 2t) is tx y  2t t3 which passes
positive, then its equation is through (9, 6)(t  3); therefore, we have
(A) y x 3  0 (B) y 3x 33  0 3 x y  6 27
(C) y 2x 12  0 (D) y x 15  0  3 x y 33  0
(IIT-JEE 2011)
250 Chapter 4 Parabola

Aliter (Using Calculus): Differentiating y2  4x with 1


respect to x, we get (A) 1 (B) 2 (C) (D) 4
2

WORKED-OUT PROBLEMS
dy 2 Solution: According to Problem 18 of the section ‘Sub-

dx y jective Problems’, the midpoints lie on a parabola whose
latus rectum is half of the latus rectum of the original
Therefore parabola. Here, the latus rectum of the original parabola is
¥ dy ´ 2 1 8 and hence the latus rectum of the required parabola is 4.
¦§ µ¶  
dx (9,6) 6 3 Answer: (D)

Hence, equation of the normal at (9, 6) is 5. If x y  k is a normal to the parabola y2  12x, then
the value of k is
y 6  3( x 9)
 y 3 x 33  0 (A) 3 (B) (C) 9 (D) 3
Answer: (B) (IIT-JEE 2000)

2. Let S be the focus of the parabola y2  8x and PQ be


Solution: We have y2  12x  4(3), x  4ax where a 
the common chord of the circle x 2 y2 2 x 4 y  0 3. Every point on the parabola is of the form (3t2, 6t).
and the given parabola. Then the area of $PQS is Suppose x y  k is normal at (3t2, 6t). Equation of the
normal at (3t2, 6t) is tx y  6t 3t 3 . Therefore
(A) 8 (B) 5 (C) 4 (D) 10
t 1 6t 3t 3
(IIT-JEE 2012)  
1 1 k
Solution: Every point on y2
 8x is of the form (2t2,
2  t  1 and k  6 3  9
4t). Substituting x  2t and y  4t in the equation of the
circle, we have Answer: (C)

4t 4 16t 2 4t 2 16t  0 6. If the line x 1  0 is the directrix of the parabola


3
 t(t 3t 4)  0 y2 kx 8  0, then one of the values of k is

 t(t 1)(t 2 t 4)  0 (A)


1
(B) 8 (C) 4 (D)
1
8 4
Since t2 t 4  0 has normal roots, the values of t are 0
and 1. Hence, the common chord end points are P(0, 0) (IIT-JEE 2000)
and Q(2, 4) and the focus is S(2, 0). Hence, the area of
Solution: The given parabola equation can be written
$PQS is
as
1
4s2 0  4 ¥ 8´
2 y2 k ¦ x µ , Y 2  k( X )
§ k¶
Answer: (C)
where
3. The orthocentre of a triangle formed by three tan-
8
gents to a parabola y2  8x lies on X  x and Y  y
k
(A) x 1  0 (B) x 2  0
Therefore
(C) x 4  0 (D) x 1  0
¥ k´
Solution: Orthocentre of a triangle formed by three Y 2  kX  4 ¦ µ X
§ 4¶
tangents to the parabola y2  4ax lies on the directrix x
a  0 (see Problem 4 in the section ‘Subjective Problems’). so that a  k/4. The equation of the directrix is
Here, a  2 gives that the orthocentre lies on the line x
2  0. ¥ 8 k´
X a  0 x ¦ µ  0
§ k 4¶
Answer: (B)
Now x – 1  0 is the directrix. This implies
4. The midpoints of the chords of the parabola y2  8x
which subtend right angle at the vertex lie on the 8 k
1
curve y2  kx, where k is equal to k 4
Worked-Out Problems 251

 32 k 2  4k x  3 2 2  y  2 2 2
 k 2 4k 32  0 Let A  (3 2 2 , 2 2 2 ) and B  (3 2 2 , 2 2 2 ). By
 (k 8)(k 4)  0 hypothesis, the normals at A, B and D meet at C. Hence,
 k  4 or 8 by Quick Look 11, the algebraic sum of the ordinates of
A, B and D is zero. This implies
Therefore, one of the values of k is 4.
2 2 2 2 2 2 y3  0
Answer: (C)
Here D  ( x3 , y3 ).
7. A normal at P(at12, 2at1) meets the curve again at
Now
Q(at22 , 2at2 ). If PQ subtends a right angle at the
vertex, then 4 x  16  x  4
(A) t12 2 (B) t22 2 Hence, the required point is (4,−4).
(C) t1  2t2 (D) t2  2t1 Answer: (B)
Solution: From Theorem 4.12, we have
9. Let P be the angle of intersection of the parabolas
2 y2  8ax and x2  27ay at the point other than the
t2  t1 origin. Then, tan P is equal to
t1
8 13 13 9
2 (A) (B) (C) (D)
or t1 t2  (4.82) 13 8 9 13
t1
Solution:
Also POQ  90o. This implies
y4  64a 2 x 2  64a 2 (27ay)  64 s 27a 3 y
¥ 2at1 ´ ¥ 2at2 ´
¦ 2 µ ¦ 2 µ  1  y( y3 64 s 27 s a 3 )  0
§ at1 ¶ § at2 ¶
 y  0, y  12a
 t1t2  4 (4.83)
From Eq. (4.82), we have The point of intersection of the parabolas other than the
origin is (18a, 12a). The equation of the tangent to y2 
t12 t1t2  2 8ax at (18a, 12a) is
which can written as [from Eq. (4.83)] y(12a) 4a(x 18a)  0
t12 4  2  3y x 18a  0
 t12 2 The slope is 1/3. The equation of the tangent to x2 27ay
Answer: (A) at (18a, 12a) is
27
8. The line x y 1  0 meets the parabola y2  4x at x(18a) a( y 12a)  0
2
A and B. Normals at A and B meet at C. If D is a point
on the curve such that CD is normal at D, then the  4 x 3 y 12a  0
coordinates of D are whose slope is 4/3. Hence
(A) (4, 4) (B) (4, −4) 4 / 3 1/ 3 9
tan Q  
(C) (2, −4) (D) (2, 4) 1 4 / 9 13
Solution: Substituting y  x 1 in y2  4x we get Aliter (Using Calculus): Differentiating y2  8ax on both
sides with respect to x, we get
x2 6x 1  0
whose roots are dy 4a

dx y
6 p 36 4
 3p 2 2 so that
2
Now ¥ dy ´ 4a 1
¦§ µ¶    m1 (say)
dx (18 a,12 a) 12a 3
x  3 2 2  y  x 1  2 2 2
252 Chapter 4 Parabola

When x2  27ay, we have Relying upon Quick Look 11 is always useful in this
case.
¥ dy ´ 2 s 18a 4
¦§ µ¶    m2
dx (18 a,12 a) 27a 3 11. The length of the normal chord of the parabola y2  4x
which makes an angle P /4 with the positive direction
Hence
of the axis is
m1 m2 1/ 3 4 / 3 9 (A) 4 2 (B) 8 2 (C) 8 (D) 4
tan Q   
1 m1 m2 1 4 / 4 13
Solution: Slope m of the chord is tan P /4  1. By
Answer: (D) Quick Look 7, the normal chord equation is y  x 2 1.
That is, y  x 3. Substituting this value in y2  4x we get
10. From the point (15, 12), three normals are drawn
to the parabola y2  4x. The centroid of the triangle (x − 3)2  4x
formed by the feet of the normals is  x2 − 10x 9  0
¥ 26 ´ Solving this, we get x  1 and 9 and so y  −2 and 6. That
(A) ¦ , 0µ (B) (4, 0)
§ 3 ¶ is, the ends of the normal chord are (1, − 2) and (9, 6).
Hence, the length of the normal chord is
¥ 16 ´
(C) ¦ , 0µ (D) (16, 0)
§ 3 ¶ (9 1)2 (6 2)2  8 2
Solution: Let P(t12 , 2t1 ), Q(t22 , 2t2 ) and R(t32 , 2t3 ) be the Answer: (B)
feet of the normals drawn from (15,12). Hence, by Theo-
rem 4.2 and Quick Look 11, t1, t2, t3 are the roots of 12. If 2x y k  0 is a normal to the parabola y2  − 8x,
then the value of k is
t3 13t 12  0 (4.84)
(A) 12 (B) 24 (C) − 12 (D) − 24
so that t1 t2 t3  0, t1t2 t2 t3 t3 t1  13, and t1t2t3 
12. Let G(x1, y1) be the centroid of $PQR. Therefore Solution: Suppose 2x y k  0 is normal at ( 2t 2 , 4t ).
However, the normal at ( 2t 2 , 4t ) is
t12 t22 t32
x1  tx y  4t 2t 3
3
2t 2t2 2t3  tx y 4t 2t3  0
and y1  1
3 Therefore
Clearly, t 1 4t 2t 3
 
y1  0 (& t1 t2 t3  0) 2 1 k
 t  2 and k  24
Now,
Answer: (B)
1
x1  [(t1 t2 t3 )2 2
3 ¤t t ]
1 2
13. The centre of the circle which cuts the parabola y2 
4x orthogonally at the point (1, 2) passes through the
1 26
 [0 2( 13)]  point
3 3
(A) (5, 3), (B) (3, 4), (C) (2, 4) (D) (4, 3)
Thus G is
Solution: Since the circle cuts the parabola y2  4x or-
¥ 26 ´ thogonally at (1, 2), the tangent to the parabola at (1, 2)
( x1 , y1 )  ¦ , 0µ
§ 3 ¶ should pass through the centre of the circle. That is,
Answer: (A) y(2) − 2(x 1)  0 passes through the centre. That is, y 
x 1 passes through (3, 4).
Note:
Answer: (B)
1. It can be observed that t  1, −3 and 4 are the roots
of Eq. (4.84) so that the feet of the normals are (1, −2), 14. If a normal chord of y2  4ax subtends a right angle
(9, −6), (16, 8) and hence the centroid is (26 / 3, 0). at the vertex, then its slope is
2. In general, the cubic equation in t similar to the 1 1
one provided in Eq. (4.84) is not so easy to solve. (A) 2 (B) (C) (D) 2
2 2
Worked-Out Problems 253

Solution: The equation of the normal with slope m is Therefore


y − mx  − 2am − am3 (4.85) a 7 2a 14
 s 16  28;  s 4  14
Suppose this chord meets the parabola at P and Q. There- m2 4 m 4
fore, by Theorem 2.33, the combined equation of the pair Hence, (x1, y1)  (28, 14) so that x1 y1  42.
of lines OP and OQ is Answer: (C)
¥ mx y ´
y2 4ax ¦ 0 17. If L y x y 1  0 is a line and S y y x x 2  0 is
§ 2am am3 µ¶
a parabola, then which of the following is true?
Now (A) L  0 and S  0 do not have common points
POQ  90o (B) L  0 cuts S  0 in two distinct points
(C) L  0 touches the parabola S  0
 Coefficient of x2 Coefficient y2
(in the above equation )  0 (D) L  0 is the directrix of the parabola S  0

4 ma Solution: Substituting y  1 x in the equation of the


 1 0 parabola, we get
2am am3
 m2 2 4  0 1 x x x2  0
 x2 2x 1  0
mp 2
Answer: (A)  ( x 1)2  0
Hence, L  0 touches the parabola at (1, 1).
15. If the line lx my n  0 touches the parabola y2  4ax, Answer: (C)
then
(A) ln  am (B) ln2  am 18. If c  1/2, then three normals can be drawn from the
point (c, 0) to the parabola y2  x out of which one
(C) l2 n  am2 (D) ln  am2 is the x-axis. If the other two normals are at right
Solution: Suppose lx my n  0 touches the parabola angles, then the value of c is
y2  4ax at (at2, 2at). However, ty  x at2 is the equation 4 3 2 3
(A) (B) (C) (D)
of the tangent at (at2, 2at). Therefore 3 4 3 2
l m n Solution: The normal at (at2, 2at), where a  1/4, which
 
l t at 2 is drawn to y2  x is
m n ¥ 1´ 1
 t tx y  2 ¦ µ t t 3
l ma § 4¶ 4
 am2  ln  4tx 4 y  2t t 3
Answer: (D) This passes through (c, 0). So we get

16. If (x1, y1) is the point of contact of the tangent paral- 4tc  2t t 3
lel to the line 4y x 3  0 and touching the parabola  t (t 2 2 4 c )  0
y2  7x, then x1 y1 is equal to
¥ 1 ´
 t  0 or t  p 4c 2 ¦§& c   4c 2  0µ¶
(A) 40 (B) 28 (C) 42 (D) 32 2
Solution: The line It is clear that t  0 gives that x-axis is the normal at (0, 0).
The other two normals are at right angles š product of
a
y  mx their slopes  −1.
m
That is
touches the parabola y2  4ax at (a /m2 , 2a /m) (see Theo-
rem 4.7). Here 4c 2 ( 4c 2)  1

1 7  4c 2  1
m ,a
4 4 3
c
4
Answer: (B)
254 Chapter 4 Parabola

19. If P is a variable point on the parabola y2  4ax whose Solution: Let P  (at2, 2at), where a  4, be a point on
focus is S, then the locus of the midpoint of the seg- the parabola y2  16x so that S  (a, 0)  (4, 0) is the focus.
ment SP is in turn a parabola whose directrix is Equation of SP is
a 8t
(A) x  (B) x  a y ( x 4)
2 2
4(t 1)
(C) x  0 (D) x  a
 2tx (t 2 1) y 8t  0 (4.86)
Solution: P  (at 2,
2at), t Z and S  (a, 0). Let M(x1,
y1) be the midpoint of SP. Then and It touches the circle (x 6)2 y2  2 which implies that

a(1 t 2 ) 2 t ( 6) ( 8 t )
x1   2
2 4t 2 (t 2 1)2
2at  4t  2 (t 2 1)
and y1   at
2
 t 2 2 2t 1  0
Therefore
 t 2 2 2t 1  0
2
2 x1 ¥y ´ 2 2 p 8 4
1  t2  ¦ 1 µ t  2 p1
a § a¶ 2
 y12  2ax1 a 2 Hence the slope of SP is

¥ a´ 2t 2( 2 p 1) 2( 2 p 1)
 y12  2a ¦ x1 µ 2
   p1
§ 2¶ t 1 (3 p 2 2 ) 1 2p2 2
Hence, the locus of (x1, y1) is the parabola That is, the chord y  p(x 4) touches the circle (x 6)2
y2  2.
¥ a´
y 2  2a ¦ x µ Aliter: Let y  m(x 4) be a focal chord of y2  16x. This
§ 2¶
chord touches the circle (x 6)2 y2  2. This implies
Substituting Y  y and X  x (a / 2) so that
6m 4m
 2
¥ a´ m2 1
y 2  2a ¦ x µ
§ 2¶  4 m2  2(m2 1)
is given by  m2  1
 m  p1
¥ a´
Y2  4¦ µ X Answer: (A)
§ 2¶

whose directrix is 21. The equation of the common tangent touching the
circle ( x 3)2 y2  9 and the parabola y2  4x
a above the x-axis is
X
2
(A) 3 y  3x 1 (B) 3 y  ( x 3)
a a
 x 
2 2 (C) 3y  x 3 (D) 3 y  (3 x 1)
x0 Solution: The line
Answer: (C) 1
y  mx
m
20. A focal chord of the parabola y2  16x touches the
circle ( x 6)2 y2  2. Then the possible values of touches y2  4x for all values of m x 0. This line also
the slope of this chord are touches the circle (x 3)2 y2  9. So we have
(A) { 1, 1} (B) { 2, 2} 1
m(3)
1º 1º m
« « 3
(C) ¬ 2, » (D) ¬2, »
­ 2¼ ­ 2¼ m2 1
Worked-Out Problems 255

2 Then the locus of the point of intersection of those


¥ 1´
 ¦ 3m µ  9(m2 1) normals is
§ m¶
1 (A) ( x y)2  2a( x y) (B) ( x y)2  2a( x y)
2 2
 9m 6  9m 9
m2 (C) ( x y)2  4a( x y) (D) ( x y)2  4a( x y)
1 Solution: Consider the common focus as the origin and
 3
m2 their axes as coordinate axes. We can suppose that the
1 parabolas are y2  4a(x a) and x2  4a(y a). Therefore,
mp
3 by Quick Look 7, their normals in terms of the slopes
are
Since the tangent is above x-axis, we have
y  m( x a) 2am am3 (4.87)
1 3
m and x  ma( y a) 2ama ama (4.88)
3
Since the normals are at right angles, we have
Therefore, the tangent equation is given by
¥ 1´
x m ¦ µ  1  ma  m
y 3 § ma ¶
3
Substituting the value of main Eq. (4.88), we have
 3y  x 3
x  m( y a) 2am am3 (4.89)
Answer: (C)
Therefore, from Eqs. (4.87) and (4.89), we get
22. The equation of a line inclined at an angle of to 45o x y
m
the line y  3x 5 and touching the parabola y2  8x x y
is
Substituting the value of m in Eq. (4.87), we obtain the
(A) y  2x 1 (B) 2x y 1  0 locus as
(C) x 2y 1  0 (D) x 2y 1  0 3
¥ x y´ ¥ x y´ ¥ x y´
Solution: Given y¦ µ ( x a) 2 a ¦ µ a¦
§ x y¶ § x y¶ § x y µ¶
y2  8x  4(2)x
 y( x y)3  ( x y)[( x a)( x y)2 2a( x y)2
so that a  2. Therefore, the line
a( x y)2 ]
2
y2  mx  ( x y)[( x y)2 ( x a) a( x y)2 ]
m
 ( x y)[ x( x y)2 a[( x y)2 ( x y)2 ]]
touches the parabola for all values of m x 0. The angle
between this tangent and the line y  3x 5 is 45°. So  ( x y)[ x( x y)2 2a( x 2 y2 )]

m 3  2a( x y)( x 2 y2 )  x( x y)( x y)2 y( x y)3


 tan 45o  1
1 3m  ( x y)2 [ x 2 xy xy y2 ]
 m 3  p(1 3m)  ( x y)2 ( x 2 y2 )
1
 m  2,  ( x y)2  2a( x y)
2
Therefore, m  2. So the tangent is Answer: (A)

y  2x 1 24. If P is a point on the line x 4a  0 and QR is the


 2x y 1  0 chord of contact of P with respect to y2  4ax, then
Answer: (B) QOR (where O is the vertex) is equal to
(A) 45o (B) 60o (C) 30o (D) 90o
23. Two equal parabolas have the same focus and their
axes are at right angles. A normal to one parabola Solution: Let P be (x1, y1) so that
is perpendicular to a normal to the other parabola. x1 4a  0 (4.90)
256 Chapter 4 Parabola

Chord of contact of P( x1 , y1 ) is (A) a pair of straight lines (B) an ellipse


yy1 2a( x x1 )  0 (C) a parabola (D) a hyperbola

 yy1 2ax 8a 2  0 [& x1  4a from Eq. (4.90)] Solution: By the two parametric equations we have

2ax y1 y x y  2t
 2
1
8a x y
t
Hence, the combined equation of the pair of lines OQ 2
and OR is
Substituting the value of t in x  t2 t 1, we have
¥ 2ax y1 y ´
y2 4ax ¦ ¥ x y´
2
¥ x y´
§ 8a 2 µ¶ x¦ ¦ 1
§ 2 µ¶ § 2 µ¶
In this equation, the coefficient of x2 the coefficient of
 4 x  ( x y)2 2( x y) 4
y2  −1 1  0. Hence, QOR  90o.
Answer: (D)  ( x y)2  2( x y 2)

which is a parabola.
25. Locus of the point of intersection of the tangents to
the parabola y2  4ax which include 60oangle is Answer: (C)

(A) y2  4ax 3( x a)2 (B) y2 4ax  2( x a)2 28. Consider the following statements:
(C) 2
y 4ax  ( x a) 2 2
(D) y 4ax  3( x a) 3 I: A parabola is symmetric about its axis.

Solution: According to Problem 28 of the section ‘Subjec- II: The curve


tive Problems’, the locus of the point of intersection of the
tangents to the parabola y2  4ax which include angle @ is x2
y x 1
2
cot 2 A ( y2 4ax)  ( x a)2
is symmetric with respect to the line x  1.
Now, substituting @  60o the required locus is Then which one of the following is true?
1 (A) Both I and II are true
( y 4ax)  ( x a)2
3 (B) Both I and II are false
 y2 4ax  3( x a)2 (C) I is true and II is false
Answer: (A) (D) I is false and II is true
Solution: y2  4ax is symmetric about the x-axis which
26. The equation of the circle with centre at the focus of
is its axis. Therefore, I is true. Now,
the parabola 8 y  ( x 1)2 and touching the parabola
at the vertex is x2
y x 1
(A) x 2 y2 4 y  0 2
 2 y  x2 2x 1
(B) x 2 y2 4 y 1  0
 2 y  ( x 1)2 2( x 1)2  2( y 1)
(C) x 2 y2 2 x 4 y  0
 X 2  2Y
(D) x 2 y2 2 x 4 y 1  0
where X  x – 1 and Y  y – 1. This implies that the curve
Solution: The parabola is X 2  4(2)Y where is symmetric about X  0 or x  1. Therefore, II is also
X  x 1, Y  y, and a  2. Therefore, the vertex  (1, true.
0) and the focus  (1, 2). Also the radius of the circle is
Answer: (A)
equal to 2. Hence, the circle equation is

( x 1)2 ( y 2)2  2 2 29. The point of intersection of the tangents drawn at


the ends of the latus rectum of the parabola y2  4x
 x 2 y2 2 x 4 y 1  0 is
Answer: (D)
(A) ( 1, 0) (B) ( 1, 1)
27. The curve described parametrically by x  t2 t 1 (C) ( 1, 2) (D) ( 1, 2)
and y  t2 t 1 represents
Worked-Out Problems 257

Solution: The ends of the latus rectum are (1, 2) and Solution: Let Q  (at12 , 2at1 ) and R  (at22 , 2at2 ). Since
(1, 2). Therefore, the tangents at these points are the normals at Q and R meet the curve again at P, we have
2y 2(x 1)  0  y  x 1 2 2
t1  t  t2 and t1t2  2 (4.91)
2 y 2( x 1)  0  y  x 1 t1 t2
Now,
Hence, the point of intersection is ( 1, 0).
Answer: (A) ( PQ)2  a 2 (t 2 t12 )2 4a 2 (t t1 )2
 a 2 (t t1 )2 [(t t1 )2 4]
30. If P, Q and R are three points on the parabola y2 
4ax at which the normals intersect at the point (h, k),  a 2 [(t t1 )2 4tt1 ][(t t1 )2 4]
then the centroid of PQR is ¨4 ·¨ 4 ·
 a 2 © 2 4tt1 ¸ © 2 4 ¸
1 4 ª t1 ¹ ª t1 ¹
(A) (h 2a, 0) (B) (h 2a, 0)
3 3
¥1 ´¥ 1 ´
2 ¥ 2a h ´  16a 2 ¦ 2 tt1 µ ¦ 2 1µ
(C) (h 2a, 0) (D) ¦ , 0µ § t1 ¶ § t1 ¶
3 § 3 ¶
Therefore,
Solution: Let P  (at12 , 2at1 ), Q  (at22 , 2at2 ) and R 
(at32 , 2at3 ). Therefore, from Theorem 4.20, t1, t2 and t3 are ¥1 ´¥ 1 ´
the roots of the equation ( PQ)2  16a 2 ¦ 2 t12 2µ ¦ 2 1µ
§ t1 ¶ § t1 ¶
at 3 (2a h)t k  0 ¨ 2 ·
©& t1  t from Eq . (4.991)¸
so that ª t1 ¹
t1 t2 t3  0 (4.90a) 16a 2 (1 t12 )3

2a h t14
t1t2 t2 t3 t3 t1  (4.90b)
a Similarly,
k
t1t2 t3  (4.90c) 16a 2 (1 t22 )3
a ( PR)2 
t24
Let G  (x1, y1) be the centroid of PQR . Therefore
Therefore,
at 2 at22 at32 a
x1  1  [(t1 t2 t3 )2 2 ¤t t ] 16a 2 (1 t12 )3 (1 t22 )3
3 3
1 2 (PQ)2(PR)2 
(t1t2 )4
Therefore, from Eqs. (4.90a) and (4.90b), we get
16a 2 (1 t12 t22 t12 t22 )3

a¨ 2(2a h) · 2 (t1t2 )4
x1  0 ¸¹  3 (h 2a)
3 ©ª a
16 2 a 4 [1 (t1 t2 )2 2t1t2 t12 t22 ]3

2a (t1t2 )4
and y1  (t1 t2 t3 )  0
3
16 2 a 4 [1 t 2 4 4]3

Hence 24
¨2 · [from Eq. (4.91) and t t1 t2  0]
G  © (h 2a), 0 ¸
ª 3 ¹  16a 4 (1 t 2 )3
Answer: (C) Answer: (D)

31. Let P(at2, 2at) be a point on the parabola y2  4ax 32. The chords of a parabola y2  6x are passing through
and Q and R be the feet of the normals drawn from the point (9, 5). Then the middle points of these
P. Then (PQ)2 (PR)2 is equal to chords lie on the curve represented by the equation
(A) 16a 4 (1 t 2 )2 (B) 16a 2 (1 t 2 )3 (A) y2 5 y 3 x 27  0
4 2 3
(C) 64a (1 t ) (D) 16a 4 (1 t 2 )3 (B) y2 5 y 3 x 27  0
258 Chapter 4 Parabola

(C) y2 5 y 3 x 27  0 4ax 4ax


(C) y2  (D) y2 
a b 2a b
(D) y2 5 y 3 x 27  0
Solution: Equation of the chord of y2  4ax, whose
Solution: midpoint is (x1, y1), is
¥ 3´
y2  6 x  4 ¦ µ x S1  S11
§ 2¶
 yy1 2a( x x1 )  y12 4ax1
so that a  3/2. The equation of the chord with midpoint
(x1, y1) is 2ax y12 2ax1
y
S1  Sn y1 y1

¥ 3´ This chord touches the parabola y2  4ax. Therefore


 yy1 2 ¦ µ ( x x1 )  y12 6 x1
§ 2¶
y12 2ax1 b ¥ b ´
 ¦§& c  is the conditionµ¶
 yy1 3( x x1 )  y12 6 x1 y1 2a / y1 m

This chord passes through the point (9, 5). So For the line y  mx c to touch y2  4bx we have

 5 y1 3(9 x1 )  y12 6 x1 2a( y12 2ax1 )  by12

 y12 5 y1 3 x1 27  0  y12 (2a b)  4a 2 x1


Therefore, (x1, y1) lies on the curve y2 5 y 3 x 27  0. Therefore, the point (x1, y1) lies on the curve
Answer: (A) 4a 2 x
y2 
2a b
33. PQ is a chord of the parabola y2  4ax such that the
normals at P and Q intersect on the parabola. Then Answer: (B)
the midpoint of the chord PQ lies on the curve
35. P is a point on the directrix of the parabola y2  4ax
(A) y2  2a( x 2a) (B) y2  2a( x a) and Q is the point of contact of a tangent drawn from
(C) y2  2a( x 2a) (D) y2  a( x 2a) P to the parabola. Then the midpoint of PQ lies on
the curve
Solution: Let P  (at12 , 2at1 ) and Q  (at22 , 2at2 ) and
let M(x1, y1) be the midpoint of PQ so that (A) y2 (2 x a)  a(3 x a)
(B) y2 (3 x a)  a(2 x a)2
2 x1  a(t12 t22 ) and y1  a(t1 t2 ) (4.92)
(C) y2 (2 x a)  a(3 x 2a)2
Since the normals at P and Q meet on the parabola, by
Theorem 4.13, we have (D) y2 (2 x a)  a(3 x a)2
t1 t2  2 (4.93) Solution: Let P  (–a, y) be a point on the directrix and
Q  (at 2 , 2at ) be the point of contact of the tangent from
From Eq. (4.92),
P. The tangent at Q is ty  x at2. This cuts the directrix
¨ y2 · y 2 4a 2 at P. So
2 x1  a [(t1 t2 )2 2t1t2 ]  a © 12 4 ¸  1
ªa ¹ a a(t 2 1)
y
 y12 4a 2  2ax1 t
 y12  2a( x1 2a) Therefore

Therefore, (x1, y1) lies on the curve y2  2a( x 2a). ¥ a(t 2 1) ´


P  ¦ a, µ (5.94a)
Answer: (C) § t ¶
Suppose M(x1, y1) is the midpoint of PQ. Therefore
34. Tangent to the parabola y2  4ax meets the parabo-
la y2  4ax in P and Q. Then the midpoint of PQ lies 2 x1  a(t 2 1)  2 x1 a  at 2
on the curve and
2 2
4a x 4a x
2
(A) y  (B) y2  a(t 2 1)
a b 2a b 2 y1  2at
t
Worked-Out Problems 259

3at 2 a of point A with respect to the circle and the parabola


 2 y1t  is
t
2
 2 y1t  3at a (5.94b) 15a 2 15a 2
(A) (B) (c) 4a2 (D) 5a2
2 4
Now from Eq. (5.94a) Solution: See Fig. 4.25. The line
2 x1 a a
t y  mx
a m
Substituting this is Eq. (5.94b) we get is always a tangent to y2  4ax, which is also the tangent
2 x1 a to the circle
2 y1  6 x1 3a a
a a2
x 2 y2 
4 y12 (2 x1 a) 2
  4(3 x1 a)2
a This implies
 y12 (2 x1 a)  a(3 x1 a) 2
a
0 0
 m  a
Therefore, (x1, y1) lies on the curve
y2 (2 x a)  a(3 x a)2 . 1 m2 2

Answer: (D)
 2  m2 (1 m2 )
36. If PNPa is a double ordinate of the parabola y2  4 x,  m4 m2 2  0
then the locus of intersection of the normal at P and
 (m2 2)(m2 1)  0
a line parallel to the axis through Pa is
 m  p1
(A) y2  4( x 2) (B) y2  4( x 3) Therefore, the tangents are y  x a and y  x a which
(C) y2  4( x 4) (D) y2  4( x 1) intersect at A( a, 0) which lies on the directrix. The
chord of contact of A( a, 0) with respect to the circle
Solution: Let P =(t 2 , 2t ) and P a  (t 2 , 2t ). The nor- x 2 y2  a 2 / 2 is
mal at P is
a
tx y  2t t3 (4.95a) x
2
The line passing through Pa and parallel to the axis is and with respect to the y2  4ax is x  a. Let PQ be
y  2t (4.95b) x  a /2 and RS be x  a so that PQRS is a trapezium
whose area is
Intersection of the line in Eq. (4.95a) and the line in
(4.95b) is 1
( PQ RS) s (Distance betweenPQ and RS)
2
¥ y´ y3
¦§ µ¶ x y  y 1 a 15a 2
2 8  (a 4a)(a ) 
2 2 4
y2
 x 2  2 y
4 S
 4 x 16  y2
 4( x 4)  y2 P
x=a
Answer: (C)
A x
37. From point A, common tangents are drawn to the
circle Q

R
a2
x 2 y2  x = −a
2
and parabola y2  4ax. The area of the quadrilateral FIGURE 4.25
formed by the common tangent, the chords of contact
Answer: (B)
260 Chapter 4 Parabola

2
38. PSQ is a focal chord of a parabola where S is the fo- Solution: See Fig. 4.26. Let P  (t1 , 2t1 ) and Q  (t22 , 2t2 ).
cus. If SP  3 and SQ  2, then the length of the latus Let L be the midpoint of
rectum is
¥1 ´
12 6 8 24 PQ  ¦ (t12 t22 ), t1 t2 µ
(A) (B) (C) (D) §2 ¶
5 5 5 5
Slope of the chord PQ is
Solution: By Theorem 4.19, part (3), we know that semi-
latus rectum is the harmonic mean (HM) between SP and 2
SQ. If l is the length of the semi-latus rectum, then t1 t2
2(SP )(SQ) 2(3)(2) 12 Hence, the equation of the perpendicular bisector of the
l  
(SP ) (SQ) 3 2 5 chord PQ is
Hence, (t1 t2 ) ¥ t2 t2 ´
y (t1 t2 )  ¦ x 1 2µ
24 2 § 2 ¶
2l 
5
Substituting y  0 in this equation, we have
Answer: (D)
¥ t2 t2 ´
M  ¦ 2 1 2 , 0µ (4.99)
39. The ordinates of the points P and Q of a parabola § 2 ¶
y2  4x are in the ratio 1:2. Then the locus of the
point of intersection of the normals at P and Q is Also
k ¥ t2 t2 ´
y2  ( x 2) 3 N  ¦ 1 2 , 0µ (4.100)
343 § 2 ¶
where the value of k is
From Eqs. (4.99) and (4.100), we get
(A) 18 (B) 36 (C) 54 (D) 12
t12 t22
Solution: Let P  (t12 , 2t1 ) and Q  (t22 , 2t2 ) so that, by OM  2
hypothesis, we have 2

(2t1 ) : (2t2 )  1 : 2 t12 t22


and ON 
 t2  2t1 (4.96) 2

Let (x1, y1) be the intersection of the normals at P and Q so that MN  OM ON  2 which is also the semi-latus
so that, by Theorem 4.11 and Eq. (4.96), we have rectum.

x1  2 t12 t1t2 t22  2 7t12 (4.97) y

and y1  t1t2 (t1 t2 )  2t12 (3t1 )  6t13 (4.98) Q

From Eqs. (4.97) and (4.98), we have L


P
3 2
¥ x1 2 ´ 6 ¥ y1 ´ y12
¦§ µ  t1  ¦§ µ 
7 ¶ 6 ¶ 36
O N M x
Hence, the locus of (x1, y1) is
36
( x 2) 3  y 2
343
Answer: (B)
FIGURE 4.26
40. PQ is a chord of the parabola y2  4x whose perpen-
dicular bisector meets the axis at M and the ordinate Answer: (A)
of the midpoint PQ meets the axis at N. Then the
length MN is equal to 41. If the tangent at two points of the parabola y2  4ax
intersect at (h, k), then the normals at these two
(A) 2 (B) 2.5 (C) 3 (D) 4 points intersect in
Worked-Out Problems 261

an angle at the focus of the parabola which equals


¥ k 2 hk ´ ¥ k 2 hk ´
(A) ¦ 2a h , (B) ¦ h 2a ,
§ a a µ¶ § a a µ¶ (A) 60o (B) 45o (C) 30o (D) 90o
Solution: Let P be (at2, 2at) so that t  2 because it lies
¥ k 2 hk ´ ¥ k 2 hk ´ on the line y  x. Suppose normal at P meets the curve at
(C) ¦ 2a h , (D) ¦ h 2a ,
§ a a µ¶ § a a µ¶ Q  (at a 2 , 2at a ). So

Solution: Let P  (at12 , 2at1 ) and Q  (at22 , 2at2 ). 2


t a 2  t  2 1 (& t  2)
Therefore the point of intersection of the tangent is t
T  (at1t2 , a(t1 t2 ))  (h, k ) Therefore, Q  (9a, 6a) and P  (4a, 4a). Now

so that ¥ 4a ´ ¥ 6a ´
Slope of SP s Slope of SQ  ¦
h  at1t2 § 4a a µ¶ ¦§ a 9a µ¶
4 ¥ 6 ´
and k  a(t1 t2) (4.101)  ¦ µ  1
3§ 8 ¶
Let N be the intersection of the normals at P and Q so
that, say, Hence, PSQ  90o.
Answer: (D)
N  (2a a(t12 t1t2 t22 ), at1t2 (t1 t2 ))  ( x, y)
Then 44. The angle between the tangents drawn from the
point (1, 4) to the parabola y2  4x is
x  2a a(t12 t1t2 t22 )  2a a ¨ª(t1 t2 )2 t1t2 ·¹
P P P P
2 2
(A) (B) (C) (D)
¨k h· k 6 4 3 2
 2a a © 2 ¸  2a h
ªa a ¹ a (IIT-JEE 2004)

¥ k ´ hk Solution: The line


and y  at1t2 (t1 t2 )  h ¦ µ 
§ a¶ a 1
y  mx
Answer: (A) m
touches y2  4x for all m x 0. This passes through (1, 4).
42. The point of intersection of the normals at A and B So
of a parabola y2  4ax meet on the line y  a. Then
the point of intersection of the tangents at A and B 1
4  m
lies on the curve m
(A) xy  a (B) x2  4ay  m2 4 m 1  0 (4.103)
(C) xy  a2 (D) x2  2ay Let m1 and m2 be the roots of Eq. (4.103) so that m1
m2  4 and m1m2  1. If P is the acute angle between
Solution: Let A  (at12 , 2at1 )
and B  so (at22 , 2at2 ) those tangents, then
that the point of intersection N of normals drawn at A
and B is m1 m2
tan Q 
N  [2a a(t12 t1t2 t22 ), at1t2 (t1 t2 )] 1 m1 m2

and lies on y  a. This implies (m1 m2 )2 4 m1 m2



1 m1 m2
at1t2 (t1 t2 )  a
 t1t2 (t1 t2 )  1 (4.102) 16 4
  3
Let T (x, y) be the intersection of the tangents at A and 2
B. Hence, x  at1t2 and y  a(t1 t2). Therefore, from Eq. Hence, Q  P / 3.
(4.102), we get
Answer: (C)
xy  (at1t2 ) s (a) s (t1 t2 )  a 2
Answer: (C) 45. If P is a point on the parabola y2  8x above the axis
and S is the focus with SP  4, then the ordinate of P is
43. A point P of the parabola y2
 4ax lies on the line (A) 4 (B) 2 (C) 8 (D) 2 2
y  x. The normal chord PQ, normal at P, subtends
262 Chapter 4 Parabola

Solution: Let P  (x, y) so that The tangent at the point (1, 2) to the parabola is
4  SP  2 x (& SP  a x) y(2) 2(x 1)  0
 x  2 and y2  16 yx 1 (4.105)
 x  2 and y  p 4 Let (h, k) be the centre of the circle that lies on the normal
Since P lies on the curve above the axis, we have y  4. to the parabola at (1, 2) whose equation is x y  3.
Hence
Answer: (A)
h k3 (4.106)
46. A circle is drawn with centre at the focus S of the
Also, since the circle touches the directrix x 1  0, we
parabola y2  4x so that a common chord of the pa- have
rabola and the circle is equidistant from the focus
and the vertex. Then the equation of the circle is (h 1)2  (h 1)2 (k 2)2  (Radius)2
9 9 Therefore, from Eq. (4.106), we get
(A) ( x 1)2 y2  (B) ( x 1)2  y2
4 16
4 h  (k 2) 2  ( 3 h 2) 2
9 9
(C) ( y 1)2 x 2  (D) ( y 1)2 x 2 
4 16  h2 6h 1  0
Solution: Since S  (a, 0)  (1, 0), the circle is of the  h  3p 2 2
form
Therefore
( x 1)2 y2  r 2 (4.104)
k  3 (3 p 2 2 )  p 2 2
Suppose AB is a common chord. Since this is equidistant
from the focus and the vertex M(1/ 2, 0) lies on AB and Since the circle is above the axis, we have k  2 2 which
AB is double ordinate of the parabola, let A  (1/ 2, y) so is the y-coordinate of the point of contact of the circle
that with the directrix.

¥ 1´ y
y2  4 ¦ µ
§ 2¶
 y p 2 (h, k)

¥1 ´ ¥1 ´
 A  ¦ , 2 µ and B  ¦ , 2 µ L(1, 2)
§2 ¶ §2 ¶
x+1=0
Since $AMS is right-angled triangle, we have
1 (−1, 0) O S(1, 0) x
SA2  SM 2 MA2  2
4
9
  (Radius)2
4
Hence, the equation of the circle is FIGURE 4.27

9 Answer: (A)
( x 1)2 y2 
4
48. The directrix of the parabola traced out by the centre
Answer: (A)
of a moving circle which touches both the line y  x
and the circle ( x 3)2 ( y 4)2  9 is
47. A circle touches the parabola y2  4x at the point (1, 2)
and also the directrix. The y-coordinate of the point of (A) x y 3  0 (B) x y 3  0
contact of the circle and the directrix is
(C) x y 3 2  0 (D) x y 2  0
(A) 2 (B) 2 (C) 2 2 (D) 4
Solution: See Fig. 4.28. The directrix is the line parallel
Solution: See Fig. 4.27. Clearly, the point (1, 2) is the to x y  0 at a distance of 3 units from x y  0 to the left
upper end of the latus rectum of the parabola of x y 0. Therefore, the required line is x y 3  0.
y2  4x
Worked-Out Problems 263

y Solution: The given equation is


y= x2  (y 4)
−x
(0, 4) (3, 4) Now
3 ¥ dy ´
¦§ µ¶  2 x1
dx ( x , y )
1 1

x
O Hence, the equation of the tangent at P (x1, y1) is
y y1  2 x1 ( x x1 )
 2 x, x y  y1 2 x12
Hence
FIGURE 4.28
¥ y 2 x12 ´
Answer: (B) A¦ 1 , 0µ
§ 2 x1 ¶
49. Let L be the point (t, 2) and M be a point on the y-axis and B  (0, y1 2 x12 )
such that the slope of LM is −t. Then, the locus of the
midpoint of LM is a parabola whose latus rectum is Thus, the area of $OAB is given by
2
1 1 1 ( y 2 x12 )
(A) 2 (B) (C) 4 (D) $OAB  (OA)(OB)  1
2 4 2 4 x1
Solution: Let M  (0, k) so that the slope of LM is 2
(4 x12 2 x12 )2 ( x12 4)
 
2 k 4 x1 4 x1
 t
t 0
So
 2 k  t 2 (4.107)
d $ 1 ¨ 4 x12 ( x12 4) ( x12 4)2 · x12 4
Let (x, y) be the midpoint of LM. Therefore  © ¸ (3 x12 4)
dx1 4 ª x12 ¹ 4 x1
2

t 2 k
x and y  Therefore
2 2
d$
Hence, from Eq. (4.107), we have 0
dx1
2 y  2 k  2 (2 t 2 ) 2
 3 x12  4 or x1 
 4 t2 3
 4 4x2
We can see easily that d $/dx1 changes sign from – to
 y  2 2x2 when x1  2 / 3 . Hence $ is minimum at x1  2 / 3 and
1 the minimum value of $ is given by
 x 2  ( y 2)
2 [(2/3) 4]2 256 3 32
$  s 
Hence, the latus rectum is 1/2. 4(2 / 3 ) 9 8 3 3
Answer: (B) Answer: (B)
50. Let P be a point on the parabola y  4 x2 lying in
Try it out Solve Worked-Out Problem 50
the first quadrant (i.e., x  0, y  0). Tangent to the
using parametric equation of the tangent.
parabola at P meets the x- and y-axis at A and B,
respectively. The minimum possible area of $OAB
(where O is the origin) is 51. Let P be the point (−3, 0) and Q be a moving point
64 32 (0, 3t). Suppose PQ is trisected at R so that R is
(A) (B) nearer to Q. The line RN which is drawn perpen-
3 3 3 3
dicular to PQ meets the x-axis at N. The locus of the
(C) 96 3 (D) 192 3 midpoint of PN is a parabola whose directrix is
264 Chapter 4 Parabola

(A) x  2 (B) y  2 53. If the line y  x touches the parabola y  x2 bx c


at the point (1, 1), then
(C) x  –2 (D) y  –2
(A) b  0, c  −1 (B) b  −1, c Z
Solution: Point R is nearer to Q implies that PR:RQ 
2:1 so that R (−1, 2t). Equation of the line through R (C) b  −1, c  −1 (D) b  −1, c  1
and perpendicular to PQ is Solution: The point (1, 1) lies on the parabola, which
1 implies that
y 2t  ( x 1) or x ty  2t 2 1
t b c0 (4.108)
and hence N  (2t 2 1, 0). Let M (x, y) be the midpoint Also, y  x touches the parabola š the quadratic
of RN so that x 2 (b 1) x c  0 has equal roots š (b 1)2 4c  0.
2t 2 1 1 2t Therefore, from Eq. (4.108), we get
x , y t
2 2 (b 1)2 4b  0
Therefore,  (b 1)2  0
2 x  2t 2 2  2 y 2 2  b  1 and c =1
 y2  x 1 Answer: (D)

Hence, the directrix is x 2  0. 54. If P(x1, y1) and Q(x2, y2) are two points on the pa-
Answer: (C) rabola y2  8ax, at which the normal meets in (18, 12),
then the length of the chord PQ is
52. A circle of radius r touches the parabola y2  4ax (a 
(A) 2 15 (B) 4 15
0) at the vertex and the centre lies on the axis of the
parabola. Further, the circle completely lies within (C) 2 13 (D) 4 13
the parabola. Then the largest possible value of r is Solution: Observe that (18,12) lies on the parabola.
(A) 2a (B) 3a (C) 4a (D) a Let
Solution: Equation of the circle is ( x y)2 y2  r 2 . P  (at 2 , 2at 2 )  (2t 2 , 4t ) (& a  2)
See Fig. 4.29. If P(at2, 2at) is any point on the parabola,
If the normal at P meets the circle at (18, 12)  (2t 21, 4t1),
then
where t1  3, by Theorem 4.12, we have
(r at 2 )2 4a 2 t 2 r r 2
2
(equality holds when t  0). This implies t 3
t
a 2 t 4 2art 2 4a 2 t 2 r 0 so that
2
 at 2r 4a r 0 t 2 3t 2  0
a
 r b (t 2 4) b 2a Hence, t  − 1, −2. Therefore, P  (2, −4) and Q  (8, −8).
2 Thus
where the equality holds when t  0. Hence the maxi-
mum value of r is 2a. PQ  (8 2)2 ( 8 4)2  52  2 13
Answer: (C)
y
P
55. P is one end of the latus rectum of the parabola
y2  4ax. If the normal at P to the parabola meets the
r
circle again in Q then the length of the chord PQ is
(A) 8a 2 (B) 4a 2
O (r, 0) x
(C) 2a 2 (D) 8a
Solution: Let P  (a, 2a) so that the value of the param-
eter t is 1. Suppose normal at P to the parabola meets the
curve again at (at 21, 2at1). Therefore,

FIGURE 4.29 2
t1  1  3
Answer: (A) 1
Worked-Out Problems 265

Hence, Q (9a 6a)and Now


(PQ)2  (9a a)2 ( 6a 2a)2  2 s 64a 2 PaQ passes through S(a, 0)
 PQ  8a 2 1
št (1 t 2 )
Answer: (A) t1 t
š tt1  1
56. In the parabola y2  4x, the tangent at the point P,
whose abscissa is equal to the latus rectum, meets which is true according to Eq. (4.110). Hence, PaQ passes
the axis at T and the normal at P cuts the curve through S(a, 0).
again at Q. Then the ratio PT:PQ is Answer: (D)
(A) 5:4 (B) 4:5 (C) 2:3 (D) 3:2
58. P is a point on the parabola y2  4x and M is the foot
Solution: We have of the perpendicular drawn from P onto the direc-
trix. S is the focus. If ΔPSM is an equilateral triangle,
y2  4ax(a  1)
then the area of the triangle is
By hypothesis, at P  (4, 4) so that the tangent equation
(A) 3 3 (B) 4 3
at P (4, 4) is 2y  x 4 which implies that
(C) 2 3 (D) 8 3
T  ( 4, 0) (4.109)
Solution: It is clear that the focus S is (1, 0) and M  (−1,
Also the normal at P(4, 4) meets the curve again at Q (t 2
2t) where p  (t 2, 2t). Since ΔPSM is equilateral, we have
2t). So
SM  SP
2
t  2  3  4 4t 2  (t 2 1)2
2
so that Q  (9, 6). Now  t 2 1  4 or t  p 3

Now P  (3, 2 3 ) implies


PT  8 2 4 2  4 5
3
Area of $ PSM  (16)  4 3
and PQ  (9 4)2 ( 6 4)2 4
Answer: (B)
 125  5 5
Hence 59. The tangents at P and Q on a parabola y2  4ax meet
at point T. If the chord PQ passes through the fixed
PT : PQ  4 5 : 5 5  4 : 5 point ( a, b) then the point T lies on the curve
Answer: (B)
(A) bx  2a(y a) (B) ay  2b(x b)
57. For the parabola y2
 4ax, let T  (−a, 0). If PPa is a (C) ax  2b(y b) (D) by  2a(x a)
double ordinate of the parabola at PT and meets the Solution: Let P  (at12 , 2at1 ) and Q  (at22 , 2at2 ) so
curve again at Q, then PaQ passes through the point that
(A) (4a, 0) (B) (3a, 0) (C) (2a, 0) (D) (a, 0) T  (at1t2 , a(t1 t2 ))
Solution: Let P  (at2,
2at). so that Pa  (at2,
–2at). Let The equation of the chord PQ is
Q  (at12 , 2ct1 ) so that the equation of the line PQ is
2
y 2at1  ( x at12 )
2 t1 t2
y 2at  ( x at 2 )
t t1
The chord PQ passes through the point (–a, b). So
The line PQ passes through T ( a, 0). So 2
b 2at1  ( a at12 )
1 t1 t2
t (1 t 2 )
t t1  b(t1 t2 ) 2at1t2 2a  0 (4.111)
 tt1  1 (4.110) Suppose (x, y)  T  [at1t2, a(t1 t2)]. Then
Now, the equation of PaQ is x  at1t2 and y  a(t1 t2 )
2 Substituting the values of x and y in Eq. (4.111), we
y 2at  ( x at 2 )
t1 t have
266 Chapter 4 Parabola

¥ y´ This line touches the parabola. So


b ¦ µ 2 x 2a  0
§ a¶ 2
c 2 3
 by  2a( x a) (1/ 3 )
Answer: (D) Therefore, the tangent equation is

60. The distance between a tangent to the parabola 3y  x 2


y 2  8x and a parallel normal which is inclined at
 x 3y 6  0 (4.112)
an angle of 30o with the axis is
Equation of the normal with slope 1/ 3 (see Quick
16 2 16 16 Look 7) is
(A) (B) (C) (D)
3 3 3 3 3
1 4 2
y x
Solution: The slope of the normal is 3 3 3 3
1 14
tan 30o   x 3y 0 (4.113)
3 3
The line with slope 1/ 3 is Hence, the distance between the lines provided in Eq.
(4.112) and (4.113) is
1
y x c 6 14/3
3 32 16
 
2
1 3 3(2) 3
Answer: (C)

Multiple Correct Choice Type Questions


1. If C is a circle described on the focal chord of the pa- 2
rabola y2  4x as diameter which is inclined at an angle  1
t1 1/t1
of 45o with the axis, then the
1
(A) radius of the circle is 2.  t1 2
t1
(B) the centre of the circle is (3, 2).
(C) the line x 1  0 touches the circle.  t12 2t1 1  0
(D) the circle x 2 y2 2 x 6 y 3  0 is orthogonal  t1  1
to C.
Therefore, P  (1, 2) and Q  (1, 2). From Eq. (4.114),
Solution: Let
the equation of the circle is x 2 y2 2 x 3  0. Hence,
¥ 1 2 ´ the radius is 2 and the centre is (1, 0).
P  (t12 , 2t1 ) and Q  ¦ 2 , µ
§ t1 t1 ¶ Answer: (A), (C)
be the extremities of the focal chord. By Theorem 4.19, 2. For the parabola y2 2 x 6 y 5  0, which of the fol-
part (4), the circle touches the directrix x 1  0. The lowing are true?
equation of the circle is
(A) Vertex is ( 2, 3)
¥ 1´ ¥ 2´
( x t12 ) ¦ x 2 µ ( y 2t1 ) ¦ y µ  0 ¥ 3 ´
§ t1 ¶ § t1 ¶ (B) Focus is ¦ , 3µ
§ 2 ¶
¥ 1´ ¥ 1´ (C) Directrix is 2x 5  0
 x 2 ¦ t12 2 µ x 1 y2 2 ¦ t1 µ y 4  0 (4.114)
§ t1 ¶ § t1 ¶ (D) Latus rectum is 2
The slope of the focal chord PQ is equal to 1. So we have Solution: Given equation is ( y 3)2  2( x 2) which
represents parabola whose
2(t1 1/t1 )
1
t12 1/t12
Worked-Out Problems 267

Vertex  (–2, 3) y

1 3 P
Focus  ¥¦ x 2  , y 3  0´µ  ¥¦ , 3´µ
§ 2 ¶ § 2 ¶
1
Directrix  x 2  0 or 2 x 5  0
2
O x
Latus rectum  4
Answer: (A), (B), (C)
G
3. If (x1, y1) and (x2, y2) are two points on the parabola Q
2
y2  8x, at which the normals to the curve intersect on
the curve, then FIGURE 4.30

(A) x1 x2  8 (B) y1 y2  16 Answer: (A), (B), (C), (D)


(C) y1 y2  32 (D) x1 x2  16
5. Let A and B be two distinct points on the parabola
Answer: (C), (D) y2  4x. If the axis of the parabola touches a circle of
Solution: If the normals at (at12 , 2at1 ) and (at22 , 2at2 ) radius r and AB is its diameter, then the slope of the
meet again on the curve, then by Theorem 4.13, line AB is
x1 x2  4a 2 and y1 y2  8a 2 . Here, a  2. 1 1 2 2
(A) (B) (C) (D)
r r r r
4. PQ is a double ordinate of the parabola y2 
4ax. If the normal at P intersects the line passing (IIT-JEE 2010)
through Q and is parallel to the axis at G, then the Solution: Let A  (t12 , 2t1 ) and B  (t22 , 2t2 ) (t1 , x t2 ). There-
locus of G is a parabola with fore, the slope of AB is
(A) vertex at (4a, 0) 2
(4.117)
(B) directrix x  3a t1 t2
(C) focus (5a, 0) Now, C is the centre of the circle described on AB as
(D) latus rectum 4a diameter (see Fig. 4.30), which is given by
Solution: Let P  (at 2 , 2at ) and Q  (at 2 , 2at ). The ¥ t12 t22 ´
normal at P is ¦ 2 , t1 t2 µ
§ ¶
tx y  2at at 3 (4.115)
Therefore, the radius is
The line passing through Q and parallel to the axis (i.e.,
t1 t2  r (& the circle touches the axis)
x-axis) is
So, the slope of AB  p2/r.
y  2at (4.116)
Answer: (C), (D)
From Eqs. (4.115) and (4.116), we have
y
G  (4a at 2 , 2at )  (h, k ) (say) A

Therefore r

r C
h  4a at 2 and k  2at
r
B
¥ k2 ´ k2
 h 4a  a ¦ 2 µ  O x
§ 4a ¶ 4a
 k 2  4a(h 4a)
 y2  4a( x 4a)
Hence, the vertex is (4a, 0), latus rectum is 4a, the FIGURE 4.31
directrix is x 4a  a or x 3a  0 and the focus 
( x 4a  a, 0)  (5a, 0). 6. The tangent PT and the normal PN to the parabola
y2  4ax at a point P on it meet its axis at points T
268 Chapter 4 Parabola

and N, respectively. The locus of the centroid of the 7. The equation of the common tangents to the parabo-
ΔPTN is a parabola whose las x2  y and (x 2)2  y are
¥ 2a ´ (A) y  4(x 1) (B) y  0
(A) vertex is ¦ , 0µ (B) direction is x  0
§ 3 ¶ (C) y  4(x 1) (D) y  30 x 50
2a (IIT-JEE 2006)
(C) latus rectum is (D) focus is (a, 0)
3
Solution: We have
(IIT-JEE 2009)
¥ 1´ ¥ 1´
Solution: See Fig. 4.32. P  (at2, 2at). Tangent at P is x2  y  4 ¦ µ y¦a  µ
§ 4¶ § 4¶
ty x at 2
Hence, any tangents to x2  y are of the form
so that T  ( at2, 0). Normal at P is
1
tx y  2at at3 x  my m x 0
4m
and hence N  (2a at2, 0).
Now P  (at2, 2at), T  ( at2, 0) and N  (2a at2, 0). This line meets the parabola y  (x 2)2 at the point
Let G(x, y) be the centroid of ΔPTN. Therefore whose ordinates are the roots of the equation
2
2a at 2 2at ¨
y  © my
1 ·

x and y
3 3 ª 4m ¹
Hence ¥3 ´ 1 1
 m2 y2 ¦ 4 mµ y 4  0 (4.118)
2
§2 ¶ 16 m 2
m
3 x 2a 2 ¥ 3 y ´
t ¦ µ This line touches the parabola y  ( x 2)2
a § 2a ¶
9 2 š The roots of Eq. (4.118) are equal
 3 x 2a  y 2
4a ¥3 ´ ¥ 1 1´
4a š ¦ 4 mµ 4 m2 ¦ 4 µ  0
 y2  (3 x 2a) §2 ¶ § 16 m2 m¶
9 9 1
4a ¥ 2a ´ š 12 m 16 m2 16 m2 4 m  0
 y2  ¦§ x µ¶ 4 4
3 3 š 2 8m  0
Therefore, 1
šm
Vertex  (2a / 3, 0) 4
Hence,
¥ 2a a ´
Focus  ¦ x  , y  0µ  (a, 0)
§ 3 3 ¶ y
x 1  y  4x 4
4
2a a a
Directrix  x  x Also the line y  0 (i.e., x-axis) touches y  x2 at (0, 0) and
3 3 3
also y  ( x 2)2 at (2, 0).
Latus rectum  4a/3
Answer: (A), (B)
y
8. Consider the parabola with its vertex at origin and the
P
axis along the x-axis. The line y  2x c, where c  0, is
a common tangent to the parabola and the circle x2
y2  5. Then the
(A) directrix is x  10
T O N x
(B) focus is (10, 0)
(C) latus rectum is 20
(D) directrix is x  10
Solution: The line y  2x c touches the circle. This
FIGURE 4.32 implies
Answer: (A), (D) c2  5(1 2 2 )  25
Worked-Out Problems 269

 c  5 (& c  0) Therefore, the vertex is (a / 2, 0), the latus rectum is 2a


and the directrix equation is
Let y2  4ax be the parabola. Since the line y  2x 5
touches the parabola we have a a
x  x0
2 2
a
5 or a  10. Answer: (A), (B), (D)
2
Therefore, the parabola is y2  40x. Hence, the focus is 10. Consider the circle C: x 2 y2 6 y 4  0 and the
(10, 0). parabola P: y2  x. Then
Answer: (A), (B) (A) the number of common tangents to C and P is 3.
9. Let P be the parabola which is the locus of the (B) the number of common tangents to C and P is 2.
midpoints of focal radii of the parabola y2  4ax. (C) x 2y 1  0 is one of the common tangents.
For P, which of the following are true? (D) x 2y 1  0 is also one of the common tan-
¥a ´ gents.
(A) Vertex is ¦ , 0µ
§2 ¶ Solution: C: x 2 ( y 3)2  5. Let Q(t2, t) be any point
(B) Latus rectum is 2a on the parabola y2  x so that the equation of the tangent
at Q is x 2ty t 2  0 which touches the circle C. So
a
(C) Directrix is x 
2 0 2t(3) t 2
(D) Directrix x 0  5
1 4t 2
Solution: Let Q (at2, 2at) be a point on the parabola  (t 2 6t )2  5 (1 4t 2 )
and S  (a, 0). M(x, y) is the midpoint of SQ. This implies
 t 4 12t 3 16t 2 5  0
at 2 a
x and y  at  (t 1)2 (t 2 10t 5)  0
2
Now  t  1,, 5 p 30

2x a 2 ¥ y´
2 Hence, the number of common tangents is 3 and
t ¦ µ x 2 y 1  0 is a common tangent when t  1.
a § a¶
Answer: (A), (C)
¥ a´
 y2  a(2 x a)  2a ¦ x µ
§ 2¶

Matrix-Match Type Questions


1. Match the items of Column I with those of Column II.
Column I Column II

Column I Column II (C) The latus rectum of a (r) 8


parabola having (3, 5)
(A) If the line x 1  0 is (p) 2 and (3, −3) as extremi-
the directrix of the ties of the latus rectum (s) −4
parabola y2 kx 8  0, is
then the value of k is (D) If (2, 0) is the vertex
(B) If l is the length of one (q) 4 and y-axis as the direc- (t) −8
side of an equilateral trix, then its focus is (a,
triangle inscribed in 0) when a equals
the parabola y2  4x
with one vertex at the Solution:
origin, then l /2 3  (A) The parabola y2 kx 8  0 is written as

(Continued) ¥ 8´
y2  k ¦ x µ
§ k¶
270 Chapter 4 Parabola

¥ k´ ¥ 8´ Column I Column II
 4¦ µ ¦ x µ
§ 4¶ § k¶ (D) Two points P(4, −4) and Q are
Hence, the directrix is on the parabola y2  4x such that (t) (2, 1)
the area of ΔPOQ (O is the
8 k vertex) is 6 sq. unit. Then, the
x 
k 4 coordinates of Q are
k 8
x  0
4 k Solution:
k 2 32 (A) Let P (x1, y1) be on the circle x2 y2  5. Then
 x 0
4k
x12 y12  5 (4.119)
By hypothesis, x  1 is the directrix. Therefore
The equation of the chord of contact of P(x1, y1)
k 2 32 with the parabola y2  4x is
 1
4k yy1 2( x x1 )  0
2
 k 4k 32  0  2 x y1 y 2 x1  0
 (k 8)(k 4)  0 However,
 k  4, 8
2x y 4  0 (4.120)
Answer: (A) j (q), (t)
is the chord of contact. Therefore, from Eqs. (4.119)
(B) From Problem 8 of the section ‘Subjective Prob-
and (4.120), we get
lems’, we have l  8a 3, where a  1. Therefore
2 y1 2 x1
l  
4 2 1 4
2 3  x1  2, y  1
Answer: (B) j (q)  P  ( 2, 1)
(C) The length of the latus rectum is (3 3) (5 3)2  8 Answer: (A) j (r)
Answer: (C) j (r) (B) Tangent to the parabola y2  4 x at (t 2 , 2t ) is ty 
x t2. This passes through the point (2, 3). So
(D) Distance of the vertex from the directrix (i.e., y-axis)
is 2 and it is equal to half the distance of the focus 3t  2 t 2
from the directrix so that the focus is (4, 0). There-
 t 2 3t 2  0
fore, a  4.
 (t 1)(t 2)  0
Answer: (D) j (q)
 t  1, 2
2. Match the items of Column I with those of Column II.
Therefore, the points of contact are (1, 2) and (4, 4).
Column I Column II Answer: (B) j (q), (s)

(A) If point P is on the circle x2 y2  (p) (9, −6) (C) Substituting


5, then the equation of the chord
5x 2 7 x
of contact with respect to the y
parabola y2  4x is y  2 (x 2). (q) (1, 2) 6
The coordinates of P are in the circle equation x2 y2  5, we get
(B) Tangents are drawn from the 2
point (2, 3) to a parabola y2  4x. (r) (−2, 1) ¥ 5x 2 7 x ´
x2 ¦ µ 5
Then, the points of contact are § 6 ¶
(C) The common chord of the circle 25 x 4 70 x 3 49 x 2
x2 y2  5 and the parabola (s) (4, 4)  x2 5
36
6y  5x2 7x passes through
 25 x 4 70 x 3 85 x 2 180  0
(Continued)
Worked-Out Problems 271

which clearly implies that x  1 is a root. So Solution:

( x 1)[25 x 3 95 x 2 180 x 180]  0 (A) It is known that the locus of the point from which
perpendicular tangents is drawn to a parabola is
 ( x 1)( x 2)[25 x 2 45 x 90]  0 the directrix of the parabola. For the parabola y2 
 ( x 1)( x 2)(5 x 2 9 x 18)  0 8x, the directrix is x 2  0 on which the points
 x  1, 2 (−2, 1) and (−2, 5) lie.
Answer: (A) j (p), (t)
Therefore, the points of intersection are (1, 2) and
( 2, 1). (B) The line
Answer: (C) j (q), (r) a
y  mx
(D) Let Q  (t2, 2t). Therefore m
touches y2  4ax at
4 4 1
1
6  Area of $ POQ  0 0 1 ¥ a 2a ´
2 ¦§ 2 , µ¶
t2 2t 1 m m
The given line is y  x 3 (a  3, m  1) and
 12  | 8t 4t 2 | hence it touches the parabola y2  12x at
 t 2 2t  p3
¥ a 2a ´
Case 1: When t2 2t 3 0, we have ¦§ 2 , µ¶  (3, 6)
m m
(t 1)(t 3)  0 Answer: (B) j (r)
 t  1, 3 (C) It is known that the line y  mx 2am am3
So Q  (1, 2), (9, 6). is a normal to the parabola y2  4ax at the point
(am2, 2am). In the present case, m  4 / 3 and
Case 2: When t2 2t 3 0, it has no real roots. a  9 / 4. Therefore
Answer: (D) j (p), (q)
¥ 9 16 ¥ 9 ´ ¥ 4 ´ ´
(am2, 2am)  ¦ s , 2 ¦ µ ¦ µ µ  (4, 6)
3. Match the items of Column I with those of Column II. §4 9 § 4¶ § 3 ¶ ¶
Answer: (C) j (q)
Column I Column II
(D) The line parallel to 4y x 3  0 is 4y x c  0.
(A) The point from which (p) (−2, 1) The line with slope m touches the parabola y2  4ax
perpendicular tangents can at the point
be drawn to the parabola y2
 8x is ¥ a 2a ´
¦§ 2 , µ¶
(B) The line x y 3  0 touches (q) (4, 6) m m
the parabola y2  12x at the 7 1
point (r) (3,−6) Here, a  and m  . Therefore, the point of
4 4
(C) 4x 3y 34  0 is normal to contact is
the parabola y2  9x at the (s) (28, 14)
point ¥ a 2a ´ ¥ 7 ¥ 7´ ´
¦§ 2 , µ¶  ¦ s 16, 2 ¦§ µ¶ s 4µ  (28, 14)
m m §4 4 ¶
(D) The line parallel to 4y x (t) (−2, 5)
3  0 touches the parabola Answer: (D) j (s)
y2  7x at the point

Comprehension Type Questions


1. Passage: For the parabola y2  4ax, the vertex is (0, 0), (i) The vertex of the parabola ( y 1)2  2( x 1) is
the focus is (a, 0) and the directrix is x a  0. Answer
(A) (1,0) (B) (2,0)
the following three questions.
(C) (1,1) (D) (0,1)
272 Chapter 4 Parabola

2
(ii) Focus of the parabola y  4( x 1) is (A) x 2y  4 (B) x 2y  4
(C) 2y  x 4 (D) x 2y 4  0
(A) (1, 0) (B) (2, 0)
(C) (1, 1) (D) (2, 2) Solution:

(iii) The directrix of the parabola ( y 2)2  4( x 1) (i) The tangent at P is


is ty  x 2t2 (4.121)
(A) x  1 (B) x  1 which passes through Q(h, k). This implies
(C) x  2 (D) x  0 tk  h 2t2 (4.122)
Solution: The equation of the chord of contact of Q(h, k)
(i) The parabola is with respect to the circle x2 y2  8 is

¥ 1´ hx ky 8  0 (4.123)
Y 2  4 ¦ µ (X )
§ 2¶ From Eqs. (4.122) and (4.123), we have
where X  x 1 and Y  y 1. Therefore, the (h 2t 2 )
vertex is hx y 8  0
t
(X  0, Y  0)  (x 1  0, y 1  0)  (1, 1) ¥ y´
 2(ty 4) h ¦ x µ  0
Answer: (C) § t¶
(ii) y2  4(x 1) is Y2  4X where X  x 1 and Y  y This line passes through the point
and a  1. The focus is
4 y
(X  1, Y  0)  (x 1  1, y  0)  (2, 0) y ,x 
t t
Answer: (B)
Therefore, the locus is
(iii) ( y 2)2  4( x 1)  Y 2  4 X where X  x 1
and Y  y − 2. The directrix equation is y x

4 y
X−a−1
 y2  4x
x−1−1
Answer: (D)
x  0
Answer: (D) (ii) The required point lies on the director circle of the
circle and directrix of the parabola. The directrix of
2. Passage: P(2t2, 4t) is a point on the parabola y2  8x the parabola is x 2  0 and the director circle of the
and Q(h, k) is a point on the tangent at P and external given circle is x 2 y2  (2 2 s 2 )2  16. Now
to the circle x2 y2  8. Answer the following ques-
x  2  4 y2  16
tions.
 y  p2 3
(i) As Q moves on the tangent at P, the locus of the
point of intersection of the chord of contact of Q Therefore, the required point  ( 2, 2 3 ).
with respect to the circle at the tangent at P is Answer: (A)
(A) y2 x2  4 (B) y2  2x (iii) The equation of the circumcircle of ΔAQB is
(C) y2 2x2 4 (D) y2  4x
( x 2 y2 8) L (hx ky 8)  0
(ii) The point in the second quadrant from which
perpendicular tangents can be drawn to both the This should pass through (0, 0) (centre of the circle)
parabola and the circle is which implies that K  1. Therefore, the circumcir-
(A) ( 2, 2 3 ) (B) ( 1, 2 ) cle of ΔAQB is

(C) ( 2 , 2 ) (D) ( 3, 2 3 ) x 2 y2 hx ky  0
(iii) If AB is the chord of contact of Q(h, k) with re-
so that (h / 2, k / 2) is its centre. If (x, y) is the circum-
spect to the circle x2 y2  8, then the circumcen-
centre of ΔAQB, then x  h / 2, y  k / 2. Substitut-
tre of ΔAQB lies on the curve (when t  2)
ing the values of h and k in Eq. (4.122), when t  2,
Worked-Out Problems 273

we have The area of ΔPQS is given by


2(2y)  2x 8 1
1( 2 2 0) 1(0 2 2 ) 1(2 2 2 2 )  4 2
 x 2y 4  0 2
Answer: (C) Therefore, $PQS : $PQR  4 : 16  1 : 4.
Answer: (C)
3. Passage: Consider the circle x2 y2  9 and the parab-
ola y2  8x. They intersect at P and Q in the first and (ii) The area of ΔPRS is given by
the fourth quadrants, respectively. The tangents to the 1
circle at P and Q intersect x-axis at R and tangents to 1(0 0) 9(0 2 2 ) 1(2 2 0)  10 2
2
the parabola at P and Q intersect x-axis at S. Answer
the following questions. Now PS  2 3 , RS  10, PR  6 2 . Therefore, the
circumradius of ΔPQS is
(i) The ratio of the areas of ΔPQS and ΔPQR is
(A) 1 : 2 (B) 1:2 (C) 1:4 (D) 1:8 ( PR)( PS)(RS) (6 2 )(2 3 )(10)
 3 3
4 $PRS 4(10 2 )
(ii) The radius of the circumcircle of ΔPRS is
Answer: (B)
(A) 5 (B) 3 3 (C) 3 2 (D) 2 3
(iii) For ΔPQR, the area  16 2. Now
(iii) The radius of the incircle of ΔPQR is
PQ  4 2 , PR  6 2 , QR  6 2
8
(A) 4 (B) 3 (C) (D) 2 Therefore, the inradius of ΔPQR is given by
3
Solution: Solving the two equations x2 y2  9 and Area 16 2
 2
y2  8x, we get P  (1, 2 2 ) and Q  (1, 2 2 ) (see Semi-perimeter 8 2
Fig. 4.33). Tangents to the parabola at P and Q, respec- Answer: (D)
tively, are
y
y(2 2 )  4( x 1)
P (1, 2√2 )
and y( 2 2 )  4( x 1)
Hence, S  (−1, 0). The tangent to the circle at P is

x(1) y(2 2 )  9
O R (9, 0) x
Hence, R  (9, 0). Therefore, we have P  (1, 2 2 ), Q  S(−1, 0)
(1, 2 2 ), R  (9, 0) and S  (−1, 0).
Q (1, −2√2 )
(i) The area of ΔPQR is given by
1
1( 2 2 0) 1(0 2 2 ) 9(2 2 2 2 )
2
1 FIGURE 4.33
 4 2 36 2  16 2
2

Integer Answer Type Questions


1. The number of points at which the parabola y2  4x The centre and the radius of the circle are (3, 0) and 2 2,
and the circle x 2 y2 6 x 1  0 touch each other is respectively. Now, the distance of the centre (3, 0) from
______. Eq. (4.124) is
Solution: Substituting y2  4x in the given circle equa- 3 0 1 4
tion, we have x 2 2 x 1  0 and hence (1, ± 2) are com-  2 2
2 2
mon points of the two curves. Also at P(1, 2), equation of
the tangent to the parabola is Hence, the line provided in Eq. (4.124) also touches the
circle in a similar manner as the two curves touch at Q.
y(2) 2(x 1)  0
Answer: 2
y x 10 (4.124)
274 Chapter 4 Parabola

2. Slope of the common tangent of the curves y2  8x and GN  ON OG  (2a at 2 ) at 2  2a


xy  1 is ______.
Hence, sub-normal  2a  4 (&a  2)
Solution: We have
Answer: 4
2 2
y  8 x  4(2) x  y  mx y
m
is a tangent of y2  8x. Substituting
P
2
y  mx
m
in xy  1, we get
2´ T O G N x
¥
x ¦ mx µ  1
§ m¶
 m2 x 2 2 x m  0 (4.125)
The line touches the parabola š the discriminant of the
quadratic equation provided in Eq. (4.125) is equal to zero FIGURE 4.34

 4 4 m3  0 m1 5. The locus of the midpoint of chord of y2  4ax which


Hence, the common tangent is y  x 1. subtends right angle at the vertex is another parable
with latus rectum ka, where the integral part of k is
Answer: 1
equal to ______.
3. If x y  a is normal to the parabola y2  12x, then the Solution: See Problem 13 in the section ‘Subjective
value of a is equal to ______. Problems’.
Solution: We have Answer: 2
y2  12x  4(3)x
6. If the normals at t1 and t2 of the parabola y2  4ax
which implies that every point on the parabola is of the meet again on the curve, then the value of t1 t2 is equal
form (3t2, 6t). The normal at (3t, 6t) is to ______.
tx y  6t 3t3 (4.126) Solution: See Theorem 4.13.
However, Answer: 2
x ya (4.127)
7. The normals at P(x1, y1) and Q(x2, y2) of the parabola
is the normal. That is, Eqs. (4.126) and (4.127) represent y2  4x meet on the curve again. Then x1 x2 is equal to
the same line. Therefore ______.
t 1 6t 3t 3 Solution: By Theorem 4.13, we have
   t  1 and a  9
1 1 a
x1 x2  4a 2  4 (& a  1)
Answer: 9
Answer: 4
4. Length of the sub-normal to the parabola y2  8x at
any point is ______. 8. The equation of the parabola with focus (−1, −1) and
directrix 2x 3y 6  0 is ax 2 2 hxy by2 2 gx
Solution: In fact, we show that the length of the sub- 2fy c  0. Then, a b is equal to ______.
normal to the curve y2  4ax at any point on the curve
is equal to 2a. Suppose that the tangent and normal to Solution: By definition,
y2  4ax at P(at2, 2at) meet axis of the curve at T and N, 2 x 3y 6
respectively. Draw PG perpendicular to the axis (see Fig.  ( x 1)2 ( y 1)2
2 2
4.34). Hence, TG  sub-tangent and GN  sub-normal 2 3
(see Definition 3.4, Chapter 3, Vol. 3, pg 225).  (2 x 3 y 6)2  13[ x 2 y2 2 x 2 y 2]
The tangent at P(at2, 2at) is ty  x at2. This meets axis
 9 x 2 12 xy 4 y2 2 x 62 y 10  0
at T( at2, 0) Normal at P(at2, 2at) is tx y  2at at3. This
meets the axis at N (2a at2, 0). Therefore, the semi-latus Therefore, a b  9 4  5.
rectum is Answer: 5
Worked-Out Problems 275

9. The latus rectum of the parabola whose vertex is at Solution: Consider y2  4ax. The normal at (at2, 2at) is
(3, 2) and focus at (5, 2) is ______.
tx y  2at at3 (4.128)
Solution: The vertex is A  (3, 2) and the focus is S  (5, Suppose y  mx c is normal at (at2, 2at). We have
2). Therefore, AS  2 (which is equal to a) and AS is hori-
zontal. Hence, the equation is of the form m 1 c
 
t 1 2at at 3
( y k )2  4a( x h)
 t  m and c  2at at3  − 2am − am3
 ( y 2)2  8( x 3)
 c 2am am3  0
Thus, the latus rectum is 8.
Here, a  2. Hence, p  4, q  2 so that p q  6.
Answer: 8
Answer: 6
10. The locus of the midpoints of chords of the parabola
y2  16x which passes through the vertex is a parabo- 13. If 3 y  bx 3, b  0 is the equation of a common
la whose length of latus rectum is ______. tangent of the circle ( x 3)2 y2  9 and the parab-
ola y2  4x, then the value of b is ______.
Solution: Let M(x1, y1) be the midpoint of a chord of y2
 16x. Hence, the equation of the chord is Solution: We know that
1
yy1 8( x x1 )  y12 16 x1 y  mx
m
This chord passes through the vertex (0, 0). This implies
touches y2  4x for all m x0. This also touches the given
8 x1  y12 16 x1 circle. So
š y12  8 x1 3m 0 (1 / m)
3
y2
Therefore, the locus of M(x1, y1) is the parabola  8x. 1 m2
Hence, the length of latus rectum is 8. 1´
2
¥
Answer: 8  ¦ 3m µ  9(1 m2 )
§ m¶
11. The locus of the midpoints of the chords of the pa- 1
 6 9
rabola 2y2  7x which are parallel to the line 3x − 2y  m2
0 is the line px qy r  0, where p q r equals 1
______.  3
m2
Solution: M(x1, y1) is the midpoint of a chord so that 1
mp
its equation is 3
7 7 Therefore, the equation of the common tangents is
yy1 ( x x1 )  y12 x1
4 2
x
yp 3
 7 x 4 y1 y  7 x1 4 y12 3
which is parallel to  y 3  (p x) 3
3x 2y  0 Hence, b  1 (& b  0).
This again implies Answer: 1
3 7
  6 y1  7 14. Through the vertex O of the parabola y2  4x, the
2 4 y1 chords OP at OQ are drawn at right angles to each
The locus of point M(x1, y1) is the line 6y − 7  0. Hence, other. Then, the equation of the locus of the midpoint
p  0, q  6, r  − 7. Thus, of chord PQ is y2  k (x b) where k b is equal to
______.
p q r  1
Solution: Let P  (t12 , 2t1 ) and Q  (t22 , 2t2 ). Now
Answer: 1 POQ  90oSo

12. If the line y  mx c is normal to the parabola y2  8x, ¥ 2t1 ´ ¥ 2t2 ´


then c pm qm3  0 where p q is equal to ______. ¦ 2 µ ¦ 2 µ  1
§ t1 ¶ § t2 ¶
276 Chapter 4 Parabola

 t1t2  4 (4.129) Therefore


Let M(x, y) be the midpoint of PQ so that y2  (t1 t2 )2  t12 t22 2t1t2  2 x 8  2( x 4)
t12 t22 So, k b  2 4  2.
x
2 Answer: 2
and y  t1 t2

SUMMARY
4.1 Definition: Let L be a straight line and S be a point 4.7 Length of the latus rectum of the parabola y2  4ax
on the plane of the line L and not on L. Then, the lo- is 4a (i.e., the coefficient of x).
cus of the point which is equidistant from the point S
and the line L is called a parabola. For this parabola, 4.8 Parametric equations: x  at2, y  2at, t Z are
S is called focus and L is called directrix. called the parametric equations of the parabola y2 
4ax. That is, for all real values of t, the point (at2, 2at)
4.2 Theorem: Standard equation of a parabola is y2  lies on the parabola y2  4ax.
4ax (a  0). The focus is (a, 0) and the directrix equa-
tion is x a  0. 4.9 Notation:

4.3 The other standard forms of parabola are as fol- S y y2 4ax


lows: S1 y y1 2a( x x1 )
1. y2  4ax [focus is (−a, 0) and directrix is S12  S21  y1 y2 2a( x1 x2 )
x − a  0] Skk  yk2 4axk
2. x2  4ay [focus is (0, a) and directrix is y a  0]
4.10 Theorem: Point (x1, y1) lies outside or inside the
3. x2  4ay [focus is (0, −a) and directrix is
parabola S y y2 4ax according as S11  0 or
y − a  0]
S11  0.
4. ( y k )2  4a( x h) [vertex is (h, k), focus is (h
a, k) and directrix is x  h − a] 4.11 Theorem (Equation of chord joining two points):
5. ( y k )2  4a( x h)2 [vertex is (h, k), focus is If P(x1, y1) and Q(x2, y2) are points on the parabola
(h − a, k) and directrix is x  h a] y2  4ax, then the equation of the chord PQ is
6. ( x h)2  4a( y k ) [vertex is (h, k), focus is S1 S2  S12
(h, k a) and directrix is y  k − a]
 [ yy1 2a( x x1 )] [ yy2 2a( x x2 )] 
7. ( x h)2  4a( y k ) [vertex is (h, k), focus
y1 y2 2a( x1 x2 )
is (h, k − a) and directrix is y  k a]
4.12. Theorem (Tangent): The equation of the tangent to
4.4 The line about which the parabola is symmetric is
S y y2 4ax  0 at (x1, y1) is
called the axis of the parabola.
S1 y yy1 2a( x x1 )  0
4.5 Note:
Tangent (Parametric form): Equation of the tangent
1. x-axis is the axis of y2  4ax and y2  4ax.
at (at2, 2at) is ty  x at2.
2. y-axis is the axis of x2  4ay and x2  4ay.
4.13 Point of intersection of tangents to y2  4ax at the
3. The line y  k is the axis of ( y k )2  p 4a( x h).
points t1 and t2 is
4. The line x  h is the equation of the axis of
( x h)2  p 4a( y k ). [at1t2 , a(t1 t2 )]

4.6 Double ordinate and latus rectum: If a line Note:


perpendicular to the axis meets the curve at P and
1. The tangents at t1 and t2 are at right angles
Pa, then PPa is called double ordinate. A double
š t1t2  1.
ordinate passing through the focus is called latus
rectum. 2. The locus of the point of intersection of perpen-
dicular tangents is the directrix.
Summary 277

4.14. Theorem: The line y  mx c (m x 0) touches the the plane of the parabola y2  4ax. Let
parabola
k 2a h
a G ,H  and $  G 2 4 H 3
2
y  4ax š c  a 3a
m
Then we have
and in the case of tangency, the point of contact is
1. only one normal from (h, k) if $  0.
¥ a 2a ´ 2. only two normals from (h, k) if .$  0.
¦§ 2 , µ¶
m m 3. three normals from (h, k) if $  0.
4.15. Theorem (Normal): Equation of the normal at (at2, 4.21. Definition (Focal chord and focal radius): If
2at) is tx y  2at at 3 . The normal in terms of its a chord of a parabola passes through the focus,
slope m is y  mx 2am am3 at the point (am3, then it is called the focal chord. If P is a point on
2am). a parabola having focus S, then SP is called focal
radius of P with respect to the parabola.
4.16. Theorem: Point of intersection of normals at t1 and
t2 is 4.22. Theorem (Properties of focal chords): Let PSQ be
a focal chord of y2  4ax. Then
[2a a(t12 t1t2 t22 ), at1t2 (t1 t2 )]
(i) If P  (at12 , 2at1 ) and Q  (at22 , 2at2 ), then t2 =
Note: The normals at t1 and t2 are at right angles
š t1t2  1. 1/t1 . Equivalently, for all t x 0, (at2, 2at) and
(a /t 2 , 2a /t ) are the ends of a focal chord.
4.17. (1) If the normal at t1 meets the parabola again at
1 1 1 2
t2 then (ii)  
SP SQ a 2a
2
t2  t1 In fact, semi-latus rectum is HM between the
t1 focal radii of a focal chord.
(2) If the normals at t1 and t2 intersect on the (iii) If PQ is a focal chord and P  (at2, 2at), then
parabola, then t1t2  2 and in such a case prod- the length PQ is equal to a[t (1/t )]2 .
uct of the abscissa  4a2 and product of the
ordinates  8a2. (iv) The tangents drawn at the extremities of focal
chords intersect on the directrix and they are
4.18. Theorem (Number of normals): From any point in at right angles to each other.
the plane of a parabola, in general, three normals (v) The circle described on a focal chord as diam-
can be drawn such that the algebraic sum of the eter touches the directrix.
ordinates of the feat of the normals is zero.
(vi) The circle described on a focal radius of a
Note: If t1, t2 and t3 are the parameters of the feet point as diameter touches the tangent at the
of normals drawn from the point (h, k), then t1, t2 vertex.
and t3 are the roots of the cubic equation (vii) The circle described on a focal radius SP of a
at 3 (2a h)t k  0 point P makes an intercept of length a 1 t 2
on the normal at P(at2, 2at).
and hence
4.23. Theorem: The orthocentre of a triangle formed by
t1 t2 t3  0
three tangents to a parabola lies on the directrix
of the parabola. Also the circumcircle of a triangle
2a h
¤t t 1 2 
a
formed by three tangents to a parabola passes
through the focus.
k
and t1t2 t3  4.24. Theorem: In general, parabola and a circle intersect
a in four points. If t1, t2, t3 and t4 are the points of
4.19. Definition (Conormal points): Points on a parabola intersection of the parabola y2  4ax and the circle
are called conormal points if the nomals at those x 2 y2 2 gx 2 fy c  0, then t1, t2, t3 and t4 are
points are concurrent at a point. roots of the equation.

4.20. Procedure to determine the number of normals a 2 t 4 2a( g 2a)t 2 4aft c  0


from a given point: Let (h, k) be a given point in
278 Chapter 4 Parabola

so that 4 f
¤ t1t2 t3 
¤t1 a
2( g 2a) c
¤ t1t2  and t1t2 t3 t4 
a a2

EXERCISES
Single Correct Choice Type Questions
1. The point of intersection of the tangents drawn at the 8. A parabola has origin as its focus and the line x  2 as
ends of the latus rectum of the parabola y2  4x is the directrix. Then, the vertex of the parabola is at
(A) ( 1, 1) (B) ( 1, 1) (A) (0, 2) (B) (1, 0)
(C) ( 1, 2) (D) ( 1, 0) (C) (0, 1) (D) (2, 0)

2. The equation of the directrix of the parabola y2 4y 9. The locus of the vertices of the family of parabolas
4 x 2  0 is
a3 x 2 a2 x
(A) x  1 (B) x  1 y 2a
3 2
3 3 is
(C) x (D) x 
2 2 3 35
(A) xy  (B) xy 
4 16
3. Two common tangents to the circle x 2 y2  2a 2 and
the parabola y2  8ax are 105 64
(C) xy  (D) xy 
64 105
(A) x  p( y 2a) (B) y  p ( x 2 a)
10. Suppose the normals at three distinct points on the
(C) x  p( y a) (D) y  p( x a) parabola y2  4ax are concurrent at point (h, k).
Then
4. The line 2bx 3cy 4d  0 passes through the
points of intersection of the parabolas x2  4ay and (A) 0  h  1 (B) 1  h  2
y2  4ax (a x 0). Then (C) h  2 (D) h  1
(A) a2 (3b − 2c)2  0 (B) d2 (3b 2c)2  0
11. If normals are drawn from the point P(h, k) to the pa-
(C) d2 (2b − 3c)2  0 (D) d2 (2b 3c)2  0 rabola y2  4ax, then the sum of the intercepts which
the normals cut off from the axis of the parabola is
5. Let P be the point (1, 0) and Q be a variable point on
the parabola y2  8x. Then, the locus of the midpoint (A) h a (B) 3(h a)
of the segment PQ is (C) 2(h a) (D) 0
(A) y2 4x 2  0 (B) y2 4x 2  0
12. P, Q and R are the feet of the normals drawn to the
(C) x2 4y 2  0 (D) x2 4y 2  0 parabola ( y 3)2  8( x 2). Then, the circumcircle
of ΔPQR passes through the point
6. The equation of a tangent to the parabola y2  8x is
y  x 2. The point on this line from which we can (A) (2, 3) (B) (3, 2)
draw the other tangent to the parabola which is per- (C) (0, 3) (D) (2, 0)
pendicular to the given tangent is
(A) (2, 4) (B) ( 2, 0) 13. If the normals at the extremities of the latus rectum
of the parabola y2  4ax meet the parabola at N and
(C) ( 1, 1) (D) (2, 0) Na, then the length NNa is
7. The angle between the tangents drawn to the curve (A) 10a (B) 20a (C) 4a (D) 12a
y  x2 5x 6 at the points (2, 0) and (3, 0) is
14. A line having slope m, and passing through the focus
P P P
(A) O (B) (C) (D) of the parabola y2  4(x 1) intersects the curve in
2 6 4 two distinct points. Then
Exercises 279

(A) m 1 TP is a parabola whose equation is

(B) m 1 (A) y2  4ax (B) y2  9ax


(C) m can be real number x 0 (C) y2  16ax (D) y2  12ax
(D) m must be rational 22. A tangent at P(h, k) (1  h  4) to the parabola
y2  4ax meets the axis at T. The line PN is drawn
15. The maximum value of a such that the circle x2 y2 
perpendicular to the axis. Then, the maximum pos-
a2 completely lies within the parabola y2  4 (x 4) is sible area of ΔPTN is
(A) 2 3 (B) 4 (A) 16 (B) 24 (C) 8 (D) 32
(C) 4 3 (D) 4 6
23. If y 3  m(x 2) touches the parabola y2  8x, then
16. The length of the shortest normal chord of the pa- m has two values m1 and m2 such that
rabola y2  4ax is
(A) m1 m2  0 (B) m1m2  2
(A) 9a (B) a 54 (C) m1m2  1 (D) m1 m2  2
(C) a 3 (D) 6a
24. y2  4ax is a variable parabola (i.e., a is a parame-
17. Tangent and normal are drawn at the point P(16, 16) ter) with focus Sa is such that for any point P on the
to the parabola y2  16x, which cut the axis of the pa- parabola, the distance PSa is equal to a constant k.
rabola at A and B, respectively. Then, the line joining Then, the locus of point P is
the point P and the circumcentre of ΔPAB makes the
(A) 4 x 2 y2 4kx  0 (B) 4 x 2 y2 4kx  0
angle with the axis of the parabola whose value is
1 (C) x 2 y2 4kx  0 (D) x 2 2 y2 4kx  0
1
(A) Tan 1 2 (B) Tan
2 (Hint: SP  x a)
¥ 4´ ¥ 3´
(C) Tan 1 ¦ µ (D) Tan 1 ¦ µ 25. The line lx my n  0 meets the parabola at points
§ 3¶ § 4¶
P and Q. The lines joining the points P and Q to the
18. The normals at the points (x1, y1) and (x2, y2) to the focus meet the parabola at Pa and Qa. Then, the equa-
parabola y2  4x meet at a point on the curve. If x1 tion of the line PaQa is
x2  4, then the value of y1 y2 is equal to (A) nx ly ma  0 (B) nx my la  0
(A) 2 (B) 2 2 (C) 4 (D) 4 2 (C) nx my la  0 (D) nx my la  0

19. PQ and PaQa are normal chords of the parabola y2  26. If (x1, y1) and (x2, y2) are extremities of a local chord
4ax. If the four points P, Q, Pa and Qa are concyclic, of the parabola y2  4x, then x1x2 y1 y2 is equal to
then the tangents at Q and Qa intersect on the (A) 3a2 (B) 2a2 (C) 2a2 (D) 3a2
(A) tangent at the vertex
(B) axis of the parabola 27. The locus of the foot of the perpendicular from
the vertex onto a chord of the parabola y2  4x
(C) directrix subtending an angle 45° at the vertex is the curve
(D) latus rectum ( x 2 y2 4 x)2  k( x 3 xy2 y2 ) where the value
of k is
20. The locus of the centre of the circle which cuts the
parabola y2  4x orthogonally at (1, 2) is the (A) 4 (B) 8 (C) 16 (D) 2

(A) line y  x 1 28. Slope of a chord PQ of the parabola y2  4ax is m


(B) line y  2x 1 (constant). Then, the normals at P and Q intersect on
(C) parabola y2  2x a normal to the parabola y2  4ax at a point whose
coordinates are
(D) circle ( x 1)2 ( y 2)2  5
¥ 4a 4a ´ ¥ 2a 4a ´
(A) ¦ 2 , (B) ¦ 2 ,
21. Let P be a point on the curve y2  8ax and T be the § m m µ¶ § m m µ¶
foot of the perpendicular drawn from the focus onto
the tangent at P. Then, the locus of the midpoint of ¥ 4a 2a ´ ¥ 4a 4a ´
(C) ¦ 2 , (D) ¦ 2 ,
§ m m µ¶ § m m µ¶
280 Chapter 4 Parabola

29. The normals at P and Q on y2  4ax meet again on 1


the parabola at R. Then, the locus of the orthocentre (A) 1 (B) 2 (C) 8 (D)
2
of ΔPQR is
35. PQ is a variable focal chord of the parabola y2  8x
(A) y2  a(x 6a) (B) y2  a(x 6a)
and O(0, 0) is its vertex. Then the locus of the cen-
(C) y2  a(x 4a) (D) x2 y2  2a2 troid of ΔOPQ is a parabola whose latus rectum is
30. If two different tangents of y2  4x are normals to the 8 4 2
(A) (B) 3 (C) (D)
parabola x2  4ay, then 3 3 3
1 1 36. Which of the following equations represent a common
(A) a  (B) a 
2 2 2 tangent to the parabolas y2  4ax and x2  32ay?
1 1 (A) x 2y 4a  0 (B) x 2y 4a  0
(C) a  (D) a 
2 2 2 (C) x 2y 4a  0 (D) x 2y 4a  0
31. The length of the normal chord of y2  4x which sub- 37. Image of the directrix of the parabola y2  4 (x 1) in
tends a right angle at the vertex is the line x 2y 3  0 is
(A) 8 2 (B) 8 3 (C) 6 3 (D) 4 2 (A) 3x 4y 16  0 (B) 3x 4y 16  0
32. Consider the following two statements: (C) x 2  0 (D) x 2  0
S1: The curve y  x 2 / 2 x 1 is symmetric about 38. Let PSPa be a focal chord, M be the foot of the per-
the line x  1. pendicular drawn from P onto the directrix of the
S2: A parabola is always symmetric about its axis. parabola y2  4ax and R be the midpoint of SM.
Then, which one of the following is true? Then, the angle between PR and SM is
(A) Both S1 and S2 are true. (A) 90° (B) 60° (C) 45° (D) 30°
(B) Both S1 and S2 are false.
39. PSQ is the focal chord of the parabola y2  4ax, where
(C) S1 is true and S2 is false.
S is the focus. If SP  4, SQ  5, then the latus rectum
(D) S1 is false and S2 is true. of the parabola is
33. If the normals to y2  4ax at points P and Q intersect
9 80
(A) 80 (B) 9 (C) (D)
at a point R(h, k) on the parabola, then the ordinates 80 9
of the point P and Q are roots of the equation 40. The parabola y  (a b) x 2 (b c) x (c a)  0 is
2 2 2 2
(A) y ky 8a  0 (B) y ky 8a  0 having the line x 1  0 as a tangent. Then the equa-
2 2 tion ax by c  0 represents
(C) y ky 8a  0 (D) y2 ky 4a 2  0
(A) a family of lines passing through a fixed point.
34. The normal at point A on the parabola y2  4ax cuts (B) a family of parallel lines.
the parabola again at point B. If the chord AB sub-
(C) a family of lines concurrent on the directrix.
tends a right angle at the vertex of the parabola, then
the square of the slope of the chord AB is (D) a family of lines concurrent at the focus of the
parabola.

Multiple Correct Choice Type Questions


1. PQ is a normal chord (normal at P) of the parabola (B) Common tangent to all the circles described on
y2  4x such that PQ subtends right angle at the focal chords as diameters is x 2  0.
vertex. Then the co-ordinates of P are (C) Focus of the parabola is (2, 0).
(A) (2, 2 2 ) (B) (2, 2 2 ) (D) Directrix of the parabola is x 2  0.
(C) (3, 2 3 ) (D) (3, 2 3 )
3. For the parabola having (2, 0) as its vertex and y-axis
as directrix, which of the following are true?
2. Consider the parabola y2  8x. Then which of the fol-
lowing are true? (A) Focus is (4, 0) (B) Focus is (−4, 0)
(A) The length of the focal chord having (2, 4) as one (C) Focus (8, 0) (D) Focus (−8, 0)
extremity is 8.
Exercises 281

4. If the circle x 2 y2 2bx  0 touches the parabola 1


(A) PQ makes angle Tan 2 with the axis
y2  4ax, then
−1
(B) PQ makes angle Tan 2 with the axis
(A) a  0, b  0 (B) a  0, b  0
(C) Length of the chord PQ is 6a 3
(C) a  0, b  0 (D) a  0, b  0
(D) Length of the chord PQ is 4a 2
5. The straight line x y 1  0 touches the parabola
8. The locus of the point of intersection of perpendicu-
(A) x 2  4 y lar normals to the parabola y2  4ax is another pa-
2 rabola with
¥ 1´ ¥ 1´
(B) ¦ x µ  ¦ y µ (A) Vertex at (2a, 0) (B) Vertex at (3a, 0)
§ 2¶ § 4¶
¥ 1´ (C) Latus rectum a (D) Directrix as x  3a
(C) ( x 1)2  2 ¦ y µ
§ 2¶
9. PQ is a normal chord of y2  4ax, normal at P. If the
2
(D) 4 x 3 x y  0 point P lies on the line y  x, then
(A) P  (4a, 4a)
6. PQ is a double ordinate of the parabola y2  4x. If
the normal at P meets the line passing through Q (B) Q  (9a, 6a)
and parallel to the axis at G, then the locus of G is a (C) Q  (9a, −6a)
parabola with (D) PQ subtends right angle at the focus
(A) Latus rectum 4
10. A square has one vertex at the vertex of the pa-
(B) Vertex at (4, 0)
rabola y2  4ax and the diagonal through this vertex
(C) Directrix as x − 3  0 lies along the axis of the parabola. If the ends of the
(D) Focus (5, 0) other diagonal lie on the parabola, then these two
vertices are
7. If PQ is the shortest normal chord of the parabola
(A) (0, 0) (B) (8a, a)
y2  4ax, then
(C) (4a, 4a) (D) (4a, −4a)

Matrix-Match Type Questions


In each of the following questions, statements are given in parallel to axis at G, then the locus of G is a parabola.
two columns, which have to be matched. The statements in For this parabola, match the items of Column I with
column I are labeled as (A), (B), (C) and (D), while those those of Column II.
in column II are labeled as (p), (q), (r), (s) and (t). Any
given statement in column I can have correct matching with Column I Column II
one or more statements in column II. The appropriate bub- (A) Length of the latus rectum of the (p) 5
bles corresponding to the answers to these questions have locus of G
to be darkened as illustrated in the following example. (q) 3
(B) Abscissa of the vertex
Example: If the correct matches are (A) m (p), (s), (B) m (C) Abscissa of the focus (r) 4
(q), (s), (t), (C) m (r), (D) m (r), (t), that is if the matches
(D) The directrix is x  a where a is (s) 6
are (A) m (p) and (s); (B) m (q), (s) and (t); (C) m (r);
equal to (t) 2
and (D) m (r), then the correct darkening of bubbles will
look as follows:
2. The focus of the parabola x 2 L y 3  0 is (0, 2).
p q r s t
Match the items of Column I with those of Column II.
A

B Column I Column II
C (A) The value of K is (p) 2
D (B) Latus rectum of the parabola is (q) 1
1. PQ is a double ordinate of the parabola y2  4x. If (Continued)
the normal at P meets the line passing through Q and
282 Chapter 4 Parabola

Column I Column II Column I Column II


(C) If k is the ordinate of the vertex, (r) 4 (A) LLa is the latus rectum of the (p) (2,0)
then 2k is equal to (s) 3 parabola. The tangent and
(D) The directrix equation is y  b, normal at L meet the axis at T
(t) 6
where 3b is equal to and N, respectively. Then, the
coordinates of the midpoint of
3. Suppose the line y  2 is the directrix and the point TN is
(0, 1) is the vertex of the parabola x 2 by c  0. (B) All variable chords of the parab- (q) (2,1)
Match the items of Column I with those of Column II. ola subtending right angle at the
vertex are concurrent at the
Column I Column II point
(A) Value of b is (p) 0 (C) If variable chords of the parab- (r) (12,0)
ola pass through a fixed point Q
(B) Value of c is (q) −4 on the axis such that the sum of
(C) Length of the latus rectum is (r) 2 the squares of the reciprocals of (s) (6,0)
(D) The ordinate of the focus is (s) −2 the two parts of chords through
Q is constant then the coordi-
(t) 4 nates of point Q are (t) (3,0)
(D) Focus of the parabola is
4. Consider the parabola y2  12x and match the items of
Column I with those of Column II.

Comprehension Type Questions


1. Passage: Consider the parabola y2  8x and answer
(A) 2 2 (B) 2 3 (C) 4 2 (D) 8 2
the following questions.
(iii) The radius of the maximum circle touching the
(i) TP and TQ are tangents to the parabola and the parabola at the vertex which is having centre
normals at points P and Q meet at a point R on on the axis of the parabola and lies completely
the curve. Then, the circumcentre of $TPQ lies within the parabola is
on the parabola whose equation is
(A) 2 (B) 4 (C) 4.5 (D) 2.5
(A) y2  2( x 2) (B) y2  x 2
(C) y2  4( x 2) (D) y2  2( x 4) 3. Passage: Consider the parabola y2  16x. Answer the
following questions.
(ii) If t1 and t2 are the parameters of points P and Q,
then the sum of the products of their abscissae (i) Let P(1, 4), Q(1/ 4, 2) and R(4, 8) be three points
and ordinates is equal to on a parabola. Then the area of the triangle
formed by the tangents to the parabola at points
(A) 48 (B) 32 (C) 16 (D) 64 P, Q and R is
(iii) The abscissa of the point of intersection of the 3 3
tangents at points P and Q is (A) (B) (C) 1 (D) 2
2 4
(A) 2 (B) 6 (C) 8 (D) 4
(ii) The locus of the midpoints of chords of the pa-
rabola which subtend right angle at the vertex is
2. Passage: Consider the parabola y2  4x and answer
the following questions: (A) y2  4( x 16) (B) y2  8( x 4)
(i) PQ is a double ordinate of length 8 units. If O is (C) y2  8( x 16) (D) y2  8( x 8)
the vertex of the parabola, then POQ is
(iii) P is a point on a parabola, M is the foot of the
(A) 45° (B) 65° (C) 90° (D) 105° perpendicular drawn from P onto the directrix
and S is the focus. If $SPM is equilateral, then
(ii) The length of the side of an equilateral triangle
the focal radius SP is equal to
inscribed in the parabola with one vertex at
(0, 0) is (A) 16 (B) 32 (C) 48 (D) 4
Answers 283

Integer Answer Type Questions


The answer to each of the questions in this section is a 4. PQ is a focal chord of a parabola y2  4ax. The nor-
non-negative integer. The appropriate bubbles below the mals at P and Q meet the curve again at Pa and Qa,
respective question numbers have to be darkened. For respectively. Then PaQa is parallel to PQ and k times
example, as shown in the figure, if the correct answer to PQ where k is equal to ______.
the question number Y is 246, then the bubbles under Y
labeled as 2, 4, 6 are to be darkened. 5. The length of the normal chord of y2  4ax which
X Y Z W subtends a right angle at the focus is ka 5, where k
0 0 0 0
is ______.
1 1 1 1
6. For the parabola y2  6x, chords are drawn through
2 2 2
the fixed point (9, 5). Then the locus of the midpoints
3 3 3 3 of these chords is a parabola whose latus rectum is
4 4 4 ______.
5 5 5 5
6 6 6 7. QR is a chord of y2  4ax and is bisected by a diam-
7 7 7 7
eter through a point P at V. Then
8 8 8 8 QV 2
 ______
9 9 9 9 SP – PV
where S is the focus.
1. If the normal to a parabola at P meets the curve
again at Q and if PQ and the normal at Q make 8. The locus of the midpoints of system of parallel
angles @ and A, respectively, with the axis, then chords having slope m of the parabola y2  4ax is the
tan 2 A tan A tan B k  0 where k is equal to ______. straight line y  ka / m, where k is ______.

2. In the parabola y2  4ax, the tangent at P whose abscis- 9. The number of normals to y2  4x drawn from the
sa is equal to the latus rectum meets the axis at T and point (1, 2) is ______.
the normal at P meets the curve again at Q. Then the
ratio PT:PQ  m:n, where m n is equal to ______. 10. The tangents to the parabola y2  4x at the points
(1, 2) and (4, 4) meet on the line y  k, where k is
3. A chord PQ is normal to the parabola y2  4ax at P ______.
and subtends a right angle at the vertex. If S is the
focus, then the ratio SQ/SP is ______.

ANSWERS
Single Correct Choice Type Questions
1. (D) 11. (C)
2. (D) 12. (A)
3. (B) 13. (D)
4. (D) 14. (C)
5. (A) 15. (A)
6. (B) 16. (C)
7. (B) 17. (C)
8. (B) 18. (D)
9. (C) 19. (B)
10. (C) 20. (A)
284 Chapter 4 Parabola

21. (B) 31. (C)

22. (A) 32. (A)

23. (C) 33. (B)

24. (B) 34. (B)

25. (B) 35. (A)

26. (D) 36. (A)

27. (C) 37. (B)

28. (A) 38. (A)

29. (A) 39. (D)

30. (A) 40. (A)

Multiple Correct Choice Type Questions


1. (A), (B) 6. (A), (B), (C), (D)

2. (A), (B), (C) 7. (A), (C)

3. (A), (B) 8. (B), (C)

4. (A), (B) 9. (A), (C), (D)

5. (A), (B), (D) 10. (C), (D)

Matrix-Match Type Questions


1. (A) m (r); (B) m (r); (C) m (p); (D) m (q) 3. (A) m (t); (B) m (q); (C) m (t); (D) m (p)

2. (A) m (p), (t); (B) m (p); (C) m (s); (D) m (s) 4. (A) m (t); (B) m (r); (C) m (s); (D) m (t)

Comprehension Type Questions


1. (i) (B); (ii) (A); (iii) (D) 3. (i) (B); (ii) (C); (iii) (A)

2. (i) (C); (ii) (D); (iii) (A)

Integer Answer Type Questions


1. 2 6. 3

2. 9 7. 4

3. 3 8. 2

4. 3 9. 1

5. 5 10. 3
Ellipse and Hyperbola 5
Contents
5.1 Ellipse
5.2 Inverted Ellipse
5.3 Hyperbola

Worked-Out Problems
Directrices Summary
Exercises
Answers
Minor axis

Focal
rectum
Latus

parameter
Ellipse and Hyperbola

An ellipse is a section of a cone by a


Major axis
Linear eccentricity plane such that the plane parallel to
it through the vertex meets the plane
of the base in a line external to it.
A conic whose eccentricity is greater
than unity is known as hyperbola.
Foci

Conjugate axis
Vertex Vertex
Transverse axis

Asymptotes
286 Chapter 5 Ellipse and Hyperbola

In this chapter, we discuss about ellipse, hyperbola and their properties. ‘Subjective Problems’ sections provide worked-out
subjective problems for the preceding sections. Students are advised to solve each and every problem to grasp the topics.

5.1 Ellipse
An ellipse is a section of a cone by a plane such that the plane parallel to it through the vertex meets the plane of the
base in a line external to it. Definition 5.1 is the plane geometrical definition.

DEFINITION 5.1 Let l be a line and S be a point on the plane of l, but not on l. Let 0  e  1 be a fixed number.
Then the locus of the point whose distance from the point S is e times the distance of the point
from the line l is called ellipse. The point S is called the focus and l is called the directrix cor-
responding to S.

T H E O R E M 5.1 The standard equation of an ellipse is


x2 y2
1
a2 b2
PROOF See Fig. 5.1. Let l be the directrix and S be the focus. Draw SZ perpendicular to l and divide SZ
internally and externally at A and Aa inFHH E ratio e : 1 (where 0  e  1). Let C be the midpointFHH
the of
E
Aa A and AaC  CA  a. We consider CS as x-axis and line through C and perpendicular to CS
as y-axis (see Fig. 5.1). We can see that C  (0, 0), A  (a, 0) and Aa  (−a, 0). Also SA  e(AZ)
and SAa  eAaZ. Therefore,
SA SAa  e(AZ AaZ)
 Aa A  e(CZ CA CZ CAa)  e(2CZ )
 2a  (2e) CZ
a
 CZ 
e
Hence, the equation of the directrix l is
a
x (5.1)
e
Also
SAa − SA  e(AaZ − AZ)
 (CS CAa) (CA CS)  e( Aa A)
 2(CS)  e(2a)
 CS  ea
 S  (ae, 0)
So the focus is
S  (ae, 0) (5.2)
Let P(x, y) be a point on the ellipse. Draw PN perpendicular to x-axis and PM perpendicular to
the directrix l. Hence, by definition,
SP  e(PN)
a
 ( x ae)2 y2  e x
e
 ( x ae)2 y2  (ex a)2
 (1 e 2 ) x 2 y2  a 2 (1 e 2 )
x2 y2
 1
a2 a 2 (1 e 2 )
5.2 Inverted Ellipse 287

Since 1 − e2  0, we write a2(1 − e2) as b2 where b  0. Hence, the locus of P is

x2 y2
1
a2 b2
y

l
B
P
M M

Z A S C S N A Z x

B

FIGURE 5.1

QUICK LOOK 1

x2 y2 5. b  a.
The properties of the curve  1 are as follows: 6. If P(x, y) is a point on the ellipse
a2 b2
1. The curve is symmetric about both axes. x2 y2

2. For any point (x, y) on the curve, we have a b x b a a2 b2
and b b y b b. then we have
3. The x-axis meets the curve at A(a, 0) and Aa(−a, 0).
SP  e(PM)  e(NZ)  e(CZ − CN)
The y-axis meets the curve at B(0, b) and Ba(0, −b).
4. For each value of x, ¥a ´
 e ¦ xµ  a ex
§e ¶
x2 7. Since the curve is symmetric about both axes, there
y pb 1 2
a must be second focus and directrix. Another focus
Sa(−ae, 0) and its corresponding directrix is
and for each value of y,
a
y2 x
x  pa 1 e
b2

DEFINITION 5.2 Major Axis, Minor Axis and Vertices In Fig. 5.1, AAa is called the major axis and BBa [where
B  (0, b) and Ba  (0, − b)] is called the minor axis. The two points A and Aa are called vertices
of the ellipse. 2a is called the length of the major axis and 2b is called the length of the minor
axis.

5.2 Inverted Ellipse


If b2  a2, then the ellipse

x2 y2
1
a2 b2
288 Chapter 5 Ellipse and Hyperbola

is called an inverted ellipse. That is, if the roles of major and minor axes are interchanged, the ellipse is called an
inverted ellipse. For the inverted ellipse, the vertices are (0, b) and (0, −b), major axis length is 2b and minor axis
length is 2a (see Fig. 5.2). Foci are (0, be) and (0, −be) and directrices are
b
y p
e
The eccentricity is given by a2  b2(1 − e2).
y

Z M y b
e
B
Z P
S

A C A x

S

B
Z M y  b
e

FIGURE 5.2

DEFINITION 5.3 Double Ordinate and Latus Rectum If a line perpendicular to major axis meets the curve
at P and Q, then PQ is called double ordinate. Double ordinate through focus is called latus
rectum.

T H E O R E M 5.2 The length of the latus rectum of the ellipse

x2 y2
1
a2 b2
is

2b2
a
PROOF Let L(ae, y) be one end of the latus rectum through the focus (ae, 0). Hence

a2 e2 y2
1
a2 b2
so that

b4
y2  b2 (1 e 2 )  [& b2  a 2 (1 e 2 )]
a2
Thus

b2
y
a

¥ b2 ´ ¥ b2 ´
and L  ¦ ae, µ , L a  ¦ ae,
§ a¶ § a µ¶
5.2 Inverted Ellipse 289

QUICK LOOK 2

x2 y2 2a 2
If b2  a2, then the length of the latus rectum of is .
a2 b2 b

DEFINITION 5.4 Centre For any conic, point C is called its centre if C bisects every chord of the conic passing
through it. For the standard ellipse, C(0, 0) is its centre.

Example 5.1

Find the equation of the ellipse whose focus is at (4, −3),  5( x 8)2 9( y 3)2  180
directrix is x 1  0 and eccentricity is 2/3.
( x 8)2 ( y 3)2
 1
Solution: Let P  (x, y). Now 36 20
SP  ePM Hence, the centre is (8, −3). Foci are
2
¥ 2´
 (SP )2  ¦ µ ( PM )2 ¥ ¥ 2´ ´
§ 3¶ ¦§ x 8  p 6 ¦§ 3 µ¶ , y 3  0µ¶  (12, 3) and (4, 3)
4
 ( x 4)2 ( y 3)2  ( x 1)2 Directrices are
9
 9( x 4) 9( y 3))  4( x 1)2
2 2 a
x 8p p 9
 5 x 2 9 y2 80 x 54 y  221 e
 x  17 and x  −1
 5( x 2 16 x 64) 9( y2 6 y 9)  221 320 81

Examples 5.2

For the ellipse 9x2 16y2  576, find the centre, foci, Centre is (0, 0). Foci are
directrices and latus rectum.
¥ 7´
( p ae, 0)  (p8 ¦ µ , 0)  (p2 7 , 0)
Solution: The given equation can be written as § 4 ¶

x 2 y2 The directrices are


1
64 36 a ¥ 4 ´ 32
xp  x p 8¦ µ p
Here, a2  64, b2  36. Now e § 7¶ 7
36  b2  a2(1 − e2)  64(1 − e2) The latus rectum is

 e2  1
36
 e
7 2b2 2(36)
 9
64 4 a 8

QUICK LOOK 3 (IMPORTANT OBSERVATION)

From the equation of the ellipse, we have the following observation: If the
perpendicular distances p1, p2 of a moving point P from
x2 y2 two perpendicular lines, say, l and la, respectively, are
1
a2 b2 connected by the equation
290 Chapter 5 Ellipse and Hyperbola

p12 p22 where a  b, the locus of P is an ellipse with centre at


1 the intersection of the lines l and la and having major
a2 b2 and minor axes along the lines la and l, respectively, and
they are of lengths 2a and 2b, respectively.

The following example illustrates the concept explained in Quick Look 3.

Example 5.3

If the focus, centre and eccentricity of an ellipse are (2, Hence


3), (3, 4) and 1/2, respectively, then find the equation of
the ellipse. a2
(3 2)2 (4 3)2  CS 2 
4
Solution: It is known that the major axis is along the a 2 2
line joining the centre (2, 3) and the focus (3, 4). Its equa-
tion is Now,
x − y 1  0 = la (say) ¥ 1´
b2  a 2 (1 e 2 )  8 ¦ 1 µ  6  b  6
The minor axis is the line through (2, 3) and perpendicu- § 4¶
lar to the major axis. Its equation is Let p1 and p2 be the distances of P(x, y) from la to l.
x y − 5  0 = l (say) Then
Now C(2, 3) is the centre and S(3, 4) is the focus. There- [( x y 1)/ 2 ]2 [( x y 5)/ 2 ]2
fore 1
6 8
a ( x y 1)2 ( x y 5)2
CS  ae  CS   1
2 12 16

Example 5.4

Find the centre and eccentricity of the ellipse which is


2 2
3(3 x 2 y 4) 2(2 x 3 y 5)  26 ¥ 2 23 ´
¦§ , µ¶
13 13
Solution: The given equation can be written as
Here,
(3 x 2 y 4)2 (2 x 3 y 5)2
1 26 2 26
(26 / 3) (26 / 2) b2  a 
2 3
Therefore, the centre of the ellipse is the point of inter-
so that
section of the lines
3x − 2y 4  0 1 1 1
a 2  b2 (1 e 2 )   (1 e 2 )  e 
3 2 3
and
2x 3y − 5  0

DEFINITION 5.5 Internal and External Points of Ellipse Since an ellipse is a closed curve, any point on the
plane of the ellipse belonging to the foci region is called internal point. Any point outside this
is called an external point.

Notations Used:

x2 y2
Sy 2
1
a b2
5.2 Inverted Ellipse 291

xx1 yy1
S1  2
1
a b2
xx2 yy2
S2  1
a2 b2
x1 x2 y1 y2
S12  S21  2
1
a b2
x12 y12
S11  1
a2 b2

QUICK LOOK 4

As in the case of circle and parabola, for the case of 2. Point (x1, y1) is an internal point
ellipse also, we have the following:
x12 y12
1. Point (x1, y1) is an external point to š S11  1 0
a2 b2
x2 y2 x12 y12
Sy 1  0 š S11  1 0
a2 b2 a2 b2

DEFINITION 5.6 Auxiliary Circle The circle described on the major axis as diameter is called auxiliary circle.
That is, for the ellipse,

x2 y2
1
a2 b2
where a  b, the circle x2 y2  a2 is called the auxiliary circle (see Fig. 5.3).
y
Auxilary circle

q 90
A C N A x

B

FIGURE 5.3

T H E O R E M 5.3 Every point on the ellipse


(P A R A M E T R I C
EQUATIONS) x2 y2
1
a2 b2
is represented by (a cos P, b sin P), where P is real.
292 Chapter 5 Ellipse and Hyperbola

PROOF Let P(x, y) be a point on the ellipse

x2 y2
1
a2 b2
Draw PN perpendicular to the major axis (i.e., x-axis) and produce NP to meet the auxiliary
circle at Q (see Fig. 5.3). Let P be ACQ. Hence, by Theorem 3.4, Chapter 3, Q  (a cos P,
a sin P). Thus, P  (a cos P, y) and P lies on the curve given by

a 2 cos2 Q y2
 1  y  p b sin Q
a2 b2
Hence, P  (a cos P, b sin P). Since the ellipse is symmetric about x-axis, Pa(a cos P, −b sin P) is
also a point on the curve. Conversely, if x  a cos P and y  b sin P, then

x2 y2
1
a2 b2
The equations x  a cos P, y  b sin P are called the parametric equation of the ellipse

x2 y2
1
a2 b2

DEFINITION 5.7 Eccentric Angle The angle Q  ACQ in Theorem 5.3 is called the eccentric angle of the
point P where P  (a cos P, b sin P).
Hereafter, when we say that P is a point on the ellipse

x2 y2
1
a2 b2
it means that its coordinates are (a cos P, b sin P) (we can assume that 0 b Q b 2P ).

T H E O R E M 5.4 1. The equation of the chord joining two points A(x1, y1) and B(x2, y2) on the ellipse
(C H O R D
EQUATION) x2 y2
Sy 2
1 0
a b2
is S1 S2  S12 and hence the equation of the tangent at (x1, y1) is
xx1 yy1
S1 y 2
1 0
a b2
2. The equation of the chord joining two points @ and A on the ellipse

x2 y2
1
a2 b2
is given by
x ¥ A B´ y ¥ A B´ ¥ A B´
cos ¦ µ¶ sin ¦§ µ¶  cos ¦§ µ
a § 2 b 2 2 ¶
and the equation of the tangent at @ is given by
x y
cos A sin A  1
a b
PROOF 1. Points A(x1, y1) and B(x2, y2) are on the ellipse. Therefore
5.2 Inverted Ellipse 293

x12 y12 x22 y22


 1
a 2 b2 a 2 b2
x12 x22 y22 y12
 0
a2 b2
b2 ( x x ) y y
 2 1 2  1 2  Slope of the chord AB
a ( y1 y2 ) x1 x2

Therefore, the equation of the chord AB is

b2 ( x1 x2 )
y y1  ( x x1 )
a 2 ( y1 y2 )

 b2 ( x x1 )( x1 x2 ) a 2 ( y y1 )( y1 y2 )  0
 (b2 xx1 a 2 yy1 ) (b2 xx2 a 2 yy2 ) (b2 x1 x2 a 2 y1 y2 ) (b2 x12 a 2 y12 )  0
¥ xx yy ´ ¥ xx yy ´ ¥ x x y y ´ ¥x 2 y 2´
 ¦ 21 21 µ ¦ 22 22 µ ¦ 1 2 2 1 2 2 µ ¦ 12 22 µ  0
§ a b ¶ § a b ¶ § a b ¶ §a b ¶
2
¥ xx yy ´ ¥ xx yy ´ ¥x y2 ´ x x yy
 ¦ 21 21 1µ ¦ 22 22 1µ ¦ 12 12 µ  1 2 2 1 2 2 1
§ a b ¶ § a b ¶ §a b ¶ a b
 S1 S2 1  S12 1
 S1 S2  S12

When x2  x1 and y2  y1, the equation of the tangent at (x1, y1) is


xx1 yy1
S1 y 1 0
a2 b2
2. Let P  (a cos @, b sin @) and Q  (a cos A, b sin A). The equation of the chord PQ is
b(sin A sin B )
y b sin A  ( x a cos A )
a(cos A cos B )
2b cos[(A B )/ 2]sin [(A B )/ 2]
 ( x a cos A )
2a sin[(A B )/ 2]sin[(A B )/ 2]
b cos [(A B )/ 2]
 s ( x a cos A )
a sin[(A B )/ 2]
Therefore

¥ A B´ ¥ A B´ ¨ ¥ A B´ ¥ A B´ ·
xb cos ¦ ya sin ¦  ab ©cos A cos ¦ µ¶ sin A sin ¦§
§ 2 µ¶ § 2 µ¶ ª § 2
µ
2 ¶ ¸¹
¥ A B´
 ab cos ¦
§ 2 µ¶

Dividing both sides by ab, we get


x ¥ A B´ y ¥ A B´ ¥ A B´
cos ¦ sin ¦ µ  cos ¦
a
µ
§ 2 ¶ b § 2 ¶ § 2 µ¶

If A  @, then the equation of tangent at (a cos @, a sin @) is


x y
cos A sin A  1
a b
294 Chapter 5 Ellipse and Hyperbola

DEFINITION 5.8 Normal If P is any point on an ellipse, then the line passing through point P and perpendicu-
lar to the tangent at P to the ellipse is called normal at point P.

T H E O R E M 5.5 Let
x2 y2
S 1 0
a2 b2
be an ellipse. Then
1. The equation of the normal at P(x1, y1) is

a 2 ( x x1 ) b2 ( y y1 )

x1 y1

2. The equation of the normal at P(a cos @, b sin @) is ax sec @ − by cosec @  a2 − b2.
PROOF 1. From Theorem 5.4, the equation of the tangent at (x1, y1) is
xx1 yy1
Sy 2
1 0
a b2
Its slope is
b2 x1
a 2 y1

Hence, the equation of the normal at (x1, y1) is

a 2 y1
y y1  ( x x1 )
b2 x1

a 2 ( x x1 ) b2 ( y y1 )
 
x1 y1

2. Since the tangent at P(a cos @, b sin @) is


x y
cos A sin A  1
a b
its slope is
b
cot A
a
Hence, the equation of the normal at P is
a sin A
y b sin A  ( x a cos A )
b cos A
ax by
  a 2 b2
cos A sin A
 ax sec A by cosec A  a 2 b2

T H E O R E M 5.6 1. The condition for the line y  mx c to touch the ellipse

x2 y2
2
1
a b2
is c2  a2m2 b2.
5.2 Inverted Ellipse 295

2. The condition for the line x cos Q y sin Q  p to touch the ellipse is

p  a 2 cos2 Q b2 sin 2 Q

PROOF 1. Suppose the line y  mx c, c x 0 touches the ellipse at (x1, y1). However, the equation of the
tangent at (x1, y1) (by Theorem 5.4) is
xx1 yy1
2
1 0
a b2
Since both equations represent the same line, we have
( x1 /a 2 ) ( y1 /b2 ) 1
 
m 1 c
2
a m b2
 x1  and y1 
c c
Since (x1, y1) lies on the curve, we have

x12 y12
1
a2 b2
a 4 m2 b4
 1
c2 a2 c 2 b2
 a 2 m2 b2  c 2
So
¥ a 2 m b2 ´
(x1 , y1 )  ¦ , µ
§ c c¶
Aliter: The abscissae of the points of intersection of the line and the ellipse are given by the
quadratic equation

x2 (mx c)2
1
a2 b2
 (b2 a 2 m2 ) x 2 2a 2 mcx c 2 a 2 a 2 b2  0 (5.3)
The line y  mx c touches the ellipse if and only if the roots of Eq. (5.3) are equal. That is if
and only if the discriminant is zero.

 4a 4 m2 c 2 4a 2 (c 2 b2 )(b2 a 2 m2 )  0
 a 2 m2 c 2 (c 2 b2 )(b2 a 2 m2 )  0
 b 2 c 2 b 2 (b 2 a 2 m 2 )  0
 c 2  a 2 m 2 b2
2. In the above condition, if we replace c with p/sin Q and m with −cot P, we have

p2 a 2 cos2 Q b2 sin 2 Q
 a 2 cot 2 Q b2 
sin 2 Q sin 2 Q
 p  a 2 cos2 Q b2 sin 2 Q

QUICK LOOK 5

1. For all values of m, the line y  mx p a 2 m2 b2 x2 y2


2
1
touches the ellipse a b2
296 Chapter 5 Ellipse and Hyperbola

2 2 touches the ellipse at


at ( a m /c, b /c), where c  p a 2 m2 b2
¥ a 2 cos Q b2 sin Q ´
¦ ,
§ p p µ¶
2. For all values of P, the line

x cos Q y sin Q  a 2 cos2 Q b2 sin 2 Q

T H E O R E M 5.7 From an external point, two tangents can be drawn to an ellipse.


PROOF Let P(x1, y1) be an external point to the ellipse

x2 y2
Sy 1 0
a2 b2
so that

x12 y12
S11  1 0 (5.4)
a2 b2

From Quick Look 5, we have that the line y  mx a 2 m2 b2 touches the ellipse. This line
passes through the point (x1, y1)

š y1 mx1  a 2 m2 b2
š ( y1 mx1 )2  a 2 m2 b2
š (a 2 x12 )m2 2 x1 y1 m b2 y12  0 (5.5)
This being a quadratic in m has two roots, say, m1 and m2, so that

2 x1 y1 b2 y 2
m1 m2  and m1 m2  (5.6)
a2 x 2 a 2 x12
Correspondingly, there are two tangents through (x1, y1). You can see that the discriminant of Eq.
(5.5) is

4 x12 y12 4(a 2 x12 )(b2 y12 )  4[ a 2 b2 a 2 y12 b2 x12 ]


¥ x 2 y2 ´
 4a 2 b2 ¦ 12 12 1µ
§a b ¶
 4a 2 b2 S11  0 [from Eq . (5.4)]

T H E O R E M 5.8 The locus of the point through which perpendicular tangents are drawn to the ellipse
x2 y2
Sy 1 0
2

a b2
is the circle x2 y2  a2 b2. This circle is called the director circle of the ellipse.
PROOF We have
The tangents are at right angles

b2 y12
š 1  m1 m2  [from Eq . (5.6) of Theorem 5.7]
a 2 x12
š a 2 x12  b2 y12
š x12 y12  a 2 b2
5.2 Inverted Ellipse 297

Therefore, the locus of (x1, y1) is x2 y2  a2 b2.


Aliter: The perpendicular tangents to the ellipse are

y  mx a 2 m2 b2  y mx  a 2 m2 b2

x a2
and y 2
b2  my x  a 2 b2 m2
m m
Squaring and adding on both sides, we have
(1 m2)(x2 y2)  (a2 b2)(1 m2)
Therefore, the locus of (x, y) is x2 y2  a2 b2.

DEFINITION 5.9 Director Circle The locus of the point through which perpendicular tangents are drawn to
an ellipse is a circle concentric with ellipse. This circle is called the director circle of the el-
lipse

x2 y2
1
a2 b2

QUICK LOOK 6

The equation of the director circle of the ellipse is given by

x2 y2 x 2 y 2  a 2 b2
1
a2 b2

Example 5.5

Find the director circle of the ellipse 9 x 2 16 y2  144. Hence, the director circle equation is
x2 y2  16 9  25
Solution: The given ellipse is

x 2 y2
1
16 9

Example 5.6

1. Prove that the condition for the line lx my n  0 may Solution:


x 2 y2 2 2 2 2 2 1. The line lx my n  0 can be written as
touch the ellipse  1 is that a l b m  n .
a 2 b2
¥ l ´ ¥ n ´
y  ¦ µ x ¦ µ
2. If § m¶ § m¶

a2 b2 (a 2 b 2 ) 2 where m x 0, and this line touches the ellipse. So from


2
2
 2 Theorem 5.6, we have
l m n
2
then prove that the line lx my  n is normal to n2 ¥ l ´
 a 2 ¦ µ b2
x2 y2 m 2 § m¶
 1.
a2 b2  n 2  a 2 l 2 b2 m 2
298 Chapter 5 Ellipse and Hyperbola

2. Suppose lx my  n is normal at (a cos P, b sin P) at


a ¥ a 2 b2 ´ b ¥ a 2 b2 ´
which the normal equation is ax sec Q by cosec Q   ¦ cos Q and  sin Q
a2 b2. Therefore l § n ¶ µ m ¦§ n µ¶
2
a sec Q b cosec Q a 2 b2 a2 ¥ a 2 b2 ´
b2 (a 2 b 2 ) 2
   2 2 ¦ µ (cos2 Q sin 2 Q ) 
l m n l m § n ¶ n2

T H E O R E M 5.9 The feet of the perpendiculars drawn from the foci onto a tangent of an ellipse lie on the auxiliary
circle of the ellipse.
PROOF It is known that

y  mx a 2 m2 b2
is a tangent to the ellipse
x2 y2
1
a2 b2
The line perpendicular to the above tangent is of the form
¥ 1´
y¦ µ x c
§ m¶
This passes through the focus S(ae, 0), as shown in Fig. 5.4, which implies that
ae ae
0 c  c
m m
Therefore, the line is
x ae
y  x my  ae
m m
So

( y mx)2 ( x my)2  a 2 m2 b2 a 2 e 2
 (1 m2 )( x 2 y2 )  a 2 m2 a 2 [& b2  a 2 (1 e 2 )]
 x 2 y2  a 2
Therefore, the foot of the perpendicular from S(ae, 0) onto a tangent lies on the auxiliary circle
x2 b2  a2

P
M

p2 p1

S C S

FIGURE 5.4
5.2 Inverted Ellipse 299

T H E O R E M 5.10 The product of the perpendiculars drawn from the foci onto a tangent of the ellipse

x2 y2
1
a2 b2
is a constant and is equal to b2.
PROOF See Fig. 5.4. Let SM  p1 and SaMa  p2 be the perpendiculars drawn from S(ae, 0) and Sa(−ae, 0),
respectively, onto a tangent
x y
cos Q sin Q  1
a b
Therefore
1 e cos Q ab
p1 
a sin 2 Q b2 cos2 Q
2

1 e cos Q ab
and p2 
a sin 2 Q b2 cos2 Q
2

Now

1 e 2 cos2 Q a 2 b2
p1 p2 
a 2 sin 2 Q b2 cos2 Q
(1 e 2 cos2 Q )a 2 b2
 (& e 2 cos2 Q  1)
a 2 sin 2 Q a 2 (1 e 2 )cos2 Q
(1 e 2 cos2 Q )b2

sin 2 Q (1 e 2 )cos2 Q
(1 e 2 cos2 Q )b2
 2 2
(& sin 2 Q cos2 Q  1)
1 e cos Q
2
b

T H E O R E M 5.11 Except the point of contact, every point on a tangent line to an ellipse is an external point to the
ellipse.
PROOF From Theorem 5.4, part (2), it is known that the equation of a tangent to the ellipse

x2 y2
2
1
a b2
at (a cos Q , b sin Q ) is
x y
cos Q sin Q  1 (5.7)
a b
Let (x1, y1) be any point on this tangent so that
x1 y
cos Q 1 sin Q  1 (5.8)
a b
Now, from Eq. (5.8), we get
2
x12 y12 x12 y12 ¥x y ´
1 1 cos Q 1 sin Q µ
2 ¦
a2 b2 a2 b § a b ¶
300 Chapter 5 Ellipse and Hyperbola

2
¥x y ´
 ¦ 1 sin Q 1 cos Q µ r 0
§ a b ¶
and the equality holds if and only if
x1 y
sin Q 1 cos Q  0 (5.9)
a b
Now, from Eqs. (5.9) and Eq. (5.8), we have
x1  a cos Q
and y1  b sin Q
That is,

x12 y12
S11  1r 0
a2 b2
and the equality holds only when (x1, y1) is the point of contact.

T H E O R E M 5.12 If M(x1, y1) is the midpoint of a chord of the ellipse


(E Q U A T I O N O F
A CHORD IN x2 y2
Sy 1 0
TERMS OF ITS a2 b2
MIDPOINT)
then the equation of the chord is
S1  S11

xx1 yy1 x12 y12


 2
1
2
1 2

a b a b2
PROOF Let A(x2, y2) and B(x3, y3) be the extremities of the chord whose midpoint is M(x1, y1) so that
x2 x3  2x1 (5.10 a)
y2 y3  2y1 (5.10 b)
Since A(x2, y2) and B(x3, y3) are the points on the curve, we have

x22 y22 x32 y32


 1
a2 b2 a2 b2
( x2 x3 )( x2 x3 ) ( y3 y2 )( y3 y2 )
 
a2 b2
b 2 ( x2 x3 ) y3 y2
 2

a ( y2 y3 ) x3 x2

which is the slope of the chord AB. From Eqs. (5.10a) and (5.10b), we have

b2 (2 x1 ) y3 y2
2

a (2 y1 ) x3 x2

which is the slope of the chord AB which is written as

b2 x1
a 2 y1
5.2 Inverted Ellipse 301

Therefore, the equation of the chord AB is

b2 x1
y y1  ( x x1 )
a 2 y1
xx1 yy1 x12 y12
 
a2 b2 a2 b2
xx1 yy1 x12 y12
 2
2
1 2
1
a b a b2
 S1  S11

T H E O R E M 5.13 The midpoints of parallel chords of an ellipse are collinear and this line passes through the center
of the ellipse.
PROOF See Fig. 5.5. Let M(x1, y1) be the midpoint of a chord of the ellipse having slope m. Since the
equation of the chord is S1  S11, we have

xx1 yy1 x12 y12



a2 b2 a2 b2
The slope of this chord is

b2 x1
m
a 2 y1
 b2 x1 a 2 my1  0

Therefore, M ( x1 , y1 ) lies on the line b2 x a 2 my  0 which passes through the centre (0, 0) of the
ellipse.

FIGURE 5.5

DEFINITION 5.10 The line of collinearity of the midpoints of parallel chords of an ellipse is called the diameter
of the ellipse.

T H E O R E M 5.14 If one diameter of an ellipse bisects the chords parallel to another, then the second one bisects the
chords parallel to the first. These types of diameters are called conjugate diameters.
PROOF From Theorem 5.13, the midpoints of chords parallel to y  mx lies on the line b2x a2my  0.
Hence, the midpoints of the chords parallel to b2x a2my  0 whose slope is

b2
a2 m
lie on the line
302 Chapter 5 Ellipse and Hyperbola

¥ b2 ´
b2 x a 2 ¦ 2 µ y  0
§ a m¶
That is, the midpoints of the parallel chords lie on the line y  mx.

T H E O R E M 5.15 The combined equation of the pair of tangents drawn from an external point (x1, y1) to the
(P A I R O F ellipse
TANGENTS)
x2 y2
Sy 2
1 0
a b2
is

S12  SS11
2
¥ xx yy ´ ¥x
2
y2 ´ ¥ x 2 y2 ´
 ¦ 21 21 1µ  ¦ 2 2 1µ ¦ 12 12 1µ
§ a b ¶ §a b ¶§a b ¶
PROOF See Fig. 5.6. Let Q(x2, y2) be a point on one of the tangents from P(x1, y1). Let R divide PQ in
the ratio 1 : L , where L 1 x 0, so that
¥ Lx x Ly y ´
R¦ 1 2 , 1 2 µ
§ L 1 L 1 ¶

Now

[(L x1 x2 )/(L 1)]2 [(L y1 y2 )/(L 1)]2


R lies on the ellipse š 1
a2 b2
On simplification, we have
L 2 S11 2 L S12 S22  0 (5.11)
FHHE
PQ touches the ellipse if and only if Eq. (5.11) has equal roots
2
 4S12 4S11S22  0
2
 S12  S11S22

Therefore, the locus of Q(x2, y2) is S12  S11S.

P (x1, y1)

C Q (x2, y2)

FIGURE 5.6
5.2 Inverted Ellipse 303

DEFINITION 5.11 Chord of Contact If the tangents drawn from an external point P to an ellipse touch the
curve at A and B, then the chord AB is called chord of contact of P with respect to the given
ellipse.

QUICK LOOK 7

As in the case of circle and parabola, we can see that is given by


the equation of the chord of contact of P(x1, y1) with
xx1 yy1
respect to the ellipse S1 y 1 0
2
a b2
x2 y2
Sy 2
1 0
a b2

Example 5.7

Two points A and B are on the ellipse 2 x 2 7 y2  14 at xx1 yy1


1 (5.13)
which the tangents drawn to the ellipse intersect at P. If 7 2
the equation of line AB is 5x 7y  7, find the coordi- is the chord of contact P(x1, y1). Therefore, from Eqs.
nates of P. (5.12) and (5.13), we get
Solution: Suppose P  (x1, y1). By hypothesis, ( x1 / 7) ( y1 / 2) 1
 
5x 7y  7 (5.12) 5 7 7
 x1  5, y1  2
is the chord of contact of P. By Quick Look 7, we have
that  P  (5, 2)

Example 5.8

Find the coordinates of the midpoint of the portion of 3(4 L 2 ) 2(9 L 2 )  12 L


the straight line x y  2 intercepted by the ellipse 3x2
2y2  6.  (30)L 2  12 L
2
 L  0 or L 
Solution: Suppose M(x1, y1) is the midpoint of the 5
chord. Therefore, by Theorem 5.12, we have Now
xx1 yy1 x12 y12 Lx0  L
2

2 3 2 3 5
However, x y  2 is the equation of the chord. Therefore Hence, the midpoint is
( x1 / 2) ( y1 / 3) ( x12 / 2) ( y12 / 3) 4 6´
  L (say)  x1 , y1  (2L , 3L )  ¥¦§ , µ
1 1 2 5 5¶
Therefore, x1  2 L , y1  3L and 3x12 2 y12  12 L implies
that

T H E O R E M 5.16 Four normals can be drawn to an ellipse from a point on the plane of the ellipse. That is, if point
P is located on the plane of an ellipse, there exist four points (not necessary all distinct) on the
ellipse at which the normals drawn to the ellipse are concurrent at P.
PROOF Let

x2 y2
Sy 1 0
a2 b2
304 Chapter 5 Ellipse and Hyperbola

be the ellipse and P(h, k) be a point in the plane of the ellipse. We know that [from Theorem 5.5,
part (2)], the normal to the ellipse at P is given by

ax sec Q by cosec Q  a 2 b2

ax[1 tan 2 (Q / 2)] by[1 tan 2 (Q / 2)] 2


  a b2
1 [ tan 2 (Q / 2)] 2 tan (Q / 2)]

ax(1 t 2 ) by(1 t 2 ) 2 ¥ Q´
  a b2 ¦§ where t  tan µ¶
1 t2 2t 2

 ax(1 t 2 )2t by(1 t 4 )  2t(1 t 2 )(a 2 b2 )  2t(1 t 2 )a 2 e 2


 byt4 2(ax a2e2)t3 2(ax − a2e2)t − by  0
This normal passes through P(h, k). This implies

bkt 4 2(ah a 2 e 2 )t 3 2(ah a 2 e 2 )t bk  0 (5.14)


Since Eq. (5.14) is a fourth-degree equation in t, it has four roots t1, t2, t3 and t4 (not necessarily
distinct) so that there exist four points on the ellipse (tr , r  1, 2, 3, 4) at which the normals drawn
are concurrent at P(h, k). Also, we have the set of equations
2(ah a 2 e 2 )
t1 t2 t3 t4 
bk
3t1t2  0
2(ah a 2 e 2 )
3t1t2 t3 
bk
bk
and t1t2 t3 t4   1
bk

T H E O R E M 5.17 A circle and an ellipse intersect at four points (real or imaginary), which are not necessarily be
distinct, such that the algebraic sum of the eccentric angles of the four points is an even multiple
of O.
PROOF Let

x 2 y2 2 gx 2 fy c  0
be a circle. Every point on the ellipse

x2 y2
1
a2 b2
is of the form (a cos Q , b sin Q ). Substituting x  a cos P and y  b sin P in the equation of the circle,
we have

a 2 cos2 Q b2 sin 2 Q 2 ga cos Q 2 fb sin Q c  0 (5.15)


Let
Q
t  tan
2
so that

1 t2 2t
cos Q  2
and sin Q 
1 t 1 t2
5.2 Inverted Ellipse 305

Substituting the values of cos Q and sin Q in Eq. (5.15) and simplifying, we obtain the following
fourth-degree equation in t:

(a 2 c 2ag ) t 4 4bft 3 2(c 2b2 a 2 ) t 2 4bft (c a 2 2ag )  0


This equation being a fourth-degree equation has four roots, say,
A B G D
t1  tan , t2  tan , t3  tan and t4  tan
2 2 2 2
Hence, we have
A 4bf
¤ tan  t1 t2 t3 t4  2 (5.16)
2 a c 2ag

A B 2(c 2b2 a 2 )
¤ tan tan  ¤ t1t2  (5.17)
2 2 c a 2 2ag
A B G 4bf
¤ tan tan tan  ¤ t1t2 t3  (5.18)
2 2 2 c a 2 2ag

A B G D c a 2 2ag
tan tan tan tan  t1t2 t3 t4  (5.19)
2 2 2 2 c a 2 2ag
Now from Eqs. (5.16) and (5.18), we have
A A B G
¤ tan 2 ¤ tan 2 tan 2 tan 2
However,

¥A B G D´
tan ¦ µ 
¤ tan(A / 2) ¤ tan(A / 2)tan(B / 2)tan(G / 2) 0
§ 2 2 2 2¶ 1 ¨
ª ¤ tan(A / 2)tan(B / 2)·¹ tan(A / 2)tan(B / 2)tan(G / 2)tan(D / 2)
Therefore
A B G D
 nP
2 2 2 2

or A + B + G + D  2nP

Subjective Problems (Sections 5.1 and 5.2)


1. Find the equation of the ellipse of eccentricity 2/3 whose 2. Find the eccentricity of the ellipse
focus is (3, −1) and whose directrix is the straight line
2x − y 16  0. ( x 1)2 ( y 2)2
4
9 1
Solution: If P(x, y) is a point on the ellipse, we have Also find the foci.
¥ 2 ´ 2 x y 16
( x 3)2 ( y 1)2  ¦ µ Solution: The given ellipse can be written as
§ 3¶ 5
2 2 2 2
( x 1)2 ( y 2)2
 45[ x y 6 x 2 y 10]  4[4 x y 4 xy 64 x 32 y 1
36 4
256]
We have a2  36 and b2  4. Now
 29 x 2 16 xy 41y2 526 x 218 y 574  0
b2  a2(1 − e2)
306 Chapter 5 Ellipse and Hyperbola

 4  36(1 e)2 AP  20 − a and PB  a  x  a cos Q and y  (20 a)


since P  (x, y). This implies
2 2
 e x2 y2
3 1
a2 (20 a)2
Hence
If PB  8, then a2  144 and b2  64 so that
¥ 6(2 2) ´
S(ae, 0)  ¦ x 1  , y 2  0µ  (1 4 2 , 2) 64
§ 3 ¶  1 e2
144
4 5
and S a( ae, 0)  (1 4 2 , 0)  e2  1 
9 9
3. An ellipse has its centre at O(0, 0) and axes along 5
 e
OX and OY, respectively. If the ellipse passes through 3
P(2, 7) and Q(4, 3), find the coordinates of foci.
y
Solution: Let the equation of the ellipse be
B
x2 y2
1 a
a2 b2
It passes through (2, 7) and (4, 3). Therefore q
P
x
4 49 20 − a
2
2
1 (5.20) y
a b q
16 9 O A x
and 2
2
1 (5.21)
a b
Solving Eqs. (5.20) and (5.21) for a2 and b2, we have FIGURE 5.7

187 187
a2  and b2  5. Let A and B be two fixed points and 2c be the distance
10 3
between them. Then, show that the locus of the point P
Hence, the ellipse equation is 10x2 3y2  187 and the such that PA PB  2a, where a  c is an ellipse having
eccentricity e is given by A and B as its foci.
187 187 HHHE
 (1 e 2 ) Solution: Take the midpoint of AB as origin O at OA
10 3 as positive x-axis. Since AB  2c, we have A  (c, 0) and
3 7 B  (−c, 0). Now, P(x, y) is a point such that
 e2  1 
10 10 PA PB  2a
Therefore, foci are given by
š ( x c)2 y 2 ( x c)2 y 2  2 a
¥ 187 s 7 ´
¦ 0, p µ š ( x c)2 y 2  4 a 2 ( x c)2 y 2 4 a ( x c)2 y 2
§ 10 ¶
š (cx a 2 )2  a 2 [( x c)2 y2 ]
4. A bar of length 20 units moves with its ends on two š x 2 (a 2 c 2 ) y 2 a 2  a 4 a 2 c 2  a 2 (a 2 c 2 )
fixed lines which are at right angles. Prove that a
marked point on the bar describes an ellipse. If the x2 y2
š 2
1 (a  c  a 2 c 2  0)
point is at a distance of 8 units from one end, find the a a c2
2

eccentricity.
This equation represents ellipse with major axis of length
Solution: See Fig. 5.7. Take OX and OY as coordinate 2a and minor axis of length 2 a 2 c 2 . Eccentricity
axes and O as (0, 0). Let AB be the bar such that AB 
20. Let point P be marked on AB such that AP  a and a 2 c 2  a 2 (1 e 2 )
PB  20 − a. Suppose OAB  Q (see Fig. 5.7). We have c
 e
a
5.2 Inverted Ellipse 307

6. For the ellipse x cos Q ¥ a ´


 ¦ 2 µ y sin Q  1 (5.22)
x 2
y 2 a §b ¶
1
a2 b2 Suppose M(x1, y1) is the midpoint of AB so that the
let the vertices be A(a, 0) and Aa(−a, 0). Suppose tan- equation of chord AB is
gent at any point of the ellipse meets the tangents at xx1 yy1 x12 y12
the vertices at L and La, respectively. Then, show that  (5.23)
the product AL·AaLa is constant and it is equal to b2. a2 b2 a2 b2
From Eqs. (5.22) and (5.23)
Proof: See Fig. 5.8. Tangent at P is
x y x1 /a 2 y1 /b2 ( x12 /a 2 ) ( y12 /b2 )
cos Q sin Q  1  
a b (cos Q )/a (a /b2 )sin Q 1

Tangents at A and Aa are x  a and x  −a, respec- x1 y1 x 2 y2


   12 12
tively, so that a cos Q a sin Q a b
b(1 cos Q ) b(1 cos Q ) 2 2
AL s Aa L a  s b x12 y12 ¥ x12 y12 ´
sin Q sin Q  2  ¦ µ
a (cos2 Q sin 2 Q ) § a 2 b2 ¶
2
L x12 y12 ¥ x 2 y2 ´
  ¦ 12 12 µ
a2 §a b ¶
P(q )
L Therefore, the locus of M(x1, y1) is

¥ x 2 y2 ´ x 2 y2
¦ 2 2µ
§a b ¶ a2
A A
C
8. Prove that the locus of the midpoint of the portion of
the tangent to

x2 y2
1
a2 b2
FIGURE 5.8
lying between the axes is
7. Point P is on the circle x2 y2  a2. AB is the chord of
contact of P with respect to the ellipse a2 b2
 22
x2 y2
x2 y2
1
a2 b2 Solution: The tangent at (a cos Q , b sin Q ) is
Then, show that the locus of the midpoint of AB is x cos Q y sin Q
the curve 1
a b
2
¥ x 2 y2 ´ x 2 y2 Therefore, A(a sec P, 0) and B(0, b cosec P) are the points
¦ 2 µ  of intersection of the tangent with the axes. If (x1, y1) is
§a b2 ¶ a2
the midpoint of AB, then
Solution: Let P(a cos P, a sin P) be a point on x2 y2  a sec Q a
a2. Then, the chord of contact AB of P with respect to x1   cos Q 
2 2 x1
the ellipse
b cosec Q b
x2 y2 y1   sin Q 
1 2 2 y1
a2 b2
Hence
by Quick Look 7, is
a2 b2
x(a cos Q ) y(a sin Q )  4(cos2 Q sin 2 Q )  4
1 x12 y12
a2 b2
308 Chapter 5 Ellipse and Hyperbola

Thus, the locus of (x1, y1) is the curve 2


Substituting x1  ae and y1  b /a, the equation of the
a2 b2 normal at L(ae, b2 /c) is given by
 4  (2)2
x2 y2 a 2 ( x ae) b2 [ y (b2 /a)]

ae b2 /a
9. The normal to the ellipse
a
 ( x ae)  ay b2
x2 y2 e
1
a2 b2 Suppose this passes through (0, −b). So we have
at point P meets the major axis at G. Find the locus
a 2  ab b2
of the midpoint of PG .
 a 2 b2  ab
Solution: Equation of the normal at (a cos Q , b sin Q ) is
ax sec P − by cosec P  a2 − b2 [see Theorem 5.5, part (2)].  a2 e2  a2 1 e2 (& b  a 1 e 2 )
Substituting y  0, we have  e4  1 e2
¥ (a 2 b2 )cos Q ´  e4 e2 1  0
G¦ , 0µ
§ a ¶

Let M(x1, y1) be the midpoint of PG. Therefore B


2 2 L
(a b )cos Q
2 x1  a cos Q and 2 y1  b sin Q
a
cos Q 2
 2 x1  (a a 2 e 2 ) and 2 y1  b sin Q
a
A A
 2 x1  a cos Q (1 e 2 ) and 2 y1  b sin Q C S

Therefore,
4 x12 4 y12
2 2 2
2
 cos2 Q sin 2 Q  1
a (1 e ) b
B
Hence, the locus of the midpoint of PG is the ellipse
FIGURE 5.9
2 2
4x 4y
1 11. Show that the locus of the midpoints of chords of the
a 2 (1 e 2 )2 b2
ellipse
10. If the normal at one end of the latus rectum of the x2 y2
ellipse 1
a2 b2
x2 y2 which pass through a fixed point (h, k) is
1
a 2 b2
passes through one end of the minor axis, then show x2 y2 hx ky
2
2
 2

that the eccentricity e satisfies the relation a b a b2
e4 e2 − 1  0
Solution: If (x1, y1) is the midpoint of a chord, then its
Solution: See Fig. 5.9. Let equation is
S1  S11 (see Theorem 5.12)
¥ b2 ´
L ¦ ae, µ
§ a¶ xx1 yy1 x12 y12
 
a2 b2 a2 b2
be one end of the latus rectum through the focus S(ae, 0).
The normal at (x1, y1) [by Theorem 5.5, part (1)] is This passes through (h, k). This implies that
a 2 ( x x1 ) b2 ( y y1 ) hx1 ky1 x12 y12
 
x1 y1 a2 b2 a2 b2
5.2 Inverted Ellipse 309

Hence, the locus of (x1, y1) is 13. If P is a point on the ellipse

x2 y2 hx ky x2 y2
 1
a2 b2 a2 b2 a2 b2

12. The tangent to the ellipse


with foci S and Sa, then show that
PSS a PS aS 1 e
x2 y2 tan s tan 
1 2 2 1 e
a2 b2
at point P (a cos A , b sin A ) meets the auxiliary circle Solution: See Fig. 5.11. We know that SP SaP  2a and
x2 y2  a2 in two points Q and R such that the chord SaS  2ae. Therefore, the semi-perimeter (s) of $PSSa is
QR subtends right angle at the centre of the ellipse.
2a 2ae
Show that the eccentricity of the ellipse is  a(1 e)
(1 sin 2 A ) 1 2 . 2
Now (see Theorem 4.6, Chapter 4, Vol. 2, p. 195), we
Solution: See Fig. 5.10. The tangent at P (a cos A , b sin A ) have
is
x cos A y sin A PSS a ( s SP )(S SS a)
1 tan 
a b 2 s(S S aP )
Therefore, the combined equation of the pair of lines PS aS ( s S aP )(S SS a)
CQ and CR (where C is the centre) is and tan 
2 s(S SP )
2
¥ x cos A y sin A ´
x 2 y2 a 2 ¦ µ 0 Therefore
§ a b ¶
PSS a PS aS s SS a
(see Theorem 2.33, Chapter 2). Now RCQ  90o. This tan s tan 
2 2 s
implies that in the above equation a ae 2ae
 (& s  a ae)
Coefficient of x2 Coefficient of y2  0 a ae
¥ a 2 sin 2 A ´ 1 e
 (1 cos2 A ) ¦ 1 
µ 0 1 e
§ b2 ¶
a 2 sin 2 A
 sin 2 A 1 0 P
a 2 (1 e 2 )
 (1 e 2 1)sin 2 A 1 e 2  0
 e 2 (1 sin 2 A ) 1  0
1
 e2  or e  (1 sin 2 A ) 1 2 S S
1 sin 2 A

Q
P
R
FIGURE 5.11

14. If a chord joining the points @ and A on the ellipse


90
A A x2 y2
C 1
a2 b2
passes through a focus, then show that
A B e 1
tan tan 
2 2 e 1

Solution: The equation of the chord joining @ and A


FIGURE 5.10
[by Theorem 5.4, part (2)] is given by
310 Chapter 5 Ellipse and Hyperbola

x cos[(A B ) 2] y sin[(A B ) 2] ¥ A B´ 16. A tangent to an ellipse


 cos ¦
a b § 2 µ¶ x2 y2
1
Suppose, this chord passes through (ae, 0). Therefore a2 b2
¥ A B´ ¥ A B´ is cut by the tangents at the vertices T and T a. Prove
e cos ¦ µ  cos ¦
§ 2 ¶ § 2 µ¶ that the circle with T and Ta as extremities of a diam-
eter passes through the foci of the ellipse.
cos[(A B )/ 2] 1
 
cos[(A B )/ 2] e Solution: See Fig. 5.13. The tangent at P(a cos P, b sin
cos[(A B )/ 2] cos[(A B )/ 2] 1 e P) is
 
cos[(A B )/ 2] cos[(A B )/ 2] 1 e x cos Q y sin Q
1
2 cos(A / 2)cos(B / 2) 1 e a b
 
2 sin(A / 2)sin(B / 2) 1 e The tangents at A(a, 0) and Aa(−a, 0) are x  a and x 
−a, respectively. Therefore
A B e 1
 tan tan 
2 2 e 1 ¥ b(1 cos Q ) ´
T  ¦ a,
§ sin Q µ¶
15. Find the locus of the foot of the perpendicular drawn
from centre of the ellipse onto any tangent to the el- ¥ b(1 cos Q ) ´
and T a  ¦ a,
lipse § sin Q µ¶
x2 y2 Hence, the circle with T and Ta as the ends of a diameter
1
a2 b2 is

Solution: The line ¥ b(1 cos Q ) ´ ¥ b (1 cos Q ) ´


( x a)( x a) ¦ y µ ¦ y µ¶
§ sin Q ¶ § sin Q
y  mx a 2 m2 b2 (5.24)
by b2 (1 cos2 Q)
is a tangent to the ellipse. The line passing through the  x 2 y2 (1 cos Q 1 cos Q) a 2 0
centre (0, 0) and perpendicular to the line which is given
sin Q sin 2 Q
in Eq. (5.24) is 2by
 x 2 y2 b2 a 2  0
x my  0 (5.25) sin Q
From Eqs. (5.24) and (5.25), we have which passes through S(ae, 0) and Sa(−ae, 0).
¥ x´ a2 x 2 T
y x¦ µ 2
b2
§ y¶ y

 x 2 y 2  a 2 x 2 b2 y 2 P
T
 ( x 2 y 2 )2  a 2 x 2 b 2 y 2
which is the required locus.

S S
M
P

FIGURE 5.13
C
17. The tangent and the normal at point P on the el-
lipse

x2 y2
1
a2 a2
FIGURE 5.12
5.2 Inverted Ellipse 311

meet the minor axis in T and G, respectively. Prove 18. If the chord joining the points P(a cos @, b sin @) and
thatTG subtends right angle at each of the foci. Q(a cos A, b sin A) cuts the major axis of the ellipse

Solution: See Fig. 5.14. Let coordinates of P be (x1, y1) x2 y2


1
so that a2 b2
x12 y12 at a distance d units from the centre, then show that
2
2
1 (5.26)
a b A B d a
tan tan 
The tangent at P(x1, y1) is 2 2 d a
xx1 yy1
2
1 Solution: The equation of the chord PQ is
a b2
x cos(A B )/2 y sin(A B )/2 ¥ A B´
so that  cos ¦
a b § 2 µ¶
¥ b2 ´
T  ¦ 0, µ The chord PQ meets the major axis at
§ y1 ¶
¥ a cos(A B )/2 ´
and the normal at P is ¦§ cos(A B )/2 , 0µ¶
a 2 ( x x1 ) b2 ( y y1 )
 By hypothesis, we have
x1 y1
a cos[(A B )/2]
so that d
cos[(A B )/2]
¥ a2 y ´ cos[(A B )/2] d
G  ¦ 0, y1 2 1 µ and S  (ae, 0)  
§ b ¶ cos[(A B )/2] a
Now, cos[(A B )/2] cos[(A B )/2] d a
 
cos[(A B )/2] cos[(A B )/2] d a
(b2 /y1 ) 0 y1 (a 2 y1 /b2 )
Slope of ST s Slope of SG  s 2 cos(A / 2)cos(B / 2) d a
0 ae 0 ae  
2 sin(A / 2)sin(B / 2) d a
b2 (b2 a 2 ) y1
 s A B d a
aey1 aeb2  tan tan 
2 2 d a
b2 a 2 a2 e2
   1
a2 e2 a2 e2
QUICK LOOK 8
Hence, GST  90o.
If the chord passes through the focus (ae, 0), then
y
d  ae so that (see Problem 14)
T
A B e 1
tan tan 
2 2 e 1

P
19. If the intercept of the line x cos @ y sin @  p made
by the ellipse
x2 y2
1
C S x a2 b2
subtends right angle at the centre, then show that the
G
line touches the circle
a 2 b2
x 2 y2 
a 2 b2

Solution: See Fig. 5.15. Let QR be the intercept of the


line x cos @ y sin @  p made by the ellipse
FIGURE 5.14
312 Chapter 5 Ellipse and Hyperbola

x2 y2 so that the slope (m2) of the normal is


1
a2 b2 a
m2  tan A
The combined equation of the pair of lines CQ and CR b
Therefore
(where C is the centre) is
2 m1 m2
x2 y2¥ x cos A y sin A ´ tan Q 
2 1 ¦ µ¶  0 1 m1 m2
2
a b § p
[(b /a) (a /b)]tan A

Now PCQ  90o. This implies that 1 tan 2 A
Coefficient of x2 Coefficient of y2  0 ¥ a 2 b2 ´
¦ µ sin A cos A
¥ 1 cos2 A ´ ¥ 1 sin 2 A ´ § ab ¶
¦ 2 µ ¦ p µ 0
§a p2 ¶ § b2 ¶ ¥ a 2 b2 ´
¦ µ sin 2A
1 1 1 § 2ab ¶
 
a2 b2 p2 a 2 b2
b
a 2 b2 2ab
 p2 
a 2 b2 Hence, the maximum value of tan P is
However, p is the distance of (0, 0) from the line x cos @ a 2 b2
y sin @  p. Hence, the line touches the circle
2ab
a 2 b2 and it is attained when
x 2 y2 
a 2 b2
P P
2A  A =
2 4
Q

21. Two points P and Q are corresponding points on an


ellipse and its auxiliary circle. Prove that the normal
to the ellipse at P and normal to the auxiliary circle
at Q intersect on the circle
90°
C x2 y2  (a b)2

Solution: See Fig. 5.16. Let


R
P  (a cos @, b sin @)
and Q  (a cos @, a sin @)
FIGURE 5.15 The normal at P to the ellipse is
ax sec @ − by cosec @  a2 − b2 (5.27)
20. Let P be any point on the ellipse whose centre is
C(0, 0). If P is the angle between the chord CP and and the normal at Q to the circle x2 y2  a2 is
the normal at P, then find tan P and show that as P y  (tan @)x (5.28)
moves on the curve, the maximum value of tan P is
From Eq. (5.28), we have
a 2 b2
y x
2ab sin A  and cos A 
2 2
x y x y2
2

Solution: Let P  (a cos @, b sin @) so that the slope


(m1) of CP is Substituting these values of sin @ and cos @ in Eq. (5.27),
we have
b sin A b
m1   tan A
a cos A a x 2 y2 x 2 y2
(ax) (by)  a 2 b2
x y
The normal at P is
ax sec @ − by cosec @  a2 − b2  x 2 y2 (a b)  a 2 b2 or x 2 y2  (a b)2
5.2 Inverted Ellipse 313

Q P

C T T
A A
C N

FIGURE 5.17

23. Let P be a point on the ellipse

FIGURE 5.16 x2 y2
2
1
a b2
22. The tangent and the normal at point P of the ellipse
and A and Aa be the vertices of the ellipse. If the
x2 y2 lines AaP and AP meet a directrix at Q and R, respec-
1
a2 b2 tively, then prove that QR subtends right angle at
the corresponding focus.
meet the major axis in T and T a such that TT a  a.
Prove that the eccentric angle P of the point P is Solution: See Fig. 5.18. We have P  (a cos P, b sin P),
given by Aa  (−a, 0) and A  (a, 0). The equation of AaP is
e2cos2 P cos P – 1  0 b sin Q
y ( x a)
where e is the eccentricity. a (1 cos Q )
¥ Q´ ¥ Q´
Solution: See Fig. 5.17. Let P  (a cos P, b sin P). The  ay  ¦ b tan µ x ¦ (ab)tan µ
tangent at P is § 2¶ § 2¶

x cos Q y sin Q Substituting x  a /e in the above equation, we get


1 (5.29)
a b ¥ a b (1 e) Q´
Q¦ , tan µ
Normal at P is §e e 2¶

ax sec Q by cosec Q  a 2 b2 (5.30) The equation of AP is

Substituting y  0 in Eqs. (5.29) and (5.30), we get b sin Q


y ( x a)
a (cos Q 1)
¥ (a 2 b 2 ) ´
T  (a sec Q , 0) and T a  ¦ cos Q , 0µ In this equation, substituting x  a /e, we get
§ a ¶
Now, ¨a b Q·
R  © , (e 1)cot ¸
TT a  a ªe e 2¹

a (a 2 b 2 ) Now, the slope of SQ is given by


 cos Q  a
cos Q a (b/e)(1 e)tan(Q / 2) 0 b (1 e)tan(Q / 2)
m1  
1 (a /e) ae a(1 e 2 )
a e 2 cos Q  a (& a 2 b2  a 2 e 2 )
cos Q b tan(Q / 2)
 m1  (5.31)
 1 e 2 cos2 Q  cos Q a (1 e)
and the slope of SR is given by
 1 e 2 cos2 Q  cos Q (& 1 r e 2 cos2 Q )
 e 2 cos2 Q cos Q 1  0 (b /e)(1 e)cot (Q / 2) b (e 1)cot (Q / 2)
m2  
(a/e) ae a(1 e 2 )
314 Chapter 5 Ellipse and Hyperbola

b cot (Q / 2)  x cot Q a cosec Q


 m2  (5.32)
a (1 e) This line touches the auxiliary circle x 2 y2  a 2 . This
implies
From Eqs. (5.31) and (5.32), we get
b tan Q / 2 b cot Q / 2 a 2 cosec 2 Q  a 2 (1 cot 2 Q )  a 2 cosec 2Q
m1 m2  s
a (1 e) a (1 e) which is true. Hence, TQ touches the auxiliary circle.
b2 1
 2
s
a (1 e 2 )
b2 Q
 2
 1
b
P
Hence, RSQ  90o.

C T x

P Q

A
A′ x
C S a
x= e

R
FIGURE 5.19

25. If the tangent at point P on the ellipse

x2 y2
1
FIGURE 5.18 a2 b2

24. P is a point on the ellipse meets a directrix at Q, then prove that PQ subtends
right angle at the corresponding focus.
x2 y2
1
a2 b2 Solution: See Fig. 5.20. The tangent at P(a cos P, b sin
P) is
and Q is the corresponding point on the auxiliary
circle x cos Q y sin Q
1
2 2 2 a b
x y a
Substituting x  a /e, we have
If the tangent at P meets the major axis at T, then
show that QT touches the auxiliary circle. ¥ a b (e cos Q ) ´
Q¦ ,
§ e e sin Q µ¶
Solution: See Fig. 5.19. P  (a cos P, b sin P) and Q  (a
cos P, a sin P). The tangent at P to the ellipse is Now
x cos Q y sin Q ¥ b sin Q ´
1 (Slope of SP) s (Slope of SQ)  ¦
a b § a (cos Q e) µ¶
Therefore, T  (a sec P, 0) and the equation of the line [b(e cos Q )/e sin Q ]
TQ is s
(a/e) ae
a sin Q 0 b sin Q b(e cos Q )e
y ( x a sec Q )  s
a cos Q a sec Q a(cos Q e) ae(1 e 2 )sin Q
sin Q cos Q ( x cos Q a)
 b2 b2
(cos2 Q 1) cos Q    1
a 2 (1 e 2 ) b2
( x cos Q a)
 Therefore, QSP  90o.
sin Q
5.2 Inverted Ellipse 315

27. Show that the locus of the midpoints of the chords of


P
the ellipse
x2 y2
Q 2
1
a b2
which subtend right angle at the centre of the
x
C S ellipse is
a 2
x= e a 2 b2 ¥ x 2 y 2 ´ x 2 y2
¦ µ 
a 2 b2 § a 2 b2 ¶ a 4 b4

Solution: See Fig. 5.21. Let PQ be a chord whose mid-


FIGURE 5.20 point is (x1, y1) which is subtending right angle at the
centre. Hence, the equation of the chord PQ is
26. Prove that the locus of the midpoints of the chords of
xx1 yy1 x12 y12
contact of points on the director circle of the ellipse 2
2
 2

a b a b2
2 2
x y
2
1 Now, the combined equation of the pair of lines CP and
a b2 CQ is
with respect to the ellipse is 2
x2 ¨ ( xx1 /a 2 ) ( yy1 /b2 ) ·
y2
2 1 © 2 2 2 2 ¸
0
¥ x 2 y2 ´ a 2 b2
2 2 2 2 ª ( x1 /a ) ( y1 /b ) ¹
¦ 2 2 µ (a b )  x y
§a b ¶
Now PCQ  90o which implies that in the above equa-
Solution: M(x1, y1) be the midpoint of the chord tion
of contact of a point P(h, k) on the director circle Coefficient of x2 Coefficient of y2  0
x 2 y2  a 2 b2 . Therefore
1 ( x12 /a 4 ) 1 ( y12 /b4 )
h 2 k 2  a 2 b2 (5.33)  0
a2 [( x12 /a 2 ) ( y12 /b2 )]2 b2 [( x12 /a 2 ) ( y12 /b2 )]2
xx1 yy1 x12 y12
and  (5.34) 1 1 ( x12 /a 4 ) ( y12 /b4 )
a 2
b 2
a 2
b 2  
a2 b2 [( x12 /a 2 ) ( y12 /b2 ))]2
Also the chord of contact of P(h, k) with respect to the
ellipse is Hence, the locus of (x1, y1) is
2
hx ky x2 y2 ¥ 1 1 ´ ¥ x 2 y2 ´
1 (5.35) ¦ 2 2 µ ¦ 2 2 µ
a2 b2 a4 b §a
4
b ¶§a b ¶
From Eqs. (5.34) and (5.35), we get
Q B
(h /a 2 ) (k /b2 ) 1
  P
( x1 / a 2 ) ( y1 /b2 ) ( x12 /a 2 ) ( y12 /b2 ) M

h k 1
   2 2 90
x1 y1 ( x1 /a ) ( y12 /b2 )
A A
x1 y1 C
h , k
( x12 /a 2 ) ( y12 /b2 ) ( x12 /a 2 ) ( y12 /b2 )
Substituting the values of h and k in Eq. (5.33), we have

x12 y12 B
 a 2 b2
[( x1 /a ) ( y12 /b2 )]2
2 2
[( x1 /a ) ( y12 /b2 )]2
2 2
FIGURE 5.21

Therefore, the locus of (x1, y1) is


28. Two tangents are drawn from a point on the ellipse
¥x 2 2 ´2
y x2 y2
x 2 y 2  ¦ 2 2 µ (a 2 b 2 ) 1
§a b ¶ a 2
b2
316 Chapter 5 Ellipse and Hyperbola

to the circle x 2 y2  c 2 . Prove that the chord of 29. Let d be the distance of a tangent at a point P of the
contact touches the ellipse ellipse from the centre. If S and Sa are the foci, then
show that
a 2 x 2 b2 y 2  c 4
¥ b2 ´
Solution: See Fig. 5.22. Let P (h, k) be a point on the ( PS PS a)2  4a 2 ¦ 1 2 µ
§ d ¶
ellipse
Solution: Let P be (a cos P, b sin P) so that the tangent
x2 y2
1 at P is
a2 b2
x cos Q y sin Q
so that 1
a b
Therefore
h2 k2
1 (5.36)
a2 b2 0 0 1
d
Chord of contact of P(h, k) with respect to the circle cos Q /a 2 sin 2 Q /b2
2
x 2 y2  c 2 is
cos2 Q sin 2 Q 1
hx ky  c 2 (5.37)  2
2

a b d2
From Eq. (5.37), we have
b2 cos2 Q b2
2  sin 2 Q 
¥ h ´ c a 2
d2
y¦ µ
§ k ¶ k
b2 cos2 Q b2
 cos2 Q  1
which touches the ellipse a2 d2
a 2 x 2 b2 y 2  c 4 ¥ b2 ´ b2
 cos2 Q ¦ 1 2 µ  1 2
x2 y2 § a ¶ d
or 1
(c 2 /a)2 (c 2 /a)2 (a 2 b 2 ) b2
 cos2 Q  1
This implies a2 d2
2 b2
¥ c2 ´ c4 ¥ h ´
2
c4  e 2 cos2 Q  1 (& a 2 b2  a 2 e 2 ) (5.38)
¦kµ  – ¦ µ d2
§ ¶ a2 § k ¶ b2
Also SP  a ea cos Q and S aP  a ea cos Q . This implies
c4 c 4 h2 c4
  (SP S aP )2  (2 ea cos Q )2
k2 a2 k 2 b2
 4a 2 e 2 cos2 Q
1 h2 1
  ¥ b2 ´
k2 a2 k 2 b2  4a 2 ¦ 1 2 µ
h2 k2 § d ¶
 1
a2 b2
30. P is a point on the ellipse
which is true by hypothesis.
x2 y2
P 1
a2 b2
Q whose foci are S and Sa. Show that the incentre of
$PSSa lies on a concentric ellipse whose eccentricity is

2e
R
1 e

where e is the eccentricity of the given ellipse.

Solution: Let P  (a cos P, b sin P ), S  (ae, 0) and Sa 


(−ae, 0). We know that
FIGURE 5.22
SSa  2ae, SP  a −ae cos P, SaP  a ae cos P
5.2 Inverted Ellipse 317

so that the perimeter of ΔPSSa  2a (1 e). Let (h, k) be [( x b2 )/c]2 y2


the incentre of $PSSa. Therefore and 4 2
1
b /c a b /c 2
2 2
2ae (a cos Q ) aea (1 e cos Q ) aea (1 e cos Q )
h The centres of the ellipses are (a 2 /c, 0) and ( b2 /c, 0).
2a (1 e) Also for both curves, the length of the minor axis is same.
2a 2 e cos Q (1 e) Hence, the common tangent touches the ellipses at

2a (1 e) ¥ a 2 ab ´ ¥ b2 ab ´
P¦ , µ and Q ¦ , µ
 h  (ae)cos Q (5.39) § c c¶ § c c¶
Similarly Hence, we have
be sin Q
k (5.40) ¨ (ab /c) 0 · ¨ (ab /c) 0 ·
1 e Slope of OP s Slope of OQ  © 2 ¸© 2 ¸
ª (a /c) 0 ¹ ª (b /c) ¹
From Eqs. (5.39) and (5.40), we get
¥ b ´ ¥ a ´
h2 k2 ¦ µ ¦ µ  1
 cos2 Q sin 2 Q  1 § a¶ § b ¶
(ae)2 [be /(1 e)]2
Thus, QOP  90o.
Therefore, (h, k) lies on the ellipse
P Q
x2 y2
1
p2 q2
where C1 O C2
2 2
b e
p2  a 2 e 2 and q2 
(1 e)2
FIGURE 5.23
If the eccentricity of this is ea, then
32. Point P is on
b2 e 2 2 2 2
 a e (1 e a ) x2 y2
(1 e)2 1
2 a2 b2
b
  1 ea2
2
a (1 e) 2 and point Q is on the auxiliary circle

1 e2 x 2 y2  a 2
  1 ea2
(1 e)2 corresponding to P. The line through P parallel to
1 e OQ (O is the centre) meets the major axis at A and
  1 ea2 minor axis at Aa. Prove that PA  b and PAa  a.
1 e
1 e 2e
 ea2  1  Solution: See Fig. 5.24. P  (a cos P, b sin P) and Q 
1 e 1 e (a cos P, a sin P). The line through point P and parallel
2e to OQ is
 ea 
1 e sin Q
y b sin Q  ( x a cos Q )
cos Q
31. Prove that a common tangent of the ellipses
Therefore, A  ( b cos Q a cos Q , 0) and Aa  (0, b sin Q
x2 2xy2 x2 y2
2x a sinP). Now,
 0 and 0
a 2 b2 c a 2 b2 c
( PA)2  (a cos Q b cos Q a cos Q )2 b2 sin 2 Q
subtends right angle at the origin.
 b2 (cos2 Q sin 2 Q )  b2
Solution: See Fig. 5.23. The given ellipses pass through  PA  b
the origin and touch at origin. Their equations can be
rewritten as Again

[( x a 2 )/c]2 y2 ( PAa)2  a 2 cos2 Q (b sin Q b sin Q a sin Q )2


1
a 4 /c 2 a 2 b2 /c 2  a 2 (cos2 Q sin 2 Q )
318 Chapter 5 Ellipse and Hyperbola

 a2 Therefore, the common tangent is 2 x 3 y  4 7 . The


 PAa  a common tangent meets the axes in the points
A (2 7 , 0) and B (0, 4 7 / 3 ). Hence, the length AB is

Q 112 196 14
28  
3 3 3
P
34. Find the point on the ellipse x 2 2 y2  6 which is
nearest to the line x y 7  0. (IIT-JEE 2003)

Solution: The given ellipse is


O A
x 2 y2
1
A 6 3
and every point on it is of the form P ( 6 cos Q , 3 sin Q ).
The nearest point on the ellipse is the point at which the
normal to the ellipse is perpendicular to the line
x y 7  0 or equivalently, the tangent is parallel to the
line (see Fig. 5.25). The tangent at P ( 6 cos Q , 3 sin Q ) is
FIGURE 5.24
x cos Q y sin Q
1
33. Find the equation of the common tangent in the first 6 3
quadrant to the circle x 2 y2  16 and the ellipse
whose slope is given by
x 2 y2
1 3 cot Q
25 4 cot Q   1
6 2
Also find the length of the intercept of the tangent
between the coordinate axes. (IIT-JEE 2005)  cot Q  2
 cos Q  2 sin Q
Solution: We know that
 cos2 Q  2 sin 2 Q
x cos Q y sin Q
1  3 sin 2 Q  1
5 2
1 2
is a tangent to the given ellipse for all real values of P.  sin Q  p and cos Q  p
3 3
This line also touches the given circle which implies that
Therefore, the nearest point in the first quadrant is
10
4 ¨ 2 ¥ 1 ´·
4 cos2 Q 25 sin 2 Q P© 6 , 3¦ ¸  (2, 1)
ª 3 § 3 µ¶ ¹
 4 (4 cos2 Q 25 sin 2 Q )  25
y
 84 cos2 Q 75  0 B
75 9
 cos2 Q  and sin 2Q 
x
+

84 84
y
=
7

Since the tangent is in the first quadrant, its slope must M


be negative. Hence, the equation of the tangent is
P
¥ 2 ´ 2
y  ¦ cot Q µ x
§ 5 ¶ sin Q x
C A
2 75 2
 y x 84
5 9 3
2 5 2 s 2 21
 y – x
5 3 3
 2 x 3y  4 7
FIGURE 5.25
5.2 Inverted Ellipse 319

35. Prove that in an ellipse, the perpendicular from a fo- meets the coordinate axes at A and B, then find the
cus upon any tangent and the line joining the centre equation of the focus of the midpoint of AB.
of the ellipse to the point of contact meet on the cor- (IIT-JEE 1999)
responding directrix. (IIT-JEE 2002)
Solution: The tangent to the ellipse is of the form
Solution: See Fig. 5.26. Let P be (a cos P, b sin P) so y  mx 25m2 4 . This line also touches the circle. So
that the tangent at P is
25m2 4  r 2 (1 m2 )
x cos Q y sin Q
1 r2 4
a b  m2  (5.41)
25 r 2
Let S  (ae, 0) be one focus. The equation of the line
through S and perpendicular to the tangent is Therefore, the common tangent is y  mx 25m2 4
where
a sin Q
y ( x ae)
b cos Q r2 4
m
This line meets the directrix x  a/e corresponding to the 25 r 2
focus s(ae, 0) which is given by Therefore,

¥ a a 2 sin Q 2 ´ ¥a b ´ ¥ 25m2 4 ´
¦ e , be cos Q (1 e )µ  ¦§ e , e tan Q µ¶ [& b2  a 2 (1 e 2 )] A ¦ , 0µ
§ ¶ ¦§ m µ¶
Now, the equation of the line CP where C  (0, 0) is
and B  (0, 25m2 4 )
¥ b sin Q ´ Let M (x1, y1) be the midpoint of AB so that
y¦ x
§ a cos Q µ¶
25m2 4
which clearly passes through 2 x1  and 2 y1  25m2 4
m
¥a b ´  4 m2 x12  4 y12
¦§ , tan Q µ¶
e e y12
 m2 
x12
M
Therefore,
y12
2 y1  25m2 4 and m2 
x12
P
a
x= e ¥ y2 ´
 4 y12  25 ¦ 12 µ 4
§ x1 ¶
 4 x12 25 y12  4 x12 y12
C S
Hence, the locus of (x1, y1) is
4 x 2 25 y2  4 x 2 y2

37. A tangent to the ellipse x 2 4 y2  4 meets the el-


lipse x 2 2 y2  6 at P and Q. Prove that the tangents
FIGURE 5.26 at P and Q of the ellipse intersect at right angles.

36. Consider the family of circles x 2 y2  r 2 , where Solution: See Fig. 5.27. The two given ellipses can be
2  r  5. If in the first quadrant, the common tan- written as
gent to a circle of this family and the ellipse
x 2 y2
1 (5.42)
x 2 y2 4 1
1
25 4 x 2 y2
and 1 (5.43)
6 3
320 Chapter 5 Ellipse and Hyperbola

Let us consider that the tangent to ellipse which is given ¥A´ ¥ B´ ¥A´ ¥G ´ ¥ B´ ¥G ´
in Eq. (5.42) meets the ellipse which is given in Eq. (5.43)  tan ¦ µ tan ¦ µ tan ¦ µ tan ¦ µ tan ¦ µ tan ¦ µ
§ 2¶ § 2¶ § 2¶ § 2¶ § 2¶ § 2¶
at P and Q. Let T be the point of intersection of the tan-
gents to Eq. (5.43) at P and Q. Let T be (h, k). Since PQ ¥A´ ¨ 1 ·
is the chord of contact of T with respect to the ellipse tan ¦ µ ©
§ 2 ¶ ª tan(A / 2)tan(B / 2)tan(G / 2) ¸¹
which is given in Eq. (5.43), the equation of PQ is
hx ky ¥ B´ ¨ 1 ·
1 tan ¦ µ ©
6 3 § 2 ¶ ª tan(A / 2)tan(B / 2)tan(G / 2) ¸¹
 hx 2ky  6
¥G ´ ¨ 1 ·
tan ¦ µ ©
¥ h ´ 3 § 2 ¶ ª tan(A / 2)tan(B / 2)tan(G / 2) ¸¹
 y¦ µ x (5.44)
§ 2k ¶ k
¨ ¥A´ ¥B ´ ¥A´ ¥ B´·
The line in Eq. (5.44) touches the ellipse in Eq. (5.42).  ©tan ¦ µ tan ¦ µ cot ¦ µ cot ¦ µ ¸
ª § 2 ¶ § 2 ¶ § 2 ¶ § 2¶¹
So
2 ¨ ¥ B´ ¥G ´ ¥ B´ ¥ G ´·
9 ¥ h ´ ©tan ¦ µ tan ¦ µ cot ¦ µ cot ¦ µ ¸
 4¦ µ 1 ª § 2 ¶ § 2 ¶ § 2 ¶ § 2¶ ¹
k 2 § 2k ¶

 h2 k 2  9  6 3 ¨ ¥G ´ ¥A´ ¥G ´ ¥A´·
©tan ¦ µ tan ¦ µ cot ¦ µ cot ¦ µ ¸
ª § 2 ¶ § 2 ¶ § 2 ¶ § 2¶¹
Thus, T(h, k) lies on the director circle of Eq. (5.43) so that
the tangents at P and Q to Eq. (5.43) are at right angles. [cos2 (A / 2)cos(B / 2)] [sin 2 (A / 2)sin(B / 2)]

Q T [cos(A / 2)cos(B / 2) sin(B / 2)(A / 2)sin(B / 2)
[cos2 (B / 2)cos2 (G / 2)] [sin 2 (B / 2)sin 2 (G / 2)]

[cos(B / 2)cos(G / 2)]–[sin (B / 2)sin(G / 2)]

x 2 + 4y 2 = 4 P [cos2 (G / 2)cos2 (A / 2)] [sin 2 (G / 2)sin 2 (A / 2)]



[cos(G / 2)cos(A / 2)].[sin(G / 2)sin(A / 2)]
x 2 + 2y 2 = 6
Therefore
4 cos(A B )/ 2 cos(A B )/ 2
FIGURE 5.27 ¤ sin A sin B
0

38. If @, A and F are the eccentric angles of three points 2 (cos A cos B )
on the ellipse
 ¤ sin A sin B
0

 sin G (cos A cos B ) sin A (cos B cos G )


x2 y2
1 sin B (cos G cos A )  0
a2 b2
 sin(B G ) sin(G A ) sin(A B )  0
at which the normals are concurrent, then show that
sin(A B ) sin(B G ) sin(G A )  0 39. If the normals at (xr, yr) (r  1, 2, 3, 4) to the ellipse

Solution: Suppose the normals at @, A and F are con- x2 y2


12

current at (h, k) and let C be the foot of the fourth nor- a b2
mal from (h, k) (see Theorem 5.16). Then we have are concurrent, then show that
A B ¥ 1 1
¤ tan tan  0 1 1´
2 2 ( x1 x2 x3 x4 ) ¦ µ  4
§ x1 x2 x3 x4 ¶
A B G D
and tan tan tan tan  1
2 2 2 2 Solution: Normal at (@, A ) is
Eliminating tan D / 2 from the above two equations gives a 2 ( x A ) b2 ( y B )

¥A ´ ¥B ´ A B
0  ¤ tan ¦ µ¶ tan ¦§ µ¶
§2 2
This passes through (h, k). This implies
5.2 Inverted Ellipse 321

a 2 ( h A ) b 2 (k B ) perpendicular to PQ and RT, respectively, where C (0,


 0) is the centre of the ellipse. Now,
A B
a 2 m2 b2
 a 2 B ( h A )  b 2 A (k B ) (CM )2 
1 m2
 [(h A )a 2 b2A ] B  b2 kA
a 2 b2 m2
 [a 2 h A (a 2 b2 )]2 B 2  b4 k 2A 2 (5.45) and (CN )2 
1 m2
Also (@, A ) lies on the ellipse, which means
Now,
A 2 B2
1 (5.46) l1  PQ  2 PM
a 2 b2
 l12  4 ( PM )2
Eliminating A from Eqs. (5.45) and (5.46) gives
 4(CP 2 CM 2 )
¥ a A 2 2´
[a 4 h2 A 2 (a 2 b2 )2 2a 2 hA (a 2 b2 )] ¦ µb
2
¨ a 2 m2 b2 ·
§ a2 ¶  4 ©a2 ¸
ª 1 m2 ¹
 b4 k 2 A 2
4
A 4 (a 2 b 2 ) 2 b 2  (a 2 b 2 ) (5.49)
 2 A 3 h (a 2 b 2 ) b 2 A 2 [ a 2 b 2 h 2 b 4 k 2 1 m2
2
a
Also
b2 (a 2 b2 )]
A [2a 2 b2 h (a 2 b2 )] a 4 b2 h2  0 ¨ a 2 b2 m2 ·
l22  4 ©a 2 ¸ (5.50)
ª 1 m2 ¹
This being a fourth-degree equation in @, we get four
values of @, say, x1, x2, x3 and x4. Therefore, 4 m2
 (a 2 b 2 )
2 2 2 2 2 1 m2
2b h(a b ) a 2a h
x1 x2 x3 x4  2 2 2 2

b (a b ) (a 2 b 2 ) Therefore, from Eqs. (5.49) and (5.50), we get
2 2 2 2 2 2
1 1 1 1 2a b h (a b ) 2(a b ) ¨ 1 m2 ·
  l12 l22  4 (a 2 b2 ) ©
x1 x2 x3 x4 a 4 b2 h 2 a2 h 2 2¸
ª1 m 1 m ¹
Hence  4 (a 2 b 2 )
¥ 1 1 1 1 ´ 2a 2 h 2(a 2 b2 )  4a 2 e 2
( x1 x2 x3 x4 ) ¦ µ  2 s 4
§ x1 x2 x3 x4 ¶ a b2 a2 h  (2ae)2
SSa)2
 (S
40. Two tangents to an ellipse
90°
x2 y2
1
a2 b2 P R
are drawn which intersect at right angles. Let l1 and l1 x l N
M 2
l2 be the intercepts on these tangents made by the
auxiliary circle x 2 y2  a 2 . Show that l12 l22 is equal
Q
to (SSa)2, where S and Sa are the foci. T

Solution: See Fig. 5.28. Let


C
y  mx a 2 m2 b2 (5.47)

x a2
and y b2
m m2
 x my  a 2 b2 m2 (5.48)
Let the intercept of Eqs. (5.47) and (5.48) on the auxil-
lary circle be l1  PQ and l2  RT. Draw CM and CN FIGURE 5.28
322 Chapter 5 Ellipse and Hyperbola

5.3 Hyperbola
Hyperbola is a conic whose eccentricity is greater than unity. In this section, we discuss the principal properties that are
common to all hyperbolas. Some of these properties are same as that of the ellipse and can be proved in a similar way.
However, the additional feature that the hyperbola possess is a pair of asymptotes (tangents with points of contact at
infinity) which gives the curve its own properties. Let us start the discussion with the formal definition of hyperbola.

DEFINITION 5.12 Let l be a straight line (called the directrix), S be a point which is not located on l (called the
focus) and e  1 be a fixed real number (called the eccentricity). Then, the locus of the point
P such that SP is equal to (e PM), where PM is the perpendicular distance of P from the
directrix, is called a hyperbola.

Example 5.9

Find the equation of the hyperbola whose directrix is 2


x y 1
x y 1  0, focus is (− 1,− 1) and eccentricity is 2. š ( x 1)2 ( y 1)2  4
2
Solution: P(x,y) is a point on hyperbola š x y 2 x 2 y 2  2 x 4 xy 2 y2 4 x 4 y 2
2 2 2

š SP  2 ( PM ) š x 2 4 xy y2 2 x 6 y  0
š (SP )2  4 ( PM )2 Thus, the equation of the hyperbola is

x 2 4 xy y2 2 x 6 y  0

T H E O R E M 5.18 The standard equation of the hyperbola is

x2 y2
1
2 2 2
a2 b2
where b  a (e 1).
PROOF See Fig. 5.29. Draw SZ perpendicular to the directrix l. Divide SZ internally and externally at
points A and Aa in the ratio e:1. Let C be the midpoint of AAa and AaC  CA  a. Since
SA : AZ  e : 1 and SAa : Aa Z  e : 1, we have
SA  e ( AZ ) and SAa  e ( Aa Z )
 SA Aa S  e ( AZ Aa Z )
 (CS CA) ( Aa C CS)  e( AAa)
 2 (CS)  e (2a)
 CS  ae (5.51)
Also
SAa AS  e ( Aa Z AZ )
 Aa A  e ( Aa C CZ AC CZ )
 2a  2e (CZ )
a
 CZ  (5.52)
e
HHHE
Now, choose CS as positive x-axis and C as origin. Hence from Eqs. (5.51) and (5.52), we have
S  (ae, 0) and the equation of CZ (i.e., directrix) is x  a /e. Now, P(x,y) is a point on the curve.
This implies
SP  ePM
2
¥ a´
 ( x ae)2 y2  e 2 ¦ x µ
§ e¶
5.3 Hyperbola 323

 ( x ae)2 y2  (ex a)2


 (e 2 1) x 2 y2  a 2 (e 2 1)
x2 y2
 1
a2 b2
where b2 = a2 (e2 1).
y
l

M P
M

x
S A Z C Z A S N

FIGURE 5.29

QUICK LOOK 9

For any point P(x, y) on the hyperbola, we have


¥ 2a ´
S aP SP  e PM a PM  e (MM a)  e (ZZ a)  e ¦ µ  2a  Aa A
§ e¶

x2 y2
5.3.1 Features of the Curve 1
a2 b2
1. The curve is symmetric about both axes.
2. To each value of x, such that x  a, there are two values for y which are equal in magnitude but opposite in sign.
To each value of y, there are two values for x.
3. The abscissae a of any point should not be between –a and a.
4. The curve cuts the x-axis at (−a, 0) and (a, 0).
5. The curve cannot intersect y-axis.
6. As y increases infinitely, x also increases infinitely.
7. Due to the symmetry of the curve, there is another focus Sa(−ae, 0); the corresponding directrix is x  a /e.

DEFINITIONS 5.13
1. Aa(−a, 0) and A(a, 0) are called the vertices of the curve and AaA is called the transeverse axis whose length is 2a.
2. If B  (0, −b) and Ba  (0, b), then BBa is called the conjugate axis whose length is 2b.
3. The double ordinate passing through the focus is called latus rectum.
4. x 2 y2  a 2 is called the auxiliary circle which touches the curve at vertices A and Aa.

x2 y2 2b2
T H E O R E M 5.19 The length of the latus rectum of the hyperbola 2
2
 1 is .
a b a
324 Chapter 5 Ellipse and Hyperbola

PROOF Let SL be the semi-latus rectum where S  (ae, 0) and L  (ae, y). Now L is a point on the curve, so

a2 e2 y2
1
a2 b2
 y2  b2 (e 2 1)
b4
 y2 
a2
b2
 SL 
a

DEFINITION 5.14 In a hyperbola, if the lengths of transeverse and conjugate axes are equal, then it is called
rectangular hyperbola.

QUICK LOOK 10

In a rectangular hyperbola, b2  a 2  a 2  a 2 (e 2 1) is 2 and the standard form of a rectangular hyperbola


 e  2 . The eccentricity of any rectangular hyperbola is x 2 y2  a 2 .

Example 5.10

For the hyperbola 4 x 2 9 y2  36, find the foci, eccentric- Foci are given by
ity, the directrices and the lengths of the axis.
¨ ¥ 13 ´ ·
(p ae, 0)  © p 3 ¦ µ , 0 ¸  ( 13 , 0)
Solution: The given hyperbola is ©ª § 3 ¶ ¸¹

x 2 y2 Directrices are given by


1
9 4
a ¥ 3 ´ 9
2 2 x  p  x  p 3¦ µ p
so that a  9, b  4. We have e § 13 ¶ 13
b2  a 2 (e 2 1) Transeverse axis length is 2a  6 and conjugate axis
2
 4  9 (e 1) length  2b  4.
4 13
 e 2  1 or e 
9 3

Example 5.11

Find the eccentricity and foci of the hyperbola Therefore, the centre is ( 4, 1). Now
9 x 2 16 y2 72 x 32 y 16  0.
b2  a 2 (e 2 1)
Solution: The given equation can be written as  9  16 (e 2 1)
5
9 ( x 2 8 x) 16 ( y2 2 y)  16 e
4
 9 ( x 4)2 16 ( y 1)2  16 144 16  144
( x 4)2 ( y 1)2 The foci are given by
 1
16 9 ( x 4  p ae, y 1  0)  ( 4 p 5, 1)  (1, 1) and ( 9, 1)
5.3 Hyperbola 325

DEFINITION 5.15 Parametric Equations For all real Q x (2 n 1) P / 2, the point (a secP, b tanP) lies on the
hyperbola
x2 y2
2
1
a b2
The equations
x  a sec Q and y = b tanQ
are called the parametric equations of the hyperbola.

DEFINITION 5.16 Position of a Point For any hyperbola, the foci region is called internal region and any
point which is not (a) inside the curve and (b) on the curve is called external point.
Notations used:
x2 y2
Sy 2
1
a b2
xx yy
S1 y 21 21 1
a b
xx yy
S2 y 22 22 1
a b
xx yy
S12 y S21  1 2 2 1 2 2 1
a b
2 2
x y
S11  12 12 1
a b
The following can be proved similar to the case of ellipse by replacing b2 with –b2 and hence only statements are given
without proofs.

x2 y2
T H E O R E M 5.20 Let S y 1  0 . Then
a2 b2
1. A point in the plane of the hyperbola is external or internal to the curve according as S11  0 or
S11  0, respectively. Note that in the case of circle, parabola and ellipse, S11  0 is the condition
for an external point and S11  0 is the condition for an internal point. However, in the case of
hyperbola, it is the reverse.
2. The equation of the chord joining two points (x1, y1) and (x2, y2) on the curve is S1 S2  S12 .
xx1 yy1
3. The equation of the tangent at (x1, y1) is S1 y 1  0.
a2 b2
a 2 ( x x1 ) b2 ( y y1 )
4. The equation of the normal at (x1, y1) is  .
x1 y1
x sec Q y tan Q
5. The equation of tangent at (a sec P, b tan P) is  1.
a b
6. The equation of the normal at (a sec P, b tan P) is
x tan Q y sec Q ¥ a 2 b2 ´
b

a
¦ µ sec Q tan Q
§ ab ¶
 ax cos Q by cot Q  a 2 b2
7. The condition for the line y  mx c to touch the hyperbola is that c 2  a 2 m2 b2 or the line
y  mx p am2 b2 touches the hyperbola.
8. The equation of the director circle is x 2 y2  a 2 b2 . This will exist only when a  b.
326 Chapter 5 Ellipse and Hyperbola

9. The equation of a tangent is x cos A y sin A  p, where p  a 2 cos2 A b2 sin 2 A .


10. All points on a tangent except the point of contact are external to the hyperbola.
11. Equation of the chord in terms of its midpoint (x1, y1) is S1  S11.
12. From any external point, two tangents can be drawn to a hyperbola and the equation of the
pair of tangents from the point (x1, y1) is S12  SS11.

DEFINITION 5.17 Asymptotes The two lines given by

x2 y2 b
2
2
 0 or y  p x
a b a

x2 y2
are called the asymptotes of the hyperbola 2
 1.
a b2
Algebraically, the two lines y  p (b /a) x satisfy the condition of a tangent, but without the existence of points of contact.
Due to this reason, asymptotes are also stated as lines through the centres which touch the curve at infinity. In general,
a line l is called an asymptote to a curve if the line l goes nearer and nearer to the curve, but never meets the curve.

x2 y2
T H E O R E M 5.21 For the hyperbola  1, the following properties hold good with regard to asymptotes:
a2 b2
1. The chord of contact of any point on an asymptote is parallel to the same asymptote.
2. The product of the perpendiculars drawn from any point on the curve onto its asymptotes is
constant.
3. The foot of the perpendicular from a focus onto an asymptote is one of the points of intersec-
tion of the corresponding directrix with the auxiliary circle.
4. If a straight line cuts the curve at P and Q and the asymptotes at R and S, then PR  QS.
5. The angle between the two asymptotes is 2 sec−1(e).
PROOF 1. P (x1, y1) is a point on the asymptote y  (b /a) x so that

b
y1  x1 (5.53)
a
The chord of contact of P(x1, y1) with respect to the hyperbola is
xx1 yy1
1
a2 b2
From Eq. (5.53), the slope of the chord of contact of

b2 x1 b2 ¥ a ´ b
P  ¦ µ
a 2 y1 a2 § b ¶ a

which is the slope of the asymptote. Hence, the chord of contact of P is parallel to the asymp-
tote on which point P lies.
2. Let P(x1, y1) be a point on the curve so that

x12 y12
1 (5.54)
a2 b2
The product of the perpendiculars drawn from P(x1, y1) onto the asymptotes is

( x1 /a) ( y1 /b) ( x1 /a) ( y1 /b) x12 y12 a 2 b2


s 
(1/ a 2 ) (1/ b2 ) (1/ a 2 ) (1/ b2 ) a2 b2 a 2 b2
5.3 Hyperbola 327

Therefore, from Eq. (5.54), we have


( x1 /a) ( y1 /a) ( x1 /a) ( y1 /a) a 2 b2
s 
(1/ a 2 ) (1/ b2 ) (1/ a 2 ) (1/ b2 ) a 2 b2

3. Let S  (ae, 0) and y  (b /a) x be an asymptote. The line passing through S and perpendicular
to the asymptote y  (b /a) x is
a
y 0 ( x ae)
b
Solving the equations
¥ b´ a
y¦ µ x and y ( x ae)
§ a¶ b
we have
2
¥ a b´ a e
x¦ µ 
§ b a¶ b
a2 e ab a3e a
 x s 2  
b a b2 a 2 e 2 e
b ¥ a´ b
and y ¦ µ 
a § e¶ e

That is, the foot of the perpendicular from S (ae, 0) onto the asymptote y  (b /a) x is (a /e, b/e)
which lies on the auxiliary circle x 2 y2  a 2 .
4. See Fig. 5.30. Let M(x1, y1) be the midpoint of PQ. We show that M(x1, y1) is also the midpoint
of RS. The equation of the chord PQ is
xx1 yy1 x12 y12
 (5.55)
a2 b2 a2 b2
Substituting y  (b /a) x and y  (b /a) x in Eq. (5.55), we have
¨ ¥ x y ´ ¥ x y ´·
R  ©a ¦ 1 1 µ , b ¦ 1 1 µ ¸
ª § a b ¶ § a b ¶¹
¨ ¥x y ´ ¥ x y ´·
and S  ©a ¦ 1 1 µ , b ¦ 1 1 µ ¸
ª § a b ¶ § a b ¶¹
Hence, the midpoint of RS is ( x1 , y1 ).
y

R
P

C M (x1, y1) x

FIGURE 5.30

5. The asymptotes are y  (b /a) x and y  (b /a) x. They are equally inclined to the x-axis. If 2P is
the angle between them, then
328 Chapter 5 Ellipse and Hyperbola

b
tan Q 
a
so that

a 2 b2 a2 e2
sec 2 Q    e2
a2 a2
and 2 Q  2 Sec 1 (e)

QUICK LOOK 11

The combined equation of the pair of asymptotes of that the combined equation of the asymptotes of a
x 2
y 2
x 2
y 2 hyperbola and the equation of a hyperbola differ by a
the hyperbola 2
2
 1 is 2
 0 which shows constant.
a b a b2

Example 5.12 (Method to Find Asymptotes)

Find the asymptotes of the hyperbola 3 x 2 5 xy 2 y2 Hence, the combined equation of the asymptotes is
17 x y 14  0. 3 x 2 5 xy 2 y2 17 x y 10  0
Solution: Since the hyperbola equation and the pair of Since 3 x 2 5 xy 2 y2  (3 x y)( x 2 y), let us consider
asymptotes equation differ by a constant, let the com- that
bined equation of the pair of asymptotes be
3 x 2 5 xy 2 y2 17 x y 10  (3 x y n1 )( x 2 y n2 )
2 2
3 x 5 xy 2 y 17 x y k  0 (5.56)
Equating the coefficients of x and y on both sides, we
Since Eq. (5.56) represents a pair of lines, from Theorem have
2.31, Chapter 2, we have
n1 3n2  17
2 2 2
¥ 1 ´ ¥ 17 ´ ¥ 5 ´ ¥ 1 ´ ¥ 17 ´ ¥ 5 ´ and 2 n1 n2  1
3( 2)k 2 ¦ µ ¦ µ ¦ µ 3¦ µ ( 2)¦ µ k ¦ µ  0
§ 2¶ § 2 ¶ § 2 ¶ § 2¶ § 2¶ § 2¶
Solving for n1 and n2 , we have n1  2 and n2  5. Hence,
49k 490
 0 the asymptotes are 3 x y 2  0 and x 2 y 5  0.
4 4
 k  10

Example 5.13

The asymptotes of a hyperbola are the lines 2 x 3 y 8  0 Since this passes through the point (5, 3), we have
and 3 x 2 y 1  0, and the curve passes through the (10 9 8)(15 6 1) k  0
point (5, 3). Find the equation of the hyperbola.
 k  110
Solution: Let the equation of the hyperbola be Hence, the equation of the hyperbola is
(2 x 3 y 8)(3 x 2 y 1) k  0 6 x 2 5 xy 6 y2 22 x 19 y 118  0

DEFINITION 5.18 Conjugate Hyperbola A hyperbola whose transverse and conjugate axes are conjugate
and transverses of another hyperbola is called conjugate hyperbola. For example, the conju-
x2 y2 y2 x2 x2 y2
gate hyperbola of  1 is  1 or  1
a2 b2 b2 a2 a2 b2
5.3 Hyperbola 329

QUICK LOOK 12

x2 y2 In fact, one can observe that


For a hyperbola  1 and its conjugate
a2 b2 ¥ x 2 y2 ´
x2 y2 Hyperbola Conjugate hyperbola  2 ¦ 2 2 µ
 1, the asymptotes are the same lines §a b ¶
a2 b2
= 2 × Asymptote
x2 y2
0
a2 b2

Example 5.14
2 2
Find the conjugate hyperbola of 3 x 5 xy 2 y 17 x y (3 x 2 5 xy 2 y2 17 x 14) Conjugate
y 14  0.  2 s Assymptotes

Solution: The asymptote of the given hyperbola (see  2 s (3 x 2 5 xy 2 y2 17 x y 10)


Example 5.12) is This gives
2 2
3 x 5 xy 2 y 17 x y 10  0 3 x 2 5 xy 2 y2 17 x y 6  0
Therefore, according to Quick Look 12, which is the equation of the conjugate hyperbola.

1 1
T H E O R E M 5.22 If e1 and e2 are eccentricities of a hyperbola and its conjugate, then  1.
e12 e22
PROOF Suppose e1 is the eccentricity of
x2 y2
1
a2 b2
and e2 is the eccentricity of
x2 y2
 1
a2 b2
Therefore

b2  a 2 (e12 1) and a 2  b2 (e22 1)


Now
(e12 1)(e22 1)  1
 e12 e22 e12 e22 0
1 1
 1
e12 e22

Now, we recall the definition of rectangular hyperbola from Definition 5.14. The hyperbola in which transverse and
conjugate axes having equal lengths is called rectangular hyperbola. Hence, the general standard equation of the rect-
angular hyperbola is x 2 y2  a 2 . To study some of the properties of the curve x 2 y2  a 2 , we transform this equa-
tion to simplest form xy  c 2 , by rotation of the axes through a certain angle.

T H E O R E M 5.23 The equation of the rectangular hyperbola with its asymptotes as the coordinate axes is
xy  c 2 , where c is constant, is called the simplest form of a rectangular hyperbola.
PROOF Let x 2 y2  a 2 be rectangular hyperbola. Now, rotate the axes about the origin through P /4 in
the clockwise direction so that the asymptotes become coordinate axes [because the angle
between the asymptotes of x 2 y2  a 2 is 2 Sec 1 ( 2 ) which equal to P /2 ]. Then
330 Chapter 5 Ellipse and Hyperbola

¥ P ´ ¥ P ´
X  x Cos 1 ¦ y sin ¦
§ 4 µ¶ § 4 µ¶
¥ P ´ ¥ P ´
and Y  x sin ¦ y cos ¦
§ 4 µ¶ § 4 µ¶
That is,
x y y x
X and Y 
2 2
Hence, the equation of the hyperbola is
2 2
¥ x y´ ¥ y x ´ 2
¦§ µ ¦ µ a
2 ¶ § 2 ¶
 4 xy  2a 2
a2
 xy 
2
We write c  a / 2 so that xy  c 2. Also xy  c 2  either both x and y are positive or both x and
y are negative.

QUICK LOOK 13

If S y xy c 2  0, then S1 y xy1 x1 y 2c 2  0 and S11  x1 y1 c 2 .

T HEOREM 5.24 The point (x, y) is on the curve xy  c 2 if and only if x  ct and y  c /t for some real number t.
(P ARAMETRIC These equations are called parametric equation of xy  c 2 .
E QUATIONS OF
xy  c 2 )

PROOF The point  ct, c /t clearly lies on the curve xy  c 2 . Conversely, if (x, y) is any point on the curve
xy  c 2 , then take either t  x /c or c / y. So x  ct and y  c /t.

Note: The parametric equations of x 2 y2  a 2 are x  a sec Q , y  a tan Q , where Q R.

T H E O R E M 5.25 Let S y xy c 2  0. Then

1. xy1 x1 y  2c 2 is the equation of the tangent at ( x1 , y1 ) . In particular, if x1  ct and y1  c /t,


then the equation of the tangent in the parametric form is
cx
cty  2c 2
t
x
or ty  2c
t

2. The equation of the normal at (ct, c /t ) is t 3 x ty c ct 4  0.


5.3 Hyperbola 331

PROOF We have S y 2 xy 2c 2  0. Hence


1. The tangent at ( x1 , y1 ) is
xy1 x1 y 2c 2  0
In particular, if x1  ct and y1  c /t, the equation of the tangent at (ct, c /t ) is
x
ty  2c
t
2. The slope of the tangent at (ct, c /t ) is 1/t 2 . Hence, the equation of the normal at (ct, c /t ) is
given by
c
y  t 2 ( x ct )
t
 ty c  t 3 ( x ct )
 t 3 x ty c ct 4  0

T H E O R E M 5.26 If the normal at (ct, c /t ) for xy  c 2 meets the curve again at (ct a, c /t a), then t 3 t a  1.
PROOF The normal at (ct, c /t ) is
t 3 x ty c t 4 c  0
This passes through (cta c/ta). This implies
¥ c´
t 3 (ct a) t ¦ µ c ct 4  0
§ ta¶
t 4
 t 3t a t 1 0
ta
 t 3 t a 2 t t at 4 t a  0
 t 3 t a (t a t ) (t a t )  0
 (t 3 t a 1)(t a t )  0
 t 3t a  1 (& t x t a)

Example 5.15

The tangent at a point P of a rectangular hyperbola 1 1 ¥ 2c ´


xy  c 2 meets the asymptotes at L and M. Prove that OL s OM  (2ct ) ¦ µ  2c 2
2 2 § t ¶
PL  PM  PO, where O is the centre of the hyperbola.
Also show that the area of $LOM is constant. which is constant.
2
Solution: See Fig. 5.31. For xy  c , the asymptotes are y
the coordinate axes. Tangent at (ct, c /t ) is
M
x
ty  2c
t P
 x t 2 y  2ct
Therefore, L  (2ct, 0) and M  (0, 2c /t ). The midpoint O L x
of LM is
¥ 2ct 0 0 ( 2c /t ) ´ ¥ c ´
¦§ , µ¶  ¦§ ct, µ¶  P (say)
2 2 t FIGURE 5.31
Since MOL  90o and P is the midpoint of LM, we have
OP  PL  PM . Also the area of $LOM is given by
332 Chapter 5 Ellipse and Hyperbola

Try it out Try Example 5.15 for the hyperbola x 2 y2  a 2 whose asymptotes are x p y  0.

T H E O R E M 5.27 The portion of the tangent to a hyperbola intercepted between its asymptotes is bisected at the
point of contact.
PROOF Let P(a sec Q , b tan Q ) be a point on
x2 y2
2
1
a b2
The tangent at P is
x sec Q y tan Q
1 (5.57)
a b
We know that the equations of the asymptotes are
¥ b´
y p¦ µ x
§ a¶

Substituting y  (b /a) x in Eq. (5.57), we get

x sec Q b tan Q x a
1x  a(sec Q tan Q )
a ab sec Q tan Q
and y  b(sec Q tan Q )
Let Q  [a(sec Q tan Q ), b(sec Q tan Q )] . Similarly, by substituting y  (b /a) x in Eq. (5.57), we
have
R  [a(sec Q tan Q ), b(tan Q sec Q )]
Now, it is easy to see that the midpoint of QR is P(a sec Q , b tan Q ).

Subjective Problems (Section 5.3)


1. Find the eccentricity and the coordinates of the hy- Solution: The given equation is
perbola 2 x 2 3 y2  5.
9( x 2 2 x) 16( y2 4 y) 89  0
Solution: The equation of hyperbola can be written as
 9( x 1)2 16( y 2)2 89 9 64  0
x2 y2 ( x 1)2 ( y 2)2
1   1
5/ 2 5/ 3 16 9
so that which is the conjugate hyperbola of
5 5 2 ( x 1)2 ( y 2)2
 (e 1) 1
3 2 16 9
5 The centre is ( x 1  0, y 2  0)  (1, 2). The eccentric-
 Eccentricity, e = ity e is given by
3
and the foci is 16  9(e 2 1)  e  5 / 3
¥ so that
5 5 ´ ¥ 5 ´
(p ae, 0)  ¦ p s , 0µ  ¦ p , 0µ
§ 2 3 ¶ § 6 ¶ ¥ 5´
be  3 ¦ µ  5
§ 3¶
2. Find the centre, vertices, eccentricity and foci of the The foci is ( x 1  0, y 2  p be) which is given by
hyperbola 9 x 2 18 x 16 y2 64 y 89  0.
(1, y  2 p 5)  (1, 3) and (1, 7)
5.3 Hyperbola 333

3. If every tangent to the hyperbola The tangent at P is


x2 y2 x sec A y tan A
2
2
1 1 (5.60)
a b a b
meets the director circle at two points P and Q, and C Since QQa is parallel to the tangent P [Eq. (5.60)], we have
is the centre of the curve, then prove that the product b sec A b ¨ cos[(G B )/ 2] ·
of the slopes of CP and CQ is equal to b2 /a 2 .  © ¸
a tan A a ª cos[(G B )/ 2] ¹
Solution: The equation of the director cir- ¥ B G ´ ¥ B G ´
 sin ¦  sin A cos ¦ (5.61)
cle is x 2 y2  a 2 b2 . It is known that the line § 2 µ¶ § 2 µ¶
y  mx a 2 m2 b2 is a tangent to the hyperbola so that
The perpendicular PN from P onto the line provided in
the combined equation of the lines CP and CQ is
Eq. (5.59) is
2
¥ y mx ´ ba sec A ab tan A
x 2 y 2 (a 2 b 2 ) ¦ µ 0
¦§ a 2 m2 b2 µ¶ b2 a 2

This being a second-degree equation in x and y repre- The perpendicular QM is


senting pair of lines, the product of their slopes (see The- ba sec B ab tan B
orem 2.32 clubbed with Theorem 2.27, Note 2, Chapter 2)
is given by b2 a 2

Coefficient of x 2 1 [(a 2 b2 )m2 /(a 2 m2 b2 )] The perpendicular Qa M a is


2
 2 2 2 2 2
Coefficient of y 1 [(aa b )/(a m b )] ba sec G ab tan G
2 2 2
b b m b2 a 2
 2 2 2
a m a
Therefore
b2 (m2 1)
 a 2 b2
a 2 (m2 1) QM – Qa M a  (sec B tan B )(sec G tan G )
2
a 2 b2
b
 a 2 b2
(1 sin B )(1 sin G )
a2  2 2
a b cos B cos G

Try it out What happens if m  p 1 ? 


a 2 b2 [1 cos[(P /2) B ]] – [1 cos[(P /2) G ]]
2 2
a b sin[(P / 2) B )] – sin[(P / 2) G ]
2 2
a b ¥ P B´ ¥P G ´
4. The chord QQa of a hyperbola is parallel to the  tan ¦ µ tan ¦ µ (5.62)
a 2 b2 § 4 2¶ § 4 2¶
tangent at P. PN, QM and QaMa are the perpen-
diculars to either of the asymptotes. Show that Now, from Eq. (5.61), we have
QM – Q a M a  ( PN )2 .
sin[(B G )/ 2]
 sin A
Solution: Let the hyperbola be cos[(B G )/ 2]
cos [(P 2 ) [( B G )/ 2]] ¥P ´
x2 y2   cos ¦ A µ
1 (5.58) cos[(B G ) / 2] §2 ¶
a2 b2
See Fig. 5.32. Let P  (a sec A , b tan A ), Q  (a sec B, b tan B ) Using componendo and dividendo, we have
and Qa  (a sec G , b tan G ). Let us consider the asymptote cos[(B G )/ 2)] cos [(P / 2) [(B G )/ 2]]
¥ b´ cos[(B G )/ 2)] cos [(P / 2) [(B G )/ 2]]
y¦ µ x (5.59)
§ a¶ 1 cos[(P / 2) A ]

The slope m of QQa is 1 cos[(P / 2 ) A ]
2 sin[(P / 4 ) (G / 2)] sin[(P / 4 ) (B / 2)]
b(tan B tan G ) b ¨ cos[(G B )/ 2] · 
 –© ¸ 2 cos[(P / 4 ) (G / 2)]cos[(P / 4 ) (B / 2)]
a(sec B sec G ) a ª sin[(B G )/ 2] ¹
334 Chapter 5 Ellipse and Hyperbola

2 sin 2 [(P / 4 ) (A / 2)] 


ab
2(tan 2 Q sec 2 Q )
 2
2 cos [(P / 4) ( A / 2)] 2
 ab
¥P G ´ ¥P B´ ¥P A´
 tan ¦ µ tan ¦ µ  tan 2 ¦ µ (5.63) y
§ 4 2¶ § 4 2¶ § 4 2¶
Therefore, from Eqs. (5.59) and (5.60), we have
a 2 b2 ¥P A´ Q
QM – Qa M a  tan 2 ¦ µ (5.64)
2
a b 2 § 4 2¶ P
Now, C x
R
ba sec A ab tan A ab
PN   sec A tan A
2 2
b a a b2
2

ab 1 sec A FIGURE 5.33



2
a b 2 cos A
6. If m x 0, then prove that the point of intersection of
ab 1 cos[(P / 2 ) A ] the lines

2
a b 2 sin[(P / 2 ) A ]
x y x y 1
 m and 
ab ¥P A´ a b a b m
 tan ¦ µ (5.65)
2
a b 2 § 4 2¶
lies on a hyperbola.
From Eqs. (5.64) and (5.65), we have QM – Qa M a  PN 2 .
Solution: Solving the given two equations, we have
y
a¥ 1´
M x ¦m µ
2§ m¶
N Q
M b¥ 1´
P and y ¦m µ
2 § m¶
C x
Q So
2 2
¥ 2x ´ ¥ 2y´
¦§ µ¶ ¦§ µ¶  4
a b

FIGURE 5.32 x2 y2
 1
a2 b2
5. Prove that the area of a triangle formed by the two as-
Hence, the lines intersect the hyperbola
ymptotes and any tangent to the hyperbola is constant.
x2 y2
Solution: See Fig. 5.33. The tangent at P(a sec Q , b tan Q ) 1
a2 b2
is
2
x y 7. Find the common tangent of the parabola y  8 x
sec Q tan Q  1 (5.66)
a b x 2 y2
and the hyperbola  1.
Asymptotes are 1 3

¥ b´ Solution: We have
y p¦ µ x
§ a¶ y2  8 x  4(2) x
Substitute y  (b /a) x in Eq. (5.66) and let Q = [a(secP + so that a  2. Hence, the line
tan Q ), b(sec Q tan Q )]. Also, substitute y  ( b /a) x and 2
let R  [a(sec Q tan Q ), b(tan Q sec Q )]. Therefore, the y  mx
m
area of $CQR is
is a tangent to the parabola y2  8 x for all real values of
1 m x 0. This also touches the given hyperbola. So
ab(tan 2 Q sec 2 Q ) ab(sec 2 Q tan 2 Q )
2
5.3 Hyperbola 335

2 The tangent at P meets the asymptotes at Q and R. If


¥ 2´ 2
¦§ µ¶  1(m ) 3 C is the centre of the curve, then show that
m
4 2
CP – CQ  a 2 b2
 m 3m 4  0
Solution: Let P  (a sec Q , b tan Q ) so that the tangent
 (m2 4)(m2 1)  0 at P is
m p 2 x y
sec Q tan Q  1 (5.67)
Hence, the common tangents are y  2 x 1 and a b
y  2 x 1. and the two asymptotes are

8. P and Q are two points on the hyperbola x y


0 (5.68)
a b
x2 y2
2
1 x y
a b2 and 0 (5.69)
a b
and C(0, 0) is its centre. If PQ subtends right angle at
Solving Eqs. (5.67) and (5.68), we have
the centre C and a  b, then show that
Q  [a(sec Q tan Q ), b(sec Q tan Q )]
1 1 1 1

CP 2 CQ2 a2 b2 and solving Eqs. (5.67) and (5.69), we have
R  [a(sec Q tan Q ), b(tan Q sec Q )]
Solution: Let CP  r1 and Q be the angle made by CP
with the positive direction of the x-axis so that Therefore
P  (r1 cos Q , r1 sin Q ) (CP )2 (CQ)2  [a 2 (sec Q tan Q )2 b2 (sec Q tan Q )2 ]
Since PCQ  90o, we have s [a 2 (sec Q tan Q )2 b2 (tan Q sec Q )2 ]
Q  (r2 cos(P / 2 Q ), r2 sin(P / 2 Q ))  (a 2 b2 )2 (sec Q tan Q )2 (sec Q tan Q )2
Q  ( r2 sin Q , r2 cos Q )  (a 2 b2 )2 (sec 2 Q tan 2 Q )2
where r2  CQ. Now, P and Q lie on the curve. This  (a 2 b 2 ) 2
implies
Hence, CP – CQ  a 2 b2 .
¥ cos Q sin Q ´ 2 2
r12 ¦ 2 2 µ  1 10. A tangent to the hyperbola
§ a b ¶
x2 y2
¥ sin 2 Q cos2 Q ´ 2
1
r22 ¦ 2 a b2
and µ 1
§ a b2 ¶
cuts the ellipse
Therefore
x2 y2
1 1 2 2
(cos Q sin Q ) (sin Q cos Q ) 2 2 1
 a2 b2
r12 r22 a2 b2
at P and Q. Show that the locus of the midpoint of
1 1 PQ is the curve

a2 b2 2
¥ x 2 y2 ´ x 2 y2
Hence
¦ 2 2µ  2 2
§a b ¶ a b
1 1 1 1

CP 2 CQ2 a2 b2 Solution: Let M ( x1 , y1 ) be the midpoint of PQ so that
the equation of the chord PQ is
9. P is a point on the hyperbola
xx1 yy1 x12 y12
2 2 2
2
 2

x y a b a b2
1
a2 b2 This chord touches the hyperbola. This implies
336 Chapter 5 Ellipse and Hyperbola

2 2 2 This meets the directrix x  a /e at point


¥ b2 ´ ¥ x12 y12 ´ 2
2 ¥ b x1 ´ 2 2 2 2 2
¦y µ ¦ 2 µ  a ¦ 2 µ b (& c  a m b ) ¨ a b (1 e cos Q ) ·
§ 1¶ §a b2 ¶ § a y1 ¶ Q= © ,
ªe e sin Q ¸¹
2
b2 ¥ x 2 y 2 ´ b2 x 2 Also S  (ae, 0) is the corresponding focus. Now,
 2 ¦ 12 12 µ  2 12 1
y1 § a b ¶ a y1 Slope of SP s Slope of SQ

¥ x 2 y2 ´
2 ¨ b tan Q 0 · ¨ [(b /e)(1 e sec Q )] 0 ·
x 2 y2 © ¸© ¸
 ¦ 12 12 µ  12 12 ª a sec Q ae ¹ ª [(a /e) ae]sin Q ¹
§a b ¶ a b
b sin Q b(1 e cos Q )
Hence, the required locus is  s
a(1 e cos Q ) a(1 e 2 ) sin Q
2
¥ x 2 y2 ´ x 2 y2 b2 b2
   1 [& b2  a 2 (e 2 1)]
¦ 2 2µ  2 2 a 2 (1 e 2 ) b2
§a b ¶ a b
Therefore, PQ subtends right angle at S(ae, 0).
11. Prove that the line y  x 2 touches the hyperbola
5 x 2 9 y2  45 and also find the point of contact.
13. A variable chord of the hyperbola
Solution: The given hyperbola is x2 y2
1
x2 y2 a2 b2
1
a2 b2 touches the circle x 2 y2  k 2 . Show that the locus of
For the line y  x 2, we have c  2 and m  1. Now, the midpoint of the chord is
2
a2m2 − b2  9(1) − 5  4  c2 ¥ x 2 y2 ´ 2¥x
2
y2 ´
¦ 2 2 µ k ¦ 4 4 µ
Hence, the line y  x 2 touches the hyperbola. Suppose §a b ¶ §a b ¶
( x1 , y1 ) is the point of contact. So
Solution: Let M ( x1 , y1 ) be the midpoint of a chord of
xx1 yy1
1
9 5 x2 y2
1
This line and y  x 2 represent the same line. There- a2 b2
fore so that its equation is
x1 / 9 y1 / 5 1 S1  S11
 
1 1 2 xx1 yy1 x12 y12
 
Hence a2 b2 a2 b2
9 5 This line touches the circle x2 + y2 = k2. So
x1  and y1 
2 2
0 0 ( x12 /a 2 ) ( y12 /b2 )
The point of contact is ( 9 / 2, 5 / 2). k
( x12 /a 4 ) ( y12 /b4 )
12. The tangent at a point P on the hyperbola 2
¥ x 2 y2 ´ ¥ x 2 y2 ´
 ¦ 12 12 µ  k 2 ¦ 14 14 µ
x2 y2 §a b ¶ §a b ¶
2
1
a b2 Therefore, the locus of ( x1 , y1 ) is
meets one of the directrices at Q. Show that PQ sub- 2
tends right angle at the corresponding focus. ¥ x 2 y2 ´ 2¥x
2
y2 ´
¦ 2 µ  k ¦ 4 µ
§a b2 ¶ §a b4 ¶
Solution: Let P  (a sec Q , b tan Q ) so that the tangent
at P is
14. A variable tangent to the hyperbola
x y
sec Q tan Q  1 x2 y2
a b 1
a2 b2
5.3 Hyperbola 337

meets the tangent at the vertex (a, 0) at R and the  x13 x1 y12  ay12
transverse axis at Q. Find the locus of the midpoint
of QR.  x13  y12 ( x1 a)

Solution: Let P  (a sec Q , b tan Q ) so that the tangent Therefore, the locus of ( x1 , y1 ) is y2 ( x a)  x 3 .
at P is
16. Find the locus of the midpoint of the normal chords
x sec Q y tan Q of a rectangular hyperbola.
1 (5.70)
a b
Solution: The equation of the chord of x 2 y2  a 2
Transverse axis is
with midpoint ( x1 , y1 ) is
y0 (5.71a)
xx1 yy1  x12 y12
The tangent at (a, 0) is
Suppose this is normal to x 2 y2  a 2 at (a sec Q , a tan Q )
xa (5.71b)
so that the equation of the normal is
Therefore,
x cos Q y cot Q  2a
¨ b(1 cos Q ) ·
Q  (a cos Q , 0) and R  ©a, ¸¹ Hence
ª sin Q
x1 y1 x12 y12
Let (h, k) be the midpoint of QR. Therefore  
cos Q cot Q 2a
Q
2 h  a(1 cos Q )  2a cos2 Therefore
2
Q x12 y12 ( x12 y12 )
 h  a cos2 (5.72) sec Q  and tan Q 
2 2ax1 2ay1

b(1 cos Q ) 2b sin 2 (Q / 2) b ¥Q´ Now


and k    tan ¦ µ
2 sin Q 4 sin(Q / 2)cos(Q / 2) 2 § 2¶ 1  sec 2 Q tan 2 Q
(5.73)
( x12 y12 )2 ( x12 y12 )2
From Eqs. (5.72) and (5.73), we get 
4a 2 x12 4a 2 y12
2
a ¥ 2k ´ Q Q ¨ y2 x 2 ·
¦ µ  sec 2 tan 2  1  ( x12 y12 )2 © 12 2 12 ¸
h § b¶ 2 2
ª 4a x1 y1 ¹
Therefore, the locus of (h, k ) is
 ( x12 y12 )3 4a 2 x12 y12  0
a 4 y2
1 Therefore, the locus of ( x1 , y1 ) is
x b2
( x 2 y 2 )3 4 a 2 x 2 y 2  0
2 2 2
15. The chords of the hyperbola x y  a touch the
parabola y2  4ax. Find the locus of their midpoint. 17. From any point on a hyperbola, tangents are drawn
to another hyperbola having the same asymptotes.
Solution: The equation of the chord of x 2 y2  a 2 Show that the chord of contact cuts off a triangle of
whose midpoint is ( x1 , y1 ) is constant area from the asymptotes.

xx1 yy1  x12 y12 Solution: Let the hyperbolas be


2
This chord touches the parabola y  4ax. This implies x2 y2
1 (5.74)
x12 y12 a ¥ a´ a2 b2
 ¦§& c  µ¶
y1 x1 /y1 m x2 y2
and k (5.75)
x 2 y12 ay a2 b2
 1  1
y1 x1
338 Chapter 5 Ellipse and Hyperbola

Let P(a sec Q , b tan Q ) be a point on the hyperbola pro- a6 [( x12 /a 2 ) ( y12 /b2 )]2 b6 [( x12 /a 2 ) ( y12 /b2 )]2
vided in Eq. (5.74). The chord of contact of P with hyper- 
x12 (a 2 b2 )2 y12 (a 2 b2 )2
bola provided in Eq. (5.75) is
x y [( x12 /a 2 ) ( y12 /b2 )]2 ¨ a6 b6 ·
sec Q tan Q  k (5.76)  1 © 2 2¸
a b (a 2 b 2 ) 2 ª x1 y1 ¹
The asymptotes are (b2 x12 a 2 y12 )2 (a6 y12 b6 x12 )
 1
¥ b´ a 4 b4 (a 2 b2 )2 x12 y12
y p¦ µ x
§ a¶
Hence, the locus of ( x1 , y1 ) is
These asymptotes meet the line provided in Eq. (5.76) at
the points a 4 b 4 ( a 2 b 2 ) 2 x 2 y 2  ( b 2 x 2 a 2 y 2 ) 2 ( a 6 y 2 b6 x 2 )
[ak(sec Q tan Q ), bk(sec Q tan Q )]
19. Prove that the locus of the midpoint of a chord of the
and [ak(sec Q tan Q ), bk(tan Q sec Q )] hyperbola ax 2 by2  1 which subtends right angle
at the centre of the curve is
and hence the area of the triangle cut off is given by
(a b)(ax 2 by2 )2  a 2 x 2 b2 y2
2
y  abk 2 sec 2 Q tan 2 Q  abk 2
2 Solution: The given conic is
which is constant.
x2 y2
1
18. Show that the locus of the midpoint of normal chords (1/a) (1/b)
x2 y2 The equation of the chord PQ in terms of its midpoint
of the hyperbola 2
 1 is
a b2 ( x1 , y1 ) is
(a6 y2 b6 x 2 )(a 2 y2 b2 x 2 )2  (a 2 b2 )2 a 4 b4 x 2 y2 axx1 byy1  ax12 by12 (5.79)

Solution: The equation of a chord with midpoint Hence, the combined equation of the pair of lines CP
( x1 , y1 ) is and CQ (C is the origin) is
2
xx1 yy1 x12 y12 ¨ axx1 byy1 ·
 (5.77) ax 2 by2 1 © 2 2 ¸
0
a2 b2 a2 b2 ª ax1 by1 ¹
Suppose the equation of the chord provided in Eq. (5.77) Since PCQ  90o in the above equation
is normal to the hyperbola at P(a sec Q , b tan Q ) whose
equation is Coefficient of x 2 Coefficient of y2  0
ax cos Q by cot Q  a 2 b2 (5.78) ¥ a 2 x12 ´ ¥ b2 y12 ´
¦a 2

2 2µ ¦
b 2 2 2µ
0
From Eqs. (5.77) and (5.78), we get § (ax1 by1 ) ¶ § (ax1 by1 ) ¶

x1 /a 2 y1 /a 2 ( x12 /a 2 ) ( y12 /b2 ) (a 2 x12 b2 y12 )


  a b 0
a cos Q b cos Q a 2 b2 (ax12 by12 )2
Hence  (a b)(ax12 by12 )2  a 2 x12 b2 y12
a 3 [( x12 /a 2 ) ( y12 /b2 )] Hence, the locus of ( x1 , y1 ) is
sec Q 
x1 (a 2 b2 )
(a b)(ax 2 by2 )2  a 2 x 2 b2 y2
b3 [( x12 /a 2 ) ( y12 /b2 )]
and tan Q 
y1 (a 2 b2 ) 20. The chords of the circle x 2 y2  a 2 touch the hy-
Now x2 y2
perbola 2

 1. Prove that the midpoints of the
1  sec 2 Q tan 2 Q a b2
chords lie on the curve ( x 2 y2 )2  a 2 x 2 b2 y2 .
5.3 Hyperbola 339

Solution: The chords of the circle x 2 y2  a 2 whose The latus rectum through S(ae, 0) is
midpoint ( x1 , y1 ) is xx1 yy1  x12 y12 . This chord touch- x  ae (5.81)
es the hyperbola. This implies The asymptote is
2 2
¥ x12 y12 ´ x y
2 ¥ x1 ´ 2 0
¦ y µ  a ¦§ y µ¶ b a b
§ 1 ¶ 1
¥ b´
 ( x12 y12 )2  a 2 x12 b2 y12  y¦ µ x (5.82)
§ a¶
Therefore, ( x1 , y1 ) lies on the curve The point of intersection of Eqs. (5.81) and (5.82) is
2 2 2
(x y )  a x b y 2 2 2 2 (ae, be). By hypothesis, the point (ae, be) lies on the
tangent provided in Eq. (5.80) which implies that
21. Prove that the locus of the midpoints of the chords of e(sec Q tan Q )  1
x 2
y 2  e  sec Q tan Q (5.83)
2
2
1
a b Now, the slope of SP is
passing through a fixed point (h, k ) is also a hyperbola b tan Q 0 b sin Q
with centre at (h / 2, k / 2). 
a sec Q ae a(1 e cos Q )
Solution: The equation of chord in terms of its mid- Using Eq. (5.83), we get
point ( x1 , y1 ) is
b sin Q b sin Q

xx1 yy1 x12 y12 a(1 e cos Q ) a[1 cos Q (sec Q tan Q )]

a2 b2 a2 b2 b sin Q b
 
This passes through the point (h, k). So a[1 1 sin Q ] a

hx1 ky1 x12 y12 which is the slope of the other asymptote

a2 b2 a2 b2 x y
0
a b
x12 hx1 y12 ky1
 2
0
a b2 23. A normal to the hyperbola
2 2 2 2
( x1 h/ 2) ( y1 k/ 2) h k x2 y2
  1
a2 b2 4a 2 4 b2 a 2
b2
Therefore, the locus of ( x1 , y1 ) is the hyperbola meets the axes at Q and R. Lines QL and RL are
drawn at right angles to the axes and meet at L.
[ x 2 (h / 2)]2 [ y (k/ 2)]2 1 ¥ h2 k 2 ´ Prove that the locus of L is the hyperbola
 ¦ 2 2µ
a2 b2 4§a b ¶
a 2 x 2 b 2 y 2  (a 2 b 2 )
22. On the hyperbola
Solution: See Fig. 5.34. The normal at P(a sec Q , b tan Q )
x2 y2 is ax cos Q by cot Q  a 2 b2 . Therefore
1
a2 b2 ¨ ¥ a 2 b2 ´ ·
Q  ©¦ µ sec Q , 0 ¸
P is a point and S is one of the foci. If the tangent at ©ª§ a ¶ ¸¹
P, the latus rectum through S and one of the asymp-
totes are concurrent, then show that focal radius SP ¨ ¥ a 2 b2 ´ ·
is parallel to the other asymptote. and R  ©0, ¦ µ tan Q ¸
©ª § a ¶ ¸¹
Solution: The tangent at P(a sec Q , b tan Q ) is If L  ( x, y), then
x sec Q y tan Q ¥ a 2 b2 ´
1 (5.80)
a b x¦ µ sec Q
§ a ¶
340 Chapter 5 Ellipse and Hyperbola

¥ a 2 b2 ´ 2B a 4 b2
and y¦ µ tan Q y1 y2 
§ b ¶ a B A 2 b4
4 2

Therefore
b 4 a 2 (a 2 A 2 )
2 2
and y1 y2 
¥ ax ´ ¥ by ´ a 4 B 2 A 2 b4
¦ 2 2µ
¦ 2 2µ
 sec 2 Q tan 2 Q  1
§ a b ¶ § a b ¶ Therefore

 a 2 x 2 b 2 y 2  (a 2 b 2 ) 2 1 1 y1 y2

y1 y2 y1 y2
y
2B a 4 b2

L
b 4 a 2 (a 2 A 2 )
R
Normal
P ( 2B )a 2

C
b 2 (a 2 A 2 )
Q
2B

b /a )(a 2 A 2 )
(b 2 2

2B 2 ¥ A 2 B2 ´
FIGURE 5.34  2
 ¦& 2 2  1µ
B B § a b ¶
24. If the tangent at (A , B ) of
Hence, A is harmonic between y1 and y2 .
2 2
x y
1 25. Prove that the chord of the hyperbola which touches
a2 b2
the conjugate hyperbola is bisected at the point of
meets the auxiliary circle at ( x1 , y1 ) and ( x2 , y2 ), contact.
then show that A is harmonic between y1 and y2 .
Solution: Let the hyperbola and its conjugate be
Solution: The tangent at (A , B ) is
x2 y2
xA yB 1
1 (5.84) a2 b2
2 2
a b
x2 y2
and the auxillary circle is and 2
 1
a b2
x 2 y2  a 2 (5.85) respectively. Let P(h, k ) be a point on
Substituting
x2 y2
2  1
x
a ¥ yB ´ a2 b2
¦§ 1 2 µ¶
A b Hence
in Eq. (5.85), we get h2 k2
2
 1 (5.86)
a 4 (b2 B y)2 a b2
y2  a 2
A2 b4 The equation of the tangent at (h, k) is
a 4 [b4 2 B b2 y B 2 y 2 ] hx ky
 y2  a 2 2
 1 (5.87)
A b 2 4 a b2

 (a 4 B 2 A 2 b 4 ) y 2 2 B a 4 b 2 y a 4 b 4 a 2 A 2 b 4  0 Let the tangent provided in Eq. (5.87) meet the hyper-


bola
Let y1 and y2 be the roots of this quadratic equation.
x2 y2
Then 1
2
a b2
5.3 Hyperbola 341

at Q and R. We have to show that (h, k) is the midpoint The line provided in Eq. (5.90) touches the circle given
of the chord QR. Now, the equation of the chord QR in by Eq. (5.91). This implies
terms of its midpoint, say, ( x1 , y1 ) is
[(0) x1 /a 2 ] [(0) y1 /b2 1]
xx1 yy1 x12 y12  ae
2
 2 2
(5.88) ( x12 /a 4 ) ( y12 /b4 )
a b2 a b
Thus, Eqs. (5.87) and (5.88) represent the same straight x12 y12 1 1
 4
4
 2 2

line. Hence a b a e a b2
2

h/a 2 k/a 2 1
2
 2
 27. Show that the locus of the point of intersection of
x1 /a y1 /b ( x12 /a 2 ) ( y12 /b2 ) tangents to the hyperbola
h k 1 x2 y2
   (5.89)
x1 y1 ( x12 /a 2 ) ( y12 /b2 ) 2
1
a b2
Therefore, whose sum of the slopes is a constant p is
x1 p( x 2 a 2 )  2 xy
h
( x1 /a ) ( y12 /b2 )
2 2

Solution: Let ( x1 , y1 ) be a point through which tan-


y1
and k gent can be drawn. It is known that y  mx a 2 m2 b2
( x12 /a 2 ) ( y12 /b2 ) touches the hyperbola. This tangent passes through
Hence, from Eq. (5.86), we have ( x1 , y1 ). This implies

h2 k2 ( x12 /a 2 ) ( y12 /b2 ) 1 ( y1 mx1 )2  a 2 m2 b2


1  
a2 b2 [( x12 /a 2 ) ( y12 /b2 )]2 ( x12 /a 2 ) ( y12 /b2 )  ( x12 a 2 )m2 2 mx1 y1 y12 b2  0
x12 y12 This equation being a quadratic in m has two roots which
  1
a2 b2 are the slopes of the tangents from ( x1 , y1 ). Therefore
Hence, from Eq. (5.89), we have 2 x1 y1
p  Sum of slopes 
h k ( 1) x12 a 2
  1
x1 y1 ( 1) Hence, the locus of the point ( x1 , y1 ) is p( x 2 a 2 ) = 2xy.

Thus, h  x1 and k  y1 . 28. Find the locus of the midpoints of the chords of
ax 2 by2  1 which touch the parabola y2  4 px.
26. Let P be a point such that the chord of contact of
x2 y2 Solution: The given hyperbola is
P with respect to the hyperbola 2
2
 1 touches
a b x 2 y2
the circle described on the line joining the foci as 1
ends of a diameter. Show that P lies on the curve 1/a 1/b
x 2 y2 1
 . The equation of the chord of the hyperbola whose mid-
a 4 b4 a 2 b2 point is ( x1 , y1 ) is

Solution: Let P  ( x1 , y1 ) so that its chord of contact axx1 byy1  ax12 by12
with respect to the hyperbola is This line touches the parabola which implies
xx1 yy1
1 (5.90) (ax12 by12 ) p
a2 b2 
by1 ax1 /by1
The equation of the circle with S(ae, 0) and Sa(−ae, 0) as
ends of a diameter is ax1 (ax12 by12 )  pb2 y12
 ax1 (ax12 by12 ) pb2 y12  0
( x ae)( x ae) y2  0

 x 2 y2  (ae)2 (5.91) Thus, the locus of ( x1 , y1 ) is ax(ax 2 by2 ) pb2 y2  0.


342 Chapter 5 Ellipse and Hyperbola

29. The chord of the circle x 2 y2  a 2 touches the 81y12


 9 x12  ( x12 y12 )2
rectangular hyperbola x 2 y2  a 2 . Show that the 4
locus of the midpoint of the chord is (x2 + y2)2 =
Therefore, the locus of ( x1 , y1 ) is
a 2 ( x 2 y2 ).
9 y2 1 2
2 2 2 x2  ( x y 2 )2
Solution: The equation of the chord of x y  a 4 9
with ( x1 , y1 ) as its midpoint is xx1 yy1  x12 y12 . This
31. In a rectangular hyperbola xy  c 2 , if m is the slope
chord touches the hyperbola x 2 y2  a 2 . So of a chord, then show that the equation of the circle
2 2 described on this chord as diameter is of the form
¥ x12 y12 ´ 2 ¥ x1 ´ 2
¦ y µ  a ¦§ y µ¶ a ¥ 1´
§ 1 ¶ 1 x 2 y2 c 2 ¦ m µ L ( x my)  0
§ m¶
 ( x12 y12 )2  a 2 ( x12 y12 )
for some real number K.
Hence, the locus of ( x1 , y1 ) is ( x 2 y2 )2  a 2 ( x 2 y2 ).
Solution: Let

30. Tangents are drawn to the circle x 2 y2  9 from ¥ c´ ¥ c´


A ¦ ct1 , µ and B ¦ ct2 , µ
point on the hyperbola § t1¶ § t 2¶

x 2 y2 be the two points on xy  c 2 such that


1
9 4 Slope of AB  m
Find the locus of the midpoint of the chord of con-
c/t1 c/t2
tact. (IIT-JEE 2005)  m
ct1 ct2
Solution: Let M ( x1 , y1 ) be the midpoint of chord of 1
 t1t2  (5.95)
contact of the circle x 2 y2  9. Hence, its equation is m
xx1 yy1  x12 y12 (5.92) The circle having A and B as extremities of a diameter is

Suppose the line provided in Eq. (5.92) is the chord of ¥ c´¥ 1´


contact of (h, k ) on the hyperbola. Therefore ( x ct1 )( x ct2 ) ¦ y µ ¦ y µ  0
§ t1 ¶ § t2 ¶
h2 k 2 ¥ 1 1´ c2
1 (5.93)  x 2 y2 c(t1 t2 ) x c ¦ µ y c 2 t1t2 0
9 4 § t1 t2 ¶ t1t2
The equation of the chord of contact of (h, k ) with ¥ 1 ´
respect to the circle x 2 y2  9 is  x 2 y2 c(t1 t2 ) x cy(t1 t2 )( m) c 2 ¦ mµ  0
§ m ¶
hx ky  9 (5.94) ¥ 1´
 x 2 y2 c 2 ¦ m µ c(t1 t2 )( x my)  0
Now, Eqs. (5.92) and (5.94) represent the same line. § m¶
Therefore Take L  c(t1 t2 ) to get the required result.
h k 9
  2 32. Prove that the orthocentre of a triangle inscribed in a
x1 y1 x1 y12
rectangular hyperbola lies on the rectangular hyper-
9 x1 9 y1 bola.
h ,k 
x12 y12 x12 y12
Solution: Suppose
From Eq. (5.93), since
¥ c´ ¥ c´ ¥ c´
2 2 P ¦ ct1 , µ , Q ¦ ct2 , µ and R ¦ ct3 , µ
h k § t1 ¶ § t2 ¶ § t3 ¶
1
9 4
be the three points on the curve xy  c2. Since, the slope of
we have QR is 1/t2 t3 , the equation of the altitude through P is
81x12 81y12 c
1 y  t2 t3 ( x ct1 )
9( x12 y12 )2 4( x12 y12 )2 t1
5.3 Hyperbola 343

c It passes through (h, k). This implies


 y  t2 t3 x ct1t2 t3 (5.96)
t1 ht 3 tk c ct 4  0
The equation of the altitude through Q is  ct 4 ht 3 kt c  0 (5.98)
c If xr  ctr and yr  c /tr for r  1, 2, 3 and 4, then t1, t2, t3
y  t3 t1 x ct1t2 t3 (5.97)
t2 and t4 are the roots of Eq. (5.98). Therefore
Subtracting Eq. (5.97) from Eq. (5.96), we have h
t1 t2 t3 t4 
¥ 1 1´ c
c ¦ µ  t3 (t2 t1 ) x c
§ t2 t1 ¶ and t1t2 t3 t4   1
c
c(t1 t2 )
  t3 (t2 t1 ) x Hence
t1t2
x1 x2 x3 x4  c (t1 t2 t3 t4 )  h
Therefore
¥ 1 1 1 1 ´ c 3 t2 t3 t4
c y1 y2 y3 y4  c ¦ µ 
x § t1 t2 t3 t4 ¶ t1t2 t3 t4
t1t2 t3
k/c
and y  ct1t2 t3 c k
1
so that xy  c 2 . This implies that the orthocentre lies on
the curve. Therefore

x1 x2 x3 x4  c 4 (t1t2 t3 t4 )  c 4 ( 1)  c 4
33. If a circle intersects xy  1 at four points (xr, yr)
(where r  1, 2, 3 and 4), then show that c4
and y1 y2 y3 y4   c4
x1 x2 x3 x4  y1 y2 y3 y4  1 t1t2 t3 t4

Solution: Let the equation of the circle be 35. Let P1, P2, P3 and P4 be four points on xy  c2 such
2 2
S y x y 2 gx 2 fy c  0 that the chord P1P2 is perpendicular to the chord
P3P4. If C is the centre and the lines CP1, CP2, CP3
In S  0, substituting y  1/x we get and CP4 make angles @, A, F and C, respectively, with
an asymptote, then show that
x 4 2 gx 3 cx 2 2 fx 1  0
tan @ tan A tan F tan C  1
This is fourth-degree equation in x. So it has four roots,
say, x1, x2, x3 and x4. Therefore,
Solution: The line through the centre is y  x tan Q so
x1 x2 x3 x4  Product of the roots  1
that it meets the curve xy  c2 at point (c cot Q , c tan Q ).
1 Hence, the four points are (c cot Q , c tan Q ) (where
and y1 y2 y3 y4  1
x1 x2 x3 x4 P  @, A, F and C ). The lines P1P2 and P3P4 are at right
angles. This means
34. If the normals at (xr, yr) (r  1, 2, 3 and 4) on the rect-
angular hyperbola xy  c2 meet in a point (h, k), then ¥ c tan B c tan A ´ ¥ c tan D c tan G ´
prove that ¦ µ¦ µ  1
§ c cot B c cot A ¶ § c cot D c cot G ¶
h  x1 x2 x3 x4
k  y1 y2 y3 y4  ( tan B tan A )( tan D tan G )  1

x1 x2 x3 x4  y1 y2 y3 y4  c 4  tan A tan B tan G tan D  ( 1)2  1

Solution: The normal at (ct, c /t ), according to Theo- 36. A circle cuts a rectangualar hyperbola xy  c2 at A, B,
rem 5.25, part (2), is C and D. If H is the orthocentre of ΔABC, then show
that H and D are the extremities of a diameter of the
xt 3 ty c ct 4  0 curve.
344 Chapter 5 Ellipse and Hyperbola

Solution: If  ctr , c /tr (where r  1, 2, 3 and 4) are Solution: The tangent and normal at P (a sec P, a tan P )
points A, B, C and D, respectively, then by Problem 33, are
the orthocentre H of triangle ABC is
x sec Q y tan Q  a (5.99)
¥ e ´ and x cos Q + y cot Q = 2a (5.100)
¦§ t t t , c t1t2 t3 µ¶
1 2 3
Therefore, a1  a cos Q , a2  2a sec Q , b1  a cot Q and b2 =
which also lies on the curve. Now, since t1t2t3t4  1, we 2a tanP . Hence
have a1a2 b1b2  2a 2 2a 2  0
1
t4  39. A rectangular hyperbola, with centre C, is cut by
t1t2 t3
a circle of radius r at four points P1, P2, P3 and P4.
so that Prove that

¥ c´ ¥ c ´ CP 12 CP 22 CP 23 CP 42  4r 2
D  ¦ ct4 , µ  ¦ , ct1t2 t3 µ
§ t4 ¶ § t1t2 t3 ¶
Solution: Let the hyperbola be xy  c2 and the circle
and hence (0, 0) is the midpoint of HD. be

S y x 2 y2 2 gx 2 fy k  0 (5.101)
37. From any point P of a rectangular hyperbola x2 − y2
 a with centre O, the perpendiculars PM and PN Substituting x  ct and y  c /t in Eq. (5.101), we get
are drawn to the principal axes (i.e., usual axes).
Show that the tangent at P is perpendicular to MN c 2 t 4 2 gct 3 kt 2 2 fct c 2  0
and that the distance of O from the tangent varies whose roots are t1, t2, t3 and t4 where
inversely as OP.
¥ c´
Pr  ¦ ctr , µ
Solution: Let P (a sec P, a tan P ) so that M  (a sec P, § tr ¶
0) and N  (0, a tan P ). The tangent at P is
(where r  1, 2, 3 and 4). Therefore
x sec Q y tan Q  a
2 g
t1 t2 t3 t4 
so that the slope of the tangent at P is c
sec Q k
 cosec Q ¤ t1t2 
tan Q c2
2 f
¤ t1t2 t3 
Slope of MN is c
a tan Q
 sin Q c2
a sec Q and t1t2 t3 t4  1
c2
Now,
Now,
4
¨ 1 1 1 1 ·
Slope of the tangent at P s Slope of MN  cosec P
s (− sin P )  −1
¤ CP
i1
i
2
 c 2 ©t12 t2 2 t32 t4 2 2 2 2 2 ¸
ª t1 t2 t3 t4 ¹

¨ 2
Hence, the tangent at P is perpendicular to MN. Also d is ¥ 1´ 1 ·
the distance of O from the tangent at P which is given by
2
©ª
2
 c ©(3t1 ) 2 3t1t2 ¦
§ ¤
t1 µ¶
2 ¤ ¸
t1t2 ¸
¹
2
a a ¨ 4 g 2 2k ¥ ¤ t t t ´ 2
d  2 3 t3 t4 ·
2
sec Q tan Q 2 OP  c2 © 2 2 ¦ 1 2 3 µ ¸
©ª c c § t1t2 t3 t4 ¶ t1t2 t3 t4 ¸
¹
¨ 4g 2 2
38. If the tangent and normal to a rectangular hyperbola 2k 4 f 2k ·
 c2 © 2 2 2 2 ¸ (&t1t2 t3 t4  1)
x 2 y2  a 2 cut off intercepts a1 and a2 on one axis ª c c c c ¹
and b1 and b2 on another axis, then show that
 4 (g 2 f 2 k)
a1a2 b1b2  0
 4r 2
Worked-Out Problems 345

40. Show that if a rectangular hyperbola cuts a circle at x1 x2 x3 x4  2 g


four points, the centre of mean position of the four
points is in midway between the centres of the two ¥ 1 1 1 1´
and y1 y2 y3 y4  k ¦ µ
curves. § x1 x2 x3 x4 ¶
k ¤ x1 x2 x3 k( 2 fk )
Solution: Let xy  k and    2f
x1 x2 x3 x4 k2
S y x 2 y2 2 gx 2 fy c  0 (5.102)
Now, the mean position of the four points is
Substituting y  k /x in Eq. (5.102), we get
¥ x1 x2 x3 x4 y1 y2 y3 y4 ´ ¥ g f ´
x 4 2 gx 3 cx 2 2 fkx k 2  0 ¦§ , µ¶  ¦§ , µ
4 4 2 2 ¶
whose roots are the abscissae of the common points of which is the midpoint of the line joining the centre
the circle and the rectangular hyperbola. If (x1, y1), (x2, (−g, −f ) of the circle and C(0, 0) of the rectangular hyper-
y2), (x3, y3) and (x4, y4) are the points of intersection, bola.
then

WORKED-OUT PROBLEMS
Single Correct Choice Type Questions
1. If m1 and m2 are the slopes of the tangents to the hy- ¥1 1 ´
perbola (A) ¦ , (B) (4, 6 )
§ 2 6 µ¶
x 2 y2
1 (C) (4, 6) (D) ( 2, 6)
25 16
(IIT-JEE 2005)
from the point (6, 2), then
Solution: Suppose 2 x 6 y  2 touches at (x1, y1). At
1 1 (x1, y1), the equation of the tangent to the hyperbola is

m1 m2
xx1 yy1
1
5 6 4 5 4 2
(A) (B) (C) (D)
6 5 5 4 That is, both equations represent the tangent at ( x1 , y1 ) .
Hence
Solution: Suppose the tangent y  mx 25m2 16
passes through the point (6, 2). That is, ( x1 /4) ( y1 /2) 1
 
(6 m 2)2  25m2 16 2 6 2

 11m2 24 m 20  0  x1  4, y1  6
The roots are m1 and m2. Therefore Hence point of contact = (4, 6 ).
Answer: (B)
24
m1 m2 
11 3. If the foci of the ellipse
20
and m1 m2  x 2 y2
11 1
16 b2
Hence and the hyperbola

1 1 24 6 x 2 y2 
  
m1 m2 20 5 144 81 25
Answer: (B)
coincide, then the value of b2 is
2 2
2. If 2 x 6 y  2 touches the hyperbola x 2 y  4,
then the point of contact is
346 Chapter 5 Ellipse and Hyperbola

(A) 1 (B) 5 (C) 7 (D) 9  c  p 65


Solution: From the ellipse equation, we have
Hence, there are two values of c, namely, 65 and 65 .
b2  a 2 (1 e 2 ). From the hyperbola equation, we have
Answer: (C)
81 144 2
 (e1 1)
25 25 6. Tangents are drawn to x 2 2 y2  2. The locus of the
81 midpoint of the intercept made by tangents between
 1  e12 the axis is
144
15 1 1 1 1
 e1  (A) 2
2
1 (B) 2
1
12 x 2y 4x 2 y2

Foci of the hyperbola are 1 1 1 1


(C) 2
2
1 (D) 2
1
2x 4y 2x y2
¥ 12 ¥ 15 ´ ´
¦§ p 5 ¦§ 12 µ¶ , 0µ¶  (p 3, 0) Solution: Let ( x1 , y1 ) be the midpoint of intercept of
the tangent between the axes. The given ellipse is
Now for the ellipse (3, 0)  (4e, 0)  e  3 / 4. Therefore x 2 y2
1
2 1
¥ 9´
b2  a 2 (1 e 2 )  16 ¦ 1 µ  7
§ 16 ¶ The tangent at ( 2 cos Q , sin Q ) is
Answer: (C) x cos Q y sin Q
1
2 1
4. The eccentricity of the hyperbola 9x2 16y2 72x
96 y 144  0 is so that it meets the axes at A( 2 sec Q , 0) and B(0, cosecP)
3 4 6 5 Since ( x1 , y1 ) is the midpoint of AB, we have
(A) (B) (C) (D)
2 3 5 4 2 x1  2 sec Q and 2 y1  cosec Q
Solution: The given equation can be written as 1 1
 cos Q  and sin Q 
9(x2 8x) 16(y2 6y) 144  0 x1 2 2 y1
2 2
 9( x 4) 16( y 3)  144 144 144 1 1
  cos2 Q sin 2 Q  1
2
( x 4) ( y 3) 2 2 x12 4 y12
 1
16 9 Therefore, the locus of ( x1 , y1 ) is
Therefore, the eccentricity is given by 1 1
1
9  16(e 1) 2
2x2 4 y2
5 Answer: (C)
 e
4
x2 y2
Answer: (D) 7. If x cos A y sin A  p touches the ellipse  1,
2
then a b2
5. The number of values of c such that the line y  4 x c
(A) p2  a 2 sin 2 A b2 cos2 A
x 2 y2
touches the ellipse  1 is (B)p2  a 2 cos2 A b2 sin 2 A
4 1
(A) 0 (B) 1 (C) p2  a 2 cosec 2A b2 sec 2 A
(C) 2 (D) infinite (D) p2  a 2 cot 2 A b2 tan 2 A
Solution: We have Solution: The given line equation can be written as

c 2  a 2 m 2 b2 y  ( cot A ) x p cosec A
It touches the ellipse. So
 c 2  4(16) 1  65
p2 cosec 2 A  a 2 ( cot A )2 b2
Worked-Out Problems 347

p2 a 2 cos2 A b2 sin 2 A 11 11 7 7
  (A) (B) (C) (D)
sin 2 A sin 2 A 3 3 3 3

 p2  a 2 cos2 A b2 sin 2 A Solution: See Fig. 5.36. Since the circle is to be largest,
it must touch the ellipse (internally) at some point, say,
Answer: (B)
P(4 cos Q , 2 sin Q ). The tangent to the ellipse P is
8. The tangents are drawn to the ellipse x cos Q y sec Q
1
x 2
y 2 4 2
1
9 5 whose slope is ( 1/ 2)cot Q . Also the line joining (1, 0)
and P is perpendicular to the tangent. So
at the ends of latus recta. The area of the quadrilateral
formed is ¥ 1 ´ (2 sin Q 0)
¦§ cot Q µ¶  1
27 27 27 2 4 cos Q 1
(A) 27 (B) (C) (D)
2 11 55 cos Q
 1
Solution: The eccentricity e is given by 4 cos Q 1

5  9 (1 e 2 )  cos Q 
1
and sin Q  p
2 2
2 3 3
 e
3 The radius of the circle is
Therefore, the foci are ( ae, 0)  ( 2, 0) and (ae, 0)  12
(2, 0). We have 2 2 12
¨¥ 4 ´ 2 ¥ 8´ ·
[(4 cos Q 1) (2 sin Q ) ]  ©¦ 1µ 4 ¦ µ ¸
¥ 2´ ©ª§ 3 ¶ § 9¶ ¸
¹
b ¥ 5´
L  ¦ ae, µ  ¦ 2, µ 33 11
§ a ¶ § 3¶  
9 3
See Fig. 5.35. The tangent at L(2, 5 / 3) is
2 x (5/3) y
1 P
9 5
2x y
 1
9 3
(1, 0)
so that P  (0, 3) and S  (9 / 2, 0). Therefore, the area of
the parallelogram is
¥1 9´
4 ( $ POS)  4 ¦ s 3 s µ  27
§2 2¶
FIGURE 5.36
P Answer: (B)
L1 L
B 10. Any ordinate NP of an ellipse

x2 y2
R A 1
C O A a2 b2
B meets the auxiliary circle at Q. Then, the locus of the
L1 L
normals at P and Q is
Q
(A) x 2 y2  (a b)2 (B) x 2 y2  (a b)2
FIGURE 5.35
(C) x 2 y2  (a 2 b2 )1/ 2 (D) x 2 y2  (a 2 b2 )1/ 2
Answer: (A)
Solution: Let P  (a cos Q , b sin Q ) so that Q  (a cos Q ,
9. The radius of the largest circle inscribed in the ellipse
a sinP ). The normal at P to the ellipse is
x 2 y2 ax by
 1 and having centre at (1, 0) is  a 2 b2 (5.103)
16 4 cos Q sin Q
348 Chapter 5 Ellipse and Hyperbola

and the normal Q to the circle is x 2 y2 x 2 y2


(A) 1 (B) 1
y  x tan Q (5.104) 12 9 12 8
Solving Eqs. (5.103) and (5.104), we have x 2 y2 x 2 y2
(C) 2 (D) 2
ax bx tan Q 12 8 8 12
 a 2 b2
cos Q sin Q Solution: The tangents at P and Q, respectively, are

 (a b) x  (a 2 b2 )cos Q x cos Q y sin Q


1 (5.105)
 x  (a b)cos Q and y  x tan Q  (a b)sin Q 12 8
x sin Q y cos Q
 x 2 y2  (a b)2 and 1 (5.106)
12 8
Therefore, the locus of the point of intersection is Squaring Eqs. (5.105) and (5.106) and adding, we have
x 2 y2  (a b)2 .
Answer: (A) x 2 y2
2
12 8
11. The smallest possible positive slope of a line whose Answer: (C)
y-intercept is 5 and which has a common point with
the ellipse 9 x 2 16 y2  144 is 14. The sum of the squares of the perpendiculars drawn
3 4 9 onto any tangent to the ellipse
(A) (B) 1 (C) (D)
4 3 16 x2 y2
2
1
Solution: Let the line be y  mx 5. Since m  0 and is a b2
to be least, the line should touch the ellipse. Hence from the two points on the minor axis, each at a dis-
2 2
5  16 m 9  m  p 1 tance of a 2 b2 from the centre, is
However, m  0  m  1. 1 2 3 2
Answer: (B) (A) a 2 (B) a (C) a (D) 2a 2
2 2

12. Consider an ellipse with foci at (5, 15) and (21, 15). If Solution: We have
the x-axis touches the ellipse, then the length of the
a 2 b2  a 2 a 2 (1 e 2 )
major axis is
 a2 e2
(A) 17 (B) 34 (C) 13 (D) 416
Solution: The major axis is y  15. The distance be- Let P  (0, ae) and Q  (0, ae). Let p1 and p2 be the
tween the foci is perpendiculars drawn from P and Q onto

2ae  16  ae  8 x cos Q y sin Q


1
a b
Since x-axis is touching the ellipse, we have b  15 so that
Therefore
152  b2  a 2 (1 e 2 )  a 2 64
8 2
p1 
;(ae sin Q/b) 1=2
 a  17 and e  (cos2 Q/a 2 ) (sin 2 Q/b2 )
17
Therefore, the length of the major axis is 2a s17  34.
p2 2 
;( ae sin Q/b) 1=2
Answer: (B) and
[(cos2 Q )/a 2 ] [(sin 2 Q )/b2 ]

13. Let P  ( 12 cos Q , 8 sin Q ) and Q  ( 12 sin Q , Hence


8 cos Q ) be the points on the ellipse
a 2 (ae sin Q b)2 a 2 (ae sin Q b)2
2 2
p12 p2 2 
x y b2 cos2 Q a 2 sin 2 Q
1
12 8 2a 2 [a 2 e 2 sin 2 Q b2 ]
Then, the locus of the point of intersection of the 
b2 cos2 Q a 2 sin 2 Q
tangents at P and Q is
Worked-Out Problems 349

2a 2 [a 2 e 2 sin 2 Q b2 ] meets the auxiliary circle at point M. Then the area of


 the triangle with vertices at A, M and the origin O is
b2 (1 sin 2 Q ) a 2 sin 2 Q
31 29 21 27
2a 2 [a 2 e 2 sin 2 Q b2 ] (A) (B) (C) (D)
 10 10 10 10
b2 (a 2 b2 )sin 2 Q
(IIT-JEE 2009)
2a 2 [a 2 e 2 sin 2 Q b2 ] 2 2 2 2
 (&a b  a e ) Solution: See Fig. 5. 37. We have
b2 a 2 e 2 sin 2 Q
A  (3, 0), B  (0, 1)
 2a 2
The equation of the line AB is
Answer: (D)
0 1
2 2
y ( x 3)
x y 3 0
15. The tangent and normal to the ellipse  1 at
4 1  x 3y  3 (5.107)
the point P (2 cos Q , sin Q ) meet the major axis at Q
and R, respectively. If QR  2, then cos Q is equal to The equation of the auxiliary circle is

1 2 2 2 3 x 2 y2  9 (5.108)
(A) (B) (C) (D)
3 3 3 3 Substituting x  3 3 y in Eq. (5.108), we get
Solution: The tangent at P is
(3 3 y)2 y2  9
x cos Q y sin Q
1  10 y2 18 y  0
2 1
9
so that Q  (2 sec Q , 0). The normal at P is  y  0,
5
2x y Now, y  0  x  3 so that the point is A(3, 0) and
 2 2 12  3
cos Q sin Q
9 12
y  x
Hence 5 5
¥ 3 cos Q ´ so that
R¦ , 0µ
§ 2 ¶
¥ 12 9 ´
M¦ ,
Now, § 5 5 µ¶

QR  2 Now, A  (3, 0), O  (0, 0) and M  ( 12 /5, 9 /5). There-


fore, area of $AOB is given by
3
 2 sec Q cos Q  2
2 1 ¥ 9´ ¥ 12 ´ 27
3¦ µ 0 ¦ 
2
 4 3 cos Q  4 cos Q 2 § 5¶ § 5 µ¶ 10

 3 cos2 Q 4 cos Q 4  0 M
2
 3 cos Q 6 cos Q 2 cos Q 4  0
 3 cos Q (cos Q 2) 2(cos Q 2)  0 B

 (3 cos Q 2)(cos Q 2)  0
2
 cos Q 
3 A
O
Answer: (C)

16. The line passing through the extremity A of the ma-


jor axis and extremity B of the minor axis of the
ellipse

x 2 y2
1
9 1 FIGURE 5.37
Answer: (D)
350 Chapter 5 Ellipse and Hyperbola

17. If P ( x, y) be any point on 16 x 2 25 y2  400 and (A) Q lies inside C but lies outside E
F1 (3, 0), F2 ( 3, 0) are the foci, then (B) Q lies outside both C and E
PF1 PF2  (C) P lies inside both C and E
(D) P lies inside C but outside E
(A) 8 (B) 6 (C) 10 (D) 12
Solution: We have
Solution: The given ellipse is
x 2 y2
x 2 y2 Sy 1
1 9 4
25 16
and S a y x 2 y2 9
In an ellipse, the sum of the focal distances of a point on
the ellipse is equal to the length of the major axis. Hence Also P  ( x1 , y1 )  (1, 2) and ( x2 , y2 )  (2, 1)  Q. Now,

PF1 PF2  2(5)  10 1 4 1


S11  1   0
Answer: (C) 9 4 9
2 2
a  1 2 9  5 0
S11
18. The radius of the circle passing through the foci of
Therefore, P lies outside E and inside C. Therefore
the ellipse
4 1
x 2 y2 S22  1  0
1 9 4
16 9
and a  4 1 9  0
S22
and having its centre at (0, 3) is
Hence, Q lies inside both E and C.
1 7 Answer: (D)
(A) 4 (B) 3 (C) (D)
2 2
(IIT-JEE 1995) 20. The locus of a variable point whose distance from (−2,
0) is 2 / 3 times its distance from the line x  9 / 2 is
Solution: Let the radius be r so that the circle equa-
tion is (A) ellipse (B) parabola
(C) hyperbola (D) pair of lines
x 2 ( y 3)2  r 2 (5.109)
Solution: By the definition of ellipse, the locus is an
The eccentricity of the ellipse is given by ellipse with focus (−2, 0), directrix x  9 / 2 and eccen-
9  16 (1 e 2 ) tricity  2 / 3  1.
9 7 Answer: (A)
 e2  1 
16 16 21. The eccentricity of the ellipse
7
 e x2 y2
4 1
a2 b2
Therefore
whose latus rectum is half of the major axis is
¥ 7´
ae  4 ¦ 1 1 1
µ 7 (A) 1 (B) (C) (D)
§ 4 ¶ 2 2 3
3
Hence, the foci are (p 7 , 0). Since the circle passes Solution: we have
through (ae, 0)  ( 7 , 0), we have
2b2
2
7 9r r  4 a
a
Answer: (A)  2b2  a 2
 a 2  2b2  2a 2 (1 e 2 )
19. Let E be the ellipse
1
x 2 y2  e2 
1 2
9 4 1
e=
and C be the circle x 2 y2  9. Let P and Q be the 2
points (1, 2) and (2, 1), respectively. Then Answer: (B)
Worked-Out Problems 351

22. The director circle of the ellipse, which is having its


x2 y2
axes along the coordinate axes, has foci at (±2, 0) and 2
1
eccentricity 1/ 2. Its equation is a b2
then the eccentric angle of the point of contact is
(A) x 2 y2  25 (B) x 2 y2  26
P P P P
(A) (B) (C) (D)
(C) x 2 y2  24 (D) x 2 y2  28 3 4 6 2
Solution: Let the ellipse be x y
Solution:  2 can be written as
x2 y2 a b
1
a2 b2 x y
1
a 2 b 2
where the eccentricity e  1/ 2.
Also x P y P
 cos sin  1
Focus  (2, 0)  ae  2  a  4 a 4 b 4
so that Q  P /4.
Again
Answer: (B)
¥ 1´
b2  a 2 (1 e 2 )  16 ¦ 1 µ  12
§ 4¶ 25. If S1 and S2 are the foci of an ellipse with major axis
Hence, the equation of the ellipse is of length 10 and P is any point on the ellipse such
that the perimeter of $ PS1S2 is 15, then the eccen-
x 2 y2 tricity of the ellipse is
1
16 12 1 1 7 3
(A) (B) (C) (D)
and the equation of the director circle is 2 4 25 4
x2 y2  a 2 b2  16 12  28 Solution: By hypothesis, we have
Answer: (D) S1S2  2ae  S1S2  10e (& 2a  10)

23. The points (5, 12) and (24, 7) are the foci of an ellipse Also
passing through the origin. Then the eccentricity of 15  PS1 PS2 S1S2  10 10e
the ellipse is
Therefore
386 286
(A) (B) 5 1
38 38 e 
10 2
286 386 Answer: (A)
(C) (D)
28 35
Solution: We have 26. If (h, k) is a variable point on the circle x 2 y2  1,
then the locus of the point (3h 2, k) is an ellipse
OS OS a  25 144 576 49  13 25  38  2a whose eccentricity is
Now 1 2 2 1 2
(A) (B) (C) (D)
SS a  2ae 3 3 3 3
 386  (38)e Solution: Let x  3h 2 and y  k so that
386 2
e ¥ x 2´ 2 2 2
38 ¦§ µ y h k 1
3 ¶
Answer: (A)
Hence, the locus of (3h 2, k) is an ellipse with centre
24. If (2, 0) and

x y 2 2
 2 1  a (1 e 2 )  e 
a b 3
touches the ellipse Answer: (B)
352 Chapter 5 Ellipse and Hyperbola

27. If P is a point on the ellipse


x2
2 2
y2  1
x y 4
1
25 16 This line touches the circle x2 y2  3. This means
lying in the first quadrant and PN is drawn perpen-
4 m2 1
dicular to major axis and produced to Q such that  3
NQ is equal to SP where S in a focus, then the focus 1 m2
of Q is
 3m 2 3  4 m 2 1
(A) 2x − 3y − 25  0 (B) 3x − 5y − 25  0
m p 2
(C) 3x − 5y 25  0 (D) 3x 5y 25  0
Therefore, the product of the slopes  −2.
Solution: See Fig. 5.38. Let P be (5 cos Q , 4 sin Q ) and
0 < Q  P / 2. Now, Answer: (B)

PN  4sinQ  N  (5cosQ , 0) 29. Two lines PQ and PR are drawn from a point on the
Let Q be (h, k). By hypothesis, NQ  SP. Also observe circle x2 y2  25 to the ellipse
that k  0. Now x2 y2
1
NQ  SP 42 b2
3 where b  4. If the parallelogram PQSR is completed
 k  a ex  5 (5 cos Q ) (5.110)
5 and S lies on the circumcircle of $PQR, then the
eccentricity of ellipse is
Since the abscissa of Q is 5 cos P, we have h  5 cos P.
From Eq. (5.110), we get 7 5 5 7
(A) (B) (C) (D)
k  5 3cos Q 3 4 3 4
¥ h´ Solution: Since the points P, Q, S and R concyclic and
 k  3¦ µ 5 PQSR is a parallelogram, the angles of the parallelogram
§ 5¶
are equal to 90o. Hence, P lies on the director circle of
 3h 5k 25  0
x 2 y2
Therefore, the locus of Q is 3x 5y 25  0. 1
4 b2
P
which is given by x2 y2  25. Hence, b  3. Therefore
9  16(1 e 2 )
9 7
 e  1 
N S 16 4
Answer: (D)

30. Point O is the centre of an ellipse with major axis


Q
AB and minor axis CD. Point F is one of the foci.
If OF  6 and the diameter of the inscribed circle of
FIGURE 5.38 $OCF is 2, then (AB)(CD) 
Answer: (D) (A) 35 (B) 45 (C) 55 (D) 65
28. The product of the slopes of the common tangents of Solution: See Fig. 5.39. Let OC  b, OF  6 and CF  a.
the ellipse Since
1  Inradius of $OCF
x 2 y2
1 (1/ 2)(b)(6)
4 1 
(1/ 2)(b 6 a)
and the circle x2 y2  3 is
we have
(A) 1 (B) −2 (C) −3 (D) −1
b 6 a  6b
Solution: The line y  mx 4 m2 1 is a tangent to
the ellipse  5b  6 a (5.111)
Worked-Out Problems 353

Also  a2 e2  a2 a2 e2
36  a2e2  a2 − b2 (5.112) 1
 e=
From Eqs. (5.111) and (5.112), we have 2
(5b − 6)2  a2  36 b2 Therefore, from Eq. (5.114), we get a  b 2. Hence from
 24b2 − 60b  0 Eq. (5.113), we get
60 5 2b2  10a  10(b 2 )
 b 
24 2
 b  5 2 and a = b 2  10
and hence
So, the equation of the ellipse is
25 13
a  5b 6  6  x 2 y2
2 2 1
100 50
Thus,
 x2 2y2  100
(AB)(CD)  (2a)(2b)  13 s 5  65
Answer: (A)
C
32. The centre of an ellipse is the origin and the x-axis is
the major axis. If it passes through the points (−3, 1)
b a
and (2, 2), then the eccentricity of the ellipse is
1
B A
2 3 2 3
O 6 F
(A) (B) (C) (D)
3 4 5 2
Solution: Let the ellipse be

x2 y2
D 1
a2 b2
FIGURE 5.39
This passes through (−3, 1) and (2, −2). So
Answer: (D)
9 1
1 (5.115)
31. An ellipse is having its axes along the x-axis and a 2
b2
y-axis and its latus rectum is of length 10 units. If the
4 4
distance between the foci is equal to the length of the and 1 (5.116)
semi-minor axis, then the equation of the ellipse is a2 b2

(A) x 2 2 y2  100 (B) 2 x 2 y2  100 Solving Eqs. (5.115) and (5.116) for a2 and b2, we get

(C) x 2 2 y2  200 (D) 2 x 2 y2  200 32 32


a2  and b2 
3 5
Solution: Let the ellipse be
Therefore
x2 y2
1 2
a2 b2 b2  a 2 (1 e 2 )  e 
5
so that
Answer: (C)
2b2 33. S and Sa are the foci of the ellipse
 10
a
x2 y2
 2b2  10a (5.113) 1
a2 b2
Also
and B is one end of the minor axis. If $BSSa is equi-
2ae  2b  b  ae (5.114) lateral triangle, then the eccentricity of the ellipse is
equal to
Now, from Eq. (5.114), we have
3 1 1 1
b2  a2(1 e2) (A) (B) (C) (D)
2 3 2 2
354 Chapter 5 Ellipse and Hyperbola

Solution: We have Solution: See Fig. 5.40. Clearly, if R(h, k) is the inter-
section of tangents at P and Q, then R lies on the director
SSa  SB  SaB
circle and hence
 SB SaB  2SSa
h2 k2  c2 d2 (5.117)
 2a  2(2ae)
1 Since PQ is the chord of contract of R, its equation is
e
2 hx ky
2
1 (5.118)
Answer: (C) c d2
However, the line provided in Eq. (5.118) touches the
34. If the eccentricity of an ellipse
ellipse
x2 y2 x2 y2
2
2
1 1
L 1 L 2 a 2
b2
is 1/ 6 , then its latus rectum is Therefore
2 2
8 10 5 7 ¥ d2 ´ 2
(A) (B) (C) (D) 2 ¥ d h´ 2
6 6 6 6 ¦ k µ a ¦ 2 µ b
§ ¶ § c k ¶
Solution: We have a2 d 4 h2
 d4  b2 k 2
2 ¥ 1´
2 c4
L 1  (L 2) ¦ 1 µ
§ 6¶ a 2 h2 b2 k 2
 1 (5.119)
 6 (L 2 1)  5L 2 10 c4 d4
 L2  4 Equations (5.117) and (5.119) hold if h2  c2 and k2  d2.
Therefore
Hence, the ellipse is
a2 b2
x2
y 2 1
1 c2 d2
5 6
P R
and the latus rectum is 90°

2a 2 2(5) 10
 
b 6 6
Answer: (B)
Q

35. A tangent is drawn to the ellipse

x2 y2
1
a2 b2
to cut the ellipse
FIGURE 5.40
x2 y2 Answer: (A)
1
c2 d2
at P and Q. If the tangents drawn at P and Q to the 36. If a tangent of slope 2 of the ellipse
ellipse
x2 y2
1
x2 y2 a2 b2
1
c2 d2 is normal to the circle x2 y2 4x 1  0, then the
are at right angles, then maximum value of ab is equal to
(A) 2 (B) 1 (C) 4 (D) 3
a2 b2 c2 d2
(A) 1 (B) 1
c2 d2 a2 b2 Solution: y  2 x 4a 2 b2 is a tangent to the ellipse.
This passes through the centre (−2, 0) of the circle. Hence
(C) a 2 d 2  b2 c 2 (D) a 2 d 2 c 2 d 2  1
Worked-Out Problems 355

From Eqs. (5.120) and (5.121), we have


0  4 4a 2 b2
x2 − 17x 60  0  (x − 12)(x − 5)  0
 4a 2 b2  16
Therefore, x  12 or 5 so that y  5 or 17. Hence
Since AM r GM, we have
F1 F2  x 2 y2  12 2 52  13
4 a 2 b2
8 r 4a 2 b2  2ab
2
P
Equality holds when 2a  b. Hence, the maximum value
of ab is 4. 90°
x
Answer: (C)
y
37. An ellipse has OB as a semi-minor and F, F a are its
foci and the angle FBF a is a right angle. Thus, the ec-
centricity of the ellipse is F2 C F1

1 2 2 2 1
(A) (B) (C) (D)
2 3 3 2
Solution: We have B  (0, b), F  (ae, 0) and F a  (−ae,
0). Now
FBF a  90o
FIGURE 5.41
 (FF a)2  (FB)2 (F aB)2
Answer: (C)
 4 a 2 e 2  (a 2 e 2 b 2 ) (a 2 e 2 b 2 )  2 a 2 e 2 2 b 2
 2a 2 e 2  2b2 39. The line PQ is a focal cord of the ellipse

 a 2 e 2  a 2 (1 e 2 ) x 2 y2
1
 2e 2  1 25 16
1 If the focus S  (3, 0) and SP  2, then the length PQ
 e=
2 is equal to
Answer: (D) (A) 8 (B) 4 (C) 10 (D) 6
Solution: Major axis length is 2 s 5  10. S  (3, 0)
38. A circle has the same centre as the ellipse
and SP  2. Since the vertex A (see Fig. 5.42) is (5, 0),
x2 y2 it follows that
1
a2 b2 SA  2 and A  P
and passes through two foci F1 and F2. P is one of the Therefore, PQ is nothing but the major axis and hence
four points of their intersection. If the major axis is PQ  10.
of length 17 and the area of the $ PF1F2 is 30, then
Q
the distance between the foci is
(A) 11 (B) 15 (C) 13 (D) 12
Solution: See Fig. 5.41. Let PF1x and PF2  y. Then
F1F2  2ae  17e
A A
Now C S
2
1
F1 PF2  90o 30  Area of $ PF1 F2  PF1 – PF2
2 P
 60  PF1 – PF2  xy (5.120)
Also FIGURE 5.42

x y  PF1 PF2  2a  17 (5.121) Answer: (C)


356 Chapter 5 Ellipse and Hyperbola

40. Point S is one of the foci of the ellipse


AB  5, OB  16 4  20 , OA  1 4  5
x 2 y2
1 We have
25 16
(OA)2 (OB)2  25  (AB)2
With S as centre and r as radius, a circle is drawn
such that it touches the ellipse and completely lies Therefore
within the ellipse. Then, the value of the radius r is P
AOB 
(A) 4 (B) 3 (A) 3 (D) 2 2
Solution: The ellipse is y

x 2 y2
 1  a  5, b  4 A B
25 16
which implies that a  5 and b  4. Also
3 O x
16  25 (1 e 2 )  e 
5
Hence, S  (3, 0) (see Fig. 5.43). Also we know that the
vertex A  (5, 0). This implies that SA  2. Since the circle
is touching the ellipse and lies completely within the FIGURE 5.44
ellipse, it follows that its radius r must be equal to 2. Answer: (B)

42. If the latus rectum of the hyperbola

x2 y2
1
a2 b2
A (5, 0)
C S subtends a right angle at the centre, then the eccen-
tricity of the hyperbola is
5 1
(A) (B) 3 1
2
2 3 3 2
FIGURE 5.43 (C) (D)
2 2
Answer: (D)
Solution: The ends of a latus rectum are
41. The angle subtended by the common tangent of the
¥ b2 ´
two ellipses L a  ¦ ae,
§ a µ¶
( x 4)2 y2
1 ¥ b2 ´
25 4 and L  ¦ ae, µ
§ a¶
( x 1)2 y2
and 1
1 4 Since LLa subtends right angle at the centre, then
at the origin is ¥ b2 /a ´ ¥ b2 /a ´
1 ¦ µ¦ µ
P P P P § ae ¶ § ae ¶
(A) (B) (C) (D)
2 4 3 6 b4

Solution: See Fig. 5.44. B(4, 2) is one end of the minor a 2 (a 2 e 2 )
axis of the ellipse 2
¥ b2 ´ 1 (e 2 1)2
( x 4) 2
y 2  ¦ 2 µ 2 
1 §a ¶ e e2
25 4
Therefore
and (−1, 2) is one end of the major axis of the second
ellipse. Therefore, (e 2 1)2  e 2
Worked-Out Problems 357

Hence ae
 b (5.123)
4 2
e 3e 1  0 2
From Eq. (5.121), we have
3 5
 e2  (&e  1) a 2 (e 2 1)  4a
2
Hence 4
a 2
(5.124)
e 1
5 1
e From Eqs. (5.122) and (5.123), we have
2
Answer: (A) a2 e2
 b2  4a
4
43. The distance between two parallel tangents of the
16
hyperbola a 2 (5.125)
e
x 2 y2
1 From Eqs. (5.124) and (5.125), we have
9 49
4 16
is 2. Then their slopes are 2

e 1 e2
5 7 4 5  e 2  4e 2 4
(A) p (B) p (C) p (D) p
2 2 5 4 2
e
Solution: The two parallel tangents are 3
Answer: (A)
y  mx p 9 m2 49
Therefore, by hypothesis 45. Consider a branch of the hyperbola

2 9 m2 49 x 2 2 y2 2 2 x 4 2 y 6  0
2
1 m2 with A as one vertex. B is one of the end points of
the latus rectum and C is the focus nearest to A.
 9 m2 49  1 m2 Then, the area of $ABC is
 8 m2  50 2 3
(A) 1 (B) 1
5 3 2
m p
2 2 2
Answer: (A) (C) 1 (D) 1
3 3
44. A hyperbola has its axes along the coordinate axes, (IIT-JEE 2008)
latus rectum is 8 and conjugate axis is half of the dis- Solution: The equation of the given hyperbola can be
tance between the foci. Then the eccentricity is written as
2 3 4 4 ( x 2 )2 ( 2 y 2)2  6 2 4  4
(A) (B) (C) (D)
3 2 3 3
( x 2 )2 2( y 2 )2
 1
Solution: Hyperbola is 4 4
x2 y2 ( x 2 )2 ( y 2 )2
1  1
a2 b2 4 2
X2 Y2
By hypothesis, we have  1
4 2
2b2
8 where X  x 2 and Y  y 2. Now
a
 b2  4a (5.122) A  ( X  2, Y  0)  ( x 2  2, y 2  0)
and 2b  ae  (2 2 , 2 )
358 Chapter 5 Ellipse and Hyperbola

The centre is ( 2 , 2 ). Let e be the eccentricity. There- Hence, the foci are (p ae, 0)  (p 1, 0). Now, let the hyper-
fore bola be

3 x2 y2
2  4(e 2 1)  e  1
2 a2 b2
Hence, so that a  sin P and the eccentricity ea is given by
C  (X  ae, Y  0) b2  a 2 (e a 2 1)  sin 2 Q (e a 2 1) (5.126)
 C  ( 2 6, 2) Also
Also aea  1  ea  cosec P (& a  sin P)
b2 2 Therefore, from Eq. (5.126),
BC   1
a 2 b2  sin2P (cosec2 P − 1)  1 − sin2 P  cos2 P
and
Therefore, the equation of the hyperbola is
¥ b2 ´
B  ¦ X  ae, Y  µ  ( 2 2 6 , 1 2 )
§ a¶ x2 y2
1
sin 2 Q cos2 Q
 (2 2 6 , 1 2 )
Answer: (A)
Therefore, the area of ΔABC is
47. If the line 2 x 6 y  2 touches the hyperbola
2 2 2 1 x2 2 y2  4, then the point of contact is
1
2 2 6 1 2 1
2 (A) ( 2, 2 ) (B) ( 5, 2 6 )
2 6 2 1
¥1 1 ´
(C) ¦ , (D) (4, 6 )
2 2 2 1 § 2 6 µ¶
1
 2 2 6 1 0 (by R 2 R 1 , R 3 R 1 ) Solution: Suppose the line 2 x 6 y  2 touches the
2
6 2 0 0 curve at (x1, y1). However, at (x1, y1), the equation of the
tangent is
1
 6 2 xx1 2 yy1  4
2
3 Hence
 1
2 x1 2 y1 4
 
Answer: (B) 2 6 2
 x1  4, y1  6
46. A hyperbola having transverse axis of length 2 sin P
is confocal with the ellipse 3 x 2 4 y2  12.Then its Thus, the point of contact is (4, 6 ).
equation is Answer: (D)
2 2 2 2
(A) x cosec Q y sec Q  1
(B) x 2 sec 2 Q y2 cosec 2 Q  1 48. For the hyperbola
2 2 2 2
(C) x sin Q y cos Q  1 x2 y2
2 2 2 2 1
(D) x cos Q y sin Q  1 cos A2
sin 2 A
(IIT-JEE 2007) where the value of @ changes, which of the following
Solution: The equation of the ellipse is remain constant?

x 2 y2 (A) Abscissae of the vertices


1
4 3 (B) Abscissae of the foci
and its eccentricity is given by (C) Eccentricity
(D) Directrices
3 1
 1 e2  e  (IIT-JEE 2004)
4 2
Worked-Out Problems 359

Solution: The eccentricity e is given by Solution: The normal at P(a sec @, b tan @) is
sin 2 A  cos2 A (e 2 1) ax cos @ by cot @  a2 b2 (5.127)
and the normal at
 e  tan 2 A 1  sec A
¨ ¥P ´ ¥P ´·
Therefore, the foci are (p cos A sec A , 0)  (p 1, 0). Q  ©a sec ¦ A µ , b tan ¦ A µ ¸  (a cosec A , b cot A )
ª § 2 ¶ § 2 ¶¹
Answer: (B)
¥ P´
49. If x  9 is a chord of contact of the hyperbola ¦§& A B  µ¶
2
x2 y2  9, then the equation of the corresponding
is
pair of tangents is
ax sin @ by tan @  a2 b2 (5.128)
(A) 9 x 2 8 y2 18 x 9  0
Now,
(B) 9 x 2 8 y2 18 x 9  0
{sin @ s [Eq. (5.127)]} − {cos @ s [Eq. (5.128)]}
(C) 9 x 2 8 y2 18 x 9  0
 by (cos @ − sin @)  (a2 b2)(sin @ − cos @)
(D) 9 x 2 8 y2 18 x 9  0
(a 2 b 2 )
(IIT-JEE 1999) y
b
Solution: Substitute x  9 in the curve equation
Therefore,
x 2 y2  9
(a 2 b 2 )
so that y2 72 or y  p 6 2. Hence, the extremities of k
b
the chord of contact are (9, p 6 2 ). The tangent at Answer: (D)
(9, 6 2 ) is
9x 6 2y 9  0 51. The distance between the foci of a hyperbola is 16
and its eccentricity is 2. Then, the equation of the
and the tangent at (9, 6 2 ) is hyperbola is
9x 6 2y 9  0 (A) x 2 y2  16 (B) x 2 y2  8
Hence the combined equation of the tangents drawn at (C) x 2 y2  32 (D) x 2 y2  24
the ends of the chord of contact is Solution: Since 2 is the eccentricity, the curve is a
(9 x 6 2 y 9)(9 x 6 2 y 9)  0 rectangular hyperbola, say, x2 − y2  a2.
Again 2ae  16, and e  2  a  4 2. Hence the
 81x 2 72 y2 9(9 x 6 2 9 x 6 2 y) 81  0 equation of the hyperbola is
 81x 2 72 y2 9(18 x) 81  0
x2−y2  32
2 2
 9 x 8 y 18 x 9  0 Answer: (C)
Answer: (D)
52. The locus of the point of intersection of two lines
50. Let P(a sec @, b tan @) and Q(a sec A, a tan A), where
3 x y 4 3k  0 and k 3 x ky 4 3  0
A B  P / 2, be two points on the hyperbola
is a hyperbola whose eccentricity is
x2 y2
1 1 3
a2 b2 (A) 2 (B) 2 (C) (D)
2 4
If (h, k) is the point of intersection of the normals at
P and Q, then k is equal to Solution: We have

a 2 b2 ¥ a 2 b2 ´ 3x y  4 3 k (5.129)
(A) (B) ¦ µ
a § a ¶ k 3 x ky  4 3 (5.130)
From Eq. (5.130), we have
a 2 b2 ¥ a 2 b2 ´
(C) (D) ¦ µ
b § b ¶ 4 3
k
(IIT-JEE 1999) 3x y
360 Chapter 5 Ellipse and Hyperbola

Substituting the value of k in Eq. (5.129), we have


and a 2 m2 b2
¥ 4 3 ´
3x y  4 3 ¦ Hence, the area of the triangle is
µ
§ 3x y ¶
3x2 − y2  48 1 a 2 m2 b2 1
a 2 m2 b2  (a 2 m 2 b 2 )
2 m 2m
x 2 y2
 1
16 48 Answer: (D)
Now,
55. In an ellipse, the distance between the foci is 6 and
48  16(e 2 1) minor axis is of length 8. Then, the eccentricity is
48 3 1 4 1
e 1  2 (A) (B) (C) (D)
16 5 2 5 5
Answer: (A)
Solution: We have 2ae  6 and b  4. Therefore
53. The equations of the pair of straight lines parallel to 16  b2  a 2 (1 e 2 )  a 2 a 2 e 2  a 2 9
the x-axis and tangent to the ellipse
a5
x 2 y2
1 Therefore
4 9
are 3
e (& ae  3)
5
(A) y  3, y  6 (B) y  3, y  6
Answer: (A)
(C) y  p6 (D) y  p 3
Solution: Let y  k be a line parallel to x-axis. The 56. The focus of an ellipse is at the origin. The directrix
points of intersections of the line with the ellipse are is the line x  4 and the eccentricity is 1/2. Then the
given by the equation length of the semi-major axis is

9x2 4k2  36 8 2 4 5
(A) (B) (C) (D)
3 3 3 3
which touches the ellipse. Solving we get
Solution: The directrix corresponding to the focus is
4k 2  36 or k  ±3 x  4. Therefore, the perpendicular distance of the di-
Therefore, the tangents are y  ±3. rectrix from the focus is
Answer: (D) a
ae  4
e
54. The area of the triangle formed by a tangent of slope a
m to the ellipse  2a  4
2
x2 y2 8
1 a
2
a b2 3
and the two axes is Answer: (A)
m 2 1
(A) (a b 2 ) (B) (a 2 b 2 ) 57. PQ is a double ordinate of
2 2m
x2 y2
m 2 2 1 1
(C) (a m b 2 ) (D) (a 2 m 2 b 2 ) a2 b2
2 2m
such that $OPQ is an equilateral triangle. If e is the
Solution: The tangent with slope m is of the form
eccentricity of the hyperbola, then
y  mx a 2 m2 b2 2 2
(A) e  (B) e 
Therefore, the intercepts of the tangent on the axes are 3 3

a 2 m2 b2 2 4
(C) 1  e  (D) e 
m 3 3
Worked-Out Problems 361

Solution: See Fig. 5.45. Let P  (a sec P, b tan P ) and 2


x 2 sec 2 Q y  25
Q  (a sec P, −b tan P ). Now,
then one of the value of P is
b tan Q P P P P
tan 30o  (A) (B) (C) (D)
a sec Q 2 4 3 6
1 b
  sin Q Solution: The hyperbola is
3 a
3b2 x2 y2
  cosec Q  1 2 1
5 5 cos2 Q
a2
 3(e 2 1)  1 and the ellipse is
1 4 x2 y2
 e2  1  1
3 3 25 cos Q 2
25
2
e Let e1 and e2 be the eccentricities of the hyperbola and
3
the ellipse. Therefore
P
5 cos2 Q  5 (e12 1)
 e12  1 cos2 Q
Also
30°
O
x 25 cos2 Q  25(1 e22 )
 e22  1 cos2 Q  sin 2 Q
By hypothesis,
Q

FIGURE 5.45 e1  3e2


Answer: (B)  e12  3e22  1 cos2 Q  3 sin 2 Q

 1 1 sin 2 Q  3 sin 2 Q
58. If ax by  1 is a tangent to the hyperbola
1 P
x2 y2  sin 2 Q  or Q 
1 2 4
a2 b2
then a2 − b2  Answer: (B)
1
(A) b2 e2 (B) 60. The distance between the tangents to the hyperbola
b2 e 2
4x2 − 9y2  1 parallel to the line 5x − 4y 7  0 is
1
(C) a2 e2 (D) 1 161
a2 e2 1 161
(A) 3 41 (B)
2 41
Solution: We know that
1 151 1 151
¥ a ´ 1 (C) (D)
y ¦ µ x 3 41 2 41
§ b¶ b
Solution: The given hyperbola is
is a tangent which implies that
x2 y2
2 4 1
1 ¥ a ´ a 1/ 4 1/ 9
 a 2 ¦ µ b2  2 b2
b 2 § b ¶ b
and the line is
 a 4 b4  1
1 1 1 ¥ 5´ 7
y ¦ µ x
 (a 2 b 2 )    § 4¶ 4
a 2 b2 a 2 a 2 (ee 2 1) a2 e2
Answer: (D) Any line parallel to this line is
¥ 5´
59. If the eccentricity of the hyperbola x2 − y2 sec2P  5 is y ¦ µ x c
§ 4¶
3 times the eccentricity of the ellipse
362 Chapter 5 Ellipse and Hyperbola

This will touch the hyperbola which implies that Hence, the tangents are
2
¥ 1´ ¥ 5´ 1 25 1 5x 161
c 2  a 2 m 2 b2  ¦ µ ¦ µ  y p
§ 4¶ § 4¶ 9 64 9 4 24

Therefore  30 x 24 y p 161  0
Hence, the distance between these tangents is
161
cp
24 2 161 161

2 2 3 41
30 24
Answer: (A)

Multiple Correct Choice Type Questions


1. For the hyperbola 9 x 2 16 y2 18 x 32 y 151  0, 2. If the circle x2 y2  a2 cuts the hyperbola xy  c2 at
which of the following are true? four points (xk, yk) (where k  1, 2, 3 and 4), then
5 (A) x1 x2 x3 x4  0 (B) y1 y2 y3 y4  0
(A) Eccentricity is
4 (C) x1 x2 x3 x4  c4 (D) y1 y2 y3 y4  c4
(B) Foci are (−4, 1) and (6, 1)
Solution: The abscissa xk (where k  1, 2, 3 and 4) are
(C) Centre is (1, −1) the roots of the equation
9
(D) Length of the latus rectum is
2 c4
x2  a2
Solution: The given equation is x2
 x 4 a2 x 2 c4  0
9( x 2 2 x) 16( y2 2 y) 151  0
Therefore
 9( x 1)2 16( y 1)2  151 9 16  144
x1 x2 x3 x4  0
( x 1)2 ( y 1)2
 1 Since the coefficient of x3 is zero, we have
16 9
¤ x1 x2  a 2 , ¤ x1 x2 x3  0, ¤ x1 x2 x3 x4  c 4
X2 Y2
 1
16 9 Now,

where X  x 1, Y  y 1. Here a 2  16, b2  9. The ¥ 1 1 1 1´


eccentricity e is given by y1 y2 y3 y4  c 2 ¦ µ
§ x1 x2 x3 x4 ¶
9 25 c 2 ( ¤ x2 x3 x4 )
9  16(e 2 1) or e 2  1  
16 16 x1 x2 x3 x4
so that c 2 (0)
 0
5 c4
e
4 Finally
The centre is given by c 2 c 2 c 2 c 2 c8
y1 y2 y3 y4  – – –   c4
( X  0, Y  0)  ( x 1  0, y 1  0)  (1, 1) x1 x2 x3 x4 c 4
The foci is given by Answers: (A), (B), (C), (D)
( X  p ae, Y  0)  ( x 1  p 5, 1)  (6, 1) and ( 4, 1) 3. On the ellipse 4x2 9y2  1, the points at which the
The latus rectum is given by tangents are parallel to the line 9y  8x are
¥2 1´ 2 1
2b2 2(9) 9
  (A) ¦ , µ (B) ¥¦ , ´µ
§5 5¶ § 5 5¶
a 4 2
Answers: (A), (C), (D)
Worked-Out Problems 363

¥ 2 1´ ¥ 2 1´ Hence, the eccentricity of the hyperbola is 5 / 3. Let


(C) ¦ , µ (D) ¦ , µ
§ 5 5¶ §5 5 ¶ x2 y2
1
Solution: The ellipse is A2 B2

x2 y2 be the hyperbola. Now,


1
(1/ 4) (1/ 9) 2
¥ 25 ´ 16A
B 2  A 2 ¦ 1µ 
§ 9 ¶ 9
The tangent at (x1, y1) is 4xx1 9yy1  1 which is parallel
to 9y  8x. This implies This implies that the equation is
4 x1 8 x2 9 y2
 1
9 y1 9 A2 16A 2
 x1  2 y1 (5.131) Also the hyperbola passes through the focus (3, 0) of the
Since (x1, y1) lies on the ellipse, we have ellipse. This implies that
9
4 x12 9 y12  1 (5.132) 1  A2  9
2
A
Substituting x1  −2y1 in Eq. (5.132), we get
16A 2
1 Also B2   B 2  16
16 y12 9 y12  1 or y1  p 9
5
Hence, the equation of the hyperbola is
2
and x1  @ x 2 y2
5 1
9 16
Therefore, the points are
One vertex is (3, 0) and the focus is
¥ 2 1´ ¥ 2 1´
¦§ , µ¶ or ¦§ , µ¶ ¥ ¥ 5´ ´
5 5 5 5 (p ae, 0)  ¦ p3 ¦ µ , 0µ  (p 5, 0)
§ § 3¶ ¶
Answers: (B), (D)
Answers: (A), (C)
4. Let a hyperbola pass through the focus of the ellipse
5. Suppose the hyperbola
x 2 y2
1 x2 y2
25 16 1
a2 b2
The transeverse and conjugate axes of this hyper-
bola coincide with the major and minor axes of the is confocal with the ellipse
ellipse. Also the product of the eccentricities of the
x 2 y2
ellipse and hyperbola is 1. Then 1
25 9
x 2 y2
(A) the equation of the hyperbola is 1 and has eccentricity is 2. Then
9 16
x 2 y2 (A) a2 b2  16
(B) the equation of the hyperbola is 1 (B) the hyperbola has no director circle
9 25
(C) the focus of the hyperbola is (5, 0) (C) the length of the latus rectum is 12
(D) a2 − b2  16
(D) the vertex of hyperbola is (5 3 , 0)
(IIT-JEE 2006) Solution: For the ellipse a2  25, b2  9 and the eccen-
tricity is given by
Solution: The eccentricity e of the ellipse is given by
4
16  25 (1 − e2) 9  25(1 − e2)  e 
5
which gives that Therefore, the foci of the ellipse are
3 ¥ ¥ 4´ ´
e (p ae, 0)  ¦ p 5 ¦ µ , 0µ  (p4, 0)
5 § § 5¶ ¶
364 Chapter 5 Ellipse and Hyperbola

Now, for the hyperbola, the foci is (p ae, 0). So


b4 cos2 Q
 b2 sin 2 Q
(p 4, 0)  (p 2a, 0) (& e  2) a2
 a2 b2
 (b2 cos2 Q a 2 sin 2 Q ) (5.133)
Therefore a2
b2  a 2 (e 2 1)  4(4 1)  12 We have PF  CT  perpendicular distance of C from the
tangent at P which is given by
Hence, the hyperbola is
1 ab
x 2 y2  (5.134)
1 (cos2 Q )/a 2 (sin 2 Q )/b2 b2 cos2 Q a 2 sin 2 Q
4 12
so that (A) We have
a 2 b2  4 12  16 PF – PG 
b 2
b cos2 Q a 2 sin 2 Q s
a
Director circle: x 2 y2  a 2 b2  8 which is not pos-
ab
sible. Hence, there is no director circle to the hyperbola.  b2
2 2 2 2
The length of the latus rectum is b cos Q a sin Q
2b2 2(12) Also
  12
a 2 2
Answers: (A), (B), (C) 2 2¥ 2 (b 2 a 2 ) ´
Pg  a cos Q ¦ b sin Q sin Q µ
§ b ¶
6. Suppose the normal at any point P on the ellipse
a 4 sin 2 Q
2 2  a 2 cos2 Q
x

y
1 b2
2 2 2
a b a
 (b2 cos2 Q a 2 sin 2 Q )
meets the major axis at G and the minor axis at g. CF b2
is drawn perpendicular to the normal where C is the Therefore
centre of the ellipse. Then
a 2
(A) PF·PG  b2 Pg  b cos2 Q a 2 sin 2 Q
b
(B) PF·Pg  a2
(B) We have
(C) PG·Pg  SP·SaP
(D) CG·CT  CS2 ab
PF – Pg  –
where S is the focus b cos Q a 2 sin 2 Q
2 2

a 2
Solution: See Fig. 5.46. Let P be (a cos Q , b sin Q ). The b cos2 Q a 2 sin 2 Q  a 2
normal at P is b

ax by (C) We have
 a 2 b2
cos Q sin Q b 2
PG – Pg  b cos2 Q a 2 sin 2 Q s
Therefore a
a 2
¥ (a 2 b 2 ´ b cos2 Q a 2 sin 2 Q
G¦ cos Q , 0µ  (ae 2 cos Q , 0) b
§ a ¶  b2 cos2 Q a 2 sin 2 Q
¥ ¥ b2 a 2 ´ ´ Also
and g  ¦ 0, ¦ µ sin Q µ
§ § b ¶ ¶ ¥ a´ ¥ a ´
SP – Sa P  e 2 ¦ a cos Q µ ¦ a cos Q µ
§ e¶ § e ¶
Now,
2
 a 2 (e cos Q 1)(1 e cos Q )
¥ ¥a b2 ´ 2´
PG 2  ¦ a cos Q ¦ cosµ (b sin Q 0)2  a 2 [1 e 2 cos2 Q ]
µ
§ § a ¶ ¶
 a 2 (a 2 b2 )cos2 Q
 a 2 sin 2 Q b2 cos2 Q
Worked-Out Problems 365

Therefore where
PG – Pg  SP – Sa P ¥ dy ´
m¦ µ
(D) We have § dx ¶ ( x , y )
1 1

¥ a 2 b2 ´ ab Now, y  0  x  x1 my1 so that by hypothesis, we have


CG – CT  ¦ µ cos Q – 2
§ a ¶ b cos Q a 2 sin 2 Q
2 x1 my1  2 x1
2 2
b(a b )cos Q
  (CS)2 Therefore, x1 my1  p 2 x1 .
2 2 2 2
b cos Q a sin Q Case 1:
n1 my1  2 x1
T
 my1  x1
P dy x
 
dx y
 xdx − ydy  0
C
Therefore, the solution is
G
F
x 2 y2
c
2 2
 x2 − y2  2c
which is a hyperbola.
Case 2:
g x1 my1  2 x1
FIGURE 5.46 dy
y 3x  0
Answers: (A), (B), (C), (D) dx
 ydy (3 x)dx  0
7. The normal to a curve at P(x, y) meets the x-axis at
G. If the distance of G from the origin is twice the Therefore
abscissa of P, then the curve is a y2 3 x 2
c
(A) circle (B) hyperbola 2 2
(C) ellipse (D) parabola
x2 y2
Solution: The normal at P(x1, y1) is  1
(2/3)c 2c
1 which is an ellipse.
y y1  ( x x1 )
m Answer: (B), (C)

Matrix-Match Type Questions


1. Consider the ellipse
Column I Column II
x 2 y2 (A) PF·PG is equal to
1 (p) 5 9 sin 2 Q
14 5
(B) PF·Pg is equal to (q) 5
The normal at P meets the major axis at G and the (r) 9 5 cos2 Q
minor axis at g. CF is drawn perpendicular to the (C) PG·Pg is equal to
(s) 9
normal at P. Match the items of Column I with those
(D) CG·CT is equal to (t) 16
of Column II.
Solution: According to Problem 6 of previous section
(Multiple Correct Choice Type Questions), we have
366 Chapter 5 Ellipse and Hyperbola

PF·PG  b2 so that
PF·Pg  a2 S  (ae, 0)  (4, 0) and Sa  ( 4, 0)
PG·Pg  SP·SaP Now,
and CG·CT  (CS)2.
12 0 15 2 15 2 12
SM  
Here, a2  14, b2  5, eccentricity e  3 / 14 and the foci
32 52 34
are (p3, 0).
15 2 12
Answer: (A) j (q); (B) j (t); (C) j (p); (D) j (s) Sa. 
34
2. Consider the ellipse
Hence
2 2
x y 306
1 SM – Sa N  9
25 9 34
with centre C and P is a point on the ellipse with eccen- Also note that SM·SaN is always equal to b2. In Problem
tric angle 45o. The normal drawn at P meets the major 6 of previous section (Multiple Correct Choice Type
and the minor axes at G and g, respectively. F is the Questions), we have proved that PF·Pg  a2 so that
foot of the perpendicular drawn from the centre C
onto the normal at P. The tangent at P meets the major PF·Pg  a2  25
axis at T. M and N are the feet of the perpendiculars Now,
drawn from the foci S and Sa onto the tangent at P.
Match the items of Column I with those of Column II. 8 2
CG·CT  – 5 2  16
5
Column I Column II
Finally,
(A) CG·CT is equal to (p) 9
4 ¨¥ 25 ´ 5 · 4 ¨ 25 5 ·
(q) 16 SP – Sa P  ¦ µ ¸– ©
(B) PF·Pg is equal to 5 ©ª§ 4 ¶ 2¹ 5ª 4
¸

(r) 17
(C) SM·SaN is equal to
(s) 15 16 ¨ 252 25 ·
 © ¸
(D) SP·SaP is equal to 25 ª 16 2¹
(t) 25
¨ 25 1 ·
 16 © ¸
Solution: See Fig. 5.47. We have ª 16 2 ¹
¥ 5 3 ´ ¥ 17 ´
P  (a cos Q , b sin Q )  ¦ ,  16 ¦ µ  17
§ 2 2 µ¶ § 16 ¶

The tangent at P is y

x y M
1 90
5 2 3 2
 3 x 5 y  15 2 (5.135) B
P
The normal at P is N

25[ x ( 5/ 2 )] 9[ y ( 3/ 2 )]

5/ 2 3/ 2 T A G A x
S C S
F 90
 5 2 x 3 2 y  25 9  16
 5 2 x 3 2 y  16 (5.136)
From Eq. (5.135), we get B

T  (5 2 , 0) (5.137) g
The eccentricity e is given by
FIGURE 5.47
4 Answer: (A) j(q); (B) j(t);(C) j(p);(D) j (r)
9  25(1 e 2 )  e 
5
Worked-Out Problems 367

3. Some ellipses are given in Column I and their eccen- (C) We have
tricities are given in Column II. Match the items of
3(x 1)2 4(y − 1)2  5 3 4  12
Column I with those of Column II.
( x 1)2 ( y 1)2
 1
Column I Column II 4 3
1 Now
(A) 9x2 25y2 − 18x − 100y − 116  0 (p)
2
3  4(1 e 2 )
3 3 1
(B) 9x2 4y2  36 (q)  e2  1 
5 4 4
(C) 3x2 4y2 6x − 8y − 5  0 2 1
(r) e
3 2
4 Answer: (C) j (p)
( x 1)2 ( y 2)2 (s)
(D) 1 5 (D) We have
9 4
5 4  9(1 e 2 )
(t)
3 4 5
 e2  1 
9 9
Solution: The eccentricity is given by the relation
5
b2  a2 (1 − e2) if a  b e
3
and a2  b2 (1 − e2) if a  b Answer: (D) j (t)
(A) We have
4. Match the items of Column I with those of Column II.
9(x − 1)2 25 (y − 2)2  116 9 100  225
Column I Column II
( x 1)2 ( y 2)2
 1 (A) The locus of the
25 9 (p) ( x 2 y2 )2  4 x 2 3 y2
point whose chord
Also of contact with
respect to the
9  25(1 e 2 ) hyperbola
9 16
 e2  1  x 2 y2
25 25 1
4 16 9
e touches the circle
5
described on the line
Answer: (A) j (s) joining the foci is
(B) We have (B) The chords of the (q) x2 y2  9
2 2
circle x y  4
x 2 y2 touch the hyperbola
1
4 9
x 2 y2
Now  1.
4 3
4  9(1 e 2 ) Then the locus of
4 5 the midpoints of
 e2  1  these chords is
9 9
(C) The director circle (r) x2 − y2  32
5 of the hyperbola
e
3
x 2 y2
Answer: (B) j (t) 1
25 16
is
(Continued)
368 Chapter 5 Ellipse and Hyperbola

Column I Column II x 2 y2
1
(D) The distance 4 3
x 2 y2 1
between the foci of (s) 
256 81 25 That is, the line xx1 yy1  x21 y21 touches the
a hyperbola is 16 hyperbola. This means
and its eccentricity
2 2
is 2. Then the ¥ x12 y12 ´ ¥ x1 ´
equations of the (t) x2 − y2  64 ¦ y µ  4 ¦§ y µ¶ 3
§ 1 ¶ 1
hyperbola is
 ( x12 y12 )2  4 x12 3 y12
Solution:
Therefore, the locus is
(A) P(x1, y1) be a point on the locus. That is
( x 2 y 2 )2  4 x 2 3 y 2
xx1 yy1
1 Answer: (B) j (p)
16 9
(C) Director circle of
touches the circle described on the line joining the
foci S(5, 0) and Sa(−5, 0) whose equation is x2 y2
1
2 2 a2 b2
x y  25
is x2 y2 a2 − b2. Hence, a2  25 and b2  16. Hence,
0 0 1 the director circle is x2 y2  9.
 5
x12 /16 2 y12 /9 2 Answer: (C) j (q)
x12 y12 1 (D) Since 2 is the eccentricity of the hyperbola, it
 2
2
 must be a rectangular hyperbola. Hence, it is of the
16 9 25
form x2 − y2  a2. By hypothesis,
Therefore, the locus is
2ae  16  2a( 2 )  16  a  4 2
x2 1 y2
2
2  Hence, the hyperbola is x2 − y2  32.
16 9 25
Answer: (D) j (r)
Answer: (A) j (s)
(B) (x1, y1) is the midpoint of a chord x2 y2  4 touch-
ing the hyperbola

Comprehension Type Questions


1. Passage: Tangents are drawn from the point P(3, 4) to (ii) The orthocentre of $PAB is
the ellipse
¥ 8´ ¥ 7 25 ´
(A) ¦ 5, µ (B) ¦ , µ
x2
y 2 § 7¶ §5 8 ¶
1
9 4 ¥ 11 8 ´ ¥ 8 7´
(C) ¦ , µ (D) ¦ , µ
touching the ellipse at A and B. Answer the following § 5 5¶ § 25 5 ¶
questions.
(iii) The equation of the locus of the point, whose
(i) The coordinates of A and B are distance from the point P and the line AB are
¥ 8 2 161 ´ equal, is
¥ 9 8 ´
(A) ¦ , µ and ¦ , µ
§ 5 15 ¶ § 5 5¶ (A) 9x2 y2 − 6xy − 54x − 62y 241  0
(B) x2 9y2 6xy − 54x − 62y − 241  0
¥ 8 2 161 ´
(B) ¦ , and (0, 2) (C) 9x2 9y2 − 6xy − 54x − 62y − 241  0
§ 5 15 µ¶
(D) x2 y2 − 2xy 27x 31y − 120  0
(C) (3, 0 ) and (0, 2)
(IIT-JEE 2010)
¥ 9 8 ´
(D) (3, 0) and ¦ , µ
§ 5 5¶
Worked-Out Problems 369

Solution: 3 4
2
cos Q  and sin Q 
(i) y  mx p 9 m 4 is a tangent to the ellipse. This 5 5
passes through (3, 4). So the corresponding point does not lie on the tangent
at P.
(4 − 3m)2 4 9m2
Answer: (D)
16 − 24m − 9m  9m2 4
2

1 (ii) The vertices are P(3, 4), A(3, 0) and B( 9 / 5, 8 / 5).


m Since AP in a vertical line, the altitude through B is
2
y  8 / 5. As orthocentre lies on the line y  8 / 5, the
Therefore, the tangent from (3, 4) is orthocentre of $PAB is (11/ 5, 8 / 5).
Answer: (C)
x 9
y 4
2 4 (iii) The equation of the line AB is
 x 2y  5 1
y ( x 3)
Suppose x − 2y 5  0 touches at (x1, y1). Hence 3
x 2y 5  0  x 3y − 3  0
xx1 yy1 Hence, the required locus is the parabola
and 1
9 4 2
¥ x 3y 3´
represent the same straight line. Therefore (x − 3)2 (y − 4)2  ¦ µ
§ 10 ¶
( x1 /9) ( y1 /4) 1
   10( x 2 y2 6 x 8 y 25)
1 2 5
 x 2 9 y2 6 xy 6 x 18 y 9
9 8
 x1  , y1   9 x 2 y2 6 xy 54 x 62 y 241  0
5 5
The point of contact is ( 9 / 5, 8 / 5). Also the line x  Answer: (A)
3 is a tangent at the vertex A(3, 0). Therefore, the
2. Passage: Tangents are drawn to the hyperbola x2 −
points of contact are ( 9 / 5, 8 / 5) and (3, 0).
9y2  9 from (3, 2). Answer the following questions.
(i) The area of the triangle formed by the tangents
Aliter: The tangent at (3 cos P, 2 sin P ) is
and the chord contact of (3, 2) is
x cos Q y sin Q (A) 6 (B) 8 (C) 10 (D) 12
1
3 2
(ii) The area of the triangle formed by the tangent to
This passes through (3, 4) which implies the hyperbola at (3, 0) and the two asymptotes is
cos Q 2 sin Q  1 (A) 3 (B) 6 (C) 9 (D) 2
2 2
 (1 cos Q )  4 sin Q (iii) The midpoint of the intercept of the tangent at
2
 1 2 cos Q cos Q  4(1 cos Q )2 (3, 0) between the asymptotes is
 5 cos2 Q 2 cos Q 3  0 (A) (1, 0) (B) (2, 0) (C) (3, 1) (D) (3, 0)
2 Solution:
 5 cos Q 5 cos Q 3 cos Q 3  0
 (5 cos Q 3)(cos Q 1)  0 (i) y  mx 9 m2 1 is a tangent to the hyperbola
 cos Q  1, 3/5
x 2 y2
When 1
9 1
cos Q  1  sin Q  0 This passes through the point (3, 2). This implies
3 4 (3m 2)2  9 m2 1
and cos Q   sin Q  p
5 5
 12 m  5
Therefore, the points of contact are (3, 0) and 5
m
( 9 / 5, 8 / 5). Note that for 12
370 Chapter 5 Ellipse and Hyperbola

Therefore, one tangent is 5x − 12y 9  0. Also the


(C) 2 11 (D) 4 11
tangent at the vertex (3, 0) passes through (3, 2).
Hence, the other tangent through (3, 2) is x  3. The (ii) The directrix of the hyperbola corresponding to
chord of contact is the focus (5, 6) is
3x 2 y (A) 2 x 2 y 11  0
1
9 1 (B) 2 x 2 y 1  0
 x − 6y  3 (C) 2 x 2 y 9  0
Therefore, the sides of the triangle are (D) 2 x 2 y 7  0

5x − 12y 9  0 (iii) Length of the latus rectum of the hyperbola


x  3 and x − 6y  3 44 22 32 42
(A) (B) (C) (D)
Solving these equations, the vertices of the triangle 10 10 10 10
are (3, 2), (3, 0) and ( 5, 4 / 3). Hence, the area of Solution:
the triangle is
(i) The centre of the hyperbola is

3 2 1 ¥ 3 5 6 2 ´
¦§ , µ  (1, 2)
1 1 ¥ 4´ 2 2 ¶
3 0 1  3 ¦ 0 µ 2(3 5) 1( 4 0)
2 2 § 3¶ Since (2, 5) lies on the auxiliary circle, its radius is
4
5 1
3 a  (1 2)2 (2 5)2  10
1 The distance between the foci is 2ae. From the coor-
 4 16 4  8
2 dinates of the foci, the distance between them is
Answer: (B)
(5 3)2 (6 2)2  128  8 2
(ii) The area of the triangle formed by two asymptotes
Now
and a tangent to the hyperbola
2 10e  8 2
x2 y2
1 4
a2 b2 e
5
is always constant which is equal to ab (see
We have
Problem 5 of the section ‘Subjective Problems
(Section 5.3)’). ¥ 16 ´
b2  a 2 (e 2 1)  10 ¦ 1µ
Answer: (A) § 5 ¶
(iii) We know that the portion of the tangent is inter-  2b  2 22
cepted between the asymptote is bisected at the Answer: (B)
point of contact (see Theorem 5.28). In fact the
asymptotes are x  p 3y and the tangent at (3, 0) (ii) Let the corresponding directrix be x y  L (be-
is x  3. Hence, the tangent at (3,0) intersects the cause the directrix is perpendicular to transverse
asymptotes at points (3, 1) and (3, − 1) so that the axis). The distance of this from the centre is a/e
midpoint of the segment is (3, 0).
1 2 L 5 5
Answer: (D)  10 s 
2 4 2 2
3. Passage: The locus of the foot of the perpendicular 5 11
 L  3 
drawn from a focus onto a tangent to the hyperbola 2 2
is the auxiliary circle. Consider the hyperbola whose Therefore, the directrix equation is 2x 2y − 11  0.
foci are (5, 6) and ( − 3, − 2). The foot of the perpen-
Answer: (A)
dicular from the focus upon a tangent is the point (2,
(iii) The length of the latus rectum is
5). Answer the following questions.
2b2 2(22) 44
(i) Length of the conjugate axis of the hyperbola is  
a 10 10
(A) 4 22 (B) 2 22
Answer: (A)
Worked-Out Problems 371

Integer Answer Type Questions


1. Let S  (1, 2) and Sa  (5, 5) be the foci of a hyper- 3 5 7
bola. For any point P on the curve, it is given that
5 8 11  0
S aP SP  3 . If ea is the eccentricity of the conjugate
hyperbola, then 4ea is equal to _______. 7 11 c

Solution: We have  3(8c 121) 5(5c 77) 7(55 56)  0


3  c 363 385 7  0
Sa P SP  3  2a  3 or a   c 15  0
2
 c  15
Therefore
Hence, c − 7  8.
5
2ae  SSa  5  3e  5  e  Answer: 8
3
Hence 5. Four times the eccentricity of the hyperbola 9x2 −
1 1 16y2 − 18x 32y − 151  0 is equal to _______.
1
e2 ea 2 Solution: The equation of the hyperbola can be writ-
1 9 16 1 ten as
 2  1 2  1 
ea e 25 25 9(x − 1)2 − 16(y 1)2  151 9 − 16  144
Answer: 5
( x 1)2 ( y 1)2
 1
2. If the angle between the asymptotes of a hyperbola is 16 9
P / 3, then the eccentricity of its conjugate hyperbola Therefore, the eccentricity e is given by
is _______.
9  16(e 2 1)
Solution: From Theorem 5.21, part (2), let ea be the ec-
9 25
centricity of the conjugate hyperbola. Therefore  e2  1 
16 16
1 1 1 3 1 5
2
2
 1 2
 1  e
e ea ea 4 4 4
Hence, ea  2.  4e  5
Answer: 2 Answer: 5

3. If p1 and p2 are perpendiculars drawn from any point 6. The radius of the director circle of the hyperbola
on the curve − x2 2y2
 2 to its asymptotes, then 3p1p2 x 2 y2
is equal to _______.  1 is _______.
4 3
Solution: By Theorem 5.21, part (2), we have Solution: The director circle of the hyperbola
a 2 b2 2(1) 2 x2 y2
p1 p2  2 2
  1
a b 2 1 3 a2 b2
Therefore, 3 p1p2  2. is
Answer: 2 x2 y2  a2 − b2
 x2 y2  4 − 3  1
4. If the equation of the asymptotes of the hyperbola Answer: 1
3x2 10xy 8y2 14x 22y 7  0
7. For the hyperbola
is
x 2 y2
3x2 10xy 8y2 14x 22y c  0 1
25 16
then the value of c − 7 is _______.
the tangent at (5, 0) is intercepted by the asymptotes
Solution: Since 3x2 10xy 8y2 14x 22y c  0 of the hyperbola at P and Q. Then, the sum of the
represent pair of lines, we have
372 Chapter 5 Ellipse and Hyperbola

coordinates of the midpoint of the segment PQ is Solution: The tangent at (a cos Q , b sin Q ) to the ellipse is
_______. x y
cos Q sin Q  1
Solution: A portion of the intercept of any tangent to a b
the hyperbola between the asymptotes is bisected by the
This meets the axes at the points A(a sec P, 0) and B(0,
point of contact (see Theorem 5.28).
bcosec P ). Let (x1, y1) be the midpoint of AB. Therefore
Answer: 5
2 x1  a sec Q and 2 y1  b co secQ
8. If A B  P / 2, then the chord joining the points a b
whose eccentric angles are A and B of the ellipse  2 cos Q  and 2 sin Q 
x1 y1
x2 y2 a2 b2
1  4(cos2 Q sin 2 Q ) 
a2 b2 x12 y12
touches the ellipse
Therefore, the locus of (x1, x2) is
2 2
x y 1
 a2 b2
a 2
b 2
k 4
x2 y2
where k is equal to _______.
Answer: 4
Solution: The chord joining A and B is
10. Through the point (3,5), the number of tangents
x cos[(A B ) / 2] y sin[(A B )/ 2] ¥ A B´
 cos ¦ drawn to the ellipses 3 x 2 5 y2  32 and 25 x 2 9 y2 
a b § 2 µ¶
450 is _______.
¥P´ 1
 cos ¦ µ  Solution: Let
§ 4¶ 2
S y 3 x 2 5 y2 32
If
and Sa y 25 x 2 9 y2 450
A B
Q Also (3, 5)  ( x1 , y1 ). Now,
2
then the equation of the chord is S11  3(9) 5(25) 32  0
Sa 11  25(9) 9(25) 450  0
x y
cos Q sin Q  1 Therefore, (3, 5) is an external point to S  0 and (3, 5)
(a/ 25 ) (b/ 2 )
lies on Sa  0. Hence, the number of tangents drawn to
Hence, the chord touches the ellipse the ellipses through (3, 5) is 3.
x2 y2 Answer: 3
2
1
(a/ 2 ) (b/ 2 )2 11. The number of distinct normals that can be drawn
x2 1 y2 from the point (0, 6) to the ellipse
or 2
2 
a b 2 x 2 y2
1
Therefore, k  2. 169 25
Answer: 2 is _______.
Solution: Observe that y-axis is normal to the ellipse
9. The locus of the midpoint of the portion of the tan-
at (0, 5) and y-axis is passing through (0, 6). Now, a nor-
gent to the ellipse
mal to the ellipse at (13 cos P, 5 sin P ) is
x2 y2
1 13 x 5y
a2 b2  169 25  144
cos Q sin Q
included between the axes is the curve This passes through the point (0, 6). So
2 2
a b 30
2
k  144
x y2 sin Q
where k is equal to _______.
Summary 373

15 5 The latus rectum is


 sin Q  
72 24 2b2 2(1)
¥ 5´ ¥ 5´  1
 Q  P Sin 1 ¦ µ or 2P Sin 1 ¦ µ a 2
§ 24 ¶ § 24 ¶
Hence, Statement (A) is true. Also, the ellipse is
Hence, the number of normals that can pass through
(0, 6) is 3. X2 Y2
1
Answer: 3 4 1
where X  x 1, Y  y 2. The foci is
12. The number of following statements which are true
in the case of the ellipse x 2 4 y2 2 x 16 y 13  0 ( X  p ae, Y  0)  ( x 1  p 3 , y 2  0)  (1 p 3 , 2)
is _______.
Hence, the distance between the foci is
(A) The latus rectum of the ellipse is 1.
(1 3 ) (1 3 )  2 3
(B) The distance between the foci is 4 3.
(C) The sum of the focal distances of a point P(x, y) Hence, Statement (B) is not true. The sum of the focal
on the ellipse is 4. distance is
(D) The line y  3 meets the tangents drawn at the SP Sa P  2a  2(2)  4
vertices of the ellipse at points P and Q. Then
Therefore, Statement (C) is true. The tangents at the ver-
PQ subtends a right angle at either of the foci.
tices are x 1  p 2 or x  3, 1. The line y  3 meets
Solution: The given ellipse equation is these tangents at (3, 3) and (−1, 3). We have P  (3, 3) and
Q  ( 1, 3)S  (1 3 , 2). Therefore
( x 1)2 4( y 2)2  13 1 16  4
¥ 3 2 ´ ¥ 3 2 ´
( x 1)2 ( y 2)2 Slope of SP s Slope of SQ  ¦
 1 § 3 1 3 µ¶ ¦§ 1 1 3 µ¶
4 1
1 1 1
 s   1
We have a 2  4, b2  1. The eccentricity e is given by 2 3 2 3 4 3
3 Therefore, PQ subtends right angle at the focus
1  4(1 e 2 )  e 
2 (1, 3 , 2) and also at the other focus. Hence, Statement
(D) is true.
Answer: 3

SUMMARY
5.1. Definition: The locus of the point whose distance 5.3. Properties of curve:
from a fixed point S is equal to e times (0  e  1)
1. Curve is symmetric about both x-axis and y axis.
the distance of the same point from a fixed line l,
That is, if (x, y) is a point on a curve, then (x, y),
which is not passing through S, is called an ellipse.
( x, y) and ( x, y) are also on the curve.
The fixed point S is called a focus and the fixed line
l is called the directrix. The fixed number e is called 2. For any point (x, y) on a curve, a b x b a and
the eccentricity. b b y b b.
3. x-axis meets a curve at A(a, 0) and Aa(−a, 0)
5.2. Standard form: The equation (called the vertices) and y-axis meets the curve at
B(0, b) and Ba (0, b).
x2 y2 4. For each real value of x( a b x b a), there are two
 1 (a 2  b 2 )
a2 b2 corresponding values for y which are given by
is called standard equation to the ellipse where b 2
yp a x2
b2  a 2 (1 e 2 ). a
374 Chapter 5 Ellipse and Hyperbola

and for each value of y( b b y b b), there are two the corresponding directrices are y  p b/e. The
corresponding values for x which are given by length of latus rectum is 2a 2 /b.
a 2
xp b y2 ( x x1 )2 ( y y2 )2
b 5.12. 1
a2 b2
5. Since a curve is symmetric about axes, there are
another focus and another directrix. where a2  b2 or b2  a2 are ellipses with (x1, y1) as
centre.
5.4. Foci: For the ellipse 5.13. Position of a point: Any point belonging to the foci
region is called an internal point. A point which is
x2 y2 neither internal nor on the curve is called external
1 point.
a2 b2
the foci are S (ae, 0) and Sa ( ae, 0). 5.14. Notation:

5.5. Directrices: x  a /e is the directrix corresponding x2 y2


Sy 2
1
to the focus (ae, 0) and x  a /e is the directrix a b2
corresponding to the focus ( ae, 0). xx1 yy1
S1  2
1
5.6 Major and minor axes: In Fig. 5.1, AAa is called a b2
major axis and its length is 2a. BBa is called minor xx2 yy2
S2  1
axis and its length is 2b. a2 b2
5.7. Double ordinate: Any chord of the ellipse which x1 x2 y1 y2
S12  S21  1
is perpendicular to the major axis is called double a 2
b2
ordinate.
x12 y12
S11  1
5.8. Latus rectum: The double ordinate passing through a2 b2
the foci is called the latus rectum and its length is
5.15. Theorem: A point (x1, y1) is external or internal to
2b2 /a.
the ellipse

5.9. If S(ae, 0) and Sa ( ae, 0) are the two foci of an x2 y2


Sy 1 0
ellipse a 2
b2
x2 y2 according as S11  0 or S11  0.
2
2
1
a b 5.16. Auxiliary circle and parametric equations: The circle
then for any point P (x, y) on the curve, x 2 y2  a 2 is called auxiliary circle for the ellipse

SP  e( PM )  a ex x2 y2
 1 (a 2  b 2 )
and Sa P  a ex a2 b2
For an inverted ellipse, the auxiliary circle is x2 y2 
and hence SP Sa P  2a (constant). b2. The equations x = acos P and y = bsinP are called
5.10. Inverted ellipse: If b2  a2, then the equation parametric equations of the ellipse and P is called
the eccentric angle of the point.
x2 y2
1 5.17. Equations of the chord:
a2 b2 1. The equations of the chord joining P(x1, y1)
represents an ellipse called inverted ellipse. That is, and Q( x2 , y2 ) on the ellipse
in the standard form, the role of a and b are inter-
x2 y2
changed. Sy 1 0
2
a b2
5.11. For an inverted ellipse, the eccentricity e is given by
a 2  b2 (1 e 2 ). For an inverted ellipse, (0, b) and is S1 S2  S12 . That is,
(0, b) are the vertices, the foci are (0, p be) and
Summary 375

¥ xx1 yy1 ´ ¥ xx2 yy2 ´ x1 x2 y1 y2 a 2 l 2 b2 m 2  n 2


¦§ 2 2 1µ¶ ¦§ 2 2 1µ¶  2 2 1
a b a b a b 2. The condition for the line lx my  n, where
2. The equation of the chord joining two points lmn x 0 may be a normal to the ellipse, is
(a cos A , b sin A ) and (a cos B, b sin B ) is
a2 b2 (a 2 b 2 ) 2

x ¥ A B´ y ¥ A B´ ¥ A B´ l 2
m 2
n2
cos ¦ sin ¦  cos ¦
a § 2 µ¶ b § 2 µ¶ § 2 µ¶
5.23. Theorem: The feet of the perpendiculars drawn
5.18. Tangent and normal: from the foci onto a tangent of an ellipse lie on the
1. The equation of the tangent at (xl, y1) is auxiliary circle of the ellipse.
xx1 yy1
S1 y 2
1 0 5.24. Theorem: The product of the perpendiculars drawn
a b2 from the foci onto a tangent of the ellipse is constant
and the normal at (xl, y1) is and equals to square of the semi-minor axis.

a 2 ( x x1 ) b2 ( y y1 ) 5.25. Theorem: Except the point of contact, every point


 on a tangent to an ellipse lies outside the ellipse.
x1 y1
2. The equation of the tangent at (a cos A , b sin A ) 5.26. Theorem (Equation of the chord in terms of its
is midpoint): If (x1, y1) is the midpoint of a chord of
the ellipse
x y
cos A sin A  1
a b x2 y2
Sy 1 0
and the normal is a 2
b2
ax sec A by cosecA  a 2 b2 then its equation is
5.19. Condition for tangency: S1  S11
1. The line y  mx c, c x 0, touches the ellipse
xx1 yy1 x12 y12
x2 y2  
1 a2 b2 a2 b2
a2 b2
5.27. Theorem: The midpoints of parallel chords of an
if and only if c 2  a 2 m2 b2 .
ellipse always lie on a line that passes through the
2. For all real values of @, the line centre of the ellipse.
x cos A y sin A  p
5.28. Diameter: The line on which the midpoints of a
where system of parallel chords of an ellipse lie is called
2 2 2 2 a diameter of the ellipse. If m is the slope of the
p  a cos A b sin A
parallel chords, then their midpoints lie on the
touches the ellipse. If y  mx c touches the el- diameter b2 x a 2 my  0.
lipse, then the point of contact is
5.29. Theorem: If one diameter of an ellipse bisects
¥ a 2 m b2 ´ the chords parallel to another diameter, then the
¦ c , cµ second one bisects the chords parallel to the first
§ ¶
diameter. Such diameters are called conjugate
5.20. Theorem: From any external point, two tangents diameters.
can be drawn to an ellipse.
5.30. Theorem (Pair of tangents): The combined equation
5.21. Director circle: The locus of the point from which of the pair of tangents drawn from a point (x1, y1) to
perpendicular tangents can be drawn to an ellipse x2 y2
is a circle called director circle of the ellipse. the ellipse S y 1  0 is
The equation of the director circle of the ellipse a2 b2
x2 y2 S12  SS11
 1 is x 2 y2  a 2 b2 .
a2 b2 2
¥ x 2 y2 ´ ¥ x 2 y2 ´
¥ xx yy ´
 ¦ 21 21 1µ  ¦ 2 2 1µ ¦ 12 12 1µ
5.22. 1. The condition that the line lx my n  0 touches § a b ¶ §a b ¶§a b ¶
x2 y2
the ellipse  1 is
a2 b2
376 Chapter 5 Ellipse and Hyperbola

5.31. Definition (Chord of contact): If the tangents 5.39. Theorem: Point (x1, y1) in the plane of the
drawn from an external point P touch the ellipse hyperbola
at Q and R, then the chord QR is called chord of
contact of P with regard to the given ellipse. If x2 y2
S 1 0
P  (x1, y1), the equation of its chord of contact is a2 a2
xx1 yy1 is an external point or an internal point according
S1 y 2
2
1 0 as S11  0 or S11  0, respectively.
a b
5.32. Theorem: Through a given point in the plane of an 5.40. 1. The equation of the chord joining two points
ellipse, in general, four normals can be drawn to (x1, y1) and (x2, y2) on the hyperbola
the ellipse.
x2 y2
S 2
1 0
5.33. Theorem: In general, a circle and an ellipse intersect a a2
at four points such that the algebraic sum of the
eccentric angles of the common points is an even is S1 S2  S12.
multiple of O. 2. The equation of the tangent at (x1, y1) is

5.34. Definition: Let l be a fixed straight line called xx1 yy1


S1 y 2

1 0
directrix, S be a fixed point (which is not located a b2
on l) called focus and e (which is greater than 1) is 3. The equation of the tangent at (a sec Q , b tan Q )
real number called the eccentricity. Then, the locus is
of the point whose distance from the fixed point S
is equal to e times the distance of the point from x y
sec Q tan Q  1
the line l is called hyperbola. a b
4. The equation of the normal at (x1, y1) is
x2 y2
5.35. Standard equation:  1 is the standard
a 2 b2 a 2 ( x x1 ) b2 ( y y1 )
equation of the hyperbola and b2  a 2 (e 2 1) if 
x1 y1
a2  b2, otherwise a 2  b2 (e 2 1).
5. The equation of the normal at (a sec Q , b tan Q )
5.36. Features of curve: is ax cos Q by cot Q  a 2 b2 .
1. Curve is symmetric about both axes. 6. The condition for the line y  mx c to touch
2. When x-axis intersects a curve at A(a, 0) and the hyperbola
Aa(–a, 0), then y-axis cannot intersect the curve.
x2 y2
3. Also (x, y) is a point as the curve š x r a. 1
a2 a2
4. Aa A is called transverse axis whose length is
2a and Ba B is called conjugate axis where Ba  is that c2  a2m2 b2 . That is, the line
(0, −b) and B  (0, b). BaB  2b is called length
y  mx p a 2 m2 b2 touches the hyperbola.
of the conjugate axis.
5. Curve is an unbounded curve. 7. For all values of @, the line x cos A y sin A  p,
6. The two foci are S (ae, 0) and Sa ( ae, 0) and where p  a 2 cos2 A a 2 sin 2 A , touches the
their corresponding directrices are x  p a /e. hyperbola.
8. The equation of the director circle is
7. x2 y2  a2 is called auxiliary circle.
x 2 y2  a 2 b2 when a r b.
8. The length of the latus rectum is 2b2 /a.
9. The equation of the chord in terms of its
9. SP Sa P  2a. midpoint (x1, y1) is S1  S11.
5.37. Parametric equations: x  a sec Q and y  b tan Q 10. Equation of the pair of tangents from an ex-
ternal point (x1, y1) is S12  SS11 .
x2 y2
are called parametric equations of 1
a2 b2 5.41. Asymptotes: The lines

5.38. External and internal points: The points belonging x y


p 0
to the region where the foci lie is called internal a b
region. The point which is neither internal nor on
the curve is called external point.
Exercises 377

b 5.46. Hyperbola Conjugate hyperbola  2 (Asymptotes).


yp x
a 5.47. If e1 and e2 are the eccentricities of a hyperbola and
look like tangents, but actually they are not the 1 1
its conjugate, then 2
 1.
tangents. These two lines are called asymptotes of e1 e2 2
the hyperbola
5.48. Rectangular hyperbola: A hyperbola in which
x2 y2 length of the transverse axis is equal to the length
2
2
1 of conjugate axis is called rectangular or equilateral
a a
hyperbola.
5.42. Angle between asymptotes: The angle between the
two asymptotes is 2 Sec 1e. 5.49. The standard equation of rectangular hyperbola is
x 2 y2  a 2 .
5.43. Few properties of asymptotes:
1. The chord of contact of any point on an asymp- 5.50. The eccentricity a rectangular hyperbola is 2
tote with respect to the hyperbola is parallel to and hence the angle between the asymptotes is
the same asymptote. ¥P´ P
2 Sec 1 2  2 ¦ µ 
2. The product of perpendiculars drawn from a § 4¶ 2
point on the curve onto its asymptotes is con-
5.51. The director circle of a rectangular hyperbola is the
a 2 b2
stant which is equal to . centre itself [i.e., {(0,0)}].
a2 b 2

3. The foot of the perpendicular from a focus onto 5.52. Simplest form: xy  c2 is the simplest form of a
an asymptote is one of the points of intersection rectangular hyperbola.
of the corresponding directrix with the auxiliary
5.53. Parametric equation of xy  c2: x  ct and y  c /t
circle.
are the parametric equations of xy  c2. Hence,
4. If a line cuts the hyperbola at P and Q and the every point on the curve xy  c2 is of the form
asymptotes at R and S, then PR  QS. (ct, c/ t ) for some non-zero, real t.
5. The combined equation of the pair of asymp-
totes and the equation of the hyperbola differ 5.54. The equation of the tangent to xy  c2 at t is
by a constant. x
ty  2c  x t 2 y  2ct
5.44. Conjugate hyperbola: If the roles of transverse and t
conjugate axes are interchanged, then the resulting 3 4
5.55. The normal at (ct, c /t ) is t x ty c ct  0.
hyperbola is called conjugate hyperbola.
5.56. If the normal at (ct, c/t ) to xy  c2 meets the curve
2 2
x y again at (ct a, c /t a), these t 3 t a  1.
5.45. 2
2
 1 is the conjugate hyperbola of
a b
5.57. A portion of a tangent to xy  c2 lying between
x2 y2 the coordinate area is bisected by the point of
 1.
a2 b2 contact.

EXERCISES
Single Correct Choice Type Questions
¥ et e t et e t ´ x 2 y2
1. For any real t, the point ¦ , is a point (C) ellipse 1
§ 2 2 µ¶ 2 1
on the
2 2
(D) circle x y  2
(A) hyperbola x 2 y2  1
x 2 y2 2. Which one of the following equations represents the
(B) hyperbola 1
2 1 x2 y2
hyperbola  1?
a2 b2
378 Chapter 5 Ellipse and Hyperbola

(A) x  a tan Q , y  b sec Q 8. Let S and Sa be the foci of the ellipse

a ¥ 1´ b ¥ 1´ x2 y2
(B) x ¦§ t µ¶ , y  ¦§ t µ¶ 1
2 t 2 t a2 b2
(C) x  cos Q , y  cosec Q PSQ and PSaR are the focal chords of the ellipse.
(D) x  a cos Q , y  b tan Q Then, the equation of chord QR is

3. If the chord joining (a cos A , b sin A ) and (acosA, x y ¥ 1 e2 ´


(A) cos Q ¦ sin Q  1
bsin A ) on the ellipse a b § 1 e 2 µ¶

x2 y2 x ¥ 1 e2 ´ y
1 (B) cos Q sin Q  1
a 2
a 2
a ¦§ 1 e 2 µ¶ b

passes through (ae, 0), then eccentricity e is equal to x ¥ 1 e2 ´ y


(C) ¦ 2µ
cos Q sin Q  1
cos(A B )/ 2 sin(A B )/ 2 a § 1 e ¶ b
(A) (B)
cos(A B )/ 2 sin(A B )/ 2
x y ¥ 1 e2 ´
(D) cos Q ¦ sin Q  1
(C)
cos(A B )/ 2
(D)
cos(A B )/ 2 a b § 1 e 2 µ¶
cos(A B )/ 2 sin(A B )/ 2

x 2 y2 9. S and Sa are the foci of


4. The latus rectum of the ellipse  1 is
9 5
x2 y2
1
(A)
5
(B)
2 5
(C)
5
(D)
10 a2 b2
3 3 3 3 and P is any point on the curve. The lines PS and PSa
5. The radius of the circle passing through the foci of the meet the curve again at Q and Qa. The tangents at Q
ellipse and Qa meet at T. As P moves on the curve, the locus
of P is
x 2 y2
1
16 9 x2 y2
(A) (1 e 2 ) (1 e 2 ) 1
which is having its centre at (0, 3) is a2 a2

7 x2 y2
(A) (B) 8 (C) 2 3 (D) 4 (B) (1 e 2 )2 2
(1 e 2 )2 2
 (1 e 2 )2
2 a a
6. The axes of an ellipse are along the coordinates axes x2 y2
and its latus rectum and eccentricity are equal to 1/ 2. (C) (1 e 2 )2 2
(1 e 2 )2 2
 (1 e 2 )2
a a
Then the equation of the ellipse is
2
x y2
(A) 9x2 12y2  1 (B) 3x2 12y2  1 (D) (1 e 2 )2 (1 e 2 )2 1
a2 a2
(C) 6x2 12y2 1 (D) 12x2 6y2 1
10. Let H and Ha be a hyperbola and its conjugate, re-
7. The normal at point P on the ellipse  16 x2 4y2 spectively. Let e and ea be the intercept of a line on
meets the x-axis at Q. If M is the midpoint of the line the positive coordinate axis where e /2 and ea / 2 are
segment PQ, then the locus of M intersects the latus the eccentricities of H and Ha, respectively. Then the
rectums of the given ellipse at the points line touches the circle

¥ 3 5 2´ (A) x2 y2  4 (B) x2 y2  3
¥ 1´
(A) ¦ p ,p µ (B) ¦ p 2 3 , p µ (C) x2 y2  2 (D) x2 y2  1
§ 2 7¶ § 7¶

¥ 3 5 19 ´ ¥ 4 3´ 11. The angle between the asymptotes of the hyperbola


(C) ¦ p ,p (D) ¦ p 2 3 , p
§ 2 4 µ¶ § 7 µ¶ x2 y2
2
1
a b2
Exercises 379

is 60o and the product of the perpendiculars drawn (A) e12 4e2 2  6 (B) 2e12 e2 2  6
from the foci onto any tangent is 9. Then the equa-
tion of the director circle of the hyperbola is (C) 4e2 2 e12  6 (D) e12 e22  6
(A) x2 y2  9 (B) x2 y2  6 17. The locus of the midpoint of chords of the hyperbola
(C) x2 y2  18 (D) x2 y2  12
x2 y2
1
12. P(A ) and Q(B ) are points on the hyperbola a2 b2
x2 y2 which pass through a fixed point (h, k) is
2
2
1
a b ¥ h k´
(A) an ellipse with centre ¦ , µ
§ 2 2¶
where A B  P . Then the chord PQ passes through
¥ h k´
(A) the centre of the hyperbola (B) hyperbola with centre ¦ , µ
§ 2 2¶
(B) a focus
¥ h k´
(C) the cone of the vertices of the hyperbola (C) a circle with centre ¦ , µ
§ 2 2¶
(D) either through (0, b) or through (0, b)
¥ h k´
(D) a parabola with focus at ¦ , µ
13. The locus of the foot of the perpendicular drawn § 2 2¶
from the centre of the hyperbola xy  c2 onto any
tangent of the same curve is 18. The ordinate MP of the ellipse

(A) ( x 2 y2 )2  4c 2 xy (B) x 2 y2  4c 2 x 2 y2 x 2 y2
1
(C) ( x 2 y2 )2  2c 2 xy (D) ( x 2 y2 )2  4c 2 xy 25 9
meets the auxiliary circle at Q. The locus of the point
14. If the ellipse of intersection of the normals at P and Q to the
ellipse and the circle, respectively, is
x2 y2
1 2
(A) x2 y2  64 (B) x2 y2  34
a b2
where a  b, and the hyperbola x 2 y2  c 2 cut (C) x2 y2  15 (D) x2 y2  8
orthogonally, then
19. The locus of the midpoint of the portion of the tan-
(A) a 2 b2  2c 2 (B) a 2 b2  2c 2 gent lying between the coordinate axes of the ellipse
(C) a 2 b2  c 2 (D) a 2 b2  2c 2 x 2 y2
1
16 9
15. The tangent at (x0, y0) to the hyperbola xy  c2 meets
the coordinate axis at P and Q. If O is the centre of is
the curve, then the circumcentre of the $OPQ lies 16 9
on the curve (A) 9 x 2 16 y2  x 2 y2 (B) 4
y2 x2
(A) x 2 y2  c 2 (B) xy  c2
4 3 16 9
(C) 4 (D) 4
c 2
2 2 2
x2 y2 x2 y2
(C) xy  (D) x y  2c
2
20. P(A ), Q(B ) and R(G ) are vertices of one equilateral
16. The director circles of the ellipse triangle which lie on the ellipse
2 2
x y x2 y2
2
2
1 1
a b a2 b2
and the hyperbola If (x1, y1) is the circumcentre of $PQR, then
2 2
x y ¤ cos(A B ) 
2
2
1
a b
respectively, are  4r2 and x2 y2  r2 and their
x2 y2 9 x12 9 y12 3
(A) 2
2
(B) 9 x12 9 y12 a 2 b2
eccentricities are e1 and e2, respectively. Then a b 2
380 Chapter 5 Ellipse and Hyperbola

9 x12 9 y12 for which the area of $PON is maximum where O


9 x12 3 9 y12
(C) 3 (D) is the centre and N is the foot of the perpendicular
a b 2 2
2a 2b 2 from O onto the tangent at P are
21. The coordinates of the point P on the ellipse
¥ 16 9 ´ ¥ 9 16 ´
(A) ¦ p , p µ (B) ¦ p , p µ
x 2
y 2 § 5 5¶ § 5 5¶
1
16 9
¥ 16 9´
(C) ¦ p , p µ (D) (p 16, p 9)
§ 25 25 ¶

Multiple Correct Choice Type Questions


1. If e1 and e2, respectively, are the eccentricities 5. An ellipse intersects the hyperbola 2 x 2 2 y2  1 or-
2 2
of the ellipse 9 x 4 y  36 and the hyperbola thogonally. The eccentricity of the ellipse is reciprocal
9 x 2 4 y2  36, then of that of the hyperbola. If the axes of the ellipse are
along the coordinate axes, then
5 13
(A) e1  (B) e2  (A) Equation of the ellipse x2 2y2  2
3 2
137 127 (B) The foci of the ellipse are (p1, 0)
(C) e12 e2 2  (D) e12 e2 2  (C) The equation of the ellipse is x2 2y2  4
36 36
(D) The foci of the ellipse are (p 2 , 0)
2. Which of the following are true to the ellipse
4 x 2 y2 8 x 2 y 1  0 ? (Hint: Conics which intersect orthogonally have the
same foci.)
(A) Foci are (1, 1 p 3 )
4 6. Let a hyperbola passes through foci of the ellipse
(B) Directrices are y  1 p
3 x 2 y2
3 1
(C) Eccentricity is 25 16
4
The transverse and conjugate axes of the hyperbola
(D) Latus rectum is 8
coincide with the major and minor axes of the ellipse.
3. Let P(x1, y1) and Q(x2, y2), y1  0, y2  0 be the end If the product of the eccentricities of the given ellipse
points of a latus rectum of the ellipse x2 4y2  4. The and hyperbola is 1, then
equation of the parabola with latus rectum PQ is x 2 y2
(A) The equation of the hyperbola is 1
(A) x 2 2 3 y  3 3 9 16
(B) x2 2 3y  3 3 x 2 y2
(B) The equation of the hyperbola is 1
9 25
(C) x2 2 3y  3 3
(C) A focus of the hyperbola is (5, 0)
(D) x 2 2 3 y  3 3
(D) A vertex of the hyperbola is (5 3 , 0)
(IIT-JEE 2008)
(IIT-JEE 2006)
4. On the ellipse 4 x 2 9 y2  1, the points at which the
7. If the normals at P to the rectangular hyperbola x2 –
tangent are parallel to the line 9y  8x are
y2  4 meet the major and minor axes at G and g, re-
¥ 2 1´ ¥ 2 1 ´ spectively, and C is the centre of the hyperbola, then
(A) ¦ , µ (B) ¦ , µ
§ 5 5¶ § 5 5¶
(A) PG  PC (B) Pg  PC
¥ 2 1´ ¥ 2 1´ (C) PG  Pg (D) Gg  2PC
(C) ¦ , µ (D) ¦ , µ
§ 5 5¶ §5 5 ¶

Matrix-Match Type Questions


In each of the following questions, statements are given in column I are labeled as (A), (B), (C) and (D), while those
two columns, which have to be matched. The statements in in column II are labeled as (p), (q), (r), (s) and (t). Any
Exercises 381

given statement in column I can have correct matching with Column I Column II
one or more statements in column II. The appropriate bub-
bles corresponding to the answers to these questions have (B) Point Z in the complex (q) a parabola
to be darkened as illustrated in the following example. plane satisfying
Z 2 Z 2  3 is
Example: If the correct matches are (A) m (p), (s), (B) m
(C) The conic represented by (r) an ellipse
(q), (s), (t), (C) m (r), (D) m (r), (t), that is if the matches
the equation
are (A) m (p) and (s); (B) m (q), (s) and (t); (C) m (r);
and (D) m (r), then the correct darkening of bubbles will ¥ 1 t2 ´ 2t
look as follows: x  3¦ 2µ
,y 
§ 1 t ¶ 1 t2
p q r s t
is
A
(D) The conic whose eccen- (s) a hyperbola
B
tricity is greater than or
C equal to 1 is
D

3. Let the foci of the hyperbola


1. Some conics are given in Column I and their direc-
trices are given in Column II. Match the items of x2 y2
Column I with those of Column II. 1
c2 d2
be the vertices of the ellipse
Column I Column II
34 x2 y2
(A) 3x2 − 4y2 6x 24y − (p) x  1 2 1
135  0 7 a2 b2

3 and the foci of the ellipse are the vertices of the hyper-
(B) 2x2 3y2 − 8x 6y − (q) x  1 p 17
23 bola. Let e1 and e2 be the eccentricities of the ellipse
70
and hyperbola, respectively. Match the items of
(C) 2x2 3y2 − 4x 12y − 2 Column I with those Column II.
(r) x  2
20  0 7
Column I Column II
(D) 3x2 − 4y2 12x 8y − (s) x  2 p 3 3
b (p) 4
40 34 (A) is equal to
(t) 1 2 x d
7
(B) e1 e2 is always greater than (q) 3

2. Match the items of Column I with those of Column II. (C) If the angle between the asymp- (r) 2
totes is 2P / 3, then 4e1 
Column I Column II
(D) If e1  1/ 2 and (h, k) is a point (s)1
(A) The locus of the point (h, k) (p) a circle of intersection of the ellipse and
such that the line hx ky  hyperbola, then 9 h2 / 2k 2 
1 touches the circles x2 y2
 4 is
(Continued)

Comprehension Type Questions


1. Passage: Consider the ellipse (i) Which line among the following lines touches
the ellipse?
x 2 y2
2 (A) 2x 3y  12 (B) x 3y  12
9 4
Answer the following questions. (C) 2x y  12 (D) x y  6
382 Chapter 5 Ellipse and Hyperbola

(ii) The product of the perpendiculars drawn from 4 5 4 5


the foci onto a tangent of the ellipse is (A) (B) (C) (D)
3 4 3 2
(A) 4 (B) 8 (C) 16 (D) 4 2
(ii) The eccentricity of the conjugate hyperbola is
(iii) The locus of the feet of the perpendiculars drawn
from the foci onto a tangent of the ellipse is 4 5 4 5
(A) (B) (C) (D)
7 3 13 21
x 2 y2
(A) 1 (B) x2 y2 9
18 8 (iii) The product of the perpendiculars drawn from
(C) x2 y2  81 (D) x2 y2  91 any point on the hyperbola onto its asymptote is
equal to
2. Passage: Consider the ellipse 144 25 9 25
(A) (B) (C) (D)
25 144 25 7
x2 y2
1 4. Passage: The circle x2 y2 − 8x  0 and the hyperbola
a2 b2
Answer the following questions. x 2 y2
1
(i) If the latus rectum is half of the major axis, then 9 4
the eccentricity of the ellipse is intersect at points A and B. Answer the following
1 2 1 1 questions.
(A) (B) (C) (D)
2 3 3 2 (i) The equation of a common tangent with positive
slope to the circle as well as to the hyperbola is
(ii) If the angle between the lines joining the foci to
an extremity of the minor axis is 90o, then the ec- (A) 2 x 5 y 20  0
centricity of the ellipse is equal to
(B) 2 x 5 y 4  0
1 3 3 1
(A) (B) (C) (D) (C) 3 x 4 y 8  0
2 2 4 2
(D) 4 x 3 y 4  0
(iii) If the latus rectum is equal to half of the minor
axis then the eccentricity is equal to (ii) The equation of the circle with AB as diameter
is
2 1 3 1
(A) (B) (C) (D) (A) x 2 y2 12 x 24  0
3 3 2 2
(B) x 2 y2 12 x 24  0
3. Passage: Consider the hyperbola − 9x2
18x 16y2
(C) x 2 y2 24 x 12  0
32y − 151  0. Answer the following questions.
(D) x 2 y2 24 x 12  0
(i) The eccentricity of the curve is
(IIT-JEE 2000)

Integer Answer Type Question


The answer to each of the questions in this section is a X Y Z W
non-negative integer. The appropriate bubbles below the 0 0 0 0
respective question numbers have to be darkened. For 1 1 1 1
example, as shown in the figure, if the correct answer to 2 2 2
the question number Y is 246, then the bubbles under Y
3 3 3 3
labeled as 2, 4, 6 are to be darkened.
4 4 4
5 5 5 5
6 6 6
7 7 7 7
8 8 8 8
9 9 9 9
Answers 383

1. The line 2x y  1 is a tangent to the hyperbola


x 2 y2
2 2
1
x y 50 20
1
a2 b2 from which a pair of perpendicular tangents are
If this line passes through the point of intersection of drawn to the ellipse
the nearest directrix with x-axis, then the eccentricity x 2 y2
of the hyperbola is _________. 1
16 9
[Hint: The line 2x 3  1 meets the x-axis at (1/2, 0).]
is_________.
2. If the line 2 x 6 y  2 touches the hyperbola − x2
2y2  4 at (x1, y1), then y12 x1 is equal to _________. 8. If the equation of the pair of asymptotes of the hy-
perbola 3x2 − 5xy − 2y2 17x y 14  0 is 3x2 −
3. The circle x2 y2
 16 cuts the hyperbola xy  1 at four 5xy − 2y2 17x y 1 k  0, then the value of k is
points (xr, yr) (where r  1, 2, 3 and 4). Then the value _________.
of x1x2x3x4 y1y2y3y4 is equal to _________.
9. If p1 and p2 are the lengths of the perpendiculars
drawn from the point ( 5 , 1) onto the asymptotes
4. If e1 and e2 are the eccentricities of the hyperbolas of the hyperbola x2 − y2  4, then p1p2 is equal to
_________.
x 2 y2 x 2 y2
 1 and  1
16 9 16 9 10. If P is the angle of rotation required to transform the
equation x2 − y2  a2 into xy  c2, then the value of
e12 e22 tan Q is _________.
respectively, then is equal to _________.
e12 e22
11. If the foci of the ellipse
5. The tangent to a rectangular hyperbola xy  1 at a
point meets the coordinate axis at P and Q. If O is x 2 y2
1
the center of the hyperbola, then the event $OPQ is 16 b2
_________ sq. unit.
and the hyperbola
6. The tangent to the ellipse x2 4y2  4 meets the el- x 2 y2 1
lipse 
144 81 25
x 2 y2 coincide, then b2 is equal to _________.
1
6 3
12. A and Aa are the vertices of the ellipse
at P and Q. If P is the angle between the tangents
drawn to the ellipse x 2 y2
1
x 2 y2 25 16
1
6 3 and P is a point on the ellipse. The tangent at P meets
at P and Q, then the value of cos P is_________. y-axis at point Q. Let R be the image of point Q in
the line y  x. Then, the circle described on the seg-
7. The number of points on the ellipse ment QR as diameter passes through a fixed point
whose abscissa is _________.

ANSWERS
Single Correct Choice Type Questions
1. (A) 4. (D)

2. (B) 5. (D)

3. (C) 6. (A)
384 Chapter 5 Ellipse and Hyperbola

7. (B) 15. (B)

8. (A) 16. (C)

9. (B) 17. (B)

10. (A) 18. (A)

11. (C) 19. (D)

12. (A) 20. (D)

13. (A) 21. (A)

14. (B)

Multiple Correct Choice Type Questions


1. (A), (B), (C) 5. (A), (B)

2. (A), (B) 6. (A), (C)

3. (B), (C) 7. (A), (B), (C), (D)

4. (B), (D)

Matrix-Match Type Questions


1. (A) m (p), (t); (B) m (s); (C) m (q); (D) m (r) 3. (A) m (s); (B) m (s), (r); (C) m (s); (D) m (p)
2. (A) m (p); (B) m (s); (C) m (r); (D) m (q), (s)

Comprehension Type Questions


1. (i) m (A); (ii) m (B); (iii) m (B) 3. (i) m (B); (ii) m (B); (iii) m (A)

2. (i) m (D); (ii) m (A); (iii) m (C)

Integer Answer Type Questions


1. 2 7. 4
2. 2 8. 9

3. 2 9. 2

4. 1 10. 1

5. 2 11. 7

6. 0 12. 0
Three-Dimensional
Geometry 6
Contents
6.1 Pre-Requisites
6.2 Coordinates, Direction Cosines
Three-Dimensional Geometry

and Direction Ratios


6.3 Plane
Z
6.4 Line

Worked-Out Problems
(x, y, z) Summary
Exercises
O z
x
Answers
y Y
X

Geometry is divided into two major


parts, namely, (a) plane geometry
and (b) solid geometry.
The branch of elementary geometry
Z that is dealt in two dimensions is
called plane geometry. Solid geome-
X try is called three-dimensional geom-
etry. The three coordinate axes divide
the space 3 into eight parts called
octants.
O
Y Y

Z
386 Chapter 6 Three-Dimensional Geometry

Geometry is a fascinating and thought-provoking topic since its origin. Greek philosopher Plato once pronounced
that “God always geometrizes” and he went on this philosophy to such an extent that he denied entry into his acad-
emy to anyone who knew nothing about geometry. Fundamentally, geometry is divided into two parts such as plane
geometry (two-dimensional geometry) and solid geometry (three-dimensional geometry). In this chapter, we learn
three-dimensional geometry for which we have to know some fundamentals of Euclidean pure geometry, solid
geometry and vectors.
The aim of this chapter is to present standard properties of lines and planes with vector algebra approach.

6.1 Pre-Requisites
Though we have discussed about vector algebra in Vol. 2, we list out some of the concepts and results (without proofs)
of vector algebra in this section for the convenience of the students. In this chapter, though coordinate methods are
explained, we give significance to vector methods since they are easy to understand. We begin the discussion with non-
empty set S (Z 3) called space and the elements of S are called points. Lines and planes are certain sub-sets of S. They
would be the central point of discussion in this chapter. First, let us discuss about the axioms on which Euclidean
geometry was built. This approach was propounded by Euclid which was further developed by Hilbert. Throughout
this chapter, Z represents the set of all real numbers.

A X I O M 6.1 Given any two distinct points, there is one andFHHH


only
E one line passing through them. If the two
points are A and B, then the line is denoted by AB.

A X I O M 6.2 Three non-collinear points determine


FHHHHHEunique plane. The plane determined by three non-collinear
points A, B and C is denoted by ABC .

A X I O M 6.3 If two distinct points lie in a plane, then the line passing through them also lies in the same
plane.

A X I O M 6.4 If two planes intersect, they intersect in a unique straight line (see the proof using coordinates in
Section 6.2).

A X I O M 6.5 On every straight line, there are at least two points. In every plane, there are at least three non-
collinear points and in the space there are at least four non-coplanar points.

Axioms 6.1–6.5 are called incidence postulates.

A X I O M 6.6 For any two points, there corresponds a non-zero real number called distance. The function which
spell this correspondence is called distance function which is denoted by d and the distance
between the points A and B is denoted by d(A, B). The distance function satisfies the following
conditions: d: S s S m Z is a function such that
1. d(A, B) r 0.
2. d(A, B)  0 if and only if A  B.
3. d(A, B)  d(B, A).
Hereafter, in this chapter we will denote d(A, B) by AB.

DEFINITION 6.1 Coordinate System Let L be a straight line and f: L m Z an injection such that
f ( P ) f (Q)  PQ  P, Q  L
Then f is called coordinate system on the line L and f(P) is called x-coordinate of P.
6.1 Pre-Requisites 387

A X I O M 6.7 Every line has a coordinate system. This is called Rooler axiom.

Note:
1. Every line may have many coordinate systems.
2. If A and B are two points on line L, then there exists a coordinate system on L such that the coordinate of A is zero
and the coordinate of B is positive.

A X I O M 6.8 If A, B and C are collinear points such that


(B E T W E E N N E S S )
AB BC  AC
then B is said to be in between A and C and is denoted by A − B − C. If A and B are two points, then
FHHHE
{P  AB | A P B} is called segment joining A and B and is denoted by AB. Generally AB is
also denoted by AB; however, as per the context, we can know whether it refers to the distance
between A and B or segment AB.
FHHHE
DEFINITION 6.2 Ray Let A and B be two distinct points. Then {P  AB A Ž BP} is called a ray from A through
point B.
HHHHE
Generally, there exists confusion while mentioning a ray andHHHHE a vector. The E is denoted by HAB
ray
HHHH HHHE . The vector with A
as initialHpoint
HHHE and B as terminal point is
HHHHE also denoted by AB . Generally
HHHHE AB denotes vector AB . Hence,
HHHHE while men-
tioning AB as ray, we mention that AB is a ray. Further, vector AB has a finite length whereas ray AB has no finite
length.

DEFINITION 6.3 Parallel Lines and Parallel Rays


1. Let L1 and L2 be two coplanar lines (i.e., in the same planes). Then L1 and L2 are said to be
parallel if either L1  L2 or L1 and L2 do not have a common point. If L1 and L2 are parallel,
then we write L1 % L2 and if they are not parallel, then we write L1 || L2 .
HHHHE HHHE HHHHE HHHE
2. Let AB and CD be two rays. They are said to be parallel either HifHHHboth
E ABHHHEand CD are
on the same line and one is a sub-set of the other or if theFHHHE AB and CD are distinct
lines
parallel lines and B and D lie on the same side of the line AC .

A X I O M 6.9 There will be one and only one line passing through a given point and parallel to a given line.
(P A R A L L E L
AXIOM)

Using the above definitions and axioms, we can prove the Euclidean geometry theorems. However, for this process, we
need some of the solid geometry theorems which we list below without their proofs.

T H E O R E M 6.1 If two distinct lines intersect, then they intersect at a unique point.

T H E O R E M 6.2 If a line, which is not in a given plane, intersects the plane, then it intersects at one and only one
point.

T H E O R E M 6.3 There is one and only one plane containing a given line and a point which is not located on the
line.

T H E O R E M 6.4 Two distinct intersecting lines determine unique plane.


388 Chapter 6 Three-Dimensional Geometry

DEFINITION 6.4 Suppose L is a line intersecting a plane E at point P. If L is perpendicular to every line in the
plane E which passes through point P, then L is said to be perpendicular to E and we write
L > E.

T H E O R E M 6.5 If a line is perpendicular to two intersecting lines at their point of intersection, then it is perpen-
dicular to the plane determined by the two intersecting lines (see Fig. 6.1).

E
L1

L2

FIGURE 6.1

T H E O R E M 6.6 There is only one plane perpendicular to a given line at a given point on it.

T H E O R E M 6.7 The locus of the point in the space which is equidistant from two given points is a plane bisecting
the segment joining the two points perpendicularly.

T H E O R E M 6.8 The lines perpendicular to the same plane are parallel.

T H E O R E M 6.9 For a given plane, there is one and only one line perpendicular to the plane at a given point
in it.

T H E O R E M 6.10 The set of all lines perpendicular to a given line at a given point constitutes a plane which is per-
pendicular to the line at that given point.

T H E O R E M 6.11 Let E be a plane and P be a point in the space, but not located in E. Let M be the foot of the per-
(T H E O R E M O F FHHHE from P onto E. Let N be the foot of the perpendicular drawn from M onto a line
pendicular drawn
THREE PER- l in E. Then, PN is perpendicular to the line l (see Fig. 6.2).
PENDICULARS)
P

M l
N

FIGURE 6.2
6.1 Pre-Requisites 389

T H E O R E M 6.12 If a plane intersects two parallel planes, then it intersects in two parallel lines (see Fig. 6.3).

L1

L2

FIGURE 6.3

T H E O R E M 6.13 If a line is perpendicular to one of the parallel planes, then it is also perpendicular to the other
planes.

T H E O R E M 6.14 Planes perpendicular to a line are parallel.

T H E O R E M 6.15 If L1, L2 and L3 are three lines such that L1 % L2 and L2 % L3 , then L1 % L3 .

T H E O R E M 6.16 The intercepts made by parallel planes on the lines perpendicular to them are equal. The length
of those equal intercepts is called the distance between the parallel planes (see Fig. 6.4).

A
C

B
D

AB = CD

FIGURE 6.4
390 Chapter 6 Three-Dimensional Geometry

T H E O R E M 6.17 If a line is parallel to another line in a plane, then the line is parallel to the plane itself (see Fig. 6.5).

L1

L2

FIGURE 6.5

T H E O R E M 6.18 Three planes, in which no two planes are parallel, intersect either in parallel lines or in concurrent
lines (see Fig. 6.6).

E3
E2 E1
E2
L1 L1
L2 L3
L1 = L2 = L3

E3
L3 L2
E1
E1 E3
E2
(a) (b) (c)

FIGURE 6.6

DEFINITION 6.5 Skew Lines Two lines are said to be skew lines if there exists no plane containing both the
lines.

Note: Skew lines never intersect and they are also non-coplanar.

T H E O R E M 6.19 If L1 and L2 are two skew lines, there exists a unique plane containing L1 and parallel to L2.

T H E O R E M 6.20 Let E be a plane and L be a line in the space. Then, the locus of the feet of the perpendiculars
(O R T H O G O N A L drawn from the point on L onto the plane E is a straight line and this line is called orthogonal
PROJECTION) projection of L on E.

L
L

E E
(a) (b)

FIGURE 6.7

Note: If a line is parallel to a plane, then the line is parallel to its orthogonal projection in the plane.
6.1 Pre-Requisites 391

DEFINITION 6.6 Angle between Line and Plane Let E be a plane and L be a line which is not parallel to E.
Let La be the projection of L on E. Then, the angle between L and La is called the angle be-
tween the line L and the plane E. If P is the angle between L and a normal to E, then either
90°−P or P−90° is the angle between L and E (see Fig. 6.8).

Normal L L

q
90q q  90
L q L
E E

Normal
(a) (b)

FIGURE 6.8

T H E O R E M 6.21 Let L1 and L2 be two skew lines. Then, there exists a unique line L perpendicular to both L1 and
L2. The intercept of L between L1 and L2 is called the shortest distance between L1 and L2 (see
Theorem 6.43, Chapter 6, Vol. 2).

6.1.1 Vectors
We shall recall some of the concepts and results of vector algebra.

E E
T H E O R E M 6.22 Let a and b be the position vectors of points A and B. If point P divides the segment AB in the
ratio l:m (l m ≠ 0), then the position vector of P is
E E
ma lb
l m

In particular, the position vector of the midpoint of AB is


E E
a b
2

T H E O R E M 6.23 E E E
1. The position vector of the centroid of a triangle whose vertices are a, b and c is
E E E
a b c
3
E E E E
2. Let a, b, c and d be the position vectors of four points A, B, C and D. Let G1, G2, G3 and G4
FHHHHE FHHHHE
be the
FHHHH FHHHHE of ΔBCD, ΔCAD, ΔABD and ΔABC, respectively. Then lines AG1 , BG2 ,
E centroids
CG3 and DG4 are concurrent. The point of concurrence is denoted by G and is called the
centre or the centroid of the tetrahedron ABCD. Its position vector is
E E E E
a b c d
4
392 Chapter 6 Three-Dimensional Geometry

E E E E E
T H E O R E M 6.24 If a and b are non-collinear vectors and r is any vector in the plane E determined by a and b,
E E
E thereE exists a unique pair of scalars x and y such that r  xa yb (see Fig. 6.9). In particular,
then
if i and j are unit vectors along the positive directions of the coordinate axes in the given plane,
E E E E
then every vector r in the coordinate plane is expressed as r  xi yj in one and only one way.

 y
b

N P (x, y)
M P 
 r
r 
j

 O  M x
O L a i
(a) (b)

FIGURE 6.9

E E E E
T H E O R E M 6.25 Suppose a, b and c are non-coplanar vectors and r is any vector
E E E Ein the space (Fig. 6.10). Then,
E E HE
there exist a unique triad of scalars (x, y, z) such that r  xa yb zc . In
E the space if i, j, k are unit
E E E
vectors along three mutually perpendicular lines, then r  xi yj zk for some unique triad of
real numbers (x, y, z).
y

  P
T b j
S 
r
U
 P
r O
 x
O R i
 
a k
V
 Q
c
(a) z (b)

FIGURE 6.10
E
In Section 6.2, we show that this (x, y, z) are the coordinates of the point whose position vector is r.

E E E
T H E O R E M 6.26 Let a, b and c be the position vectors of three points A, B and C, respectively. Then the points
A, B and C are collinear ifE and only if there exist scalars l, m and n (where all scalars are not equal
E E E
to zero) such that la mb nc  0 and l m n  0.

E E E E E
T H E O R E M 6.27 1. The equation of the line passing through the point A( a ) and parallel to a vector b is r  a tb,
where t  Z (see Fig. 6.11).
E E E E E
2. The equation of the line passing through two points A (a ) and B (b) is r  (1 t )a tb,
where t  Z.

A 
b

FIGURE 6.11
6.1 Pre-Requisites 393

E E
T H E O R E M 6.28 1. The equationE of the plane passing through the point A (a ) and parallel to the vectors b and
E E E E
(P A R A M E T R I C c is r  a tb sc , where t, s  Z .
E E E
VECTORIAL 2. The equationE of the plane passing through two points a and b and parallel to vector c is
EQUATIONS OF E E E
r  (1 t )a tb sc , where t, s  Z .
A PLANE) E E E
3. The equation of the plane passing through three non-collinear points a, b and c is
E E E E
r  (1 t s)a tb sc .

E E E E
T H E O R E M 6.29 Four points a, b, c and d in the space are coplanar if andE only E exist scalars x, y, z and u
ifEthere
E E
(where all scalars are not equal to zero) such that xa yb zc ud  0 and x y z u  0.

E HHHE E HHHE
DEFINITION 6.7 Angle between Vectors Let a  OA and b  OB be two non-zero vectors (see Fig. 6.12).
E
Then,
E the measure of AOB which does not exceed 180° is called the angle between a and
E E
b which is denoted by (a, b).

B

b


q = (a ,b )
O  A
a

FIGURE 6.12

Note: See Fig. 6.13.


E E
1. 0o b (a, b) bE180o.E Unlike the Eangle between the two lines, the angle between two non-zero vectors is only one angle
E E E
so that (a, b)  (b, a )  ( a, b).
2. Let K and μ be two positive real numbers. Then
E E E E E E
(i) (L a, Mb)  (a, b)  ( L a, Mb)
E E E E E E
(ii) ( a, b)  (a, b)  180o (a, b)
E E E E E E
(iii) ( L a, Mb)  (L a, Mb)  180o (a, b)

m
b la


b

a
q
180q
q



a
b

 m
b
l a

FIGURE 6.13

E HHHE E HHHE E HHHHE


DEFINITION 6.8 Right-Handed and Left-Handed Systems Let a  OA, b  OB E and c  OC (see Fig. 6.14).
E
When observed from point C, if the E rotation of a towards b is in anticlockwise sense not
E E E E E
exceeding 180°, then the triad (a, b, c ) is said to be in right-handed system. If (a, b, c ) is not a
right-handed system, then it is called left-handed system.
394 Chapter 6 Three-Dimensional Geometry


b

O  A
a

c
C

FIGURE 6.14

Note:
E E E E E E E E E
1. Triads (a, b, c ), (b, c , a ) and (c , a, b) all exist in one system only. That is, all exist in right-handed system or all exist
in left-handed system.
2. In a system, if two vectors are interchanged, then the system changes.
3. In a system, if one vector is replaced by its additive inverse, then the system changes.
E E
DEFINITION 6.9 Scalar Product or Dot Product LetEa and b be two vectors. Then the scalar product (or dot
E E E
product) denoted by a – b of a and b is defined as follows:
E E E E
1. a – b  0 if one of a, b is a null vector
E E E E E E
2. If a, b are non-zero vectors and P is the angle between them, then a – b  a % b cos P.

Note:
E E
1. a – b is a real number (scalar).
E E E E
2. When a – b    0, the angle P between a and b becomes acute or right angle or obtuse accordingly.
E E E E E E E E E E E E E E
3. a – b  b – a, a – ( b)  ( a ) – b  (a – b), (L a ) – ( Mb)  LM(a – b), where K and μ are scalars.
E E HHHE E HHHHE
DEFINITION 6.10 Orthogonal Projection Let 0 x a  OA and b  LM in the space (see Fig. 6.15). Draw LP
E HHHE
and MQ perpendicular
E to the support of the vector a . Then PQE is called the projection
E H HHE E
vector of b on a and PQ is called the orthogonal projection of b on a.

b M
L

O P Q A

FIGURE 6.15
E E E
Note:
E In the definition of projection of
E b on a, we considered the supporting line of a and hence the projection of
E E
b on a is equal to the projection of b on any vector which is collinear with a.

E E
T H E O R E M 6.30 Projection of b on a is
E E
¥ b – aE ´ E ¥ b – aE ´ ¥ aE ´
¦ E 2 µ a ¦ E µ ¦ E µ
§ a ¶ § a ¶§ a¶
E E
¥ b–a´ E
¦ E µ e
§ a ¶
E E E E
where e is the unit vector along the direction of a. The magnitude of the projection of b on a is
E E
b–a
E
a
6.1 Pre-Requisites 395

E E
DEFINITION 6.11 Suppose a and b are not collinear vectors. Then the vector
E
¥ b – aE ´ E
¦ E2µa
§ a ¶
E E
is called the component of b along a and
E
E (b – aE) E
b E 2 a
a
E E
is called the component of b perpendicular to a.
E E E
Hereafter, (i , j , k ) denotes the right-handed system of mutually perpendicular unit vectors.
E E E E E E E E
FORMULA 6.1: If a  a1 i a2 j a3 k and b  b1 i b2 j b3 k , then
E E
a – b  a1b1 a2 b2 a3 b3
E E
a–b a1b1 a2 b2 a3 b3
and cos Q  E E 
a%b a1 a22 a32 b12 b22 b32
2

E E E
Hereafter, we denote a – a by a 2 .

6.1.2 Equations of a Plane Using Scalar Product

T H E O R E M 6.31 Let E be a plane, O be its origin and ON be perpendicular drawn to E (Fig. 6.16). If ON  p and
HHHHE
E E E
n is the unit vector in the direction of ON , then the equation of the plane E is r – n  p. In par-
E E
ticular, if the plane passes through the origin, then its equation is r – n  0.

N 90

E
 
n r

FIGURE 6.16

E E E E
T H E O R E M 6.32 If q is any real number and m x 0, then the equation r – m  q represents the plane perpendicular
E E
to the direction of p m and the plane is at a distance of q / m from its origin.

E E E E
T H E O R E M 6.33 The equation (r a ) – m  0 represents a plane passing through the point a and the plane is per-
E
pendicular to the direction of m.
E E
E E E a–m
Note: The distance of the plane (r a ) – m  0 from the origin is E .
m

E E E E E
T H E O R E M 6.34 The distance of a point A(A ) from the plane r – n  p is A – n p .
396 Chapter 6 Three-Dimensional Geometry

DEFINITION 6.12 The angle between two planes is defined to be the angle between their normals. If P is the
E E E E
angle between the planes r – m1  q1 and r – m2  q2 , then
E E
m1 – m2
cos Q  E E
m1 m2

6.1.3 Cross Product


E E E E
DEFINITION 6.13 ConsiderE two vectors a and b. Then the cross product of a and b (in this order), denoted
E
by a s b, is defined as follows:
E E E E E
1. a s b  0 if either a or b is a null vector or they are collinear vectors.
E E
2. If a and b are non-zero and non-collinear vectors, then
E E E E E
a s b  ( a b sin Q )n
E E E E E E E E
where Q  (a, b) and n is the unit vector perpendicular to both a and b such that (a, b, n)
is a right-handed triad.

Note:
E E E E E
1. Since sin P > 0 (for E 0 < P <180°), the vector a s b is in the direction of n and also a s b is perpendicular to the
E
plane of a and b.
E E E
asb E
2. The unit vectors p E E are perpendicular to the plane of a and b.
asb
E E E E
3. b s a  (a s b).
E E E E E E
4. a s ( b)  ( a ) s b  (a s b).
E E E E
5. (L a ) s ( Mb)  (LM )(a s b)  L , M Z.

T H E O R E M 6.35 The distributive laws are as follows:


E E E E E E E
1. a s (b c )  a s b a s c
E E E E E E E
2. (a b) s c  a s c b s c
E E E
Also, if (i , j , k ) is a right-handed system of mutually perpendicular unit vectors triad, then we
E E E E E E E E E E E E E E E
have i s j  k  ( j s i ), j s k  i  (k s j ) and k s i  j  (i s k ).

E E E E E E E E
FORMULA 6.2: If a  a1 i a2 j a3 k and b  b1 i b2 j b3 k , then
E E E
i j k
E E
a s b  a1 a2 a3
b1 b2 b3

Note:
E E
1. a s b  ¤ (a b a3 b2 )2 .
2 3
E
E
2. If P is the angle between a and b, then
E E
asb
sin Q 
a12 a22 a32 b12 b22 b32
6.1 Pre-Requisites 397

DEFINITION 6.14 Vector Area Let D be a plane region enclosed by a curve C (see Fig. 6.17). Let P1, P2 and
E
P3 be three points on the closed curve C and Δ be the real area of D. If n is the unit vector
E
perpendicular to the plane region D, such that from the side of n, the points P1, P2 and P3
E
(in this order) are in anticlockwise sense, then Δ n is called the vector area of D.
P3
P3

n

D
P2
P2 D

 P1
n
P1
(a) (b)

FIGURE 6.17

E E
Note: If n is a unit vector perpendicular to the plane of D and Δ is the real area of D, then p ( $n) is the vector area
of D. Magnitude of the vector area is the real area.

T H E O R E M 6.36 If A, B and C are three non-collinear points (see Fig. 6.18), then the vector area of ΔABC is
1 HHHHE HHHHE 1 HHHE HHHE 1 HHHE HHHE
( AB s AC )  ( BC s BA)  (CA s CB)
2 2 2
E E E
Further, if a, b and c are the position vectors of the points A, B and C, respectively, then its vector
area is
1 E E E E E E
(b s c c s a a s b )
2
and its real area is
1 E E E E E E
bsc c sasasb
2

C
B

FIGURE 6.18

Note:
E E E E E E
1. The vector area of a parallelogram with a and b as adjacent side vectors is a s b and its real area is a s b .
1 HHHHE HHHHE
2. The real area of a parallelogram (in fact, any quadrilateral ABCD) in terms of its diagonal vectors is AC s BD .
2
E E E E E E E E E E
3. Three points a, b and c are collinear š b s c c s a a s b  0.
398 Chapter 6 Three-Dimensional Geometry

E E E E E E
DEFINITION 6.15 Scalar Triple Product E a – (b s c ) is called the scalar triple product of vectors a, b and c
E E
which is denoted by [a b c ].

E E E E E E
T H E O R E M 6.37 1. a – (b s c )  (a s b) – c (i.e., placement of dot and cross can be interchanged).
E E E EEE
2. If any two of three vectors a, b and c are collinear, then [a b c ]  0.
E E E EEE
3. The vectors a, b and c are coplanar š [a b c ]  0.
E E E EEE
4. The volume V of the parallelepiped with coterminous edges a, b and c is V  p [a b c ].
E E E
Accordingly (a, b, c ) is considered as a right-handed system or a left-handed system.
EEE
5. The value of [a b c ] changes if any two vectors are interchanged or any of the vectors is
replaced by its additive inverse.
EEE EEE EEE
6. [a b c ]  [b c a ]  [c a b]. That is, the value does not change, by cyclically permuting the
vectors.

FORMULA 6.3
E E E E E E E E E E E E
1. If a  a1 i a2 j a3 k , b  b1 i b2 j b3 k and c  c1 i c2 j c3 k , then

a1 a2 a3
EEE
[a b c ]  b1 b2 b3
c1 c2 c3
E E E E E E E E E E E E E E E
2. If the vectors l , m and n are non-coplanar and a  a1 l a2 m a3 n, b  b1l b2 m b3 n and c  c1l c2 m c3 n, then

a1 a2 a3
EEE E E E
[a b c ]  b1 b2 b3 [l m n]
c1 c2 c3
E E E
Note: The vectors a, b and c are coplanar if and only if

a1 a2 a3
b1 b2 b3  0
c1 c2 c3

T H E O R E M 6.38 1. Let V be the volume of the tetrahedron ABCD. Then

1 HHHHE HHHHE HHHHE


V [ AB AC AD]
6
1 E E E E E E
 [b a c a d a ]
6
E E E E
where a, b, c and d are the position vectors of the points A, B, C and D, respectively.
2. The volume of triangular prism ABCDEF is given by
1 HHHHE HHHHE HHHHE
[ AB AC AD]
2
HHHHE HHHHE HHHHE
3. Four points A, B, C and D are coplanar if and only if [ AB AC AD]  0.
6.2 Coordinates, Direction Cosines and Direction Ratios 399

6.2 Coordinates, Direction Cosines and Direction Ratios


FHHE FHHE
Let O be a point in the space called origin of reference and x a x and ya y be the two perpendicular lines passing through
FHHE FHHE
O called x- and y-axes, respectively (see Fig. 6.19). The plane determined by the lines x a x and ya y (see Theorem 6.4) is
FHHE FHHE
called xy-plane. Let za z be the line perpendicular to xy-plane at point O (Theorem 6.9). za z is called z-axis. On
HHHE HHHE HHHE
Ox, Oy and Oz, mark points i, j and k, respectively, FHHE that FOi
FHHE such  Oj  Ok. Those points are called unit points.
HHE
According to Rooler axiom (Axiom 6.7) on x a x, ya y and za z, there exist coordinate systems with O as origin and i,
j and k as unit points. We call the coordinates of points on x-, y- and z-axes as x-coordinate, y-coordinate FHHE pointHHHEand
z-coordinate, respectively, and they are denoted
HHHE by x,
HHHE y and z, respectively. The plane determined by xa x
HHHE and a yE is
yFHH
called xy-plane. The plane determined by ya y and za z is called yz-plane and the plane determined by za z and x a x is
called zx-plane. These are also called coordinate planes. Oxyz is called frame of reference.
z

x
y
O

y
x

z

FIGURE 6.19

6.2.1 Coordinates of a Point


Let P be any point in the space. Let the feet of the perpendiculars drawn from P onto x-, y- and z-axes be L, M and
L, respectively. Suppose coordinates of L, M and N are x, y and z, respectively. These are called x-, y- and z- coordi-
nates of P, respectively, and we write P  (x, y, z). That is, with reference to a frame of axes, to each point P in the
space, there corresponds an ordered triad (x, y, z) of real numbers. Conversely, suppose (x, y, z) is an ordered triad
of real numbers. Let L, M and N be the points on x-, y- and z-axes such the x is the coordinate of L, y is the coordi-
nate of M and z is the coordinate of N. At L, M and N, draw the planes perpendicular to x-, y- and z-axes, respec-
tively, meeting at point P (see Fig. 6.20). Since the line PL lies in the plane perpendicular to x-axis at L, it follows
that PL is perpendicular to x-axis (see Definition 6.4). Similarly, the lines PM and PN are perpendicular to y-axis
and z-axis, respectively. Hence x, y and z are the coordinates of P where P  (x, y, z). That is, to each ordered trial
(x, y, z) of real numbers, there corresponds unique point P such that P  (x, y, z) with reference to Oxyz. Thus, with
reference to a frame of reference, we established a one-to-one correspondence (bijection) between the set of all
points in the space and the set or all ordered triad (x, y, z) of real numbers. This space is called three-dimensional
space which is denoted by Z 3 .
The three coordinate axes divide the space Z 3 into eight parts called octants. Table 6.1 shows the signs of the coor-
dinates of any point in the space. Since PQ is perpendicular to zx-plane at Q, by Theorem 6.5, it is perpendicular to
both QL and QN so that OLQN is a rectangle and hence OL  NQ and ON  QL. Also OQPN is a rectangle, so we
have Ox  PQ. Thus,
QN  OL  x , PQ  ON  y and LQ  ON  z

That is, if P  ( x, y, z), then x , y and z are the distances of the point P from yz-, zx- and xy-planes, respectively.
Also by Pythagoras theorem,

PL2  PQ2 QL2  y2 z2

 PL  y 2 z2
400 Chapter 6 Three-Dimensional Geometry

That is, the distance of P(x, y, z) from x-axis  PL  y2 z2 . Similarly, the distance P( x, y, x) from y- and z-axes,
respectively, are z2 y2 and x 2 y2 . Also PL is perpendicular to X-axis which implies that PLO is 90o so that
OP 2  PL2 OL2  y2 z2 x 2
and OP  x 2 y2 z2

Table 6.1
Octant with bonding lines Oxyz Oxayz Oxyaz Oxyza Oxayaz Oxayza Oxyaza Oxayaza
Sign of the coordinates (x, y, z) , , −, , ,− , ,− −,−, −, ,− ,−,− −,−,−

O
L x

N
Q

FIGURE 6.20

QUICK LOOK 1

We can describe the coordinate axis and the coordi- 3. z-axis  { p(0, 0, z) z Z}
nate planes as follows:
4. xy-plane  { p( x, y, z) z  0}  { p( x, y, 0) x, y  Z}
1. x-axis  {P ( x, y, z) y  0, z  0}  {P ( x,0, 0) x Z} 5. yz-plane  { p(0, y, z) y, z  Z}
2. y-axis  {P(0, y, 0 y Z)} 6. zx-plane  { p( x, 0, z) x, z Z}

E E E HHHE HHHE HHHE E E E


Note: If i , j and k are unit vectors
E along Ox, Oy, and Oz, then i  (1, 0E, 0), j  (0, 1, 0) and k  (0, 0, 1) and
HHHE E E E E
P  ( x, y, z)  OP  xi yj zk. Hereafter, we identify the vector xi yj zk with the point (x, y, z).

T H E O R E M 6.39 If A  (x1, y1, z1) and B  (x2, y2, z2), then the distance between A and B is

( x2 x1 )2 ( y2 y1 )2 (z2 z1 )2
PROOF We have
HHHHE HHHE HHHE E E E
AB  OB OA  ( x2 x1 ) i ( y2 y1 ) j (z2 z1 )k
and hence
E
AB  ( x2 x1 )2 ( y2 y1 )2 (z2 z1 )2
6.2 Coordinates, Direction Cosines and Direction Ratios 401

T H E O R E M 6.40 Let A  ( x1 , y1 , z1 ) and B  ( x2 , y2 , z2 ). Suppose point P divides AB in the ratio l:m (l m x 0).
Then
¥ lx mx1 ly2 my1 lz2 mz1 ´
P¦ 2 , ,
§ l m l m l m µ¶

In particular, the midpoint of AB is


¥ x1 x2 y1 y2 z1 z2 ´
¦§ , , µ
2 2 2 ¶
HHHE E E E HHHE E E E
PROOF We have OA  x1 i y1 j z1k and OB  x2 i y2 j z2 k and also AP:PB  l : m. Hence, by
Theorem 6.22, we have
HHHE HHHE E E E
E E E HHHE l OB mOA (lx2 mx1 )i (ly2 my1 ) j (lz2 mz1 )k
xi yj zk  OP  
l m l m
Therefore
lx2 mx1
x
l m
ly2 my1
y
l m
lz2 mz1
and z
l m

QUICK LOOK 2

1. If A ( x1 , y1 , z1 ), B ( x2 , y2 , z2 ) and C ( x3 , y3 , z3 ) are 2. If ( xr , yr , zr ) (where r  1, 2, 3 and 4) are the ver-


the vertices of a triangle, then the coordinates of its tices of a tetrahedron, then the coordinates of its
centroid are given by centre are given by

¥ x1 x2 x3 y1 y2 y3 z1 z2 z3 ´ ¥ x1 x2 x3 x4 y1 y2 y3 y4 z1 z2 z3 z4 ´
¦§ , , µ¶ ¦§ , , µ¶
3 3 3 4 4 4

Example 6.1
HHHE
Show that the points A  ( 1, 3, 4), B  ( 2, 1, 4) and Also BC  (5, 12, 9). This implies
C  (3, 11, 5) form an isosceles triangle. HHHE
BC  52 ( 12)2 9 2  250
Solution: We have HHHE
HHHHE HHHE HHHE Finally CA  ( 4, 8, 1). This implies
AB  OB OA  ( 1, 4, 8) HHHHE
HHHHE AC  4 2 8 2 12  9
 AB  12 4 2 8 2  9
Since AB  9  AC , $ ABC is isosceles.

Example 6.2

Show that the points A  (1, 1, 2), B  (0, 1, 1) and C  Solution: We have
(2, 3, 3) are collinear and find the ratio in which B
( AB)2  (1 0)2 ( 1 1)2 (2 1)2  6
divides the segment AC.
( BC )2  (2 0)2 ( 3 1)2 (3 1)2  24
402 Chapter 6 Three-Dimensional Geometry

( AC )2  (2 1)2 ( 3 1)2 (3 2)2  6 ¥ 2 L 1 3L 1 3L 2 ´


(0, 1, 1)  B  ¦ , , ,
Therefore § L 1 L 1 L 1 µ¶

AB AC  2 6  BC Therefore

This implies that A, B and C are collinear. Suppose B 3L 1 3L 2


2 L 1  0,  1, 1
divides AC in the ratio L : 1. Therefore, from Theorem L 1 L 1
6.40, we get From these three equations, we get L  1 / 2. Hence, B
divides AC externally in the ratio 1:2.

Example 6.3
If A  (2, 3, 4) and B  ( 3, 5, 4), find the ratio in which Since P belongs to yz-plane, its x-coordinate should be
yz-plane divides the segment AB. zero so that
FHHHE 2 3L  0  L  2 / 3
Solution: Suppose yz-plane meets the line AB at P and
AP : PB  L : 1. Therefore, from Theorem 6.40, we get The ratio is 2:3.

¥ 2 3L 5 L 3 4 4 L ´
P¦ , ,
§ L 1 L 1 L 1 µ¶

Example 6.4

Find the centre and radius of the sphere passing through c


the points O(0, 0, 0), A(a, 0, 0), B(0, b, 0) and C(0, 0, c). and HO  HC  z 
2
Solution: Let H(x, y, z) be the centre of the sphere so Therefore, the centre H is given by
that HO  HA  HB  HC  Radius. Now ¥ a b c´
¦§ , , µ¶
2 2 2
HO  HA  ( x a) y z  x y z 2 2 2 2 2 2
a and the radius is given by
x
2
1 2
Similarly HO  a b2 c 2
2
b
HO  HB  y 
2

Example 6.5

Suppose A  ( 2, 2, 3) and B  (13, 3, 13). If P is a vari-  9 PA2  4 PB2


able point such that 3PA  2 PB, then find the equation
of the locus.  9[( x 2)2 ( y 2)2 (z 3)2 ]
 4[( x 13)2 ( y 3)2 (z 13)2 ]
Solution: Suppose P  ( x, y, z). Now
 5( x 2 y2 z2 ) 140 x 60 y 50z 1235  0
3PA  2 PB  x 2 y2 z2 28 x 12 y 10z 247  0

DEFINITION 6.16 Direction Ratios and Direction Cosines Let L be a line in the space and La be a line pass-
ing through the origin and parallel to L (see Fig. 6.21). If P (a, b, c) ≠ (0, 0, 0) is a point on
La, then a, b and c are called direction ratios of the line L or the vector (a, b, c), where at least
6.2 Coordinates, Direction Cosines and Direction Ratios 403

one of a, b and c is non-zero, is called the direction ratio of L. Generally, “DRs” denotes
HHHHE HHHE
the direction ratios. If Q x (0, 0, 0) is any point on La, then OQ  L OP  (L a, L b, L c) are
also DRs. Hence, a straight line has many DRs and it is known that the vectors (a, b, c) and
(L a, L b, L c) are parallel. In particular, if P (a, b, c) ≠ (0, 0, 0) is on L a and OP  1, then a, b
and c are called the direction cosines (DCs) of the line L (a, b and c are DCs of L). If a, b and
c are DCs of L, then −a, −b, and −c are also DCs of L. Generally, DCs of a line are denoted
by (l, m, n). Remember that (−l, −m, −n) are also DCs.
y
L
L

P (a, b, c)
O x

FIGURE 6.21

QUICK LOOK 3

1. DRs of a line L means any vector in the space par- 2. DCs of L means any unit vector parallel to the line.
allel to L. 3. If (l, m n) are DCs of a line L, then l 2 m2 n2  1.

HHHHE HHHHE
DEFINITION 6.17 DCs of Array Suppose AB is an ray in the space. Then any unit vector parallel to AB in
HHHHE
the direction of the ray AB is called DC of the ray AB.

QUICK LOOK 4

See Fig. 6.22. HHHE HHHE


5. The DCs
HHHE of Ethe positiveE coordinate axes E Ox, Oy
1. A line has two sets of DCs. and Oz are i  (1, 0, 0), j  (0, 1, 0) and k  (0, 0, 1),
2. A ray has only one set of DCs. respectively.
3. If (l, m, n) are the DCs of a line, then we have 6. The DRs of parallel lines are proportional.
l 2 m2 n2  1 and hence if we know any two val-
y
ues of l, m and n, and the sign of the third, then we
Q
can write the DCs of the line. P
4. If (a, b, c) are the DRs of a line L, then the DCs of
A (l, m, n)
L are
a
¥ a b c ´ O
x
¦ , , µ
¦§ a 2 b2 c 2 a 2 b2 c 2 a 2 b2 c 2 µ¶

¥ ´ z
a b c
and ¦ , , µ
¦§ a 2 b2 c 2 a 2 b2 c 2 a 2 b2 c 2 µ¶ FIGURE 6.22

T H E O R E M 6.41 HHHE
If a ray PQ makes angles A , B and G with the positive directions of the axes, then the DCs of the
HHHE
ray PQ are (cos A , cos B , cos G ).
404 Chapter 6 Three-Dimensional Geometry

HHHE HHHE HHHE


PROOF Suppose the DCs of the ray PQ are (l, m, n). Let A  (l, m, n) so that OA is parallel to PQ and
HHHE HHHE HHHE HHHE HHHE E
OA  1. Further, the vector OA makes angles A , B and G with Ox, Oy and Oz. Since i is the
HHHE
unit vector in the direction of Ox, we have
HHHE E HHHE E
OA – l  OA l cos A  cos A  l  cos A
Similarly m  cos B and n = cos G . Also

l 2 m2 n 2  1
 cos2 A cos2 B cos2 G  1
HHHE HHHE HHHE
Note: If (l, m, n) are the DCs of the line PQ, then (−l, −m, −n) are the DCs of the ray QP. The angles made by QP
HHHE HHHE HHHE HHHE
with Ox, Oy and Oz are P A , P B and P G so that the DCs of QP are cos A , cos B and cos G . Hence, for the
FHHE
line PQ, both (cos A , cos B , cos G ) and ( cos A , cos B, cos G ) are DCs.

T H E O R E M 6.42 The DRs of the line joining A ( x1 , y1 , z1 ) and B (x2 , y2 , z2 )are (x2 x1 , y2 y1 , z2 z1 ).
PROOF Let O(0, 0, 0) be the origin (see Fig. 6.23). Now, complete the parallelogram OABC so that
HHHHE HHHHE HHHE HHHE
OC  AB  OB OA  ( x2 x1 , y2 y1 , z2 z1 )

A B

O C

FIGURE 6.23

HHHHE
T H E O R E M 6.43 HHHm,
Suppose (l, HHHHEDCs of a ray AB. If P  ( x1 , y1 , z1 ) and Q  ( x2 , y2 , z2 ), then the projection
E n) are
vector of PQ on AB is
E E E
[l( x2 x1 ) m ( y2 y1 ) n (z2 z1 )](li mj nk )

and it modulus is l( x2 x1 ) m ( y2 y1 ) n(z2 z1 ) .


E E E E HHHHE
PROOF The vector e  li mj nk is Hthe
HHE unitHHHH
vector
E in the direction
HHHE E of AB. Therefore, from Theorem
E
6.30, the projection vector of PQ on AB is given by  ( PQ – e ) e . That is
E E E
[l( x2 x1 ) m( y2 y1 ) n(z2 z1 )](li mj nk )

T H E O R E M 6.44 Suppose (l1, m1, n1) and (l2, m2, n2) are the DCs of the lines L1 and L2 , respectively. If P is the
angle between L1 and L2 , then cos Q  l1l2 m1m2 n1n2 .
HHHE HHHE
PROOF Let A  (l1 , m1 , n1 ) and B =(l2 , m2 , n2 ) so that OAHand
HHE OB HHHE unit vectors parallel to L1 and L2 ,
are
respectively. Hence, Q is also the angle between OA and OB. Therefore
HHHE HHHE
OA – OB HHHE HHHE
cos Q  HHHE HHHE  l1l2 m1 m2 n1 n2 (& OA  1  OB )
OA OB
6.2 Coordinates, Direction Cosines and Direction Ratios 405

Example 6.6
E E E
If (l1 , m1 , n1 ) and (l2 , m2 , n2 ) are the DCs of the lines i j k
L1 and L2 , respectively, then show that the DRs of a line HHHE HHHHE
OA s OB  l1 m1 n1
perpendicular to both L1 and L2 are (m1n2 m2n1,
l2 n1 l1 n2 , l1 m2 l2 m1 ). l2 m2 n2
E E E
 (m1n2 m2 n1 ) i (l1n2 l2 n1 ) j (l1 m2 l2 m1 ) k
Solution: Let A  (l1 , m1 , n1) and B =(l2 , m2 , n2 ) so that
the DRs of the line perpendicular to both L1 and L2 are  (m1 n2 m2 n1 , l2 n1 l1 n2 , l1 m2 l2 m1 )
given by

Example 6.7
If the DCs of two lines satisfy relations l m n  0 and l m n
mn − 2nl − 2lm  0, then find them. Also show that one of Case 1: l  m, 2 m  n   
1 1 2
the angles between the lines is 2P / 3.
l m n
Solutions: We have Case 2: l  n, m  2 n   
1 2 1
l m n0 (6.1) Hence, the DRs of the lines are (1, 1, −2) and (1, −2, 1)
mn 2 nl 2lm  0 (6.2) and the DCs are given by

From Eq. (6.10), we have l  (m n) and substituting ¥ 1 1 2 ´


¦§ , , µ
the value of l in Eq. (6.2), we have 6 6 6¶
mn 2 n(m n) 2 m(m n)  0
¥ 1 2 1 ´
 2 m2 5mn 2 n2  0 and ¦§ , , µ
6 6 6¶
 (2 m n)(m 2 n)  0
If Q is the angle between them, we get
 2 m  n or m  2 n
1 2 2 1
If we substitute 2m  − n in l  − (m n), we have cos Q  
6 6 6 2
l  (m 2 m)  m
so that 2P / 3.
and m  2 n  l  (m n)  n

Example 6.8

The DCs of two lines satisfy the equations 2l 2 m n  0  m  2l, l  2m


and mn nl lm  0. Show that the two lines are at right Now m  2l and n = 2l 2 m  2l 4l  2l. This implies
angles to each other. that

Solution: We have l m n l m n
    
1 2 2 1 2 2
2l 2 m n  0 (6.3)
Hence, l  2 m and n  2l 2 m  2 m. This implies
mn nl lm  0 (6.4)
that
From Eq. (6.3), we have n  2l 2 m. Substituting the
value of n in Eq. (6.4), we have l m n
 
2 1 2
m(2l 2 m) l(2l 2 m) lm  0 Therefore, the DRs of the lines are (−1, 2, 2) and (2, −1,
 2l 2 5lm 2 m2  0 2) and the dot product is given by −2 − 2 4  0. Hence,
the lines are at right angles to each other.
 (2l m)(l 2 m)  0
406 Chapter 6 Three-Dimensional Geometry

Example 6.9

If L1 and L2 are two straight lines whose DCs satisfy the Therefore,
equations al bm cn  0 and ul 2 vm2 wn2  0, then l1l2 m1 m2 n1 n2
prove that    k (say)
c v b w c u a w b u a2 v
2 2 2 2 2
2 2
(a) a2 (v w) b (w v) c (u v)  0 if L1 and L2 are at
right angles (a) As L1 is perpendicular to L2, we have
2 2 2
a b c l1l2 m1m2 n1n2  0
(b)  0 if L1 and L2 are parallel.
u N w
 k[c 2 v b2 w c 2 u a 2 w b2 u a 2 v]  0
Solution: Eliminating n from the given two equations,  a 2 (v w) b2 (w u) c 2 (u v)  0
we have
(b) As L1 is parallel to L2, we have
(c 2 u a 2 w)l 2 2abw lm (c 2 v b2 w) m2  0
Dividing this equation with m2, we have l1 l
 2
m1 m2
2
¥ l´ ¥ l´
(c 2 u a 2 w) ¦ µ 2abw ¦ µ c 2 v b2 w  0 (6.5) That is, the roots of Eq. (6.5) are equal. Therefore, we
§ m¶ § m¶
get
which is a quadratic equation in l / m. If (l1 , m1 , n1 ) and
(l2 , m2 , n2 ) are the DCs of the two given lines, then l1 /m1 4a 2 b2 w 2  4(c 2 u a 2 w)(c 2 v b2 w)
and l2 /m2 are the roots of Eq. (6.5) so that
 a 2 b2 w 2  c 4 uv b2 c 2 uw a 2 c 2 vw a 2 b2 w 2
¥ l1 ´ ¥ l2 ´ c 2 v b2 w
¦§ m µ¶ ¦§ m µ¶  c 2 u a 2 w  c 2 (c 2 uv b2 wu a 2 vw)  0
1 2

Therefore  a 2 vw b2 uw c 2 uv  0 (6.6)

l1l2 m1 m2 Dividing Eq. (6.6) by uvw, we have



c 2 v b2 w c 2 u a 2 w a 2 b2 c 2
0
Similarly, we can show that u v w
m1 m2 n1 n2

c u a w b u a2 v
2 2 2

Example 6.10

If (lr , mr , nr )(where r  1, 2 and 3) are the DCs of three Solution: It is known that the lines are coplanar if and
concurrent lines, then show that the condition for them only if the vectors (lr , mr , nr ) are coplanar. These vectors
to be coplanar is that are coplanar if and only if their scalar triple product is
zero. That is, if and only if
l1 m1 n1
l2 m2 n2  0 l1 m1 n1
l3 m3 n3 l2 m2 n2  0
l3 m3 n3

Example 6.11

If a line makes angles A , B, G and D with the diagonals of a Solution: Suppose A  (1, 0, 0), B  (0, 1, 0) and C(0, 0, 1)
cube, then show that are the unit points on the coordinate axes and consider
the cube OADCBEFG (see Fig 6.24) so that D  (1, 0, 1),
4 E  (1, 1, 0), F  (1, 1, 1) G  (0, 1, 1). Suppose the given
cos2 A cos2 B cos2 G cos2 D 
3 line makes an angle A with the diagonal OF. If (l, m, n)
are the DCs of the given line, then
6.2 Coordinates, Direction Cosines and Direction Ratios 407

(l, m, n) – (1, 1, 1) l m n 1
cos A    [4(l 2 m2 n2 )]
(1) 12 12 12 3 3
HHHHE HHHE HHHHE 4
Also BD  (1, 1, 1), CE  (1, 1, 1),  (& l 2 m2 n 2  1)
E AG  ( 1, 1, 1).
and
HHHH 3
Suppose B is the angle made by AG with the line. We
have
B (0, 1, 0)
l m n E
cos B 
3
l m n G
cos G  F
3
l m n
and cos D =
3
A (1, 0, 0)
O
Therefore

cos2 A cos2 B cos2 G cos2 D


1
 [(l m n)2 ( l m n)2 (l m n)2 (l m n)2 ] C (0, 0, 1) D
3
FIGURE 6.24

Example 6.12
HHHHE HHHHE HHHHHE
If the DCs of the rays AB and AC are (l1 , m1 , n1 ) and Suppose ON1 is the second bisector of the angle Q ,
(l2 , m2 , n2 ), respectively, then show
HHHHE that DCs ofHHH
the
E angle that is, bisector of P Q . Now
bisectors of the rays AB and AC are HHHHHE HHHHHE HHHHHE HHHE HHHHHE HHHE
ON1 > OM1 and OM1 > PQ  ON1 PQ
(l1 p l2 , m1 p m2 , n1 p n2 ).
HHHE
HHHHE HHHHE and (l2 l1 , m2 m1 , n2 n1 ) are the DRs of PQ. Hence,
Solution: Let AM and AN be the angle bisectors HHHHHE
HHHHE HHHHE the DRs of ON1 are also (l2 l1 , m2 m1 , n2 n1 ) or
of the angle between AB and AC . If P  (l1 , m1 , n1 ) (l1 l2 , m1 m2 , n1 n2 ).
HHHE
and Q  (l2 , m2 , n2 ), then we know that OP is parallel
HHHHE HHHHE HHHHE
to AB and OQ is parallel to AC and hence the angle N1
HHHHE HHHHE HHHE
between AB and AC is same as the angle between OP P (l1, n1, m1)
HHHHE HHHE HHHHE HHHHHE
and OQ. Let (OP, OQ)  Q . Let OM1 be one of the an-
HHHE HHHHE HHHHE
gle bisectors of OP and OQ. Suppose AM is paral-
HHHHHE
lel to OM1 . Suppose PQ meets OM at R (see Fig. 6.25). q/2
HHHE HHHHE
Since OP and OQ are unit vectors, $OPQ is isosceles O q/2 R M1
so that R is the midpoint of PQ and hence
¥ l l m m2 n1 n2 ´
R¦ 1 2 , 1 , µ Q (l2, n2, m2)
§ 2 2 2 ¶
HHHHHE HHHHHE
HHHHRE lies on OM1 , and the fact that OM1 is parallel
Since FIGURE 6.25
to AM it follows that (l1 l2 , m1 m2 , n1 n2 ) are the
HHHHE
DRs of AM .

Example 6.13

If a, b and c are the lengths of the edges of a rectangular ¥ p a 2 p b2 p c 2 ´


parallelepiped, then show that the angle between any Cos 1 ¦ 2 µ
two diagonals of it is § a b2 c 2 ¶
where all signs or all are − signs should not be taken.
408 Chapter 6 Three-Dimensional Geometry

HHHE HHHE HHHHE


Solution: Let OA, OB and OC be coterminous ¥ a 2 b2 c 2 ´
edges of a rectangular parallelepiped (see Fig. 6.26). Cos 1 ¦ 2 µ
HHHE HHHE HHHHE § a b2 c 2 ¶
Suppose OA, OB and OC are x-, y- and z-axis, re-
¥ a 2 b2 c 2 ´
spectively. Also
HHHE we haveHHH E  a, OB  b Hand
OA HHHE OC  c. and Cos 1 ¦ 2 µ
Therefore, OA  (a, 0, 0), OB  (0, b, 0) and OC  (0, 0, c). § a b2 c 2 ¶
Consider the HHH angle
E between the HHHHdiagonals
E BD and
OE. Now,
HHHHE HHHE OE  ( a, b, c ) and BD  ( a, b, c ). Let y
( BD, OE)  A so that
HHHE HHHHE B(0, b, 0)
OE – BD a 2 b2 c 2 F(a, b, 0)
cosA  HHHE HHHHE  2 2 2
OE BD a b c
E(a, b, c)
¥ a 2 b2 c 2 ´
 A  Cos 1 ¦ 2 µ
§ a b2 c 2 ¶
Similarly, the other angles are
O
A(a, 0, 0) x
¥ a 2 b2 c 2 ´
Cos 1 ¦ 2 µ
§ a b2 c 2 ¶

¥ a 2 b2 c 2 ´ C(0, 0, c) D(a, 0, c)
Cos 1 ¦ 2 µ z
§ a b2 c 2 ¶
2
FIGURE 6.26
b2 c 2 ´
1 ¥ a
Cos ¦ 2 µ
§ a b2 c 2 ¶

QUICK LOOK 5

Since two diagonals are not parallel, in the formula or all – signs. Further, if a  b  c, then the angle is
¥ p a 2 p b2 p c 2 ´ Cos 1 (1/ 3).
Cos 1 ¦ 2 µ,
we should not take all signs
§ a b2 c 2 ¶

Example 6.14

Suppose (lr, mr, nr) (where r  1, 2 and 3) are the DCs of and l32 m32 n32  1
the three mutually perpendicular lines. The DRs of
another line L are proportional to (l1 l2 l3, m1 m2 Therefore,
m3, n1 n2 n3). Find the DCs of L and show that L is (l1 l2 l3 )2 (m1 m2 m3 )2 (n1 n2 n3 )2
equally inclined to all the three lines.
 3 2(0 0 0)  3
Solution: Let L1, L2 and L3 be the lines. Since these Hence, the DCs of L are
lines are mutually perpendicular to each other, we have
¥ l1 l2 l3 m1 m2 m3 n1 n2 n3 ´
l1l2 m1m2 n1n2  0 ¦§ , , µ¶
3 3 3
l2 l3 m2 m3 n2 n3  0
Suppose P is the angle between L and L1. Therefore
l3l1 m3 m1 n3 n1  0
l1 (l1 l2 l3 ) m1 (m1 m2 m3 ) n1 (n1 n2 n3 )
cos Q 
l12 m12 n12  1 3 3 3
l22 m22 n22  1 l12 m12 n12 1 1
 ( 0) 
3 3 3
6.2 Coordinates, Direction Cosines and Direction Ratios 409

1 Similarly, the angles made by the line L with L1 and L2


 Q  Cos 1 are equal to Cos 1 (1/ 3 ).
3

Example 6.15

Show that in a tetrahedron OABC, the opposite pairs of we have


edges are at right angles if and only if (OA)2 ( BC )2  E2 E E E2 E E
(OB)2 (CA)2  (OC )2 ( AB)2 . b (c a)2  c (b a)2
E E E E
c–a a–b
Solution: See Fig. 6.27. In the tetrahedron OABC,
(OC , AB),( AC , OB) and ( BC , OA) are the pairs of op- E E E HHHE HHHE
HHHE E HHHE E HHHHE E Therefore, a – (c b)  0  OA and BC are at right angles.
posite edges. Let OA  a, OB  b and OC  c. Assume
that O

(OA)2 ( BC )2  (OB)2 ( AC )2
E E E E2 E E
 a 2 (c b ) 2  b ( c a ) 2
E 2 E 2 E2 E E E 2 E 2 E2 E E C
 a b c 2(b – c)  a b c 2c – a
E E E E
 b– c  c –a A B
E E E
 c – (b a)  0 FIGURE 6.27
HHHHE HHHHE
This implies that OC is perpendicular to AB. Similarly, if
(OB) 2 ( AC )2  (OC) 2 ( AB)2

Example 6.16

Find the equation to the locus of a point whose distance


x 2 y2  3 ( x 1)2 ( y 2)2 (z 3)2
from the z-axis is three times the distance from the point
(−1, 2, −3).  x 2 y2  9 ¨ª( x 1)2 ( y 2)2 (z 3)2 ·¹

Solution: P(x, y, z) is a point whose distance from the  8 x 2 8 y2 9z2 18 36 y 54z 126  0
z-axis is three times its distances from the point (−1, 2, −3).
This implies

Example 6.17

Three vertices of a parallelogram ABCD are A (1, 2, 3), Therefore


B (−1, −2, −1) and C (2, 3, 2). Find the coordinates of the
x 1 3
fourth vertex D.  x4
2 2
Solution: In a parallelogram, diagonals bisect each y 2 5
 y7
other. Therefore, if D ( x, y, z) is the fourth vertex, then 2 2
we have
z 1 5
and  z6
¥ 1 2 2 3 3 2´ 2 2
¦§ , µ  Midpoint of AC
2 2 2 ¶
Therefore, D  (4, 7, 6).
 Midpoint of BD
¥ x 1 y 2 z 1´
¦ , , µ
§ 2 2 2 ¶
410 Chapter 6 Three-Dimensional Geometry

Example 6.18

Find the length and the direction cosines of a line seg- y


ment whose projections on the coordinate axes are 6, −3
and 2, respectively.

Solution: See Fig. 6.28. Let P  ( x1 , y1 , z1 ) and Q (x2,


P (x1, y1, z1)
y2, z2) be the extremities of the given line segment. Let
PA and PB be perpendicular to the z-axis. Now, O x
HHHHE A
OA  z1, OB  z2  AB  z2 z1
Therefore, z2 z1  2. Similarly, x2 x1  6 and y2 y1 =
3. Therefore Q (x2, y2, z2) B

z
PQ  6 2 ( 3)2 2 2  7
FIGURE 6.28
Hence, the DRs of PQ are (6, 3, 2) and the DCs of PQ
¥ 6 3 2 ´
are ¦ , , µ .
§ 7 7 7¶

6.3 Plane
HHHE
Suppose P ( x, y, z) is a point with reference to the frame of reference Oxyz. In this caseHHH
OP
E Eis a vector. Conversely, if
E
r is any vector in the space, there corresponds a unique point P in the space such that OP  r . Hence, there is one-to-
one correspondence between the set of space vectors and the set of all points in the space. Hence, hereafter, we iden-
tify space with vector space and denote it by Z 3 .

DEFINITION 6.18 Let f : Z 3 m Z be a function. Then the set S  [P( x, y, z) | f ( x, y, z)  0] is called a surface
represented by the equation f ( x, y, z)  0.
One equation represents only one surface. However, a surface may be represented by more than one equations,
because, if f ( x, y, z)  0 represents a surface S and L x 0, then L f ( x, y, z)  0 also represents the same surface S.

DEFINITION 6.19 If f is a polynomial in x, y and z, then the surface represented by f ( x, y, z)  0 is called alge-
braic surface.

DEFINITION 6.20 If a, b, c and d are real numbers and at least one of a, b and c is not zero, then the equation
ax by cz d  0 is called first-degree equation in x, y and z. The surface represented by
first-degree equation is called first-degree surface.

T H E O R E M 6.45 The equation of a plane is a first-degree equation in x, y and z (normal form).


PROOF See Fig. 6.29. Let E be a plane and O be its origin. Let N be the foot of Hthe HHHE perpendicular drawn
E E
from the
HHHHE HHHE origin O onto E. Let n be the unit vector in the direction of ON . If n  (l , m, n), then
ON  pn where p  ON .
HHHE HHHHE
P( x, y, z) is any point in E š NP – ON  0
E E E
š (r pn) – pn  0
E E
š r –n  p
HHHE E E HE E E E H
E
Now OP  xi yj zk and n  li mj nk. Then
E E
r – n  p š lx my nz  p
E
Since n  (l, m , n) is a unit vector, l, m , n x 0. Here, the equation of the plane E is lx my nz p  0,
a first-degree equation in x, y and z and this equation is called normal form of the plane.
6.3 Plane 411

E
 
n r

FIGURE 6.29

QUICK LOOK 6

1. If E passes through origin O, then P  0 so that its Hence, the plane lx my  p is parallel to z-axis. Sim-
equation is lx my nz  0. ilarly, the plane my nz  p is parallel to x-axis and
2. l x 0, m x 0  (l, m, 0) – (0, 0, 1)  0. Hence, the line lx nz  p is parallel to y-axis.
having (l, m, 0) as DRs is perpendicular to the z-axis.

DEFINITION 6.21 If a plane meets the axes at (a, 0, 0),(0, b, 0) and(0, 0, c), then a, b and c are called the inter-
cepts of the plane on the axes.

T H E O R E M 6.46 If abc x 0 , then the equation of the plane having intercepts on coordinate axes a, b and c is
x y z
1
a b c
This form is called intercept form.
E
PROOF Suppose E is the plane meeting the axes at A(a, 0, 0), B(0, b, 0) and C (0, 0, c). Let n  (l, m, n) be
the unit vector perpendicular to the plane E so that the equation of the plane is of the form
lx my nz  p (6.7)

Now A(a, 0, 0) belongs to E. This implies


p
(a, 0, 0) – (l, m, n)  p  al  p  l =
a
Similarly m = p / b and n  p / c. Hence, from Eq. (6.7), the equation of the plane is
x y z
1
a b c
Aliter: P( x, y, z) is a point in the plane E š the four points A, B, C and P are coplanar. This is
HHHHE HHHHE HHHHE
possible if and only if the vectors AP, AB and AC  0. This means
HHHHE HHHHE HHHHE
[ AP AB AC ]  0
x a y z
 a b 0  0
a 0 c
 bc( x a) cay abz  0
 bcx cay abz  abc
x y z
 1
a b c
412 Chapter 6 Three-Dimensional Geometry

T H E O R E M 6.47 A first-degree equation in x, y and z represents a plane.


PROOF Suppose
ax by cz d  0 (6.8)
is a first-degree equation in x, y and z. Since both equations ax by cz d  0 and (ax by
cz d)  0 represent the same surface (see Quick Look 6), we can suppose that d r 0. Now, we
can write Eq. (6.8) as
a b c d
x y z
2 2 2 2 2 2 2 2 2
a b c a b c a b c a b2 c 2
2

Take
a
l
a b2 c 2
2

b
m
a b2 c 2
2

c
and n
a b2 c 2
2

E E E
If n  (l, m, n), then n is a unit vector. If r  ( x, y, z) and
d
p
a b2 c 2
2

E E
then Eq. (6.8) becomes r – n  p, which represents a plane.

QUICK LOOK 7

1. Since is a normal to the plane provided in Eq. (6.8), it follows


that (a, b, c) are the DRs of the plane ax by cz
¥ a b c ´ d  0.
(l, m, n)  ¦ , , µ
¦§ a 2 b2 c 2 a 2 b2 c 2 a 2 b2 c 2 µ¶ d
2. p  is the distance of the plane from
a 2 b2 c 2
the origin.

DEFINITION 6.22 If one of a, b and c is not zero, then the equation ax by cz d  0 is called general equation
of a plane.

T H E O R E M 6.48 Two first-degree equations


a1 x b1 y c1z d1  0 (6.9)
and a2 x b2 y c2 z d2  0 (6.10)
represent the same plane if and only if a1 : b1 : c1 : d1  a2 : b2 : c2 : d2 .
PROOF Suppose a1 : a2  b1 : b2  c1 : c2  d1 : d2  k x 0, then
a2 x b2 y c2 z d2  0 š k(a1 x b1 y c1z d1 )  0
Therefore, Eqs. (6.9) and (6.10) represent the same plane.
Conversely, suppose Eqs. (6.9) and (6.10) represent the same plane, then (a1 , b1 , c1 ) and (a2 , b2 , c2 )
are the DRs of the normal to the plane [Quick Look 8, part (1)]. Therefore
(a2 , b2 , c2 ) = k(a1 , b1 , c1 )
6.3 Plane 413

for some k x 0. If ( x1 , y1 , z1 ) is a point of the plane represented by Eqs. (6.9) and (6.10), we have
d2   a2 x1 b2 y1 c2 z1  k(a1 x1 b1 y1 c1z1 )  kd1
Therefore, a2 : a1  b2 : b1  c2 : c1  d2 : d1 .

QUICK LOOK 8

The equation of the plane passing through the point Hence, we can write the equation of the plane if we
E
A( x1 , y1 , z1 ) which is having n  (a, b, c) as normal is know one point in the plane and DRs of the normal to
a( x x1 ) b( y y1 ) c(z z1 )  0 (see Theorem 6.33). the plane.

T H E O R E M 6.49 The equation of the plane determined by three non-collinear points A( x1 , y1 , z1 ), B( x2 , y2 , z2 ), and
C ( x3 , y3 , z3 ) is
x x1 y y1 z z1
x2 x1 y2 y1 z2 z1  0
x3 x1 y3 y1 z3 z1

PROOF We have
HHHHHHE
P ( x, y, z) is a point in the plane ABC
HHHHE HHHHE HHHHE
š AP, AB, and AC coplanar vectors
HHHHE HHHHE HHHHE
š ¨ª AP, AB, AC ·¹  0
x x1 y y1 z z1
š x2 x1 y2 y1 z2 z1  0
x3 x1 y3 y1 z3 z1

The points ( xr , yr , zr ) (where r  1, 2, 3 and 4) are coplanar if and only if


x4 x1 y4 y1 z4 z1
x2 x1 y2 y1 z2 z1  0
x3 x1 y3 y1 z3 z1

T H E O R E M 6.50 The distance of the plane ax by cz d  0 from a point A( x1 , y1 , z1 ) is


ax1 by1 cz1 d
a 2 b2 c 2
E E
PROOF Let E be the given plane and r – n  p be its equation. Let B(b) be a point which is not equal to N
E E
(see Fig. 6.30) where N is the foot of the perpendicular from A onto E so that b – n  p. Therefore
HHHHE HHHHE
HHHHE HHHHE AB – AN
AN  Projection of AB on AN  HHHHE
AN
E E E
(b a ) – n
 E
n
HE E E
p a–n
 E
n
414 Chapter 6 Three-Dimensional Geometry

E E
Now, a  ( x1 , y1 , z1 ), n  (a, b, c) and p  d imply
ax1 by1 cz1 d
AN 
a 2 b2 c 2

A (x1, y1, z1)

90°
B N

FIGURE 6.30

T H E O R E M 6.51 The equation of any two parallel planes are of the form ax by cz d1  0 and ax by cz d2  0
and the distance between them is equal to
d1 d2
a b2 c 2
2

PROOF Let
E1 y a1 x b1 y c1z d1  0
and E2 y a2 x b2 y c2 z d2  0
E E
be two parallel planes so that n1  (a1 , b1 , c1 ) and n2  (a2 , b2 , c2 ) are the normal vectors to
E E
E1 and E2 , respectively (see Quick Look 7). Since E1 and E2 are parallel, their normals n1 and n2
HHE HE H
are parallel. Hence, let n2 =kn1 so that a2  ka1 , b2  kb1 and c2 = kc1 . Thus, the equation of E2 is
k(a1 x b1 y c1z) d2  0. Dividing by k, we have E2 y a1 x b1 y c1z d2  0 (here, in the place of
d2 / k, we take d2). Let E1 y ax by cz d1  0 and E2 y ax by cz d2  0 be the two parallel
planes. Let A( x1 , y1 , z1 ) be a point in E2 so that
ax1 by1 cz1 d2  0 (6.11)
Now,
Distance between E1 and E2  Distance of E1 from point A
ax1 by1 cz1 d1
 (Theorem 6.49)
a 2 b2 c 2
d2 d1
 [by Eq. (6.11)]
a 2 b2 c 2

Notation: If E denotes ax by cz d, then we denote ax1 by1 cz1 d by E11 and ax2 by2 cz2 d2 by E22.

T H E O R E M 6.52 The ratio in which the plane E y ax by cz d  0 divides the segment joining the points
A( x1 , y1 , z1 ) and B( x2 , y2 , z2 ) is –E11:E22.
PROOF See Fig. 6.31. Let E y ax by cz d  0 be the plane meeting the line AB at P and suppose
AP : PB  L1 : L 2 so that
¥ L x L 2 x1 L1 y2 L 2 y1 L1z2 L 2 z1 ´
P¦ 1 2 , ,
§ L1 L 2 L1 L 2 L1 L 2 µ¶
6.3 Plane 415

Since P E, we have

¥ L x L 2 x1 ´ ¥ L x L 2 x1 ´ ¥ L x L 2 x1 ´
a¦ 1 2 µ b¦ 1 2 µ c¦ 1 2 d0
§ L 1 L 2 ¶ § L 1 L 2 ¶ § L1 L 2 µ¶
 L1 (ax2 by2 cz2 d2 ) L 2 (ax1 by1 cz1 d)  0
L1 (ax1 by1 cz1 d) E11
  
L2 ax2 by2 cz2 d E22

A P B A B P

(a) (b)

FIGURE 6.31

Note
1. Points A and B lie on the opposite sides of the plane E  0 š the ratio –E11:E22 is positive š E11 and E22 should have
opposite signs. A and B lie on the same side of E š the ratio −E11:E22 < 0 š E11 and E2 should have the same sign.
2. Let O  (0, 0, 0), A  ( x1 , y1 , z1 ) and E y ax by cz d  0, then A lies on non-origin side of the plane E  0 š
E11 and E22  d have opposite signs and A lies on the origin side š E11 , d have the same sign.
For example, if E y x 2 y 3z 1 and A  (1, 2, 3), then E11  1 4 6 1  10 and d  1 so that the point A lies on the
non-origin side of E  0.

6.3.1 System of Planes


We have discussed that the first-degree equation ax by cz d  0 represents a plane. This equation is called general
equation of a plane. Since one of a, b and c is not equal to zero, we can consider a x 0. Then, the plane equation can be
written as
b c d
x y z  0
a a a
Hence, if the values of b /a, c /a and d /a are given, then we can determine the plane equation. If the three ratios
b /a, c /a and d /a are to be determined uniquely, we need three conditions. If lesser number of conditions is given, then
the ratios b /a, c /a and d /a can be determined in many ways. The planes so obtained are called system of planes.
For example, if we fix a, b and c and vary d, then ax by cz d  0 represents system of parallel planes. For all those
planes (a, b, c) is the normal. If ( x1 , y1 , z1 ) is a fixed point, then for different values of a, b and c, the equation
a( x x1 ) b( y y1 ) c(z z1)  0
represents the planes passing through the point ( x1 , y1 , z1 ).

T H E O R E M 6.53 Let E1 y a1 x b1 y c1z d1  0, E2 y a2 x b2 y c2 z d2  0 be the two intersecting planes which


intersect in a line L (since there are two planes, when they intersect, they intersect in a line). If
L1 and L 2 are any two real numbers such that at least one of them is not zero, then the equation
L1 E1 L 2 E2  0 represents plane passing through the line L.
PROOF Let
E y L1 E1 L 2 E2  (L1a1 L 2 a2 ) x (L1b1 L 2 b2 ) y (L1c1 L 2 c2 )z (L1d1 L 2 d2 )  0
Now
L1a1 L 2 a2  L1b1 L 2 b2  L1c1 L 2 c2  0
 a1 : a2  b1 : b2  c1 : c2  L1 : L 2
416 Chapter 6 Three-Dimensional Geometry

Therefore, E1  0 and E2  0 are parallel planes, which is not true in this case. Therefore
E y L1 E1 + L 2 E2  0
is a first-degree equation and hence it represents a plane. Since the line L is contained in both
E1  0 and E2  0, it is also contained in the plane represented by L1 E1 + L 2 E2  0.

QUICK LOOK 9

L1 E1 + L 2 E2  0 represents E1  0, if L 2  0 and L1  1, can take the equation of any plane other than E1 and
and represents E2  0, if L1  0 and L 2  1. Hence, we E2 passing through the line L as E1 + L E2  0 (K is a
parameter).

Theorem 6.54 is the converse of Theorem 6.53.

T H E O R E M 6.54 If E1 y a1 x b1 y c1z d1  0, E2 y a2 x b2 y c2 z d2  0 are two intersecting planes which inter-


sect in a line L, then the equation of any plane passing through line L is of the form L1 E1 L 2 E2  0
where | L 1| |L 2 | x 0.
E E
PROOF Suppose E y px qy rz t  0 is a plane containing the line L. Let n1  (a1 , b1 , c1) and n2  (a2 , b2 , c2 )
E E
be the normals to E1  0 and E2  0, respectively. Hence, n1 s n2 is parallel to the line L so that
(b1c2 b2 c1 , c1a2 c2 a1 , a1b2 a2 b1 ) are the DRs of the line L. Since one of the DRs is not zero, we
consider that a1b2 a2 b1 x 0. Therefore, the equations a1 x a2 y  p and b1 x b2 y  q are uniquely
solvable for a non-zero solution and let x  L1 and y  L 2 be the solution. Therefore,
a1 L1 a2 L 2  p and b1 L1 b2 L 2  q. Let k  r L1c1 L 2 c2 and l  t L1d1 L 2 d2 so that
E y L1(a1 x b1 y c1z d1 ) L2 (a2 x b2 y c2 z d2 ) kz l  0 (6.12)
Suppose P ( x1 , y1 , z1 ) and Q ( x2 , y2 , z2 ), where z1 x z2 , are points on line L. Therefore,
a1 x1 b1 y1 c1z1 d1  0
and a2 x1 b2 y1 c2 z1 d2  0
so that point P( x1 , y1 , z1 ) satisfies the equation E  0 and similarly point Q ( x2 , y2 , z2 ) satisfies
E  0 [from Eq. (6.12)]. Therefore, kz1 l  0, kz2 l  0 and z1 x z2  k = 0 and then l  0. Hence,
the equation of the plane E  0 is of the form L1 E1 L 2 E2  0.

T H E O R E M 6.55 Let E1 y a1 x b1 y c1z d1  0 and E 2 y a2 x b2 y c2 z d2  0 be the two intersecting planes.


Then, the equations of the planes bisecting between the angles are
a1 x b1 y c1z d1 a2 x b2 y c2 z d2
p
2 2 2
a b c a 2 b 2 c2

PROOF We have
P( x, y, z) is a point on a bisecting plane of the given planes
š The distance of P from both the planes are equal
a1 x b1 y c1z d1 a2 x b2 y c2 z d2
š  (by Theorem 6.12)
a 2 b 2 c2 a 2 b 2 c2
a1 x b1 y c1z d1 a2 x b2 y c2 z d2
š p
2 2 2
a b c a 2 b 2 c2
6.3 Plane 417

QUICK LOOK 10

To determine the plane bisecting the acute angle between the plane E and one of E1 and E2 and one of
between the planes, consider one of the planes E1 or the bisecting planes, say, is less than Tan 1 1, then E is
E2 and one of the bisecting planes, say, E. If the angle the acute angle bisector plane.

Note: As in the case of straight lines, on the similar lines, we can find acute and obtuse angle bisecting planes and
plane bisecting the origin angle (see Theorems 2.25 and 2.26 and Quick Look 14, Chapter 2).

Example 6.19

Find a point on x-axis which is equidistant from points  ( x 4)2 ( 3)2 7 2  ( x 2)2 ( 1)2 12
A (4, 3, 7)and B (2, 1, 1) and also find the equation of
the plane passing through this point and perpendicular  4 x  68
to the line AB.  x  17
Hence, P (17, 0, 0). Now, from Quick Look 8, the equation
Solution: Let P ( x, 0, 0) be a point on the x-axis.
of the plane passing through P (17, 0, 0) and perpendicular
Therefore, HHHHE
to AB  ( 2, 2, 6) is 2( x 17) 2( y 0) 6( x 0)  0
PA  PB
 2 x 2 y 6z 34  0
 ( PA)2  ( PB)2  x y 3z 17  0

Example 6.20

Find the equation of the plane passing through the point E E HE


i j k
( 1, 3, 2) and perpendicular to both the planes E E HE
E1 y x 2 y 2z 5  0 and E2 y 3 x 3 y 2z 8  0. 1 2 2  2i 4 j 3k
3 3 2
Solution: Let E  0 be the required plane. Since E is
perpendicular to both E1  0 and E2  0, its normal is Therefore
parallel to the cross product of the normals of E1  0 and E y 2 ( x 1) 4( y 3) 3(z 2)  0
E2  0. Therefore, the normal of E is
y 2 x 4 y 3z 8  0
y 2 x 4 y 3z 8  0

Example 6.21

Two sets of axes have the same origin. If a plane makes Since the perpendicular distance from origin onto the
intercepts a, b and c on one set of the axes and a1 , b1 and c1 plane is the same, we have (see Theorem 6.50)
on another set of the axes, then show that
1 1
a 2 b 2 c 2  a1 2 b1 2 c1 2 . 
1 1 1 1 1 1

Solution: Let E be the given plane. Therefore, its equa- a 2 b2 c 2 a12 b12 c12
tions are
1 1 1 1 1 1
x y z  
1 a2 b2 c2 a12 b12 c12
a b c
x y z
and 1
a1 b1 c1
418 Chapter 6 Three-Dimensional Geometry

Example 6.22

Find the equation of the plane passing through the Since it passes through (2, 1, 3), we have
point (2, 1, 3) and parallel to the plane E1 y 3 x 4 y 7z
5  0. 3(2) 4( 1) 7(3) d  0
 d  19
Solution: Let E be the required plane. Since E is par-
allel to the plane E1  0, E is of the form So
E y 3 x 4 y 7z 19  0
3 x 4 y 7z d  0

Example 6.23

A plane intersects the coordinate axes in A, B and C, and the equation of the plane is
respectively. If (A , B, G ) is the centroid of $ABC, then
x y z
show that the equation of the plane 1 (6.13)
a b c
x y z
3 Since (A , B, G ) is the centroid, we have
A B G
a b c
A ,B ,G 
3 3 3
Solution: Let A  (a, 0, 0), B  (0, b, 0) and C  (0, 0, c)
 a  3A , b  3B, c  3G
so that
so that from Eq. (6.13), the equation of the planes is
¥ a b c´
G¦ , , µ
§ 3 3 3¶ x y z
3
A B G

Example 6.24

If the sum of the squares of the intercepts of a variable x y z


plane on the coordinate axis is constant k2, then show  1
(x12 y12 z12 )/x1 (x12 y12 z12 )/y1 (x12 y12 z12 )/z1
that the locus of the foot of the perpendicular drawn
from the origin onto the plane is Therefore, the intercepts of the plane on the axes are
2 2 2 2 2
(x y z ) (x y 2 z 2)  k2 x12 y12 z12 x12 y12 z12 x12 y12 z12
, ,
x1 y1 z1
Solution: Let E be the variable plane and a, b and c
be the intercepts of the plane E on the coordinate axes. Hence by Eq. (6.14), we have
Therefore, by hypothesis, we have
( x12 y12 z12 )2 ( x12 y12 z12 )2 ( x12 y12 z12 )2
2 2
a b c k 2 2
(6.14)  k2
x12 y12 z12
Let P( x1 , y1 , z1 ) be the foot of the perpendicular from
the origin onto E. Hence, by Quick Look 8, the equation Therefore, the locus is
of the plane is
¥ 1 1 1´
x1 ( x x1 ) y1 ( y y1 ) z1 (z z1 )  0 ( x 2 y 2 z2 )2 ¦ 2 2 2 µ  k 2
§x y z ¶
 xx1 yy1 zz1  x12 y12 z12

Example 6.25

A variable plane is moving such that its distance from C, respectively. Show that the locus of the centroid of
the origin is 3p (constant) and meets the axes at A, B and $ ABC is x 2 y 2 z 2  p 2 .
6.3 Plane 419

Solution: Let A  (a, 0, 0), B  (0, b, 0), C  (0, 0, c). Let By hypothesis, we have
G = (x1, y1, z1) be the centroid so that 1
 3p
a b c (1 / 9 x12 ) (1 / 9 y12 ) (1 / 9z12 )
x1  , y1  , z1 
3 3 3
1 1 1 1
 
The equation of the plane is 9 x12 9 y12 9z12 9 p2
x y z
1 Therefore, the locus of G( x1 , y1 , z1 ) is
a b c
so that 1 1 1 1
2
2
2

x y z p2
x y z
1
3 x1 3 y1 3z1

Example 6.26

A variable plane is passing through a fixed point (a, b, c) Since this plane is passing through (a, b, c), we have
and meeting the coordinate axes at A, B and C . Then
a b c
show that the locus of the point of intersection of the 1 (6.15)
planes through A, B and C and parallel to the coordinate A B G
planes is Now, the equation of the plane passing through A(A , 0, 0)
a b c and parallel to yz-plane is x  A . Similarly, the other two
1 planes are y  B and z  G . Since the intersection of these
x y z
planes is (A , B, G ), from Eq. (6.15), the locus of the point
Solution: Let A  (A , 0, 0), B  (0, B, 0) and C  (0, 0, G ) (A , B, G ) is
so that equation of the plane is a b c
1
x y z x y z
1
A B G

Example 6.27

P is a variable point in the plane Since P belongs to the plane


x y z x y z
1 1
a b c a b c
HHHE
The plane passing through P and perpendicular to OP we have
(O is the origin) meets the coordinate axes at A, B and C.
A B G
Show that the equation to the locus of the point of inter- 1 (6.17)
section of the planes through A, B and C and parallel to a b c
the coordinate planes is The plane represented by Eq. (6.16) meets the coordi-
nates axes at
x 2 y 2 z 2  (ax) 1 (by) 1 (cz) 1
¥ A 2 B2 G 2 ´
Solution: Suppose P  (A , B, G ). Therefore, the equa- A¦ , 0, 0µ
§ A ¶
E the plane passing through P and perpendicular
tionHHHof
to OP is ¥ A 2 B2 G 2 ´
B ¦ 0, ,G µ
A ( x A ) B( y B ) G ( x G )  0 § B ¶

 A x By G z  A 2 B2 G 2 (6.16) ¥ A 2 B2 G 2 ´
and C ¦ 0, 0, µ
§ G ¶
420 Chapter 6 Three-Dimensional Geometry

Hence, the equations of the planes passing through A, B A 2 B2 G 2


and C and parallel to coordinate planes are and z1 
G
A 2 B2 G 2 Therefore, from Eq. (6.17), we have
x
A
1 1 1 (A / a) (B / b) (G / c) 1
A B2 G 2
2
 2 2 2
 2
y ax1 by1 cz1 A B G A B2 G 2
B
Also
A 2 B2 G 2
and z 1 1 1 A 2 B2 G 2 1
G   2
x12 y12 z12 (A B G ) A B 2 G 2
2 2 2 2
Suppose the point of intersection of these three planes is
Q( x1 , y1 , z1 ) so that Therefore
A 2 B2 G 2 1 1 1 1 1 1
x1  
A x12 y12 z12 ax1 by1 cz1
A B2 G 2
2
Hence, the locus of Q( x1 , y1 , z1 ) is
y1 
B
1 1 1 1 1 1
2
2
2

x y z ax by cz

Example 6.28

A plane E is intersecting three mutually perpendicular a2


planes on the sides AB, BC and CA of $ABC . If the  cot A 
angles between the plane E and the three planes are @, A b2 c 2 c 2 a 2 a 2 b2
and F, respectively, then prove that cos2 G  cot B cot C ,
Similarly
cos2 B  cot C cot A and cos2A  cot A cot B.
b2
Solution: Without loss of generality, we can consider cot B 
b2 c 2 c 2 a 2 a 2 b2
that the three mutually perpendicular planes are the co-
ordinate planes. Suppose the equation of the plane E is c2
and cotC 
x y z b2 c 2 c 2 a 2 a 2 b2
1
a b c
If @ is the angle between the plane E and yz-plane, then
so that A  (a, 0, 0), B  (0, b, 0) and C  (0, 0, c). Hence, the
HHHHE HHHHE (1 / a) 0 0 bc
DRs of AB and AC , respectively, are ( a, b, 0) and cos A  
2 2 2
( a, 0, c). Therefore, from the dot product and cross prod- (1 / a ) (1 / b ) (1 / c ) b c c 2 a 2 a 2 b2
2 2

uct of vectors, we have


Therefore
2 2 2 2 2 2
b c c a a b b2 c 2
tan A  2 cos2 A   cot B cot C
a b2 c 2 c 2 a 2 a 2 b2
Similarly, we can prove the other parts.

Example 6.29

Let E1 y 2 x 7 y 4z 3  0 and E2 y 3 x 5 y 4z 11  0 Solution: Let the required plane be E. Then by Theo-


be two intersecting planes. Find the equation of the plane rem 6.54, we have E  E1 L E2 for some K. That is,
passing through the intersecting line of E1  0 and E2  0
E y  2 x 7 y 4z 3 L  3 x 5 y 4z 11  0
and also through the point (−2, 1, 3).
6.4 Line 421

Now, E passes through the point ( 2, 1, 3). So Therefore, the plane

( 4 7 12 3) L ( 6 5 12 11)  0 1
E y (2 x 7 y 4z 3) (3 x 5 y 4z 11)  0
6
 2 12 L  0
 E y 15 x 47 y 28z 7  0
1
L 
6

Example 6.30
Find the equation of the plane passing through the inter- E  0 is perpendicular to the plane 5 x 3 y 6z 8  0. So
section of the planes E1 y x 2 y 3z 4  0 and their normals are at right angles. This means
E2 y 2 x y z 5  0 and perpendicular to the plane
5 x 3 y 6 z 8  0. 5(1 2 L ) 3 – (2 L ) 6(3 L )  0
 7 L  29
Solution: Let E y E1 L E2  0 be the required plane. 29
That is, L
7
E y (1 2 L ) x (2 L ) y (3 L )z 4 5L  0 Therefore, E y 51x 15 y 50z 173  0.

Example 6.31

Let E1 y x 2 y 2z 9  0 and E2 y 4 x 3 y 12z 13  0 1326


be two planes. Find the following planes: (a) The bisect-  sec Q 
17
ing planes of the angle between E1 and E2. (b) The acute
angle bisector plane of E1 and E2. (c) The angle which Now
contains the origin. 1326 1027
tan 2 Q  sec 2 Q 1  1
289 289
Solution:
1027
(a) Bisecting planes are  tan Q  1
17
x 2 y 2z 9 4 x 3 y 12z 13
p Therefore, Eq. (6.18) represents the obtuse angle
12 2 2 2 2 4 2 ( 3)2 12 2 bisecting plane and hence Eq. (6.19) represents
acute angle bisecting plane.
That is, (c) The constant terms of the given planes are −9 and
x 35 y 10z 156  0 (6.18) 13. Multiply E1  0 with (−1) so that
and 25 x 17 y 62z 78  0 (6.19) E1 y x 2 y 2z 9  0
(b) Let P be the angle between E1 and the plane pro- and E2 y 4 x 3 y 12z 13  0
vided in Eq. (6.18). Therefore
Now,
1(1) 2(35) 2( 10) 17 a1a2 b1b2 c1c2  4 6 24  18  0
cos Q  
2 2 2 2 2 2 1326
1 2 2 1 35 10 Hence, the origin lies in the acute angle region.

6.4 Line
It is said that if two planes intersect, they intersect in a straight line (Axiom 6.4) and the equation of a plane is a first-
degree equation in x, y and z. Hence, a straight line can be represented by a pair of intersecting planes. For example,
x-axis is the intersection of xy-plane and zx-plane and hence x-axis equations are y  0 and z  0. Similarly, y-axis
equations are z  0 and x  0 and z-axis equations are x  0 and y  0.
422 Chapter 6 Three-Dimensional Geometry

T H E O R E M 6.56 The equations of the line passing through the point A( x1 , y1 , z1 ) (see Fig. 6.32) and having the
(P A R A M E T R I C DCs (l, m, n) are x  x1 lr, y  y1 mr and z  z1 nr, where r is real.
FORM OF
A STRAIGHT
LINE)
E E
PROOF It is known that the equation of the line passing through the point a and parallel to a vector b is
E E E E E E H
E
r  a tb, t Š Z (see Theorem
E 5.27, p. 333, Vol. 2). Now, take a  ( x1 , y1 , z1 )  x1 i y1 j z1 k and
E E E E E E HE
b  (l, m, n)  li m j nk. Let r  ( x, y, z)  xi yj zk be any point on the line. Then
E E HE E E HE E E HE
xi y j zk  ( x1 i y1 j z1 k ) t(li m j nk )
 x  x1 tl, y  y1 tm, z  z1 tn
Replacing t by r, we have
x  x1 lr, y  y1 mr and z  z1 nr

(l, m, n)

A (x1, y1, z1) P (x, y, z)

FIGURE 6.32

Before pronouncing the symmetric form of a line we adopt the following convention.
Convention: Let a1 , a2 ,!, an and b1 , b2 , !, bn be real numbers. We write
a1 a2 a
 !  n
b1 b2 bn
only to mean that a1 : a2 : a3 : ! : an  b1 : b2 : b3 : ! : bn . If some bi’s are zeros, then we mean that the corresponding ai’s
are zeros. In this case the parametric form of the line can be written as
x x1 y y1 z z1
 
l m n
which is called the symmetric form of a line.

QUICK LOOK 11 (SYMMETRIC FORM OF A LINE)

The equation of the line passing through the point Here, DCs may be replaced DRs as well. Also
( x1 , y1 , z1 ) having (l, m, n) as DCs is
( x x1 )2 ( y y1 ) 2 (z z1 )2  r 2
x x1 y y1 z z1
  Hence, r gives us the distance of ( x, y, z) from the
l m n
given point ( x1 , y1 , z1 ).

T H E O R E M 6.57 If E1 y a1 x b1 y c1z d1  0 and E2 y a2 x b2 y c2 z d2  0 be the two intersecting planes where


L is their line of intersection, then determine the symmetric form of the line L.
E E
PROOF Let n1  (a1 , b1 , c1 ) and n2  (a2 , b2 , c2 ) which are the normals of E1  0 and E2  0, respectively.
E E
Hence, the line L is parallel to n1 s n2 so that the DRs of the line L are (b1c2 b2 c1 , c1a2 c2 a1 , a1b2
a2b1). Now, we find a point on L. Since one of the DRs is not zero, we consider that (a1b2 a2 b1 ) x 0.
Therefore, the equations a1 x b1 y  d1 and a2 x b2 y  d2 have unique solutions
6.4 Line 423

b1d2 b2 d1
x
a1b2 a2 b1

d1a2 d2 a1
and y
a1b2 a2 b1
so that the point (x, y, 0) lies on L. Therefore, the symmetric form of L is
x [(b1d2 b2 d1 ) / (a1b2 a2 b1 )] y [(d1a2 d2 a1 ) / (a1b2 a2 b1 )] z 0
= 
b1c2 b2 c1 c1a2 c2 a1 a1b2 a2 b1

T H E O R E M 6.58 The plane E y ax by cz d  0 contains the line


x x1 y y1 z z1
L:  
l m n
if and only if (a) al bm cn  0 and (b) ax1 by1 cz1 d  0.
PROOF Suppose the line L is contained in the given plane. Since ( x1 , y1 , z1 ) lies on L and L is contained
in E = 0 we have
ax1 by1 cz1 d  0
Also the normal (a, b, c) of E  0 is perpendicular to the line L. This implies
(a, b, c) – (l, m, n)  0
 al bm cn  0
Conversely, assume that
ax1 by1 cz1 d  0 (6.20)
and al bm cn  0 (6.21)
Since one of a, b and c is not zero, we have that the vector (a, b, c) is normal to the plane
E y ax by cz d  0. Also from Eqs. (6.20) and (6.21), we have that ( x1 , y1 , z1 ) lies in
E  0 and al bm cn  0 which implies that (l, m, n) is perpendicular to the normal (a, b, c) or E
 0. Hence, L must lie in the plane E  0.

QUICK LOOK 12

1. To show that a line lies in a plane, it is enough if we x x1 y y1 z z1


show one point of the line belongs to the plane and  
l m n
the normal to the plane is perpendicular to the line.
is
2. If al bm cn  0 then the equation of the plane
containing the line a( x x1 ) b( y y1 ) c(z z1 )  0
In the relation al bm cn  0, if a, b and c vary, then
we obtain system of planes containing the line.

6.4.1 Discussion on Nature of Three Planes


If 01 , 0 2 and 0 3 are three planes, any one of the following three cases may arise:
Case 1: All three planes 01 , 0 2 and 0 3 are parallel to each other (i.e., their normals are parallel).
Case 2: Two planes are parallel and the third plane intersects them in parallel lines (see Fig. 6.3, Theorem 6.12).
Case 3: No two planes of 01 , 0 2 and 0 3 are parallel. In this case, there are three sub-cases:
424 Chapter 6 Three-Dimensional Geometry

1. All three planes interest at a point. That is, the three lines of intersection of the planes, which are taken
pairwise, are concurrent [see Fig 6.6(a), Theorem 6.18]. Example: every corner of a room is a point of in-
tersection of three planes.
2. All three planes intersect in a single line [see Fig 6.6(b), Theorem 6.18].
3. Every two planes intersect in a line which is parallel to the third plane. In this case, the planes form a
triangular prism [see Fig 6.6(c), Theorem 6.18]. That is, if 01 † 0 2  L1 , 0 2 † 0 3  L2 and 0 3 † 01  L3 ,
then L1 0 3 , L2 01 and L3 0 2 . Suppose the equation of the planes 01 , 0 2 and 0 3 are
E1 y a1 x b1 y c1z d1  0 (6.22)
E2 y a2 x b2 y c2 z d2  0 (6.23)
E3 y a3 x b3 y c3 z d3  0 (6.24)
If a1 : a2 : a3 :  b1 : b2 : b3  c1 : c2 : c3 , then the three planes are parallel. Suppose 01 and 0 2 are parallel
and they interest 0 3 so that L2 and L3 are parallel. Then, E2  0  E3 represents L2 and E1  0  E3
represents L3 .
In the above three cases, let us discuss Case 3 (when no two planes out of 01 , 0 2 and 0 3 are parallel) in detail. Let us
consider that
a1 b1 c1
$  a2 b2 c2
a3 b3 c3
d1 b1 c1
$ 1  d2 b2 c2
d3 b3 c3
a1 d1 c1
$ 2  a2 d2 c2
a3 d3 c3
a1 b1 d1
and $ 3  a2 b2 d2
a3 b3 d3

1. Suppose $ x 0. Then, by Cramer’s rule (Theorem 8.46, p. 413, Vol. 1), the three Eqs. (6.22)–(6.24) have unique solu-
tion and hence the three planes are concurrent at a point.
2. Suppose $  0 and $ 3  0. Since 0 2 † 0 3  L2 the DRs of L2 are
(a2 , b2 , c2 ) s (a3 , b3 , c3 )  (b2 c3 b3 c2 , c2 a3 a2 c3 , a2 b3 a3 b2 )
Now,
$  0  a1 (b2 c3 b3 c2 ) b1 (c2 a3 c3 a2 ) c1 (a2 b3 a3 b2 )  0
So the line L2 is parallel to the plane 01 .
Also if a2 b3 a3 b2 x 0, then by Theorem 6.57, the point
¥ b2 d3 b3 d2 d2 a3 d3 a2 ´
¦§ a b a b , a b a b , 0µ¶
2 3 3 2 2 3 3 2

lies on L2 . Therefore
¥ b d b d ´ ¥ d a d a ´ $3
a1 ¦ 2 3 3 2 µ b1 ¦ 2 3 3 2 µ c1 (0) d1  0 (& $ 3  0)
§ a2 b3 a3 b2 ¶ § a2 b3 a3 b2 ¶ a2 b3 a3 b2

That is, a point on the line L2 also lies in the plane 01 and L2 is parallel to 01 so that L2 lies in 01 . Therefore
L1  L2  L3 . Similarly, $  0  $ 1  0 and $  0  $ 2  0.
6.4 Line 425

3. Suppose $  0, $ 3 x 0. $  0 implies that the line L2 is parallel to 01 . Also


$3
$3 x 0  x0
a2 b3 a3 b2

so that the line L2 does not lie in 01 . In this case, triangular prism is formed.

QUICK LOOK 13

If $  0, then either all $ j (where j  1, 2 and 3) are zero or all $ j (where j  1, 2 and 3) are non-zero.

6.4.2 Method to Solve Problems on Three Planes


1. Check whether all three planes are parallel. If all the three planes are parallel, then no common solution exists for
the three equations.
2. If two planes are parallel and the third intersects both in parallel lines, then there exists no common solution.
3. If $ x 0, then all three planes meet at a unique point.
4. If $  0 and one of $ 1 , $ 2 and $ 3 is not zero, then the three planes form a triangular prism.
5. If $  0 and $ 1  $ 2  $ 3  0, then the equations have infinitely many solutions and the three planes intersect in
a single line.

Example 6.32

Discuss the nature of the following three planes: 2. We have


1. 2 x y z 4  0, y z 4  0, 3 x 2 y z 8  0 4 5 2
 4 ( 32 4) 5(40 4) 2(10 8)
2. 4 x 5 y 2z 2  0, 5 x 4 y 2z 2  0, 2 x 2 y $  5 4 2
 144 180 36  0
8z 1  0 2 2 8
3. 2 x 3 y z 2  0, 3 x 3 y z 4  0, x y 2z 5  0 2 5 2
 2( 32 4) 5(16 2) 2(4 4)
$ 1  2 4 2
Solution:  72 90  162 x 0
1 2 8
1. We have
Hence, the three planes form a triangular prism (in
2 1 1 this case, the system of equation has no common solu-
$  0 1 1  2(1 2) 1(0 3) 1(0 3)  6 6  0 tion).
3 2 1 3. We have
4 1 1 1 1 1 2 3 1
 2(6 1) 3(6 1) 1( 3 3)
$ 1  4 1 1  4 2 1 1  4(0)  0 $ 3 3 1
 14 15 6  5 x 0
8 2 1 2 2 1 1 1 2
Hence, the three planes intersect in a single line (thus, Hence, the three planes are concurrent at a point (i.e.,
the three equations have infinitely many common the three equations have unique solution).
solutions).

T H E O R E M 6.59 The equation of the plane passing through the line


x x1 y y1 z z1
 
l1 m1 n1
426 Chapter 6 Three-Dimensional Geometry

x x2 y y2 z z2
and parallel to the line  
l2 m2 n2
is
x x1 y y1 z z1
l1 m1 n1  0
l2 m2 n2

PROOF Let L1 and L2 be the given lines and A  ( x1 , y1 , z1 ) (see Fig. 6.33). Let
E E E E
n1  (l1 , m1 , n1 ) and n2  (l2 , m2 , n2 ) so that n1 s n2 is perpendicular to the required plane. Now

P( x, y, z) is any point in the plane


HHHHE E E
š AP – (n1 s n2 )  0
x x1 y y1 z z1
š l1 m1 n1  0
l2 m2 n2

L2

A (x1, y1, z1)


L1
E
FIGURE 6.33

QUICK LOOK 14

The equation of the plane containing both the lines x x1 y y1 z z1


x x1 y y1 z z1 x x2 y y2 z z2 l1 m1 n1  0
  and  
l1 m1 n1 l2 m2 n2 l2 m2 n2
is

Example 6.33

If the points A(a, b, c), B(a a, ba, c a) and the origin are col- a a ba c a
  
linear, then show that a b c
aa a bba cc a  a 2 b2 c 2 a a 2 ba 2 c a 2 a a ba c a
   L (say)
FHHHE a b c
Solution: The equation of the line AB is Therefore
x a y b z c (a 2 b2 c 2 )(a a 2 ba 2 c a 2 )  L 2 (a 2 b2 c 2 )2
 
a a a ba b c a c  (L aa L bb L cc)2
This passes through origin which means  (a aa ba b c ac) 2
a b c Hence
 
a a a b ba c c a
a a a b ba c c a a a a b a b c ac  a 2 b 2 c 2 – a a 2 b a 2 c a 2
  
a b c
6.4 Line 427

Example 6.34

Find the shortest distance and the equation of the line con- HHHE L1 and L2 , respectively. The
Let the given skew lines Fbe
taining the shortest distance segment of the skew lines plane containing L1 and LM is
x 3 y 4 z 2 x 3 y 4 z 2
 
1 2 1 1 2 1 0
x 1 y 7 z 2
and   1 3 2
1 3 2
 13 x 4 y 5z 45  0 (6.25)
Solution:
E See Fig. 6.34. We HE have A  (3, 4, 2), E HE
b  ( 1, 2, 1), C  (1, 7, 2) and d  (1, 3, 2). Let LM be Here we have used (1, 3, 5)  b s d. HHHHE
the shortest distance between the lines. Hence, by Theo- Also, the equation of the plane containing L2 and LM is
rem 6.44, p. 413, Vol. 2, we have x 1 y 7 z 2
HHHHE E HE
AC – (b s d) 1 3 2 0
LM  E HE  35 1 3 5
bsd
 3 x y 10  0 (6.26)
L
A (3, 4, −2) Intersection
FHHHE of the planes provided in Eqs. (6.25) and
(6.26) is LM . Hence, the equation of the line containing
the shortest distance segment is
13x 4y 5z 45  0  3 x y 10
M

C (1, −7, −2)

FIGURE 6.34

Example 6.35

Suppose 5 x y z  0  x 2 y z 3 and 7x 4y 2z  0  Now,


x y z 3 represent skew lines. Find the shortest dis- 5 L (1 2 L )
tance between them and also find the equation of the   10 L 4 M LM  13 (6.31)
7 M (4 M )
shortest distance.
1 2L L 1
  8 L 2 M LM  2 (6.32)
Solution: Let the lines 4 M M 2
5x y z  0  x 2 y z 3 (6.27) 5 L L 1
  9L 6 M  3 (6.33)
and 7 x 4 y 2z  0  x y z 3 (6.28) 7 M M 2
be L1 and L2, respectively. The equation of any plane Adding Eqs. (6.31) and (6.32), we get
passing through L1, by Theorem 6.54, is
18 L 6 M  15
(5 x y z) L ( x 2 y z 3)  0 (6.29)
 6L 2 M  5 (6.34)
and the equation of the plane passing through L2 is
Solving Eqs. (6.33) and (6.34) for L and M, we have
(7 x 4 y 2z) M( x y z 3)  0 (6.30) L  2 and M  7 / 2. Substituting the values of L and M in
Eq. (6.29), we get the planes passing through L1 and L2 ,
Equations (6.29) and (6.30) represent parallel planes.
which are parallel to each other, as
This means
7 x 5y z 6  0 (6.35)
5 L 1 2L L 1
 
7 M 4 M M 2 and 7 x 5y z 7  0 (6.36)
428 Chapter 6 Three-Dimensional Geometry

The distance between the planes provided in Eqs. (6.35) Also the planes represented by Eq. (6.30) is perpendicu-
and (6.36) is lar to the plane represented by Eq. (6.27). So
6 7 13 7(7 M ) 5( 4 M ) 1( M 2)  0

7 2 ( 5)2 12 75  13M  67

The planes represented by Eqs. (6.29) and (6.35) are at 67


M
right angles. This implies 13

7(5 L ) 5( 1 2 L ) 1(L 1)  0 Substituting the value of M  67 / 13 in Eq. (6.30), we


have
13
L 
6 8 x 5 y 31z 67  0 (6.38)
That is the equation of the plane passing through L1 and which is the plane passing through the line L2 and per-
perpendicular to the plane provided in Eq. (6.35) is pendicular to the plane represented by Eq. (6.30). From
Eqs. (6.37) and (6.38), the equation of the line having the
¥ 13 ´ ¥ 26 ´ ¥ 13 ´ 39 shortest distance is
¦§ 5 µ¶ x ¦§ 1 µ¶ y ¦§ 1µ z  0

6 6 6 6
17 x 20 y 19z 39  0  8 x 5 y 31z 67
 17 x 20 y 19z 39  0 (6.37)

Example 6.36
Find the equation of the line intersecting the lines is parallel to the line
2 x y 1  0  x 2 y 3z, 3x y z 2  0  4 x 5 y 2z 3
x 1 y 2 z 3
x 1 y 2 z 3  
and parallel to the line   . 1 2 3
1 2 3
Therefore
Solution: The required line equation is
1(2 L ) 2(1 2 L ) 3(3L )  0  1(3 4 M ) 2 ( 1 5M )
(2 x y 1) L ( x 2 y 3z)  0
3(1 2 M )
 (3 x y z 2) M(4 x 5 y 2z 3)  6 L  4 and 8M = 4
so we have 2 1
L  and M =
(2 L ) x (1 2 L ) y 3L z 1  0 3 2
 (3 4 M ) x ( 1 5M ) y (1 2 M )z 2 3M (6.39) Substituting the values of L  2 / 3 and M  1/ 2 in Eq.
(6.39), the required line equation is
4 x 7 y 6z 3  0  2 x 7 y 4z 7

Example 6.37

Find the angle between the planes x y 2z 9  0 two planes, respectively. Suppose Q is the angle between
and x + 2y + z 5= 0. the normals. Hence
E E
n1 – n 2 1 2 2 1
Solution: The angle between the two planes is equal cos Q = E E  
to the angle between their normals (see Definition 6.12). n1 – n 2 6 6 2
E E
n1  (1, 1, 2) and n2  (1, 2, 1) are normals to the given
Therefore, Q  P / 3.

Example 6.38

Find the foot of the perpendicular and also the perpen- x 1 y 2 z 3


dicular distance of the point (5, 9, 3) from the line = 
2 3 4
6.4 Line 429

Solution: See Fig. 6.35. Let M (x, y, z) be the foot of HHHHE


PM  (5 3)2 (9 5)2 (3 7)2
the perpendicular drawn from P onto the given line.
Therefore, x  1 2K, y  2 3K and z  3 4K for some  4 16 16
real K. Therefore, 6
HHHHE
PM  (2 L 4, 3L 7, 4 L ) P (5, 9, 3)
HHHHE
PM is perpendicular to the line
HHHHE
 PM – (2, 3, 4)  0
 2(2L 4) 3(3L 7) 4(4L )  0
 29 L  29 A (1, 2, 3) M
L 1
(2, 3, 4)
Therefore, the foot of the perpendicular M  (1 2, 2 3,
FIGURE 6.35
3 4)  (3, 5, 7). Also

Example 6.39
Find the symmetric form of the line x 2y − z − 6  0  Also, substitute z  0 in the plane equations so that 2x
2x 3y − z − 8. 2y  6 and 2x 3y  8. Solving these equations, we get x 
1 and y  2. Therefore, (1, 2, 0) is a point on the line L.
Solution: Let E1 y 2x 2y − z − 6  0  2x 3y − z − 8 Hence, the equation of the line is
y E2 and L be the line represented by the planes E1  0 
x 1 y 2 z 0
E2. Therefore, the DRs of L is  
E E E 1 0 2
i j k
E E
(2, 2, 1) s (2, 3, 1)  2 2 1  i + 2k
2 3 1

Example 6.40

Find the angle between the lines Let L2 be the line


x 2y − 2z − 11  0  x − 2y z − 9 x 3 y+ 5 z 1
= =
x 3 y 5 z 1 1 3 2
and  
1 3 2 Let P be the acute angle between L1 and L2. Therefore,

Solution: Let L1 be the line ( 2, 3, 4) – (1, 3, 2) 1


cos Q  
x 2y − 2z − 11  0  x − 2y z − 9 2 2 2 2 2 2 406
2 3 4 1 3 2
so that DRs of L1 is
E E E
i j k
E E E
1 2 2  2 i 3 j 4k
1 2 1

Example 6.41

Find the equation of the plane through the point (1, 4, − 2) Solution: Let L be the line of intersection of the
and perpendicular to the line of intersection of the planes planes. Hence, the DRs of L is
x y z − 10  0 and 2x − y 3z − 18  0.
430 Chapter 6 Three-Dimensional Geometry

E E E Since the required plane is perpendicular to L, the DRs


i j k
E E E of the normal of the plane are the DRs of L. That is (4,
1 1 1  4 i j 3k −1, − 3). Hence the equation of the plane is
2 1 3
4(x − 1) − 1(y − 4) − 3(z 2)  0
 4x − y − 3z − 6  0

Example 6.42

Find the point of intersection of the line  1 6L  0


x 2 y 1 z 3 1
  L 
3 2 2 6
and the plane 2x y − z − 3  0. Hence, the required point is

Solution: Any point on the given line is of the form (2 ¨ ¥ 1´ ¥ 1´ ¥ 1´ · ¥ 5 2 8 ´


© 2 3 ¦§ 6 µ¶ , 1 2 ¦§ 6 µ¶ , 3 2 ¦§ 6 µ¶ ¸  ¦§ 2 , 3 , 3 µ¶
− 3K, 1 2K, 3 2K). This point lies on the given plane. So ª ¹

2(2 3L ) (1 2 L ) (3 2 L ) 3  0

WORKED-OUT PROBLEMS
Single Correct Choice Type Questions
1. The angle between the lines whose direction cosines Substituting m  (l n) in Eq. (6.42), we have
satisfy the relations l m n  0 and 3lm − 5mn 2nl
 0 is ¥ l1 ´ ¥ l2 ´ 5
¦§ n µ¶ ¦§ n µ¶  3 (6.43)
1 2
P P
(A) (B)
4 2 Therefore, from Eqs. (6.42) and (6.43), we have
P ¥ 3 ´ l1l2 m1m2 n1n2
(C) (D) Cos 1 ¦
3 § 16 µ¶ 5

2

3
L (say)

Solution: We have Therefore

l m n0 (6.40) (l1 , m1 , n1 ) – (l2 , m2 , n2 )  l1l2 m2 m2 n1n2


 L ( 5 2 3)
3lm − 5mn 2nl  0 (6.41)
 L (0)  0
From Eq. (6.40), we have n  − (l m). Substituting this
value of n in Eq. (6.41), we have Therefore, the two lines are at right angles.
Answer: (B)
3lm 5m (l m) − 2l (l m)  0
2. The direction cosines of the line joining the points
 2l 2 6lm 5m2  0
P(4, 3, −5) and Q(−2, 1, −8) are
2
¥ l´ ¥ l´
2¦ µ 6¦ µ 5 0
§ m¶ § m¶ (A) ¥ 6 , 2 , 3 ´ or ¥ 6 , 2 , 3 ´
¦§ µ ¦§ µ
7 7 7¶ 7 7 7¶
If l1 / m1 and l2 /m2 are the roots of this quadratic in l /m, ¥ 6 2 3 ´ ¥ 6 2 3 ´
(B) ¦§ , , µ¶ or ¦§ , , µ¶
then 7 7 7 7 7 7
¥ l1 ´ ¥ l2 ´ 5 (C) ¥ 6 2 3 ´ ¥ 6 2 3 ´
¦§ m µ¶ ¦§ m µ¶  2 (6.42) ¦§ , , µ¶ or ¦§ , , µ¶
7 7 7 7 7 7
1 2
Worked-Out Problems 431

¥ 6 2 3 ´ ¥ 6 2 3 ´ are
(D) ¦ , , µ or ¦ , , µ
§7 7 7 ¶ § 7 7 7¶ (A) (5, 4, 1) (B) (5, 4, −1)
FHHE
Solution: The DRs of the line PQ are (C) (−5, 4, −1) (D) (5, −4, 1)
E E
( 2 4, 1 3, 8 5)  ( 6, 2, 3) Solution: We have n1  (2, 3, 2) and n2  ( 1, 2, 3)
as the DRs of the two given lines, respectively. Hence,
Therefore E E
n1 s n2 is the normal to the plane determined by the
FHHE given lines is
PQ  6 2 2 2 32  7
E E E
FHHE i j k
Therefore, the DCs of the line PQ is E E E
2 3 2 = 5i 4j + k
¥ 6 2 3 ´ ¥ 6 2 3´ 1 2 3
¦§ , , µ¶ or ¦§ , , µ¶
7 7 7 7 7 7
Answer: (A) Therefore, the DRs of the normal are (−5, −4, 1) or (5,
4, −1).
3. If A  (3, 6, 4), B  (2, 5, 2), C  (6, 4, 4) and D  Answer: (B)
(0, 2, 1), then the length of the projection of AB on
CD is equal to 6. The point of intersection of the lines
(A) 4 (B) 3 (C) 1 (D) 2 x 1 y 2 z 3 x 4 y 1
HHHHE HHHE   and  z
Solution: AB  ( 1, 1, 2) and CD  ( 6, 2, 3) are 2 3 4 5 2
FHHHE FHHE
the DRs of the lines AB and CD . Therefore, the mag- is
FHHHE FHHE
nitude of the projection of AB on CD (see Theorem
(A) (1, 1, 1) (B) (1, 1, − 1)
6.29) is
HHHHE HHHE (C) (− 1, 1, − 1) (D) (− 1, − 1, − 1)
AB – CD ( 1)( 6) ( 1)( 2) ( 2)( 3) 14 Solution: Let the given lines be L1 and L2, respectively.
HHHE   2
CD ( 6)2 ( 2)2 ( 3)2 7 Every point on L1 is of the form (1 2t, 2 3t, 3 4t) and
every point on L2 is of the form (4 5s, 1 2s, s). These
Answer: (D) two lines intersect at same point P. This implies

4. The DRs of the line 3x 1  6y − 2  1 − z are 1 2t  4 5s 2t − 5s  3 (6.44)

¥1 ´ ¥ 1 ´ 2 3t  1 2s 3t − 2s  − 1 (6.45)
(A) ¦ , 6, 1µ (B) ¦ 3, , 1µ
§3 ¶ § 6 ¶ 3 4t  s 4t − s  − 3 (6.46)
¥1 1 ´ ¥ 1 1 ´ Solving Eqs. (6.44) and (6.45), we have t  − 1 and s  − 1
(C) ¦ , , 1µ (D) ¦ , , 1µ
§3 6 ¶ §3 6 ¶ which also satisfy Eq. (6.46). Therefore, the point of inter-
section is (−1, −1, −1).
Solution: We have Answer: (D)
x ( 1/ 3) y ( 1/ 3) z 1
3x 1  6 y 2  1 z    7. The foot of the perpendicular drawn from the point
1/ 3 1/ 6 1
(1, 0, 2) onto the line
Therefore, the DRs are
x 1 y 2 z 1
¥1 1 ´  
3 2 1
¦§ , , 1µ¶
3 6 is
Answer: (C)
¥ 1 3 ´ ¥2 ´
(A) ¦ , 1, µ (B) ¦ , 1, 1µ
5. The DRs of the line perpendicular to the plane deter- §2 2¶ §3 ¶
mined by the lines
¥ 2 1 ´
(B) ¦ , , 2µ (D) (1, 2, 1)
x y 1 z 2 x 3 y 2 z 1 §3 2 ¶
  and  
2 3 2 1 2 3
432 Chapter 6 Three-Dimensional Geometry

Solution: Any point on the given line is of theHHH form


E 9. The image of the point (1, 6, 3) in the line
Q  (−1 3K, 2 − 2K, − 1 − K). LetHHHPE  (1, 0, 2). PQ is x y 1 z 2 is
perpendicular to the given line  PQ and the vector (3,  
1 2 3
− 2, − 1) are at right angles. So
(A) (1,0,7) (B) (3, −2, 3)
(3L 2, 2 2 L , 3 L ) – (3, 2, 1)  0
(C) (3, −1, 1) (D) (−3, 0, 1)
 9 L 6 2 (2 2 L ) 1 ( 3 L )  0
Solution: Let P  (1, 6, 3) and Q(K, 1 2K, 2 3K) be
 14 L  7
two points on the line so that
1 HHHE
L 
2 PQ  (L 1, 2 L 5, 3L 1)
HHHE
Therefore Now, PQ is perpendicular to the given line. This implies
¥ 1 3´ (L 1, 2 L 5, 3L 1) – (1, 2, 3)  0
Q  ¦ , 1, µ
§2 2 ¶
 (L 1) 2(2 L 5) 3(3L 1)  0
is the foot of the perpendicular from P onto the line.
 14 L  14
Answer: (D)
L 1
8. The lines Therefore, the foot of the perpendicular from P onto the
x 1 y 1 z 1 line is Q(1, 3, 5). If Pa(x, y, z) is the image of P in the given
  line, then Q(1, 3, 5) must be the midpoint of PPa. Hence,
3 2 5
we get
x 2 y 1 z 1
and   x 1 y 6 z 3
4 3 2  1,  3, 5
2 2 2
intersect at
Therefore, Pa  (1, 0, 7).
(A) (1, 3, 2) (B) (2, 3, 1)
Answer: (A)
(C) no point (D) (2, −3, −1)
Solution: Let L1 and L2 be the two given lines so that 10. The angle between the lines whose DRs are (1, 1, 2)
the general points on L1 and L2 are (1 3t, − 1 2t, 1 5t) and ( 3 , 3 , 0) is
and (2 4s, 1 3s, − 1 − 2s), respectively. The two lines L1
(A) 60° (B) 45° (C) 90° (D) 75°
and L2 intersect. This implies for same t and s we have
E E
Solution: Let n1  (1, 1, 2) and n2  ( 3, 3, 0) and P
1 3t  2 4s 3t − 4s  1 (6.47)
be the acute angle between the lines. Therefore
1 2t  1 3s 2t − 3s  2 (6.48)
E E 3 3 0
 5t  1 2s 5t 2s  −2 (6.49) n1 – n2
cos Q  E E  0
n1 n2 6 18
Solving Eqs. (6.47) and (6.48), we have t  − 5 and s  − 4
which do not satisfy Eq. (6.49). Hence L1 and L2 do not Therefore, P  90°.
intersect.
Answer: (C)
E
Aliter: Let A  (1, −1, 1), B  (2, 1, −1), n1  (3, 2, 5) and
E E E
n2  (4, 3, −2). Also n1 and n2 are not parallel  the 11. If A  (1, −1, 2), B  (3, 4, −2), C  (0, 3, 2)
FHHH E(3, 5,
E and DFHH
lines are not parallel. Now 6), then the angle between the lines AB and CD is
1 2 2 (A) 60° (B) 90° (C) 75° (D) 45°
HHHHE E E FHHHE FHHE
[ AB n1 n2 ]  3 2 5 Solution: The DRs of two lines AB and CD be
E
4 3 2 (2, 5, −4) and (3, 2, 4), respectively. Let n1  (2, 5, 4)
E
and n2  (3, 2, 4). If P is the angle between the lines, then
 1 ( 4 15) 2 ( 6 20) 2(9 8) E E
n –n 2(3) 5(2) ( 4)4
cosQ  E 1 E2  0
 19 52 2 x 0 n1 n2 2 2 52 4 2 32 2 2 4 2
Therefore, the lines are not coplanar and hence they are
Therefore, Q  90o.
skew lines.
Answer: (B)
Answer: (C)
Worked-Out Problems 433

12. The points A(4, 2, 4), B(10, 2, −2) and C(2, 0, −4) are 15. The equation of the locus of a point whose distance
the vertices of from the yz-plane is twice its distance from the point
(4, −2, 1) is
(A) an isosceles, but not an equilateral triangle
(B) an isosceles and a right-angled triangle (A) 3 x 2 4 y2 4z2 32 x 16 y 8z 84  0
(C) right-angled triangle only (B) 3 x 2 4 y2 4z2 32 x 16 y 8z 84  0
(D) an equilateral triangle (C) 3 x 2 4 y2 4z2 32 x 16 y 8z 84  0
Solution: We have (D) 3 x 2 4 y2 4z2 32 x 16 y 8z 84  0

( AB)2  (10 4)2 (2 2)2 ( 2 4)2  36 36  72 Solution: The distance of a point from yz-plane is the
numerical value of the x-coordinate of the point. P(x, y,
( BC )2  (10 2)2 (2 0)2 ( 2 4)2  64 4 4  72 z) is a point on the locus
(CA)2  (4 2)2 2 2 (4 4)2  4 4 64  72 2
 4[( x 4)2 ( y 2)2 (z 1)2 ]  x
Since AB  BC  CA, the triangle is equilateral.  3x 2 4 y2 4z2 32 x 16 y 8z 84  0
Answer: (D) Answer: (C)
13. The locus of the point equidistant from the points 16. The equation of the locus of a point whose distance
(1, −2, 3) and (−3, 4, 2) is from (0, 0, −2) is one-third of its distance from the
(A) 8x 12y + 2z + 15= 0 plane z 18  0 is
(B) 8x + 12y 2z + 15= 0 (A) 9 x 2 9 y2 8z2  188
(C) 8x 12y 2z + 15= 0 (B) 9 x 2 9 y2 9z2  288
(D) 8x 12y + 2z 15= 0 (C) 9 x 2 9 y2 8z2  288
Solution: Let A  (1, −2, 3) and B  (−3, 4, 2). P(x, y, z) (D) 9 x 2 9 y2 8z2  288
is a point on the locus š PA  PB. So
Solution: P(x, y, z) is a point on the locus . So
( PA)2  ( PB)2
1
x 2 y2 (z 2)2  z 18
 ( x 1)2 ( y 2)2 (z 3)2  ( x 3)2 ( y 4)2 (z 2)2 3
 8 x 12 y 2z 15  0  9 x 2 9 y2 9(z 2)2  (z 18)2
Answer: (A)  9 x 2 9 y2 8z2  288
Answer: (C)
14. Given the points A(3, 2, 0) and B(2, 1, −5), the locus
of the point P(x, y, z) such that PA is perpendicular
17. The equation of the plane passing through the points
to PB is
(1, −2, 2) and (−3, 1, −2) and perpendicular to the
(A) x 2 y2 z2 5 x 3 y 5z 8  0 plane 2x y − z 6  0 is
(B) x 2 y2 z2 5 x 3 y 5z 8  0 (A) x 12 y 10z 5  0
2 2 2 (B) x 12 y 10z 5  0
(C) x y z 5 x 3 y 5z 8  0
(D) x 2 y2 z2 5 x 3 y 5z 8  0 (C) x 12 y 10z 10  0
(D) x 12 y 10z 10  0
Solution: We can write
HHHHE Solution: Let E y ax by cz d  0 be the required
AP  ( x 3, y 2, z) plane. It passes through (1, −2, 2) and (−3, 1, −2). This
HHHE implies
BP  ( x 2, y 1, z 5)
and a 2b + 2c + d = 0 (6.50)
Therefore and 3a b 2c d  0 (6.51)
HHHHE HHHE
AP – BP  0 E  0 is perpendicular to the plane 2x y − z 6  0. This
 ( x 3)( x 2) ( y 2)( y 1) z(z 5)  0 gives
2a b − c  0 (6.52)
 x 2 y2 z2 5 x 3 y 5z 8  0
Subtracting Eq. (6.51) from Eq. (6.50), we get
Answer: (B) 4a −3b 4c  0 (6.53)
434 Chapter 6 Three-Dimensional Geometry

From Eqs. (6.52) and (6.53), we get E E E


i j k
E E E E E
12a b  0  b  −12a n1 s n2  7 3 1  4 i 11 j 5k
Therefore, 4 1 1
c  2a b  − 10a is the normal to the required plane. Therefore, the
and d  −a 2b − 2c  − a 2(− 12a) 20a  −5a required plane equation is

Hence, 4 ( x 3) 11 ( y 2) 5(z 4)  0
 4 x 11y 5z 10  0
E y ax by cz d y ax 12ay 10az 5a  0
Answer: (A)
E y x − 12y − 10z − 5  0
FHHE
Answer: (A) 21. Let P  (−3, 1, 1) and Q  (3, 4, 2). R divides PQ
in the ratio PR: PQ  1:3.
FHHEThen, the equation of the
18. The equation of the plane parallel to the plane 2x − plane perpendicular to PQ at R is
3y − 6z − 14  0 and at a distance of 5 units from the
(A) 18 x 9 y 3z  8 (B) 18 x 9 y 3z  4
origin is
(C) 9 x 18 y 3z  4 (D) 3 x 9 y 18z  8
(A) 2 x 3 y 6z p 25  0
Solution: See Fig. 6.36.
(B) 2 x 3 y 6z p 35  0
PR : PQ  1 : 3  3PR  PQ
(C) 2 x 3 y 6z p 35  0
 3PR  PR RQ
(D) 2 x 3 y 6z p 45  0
 2 PR  RQ
Solution: Required plane is of the form 2x − 3y − 6z
d  0. Since its distance from origin is 5, we have Therefore, PR:RQ  1:2. Hence

d ¥ 6 3 2 4 2 2´ ¥ 4´
 5  d  p 35 R¦ , ,  ¦ 1, 2, µ
2 2 32 6 2 § 1 2 3 3 µ¶ § 3¶
HHHE
Answer: (B) The normal to the required plane is PQ  (6, 3, 1). Hence,
the equation of the required plane is
19. If the equation of the plane parallel to the plane 2x − 4´
¥
3y − 5z 6  0 and passing through the point (−1, 2, 6( x 1) 3 ( y 2) 1 ¦ z µ  0
§ 3¶
4) is 2x − 3y − 5z d  0, then the value of d is
 18 x 9 y 3z 4  0
(A) 8 (B) 18 (C) −18 (D) 28
Solution: Since 2x − 3y − 5z d  0 passes through (−1, 1 2
2, 4), we have P R Q
(−3, 1, 1) (3, 4, 2)
2(−1) − 3(2) − 5(4) d  0
FIGURE 6.36
 d  28
Answer: (B)
Answer: (D)
22. From the point P(a, b, c), perpendiculars PL, PM,
20. The equation of the plane passing through the point and PN are drawn to the coordinate planes. If the
(3, −2, 4) and perpendicular to each of the planes 7x equation of the plane LMN is
− 3y z − 5  0 and 4x − y − z 9  0 is
x y z
k
(A) 4 x 11y 5z 10  0 a b c
(B) x y z 5  0
then the value of k is
(C) 2 x y 2z 16  0
(D) 3 x 2 y z 1  0 (A) 1 (B) −1 (C) −2 (D) 2
E E
Solution: Let n1  (7, 3, 1) and n2  (4, 1, 1) so that Solution: It is known that L  (a, b, 0), M  (0, b, c) and
E E
n1 and n2 are the normals to the respective planes. Hence N  (a, 0, c) so that the equation of LMN is
Worked-Out Problems 435

x a y b z k k k
 1
a b 0 0 a b c
0 b c Therefore, the plane passes through the fixed point
(k, k, k).
 bc ( x a) ca ( y b) (ab)z  0
Answer: (B)
 bcx cay abz  2abc
x y z
 2 26. The value of k such that the line
a b c
Answer: (D) x 4 y 2 z k
 
1 1 2
23. If the planes L x 3 y 7z 1  0 and 5x + 6y M z  0 lies in the plane 2x − 4y z − 7  0 is
are parallel, then the integral part of K μ (i.e., K
(A) 7 (B) −7
μ) is
(C) no real value (D) 4
(A) 15 (B) 17 (C) 14 (D) 16
(IIT-JEE 2003)
Solution: The planes are parallel. So
Solution: Every point on the given line is of the form
L 3 7 (4 K, 2 K, k 2K), where K  ℝ. This point lies in the
 
5 6 M plane
5 2x 4y z 7  0
 L  , M  14
2 So
¨5 · ¨ 32 · 2(4 L ) 4 (2 L ) l (k 2 L ) 7  0
Therefore, [L M ]  © 14 ¸  © ¸  16.
ª2 ¹ ª2¹ for all K  ℝ.
Answer: (D)
In particular, for K  0, we have
24. The distance between the parallel planes 3x − 2y 6z 8 − 8 k − 7  0 k  7
8  0 and 3x − 2y 6z − 6  0 is
Note that the point (4, 2, k) lies on the given line and
3 4 3 1 hence it lies in the given plane. So
(A) (B) (C) (D)
7 7 7 7 2(4) − 4(2) k − 7 0
Solution: The distance between the planes, by Theo- k7
rem 6.51, is
Answer: (A)
8 ( 6) 14
 2 27. If the lines
2 2
3 ( 2) 6 2 7
x 1 y 1 z 1
Answer: (C)  
2 3 4
25. If the sum of the reciprocals of the intercepts of a
x 3 y k z
and  
plane on the coordinate axis is the constant 1/ k, 1 2 1
then the plane passes through the point intersect, then the value of k is
1 1 1 3 9 2 3
(A) ¥¦ , , ´µ (B) (k, k, k ) (A) (B) (C) (D)
§ k k k¶ 2 2 9 2
¥ 2 2 2´ (IIT-JEE 2004)
(C) (k 2 , k 2 , k 2 ) (D) ¦ , , µ
§ k k k¶ E
Solution: Let A  (1,−1, 1), B  (3, k, 0), n1  (2, 3, 4)
E HHHHE E E
Solution: Let the plane be and n2  (1, 2, 1). The lines intersect š AB, n1 , n2 are
coplanar. Hence
x y z
1
c b c 2 k 1 1
1 1 1 1 2 3 4 0
and 
a b c k 1 2 1
 2(3 8) (k 1)(2 4) 1 (4 3)  0
436 Chapter 6 Three-Dimensional Geometry

 10 2k 2 1  0 ¨ 1 2 1· ¨ 4 ·
 2k  9 ©
^ ©0 3 0 ¸ X  ©© 10 ¸¸ R 3 R 2
¸
9 ©ª0 0 L 1¸¹ ©ª 2 ¸¹
k 
2
The system is inconsistent if K  1.
Answer: (B)
Answer: (B)
28. The value of K such that the planes 2x − y 2z 
29. A plane is parallel to two lines whose DRs are (1, 0,
2, x − 2y z  −4, x y Kz  4 form a triangular
−1) and (−1, 1, 0) and it contains the point (1, 1, 1). If
prism is
it cuts coordinate axis at A, B and C, then the volume
(A) 3 (B) 1 (C) 0 (D) −3 of the tetrahedron OABC (in cubic units) is
(IIT-JEE 2004) 11 9 7
(A) (B) (C) (D) 6
2 2 2
Solution: Since the planes are not parallel, from Sec-
tion 6.4.1, the planes form a triangular prism if (IIT-JEE 2004)
E E
2 1 2 Solution: Let n1  (1, 0, 1) and n2  ( 1, 1, 0). Since
E E
$  1 2 1  0 the required plane is parallel to n1 and n2 , its normal
E E E
1 1 L ¨ i j k·
E E © ¸ E E E
n1 s n2  © 1 0 1¸  i j k  (1, 1, 1)
2 1 2
© 1 1 0 ¸
and $ 3  1 2 4  8 x 0 ª ¹
1 2 4 Since the plane also passes through the point (1, 1, 1), its
Also equation is
1(x − 1) 1(y − 1) 1(z − 1)  0
$  0  2 ( 2 L 1) 1 (L 1) 2 (1 2)  0
 3L 2 1 6  0 x y z3
L 1 x y z
 1
3 3 3
Aliter: The planes form a triangular prism if and only if Therefore, A  (3, 0, 0), B  (0, 3, 0) and C  (0, 0, 3).
the planes are not parallel and the planes are not having Hence, the volume of the tetrahedron OABC is
common solutions. We have
¨ 3 0 0·
¨ 2 1 2 · ¨ 2· 1© 27 9
© 1 2 1¸ X  © 4 ¸ © 0 3 0 ¸¸  
6 6 2
© ¸ © ¸ ©ª0 0 3¸¹
©ª 1 1 L ¸¹ ©ª 4 ¸¹
Answer: (B)
where
30. The equation of the plane containing the straight
¨ x·
line
X  ©© y¸¸ x y z
©ª z ¸¹  
2 3 4
which is perpendicular to the plane containing the
Interchanging R1 and R2, we get
lines
¨ 1 2 1· ¨ 4 ·
© 2 1 2 ¸ X  © 2 ¸ x y z x y z
  and  
© ¸ © ¸ 3 4 2 4 2 3
©ª 1 1 L ¸¹ ©ª 4 ¸¹ is
¨ 1 2 1· ¨ 4 ·
© (A) x 2y − 2z  0 (B) 3x 2y − 2z  0
^ ©0 3 0 ¸ X  ©© 10 ¸¸ R 2 2R 1 , R 3 R 1
¸
(C) x − 2y z  0 (D) 5x 2y − 4z  0
©ª0 3 L 1¹¸ ©ª 8 ¸¹
(IIT-JEE 2010)
Worked-Out Problems 437
HHHE
Solution: The normal to the plane containing the given Also PQ is normal to the plane provided in Eq. (6.57),
lines is so that
E E E (x1 − 1, y1 2, z1 − 1)  L (1, 2, −2)
i j k
E E E
(3, 4, 2) × (4, 2, 3)  3 4 2  8 i j 10k Since (1, 2, −2) is normal to the plane provided in Eq.
(6.56), we have
4 2 3
x1  1 K, y1  −2 2K, z1  1 − 2K
Let E be the required plane so that the normal to E is
Substituting the values of x1, y1 and z1 in Eq. (6.56), we
perpendicular to (8, −1, −10). Let E y ax by cz d  0.
have
E  0 passes through (0, 0, 0) d  0. Also E  0 con-
tains (1 K) 2(− 2 2K) − 2(1 − 2K)  10
x y z  9K − 5  10
 
2 3 4 5
K
3
Therefore
Therefore
2a 3b 4c  0 (6.54)
5 8
Further, the normal (a, b, c) is perpendicular to (8, −1, x1  1 L  1 
−10). Therefore 3 3
10 4
8a − b − 10c  0 (6.55) y1  2 2 L  2 
3 3
From Eqs. (6.54) and (6.55), we get
10 7
and z1  1 2 L  1 
a b c 3 3
 
1 2 1 So
Therefore, the equation of E is x − 2y z  0. ¥ 8 4 7 ´
Answer: (C) Q ( x1 , y1 , z1 )  ¦ , , µ
§3 3 3 ¶
Answer: (A)
31. If the distance of the point P(1, −2, 1) from the plane
x 2y − 2z  A , where A > 0, is 5, then the foot of
32. A line with positive direction cosines passes through
the perpendicular from point P to the plane is
the point P(2, −1, 2) and makes equal angles with the
¥ 8 4 7´ ¥ 4 4 1´ coordinate axes. The line meets the plane 2x y z 
(A) ¦ , , (B) ¦ , ,
§ 3 3 3 µ¶ § 3 3 3 µ¶ 9 at Q. The length of the line segment PQ is equal to

¥ 1 2 10 ´ ¥ 2 1 5´ (A) 1 (B) 2 (C) 3 (D) 2


(C) ¦ , , µ (D) ¦ , , µ
§3 3 3 ¶ § 3 3 2¶ (IIT-JEE 2009)
(IIT-JEE 2010) Solution: Since the line makes equal angles with the
axes and the DCs are positive, the DCs are
Solution: By hypothesis, we get
¥ 1 1 1 ´
¦§ , , µ
1 2( 2) 2(1) A 3 3 3¶
5
12 2 2 ( 2)2 Hence, the equation of the line is

 @ 5  ± 15 x 2 y 1 z 2
 
1/ 3 1/ 3 1/ 3
 @  10, − 20
Since @ > 0, we have @  10. Therefore, the plane is Therefore, every point on the line is of the form
t t t
x 2y − 2z  10 (6.56) x2 , y  1 , z 2
3 3 3
Let Q(x1, y1, z1) be the foot of the perpendicular from
P(1, −2, 1) onto the plane provided in Eq. (6.56). There- This point lies on the plane 2x y z  9. So we have
fore ¥ t ´ ¥ t ´ ¥ t ´
2¦2 µ ¦ 1 µ ¦2 µ 9
x1 2y1 − 2z1  10 (6.57) § 3¶ § 3¶ § 3¶
438 Chapter 6 Three-Dimensional Geometry

4t Since the plane passes through the point (1, −2, 1), its
5 9 equation is
3
t  3 − 3(x − 1) − 3(y 2) 0(z − 1)  0

Therefore, Q  (3, 0, 3). This gives 3x 3y − 3 6  0


x y 1  0 (6.58)
PQ  (3 2)2 (0 1)2 (3 2)2  3
The distance of the point (1, 2, 2) from the plane pro-
Answer: (C) vided in Eq. (6.58) is

33. Let P(3, 2, 6) be a point


E in space E QE be a point
E and 1 2 1
E E E 2 2
on the line r  (i j 2k ) L ( 3i j 5k ). Then the
FHHE 12 12
value of K for which the vector PQ is parallel to the
plane x − 4y 3z  1 is Answer: (D)
1 1 1 1
(A) (B) (C) (D) 35. If the equation of the plane passing through the in-
4 4 8 8 tersection of the planes x 2y z − 1  0 and 2x y
(IIT-JEE 2009) 3z − 2  0 and perpendicular to the plane x y z − 1
 0 is x ky 3z − 1  0, then the value of k is
Solution: The given line
E E E E E E E (A) 4 (B) − 4 (C) 2 (D) − 2
r  (i j 2k ) L ( 3i j 5k )
Solution: Let E1  0 and E2  0 be the given two planes
is written in the parametric form as and E  0 be the required plane. Since E  0 passes
x  1 − 3K, y  −1 K, z  2 5K through the line of intersection of E1  0 and E2  0,

Therefore E  E1 KE2  0
FHHE E y (x 2y z − 1) K(2x y 3z − 2)  0
PQ  (1 3L 3, 1 L 2, 2 5L 6)
 ( 3L 2, L 3, 5L 4) E y (1 2K) x (2 K) y (1 3K) z − 1 − 2K  0
FHHE This is perpendicular to the plane x y z − 1  0. So
PQ is parallel to the plane x − 4y 3z  1
FHHE 1(1 2K) 1(2 K) 1(1 3K)  0
 PQ is perpendicular to the normal of the plane
 6K 4  0
 1(− 3K − 2) − 4(K − 3) 3(5K − 4)  0
2
 8K − 2  0  L
3
1
L Therefore
4
Answer: (B) ¥ 4´ ¥ 2´ 4
E y ¦ 1 µ x ¦ 2 µ y (1 2) z 1  0
§ 3¶ § 3¶ 3
34. A plane which is perpendicular to two planes 2x − 2y  E y x 4 y 3z 1  0
z  0 and x − y 2z  4 passes through (1, −2, 1).  E y x 4 y 3z 1  0
The distance of the plane from the point (1, 2, 2) is
(A) 0 (B) 1 (C) 2
(D) 2 2 Hence, k  − 4.
(IIT-JEE 2006) Answer: (B)
E E
Solution: We have n1  (2, 2, 1) and n2  (1, 1, 2). 36. The equation of the plane passing through the line of
The normal to the required plane is intersection of the planes x 2y 3z  2 and x − y z
E E E  3 and at a distance 2 / 3 from the point (3, 1, −1)
i j k
E E is
n1 s n2  2 2 1
(A) 5x − 11y z  17 (B) 2 x y 3 2 1 0
1 1 2
E E (C) x y z  3 (D) x 2 y 2 1  0
 3i 3 j
(IIT-JEE 2012)
Worked-Out Problems 439

Solution: The required plane is of the form Therefore


(x 2y 3z − 2) K(x − y z − 3)  0 P  (1 L , 1 4 L , 4 L )
(1 K) x (2 − K) y (3 K) z − 2 − 3K  0 (6.59) ¥ 1 4 1 ´ ¥  1 13 ´
 ¦ 1 , 1 , 4 µ  ¦ , , µ
§ 3 3 3¶ § 3 3 3 ¶
The plane provided in Eq. (6.59) is at a distance of 2 / 3
from (3, 1, −1). So Now, S  (1 − K, − 1 − 4K, 4 − K) is a point on QR and T 
HHE HHHE
(2, 1, 4) and TS is perpendicular to QR. This implies
(1 L )(3) (2 L )(1) (3 L )( 1) 2 3L 2 HHE HHHE
 TS – QR  0
2 2 2 3
(1 L ) (2 L ) (3 L )
 (1 − K − 2, − 1 − 4K − 1, 4 − K − 4) – (− 1, − 4, − 1)  0
3 2 3 2 2L 2
   (− K − 1, − 2 − 4K, − K) – (− 1, − 4, − 1)  0
3L 2 4 L 14 3
 (K 1, 2 4K, K) – (1, 4, 1)  0
 12K2  4(3K2 4K 14)  K 1 8 16K K  0
 16K  −56  18K  − 9
7
L  1
2 L 
2
From Eq. (6.59), the required plane is
Therefore
7
x 2 y 3z 2 ( x y z 3)  0 ¥ 1 4 1´ ¥ 3 9 ´
2 S  ¦ 1 , 1 , 4 µ  ¦ , 1, µ
§ 2 2 2¶ § 2 2¶
 −5x 11y − z 17  0
Hence
 5x − 11y z − 17  0 2 2 2
¥ 4 3 ´ ¥ 1 ´ ¥ 13 9 ´
Answer: (A) ( PS)2  ¦ µ ¦ 1µ ¦ µ
§ 3 2¶ § 3 ¶ § 3 2¶
37. The point P is the intersection of the line joining 1 4 1 18 1
Q(2, 3, 5) and R(1, −1, 4) with the plane 5x − 4y − z −   
36 9 36 36 2
1  0. If S is the foot of the perpendicular drawn from
the point T(2, 1, 4) to QR, then the length of the line 1
 PS 
segment PS is 2
1
(A) (B) 2 (C) 2 (D) 2 2 T (2, 1, 4)
2
(IIT-JEE 2012) 90° Q (2, 3, 5)
HHHE
Solution:FHHEWe have QR  (−1, −4, −1). The equation of S
the line QR is R (1, -1, 4)

x 1 y 1 z 4
  L (say)
1 4 1 P

Therefore, every point on the line QR is given by 5x − 4y − z − 1 = 0

x  1 − K, y  − 1 − 4K, z  4 − K
Point P lies in the plane 5x − 4y − z − 1  0. This implies FIGURE 6.37
Answer: (A)
5(1 − K) − 4(−1 − 4K) − (4 − K) − 1  0
 5 4 − 4 − 5K 16K K − 1 0 38. Suppose a, b and c are real and

 12K  −4 ¨ 1 9 7·
1 ;a bc = ©©8 2 7¸¸  [0, 0, 0]
L ©ª 7 3 7 ¸¹
3
440 Chapter 6 Three-Dimensional Geometry

If the point P(a, b, c), which is satisfying the above 3ts 9t 6s 18  12ts − 6s 16t − 8
condition, lies in the plane 2x y z  1, then the
9ts  −7t 12s 26 (6.63)
value of 7a b c is
(ii)  (iii)  (2t − 1) (s 1)  (t − 1)(s 3)
(A) 0 (B) 12 (C) 7 (D) 6
(IIT-JEE 2011)  2ts 2t − s − 1  ts 3t − s − 3

Solution: We have  ts  t − 2
t 2
¨ 1 9 7·  s (6.64)
t
;a bc = ©©8 2 7¸¸  [0, 0, 0] Substituting the value of s in Eq. (6.63), we have
©ª 7 3 7 ¸¹
2t2 − 7t 3  0
 [a 8b 7c, 9a 2b 3c, 7a 7b 7c]  [0, 0, 0]
Hence t  3 or 1 / 2.
Therefore
t 2 1
a 8b 7c  0 (6.60) Case 1: When t  3 and s  
3 3
9a 2b 3c  0 (6.61) (i)  (iii) 3ts  − 5t 12s 14 (6.65)
a b c0 (6.62) Now, t  3 and s  1/ 3 also satisfy Eq. (6.65). Therefore,
P  (5, −5, 2) and Q  (10 / 3, 10 / 3, 4 / 3). Hence
Hence
25 25 4 54
9a 2b 3(− a − b)  0 (& c  − a − b) ( PQ)2   6
9 9 9 9
 6a − b  0, c  − 7a
Case 2: When t  1/ 2 and s  − 3 do not satisfy Eq. (6.65),
a b c P  (5, −5, 2) and Q  (10 / 3, 10 / 3, 4 / 3). Hence,
  
1 6 7
PQ  6
P(a, b, c) lies on the plane 2x y z  1. This implies
Answer: (B)
2a b c  1
 2a 6a − 7a  1 (& b  6a, c  − 7a) 40. A variable plane at a distance of 1 unit from the ori-
gin cuts the coordinate axes at A, B and C. If the cen-
a1 troid D(x, y, z) of ΔABC satisfies the relation
Therefore 1 1 1
2
2
k
7a b c  7(1) 6 − 7  6 x y z2
Answer: (D)
then the value of k is

39. A line through origin meets the lines


1
(A) 9 (B) (C) 1 (D) 3
3
x 2 y 1 z 1 x (8 / 3) y 3 z 1
  and   Solution: Let A  (a, 0, 0), B  (0, b, 0) and C  (0, 0, c).
1 2 1 2 1 1
The equation of the plane ABC is
at P and Q, respectively, then the length PQ is
x y z
1
(A) 5 (B) 6 (C) 7 (D) 2 2 a b c
(IIT-JEE 2010)
FHHE Since the distance of the plane ABC from origin is 1, we
Solution: The line PQ passes through (0, 0, 0). This have
FHHE FHHHE
implies OP  L (OQ) . Therefore 1
1
2 t 2t 1 t 1 (1 / a ) (1 / b2 ) (1 / c 2 )
2
 
(8 / 3) 2 s 3 s s 1


L L
(ii) (iii) 1 1 1
(i)  1 (6.66)
a2 b2 c2
(i)  (ii)  (2 t)(3 s)3  (2t − 1)(8 6s)
Worked-Out Problems 441

The centroid D( x, y, z) is (a/3, b/3, c/3). This gives P (3, 5, 7)

a b c
x  , y ,z
3 3 3
So
Q
1 1 1
k
x2 y2 z2
9 9 9
 k
a 2 b2 c 2 P
9k
FIGURE 6.38
Answer: (A)
Answer: (B)
41. The normal to the plane passing through the points
43. The planes E1 y x − y − z − 4  0 and E2 y x y 2z
A  (2, 1, −1), B  (1, 1, 1) and C  (3, 3, 0) is
− 4  0 intersect in line L. The plane E1  0 is rotated
(A) (4, − 3, 2) (B) (4, 3, 2) about the line L through 90°. The equation of E1  0
(C) (−4, 3, 2) (D) (4, 3, −2) in its new position is

Solution: The normal to the plane ABC is (A) 5x y − 4z − 20  0


E E E (B) 5x − y 4z − 20  0
i j k
HHHHE HHHHE E E E (C) 5x y 4z − 20  0
AB s AC  1 0 2  4 i 3 j 2k  (4, 3, 2)
(D) 5x y 4z 20  0
1 2 1
Solution: See Fig. 6.39. Let E be the required plane.
Answer: (A) Therefore,
E y E1 KE2  0
42. The image of the point P(3, 5, 7) in the plane 2x y
z  0 is E y (x − y − z − 4 ) K (x y 2z − 4 )  0

(A) (9, 1, −1) (B) (−9, −1, 1) y(1 K)x (K − 1)y (2K − 1)z − 4 − 4K  0
(C) (−9, 1, −1) (D) (9, −1, −1) E is perpendicular to E1. This implies

Solution: See Fig. 6.38. Let Q(x1, y1, z1) be the foot of (1 K)(1) (K − 1)(− 1) (2K − 1)(−1)  0
the perpendicular from P(3, 5, 7) onto the plane. There-  − 2K 1 1 1  0
HHHE
fore, PQ is parallel to the normal to the plane. So 3
L
(x1 − 3, y1 − 5, z1 − 7)  K(2, 1, 1) 2
 x1  3 2K, y1  5 K and z1  7 K Therefore

(x1, y1, z1) lies in the plane. This implies ¥ 3´ ¥3 ´


E y ¦ 1 µ x ¦ 1µ y (3 1)z 4 6  0
§ 2¶ §2 ¶
2(3 2K) (5 K) (7 8)  0
E y 5x y 4z − 20  0
 6K  −18
 K  −3
Therefore, Q  (x1, y1, z1)  (−3, 2, 4). If Pa(@, A, F ) is the 90°
HHHHE
image of P(3, 5, 7), then Q is the midpoint of PPa which E
implies that
A 3
 3@  9
2 L
B 5
2  A  1 E1
2
G 7 E2
and 4 F  1
2 FIGURE 6.39
Hence, the image of P  Pa  (@, A, F)  (−9, −1, 1). Answer: (C)
442 Chapter 6 Three-Dimensional Geometry

44. The equation of the plane passing through the points x y 2 z x 1 y 2 z


(3, 4, 1) and (0, 1, 0) and parallel to the line (C)   (D)  
2 3 4 2 3 4
x 3 y 3 z 2 Solution: Let L be the line of intersection. We know
  E E
2 7 5 that n1  (4, 4, 5) and n2  (8, 12, 13) are the normals
is of the given planes. Therefore, the DRs of the line L is
E E E
(A) 8x 13y 15z 13  0 i j k
E E
(B) 8x 13y − 15z − 13  0 n1 s n2  4 4 5
(C) 8x − 13y − 15z 13  0 8 12 13
(D) 8x − 13y 15z 13  0 E E E
 8 i 12 j 16k
Solution: Let the required plane be E y ax by cz
Therefore, the DRs of L is (2, 3, 4). Also, substituting z  0
d  0. Since it passes through (3, 4, 1) and (0, 1, 0), we
in both plane equations, we get
have
3a 4b c d  0 (6.67) 4x 4y  12 and 8x 12y  32
and b d0 (6.68) Solving these equations, we get x  1 and y  2. Hence, (1,
2, 0) is a point on the line L. Thus, the equation of the
Also since the plane E  0 is parallel to the given line, we
line is
have
2a 7b 5c  0 (6.69) x 1 y 2 z 0
 
2 3 4
From Eq. (6.68), we have d  −b. Substituting this value
Answer: (A)
in Eq. (6.67), we have
3a 3b c  0 (6.70) 46. The equation of the plane passing through the point
(1, −1, 2) and perpendicular to each of the planes
From Eqs. (6.69) and (6.70), we get
2x 3y − 2z  5 and x 2y − 3z  8 is
2a 7b 5(−3a − 3b)  0
(A) 5x 4y − z  7 (B) 5x − 4y z  7
 −13a − 8b  0 (C) 5x − 4y − z  7 (D) 5x 4y z  7
Therefore E E
Solution: Let n1  (2, 3, 2) and n2  (1, 2, 3) which
8 are the normals of the given planes, respectively, so that
a b the normal to the required plane is
13
E E E
24 15b i j k
c  3a 3b  b 3b  E E
13 13 n1 s n2  2 3 2
Hence 1 2 3
E E E
¥ 8 ´ 15  5i 4 j k
a  ¦ µ b, c  b, d  b
§ 13 ¶ 13
The required plane passes through the point (1, −1, 2).
Therefore, the equation of the plane is Hence its equation is

8 −5(x − 1) 4(y 1) 1(z − 2)  0


¥ 15 ´
bx by ¦ bµ z b  0
13 § 13 ¶  5x − 4y − z  7
Answer: (C)
 8x − 13y 15z 13  0
Answer: (D) 47. The coordinates of the point where the line

45. The symmetric form of the line of intersection of the


x 1 y 2 z 3
 
two planes 4x 4y − 5z − 12  0 and 8x 12y − 13z − 2 3 4
32  0 is meets the plane x y 4z − 6  0 are
x 1 y 2 z x 1 y 2 z 1 (A) (−1, −1, −1) (B) (1, −1, −1)
(A)   (B)  
2 3 4 2 3 4 (C) (−1, 1, −1) (D) (1, 1, 1)
Worked-Out Problems 443

Solution: P  (−1 2K, −2 3K, −3 4K) is a point on (A) (3a, 2a, 3a), (a, a, 2a)
the given line. Point P belongs to the given plane implies (B) (3a, 2a, 3a), (a, a, a)
that
(C) (3a, 3a, 3a), (a, a, a)
(−1 2K) (−2 3K) 4(−3 4K)  6 (D) (2a, 3a, 3a), (2a, a, a)
 21K  21 Solution: The given line L1 is
 K1 x y a z
 
Therefore, the required point is (−1 2, −2 3, −3 4)  1 1 1
(1, 1, 1).
P is a point on L1. Therefore
Answer: (D)
P  (t, −a t, t) (6.71)
48. The equation of the plane containing both lines and the given line L2 is
x 3 y 1 z 2 x 7 y z 7 x a y z
  and    
2 3 1 3 1 2 2 1 1
is Q is a point on L2. Therefore
(A) x y − z  0 (B) x y z  0 Q  (−a 2s, s, s) (6.72)
(C) x y z  1 (D) x y z  2 FHHE
From Eqs. (6.71) and (6.72), the DRs of PQ are
Solution: The DRs of the required plane is
E E E (2s a t, s t a, s t )
i j k‘ Therefore, by hypothesis, we have
E E E
2 3 1  7 i 7 j 7k
2s a t s t a s t
3 1 2  
2 1 2
Therefore, (1, 1, 1) is the normal vector which passes Hence
through the point (7, 0, −7). So we have
2s a t s t a
1(x − 7) 1(y − 0) 1(z 7)  0   t  3a
2 1
x y z0 s t a s t
Answer: (B)   s t  2a  s  a (& t  3a)
1 2

49. The equation of the plane passing through the inter- Therefore, P  (3a, 2a, 3a) and Q  (a, a, a).
section of the planes x 2y 3z 4  0, 4x 3y Answer: (B)
2z 1  0 and the origin is
(A) x y z  0 (B) 3x 2y z 1  0 51. If the angle P between the line
(C) 2x 3y z  0 (D) 3x 2y z  0 x 1 y 1 z 2
 
Solution: The required plane is of the form 1 2 2
(x 2y 3z 4) K(4x 3y 2z 1)  0 and plane 2x y Lz 4  0
This passes through (0, 0, 0). This implies is such that sin Q  1/ 3, then the value of K is
4 K0K−4 5 3 3 4
(A) (B) (C) (D)
Therefore, the required plane is 3 5 4 3
E
Solution: The DRs of the line  (1, 2, 2)  n1 . The DRs
−15x − 10y − 5y  0 E
of the normal to the plane  (2, 1, L )  n2 . By hypoth-
 3x 2y z  0 E E
esis, 90° − P is the angle between n1 and n2 . Therefore
Answer: (D)
E E
n1 – n2 2 2 2 L
cos(90o Q )  E E 
50. A line with DCs proportional to 2, 1, 2 meets each of n1 n2 3 2 2 12 L
the lines x  y a  z and x a  2y  2z. The coordi-
nates of each of the points of intersection are
444 Chapter 6 Three-Dimensional Geometry

Hence 1 1 2
1 2 L 5 k 2 3 0
  5 L  4L  L 
3 3 5 L 3 3 k 2
Answer: (A)  1(4 3k ) 1(2k 9) 2(k 2 6)  0
 2k 2 5k 25  0
52. If the lines
 2k 2 5k 25  0
x 1 y 2 z 3
  Therefore, the integer value of k  5 or 5 / 2.
k 2 3
Answer: (C)
x 2 y 3 z 1
and  
3 k 2 53. The line joining the points (5, 1, a) and (3, b, 1) cross-
intersect at a point, then the integer value of k is es the yz-plane at the point (0, 17 / 2, 13 / 2). Then
(A) 2 (B) −2 (C) −5 (D) 5 (A) a  6, b  4 (B) a  8, b  2
Solution: P  (1 kt, 2 2t, 3 3t) is a point on the (C) a  2, b  8 (D) a  4, b  6
first line and Q  (2 3s, 3 ks, 1 2s) is a point on the
Solution: Let P  (5, 1, a), Q  (3, b, 1) and R  (0, 17 / 2,
second line. The lines intersect. So FHHE
13 / 2).The line PQ meets yz-plane in R(0, 17 / 2, 13 / 2).
HHHE HHHE
1 kt  2 3s kt − 3s  1 (6.73) So the vectors PQ and QR are parallel vectors. This
2 2t  3 ks  2t − ks  1 (6.74) gives
HHHE HHHE
and 3 3t  1 2s  3t − 2s  − 2 (6.75) PQ  LQR
From Eqs. (6.73)–(6.75) and eliminating t and s, we have ¥ 17 13 ´
 ( 2, b 1, 1 a)  L ¦ 3, b, 1µ
2k2 5k − 25  0 § 2 2 ¶

whose roots are −5 and 5 / 2. Integer value of k is −5. 2 b 1 1 a


  
3 17 2 b 13 2 1
Aliter: We know that the points A(1, 2, 3) and B(2,
3, 1) are points on the first and the second line, respec- 2 2(b 1) 2(a 1)
  
tively. Also the DRs of the lines are (k, 2, 3) and (3, k, 2), 3 17 2b 15
HHHHE
respectively. The lines intersect. So AB, (k, 2, 3) and (3,  a  6, b  4
k, 2) are coplanar vectors. Therefore
Answer: (A)

Multiple Correct Answer Type Questions


1. The equation of a plane containing the line x y z  0
x 1 y 1 z 4 x y 2z  3
 
1 2 1
3x 2 y z  2
is Answer: (A), (B), (C)
(A) x y z  0 (B) 4x y − 2z  3
2. Which of the following equations represent the line
(C) 3x 2y z  2 (D) 3x 2y z  0
of intersection of the planes 4x − 4y − z 11  0  x
Solution: The required plane is of the form 2y − z − 1?
a(x − 1) b(y 1) cz  0 x 2 y z 3 2 x 13 4 y 15 z
(A)   (B)  
Since (a, b, c) is normal to the plane, we have 1 1 4 4 4 4
a − 2b c  0 (6.76) x y 2 z 3 x 4 y 4 z 11
(C)   (D)   .
We know that a  1, b  1, c  1; a  4, b  1, c  −2 and 2 1 4 2 2 2
a  3, b  2, c  1 satisfy Eq. (6.76). Substituting these in
the equation of the plane we get
Worked-Out Problems 445

Solution: We have and L3  P1 † P2 be their lines of intersection two by


E E E two. Then
i j k
E E
4 4 1  6 i 3 j 12k (A) L1, L2, L3 are parallel.
1 2 1 (B) At least two of L1, L2, L3 are parallel.
(C) L1, L2, L3 intersect in a point.
Hence, the DRs of the line are (2, 1, 4). In the given plane
equations, substitute x  0 so that we have 4 y 8  11, (D) The three planes do not have a common point.
2 y z  1. On solving these equations, we have (0, 2, 3) is Solution: The DRs of L1 are
a point on this line. Hence, the line equation is E E E
x y 2 z 1 i j k
E E
  1 1 1  4 j 4k
2 1 4
1 3 3
Similarly, taking z  0 and solving the two equations for
x and y, we have x  13 / 2 and y  15 / 4 so that
The DRs of L2 are
( 13 / 2, 15 / 4, 0) is a point on the line. Hence, the equa-
E E E
tion of the line is i j k
E E
x ( 13 2) y (15 4) z L2  1 3 3  2 j 2k
 
2 1 4 1 1 1
2 x 13 4 y 15 z
   The DRs of L3 are
4 4 4
E E E
It is clear that (2, 0, 3) is not a point on the line and (−2, i j k
E E
2, 2) is not parallel to (2, 1, 4). 1 1 1  2 j 2k
Answer: (B), (C) 1 1 1

3. A point on the line Therefore,


x 1 y 2 z 1 −2(DRs of L2)  DRs of L1  −2(DRs of L3)
 
1 2 3
Hence, the lines L1, L2 and L3 are parallel.
at a distance of 6 units from the origin is Answer: (A), (B), (D)
(A) ¥ 5 10 13 ´ (B) ( 1, 2, 1)
¦§ , , µ 5. Consider the planes 3x − 6y − 2z  15 and 2x y −
7 7 7 ¶ 2z  5. Then
¥ 5 10 13 ´ E E E
(C) (1, 2, 1) (D) ¦ , , µ (A) Vector 14 i 2 j 15k is parallel to their line of
§ 7 7 7¶ intersection.
Solution: Any point P on the given line is of the form (B) The two planes are intersecting.
(1 t, 2 2t, −1 3t). If O is the origin and OP  6, we (C) The symmetric form of their line of intersection
have x 3 y 1 z
is   .
(1 t )2 4(1 t )2 (3t 1)2  6 14 2 15
x y z
 14t 2 4t 6  6 (D) The line of intersection meets the line  
1 1 1
2 at (1, 1, 0).
 t  0,
7 Solution: The DRs of the line of intersection of the
Case 1: t  0  P =(1, 2, 1). planes are
E E E
2 ¥ 2 4 6 ´ ¥ 5 10 13 ´ i j k
Case 2: t   P  ¦1 , 2 , 1 µ  ¦ , , µ. E E E
7 § 7 7 7¶ § 7 7 7 ¶ 3 6 2  14 i 2 j 15k
Answer: (A), (C) 2 1 2

4. Consider the planes P1: x − y z  1, P2: x y − z  − 1 Clearly, the planes are not parallel. If we substitute z  0
and P3: x − 3y 3z  2. Let L1  P2 † P3 , L2  P3 † P1 in both plane equations and solve for x and y, we have
446 Chapter 6 Three-Dimensional Geometry

x  3 and y  − 1 so that the equation of the line of inter- 7. The DRs of the normal to the plane passing through
section is (1, 0, 0), (0, 1, 0) which makes an angle P /4 with plane
x y  3 are
x 3 y 1 2
  (A) (1, 2 , 1) (B) (1, 1, 2 )
14 2 15
Answer: (A), (B), (C) (C) (1, 1, 2) (D) ( 1, 1, 2 )
Solution: The equation of the plane is
Try it out Try that Choice (D) is not correct in a( x 1) by cz  0 (6.78)
Problem 5 above.
It passes through (0, 1, 0). So
−a b0
6. Let P be the point (1, 2, 3) and E y x y z 1  0 be
ab (6.79)
a plane. Then
Also
(A) P lies on the origin side of the plane E  0.
(B) Distance of P from the plane E is 7 / 2 . 1 P a b
 cos 
(C) Foot of the perpendicular from P on the plane E 2 4 1 1 0 2 a 2 b2 c 2
2 2
 0 is ( 4 / 3, 1/ 3, 2 / 3).
1 | 2a |
(D) The line joining origin to P meets the plane E   
0 at the point (1, 1, 1). 2 2 2a 2 c 2
Solution:  2a 2 c 2  4a 2
(A) We have  2a 2  c 2
P  ( x1 , y1 , z1 )  E11  1 2 3 1  7  0 Therefore, a  b and c  2a. Hence, a : b : c  1 : 1 : 2 .
O (0, 0, 0) E22  1 > 0 The DRs of the normal of the plane are (1, 1, 2 ) or
Hence, from the Note under Theorem 6.52, it is ( 1, 1, 2 ).
clear that P lies on the origin side of the plane. Answer: (B), (D)
(B) Distance of P(1, 2, 3) from the plane is
8. Consider the planes x − 2y 3z  1, − x y − 2z  k
1 2 3 1 7 7 and x − 3y 4z  1. Then
 
12 2 2 32 14 2 (A) The three planes have no common point for k ≠ 3.
(B) The three planes have infinitely many common
(C) Let Q(x1, y1, z1) be the foot of the perpendicular
points for k ≠ 3.
from P onto the plane E  0. Therefore
(C) The three planes have infinitely many common
x1 y1 z1 1 0 (6.77)
points for k  3.
HHHE HHHE
Also PQ is normal to the plane E  0  PQ  (D) The three planes have unique common point,
L (1, 1, 1). So for k  3

x1 1  L , y1 2  L , z1 3  L Solution: Consider the matrix equation


 x1  L 1, y1  L 2, z1  3 L ¨ 1 2 3· ¨ 1·
© 1 ¸
1 2 ¸ X  ©© k ¸¸
Substituting the values of x1, y1 and z1 in Eq. (6.77), ©
we have ©ª 1 3 4 ¸¹ ©ª 1¸¹
3K 7  0  L  7/ 3 where
Therefore ¨ x·
Q  ( x1 , y1 , z1 )  (1 L , 2 L , 3 L ) X  ©© y¸¸
¥ 7 7 7´ ©ª z ¸¹
 ¦1 , 2 , 3 µ
§ 3 3 3¶ Add R1 to R2 and subtract R1 from R3 to get
¥ 4 1 2´ ¨ 1 2 3· ¨ 1 ·
¦ , ,
§ 3 3 3 µ¶ ©0 1 1¸ X  ©k 1¸
© ¸ © ¸
Answer: (A), (B), (C) ©ª0 1 1¸¹ ©ª 2 ¸¹
Worked-Out Problems 447

¨ 1 2 3· ¨ 1 ·  x z  5, y z  2
©0 1 1¸ X  © k 1¸ (R m R R ) (6.80)
© ¸ © ¸ 3 3 2  x  5 z, y  z 2
©ª0 0 0 ¸¹ ©ª 3 k ¸¹ Therefore
If k  3, then the system is consistent and it can be ¨ 5 z· ¨ 1· ¨ 5·
X  ©© z 2 ¸¸  z ©© 1 ¸¸ ©© 2 ¸¸
written as
¨ 1 2 3· ¨ 1· ©ª z ¸¹ ©ª 1 ¸¹ ©ª 0 ¸¹
©0 1 1¸ X  © 2 ¸
© ¸ © ¸ is a solution. That is, in this case, all three equations have
©ª0 0 0 ¸¹ ©ª 0 ¸¹
infinitely many solutions. That is, they intersect in a line.
Also R1 2R2 gives If k ≠ 3, then 0 L 0 y 0z  3 k x 0, which is illogical by
Eq. (6.80). Hence, there is no common point.
¨ 1 0 1· ¨ 5· Answer: (B), (C)
©0 1 1¸ X  © 2 ¸
© ¸ © ¸
©ª0 0 0 ¸¹ ©ª 0 ¸¹

Matrix-Match Type Questions


1. Match the items of Column I with those of Column II. x 11 y 2 z 8
 
10 4 11
Column I Column II
(A) The foot of the perpen- (p) (−1, 2, −3) Therefore,
dicular drawn from the x1 11 10t
point (2, −1, 5) onto the
line y1  − 2 − 4t
z1  − 8 − 11t
x 11 y 2 z 8 HHHE
 
10 4 11 for some t  Z . PQ is perpendicular to the given
line. So
is HHHE
(B) The coordinates of the (q) (2, 0, 1) PQ – (10, 4, 11)  0
point on the line joining  (9 10t, 1 4t, 13 11t ) – (10, 4, 11)  0
the points A(1, 2, 3) and
 10(9 10t ) 4( 1 4t ) 121t 143  0
B(3, 5, 9) at a distance 14
units from the midpoint  237t 237  0
of AB are t  1
(C) The point of intersection (r) (1, 2, 3)
and hence
of the planes x 2y − z 
6, 2x − y 3z  − 13 and Q  (x1, y1, z1)  (1, 2, 3)
¥ 19 ´
3x − 2y 3z  − 16 is (s) ¦ 6, , 18µ Answer: (A) m (r)
§ 2 ¶
(D) (1, 3, −2) are the DRs of FHHHE
(B) The equation of the line AB is
line L1 and (−2, 2, 4) are
the DRs of another line (t) ¥ 2, 5 , 6´ x 1 y 2 z 3
¦§ µ¶  
L2. If a line L is perpen- 2 2 3 6
dicular to both L1 and L2,
and hence any point on this line is of the form P(1
then the DRs of L are
2t, 2 3t, 3 6t). Also
Solution: ¥ 7 ´
Q  Midpoint of AB  ¦ 2, , 6µ
(A) Let P  (2, −1, 5) and Q  (x1, y1, z1) be the foot of § 2 ¶
the perpendicular from P onto the line
448 Chapter 6 Three-Dimensional Geometry

Now Column I Column II


PQ  14 (C) The equation of
the locus of the
 ( PQ)2  (14)2 point whose dis-

2 tance from (2,
¥
 (2t 1)2 ¦ 3t µ (6t 3)2  196 −1, 3) is 4 is
§ 2¶
(D) The equation of (s) x 2 y2 z2  0
(6t 3)2 the locus of the
 (2t 1)2 (6t 3)2  196
4 point whose dis-
tance from the
 (16t 2 16t 4) 5(6t 3)2  784 y-axis is equal 2 2 2
to its distance (t) x y z 4 x 2 y 6z
 196t 2 196t 735  0
from the point 20
whose roots are 5 / 2, 3 / 2. Now (2, 1, −1) is

3 ¥ 5 ´ Solution:
t  P  ¦ 2, , 6µ
2 § 2 ¶
(A) The distance of a point from z-axis is x 2 y2 . The
5 ¥ 19 ´ distance of the point from xy-plane is |z|. Therefore
and t  P  ¦ 6, , 18µ
2 § 2 ¶
x2 y2  z2 or x2 y2 − z2  0
Answer: (B) m (s), (t) Answer: (A) m (s)
(C) By solving the three equations simultaneously, we (B) Since the sphere touches yz-plane, its radius is |x|.
get that x  − 1, y  2, z  − 3. Hence, the equation of the sphere is
Answer: (C) m (p)
( x 3)2 ( y 1)2 (z 2)2  32
(D) The DRs of L are givne by
E E E  x 2 y 2 z2 6 x 2 y 4 z 5  0
i j k
E E E E Answer: (B) m (p)
1 3 2  16 i 8k  8(2 i k )
(C) The locus is
2 2 4
( x 2)2 ( y 1)2 (z 3)2  16
Therefore, the DRs of L  (2, 0, 1).
Answer: (D) m (q) x 2 y 2 z2 4 x 2 y 6 z 2  0

Answer: (C) m (t)


2. Match the items of Column I with those of Column II.
(D) We have
Column I Column II
x 2 z2  ( x 2)2 ( y 1)2 (z 1)2
(A) The equation of (p) x 2 y2 z2 6 x 2 y 4z
the locus of a  x 2 z2  ( x 2)2 ( y 1)2 (z 1)2
point whose dis- 5 0
 y 2 4 x 2 y 2z 6  0
tance from the
z-axis is equal Answer: (D) m (q)
to its distance
from the xy- 3. Match the items of Column I with those of Column II.
plane is
(B) The equation of (q) y2 2 y 4 x 4z 6  0 Column I Column II
the sphere with (A) The equation of the plane (p) x 2y 3z
centre at (3, −1, parallel to the plane passing 3
2) and touching through the points (1, 1, 1),
yz-plane is (2, 3, 5) and (−1, 0, 2) which
is at a distance of 2 units
(Continued) from the given plane is
(Continued)
Worked-Out Problems 449

Column I Column II (B) Every point on the lines L1 and L2 is of the form
P(1 3t, 2 t, 3 2t) and Q(3 s, 1 2s, 2 3s),
(B) The equation of the plane (q) 19(x 5y − respectively. Now P  Q gives
passing through the inter- 4z) − 300
section of the lines 0 1 3t  3 s  3t s  2 (6.81)
x 1 y 2 z 3 2 t  1 2s  t 2s  1 (6.82)
L1 :  
3 1 2
3 2t  2 3s  2t 3s  1 (6.83)
x 3 y 1 z 2
L2 :   (r) 2 x 3 y z
1 2 3 From the above equations t  1 and s  1 and the
2 14  0 point
HHHE of intersection of the lines is (4, 3, 5). Now
and perpendicular to the
OP  (4, 3, 5). The required plane is
line joining the point of their
intersection and origin is 4( x 4) 3( y 3) 5(z 5)  0
(C) A plane containing the line (s) 2 x 3 y z  4 x 3 y 5z 50  0
x 1 y 2 z 3 2 14  0
  is Answer: (B) m (t)
1 2 3
(D) The equation of the plane (C) The line
which is parallel to the
plane x 5y − 4z 5  0 (t) x 2y 3z − x 1 y 2 z 3
 
and whose sum of the 14  0 1 2 3
intercepts on the coordi-
nate axes is 15 is passes through (1, 2, 3). Since the plane passes
through (1, 2, 3) which is having normal DRs (1, 2,
Solution: 3), its equation is

(A) The equation of the plane passing through the 1(x 1) 2(y 2) 3(z 3)  0
points (1, 1, 1) (2, 3, 5) and (−1, 0, 2), from Theorem  x 2y 3z  0
6.49, is Answer: (C) m (p)
x 1 y z 2
(D) The plane parallel to the plane x 5y − 4z 5  0 is
1 2 4 0 x 5y − 4z  k. Now,
2 1 1
15  Sum of the intercepts on the axes
 6( x 1) 9 y 3(z 2)  0 k k
k
 6 x 9 y 3z  0 5 4
 2 x 3y z  0
20k 4k 5k 19k
Any plane parallel to the above plane is of the form  
20 20
2x − 3y z d  0. Since the distance between those
two planes is 2, we have Therefore, k  300 / 19. Hence, the equation of the
plane
| d 0|
2 300
2 ( 3)2 12
2
x 5 y 4z 
19
 d  p 2 14 Answer: (D) m (q)
Answer: (A) m (r), (s)

Comprehension Type Questions


1. Passage: The equation of the plane passing through (i) The equation of the plane passing through the
a point (x0, y0, z0) and having the vector (a, b, c) as points A(2, 1, 0), B(5, 0, 1) and C(4, 1, 1) is
normal is
(A) x y − 2z − 3  0
a(x − x0) b(y − y0) c(z − z0)  0 (B) x − y 2z − 3  0
and this plane is also parallel to the plane ax by cz (C) x y 2z − 3  0
d  0. Answer the following questions. (D) x y − 2z 3  0
450 Chapter 6 Three-Dimensional Geometry

(ii) The equation of the plane passing through the is the cross product of the normals (a1, b1, c1) and (a2, b2,
point (2, −3, 1) and perpendicular to the vector c2) of the given planes. Answer the following ques-
E E E
3i 4 j 7k is tions.

(A) 3x 4y 7z 11  0 (i) The equation of the line drawn from (1, −1, 0) to
intersect the lines
(B) 3x 4y 7y − 1  0
x 2 y 1 z 3 x 4 y z 1
(C) 3x 4y 7z 12  0   and  
2 3 4 4 5 2
(D) 3x 4y 7z − 12  0
orthogonally is
(iii) The equation of the plane through the point (−3,
−3, 1) and normal to the line joining the points
x 1 y 1 z 1
(2, 6, 1) and (1, 3, 0) is (A)  
14 12 2
(A) x 3y z − 11  0
x 1 y 1 z
(B) 2x y z 11  0 (B)  
14 12 2
(C) x 3y z 11  0
x 1 y 1 z
(D) x 2y 3z − 11  0 (C)  
14 12 2
Solution:
x 1 y 1 z 1
(i) The normal to the required plane is (D)  
14 12 2
E E E
i j k (ii) The symmetric form of the equation of the line
HHHHE HHHHE E E E 4x − 4y − z 11 0  x 2y − z − 1 is
AB s AC  3 1 1  i j 2k
2 0 1 x 2 y z 3
(A)  
1 1 4
The plane is passing through A(2, 1, 0). Hence, its
x 2 y 2 z
equation is (B)  
2 1 4
−1(x − 2) − 1(y − 1) 2(z − 0)  0
x y 2 z 3
−x − y 2z 3  0 (C)  
2 1 4
x y − 2z − 3  0 x 3 2 y 5 4 z
Answer: (A) (D)  
2 1 4
(ii) The equation of the plane through
E E (2,E −3, 1) and Solution:
perpendicular to the vector 3i 4 j 7k is
(i) The DRs of the required line are given by
3(x − 2) 4(y 3) 7(z − 1)  0 E E E
 3x 4y 7z − 1  0 i j k
E E E
Answer: (B) 2 3 4  14 i 12 j 2k
4 5 2
(iii) The normal to the plane is (1 − 2, 3 − 6, −1)(−1, −3,
−1). Hence the equation of the plane is
Therefore, the DRs of the line are (14, −12, 2). The
−1(x 3) − 3(y 3) − 1(z − 1)  0 line passes through (1, −1, 0). Hence, its equation is
 −x − 3y − z − 12  0 x 1 y 1 z
 
 x 3y z 11  0 14 12 2
Answer: (C) Answer: (B)
(ii) The DRs of the line of intersection are
2. Passage: The equation of the line passing through a
E E E
point (x0, y0, z0) and having DRs (l, m, n) is i j k
E E E E E E
x x0 y y0 z z0 4 4 1  6 i 3 j 12k  3(2 i j 4k )
 
l m n 1 2 1
Also the DRs of the line represented by the planes
The DRs are (2, 1, 4). In the given equations, substi-
a1x b1y c1z d1  0  a2x b2y c2z d2
tuting z  0, we get
Worked-Out Problems 451

4x − 4y  − 11 Now
E E E
and x 2y  1 i j k
HE
H HHE E E E
Solving the equations, we get n1 s n2  3 1 2  i 7 j 5k
3 5 1 2 3
x , y
2 4 HE
H HHE
n1 s n2  ( 1)2 ( 7)2 (5)2  75  5 3
Hence, ( 3 / 2, 5 / 4, 0) is a point on the line. There-
fore, its equation is Therefore
HE
H HHE E E E
x (3 / 2) y (5 / 4)) z n1 s n2 i 7 j 5k
  H HHE 
HE
2 1 4 n1 s n2 5 3
Answer: (D)
Answer: (B)
HHHHE
3. Passage: Consider the lines (ii) A  (−1, −2, −1) and B  (2, −2, 3) so that AB  (3, 0, 4).
Now
x 1 y 2 z 1 x 2 y 2 z 3
L1 :   and L2 :  
3 1 2 1 2 3 3 0 4
HHHHE HE
H HHE
Answer the following questions. ¨ AB n1 n2 ·  3 1 2  3(3 4) 4(6 1)
ª ¹
(i) Unit vector perpendicular to both L1 and L2 is 1 2 3
E E E E E E  3 20 x 0
i 7 j 7k i 7 j 5k
(A) (B)
99 5 3 Therefore, L1 and L2 are skew lines. Hence, the
E E E E E E shortest distance is
i 7 j 5k 7 i 7k k
(C) (D) HHHHE E E
5 3 99 AB – (n1 s n2 ) (3, 0, 4) – ( 1, 7, 5) 17
E E  
(ii) The shortest distance between L1 and L2 is n1 s n2 5 3 5 3
17 41 17 Answer: (D)
(A) 0 (B) (C) (D)
3 5 3 5 3 (iii) The plane passingH through
HE HHE the point (−1, −2, −1)
(iii) The distance of the point (1, 1, 1) from the plane whose normal is n1 s n2 is
passing through (−1, −2, −1) and whose normal is −1(x 1) − 7(y 2) 5(z 1)  0
perpendicular to both L1 and L2 is
x 7y − 5z 10  0 (6.84)
2 7 13 23
(A) (B) (C) (D) Now, the distance of the plane provided in Eq.
75 75 75 75
(6.84) from the point (1, 1, 1) is
(IIT-JEE 2008)
1 7 5 10 13
Solution: L1 and 
HE
H HHE L2 are parallel to the vectors 2 2 2 75
n1  (3, 1, 2) and n2  (1, 2, 3), respectively. 1 7 5
HE
H HHE Answer: (C)
(i) Unit vector perpendicular to both n1 and n2 is
HE
H HHE
n1 s n2
p HE
H HHE
n1 s n2

Integer Answer Type Questions


1. If the distance between the plane ax − 2y z  d and
HE
H HHE
Solution: Let n1  (2, 3, 4) and n2  (3, 4, 5). So
the plane containing the lines E E E
i j k
x 1 y 2 z 3 x 2 y 3 z 4 HE
H HHE E E E
  and   n1 s n2  2 3 4  i 2j k
2 3 4 3 4 5
3 4 5
is 6, then the value |d| is _______.
452 Chapter 6 Three-Dimensional Geometry

Hence, the equation of the plane containing the given Let D  (x1, 0, 0), E  (0, y1, 0) and F  (0, 0, zl) which are
lines is the midpoints of the sides BC, CA and AB, respectively.
Then
−1(x − 1) 2(y − 2) − 1(z − 3)  0
1
 x − 2y z  0 EF || BC and EF  BC  4( y12 z12 )  a 2
2
This plane is parallel to the given plane if a  1. Hence,
the distance between the plane is Similarly

d 4( x12 y12 )  c 2 and 4(z12 x12 )  b2


 6
12 2 2 12 Therefore
Thus, |d|  6. 8( x12 y12 z12 )  a 2 b2 c 2
Answer: 6
A
2. Point (@, A, F) lies in the plane x y z  2. Let
E E E E E E E E
a  A i B j G k and k s (k s a )  0, then F  _______.
(IIT-JEE 2006)
Solution: By hypothesis, we have F (0, 0, z1) E (0, y1, 0)

@ A F2 (6.85)
Therefore
E E E E E E E E E E
0  k s (k s a )  (k – a )k (k – k )a B D (x1, 0, 0) C
E E E
0G k a FIGURE 6.40
E E Answer: 8
a G k
so that @  0 and A  0. Hence, from Eq. (6.85), we get 5. If the point S(3, 3, k) lies in the plane of the triangle
F  2. with vertices A(1, 1, 1), B(0, −1, 0) and C(2, 1, −1), then
Answer: 2 the value of k is equal to _______.
HHHHE HHHHE
Solution: AB  ( 1, 2, 1) and AC  (1, 0, 2) so
3. The distance between the planes 2x − y 2z − 4  0
that
and 2x − y 2z − 1  0 is _______. E
E E
Solution: The distance is i j k
HHHHE HHHHE
AB s AC  1 2 1  4 x 3 y 2z
4 1 3
 1 1 0 2
2
2 ( 1) 2 23 2

Since the plane having (4, −3, 2) as normal passes through


Answer: 1
the point B(0, −1, 0), its equation is
4. A triangle is so placed that the midpoints of its sides are 4(x − 0) − 3(y 1) 2(z − 0)  0
placed on the coordinate axes. If a, b and c are the sides
4x − 3y 2z − 3  0
of the triangle, then the equation of the triangle is
x y z S(3, 3, k) belongs to the plane. So
1
x1 y1 z1 4(3) − 3(3) 2k − 3  0  k  0
where a 2 b2 c 2  k( x12 y12 z12 ). The value of k is Answer: 0
equal to _______.
Solution: See Fig. 6.40. Let the intercepts of the plane 6. If the coordinates of the points A, B, C and D are (3, 6,
on the axes be (x1, 0, 0), (0, y1, 0) and (0, 0, z1) so that the 4), (2, 5, 2), (6, 4, 4) and (0, 2, 1), respectively, then the
HHHHE HHHE
equation of the plane is length of the projection of AB on CD is _______.
HHHHE HHHE
x y z Solution: AB  ( 1, 1, 2) Hand HHHE CD HHHE ( 6, 2, 3).
1 The length of the projection of AB on CD is
x1 y1 z1
Worked-Out Problems 453

HHHHE HHHE 9. If the equation of the plane through the intersection


AB – CD 6 2 6
HHHE  2 of the planes x 2y 3z − 4  0 and 2x y − z 5 
CD 7 0 and perpendicular to the plane 5x 3y 6z 8  0
is ax by cz 173  0, then b − 9 (a c) is equal to
Answer: 2 _______.

7. If the plane 2ax − 3ay 4az 6  0 passes through the Solution: The required plane is of the form
midpoint of the centres of the spheres (x 2y 3z − 4) L (2x y − z 5)  0
x2 y2 z2 6x − 8y − 2z − 13  0 whose normal is (1 2 L , 2 L , 3 L ). This plane is per-
and x2 y2 z2 − 10x 4y − 2z − 8  0 pendicular to the plane 5x 3y 6z 8  0. So we have

then a is equal to _______. 5(1 2 L ) 3 (2 L ) 6(3 L )  0

Solution: The centre of the sphere  7 L  29

x2 y2 z2 2ux 2vy 2wz c  0 29


L 
7
is (− u, − v, − w). Hence, the centre of the given spheres are
A(−3, 4, 1) and B(5, −2, 1) whose midpoint is (1, 1, 1). Since Therefore, the required plane is
(1, 1, 1) lies on the plane 2ax − 3ay 4az 6  0, we have
29
( x 2 y 3z 4) (2 x y z 5)  0
2a − 3a 4a 6  0  a  −2 7
Answer: 2  51x 15 y 50z 173  0

8. If d is the shortest distance of the point (1, 2, −1) from comparing this with ax + by + cz + 173 = 0 we get a  51,
the surface x2 y2 z2  54, then d / 6 is equal to b  15, c  50.
_______. so that

Solution: See Fig. 6.41. The equation b − 9 (a c)  15 − 9  6

x2 y2 z2  54 Answer: 6
is a sphere with centre (0, 0, 0) and radius  54  3 6.
10. The equation of the planes passing through the line of
P(1, 2, −1) is a point inside the sphere such that
intersection of the planes x 3y 6  0  3x − y − 4z
2
OP  1 2 ( 1)  6 2 2 which is at a distance of 1 unit from the origin are 2x
y − 2z d1  0 and x − 2y − 2z d2  0. Then − d1d2
Thus distance of P from the surface of the sphere is is equal to _______.
d  3 6 62 6 Solution: The required plane is of the form
Therefore ( x 3 y 6) L (3 x y 4z)  0

d 2 6  (1 3L ) x (3 L ) y 4 L z 6  (6.86)
 2
6 6 The distance of this plane provided in Eq. (6.86) from the
origin  1. This gives
6
1
(1 3L ) (3 L )2 16 L 2
2

P (1, 2, −1)  36  26 L 2 10
O (0, 0, 0)
L p1
Therefore, the required planes are obtained by substitut-
ing L  p 1 in Eq. (6.86) and they are 2x y − 2z 3  0
and x − 2y − 2z − 3  0. Therefore

FIGURE 6.41
d1d2  3( 3)  9
Answer: 2 Answer: 9
454 Chapter 6 Three-Dimensional Geometry

SUMMARY
6.1. Theorem: The distance between the points A(x1, y1, 6.7. Theorem: If (l, m, n) are the direction cosines of a
FHHHE
z1) and B(x2, y2, z2) is line AB and P  (x1, y1, z1) and Q  (x2, y2, z2), then
FHHE FHHHE
the projection vector of PQ on AB is
( x1 x2 )2 ( y1 y2 )2 (z1 z2 )2 E E E
l ( x2 x1 ) i m ( y2 y1 ) j n (z2 z1 ) k
6.2. Theorem (Section formula): Let A  (x1, y1, z1) and
B  (x2, y2, z2) be two points and P(x, y, z) be a point 6.8. Theorem: If (l, m, n) are the DCs of the normal of
FHHHE
on the line AB such that a plane whose distance from the origin is p, then
AP : PB  l : m (l m x 0) the equation of the plane is lx my nz  p. This
equation of the plane is called normal form.
Then
6.9. Definition: If a plane meets x-, y- and z-axes at the
lx2 mx1 ly my1 lz mz1 points (a, 0, 0), (0, b, 0) and (0, 0, c), respectively,
x , y 2 ,z 2
l m l m l m then a, b, c are called the intercepts of the plane on
the coordinate axes.
The coordinates of midpoint are
¥ x1 x2 y1 y2 z1 z2 ´ 6.10. Theorem (Equation of the plane in intercept form):
¦§ , , µ If a, b, c are the intercepts (abc x 0) of a plane on
2 2 2 ¶
the coordinate axes, then its equation is
6.3. The centroid of a triangle with vertices A(x1, y1, z1),
B(x2, y2, z2) and C(x3, y3, z3) is x y z
1
a b c
¥ x1 x2 x3 y1 y2 y3 z1 z2 z3 ´
¦§ , , µ¶ 6.11. Definition: If a, b, c and d are real and at least one
3 3 3
of a, b and c is not zero, then the equation ax by
and centroid of a tetrahedron with vertices (xr, yr, zr) cz d  0 is called a first-degree equation in x, y
(where r  1, 2, 3 and 4) is and z.
¥ x1 x2 x3 x4 y1 y2 y3 y4 z1 z2 z3 z4 ´ 6.12. Theorem: The equation of a plane is a first-degree
¦§ , , µ¶
4 4 4 equation in x, y and z and every first-degree
6.4. Definition (DCs and DRs of a line): Let L be any equation in x, y and z represents a plane.
line in the space. Any unit vector parallel to the 6.13. Theorem: (1) Two first-degree equations
line L is called DC (direction cosine) of L and any
vector parallel to L is called DR (direction ratio) a1x b1y c1z d1  0 and a2x b2y
of the line L. If a line makes angles @, A and F with c2z d2  0
the x-, y- and z-axes, respectively, and (l, m, n) are represent the same plane if and only if
the DCs of L, then l  cos A , m  cos B, n  cos G or
a1 : b1 : c1 : d1  a2 : b2 : c2 : d2
l  cos A , m  cos B, n  cos G . That is, (cos @,
cos B, cos G ),( cos A , cos B, cos G ) are the DCs of (2) Two first-degree equations
L. a1x b1y c1z d1  0 and a2x b2y c2z d2  0
represent parallel planes if and only if a1 : b1 : c1 
6.5. Theorem: If (l, m, n) are the DCs of a line, then l2
a2 : b2 : c2 .
m2 n2  1.
Note: If (a, b, c) are the DRs of a line, then its DCs 6.14. Theorem: (1) The perpendicular distance of a point
are P(x1, y1, z1) from a plane ax by cz d  0 is

¥ ´ ax1 by1 cz1 d


pa pb c
¦ , ,p µ a 2 b2 c 2
¦§ a 2 b2 c 2 a 2 b2 c 2 a 2 b2 c 2 µ¶
(2) The distance between two parallel planes ax
where we have to take all signs or all – signs.
by cz d1  0 and ax by cz d2  0 is
6.6. Theorem: The direction ratios of the line joining
d1 d2
the two points A(x1, y1, z1) and B(x2, y2, z2) are (x2
− x1, y2 − y1, z2 − z1). a 2 b2 c 2
Summary 455

6.15. Theorem: The equation of the plane passing Case 1: If a1a2 b1b2 c1c2 > 0, then the origin lies
E
through the point (x1, y1, z1) and having n  (a, b, c) on the obtuse angle region and hence the acute
as normal is bisector is
a(x − x1) b(y − y1) c(z − z1)  0 a1 x b1 y c1z d1 a2 x b2 y c2 z d2

6.16. Theorem: The equation of the plane determined by a12 b12 c12 a22 b22 c22
three non-collinear points (xr, yr, zr) (where r  1, 2 Case 2: If a1a2 b1b2 c1c2 < 0, then the origin lies
and 3) is on the acute angle region and hence the acute angle
x x1 y y1 z z1 bisector is
x2 x1 y2 y1 z2 z1  0 a1 x b1 y c1z a2 x b2 y c2 z d2

x3 x1 y3 y1 z3 z1 a12 b12 c12 a22 b22 c22

6.17. Notation: If E y ax by cz d, then E11 means 6.22. Foot of the perpendicular and image: Let E yax
ax1 by1 cz1 d. by cz d  0 be a plane and P(x1, y1, z1) be a point
which is not on the plane.
6.18. Theorem: If a plane E  0 cuts a line joining two
(i) If ( x1 a, y1 a, z1 a) is the foot of the perpendicu-
points A(x1, y1, z1) and B(x2, y2, z2) at a point P,
lar drawn from P(x1, y1, z1) onto the plane,
then the ratio
then
AP : PB  E11 : E22 x1a x1 y1a y1 z1a z1 (ax1 by1 cz1 d)
  
Further, A and B are on opposite sides the plane E a b c a 2 b2 c 2
 0 if E11 and E22 are of opposite signs and A and B
lie on the same side if E11 and E22 are of same signs. (ii) If ( x1aa, y1aa, z1aa) is the image of P(x1, y1, z1) in
In particular, if E  ax by cz d  0, then a point the plane E  0, then
(x1, y1, z1) lies in the origin side if E11 and d are x1aa x1 y1aa y1 z1aa z1 (ax1 by1 cz1 d)
   2
of same signs and (x1, y1, z1) lies on the non-origin a b z a 2 b2 c 2
side of the plane, if E11 and d have opposite signs.

6.19. Theorem: If E1  0 and E2  0 are two intersecting 6.23. Angle between two planes: The angle between two
planes which intersect in a line L, then any plane planes is defined to be the angle between their
passing through the line L is of the form normal vectors. If a1x b1y c1z d1  0 and a2x
b2y c2z d2  0 are two planes, then the angle
K1E1 + K2E2 = 0 between the two planes is
where L1 L 2 x 0. For practical purpose, we can
¥ a1a2 b1b2 c1c2 ´
consider such a plane as Cos 1 ¦ µ
E1 L E2  0 ¦§ a 2 b2 c 2 a 2 b2 c 2 µ¶
1 1 1 2 2 2

The converse of this theorem is also true. 6.24. Angle between a line and a plane: If Q is the angle
6.20. Theorem: If E1 y a1 x b1 y c1z d1  0 and E2 y a2x between a line and the normal to a plane, then
b2y c2z d2  0 are intersecting planes, then the 90o Q is called the angle between the line and the
equations of the plane bisecting the angle between plane.
the planes are 6.25. Equation of a line (Symmetric form): The equation
a1 x b1 y c1z d1 a2 x b2 y c2 z of the line which is passing through a point (x0, y0,
p
z0) and having the DRs (a, b, c) is
a12 b12 c12 a22 b22 c22
x x0 y y0 z z0
6.21. To find the acute angle bisector plane: Let E1 ya1x +  
a b c
b1 y c1z d1  0 and E2 y a2 x b2 y c2 z d2  0
be two intersecting lines which do not pass through 6.26. Parametric form: If (x, y, z) is any point on the
the origin. Make both d1 and d2 positive. Then, the line
origin angle bisector is always with signs. That is, x x0 y y0 z z0
 
a1 x b1 y c1z d1 a2 x b2 y c2 z d2 a b c

a12 b12 c12 a22 b22 c22 then x  x0 L a, y  y0 L b and z  z0 L c, where
L Z . These equations are called parametric
456 Chapter 6 Three-Dimensional Geometry

equations of the line passing through the point (x0, a1 b1 c1 d1 b1 c1


y0, z0) and having DRs (a, b, c).
$  a2 b2 c2 , $ 1  d2 b2 c2 ,
6.27. To write the line of intersection of two planes in a3 b3 c3 d3 b3 c3
symmetric form: Suppose the planes E1 y a1x +
a1 d1 c1 a1 b1 d1
b1 y c1z d1  0 and E2 y a2x b2y c2z d2  0
intersect in the line L so that L is parallel to the $ 2  a2 d2 c2 , $ 3  a2 b2 d2
cross product of the normals (a1, b1, c1) and (a2, b2, a3 d3 c3 a3 b3 d3
c2) of the planes. If a1b2 a2 b1 x 0, then substitute
z  0 in the plane equations and solve for x and y. Case 1: If a1 : a2 : a3  b1 : b2 : b3  c1 : c2 : c3 , then the
Suppose (x0, y0, 0) is a point on the line. Then, the planes are parallel and hence no common points.
symmetric form of the line L is Case 2: If $ x 0, then the three planes intersect in
x x0 y y0 z 0 unique point.
  . Case 3: If $  0 and one of $ 1 , $ 2 and $ 3 is zero
b1c2 b2 c1 a2 c1 a1c2 a1b2 a2 b1
and hence all of them are zero, then the three
6.28. Nature of three planes: Let E1 y a1x b1y c1z
planes intersect in a single line.
d1  0, E2 y a2 x b2 y c2 z d2  0 and E3 y a3x Case 4: If $  0 and at least one of $ 1 , $ 2 and $ 3
b3 y c3 z d3  0 be three planes and is not zero, then the three planes form a triangular
prism. That is, the planes intersect pairwise in par-
allel lines.

EXERCISES
Single Correct Choice Type Questions
1. The equation of the plane passing through the line of 5. Point of intersection of the lines
intersection of the planes x y z  1 and 2x 3y − z x 4 y 6 z 1
 − 4 and parallel to x-axis is  
3 5 2
(A) y 4z − 7  0 (B) 2y − 4z − 7  0
and 3x − 2y z 5  0  2x − 3y 4z − 4
(C) y − 2z 7  0 (D) y − 4z 7  0 is
2. The equation of the plane determined by the points (A) (2, 4, −3) (B) (−4, −6, 1)
(0, 4, 3) (−2, − 2, 1) and (1, 1, −1) is (C) (−2, 4, 3) (D) (−2, 4, −3)
(A) 9x 5y 6z 2  0
6. The planes 2x y z 4  0, 3x 2y z 8  0 and
(B) 9x − 5y 6z 2  0 y−z 40
(C) 9x − 5y − 6z 2  0 (A) do not have common points.
(D) 9x − 5y 6z − 2  0 (B) have a unique common point.
(C) form a triangular prism.
3. The image of the point (2, −1, 3) in the plane 3x − 2y (D) intersect in a line.
z  9 is
(Hint: Use item 6.26 of the section “Summary”)
(A) (1, 1, 8) (B) (2, −1, 2)
7. The vertices of a triangle ABC are A(−1, −2, −3),
¥ 8 3 19 ´
(C) (−2, −2, −2) (D) ¦ , , µ B(−1, 2, 3) and O(0, 0, 0). The direction ratios of the
§7 7 7 ¶
internal bisector of the angle AOB are
4. The equation of the plane through the line (A) 0, 0, 1 (B) −1, 0, 0
x 1 y 2 z 3 (C) −1, 1, 1 (D) −1, −1, −1
 
2 3 4
8. If the line
and parallel to y-axis is
x y z
(A) 2x z 1  0 (B) 2x − z 1  0  
1 1 1
(C) x − 2z 1  0 (D) x 2z − 1  0 meets the line
Exercises 457

x sin A y sin B z sin C 2d 2  0 to the intersection of the planes 3x − y − 3z 32  0


and 3x 2y − 15z − 8  0 are
 x sin 2 A y sin 2 B z s sin 2C d 2
(A) (9, 1, 7) (B) (7, 9, 1)
then the value of
(C) (−7, −1,− 9) (D) (−7, −9, −1)
A B C
sin sin sin 16. The distance of the point (−2, 3, −4) from the line
2 2 2
where A B C  P is x 2 2 y 3 3z 4
 
3 4 5
1 1 1 1
(A) (B) (C) (D) measured parallel to the plane 4x 12y − 3z 1  0
16 32 8 12
is
9. The image of the point (−1, 3, 4) on the plane x −2y 
0 is 15 17 13
(A) (B) (C) (D) 6
2 2 2
¥ 17 19 ´
(A) (15, 11, 4) (B) ¦ , , 1µ
§ 3 3 ¶ 17. The line
¥ 17 19 ´ x 2 y 1 z 2
(C) (8, 4, 4) (D) ¦ , , 4µ  
§ 3 3 ¶ 3 4 1
10. The plane 2x y 2z − 9  0 meets the coordinate meets the plane x − y z  5 at Q. If P is the point
axes at points A, B and C. Then the orthocentre of (−1, −5, −10), then the distance PQ is
$ ABC is 13
(A) 10 (B) 11 (C) (D) 13
¥ 3 3´ 2
(A) ¦ , 3, µ (B) (2, 1, 2)
§ 2 2¶
18. If the lines
¥ 9 27 9 ´ ¥ 3 1 3´ x 1 y 2 z 3
(C) ¦ , , µ (D) ¦ , , µ
§ 7 7 7¶ § 13 13 13 ¶  
3 2L 2
11. The foot of the perpendicular drawn from (0, 0, 0) to x 1 y 5 z 6
the line joining the points (−9, 4, 5) and (10, 0, −1) is and  
3L 1 5
(A) (3, 4, 5) (B) (1, 2, 2)
are at right angles to each other, then L is equal to
¥ 58 112 109 ´
(C) (4, 5, 3) (D) ¦ , , 9 9 10 10
§ 59 59 59 µ¶ (A) (B) (C) (D)
7 7 7 7
12. The distance of the point (−2, 3, 1) from the line 19. The shortest distance between any two opposite edg-
through the point (−3, 5, 2) and making equal angles es of a tetrahedron formed by the planes y z  0, z
with the coordinate axes is x  0, x y  0 and x y z  1 is
2 16 5 14 1 2 1
(A) (B) (C) (D) (A) (B) (C) (D) 2 3
3 3 3 3 3
6 3
13. The shortest distance between the lines
20. The equation of the plane containing the line
x 1 y 2 z 3 x 5 y 4 z 5
  and   is y z
2 3 4 4 4 5  1, x  0
b c
(A) 0 (B) 2/3 (C) 2 3 (D) 3 2
and parallel to the straight line
14. The point of intersection of the lines whose paramet- x z
ric equations are x  3r 3, y  − 4r 2, z  r − 1 and  1, y  0
a c
x  − 6t 1, y  3t − 2 and z  1 is
is
(A) (9, 6, −1) (B) (9, −6, 1)
x y z x y z
(C) (−9, 6, −1) (D) (9, 6, 1) (A) 1 (B) 1 0
a b c a b c
15. The coordinates of the point on the line joining the x y z x y z
(C) 1 0 (D) 1
points (−3, 7, −13) and (−6, 1, −10) which is nearest a b c a b c
458 Chapter 6 Three-Dimensional Geometry

Multiple Correct Choice Type Questions


1. For the lines and
x 3 y 8 z 3 x  2 3t, y  4t and z2
 
3 1 1 Identify the correct equations from the following:
x 3 y 7 z 6
and   (A) 4x 3y − 10  0  2y 4z − 4
3 2 4
(B) 4x 3y 10  0  2y 4z − 4
choose the correct choices.
(C) 4x − 3y − 8  0  z − 2
(A) Coplanar and non-parallel. (D) 4x 3y 8  0  z − 2
(B) Skew lines.
(C) The shortest distance between them is 3 30. 4. The symmetric and the parametric forms of the line
2x − y z − 5  0 x y − 2z − 1 are
(D) Standard deviation is 0.
x 2 y 1 z
2. Consider the line (A)  
1 5 3
x 1 y 2 z 3
L:   x 2 y 1 z
2 3 4 (B)  
2 10 6
and the point P(5, 9, 3) in the space. Then
(C) x  2 t, y  −1 5t, z  3t
(A) the distance of P from the line is 6. (D) x  2 2t, y  1 5t, z  − 3t
(B) the distance of P from the line is 6.
(C) the foot of the perpendicular from P onto L is 5. Consider the plane 2x y z  0 and the point P(3, 5,
(3, 5, 7). 7). Then, the
(D) the equation of the perpendicular from P onto (A) foot of the perpendicular from P onto the given
the line L is plane is (−3, 2, 4).
x 5 y 9 z 3 (B) image of P in the given plane is (−9, −1, 1).
 
2 4 4 (C) equation of the line joining the points (3, 5, 7)
x 3 y 5 z 7
3. For the two lines
and (−9, −1, 1) is   .
6 6 4
x 1 y 2 z (D) distance of (−9, −1, 1) from the plane 2x y z 
  0 is 9.
3 4 2

Matrix-Match Type Questions


In each of the following questions, statements are given in p q r s t
two columns, which have to be matched. The statements in A
column I are labeled as (A), (B), (C) and (D), while those
B
in column II are labeled as (p), (q), (r), (s) and (t). Any
given statement in column I can have correct matching with C
one or more statements in column II. The appropriate bub- D
bles corresponding to the answers to these questions have
to be darkened as illustrated in the following example.
1. Match the items of Column I with those of Column II.
Example: If the correct matches are (A) m (p), (s), (B) m
(q), (s), (t), (C) m (r), (D) m (r), (t), that is if the matches Column I Column II
are (A) m (p) and (s); (B) m (q), (s) and (t); (C) m (r);
(A) The equation of the (p) 2 x 3 y 2 3 z 11  0
and (D) m (r), then the correct darkening of bubbles will
plane which is at a
look as followes:
distance 10 from the
origin, and whose
normal has DRs (3, 2,
6) is
(Continued)
Exercises 459

Column I Column II Column I Column II


(B) The equation of the (q) 3x 2y 6z  70 (D) The equation of the plane (s) 2x − y z − 7  0
plane which is at a dis- passing through the point
tance of 5 units from (2, 1, −1) and passing
the origin and is per- through the line of inter-
pendicular to the section of the plane x (t) x 9y 11z  0
vector (2, −3, 6) is (r) 2 x 3 y 2 3 z 1  0 3y −z  0  y 2z is
(C) The equation of the
plane which is at a 3. Match the items of Column I with those of Column II.
distance of one unit
from the plane Column I Column II
2 x 3 y 2 3 z 6  0 (s) 2x − y 3z 8  0
(A) The distance of the point (2, 5, (p) 105
is
−3) from the plane 6x − 3y
(D) The equation of the 2z − 4  0 is
plane passing through
(B) If the lines
the point (1, 4, −2)
(t) 2x − 3y 6z  35
and parallel to the x 1 y 1 z 1
  (q) 7
plane 2x − y 3z  0 2 3 4
is
x 3 y k z
and  
2. Match the items of Column I with those of Column II. 1 2 1
intersect, then the distance
Column I Column II between the points (1, −1, 1) 10
(r)
and (3, 2k, 0) is 3 3
(A) The equation of the (p) 3x 4y − 5z −9  0
plane passing through (C) The distance of the point (1, 0,
the point (−1, −1, 2) and −3) from the plane x − y − z −
perpendicular to the 9  0 measured parallel to the
planes 3x 2y − 3z −1  0 line
10
and 5x − 4y z −5  0 is (s)
x 2 y 2 z 6 3
(B) The equation of the (q) x 11y 9z  0  
2 3 6
plane passing through
the points (2, 2, 1) and is
(9, 3, 6) and perpendicu- 13
(D) The distance of the line (t)
lar to the plane 2x 6y 7
6z −1  0 is x 2 y 2 z 3
 
(C) The equation of the (r) 5x 9y 11z  0 1 1 4
plane passing through from the plane x 5y z −
the point (3, 1, 2) and 5  0 is
perpendicular to the
vector (2, −1, 1) is

(Continued)

Comprehension Type Questions


1. Passage: Consider the plane x y − z − 1  0, the point (i) Equation of the plane containing the line L and
P(1, 2, −3) and the line the point P(1, 2, −3) is
x 1 y 2 z 3 (A) x 3y − 7  0 (B) 3x y − 5  0
L:  
3 1 4 (C) x − 3y 5  0 (D) x 3y − z − 7  0
Answer the following questions.
460 Chapter 6 Three-Dimensional Geometry

(ii) The distance between the points on the line L (iii) If a b c  0 and a 2 b2 c 2  ab bc ca, then
which are at a distance of 2 / 3 from the plane is the
2 (A) three planes have unique common point.
(A) 4 3 (B)
3 (B) common solutions line on a line only.
(C) 104 (D) 2 3 (C) three planes are identical.
(iii) If Q is a point on the line L such that PQ is paral- (D) three equations represent Z 3 .
lel to the plane, then the coordinates of Q are
3. Passage: If a plane meets the coordinate axes at the
(A) (−8, 5, −9) (B) (4, 1, 7) points (a, 0, 0), (0, b, 0) and (0, 0, c), then its equation
(C) (−5, 4, −5) (D) (10, −1, 15) is
x y z
2. Passage: Consider the planes ax by cz  0, bx cy 1
a b c
az  0 and cx ay bz  0. Answer the following
questions. (i) A plane cuts the coordinate axes at A(2, 0, 0),
2 2 2 B(0, 3, 0) and C(0, 0, 1). Then the area of the tri-
(i) If a b c x 0 and a b c  ab bc ca,
angle ABC is
then the
11 9 7 5
(A) three planes have no non-zero solution. (A) (B) (C) (D)
2 2 2 2
(B) three planes intersect in a single point only.
¥ 1 2 2 2 2 ´
(C) three planes intersect in a line. ¦§ Hint : Area  b c c a a 2 b2 µ
2 ¶
(D) three planes are identical.
(ii) If a b c  0 and a 2 b2 c 2 x ab bc ca, then (ii) The volume of the tetrahedron OABC, where O
the is the origin, is
1
(A) planes intersect in the line x  y  z. (A) 6 (B) 1 (C) 2 (D)
6
(B) planes do not have common point. (iii) The centroid of tetrahedron OABC is
(C) planes have infinitely many common points
among which (1, 2, 3) is one such common ¥ 1 3 1´ ¥ 1 3 1´
(A) ¦ , , µ (B) ¦ , , µ
point. § 2 4 4¶ § 4 4 4¶
(D) planes form a triangular prism. ¥ 2 1´ ¥ 1 1 1´
(C) ¦ , 1, µ (D) ¦ , , µ
§ 3 3¶ § 3 3 3¶

Integer Answer Type Questions


The answer to each of the questions in this section is a 1. If a line makes angles @, A, F and C with the diagonals
non-negative integer. The appropriate bubbles below the of a cube, then
respective question numbers have to be darkened. For
example, as shown in the figure, if the correct answer to 3(cos2 A cos2 B cos2 G cos2 D )
the question number Y is 246, then the bubbles under Y is ______.
labeled as 2, 4, 6 are to be darkened.
X Y Z W 2. A(3, 2, −4), B(5, 4, −6), and P(9, 8, −10) are three col-
0 0 0 0 linear points. If the ratio AP:PB  l:m, then l m is
1 1 1 1 equal to (l and m do not have common factor s x 1 )
2 2 2 ______.
3 3 3 3
3. If yz-plane divides the line joining the point A(−2, 4, 7)
4 4 4
and B(3, −5, 8) in the ratio 2/ k, then the value of k is
5 5 5 5 ______.
6 6 6
7 7 7 7 4. If the lines x  ay b, z  cy d and x  aay ba, z  cay
8 8 8 8 da are perpendicular to each other, then aaa cca
9 9 9 9 k  0, where the value of k is______.
Answers 461

5. If p is the shortest distance between the lines 6. The equation of the plane passing through the points
(−1, 1, 1) and (1, −1, 1) and perpendicular to the plane
x 1 y 2 z 1 x 2 y 1 z 1
  and   x 2y 2z − 5  0 is 2x 2y − 3z d  0, where the
2 1 2 2 1 2 value of d is______.
then the integral part of p ([p]) is______.

ANSWERS
Single Correct Choice Type Questions
1. (A) 11. (D)

2. (B) 12. (D)

3. (D) 13. (A)

4. (B) 14. (B)

5. (A) 15. (C)

6. (D) 16. (B)

7. (B) 17. (D)

8. (A) 18. (C)

9. (D) 19. (B)

10. (B) 20. (C)

Multiple Correct Choice Type Questions


1. (B), (C) 4. (A), (B), (C)

2. (A), (C) 5. (A), (B), (C)

3. (A), (C)

Matrix-Match Type Questions


1. (A) m (q); (B) m (t); (C) m (p), (r); (D) m (s) 3. (A) m (t); (B) m (p); (C) m (q); (D) m (r)

2. (A) m (r); (B) m (p); (C) m (s); (D) m (t)

Comprehension Type Questions


1. (i) m(A); (ii) m (C); (iii) m (A) 3. (i) m(C); (ii) m (B); (iii) m (A)

2. (i) m(D); (ii) m (A); (iii) m (D)

Integer Answer Type Questions


1. 4 4. 1
2. 5 5. 3

3. 3 6. 3
Index
A Centres of similitude, 208 of midpoint, 454
Centroid, 5 of a point, 399–401
Abscissae of the points of intersection, coordinates of, 401 rectangular, 2
295 position vector of, 391 rectangular cartesian, 2
Acute angle bisector, identification of, of tetrahedron, 391 of a tetrahedron, 401
43–45 of a triangle, 454 Cross product of vectors, 396
Algebraic surface, 410 Ceva’s theorem, 52–53 Curve
Angle Changes of axes, 7 conjugate axis of, 323
of intersection, 154 Chord features of, 323, 376
of intersection formula, 207 of contact, 135, 206, 224 latus rectum of, 323
of intersection of circles, 206 equation of, on the ellipse, 292–293 properties of, 373–374
between a line and a plane, 455 equation of, using midpoint, 226 transeverse axis of, 323
between line and plane, 391 of parabola, 224 vertices of, 323
between two lines, 33–34 Circle (s), 125
between two non-zero vectors, 393 angle of intersection of, 206
auxiliary, 291
between two planes, 396, 455
classification of points in a plane
D
between vectors, 393
Area of a triangle, 4 w.r.t. a, in the same plane, 133–
Degenerate cones, 212
Asymptotes of hyperbola, 322, 326 134
Diameter of the ellipse, 301
angle between two, 327, 377 common tangents of two, 159–160,
Diameter of the parabola, 226–227
chord of contact of, 326 207
Direction cosines (DCs), 402–403, 454
combined equation of pair of, 328 nine-point, 6
of array, 403
of conjugate, 329 orthogonal, 154, 206
of lines, 403
foot of the perpendicular from a parametric equations of, 205
of rays, 403
focus, 327 point, 205
theorems, 403–404
method to find, 328 position of a point with respect a,
Direction ratios (DRs), 402–403, 454
product of the perpendiculars drawn 206
of the normal to plane, 413
from focus, 326–327 relation between a line in its plane
of the normal to the plane, 412
slope of, 326 and, 128–131, 205
of parallel lines, 403
tamgent intercepted between, 332 relation between two, 150–152, 206
of plane, 412
Auxiliary circle of ellipse, 291, 323, 374 universal equation of a, 205
theorems, 404
tangent of an ellipse, 298 Circular cone, 212
Director circle, 134, 206, 221
Axes/axis Circumcentre of triangle, 5
of ellipse, 296–297
changes of, 7 Circumcircle of triangle, 5
Directrix, 213
conjugate, 323 Pedal line or Simson’s line, 6
definition, 373
conjugate, of curve, 323 Circumradius of triangle, 5
of hyperbola, 322
coordinate, 400 Classification of points in a plane
of parabola, 221
of ellipse, 287 w.r.t. a circle in the same plane,
Distance between two points, 3
intercepts of plane on, 411 133–134
Distance function, 386
of inverted ellipse, 288 Collinear vectors, 394–395
Double ordinate, 215, 288
radical, 157–158, 207 Common tangents of two circles, 159–
of ellipse, 374
rotation of, 8–9 160, 207
transeverse, of curve, 323 Condition for line to be tangent, 206
Cone (s), 212
degenerate, 212 E
Conic, 212, 322
B Conjugate axis, 323 Eccentric angle on ellipse, 292
Conjugate diameters, 301 algebraic sum of four points, 304–305
Base curve, 212 Conjugate hyperbola, 328 Eccentricity, 213
Betweenness, 387 asymptote of, 329 definition, 373
eccentricities of, 329 Ellipse, 213, 285
Conormal points, 231, 277 chord of contact of, 303, 374, 376
C Coordinate system, 386–387 condition for tangency, 294–295, 375
of centroid, 401 definition, 373
Cardon’s method, 231 coordinate axis, 400 diameter of, 301
Centre of cone, 289 coordinate planes, 399–400 director circle of, 296–297, 375
464 Index

directrix of, 286, 374 Focus of ellipse, 373–374 L


double ordinate of, 374 Focus of general conic, 213
eccentric angle of point on, 292 Frame of reference, 399, 410 Latus rectum, 215
equation of a tangent, 299, 375 of curve, 323
equation of normal on, 294 of ellipse, 288, 374
equation of the chord joining two
points, 292–293
G length of, 215
Left-handed system, 393
external point of, 290 General conic, 213 Length
foci region of, 325 General equation of a chord, 206
focus of, 286 of a circle, 205 of latus rectum of hyperbola, 323–
interception with circle, 304–305 of a plane, 412, 415 324
internal point of, 290 procedure to find the lines of latus rectum of the ellipse, 288
inverted, 287–288, 374 represented by second-degree (S of the latus rectum of the parabola,
latus rectum of, 288, 374  ax2 2hxy by2 2gx 2fy c 215
major axis of, 287, 374  0), 50 of major axis of ellipse, 287
midpoint of a chord of, 300–301, 375 of a straight line, 30 of minor axis of ellipse, 287
minor axis of, 287, 374 Generator of the cone, 212 of a tangent, 206
number of normals from a point to, Guiding curve, 212 Line (s). see also Straight line
303–304 angle between two, 33–34
pair of tangents drawn from an coordinate system, 387
external point, 302
parallel chords of an, 301 H direction cosines (DCs) of, 403
equations, through the point, 422,
parametric equation, 291–292 425–426
perpendicular tangents to, 297 Horizontal line, 22
Hyperbola, 213, 285 general equation of a straight, 30
as a section of cone, 286 general equation of second-degree
standard, 289 asymptotes of, 322, 326, 376–377
conjugate, 328, 377 (S  ax2 2hxy by2 2gx 2fy
standard equation of, 286–287, 373 c  0), 50
tangents from an external point to, definition of, 322
directrix of, 322 horizontal, 22
296 intercepts of a, 23
vertices of, 287 eccentricity of, 322
external region of, 325, 376 normal form of straight, 27–28
Equation of a chord, 206 pair of, 42–51
of contact, 206 foci region of, 325
focus of, 322 parallel, 387
joining two points, 276 parametric form of, 455–456
joining two points on parabola, 218 internal region of, 325, 376
length of latus rectum of, 323–324 Pedal, 6
in terms of its midpoint, 206 relation between a circle in its plane
Equation of a tangent, 276 locus of, 322
parametric form, 325, 376 and, 128–131, 205
Cartesian form, 206 Simson’s, 6
parametric form, 206 rectangular, 324, 329, 377
standard equation of, 322, 376 skew, 390
Equation of straight line in normal S– Sa 0, 207
form, 27–28 straight, 22
Euclidean geometry theorems, 387 symmetric form of, 29, 422–423,
Euclidean pure geometry, 386 I 455–456
Excentre, 5 vertical, 22
External point of ellipse, 290 Image or a reflection of point, 34 Locus, 6–7
External region of hyperbola, 325 Incentre, 5 equation of, 7
condition for, 325 Incidence postulates, 386 of hyperbola, 322
Intercepts
of a line, 23
F of plane on axes, 411, 454
M
Internal point of ellipse, 290
First-degree equation, 30, 31–32, 410, Internal region of hyperbola, 325
condition for, 325 Major axis of ellipse, 287
412 Menelaus’ theorem, 52
First-degree expression in x and y, 30 Intersection in a plane, 387–388
Inverted ellipse, 287–288 Minor axis of ellipse, 287
First-degree surface, 410
Focal chord, 228–230 directrices of, 288
properties of, 277 foci of, 288
Focal radius, 228, 277 major axis length of, 288 N
Foci region minor axis length of, 288
of ellipse, 325 Nine-point centre, 6
of hyperbola, 325 Nine-point circle, 6
Index 465

Non-collinear points, 386, 455 Parametric equations Rectangular hyperbola, 324, 329
vector area of, 397 of circle, 205 eccentricity of, 324
Non-collinear vectors, 392 of ellipse, 291–292, 374 equation of tangent in parametric
Non-coplanar vectors, 392 of hyperbola, 325 form, 330–331
Non-degenerate cones, 212 of line, 455–456 normals to, 331
Non-empty set, 386 of parabola y2  4ax, 215 parametric equation of, 330, 377
Normal chord, 224 of rectangular hyperbola, 330 simplest form of, 329–330, 377
Normal form of plane, 410 of straight line, 422 standard form of, 324, 377
Normals to rectangular hyperbola, 331 of a tangent, 206 Relation
Normal to an ellipse, 294 vectorial equations of a plane, 393 between a circle and a line in its
number of, from a point, 303–304 Pedal line, 6 plane, 128–131, 205
Normal to a parabola, 222, 277 Plane geometry, 385 between two circles, 150–152, 206
number of, from a point, 231, 277 algebraic surface, 410 Right circular cone, 212–213
point of intersection of, 277 angle between two planes, 455 Right-handed system, 393, 395
procedure to determine number of, bisecting planes, equations of, 416 of mutually perpendicular unit
231 distance between two parallel planes, vectors triad, 396
413 Rooler axiom, 387
first-degree equation, 410, 412, 454 Rotation of axes, 8–9
O general equation of a plane, 412, 415
intercepts of plane on axes, 411, 454
Octants, 385, 399 nature of three planes, 423–425
normal form of plane, 410
S
with bonding lines, 400
Ordinates, algebraic sum at common ratio of segments joining points, S1, property of, 206
points, 233 414–415 Scalars, 392
Origin and non-origin sides, 38 surface, 410 scalar product (or dot product), 394
Origin of reference, 399 system of planes, 415–417 Scalar triple product of vectors, 398
Orthocentre, 5 Point Section formulae, 3
Orthogonal circles, 154, 206 classification of, 215–216 Semi-vertical angle of the cone, 213
intersection of two, 207 perpendicular distance from plane, Shifting of origin, 7–8
Orthogonal projection, 394 454 Sign of coordinates of a point, 400
Orthogonal projection in a plane, 390 position on hyperbola, 325, 374 Simson’s line, 6
unit, 399 Skew lines, 390
Point, power of a, 157, 207 shortest distance between, 391
formula, 207
P Point circle, 205
Slope, 22
Solid geometry, 385
Points (elements of space) Space, 386
Parabola, 211, 213 distance in terms of, 386
chord of a, 224 lines and planes, 386
incidence postulates of, 386 points (elements of space), 386
chord of contact, 224 non-collinear, 386
condition for the line y  mx c to Standard equation
Position vector of centroid of a triangle, of ellipse, 286–287, 373
touch, 219–220 391
diameter of the, 226–227 of hyperbola, 322
Power of a point, 157, 207 of parabola, 214–215
directrix of, 221 formula, 207
equation of the chord joining two of rectangular hyperbola, 324, 377
Projection vector, 394 Straight line, 22
points on, 218 Pythagoras theorem, 399
equation with vertext, 215 angle between two lines, 33–34
length of the latus rectum of, 215 distance between the parallel lines,
normal to a, 222 37
parametric equations of expression R equation in normal form, 27–28
y2  4ax, 215 in the form of intersecting planes,
point of intersection of the tangents Radical axis, 157–158, 207 421
to the, 218–219 equation of, 207 general equation of, 30
standard equation of a, 214–215, 276 Radical centre, 207 intercept of, 23–25
tangents from external point of a, Ray (s), 387 normal form, 27–28, 32
221 direction cosines (DCs) of, 403 parametric form of, 422
Parallel axiom, 387 parallel, 387 points in, 386
Parallel chords of an ellipse, 301 Real area of a parallelogram, 397 point-slope form, 23
Parallel lines, 387 Rectangular cartesian coordinates, 2 slope-intercept form, 24
direction ratios (DRs) of, 403 quadrants, 2 slope of, 22
Parallel rays, 387 Rectangular coordinates, 2 two-point form, 26
466 Index

Symmetric form of a line, 29, 422–423, area of a, 4 plane, 393


455–456 circumcentre of, 5 position vector, 391
Symmetric form of line, 29 circumcircle of, 5 projection vector, 394
System of planes, 415–417 circumradius of, 5 scalar triple product of vectors, 398
orthocentre of a, 277 vector area, 397
position vector of centroid of a, 391 Vector area, 397
T of non-collinear points, 397
of a parallelogram, 397
Tangent (s) U Vertex of the cone, 212
Vertical line, 22
of an ellipse on auxiliary circle, 298
from an external point to ellipse, 296 Unit points, 399 Vertices of curve, 323
common, of two circles, 159–160, 207 Universal equation of a circle, 205 Vertices of ellipse, 287
condition for line to be, 206 Volume
equation of a, 276 of tetrahedron, 398
of triangular prism, 398
equation of a, on ellipse, 299, 375
from external point of a parabola,
V
221 Vector algebra, 391
length of a, 206 angle between two non-zero vectors, X
method to find common, 208 393
pair, drawn from an external point angle between vectors, 393 x-coordinate of P, 386
on ellipse, 302 collinear, 394–395 x-coordinate point, 399
pair of, 225 cross product, 396 x-intercept of the line, 23
parametric equations of, 206 direction cosines (DCs), 402–403 xy-plane, 399
perpendicular to ellipse, 297 direction ratios (DRs), 402–403
point of intersection to parabola, distributive laws, 396
218–219
of rectangular hyperbola, 330–331
equation of line passing through Y
point and parallel to vector, 392
between two circles, 159–160, 207, 208 equation of line passing through two y-coordinate point, 399
Theorem of three perpendiculars, 388 points, 392 y-intercept of the line, 23
Three-dimensional geometry, 385 equation of plane passing through
Three-dimensional space, 399 points, 393
Three planes, nature of, 423–425, 456
method of solving problems of, 425
non-collinear, 392
non-coplanar, 392
Z
Transeverse axis of curve, 323 orthogonal projection, 394
Triangle z-coordinate point, 399
parametric vectorial equations of a zx-plane, 399

You might also like